Question Review Flashcards

(1714 cards)

1
Q

A 70-year-old male presents to his GP with cough. The chest radiograph shows
bilateral egg shell calcifications in the hilar regions.
Which of the following is the least likely diagnosis?
(a) Silicosis
(b) Asbestosis
(c) Coal workers pneumoconiosis
(d) Sarcoidosis
(e) Histoplasmosis

A

(b) Asbestosis
All the other given options are known to cause egg shell calcification of the hilar
lymph nodes.

How well did you know this?
1
Not at all
2
3
4
5
Perfectly
2
Q

A chest radiograph shows diffuse lung disease with fibrotic changes predominantly
affecting the upper lobes.
What is the most unlikely diagnosis?
(a) Sarcoidosis
(b) Cystic fibrosis
(c) Allergic bronchopulmonary aspergillosis
(d) Langerhans cell granulomatosis
(e) Scleroderma

A

(e) Scleroderma
Other conditions cause predominantly upper zone disease.

How well did you know this?
1
Not at all
2
3
4
5
Perfectly
3
Q

A 70-year-old retired miner presents with shortness of breath for several months.
There is no other significant medical history. The chest radiograph shows calcified
pleural plaques at both lung bases and bi-basilar interstitial shadowing. CT
shows extensive pleural thickening and calcified pleural plaques with bi-basal,
peripheral, interstitial shadows and honeycombing. No lymphadenopathy seen.
The most likely diagnosis is?
(a) Tuberculosis
(b) Asbestosis
(c) Silicosis
(d) Empyema
(e) Sarcoidosis

A

(b) Asbestosis
This is defined as interstitial pulmonary fibrosis in association with asbestos
exposure (pleural plaques and calcification). Disease progression is from bases to
apices and honeycombing is seen later in the disease. Lymphadenopathy is usually
absent, and its presence should suggest alternate diagnosis

How well did you know this?
1
Not at all
2
3
4
5
Perfectly
4
Q

A 20-year-old woman is brought to the Accident & Emergency Department by
ambulance after being found unresponsive on the street. Examination shows
pinpoint pupils and induration in the right groin. The chest radiograph shows
bilateral patchy diffuse air space shadowing predominantly in the middle
and upper zones with central peribronchial cuffing. No pleural effusion or
pneumothorax seen.
The most likely diagnosis is?
(a) Pulmonary oedema secondary to opiate overdose
(b) Acute respiratory distress syndrome
(c) Lung contusion
(d) Renal failure
(e) Fat embolism

A

(a) Pulmonary oedema secondary to opiate overdose
Pin point pupils and right groin infection suggests intravenous drug abuser.
Radiographic findings of non-cardiogenic pulmonary oedema are non-central,
extensive, patchy, bilateral airspace shadowing with indistinct vessels and
peribronchial cuffing.
Cardiogenic oedema is characterised by cardiac enlargement, pleural effusions,
upper lobe venous diversion, Kerley-B lines and peribronchial cuffing.

How well did you know this?
1
Not at all
2
3
4
5
Perfectly
5
Q

A 60-year-old patient under treatment for lymphoma presents with chest pain.
The chest radiograph and blood results are normal. A V/Q scan shows normal
perfusion and patchy areas of ventilation defects in the lungs.
Which of the following is the unlikely diagnosis?
(a) Asthma
(b) Chronic obstructive pulmonary disease
(c) Acute bronchitis
(d) Sarcoidosis
(e) Pulmonary embolism

A

(e) Pulmonary embolism is the unlikely diagnosis
Pulmonary embolism will demonstrate abnormal perfusion defects with or without
ventilation defects

How well did you know this?
1
Not at all
2
3
4
5
Perfectly
6
Q

A 55-year-old woman presents with left-sided ptosis and shoulder pain. The chest
radiograph shows a mass in the left lung apex. CT confirms a large superior sulcus
tumour eroding through the posterior chest wall and rib.
What is the most likely diagnosis?
(a) Adenocarcinoma
(b) Squamous cell carcinoma
(c) Small cell undifferentiated carcinoma
(d) Undifferentiated large cell carcinoma
(e) Scar carcinoma

A

(b) Squamous cell carcinoma
Superior sulcal tumours are frequently squamous cell carcinomas. They may lead
to atrophy of muscles secondary to brachial plexus involvement or/and Horner’s
syndrome secondary to involvement of sympathetic chain and stellate ganglion.

How well did you know this?
1
Not at all
2
3
4
5
Perfectly
7
Q

A 64-year-old non-smoker presents with right chest pain and cough. CT shows a
3 cm spiculated mass in the right upper lobe, abutting the lateral chest wall.
The likely histology is expected to be?
(a) Adenocarcinoma
(b) Squamous cell carcinoma
(c) Small cell undifferentiated carcinoma
(d) Undifferentiated large cell carcinoma
(e) Oat cell cancer

A

(a) Adenocarcinoma
Adenocarcinoma is the most common type associated with non-smokers and is
usually seen in the periphery.
Squamous cell carcinoma, small cell undifferentiated type and undifferentiated large
cell cancers are strongly associated with smoking.

How well did you know this?
1
Not at all
2
3
4
5
Perfectly
8
Q

A 65-year-old man with history of stroke presents with chest pain. The chest
radiograph shows a thin curvilinear area of calcification in the lower part of left
heart border.
What is the likely site of calcification?
(a) Left atrium
(b) Left ventricle
(c) Right atrium
(d) Left descending coronary artery
(e) Mitral valve

A

(b) Left ventricle
This is the typical site for left ventricular calcifications.
Valvular calcifications are located within the heart. Coronary artery calcifications are
seen along the upper part of left heart border and have a ‘tram-track’ appearance.

How well did you know this?
1
Not at all
2
3
4
5
Perfectly
9
Q

A 56-year-old patient with history of cardiac valve replacement presents with
acute-onset chest pain. A frontal chest radiograph shows an enlarged heart with
laterally displaced left cardiac apex and a metallic ring shadow is seen to be
overlapping the spine and horizontally positioned.
Which cardiac valve is this likely to be?
(a) Aortic
(b) Mitral
(c) Tricuspid
(d) Pulmonary
(e) Mitral or aortic

A

(a) Aortic valve
The aortic and mitral valves are seen adjacent to the spine and can be difficult to
separate. However, the aortic valve is usually seen horizontally situated while the
mitral valve is generally situated vertically. On a lateral projection, if a line is drawn
from the carina to the anterior costophrenic angle, the aortic valve lies above this
line and the mitral valve below it.

How well did you know this?
1
Not at all
2
3
4
5
Perfectly
10
Q

. A 60-year-old recently retired postman presents with chronic cough. The chest
radiograph shows soft tissue opacity extending from the right hilum to the lateral
chest wall, with loss of the right heart border. There is loss of right lung volume
and the right costophrenic angle is seen. Bronchoscopy demonstrates a large
endobronchial mass.
What is the most likely bronchus involved?
(a) Right upper lobe bronchus
(b) Right middle lobe bronchus
(c) Right lower lobe bronchus
(d) Bronchus intermedius
(e) Right lower lobe apical segment bronchus

A

(d) Bronchus intermedius
The chest radiograph findings are suggestive of combined right middle lobe
and right lower lobe collapse secondary to tumour obstructing the bronchus
intermedius.

How well did you know this?
1
Not at all
2
3
4
5
Perfectly
11
Q

A 40-year-old man with a history of intravenous drug abuse presents with back
pain. CT shows an infrarenal aortic aneurysm and left psoas abscess.
What is the most likely finding on CT?
(a) Lobulated, saccular aneurysm
(b) Fusiform aneurysm
(c) Pseudoaneurysm
(d) Periaortic gas
(e) Extensive mural thrombus

A

(a) Lobulated, saccular aneurysm
The patient is likely to have a mycotic aortic aneurysm. Mycotic aneurysms are
commonly saccular and lobulated and less commonly fusiform. They may be
associated with psoas abscess, discitis or osteomyelitis.

How well did you know this?
1
Not at all
2
3
4
5
Perfectly
12
Q

A 64-year-old woman known to have chronic rheumatoid arthritis presents with
shortness of breath.
The most common feature seen on the chest radiograph is?
(a) Pleural effusion
(b) Rheumatoid nodule
(c) Diffuse interstitial fibrosis
(d) Bronchiectasis
(e) Pericardial effusion

A

(a) Pleural effusion
This is the most common finding on a chest radiograph in patients with chronic
rheumatoid arthritis, seen in more than 90% cases.

How well did you know this?
1
Not at all
2
3
4
5
Perfectly
13
Q

A 45-year-old woman had allogenic bone marrow transplant for treatment of
leukaemia. Two weeks later she developed cough and shortness of breath. CT
demonstrates bilateral ground-glass shadowing, thickened interstitial lines and
bilateral pleural effusion.
What is the most likely diagnosis?
(a) Bronchiolitis obliterans
(b) Drug toxicity
(c) Pulmonary oedema
(d) Diffuse alveolar haemorrhage
(e) Bronchiolitis obliterans organising pneumonia

A

(c) Pulmonary oedema
This is usually secondary to fluid overload and associated renal dysfunction.
Bronchiolitis obliterans and bronchiolitis obliterans organising pneumonia are
late complications seen after 3 months. Drug toxicity and alveolar haemorrhages
may present with ground-glass shadowing but do not show pleural effusions or
interstitial involvement.

How well did you know this?
1
Not at all
2
3
4
5
Perfectly
14
Q

A 40-year-old man presents with right knee pain. Plain radiography shows a large
joint effusion. MRI of the knee shows multiple foci of low signal intensity seen in
the synovium on T1, T2 and gradient-echo sequences. There is a moderate joint
effusion.
The most likely diagnosis is?
(a) Haemangioma
(b) Pigmented villonodular synovitis
(c) Rheumatoid arthritis
(d) Synovial sarcoma
(e) Synovial chondromatosis

A

(b) Pigmented villonodular synovitis
This is a benign pathology affecting usually the knee joint. It shows no calcifications,
osteoporosis or erosions (until late). MRI is diagnostic, the lesions returning low
signal on all sequences due to iron (haemosiderin)

How well did you know this?
1
Not at all
2
3
4
5
Perfectly
15
Q

A 67-year-old man with history of lung cancer and renal transplant had a bone
scan. There are multiple focal areas of increased tracer uptake in the left ribs,
arranged in a linear pattern. Increased tracer uptake is also identified along the
cortices of both humerus and radius bones bilaterally. No renal uptake is seen.
The most likely diagnosis for this appearance is?
(a) Hypertrophic osteoarthropathy with rib metastases
(b) Hypertrophic osteoarthropathy with rib fractures
(c) Normal uptake in lower limbs with rib fractures
(d) Normal uptake in lower limbs with rib metastases
(e) Diffuse skeletal metastases

A

(b) Hypertrophic osteoarthropathy (HPOA) with rib fractures
On bone scan multiple areas of uptake in a linear pattern suggests traumatic
injury to ribs. HPOA is characterised by bilateral symmetrical tracer uptake on
bone scanning, involving the diaphyseal and metaphyseal regions of long bones.
Characteristically a periosteal reaction is seen along the shafts of involved bones.
This pattern of uptake is called a ‘double-stripe’ or ‘parallel-track’ sign and is
characteristic of HPOA.

How well did you know this?
1
Not at all
2
3
4
5
Perfectly
16
Q

A 62-year-old man presents with sudden-onset pain after minor injury. Plain
radiograph shows subchondral sclerosis in the medial femoral condyle and joint
effusion. MRI shows a diffuse oedema in the subchondral bone of medial femoral
condyle with a crescentic linear fracture in a subchondral location.
The most likely diagnosis is?
(a) Spontaneous osteonecrosis of knee
(b) Osteoarthritis
(c) Osteochondritis desiccans
(d) Calcium pyrophosphate deposition disease
(e) Gout

A

(a) Spontaneous osteonecrosis of knee
Typical subchondral fracture in elderly patient after minor knee injury.

How well did you know this?
1
Not at all
2
3
4
5
Perfectly
17
Q

An 18-year-old man presents with progressive swelling of right knee. Radiographs
show large joint effusion in the suprapatellar pouch. MRI shows marked synovial
thickening and large synovial fronds which return high signal on T1, T2 and proton
density images. The lesions are low signal on STIR images.
The most likely diagnosis is?
(a) Synovial lipoma
(b) Synovial osteochondromatosis
(c) Hypertrophic synovitis
(d) Pigmented villonodular synovitis
(e) Lipoma arborescens

A

(e) Lipoma arborescens
This condition is seen most commonly in the suprapatellar pouch, with small to large
frond-like masses arising from synovium. On MRI, the masses show characteristic
signal of fat on all sequences. Saturation on STIR images is diagnostic.

How well did you know this?
1
Not at all
2
3
4
5
Perfectly
18
Q

A 35-year-old woman presents with swelling in the thigh. The radiograph shows a
bony excrescence from the femoral cortex without medullary continuity. On MRI
there is a soft tissue surrounding the bony excrescence, which returns high signal
on T1 and T2 and homogenous low signal on STIR.
The most likely diagnosis is?
(a) Osteochondroma
(b) Osteosarcoma
(c) Liposarcoma
(d) Parosteal lipoma
(e) Intramuscular lipoma

A

d) Parosteal lipoma
These are benign tumours of adipose tissue which are intimately related to
the periosteum. They often contain bony excrescences that may resemble
osteochondroma but, unlike osteochondroma, they do not communicate with the
medullary cavity of parent bone. MRI is diagnostic, confirming the juxtacortical
benign nature of the fatty lesion and non-communication of the bony lesion with
the medulla of the adjacent bone.

How well did you know this?
1
Not at all
2
3
4
5
Perfectly
19
Q

A 33-year-old man presents with a 2-year history of a hard lump on the left middle
finger. A radiograph shows a 2 cm, well-defined, round, densely sclerotic lesion
attached to the cortex of the proximal phalanx of the left middle finger. No cortical
erosion or periosteal reaction is seen. A bone scan shows no tracer uptake.
The most likely diagnosis is?
(a) Enostosis
(b) Osteoma
(c) Parosteal osteosarcoma
(d) Osteochondroma
(e) Myositis ossificans

A

(b) Osteoma
The best diagnostic clue is the densely sclerotic, well-defined lesion attached to the
parent bone. Latent lesions show no tracer uptake.

How well did you know this?
1
Not at all
2
3
4
5
Perfectly
20
Q

A 60-year-old presents with left groin pain. Ultrasound shows a 2 cm hypoechoic
lesion bulging medial to the epigastric vessels on Valsalva manoeuvre and absent
on rest.
What is the most likely diagnosis?
(a) Direct inguinal hernia
(b) Indirect inguinal hernia
(c) Obturator hernia
(d) Spigelian hernia
(e) Femoral hernia

A

(a) Direct inguinal hernia
A direct inguinal hernia is seen medial to the inferior epigastric vessels whereas an
indirect hernia is seen lateral to them.

How well did you know this?
1
Not at all
2
3
4
5
Perfectly
21
Q

A 40-year-old man presents with right groin pain. Ultrasound shows a 3 cm
echogenic soft tissue mass distending the right inguinal canal on straining, and
which goes away on relaxation.
What is the most likely diagnosis?
(a) Direct inguinal hernia
(b) Indirect inguinal hernia
(c) Femoral hernia
(d) Obturator hernia
(e) Lymph node

A

(b) Indirect inguinal hernia
An indirect inguinal hernia protrudes through the internal inguinal ring and extends
along the inguinal canal parallel to its long axis.

How well did you know this?
1
Not at all
2
3
4
5
Perfectly
22
Q

A 70-year-old man presents after falling down five stairs and sustaining injury
to the neck. An open-mouth view shows increased space between the dens and
medial border of lateral masses of C1. CT shows fracture of the anterior and
posterior arch of the C1 vertebra.
What is the most likely diagnosis?
(a) Hangman’s fracture
(b) Clay shoveller’s fracture
(c) Jefferson fracture
(d) Extension teardrop fracture
(e) Flexion teardrop fracture

A

(c) Jefferson fracture
Jefferson fracture involves the C1 vertebra. There is a comminuted fracture of the
C1 ring, at least through two places. Plain radiography using an open-mouth view
demonstrates lateral displacement of the lateral masses.

How well did you know this?
1
Not at all
2
3
4
5
Perfectly
23
Q

A 14-year-old boy presents with persistent right hip pain after a recent injury.
Radiographs confirm the diagnosis of slipped capital femoral epiphysis.
What is the Salter–Harris classification of this condition?
(a) Type I
(b) Type II
(c) Type III
(d) Type IV
(e) Type V

A

(e) Type I Salter–Harris injury
Slipped capital femoral epiphysis is classified as a type I Salter–Harris injury because
there is a slipped epiphysis due to the shearing force of an injury, separating
the epiphysis from the metaphysis. There is no fracture of the metaphysis or the
epiphysis itself.

How well did you know this?
1
Not at all
2
3
4
5
Perfectly
24
Q

A 65-year-old woman is admitted with abdominal pain. ERCP shows generalised
dilated intrahepatic and extrahepatic ducts with multifocal strictures and small
diverticulae formation.
The most likely diagnosis is?
(a) Primary sclerosing cholangitis
(b) Choledochocoele
(c) Caroli’s syndrome
(d) Cholangiocarcinoma
(e) Primary biliary cirrhosis

A

(a) Primary sclerosing cholangitis
These features are typically diagnostic of primary sclerosing cholangitis.
Caroli’s disease is rare condition which manifests in childhood, adolescents and
into the third decade. Appearances can be similar to primary sclerosing cholangitis.
Choledochocoele is seen in young adults: there is a sac-like dilatation of the
intramural segment of the common bile duct which prolapses into the duodenum;
there are scattered dilated intrahepatic ducts with no apparent connection to
main bile ducts. Caudate lobe hypertrophy is seen in primary biliary cirrhosis.
Cholangiocarcinoma may be seen as mass lesion with focal duct dilatation; no
generalised strictures and diverticulae are seen.

How well did you know this?
1
Not at all
2
3
4
5
Perfectly
25
To differentiate between a renal cell carcinoma which has spread into the perinephric fat and an angiomyolipoma with haemorrhage, the best diagnostic modality would be? (a) Ultrasound (b) CT scan (c) MRI (d) Angiogram (e) Cannot be differentiated
(b) CT scan Demonstration of fat within the mass will point to a diagnosis of angiomyolipoma.
26
Which of the following investigations is the most sensitive test for localisation of Islet cell tumours? (a) Transhepatic portal venous sampling (TPVS) (b) Contrast enhanced MRI (c) Endoscopic ultrasound (d) Selective arteriography (e) Arterial stimulation and venous sampling (ASVS)
(e) Arterial stimulation and venous sampling This involves selective pancreatic arterial injection of a secretogogue and the hepatic venous flow is sampled. Lesions not seen on cross section imaging can be detected. The sensitivity of ASVS is same as TPVS for insulinoma but better for gastrinoma.
27
A 60-year-old man was admitted with intermittent abdominal pain. CT scan shows an ill-defined soft tissue mass in the bowel mesentery, with extensive calcification within. Strands of soft tissue are seen radiating into the surrounding fatty mesentery. The adjacent bowel loops show retraction. MRI shows low signal on T1 and T2 images. The most likely diagnosis is? (a) Carcinoid syndrome (b) Fibrosing mesenteritis (c) Mesenteric panniculitis (d) Desmoid tumour (e) Old tuberculosis
b) Fibrosing mesenteritis This is the classical appearance for fibrosing mesenteritis.
28
A 48-year-old woman presents with history of upper abdominal pain, weight loss and bilateral ankle oedema. CT abdomen shows thickened gastric wall with prominent mucosal folds affecting the upper part of the stomach and greater curvature, while the antrum, pylorus and rest of the bowel appear normal. The most likely diagnosis is? (a) Crohn’s disease (b) Gastric carcinoma (c) Lymphoma (d) Ménétrier’s disease (e) Eosinophilic gastritis
(d) Ménétrier’s disease This is characterised by hypertrophy of gastric folds affecting the greater curvature while usually sparing the antrum, hypoproteinemia (causing ankle oedema) and hypochlorhydria. Lymphoma involves any part of stomach and antrum. Eosinophilic gastritis often affects the antrum and the proximal small bowel. Crohn’s disease shows multiple aphthous ulcers and commonly affects antrum and pylorus. It usually affects the terminal ileum as well.
29
A 6-year-old girl presents with gradually increasing abdominal mass. Plain radiography shows a soft tissue mass displacing the bowel loops, with small calcifications. Ultrasound reveals a 10 cm, thin walled, cystic lesion in the mid abdomen, with multiple internal septations and small internal echoes. The most likely diagnosis is? (a) Duplication cyst (b) Mesenteric cyst (c) Neuroblastoma (d) Ovarian cyst (e) Lymphoma
(b) Mesenteric cyst Mesenteric cysts are rare and are found in the mesentery and omentum. They are true congenital abnormalities and arise due to sequestration of mesenteric lymphatics. Imaging features are typical as given in this case history. CT scan defines the anatomic margins of the cyst but septations are poorly seen on CT. MRI features vary according to the contents of the cyst.
30
A 45-year-old man presents with acute-onset abdominal pain. Imaging confirms the diagnosis of intussusception. Which of the following is the most likely cause? (a) Meckel’s diverticulum (b) Aberrant pancreas (c) Chronic tuberculosis ulcer (d) Scleroderma (e) Idiopathic
e) Idiopathic Twenty per cent of all adult intussusceptions are idiopathic. The rest of the given causes are rare.
31
A 60-year-old man presents with constipation and painful defecation. CT shows a non-enhancing, well-defined, lobulated, and homogenous, low-attenuation lesion in the retrorectal space. The lesion shows thin peripheral calcification. On MRI, the lesion returns intermediate signal on T1 with areas of high signal within and high signal on T2 with septae. What is the most likely diagnosis? (a) Enteric cyst (b) Dermoid cyst (c) Sacrococcygeal teratoma (d) Anal duct cyst (e) Rectal leiomyosarcoma
(a) Enteric cyst Enteric cyst may be septated and filled with mucoid contents which return high signal on T1 images. On CT they have the characteristic features as in the case. Dermoid cysts contain skin appendages and commonly contain fat. Sacrococcygeal teratomas are usually seen in paediatric age group. On CT and MRI, they appear as heterogeneously enhancing lesions with both cystic and solid components.
32
A 50-year-old woman with a history of bleeding per rectum presents for a barium enema. The examination shows multiple worm-like projections attached by their bases to the sigmoid colon. The most likely diagnosis is? (a) Postinflammatory polyps (b) Juvenile polyps (c) Familial adenomatous polyposis (d) Faecal residue (e) Colonic ulceration
(a) Postinflammatory polyps Filiform polyps adhering to the colon by their bases are typical of postinflammatory polyps, which can be seen in ulcerative colitis.
33
A 47-year-old diabetic man with recent renal transplant, presents with dysphagia. Double-contrast barium swallow shows longitudinally oriented filling defects in the upper and mid oesophagus. CT shows circumferential thickening of the upper half of oesophagus. What is the most likely diagnosis? (a) Reflux oesophagitis (b) Viral oesophagitis (c) Candida oesophagitis (d) Oesophageal varices (e) Carcinoma
(c) Candida oesophagitis This is seen in immunocompromised patients and is caused by Candida species. It spares the lower oesophagus and typically shows longitudinally oriented filling defects on double-contrast barium swallow. Reflux usually extends proximally from gastro-oesophageal junction with ulcers in distal oesophagus. Viral oesophagitis (due to herpes virus or cytomegalovirus) usually shows multiple discrete ulcers. Cytomegalovirus may show giant ulcers (> 1 cm). Varices show serpiginous longitudinal defects, best seen on mucosal relief views
34
A 28-year-old man smelling of alcohol presented to the casualty department with chest pain and vomiting. The chest radiograph shows pneumomediastinum, surgical emphysema and left hydropneumothorax. What is the most likely diagnosis? (a) Mallory–Weiss syndrome (b) Boerhaave syndrome (c) Spontaneous pneumothorax (d) Post traumatic (e) None of the above
(b) Boerhaave syndrome This is characterised by spontaneous distal oesophageal perforation following vomiting or violent straining. The tear usually is seen at the left lateral wall of distal oesophagus just above the oesophago-gastric junction. Diagnosis is made by demonstrating leak of extraluminal air and contrast around oesophagus. A perforated Mallory–Weiss tear is a Boerhaave syndrome.
35
A 37-year-old woman presents with epigastric pain and weight loss. Barium meal shows multiple small barium collections with surrounding lucent halos in the distal body and antrum of stomach. The most likely diagnosis is? (a) Gastric metastases (b) Gastric varices (c) Erosive gastritis (d) Gastric polyps (e) Menetrier disease
(c) Erosive gastritis The condition is characterised by aphthous ulcers seen as multiple small mucosal mounds, each having a central and superficial erosion (not penetrating the muscularis), in the antrum and body of stomach. The central erosion collects barium, with the mucosal mounds representing surrounding lucencies.
36
A 70-year-old man with history of history of alcoholism, presents with progressive dysphagia. Barium studies of the upper gastrointestinal tract show irregular narrowing of the oesophagus just above the gastro-oesophageal junction, with abrupt transition to normal mucosa proximally. What is the most likely diagnosis? (a) Ulcerating carcinoma of the oesophagus (b) Barrett’s oesophagus (c) Schatzki rings (d) Oesophageal polyps (e) Achalasia of oesophagus
(a) Ulcerating carcinoma of the oesophagus In patients with risk factors and irregular stricture in the oesophagus, carcinoma should be considered as first diagnosis and direct visualisation with histopathology is recommended.
37
A 70-year-old woman under investigation for anaemia and upper abdominal pain has barium study of the upper gastrointestinal tract, which shows a featureless tubular narrow stomach. The area gastricae are absent. What is the most likely diagnosis? (a) Helicobacter pylori gastritis (b) Crohn’s disease (c) Atrophic gastritis (d) Ménétrier’s disease (e) Zollinger–Ellison syndrome
(c) Atrophic gastritis In elderly patients, this is associated with pernicious anaemia. The condition is characterised by loss of parietal cells leading to achlorhydria and atrophy of mucosa and mucosal glands. Radiographic findings as described in the case history are typical of atrophic gastritis. The condition is associated with malignancy.
38
A 23-year-old woman presents with recurrent cyclical lower abdominal pain. Ultrasound shows a 4 cm heterogenous cystic mass in the pelvis related to the left ovary. On MRI, the lesion has predominantly high signal on T1, T2 and STIR sequences. What is the most likely diagnosis? (a) Metastasis (b) Krukenberg tumour (c) Ovarian dermoid (d) Endometrioma (e) Ectopic pregnancy
d) Endometrioma MRI is highly sensitive and specific in the diagnosis of endometrioma. The endometrioma returns high signal on T1 and T2 and STIR sequences due to blood products. On ultrasound the lesion may show diffuse homogenous low-level internal echoes (haemorrhagic debris). Other features may include septations or echogenic material suggesting a clot.
39
A 65-year-old man with known abdominal aortic aneurysm and under follow up for lymphoma, presents with backache. Contrast-enhanced CT shows a doughnut shaped soft tissue mass surrounding the lower part of aneurysmal abdominal aorta and the ureters are pulled medially with bilateral hydronephrosis. What is the most likely diagnosis? (a) Retroperitoneal fibrosis (b) Lymphoma recurrence (c) Aneurysm leak (d) Radiation injury (e) None of the above
(a) Retroperitoneal fibrosis This is hard fibrous tissue enveloping the retroperitoneum, including the great vessels, ureters and the lymphatics. The plaque typically begins around the aortic bifurcation and extends cephalad to the renal hilum, and it rarely extends below the pelvic brim.
40
A 60-year-old woman had a screening mammogram which shows a densely calcified lesion in the right breast. The lesion is smoothly marginated and has soft tissue density with dense coarse ‘popcorn’ calcification. What is the most likely diagnosis? (a) Breast carcinoma (b) Ductal carcinoma in situ (c) Fibroadenoma (d) Fibroadenosis (e) Fat necrosis
(c) Fibroadenoma Fibroadenomas are benign lesions often seen in young women. With advancing age, they shrink and may degenerate. This can then calcify resulting in a typical ‘popcorn’ type calcification.
41
A 35-year-old woman with a strong family history of breast cancer presents with a breast lump. Ultrasound shows a hypoechoic lesion with internal echoes. Gadolinium-enhanced contrast imaging demonstrates a 2 cm, non-enhancing, oval lesion in the right breast. What is the most likely diagnosis? (a) Fat necrosis (b) Fibroadenoma (c) Cyst (d) Carcinoma (e) Radial scar
(c) Cyst Other lesions are known to show contrast enhancement.
42
A 37-year-old woman presents with menstrual irregularities. Ultrasound shows a right adnexal abnormality. MRI shows a 3 cm well-defined lesion in the right adnexa which returns high signal on T1. What is the most likely diagnosis? (a) Fibroma (b) Brenner tumour (c) Ovarian dermoid (d) Pedunculated leiomyoma (e) Fibrothecoma
(c) Ovarian dermoid Fibrous lesions in the adnexa are of low signal intensity on MRI. Ovarian dermoids return high signal because of their fat content, with signal drop-out on fat suppression images. Other causes of lesions which may return high signal on T1 include endometrioma, mucinous cystic neoplasm, haemorrhagic cysts and ovarian carcinoma.
43
Zonal anatomy of the prostate is best seen in which of the following sequences? (a) T1-weighted images (b) T2-weighted images (c) Proton density (d) STIR (e) T1 fat saturation
(b) T2 weighted images These demonstrate the zonal anatomy of the prostate well. The prostatic urethra serves as a reference point. The peripheral zone returns high signal compared with the central or transitional zones.
44
A sick neonate has an anteroposterior radiograph of the chest and abdomen. It shows an umbilical catheter line traversing initially caudally and then cephalad, and the tip lies to the left of T3 vertebral body. The catheter is in? (a) Correctly placed umbilical arterial line (b) Correctly placed umbilical vein line (c) High umbilical arterial line (d) High umbilical vein line (e) Low umbilical arterial line
c) A high umbilical arterial line The umbilical arterial line passes caudad into the internal and common iliac arteries and then courses cephalad in the aorta. The tip should be above the level of celiac axis (T6–T10), or below the renal arteries (L3–L5). An umbilical vein catheter courses directly cephalad on the right side. The tip should lie above the liver and not passed into a tributary vein.
45
A 10-year-old child presents with a lump in the scalp. The skull radiograph shows a lucent lesion with sclerotic margins. The most likely diagnosis is? (a) Dermoid cyst (b) Aneurysmal bone cyst (c) Histiocytosis X (d) Neuroblastoma metastasis (e) Osteosarcoma metastasis
(a) Dermoid They are usually an incidental finding. They have a characteristic appearance of a central lucent area with sclerotic margins.
46
A 13-year-old child presents with pain in the leg. Radiography shows a well- defined, eccentric, radiolucent lesion with a thin sclerotic border towards the medulla in the proximal tibia. No periosteal reaction seen. On MRI, the lesion returns low signal on T1 and T2. The most likely diagnosis is? (a) Chondromyxoid fibroma (b) Non-ossifying fibroma (c) Intraosseous ganglion (d) Brodie’s abscess (e) Simple bone cyst
(b) Non-ossifying fibroma or a fibrous cortical defect These benign lesions typically present like this. On MRI, they appear as low signal on T1 and T2 images due to hypocellular fibrous tissue within. Chondromyxoid fibromas show a bulging cortex and geographical bone destruction with calcifications and septations. On MRI they are hyperintense on T2. Intraosseous ganglion is hyperintense on T2. Aneurysmal bone cysts show fluid–fluid levels and are heterogenously hyperintense on MRI.
47
A 6-year-old girl presents with ongoing back pain. Radiograph of the spine shows a flattened sclerotic T6 vertebral body with normal adjacent discs. What is the most likely diagnosis? (a) Trauma (b) Tuberculosis (c) Langerhans cell histiocytosis (d) Leukaemia (e) Morquio’s syndrome
c) Langerhans cell histiocytosis This predominantly affects children and is characterised by clonal proliferation of abnormal Langerhans cells; histiocytes capable of migrating from skin to lymph nodes. This is the most likely diagnosis in a child presenting with vertebra plana with sparing of the disc spaces.
48
A 14-year-old girl presents after a twisting injury with inability to weight bear on the leg. Radiographs of the ankle and leg show a type III Salter–Harris fracture on the anteroposterior view. CT of the ankle confirms the fracture. The coronal reformats show that there is partial fusion of the medial part of distal tibial epiphyseal plate. What is the most likely diagnosis? (a) Le Fort fracture (b) Tillaux fracture (c) Maisonneuve fracture (d) Bennett fracture (e) Pilon fracture
(b) Tillaux fracture This is a Salter–Harris type III fracture of the anterolateral distal tibial epiphysis resulting from an abduction and external rotation injury. This is seen in adolescents since the distal tibial epiphysis fuses in a medial to lateral direction and is not seen in adults where the growth plate has fused. Pilon fractures are comminuted fractures of the plafond. Maisonneuve fracture involves tearing of syndesmosis, posterior malleolus, capsular injury and fracture of the proximal fibula. Le Fort fracture involves the distal fibula and the anterior tibiofibular ligament
49
A 4-year-old Caucasian child presents with loss of vision. CT of the head shows a well circumscribed suprasellar cystic mass with rim calcifications. On MRI, the pituitary gland appears normal and the lesion has a fluid-fluid level. The lesion returns high signal on T1, T2 and FLAIR sequences. There is minimal peripheral enhancement with gadolinium. The most likely diagnosis is? (a) Rathke’s cleft (b) Epidermoid cyst (c) Pituitary adenoma (d) Craniopharyngioma (e) Suprasellar arachnoid cyst
(d) Craniopharyngioma Craniopharyngioma is the most common suprasellar tumour in paediatrics and usually cystic. Calcification is seen in 90% of the cases. These lesions contain highly proteinaceous fluid, cholesterol and blood products resulting in high signal on T1, T2 and FLAIR images.
50
A 1-day-old neonate presents with respiratory distress. The chest radiograph shows soft tissue shadowing in the right lower zone. On day 4, CT of the chest shows multiple small cysts of varying sizes containing air with resolution of the soft tissue density. What is the most likely diagnosis? (a) Bronchopulmonary sequestration (b) Congenital diaphragmatic hernia (c) Pneumonia (d) Congenital cystic adenomatoid malformation (e) Bronchogenic cyst
(d) Congenital cystic adenomatoid malformation These are multicystic lesions filled with air. They communicate with the bronchial tree and are filled with air early in life. Most lesions are confined to a single lobe and are solitary. Sequestration does not contain air in the neonatal period and is only filled with air if infected.
51
A term neonate presents with bilious vomiting, abdominal distension and failure to pass meconium. An abdominal radiograph shows dilated loops of bowel. A contrast enema shows a micro colon with ‘rabbit pellet’ filling defects in the ileum. What is the most likely diagnosis? (a) Meconium ileus (b) Hirschsprung’s disease (c) Imperforate anus (d) Meconium plug syndrome (e) Ileal atresia
(a) Meconium ileus Meconium ileus is almost diagnostic of cystic fibrosis and presents in neonates. Family history of CF may be present. The abnormally thick meconium obstructs the ileum and a water soluble enema may relieve the impaction. This disorder produces the smallest of microcolons because the obstructing meconium leaves the colon unused. Meconium plug syndrome is also known as functional immaturity of the colon. It is a temporary functional obstruction which often occurs at the splenic flexure, and is a common cause of neonatal obstruction.
52
A toddler presents with urinary retention and abdominal distension. CT shows a large pelvic mass with calcifications. T2 and STIR images on MRI demonstrate a large, predominantly solid, mixed signal intensity mass in the presacral region, which extends in between the sacral segments, encasing the sacrum. The bladder and rectum are displaced anteriorly but not invaded. What is the most likely diagnosis? (a) Ovarian teratoma (b) Neuroblastoma (c) Sacrococcygeal germ cell tumour (d) Anterior meningocele (e) Duplication rectum
(c) Sacrococcygeal germ cell tumour These are relatively rare tumours and can be benign or malignant. Calcification is seen on CT in more than half of cases, more frequently in benign lesions. Direct invasion of surrounding structures suggest malignancy. Ovarian teratoma, neuroblastoma and gastrointestinal duplication cysts are all rare and do not (usually) extend around the sacrum. Meningoceles are cystic structures.
53
Transcranial ultrasound of a preterm infant with feeding difficulties shows echogenic shadowing filling the lumen of right lateral ventricle. There is also dilatation of the lateral ventricles. What is the most likely diagnosis? (a) Normal choroid plexus (b) Subependymal haemorrhage (c) Subependymal haemorrhage with ventricular dilatation (d) Subependymal haemorrhage without ventricular dilatation (e) Periventricular haemorrhage
(c) Subependymal haemorrhage with ventricular dilatation The differential diagnosis is from a normal choroid plexus. However, if the blood fills the ventricle then the diagnosis is easy.
54
A 40-year-old housewife presents with severe left L4 radiculopathy. 1 year ago she had a L4/5 discectomy. A gadolinium-enhanced MRI of the lumbar spine was performed. Post surgical fibrosis and epidural scar is diagnosed on T1-enhanced images by? (a) An enhancing epidural tissue at L4/5 compressing on left L4 nerve root (b) A non-enhancing epidural tissue at L4/5 compressing the left L4 nerve root (c) An enhancing mass within the spinal canal at L4/5 compressing left L4 root (d) A non-enhancing mass in the spinal canal at L4/5 compressing left L4 root (e) An enhancing mass at L5/S1 compressing left L4 root
(a) An enhancing epidural tissue at L4/5 compressing on left L4 nerve root Scar tissue seen after previous disc resection shows enhancement with gadolinium, while a recurrent or sequestrated disc is unlikely to..
55
A 40-year-old teacher presents with a history of hearing loss in the left ear. Gadolinium-enhanced MRI shows a non-enhancing lesion in the left cerebellopontine angle (CPA). The lesion is isointense to CSF on T1 and T2 sequences. On FLAIR imaging, the lesion shows incomplete attenuation of fluid signal and on diffusion-weighted imaging it returns a bright signal. The most likely diagnosis is? (a) Arachnoid cyst in the left CPA (b) Schwannoma in the left CPA (c) Epidermoid cyst in the left CPA (d) Lipoma in the left CPA (e) Cystic meningioma in the left CPA
(c) Epidermoid cyst This resembles CSF on non-enhanced CT. On MRI, the lesion is isointense, or slightly hyperintense to CSF. Incomplete attenuation on FLAIR and high signal on diffusion (suggesting restricted diffusion) concludes the diagnosis. Non enhancement is the rule.
56
A 27-year-old woman presents to the Accident & Emergency Department with headaches. A CT scan of the head shows widely spaced lateral ventricles, dilatation of the trigones and occipital horns of lateral ventricles with an upward displacement of the dilated 3rd ventricle. The underlying abnormality in the brain is? (a) Midline arachnoid cyst (b) Agenesis of the corpus callosum (c) Prominent cavum septum pellucidum (d) Hydrocephalus (e) Lobar holoprosencephaly
b) Agenesis of corpus callosum This is associated with parallel, widely spaced lateral ventricles that may appear crescent shaped. There is dilatation of trigones and the occipital horn of lateral ventricles, along with a high riding 3rd ventricle. Callosal agenesis is associated with Dandy–Walker syndrome, Chiari malformations and fetal alcohol syndrome.
57
A 25-year-old man presents with painless progressive proptosis. CT shows a soft tissue lesion with microcalcifications in the left orbit. On MRI, T1 axial and coronal images show a soft tissue mass isointense to adjacent muscles in the extraconal plane while the T2 images show the hyperintense septated lesion. The lesion avidly enhances with gadolinium. What is the most likely diagnosis? (a) Cavernous haemangioma (b) Orbital pseudotumour (c) Neurofibroma (d) Dermoid cyst (e) Haemangiopericytoma
a) Cavernous haemangioma This is the most common orbital mass in adults and is the most common vascular malformation of the orbit. CT demonstrates the location of the lesion and microcalcifications. Remodelling of the bone may be seen. MRI features are typical as given.
58
A 32-year-old man with a 3 month history of headaches presents to the Accident & Emergency Department with tonic-clonic seizures. MRI shows a 5 cm intra- axial lesion in the left frontal lobe. The lesion appears hypointense on T1 and hyperintense on T2 to brain parenchyma. No significant surrounding oedema is seen and there is no enhancement with gadolinium. The most likely diagnosis is? (a) Oligodendroglioma (b) Astrocytoma (c) Arachnoid cyst (d) Metastases (e) Lymphoma
(b) Astrocytoma These MRI appearances are typical of a grade II astrocytoma. Grade III are more infiltrative and show more surrounding oedema. Oligodendrogliomas show calcifications. Arachnoid cysts show CSF density on all sequences. Metastatic lesions and lymphoma enhance with gadolinium.
59
A 70-year-old man was admitted with left sided hemiparesis. Brain CT shows an area of low attenuation in the right lentiform nucleus. Which of the following artery is involved? (a) Anterior choroidal branches (b) Posterior cerebral artery (c) Lateral lenticulostriate branches of the middle cerebral artery (d) Medial lenticulostriate branches of the middle cerebral artery (e) Posterior choroidal branches
(c) Lateral lenticulostriate branches of middle cerebral artery The basal ganglia derive their blood supply from the lenticulostriate arteries. A portion of the anterior limb of internal capsule and the head of caudate nucleus is supplied by the medial lenticulostriate arteries. The lateral lenticulostriate arteries supply the lentiform nucleus and parts of the caudate nucleus and internal capsule.
60
A 42-year-old man presents in the Accident & Emergency Department with epileptic seizure. Head CT shows asymmetrical white matter oedema in the left parietal region with a mass effect. Post-contrast study shows a large, irregular and peripheral enhancing lesion with a central area of low attenuation. What is the most likely diagnosis? (a) Lymphoma (b) Metastasis (c) Glioblastoma multiforme (d) Toxoplasmosis (e) Cerebral abscess
(c) Glioblastoma multiforme These tumours are typically inhomogeneous on CT and MRI, showing irregular areas of peripheral enhancement. Tumour necrosis is a hallmark of glioblastoma multiforme.
61
A 42-year-old Caucasian woman presents with multiple fits. CT of the head shows multiple, small enhancing lesions in the cortical and subcortical areas. On MRI, these lesions return low signal on T2 and hyperintense on post-gadolinium T1. What is the most likely diagnosis? (a) Tuberous sclerosis (b) Calcifications (c) Melanoma metastases (d) Haemorrhagic metastases (e) Lymphoma
c) Melanoma metastases The T2 shortening effect is attributed to the paramagnetic effects of iron and copper bound to melanin.
62
A 45-year-old man presents with deafness and left ear discharge. CT of the petrous and mastoids shows a soft tissue mass in the attic with erosion of the scutum. No contrast enhancement is seen. What is the most likely diagnosis? (a) Glomus tympanicum (b) Pars tensa cholesteatoma (c) Pars flaccida cholesteatoma (d) Cholesterol granuloma (e) Congenital cholesteatoma
(c) Pars flaccida cholesteatoma This typically causes erosion of the scutum, ossicles or the lateral epitympanic wall. Pars flaccida is a small superior portion of the tympanic membrane. Scutum erosion is common and three-quarters of cases may have erosion of the ear ossicles.
63
A 9-year-old boy presents with chronic right facial pain. Radiography shows an opaque right maxillary antrum. A CT scan of the paranasal sinuses shows that the right maxillary antrum is filled with soft tissue with destruction of medial and posterior bony walls. No significant sinus mucosal disease is seen in the other paranasal sinuses. What is the most likely diagnosis? (a) Fungal infection (b) Allergic sinusitis (c) Rhabdomyosarcoma (d) Antrochoanal polyp (e) Acute sinusitis
(c) Rhabdomyosarcoma Bone destruction suggests an aggressive lesion at this site and rhabdomyosarcoma would be the most likely diagnosis in a young person.
64
A 34-year-old female presented with hypertension and an ejection systolic murmur. Which finding is most likely to support a diagnosis of true coarctation compared to pseudocoarctation? a Figure`3' sign on angiogram b An associated bicuspid aortic valve C Rib notching d High positioned aortic arch e Dilatation of the distal aorta
Answer C: Rib notching Rib notching is not seen in pseudocoarctation. This entity , once thought benign, is due to kinking of the aorta. Although there is no obstruction to flow, the abnormal anatomy can lead to aneurysmal dilatation distal to the abnormality .
65
A previously fit and well 45-year-old male presented with fever, abdominal pain and weight loss. Clinical examination was unremarkable. The erythrocyte sedimentation rate was raised but his leucocyte count was normal. After referral to the surgical team a CT of his abdomen was performed. This showed thickening of the wall of the ascending colon with pericolic fat stranding. Contrast was seen within the superior mesenteric vessel and there was a wedge-shaped area of low attenuation in the spleen. A selective angiogram of the superior mesenteric artery was performed, which demonstrated multiple aneurysms measuring between 1 and 5 mm. What is the most likely diagnosis? a Ischaemic colitis b Systemic lupus erythematosus C Polyarteritis nodosa d Rheumatoid vasculitis e W egener's granulomatosis
Answer C: Polyarteritis nodosa The angiogram findings are classic although not pathognomonic of P AN. With no history of SLE or rheumatoid it makes these less likely . Up to two-thirds of people with PAN have bowel symptoms
66
A 39-year-old male smoker was referred to a cardiologist with chest pain. A cardiac magnetic resonance examination was requested as part of his workup. This showed patchy multifocal delayed hyperenhancement within the basal interventricular septum. What is the most likely diagnosis? a Sarcoidosis b Amyloidosis c Ischaemic myocardium d Myocarditis e Hypertrophic Cardiomyopathy
Answer E: Hypertrophic cardiomyopathy The location and pattern of enhancement is typical of this diagnosis. In amyloidosis the hyperenhancement is global and in sarcoidosis and myocarditis it affects the epicardial or mid-myocardial regions.
67
A 2 5-year-old male was referred for a routine testicular screening ultrasound. He is known to have a cardiac myxoma and has multiple pigmented lesions on his face and lips. What is the most likely unifying diagnosis? a Carney's syndrome b Peutz Jeghers syndrome c W aardenburg's syndrome d Cronkhite-Canada syndrome e Gorlin's syndrome
Answer A: Carney's Syndrome Carney's Syndrome or Complex refers to a familial neoplastic lentiginous syndrome consisting of the following: primary pigmented nodular adrenocortical disease, lentigines, ephelides, blue nevi of the skin and mucosa, various tumours (including myxomas of the skin, heart and breast) and Sertoli-cells tumours of the testes.
68
A 45-year-old man presents with worsening dyspnoea over a two-year period. Chest radiography demonstrates enlarged central pulmonary arteries and elevation of the cardiac apex. Which of the following is most likely? a V entricular septal defect b Patent ductus arteriosus c Atrial septal defect d Partial anomalous pulmonary venous drainage e Endocardial cushion defect
11 Answer C: Atrial septal defect ASD is the most common presenting left to right shunt in adulthood.
69
A 56-year-old female was referred to the cardiology outpatient clinic with recent onset exertional dyspnoea. An echocardiogram showed left ventricular dysfunction and a cardiac MRI was requested to identify the cause. Cine images revealed focal hypokinesis in the anteroseptal wall and delayed enhanced images show increased signal in the subendocardium. What is the most likely diagnosis? a Myocarditis b Myocardial infarction C Hypertrophic cardiomyopathy d Amyloidosis e Tako-tsubo cardiomyopathy
Answer B: Myocardial infarction Purely subendocardial delayed hyperenhancement in a recognised vascular territory is classical of myocardial infarction. The anteroseptal wall is supplied by the left anterior descending artery .
70
A 56-year-old male patient presented with headache and swelling of his face and neck. A chest radiograph revealed several parenchymal opacities of varying sizes and widening of the superior mediastinum. What is the most likely cause of this presentation? a Superior vena cava obstruction (SVCO) secondary to metastatic renal cell carcinoma b SVCO secondary to tuberculous lymphadenopathy C SVCO secondary to metastatic bronchogenic carcinoma d SVCO secondary to metastatic thyroid carcinoma e SVCO secondary to primary pulmonary lymphoma
Answer C: SVCO secondary to metastatic bronchogenic carcinoma Malignant lesions account for 80-90% of SVCO, and of these bronchogenic carcinoma accounts 50% of cases.
71
A 67-year-old retired musician was admitted to the acute medical ward at 7.00 am with dyspnoea. Examination revealed central cyanosis, tachypnoea and bilateral crepitations, which were most marked at the bases. Initial investigations revealed hypoxia, tachycardia and an abnormal ECG. A chest radiograph demonstrated perihilar alveolar opacification, interstitial thickening, small bi-basal effusions and upper lobe blood diversion. On further questioning it is apparent that the patient had been increasingly breathless for several months and subsequent echocardiography demonstrated cardiomyopathy . What echocardiographic finding would be most suggestive of a restrictive rather thana dilated cause for the cardiomyopathy? a Decreased systolic function b Isolated diastolic dysfunction C Cardiac mural thrombus d Reduced cardiac output e Increased L V cavity size
Answer B: Isolated diastolic dysfunction One of the hallmarks of restrictive cardiomyopathy is diastolic dysfunction, whereas systolic dysfunction is typical of dilated cardiomyopathy . The presence of mural thrombus and increased L V cavity size can be seen in both forms of cardiomyopathy .
72
Y ou are asked to review a chest radiograph of an 18-year-old female inpatient. The cardiac apex lies on the right side. The aortic knuckle is also seen on the right. Gas within the stomach is seen under the left hemidiaphragm. Otherwise the cardio-mediastinal appearances are unremarkable. The appearance and orientation of the ribs is normal. What is the most likely explanation for these findings? a Dextrocardia b Situs invertus c Situs solitus d Levoposition due to pectus excavatum e Asplenia
Answer A: Dextrocardia The appearances are those of dextrocardia. Situs invertus describes reversal of the thoracic and abdominal organs in a mirror-image fashion. Situs solitus refers to normal orientation. Severe pectus excavatum can lead to an abnormal position of the heart towards the right of the midline; however, the ribs would have an abnormal orientation (horizontal posterior ribs and steeply sloping anterior ribs - the so-called 7 appearance).
73
A 43-year-old woman presents with fever, malaise, arthralgia and myalgia. She also complains of pain in her left arm after activity and on examination her left radial pulse is weak. Her ESR is elevated but the other blood tests are unremarkable. A chest radiograph demonstrates an undulating contour of the lateral margin of the descending aorta and CT angiography reveals multifocal areas of thickening and enhancement of the wall of the thoracic aorta and left subclavian artery . Non-occlusive thrombus is seen within the left subclavian artery . What is the most likely diagnosis? a Polyarteritis nodosa b Rheumatoid vasculitis C Churg-Strauss syndrome d Microscopic polyangiitis e Takayasu's disease
Answer E: Takayasu's disease Takayasu's disease causes granulomatous inflammation of large arteries. Polyarteritis nodosa affects medium-sized vessels. The other options are causes of small vessel vasculitis.
74
A 62-year-old gentleman presented with syncopal episodes and intermittent pain and paraesthesia in his right hand especially when exerting his right arm. An MRI demonstrated obstruction of his right subclavian artery . Where in the artery is the obstruction most likely to be located? a First part of the artery , at its origin b First part of the artery , just distal to the right vertebral artery C Second part of the artery , just distal to the deep cervical artery d Second part of the artery , just proximal to dorsal scapular artery e just lateral to the lateral border of the scalenus anterior muscle
Answer A: First part of the artery , at its origin In the subclavian steal syndrome, there is stenosis of the first part of the subclavian artery proximal to origin of the vertebral artery . The scalenus anterior muscle divides the subclavian artery into three parts.
75
A 3 9-year-old gentleman with frequent respiratory tract infections and recent onset stridor was assessed with a CT of his thorax. A vessel was seen passing above the right main stem bronchus and coursing between the trachea and oesophagus. The trachea was deviated to the left and there was atelectasis in the right upper lobe. Which abnormal vessel is most likely to be present? a Aberrant left pulmonary artery b Aberrant right pulmonary artery C Double aortic arch d Aberrant right subclavian artery e Aberrant left subclavian artery
Answer A: Aberrant left pulmonary artery An aberrant left pulmonary artery arises from the right pulmonary artery and passes above the right main bronchus and between the trachea and oesophagus to reach the left lung. The right main stem bronchus may be bowed anteriorly and the trachea deviated to the left. There is an anterior indentation of the oesophagogram. Atelectasis and/or obstructive emphysema may be seen in the right (and/or left) upper lobe. It is associated with stenosis of the trachea, patent ductus arteriosus and absence of the pars membranacea.
76
A patient is awaiting investigation and treatment of a superficial neck mass, suspicious for non-Hodgkin's lymphoma. Their chest radiograph shows bilateral hilar lymphadenopathy . When is the best time to perform PET CT in view of gaining a histological diagnosis and commencing treatment? a W ait 1 week after neck dissection b W ait 4 weeks after neck dissection C W ait 4-6/52 after start of chemotherapy d Within 1 week of commencing chemotherapy e W ait 4-6 weeks after starting radiotherapy
Answer C: W ait 4-6/52 after start of chemotherapy Performing PET imaging after a procedure will produce inaccurate results as traumatised tissues have an increased metabolism and will mask any adjacent uptake due to pathology . Imaging should ideally wait six weeks post surgery , one week post biopsy , six weeks post chemotherapy and six months post radiation to avoid false positive uptake. However, some tumours such as GIST can show reduced metabolic activity and hence reduced uptake as soon as 24 hours following commencement of chemotherapy where conventional imaging will appear no different for weeks.
77
A 40-year-old female presented with shortness of breath and her chest radiograph was normal. She underwent a ventilation/perfusion study to investigate a possible pulmonary embolus. This showed two small, unmatched subsegmental defects in the left apical region. The ventilation images are normal. What is the correct report for this study? a Normal study b V ery low probability C Low probability d High probability , e Intermediate probability
Answer C: Low probability The Prospective Investigation of Pulmonary Embolus Diagnosis (PIOPED) criteria give a range of findings that can be reported, from normal study to high probability . The reporter should be familiar with the segmental anatomy and these criteria and the report should be taken into account along with the clinical probability .
78
An infant who is failing to thrive with difficulty feeding presents with increased work of breathing. A chest radiograph shows moderate cardiomegaly , bulky pulmonary vessels and fluid in the fissures. What is the most likely diagnosis? a Atrial septal defect b V entricular septal defect c Patent ductus arteriosus d Pulmonary artery stenosis e Aortic coarctation
Answer B: V entricular septal defect This child has a ventricular septal defect (VSD) causing a left to right shunt and has right and left-sided cardiac enlargement (enlarged left atrium). Seventy-five to eighty per cent of VSDs are membranous' (opening in the upper section of the ventricular septum near the valves) and 10-15 % are muscular' (opening in the lower section of the ventricular septum). Congestive heart failure rarely occurs in patients with PDA if the left to right shunt is large.
79
A routine baby check on a neonate born at term reveals bounding peripheral pulses and a continuous murmur, loudest under the clavicle. What is the most likely diagnosis? a Patent ductus arteriosus b V entricular septal defect C Atrio-ventricular septal defect d Pulmonary stenosis e Transposition of the great arteries
Answer A: Patent ductus arteriosus In patent ductus arteriosus (PDA) there is a persistent connection between the left pulmonary artery and descending aorta. This causes increased volume of blood to flow from the aorta through the PDA to the pulmonary artery and lungs and then to the left atrium, resulting in left atrial and ventricular enlargement, bounding peripheral pulses, and a continuous murmur. The right ventricle may be enlarged with pulmonary hypertension.
80
A 32-year-old man developed a low-grade fever and weight loss. He was previously well and had never smoked. CT shows lymphadenopathy on both sides of the diaphragm and a sample taken at mediastinoscopy showed Reed- Sternberg cells. There were no further positive findings in the rest of the thorax or abdomen. Where in the thorax is the lymphadenopathy most likely? a Anterior mediastinum b Middle mediastinum C Posterior mediastinum d Superior mediastinum e Hilar nodes
Answer A: Anterior mediastinum The Reed-Sternberg cell, although not common, is characteristic of Hodgkin's disease (which affects T -cells). The nodular sclerosing subtype is the most common and carries a relatively good prognosis. There is a bimodal distribution with peaks in the 25-30 and 75-80 age groups. Anterior mediastinal and retrosternal nodes are more commonly involved. The presence of a pleural effusion is not of prognostic significance.
81
A 75-year-old gentleman who had worked in the construction industry had a chest radiograph prior to an elective cholecystectomy . Multiple calcified pleural plaques were visible bilaterally with lower zone predominant reticular opacification. What is the most likely pleural manifestation of this disease? a Diffuse pleural thickening b Pleural effusion C Focal pleural plaques d Pleural calcification e Mesothelioma
Answer B: Pleural effusion Pleural effusion is often the earliest abnormality in asbestos-related pleural disease but focal pleural plaques are more common during the later part of the disease. V ery fine fibres such as crocidolite are more likely to result in extensive pleural disease.
82
A patient with long-standing severe rheumatoid arthritis developed progressive dyspnoea. What is the most frequent intra-thoracic manifestation of the disease? a Pleural disease b Interstitial fibrosis C Bronchiectasis d Bronchiolitis obliterans e Pulmonary nodules
Answer A: Pleural disease Rheumatoid lung occurs more frequently in males with rheumatoid arthritis (although rheumatoid arthritis is more common in females). Pleural involvement is the most common thoracic manifestation while rheumatoid nodules are the rarest.
83
A 56-year-old man presented with recurrent episodes of fever, dry cough and dyspnoea. On specific questioning he states he breeds pigeons. What feature would make the diagnosis of extrinsic allergic alveolitis (EAA) less likely? a Pleural effusion b A normal chest radiograph c Fibrosis of middle and lower zones d Traction bronchiectasis e Diffuse ground-glass attenuation on high resolution CT
Answer A: Pleural effusion In both the acute and late-phases of EAA a normal chest radiograph is seen in 30- 95%. No radiological study is pathognomonic, but appearances that would support a diagnosis include patchy non-specific pneumonitis, small pulmonary nodules (which may be so small they give the appearance of ground-glass consolidation) and more chronic changes reflecting healing fibrosis such as bronchiectasis and scarring. Although the horizontal fissure may become thickened, in general pleural disease is not a feature.
84
A 54-year-old female presented with progressive dyspnoea and diffuse pulmonary opacities on her chest radiograph. An HRCT revealed interlobular septal thickening bibasally . What further findings would suggest a diagnosis of lymphangitis carcinomatosis rather than cardiogenic pulmonary oedema? a Nodular interlobular septal thickening b Pleural fluid C Ground-glass opacification d Centrilobular nodules e Mediastinal lymphadenopathy
Answer A: Nodular interlobular septal thickening In lymphangitis carcinomatosis the interlobular septal thickening is caused by tumour infiltration and is more often irregular or nodular in appearance. The remaining features would not be unusual in either condition.
85
A 50-year-old man who presented with progressive dyspnoea had a chest radiograph that demonstrated multiple opacities between 0.5 and 2 mm in size, which were noted to be more dense than soft tissue. Which of the following diagnoses is most likely? a Fungal infection such as histoplasmosis b Coal miners' pneumoconiosis C Sarcoidosis d Acute extrinsic allergic alveolitis e Silicosis
Answer E: Silicosis All the other conditions would produce nodules of soft tissue density . In pure silicosis the nodules are very well defined and very dense. There is also relative sparing of the bases and apices with septal lines on HRCT.
86
A 64-year-old female was seen in the Emergency Department for dyspnoea. She was otherwise fit and well and had no significant past medical history . A full blood count showed a mild neutrophilia and a chest radiograph revealed significant elevation of the right hemidiaphragm. What is the most likely cause? a Herpes zoster infection b Previous iatrogenic trauma C Compression of the phrenic nerve by a tumour d Peripheral neuropathy secondary to cervical spondylosis e Compression of the phrenic nerve caused by a thoracic aortic aneurysm
Answer C: Compression of the phrenic nerve by a tumour This is largely a question of probability , as the clinical history is non-specific. A tumour causing compression is the commonest cause in adults.
87
At the weekly respiratory multidisciplinary team meeting you are asked to review a chest radiograph and CT of a 39-year-old man who presented with cough, fever, dyspnoea and chest pain. Blood biochemistry had demonstrated renal impairment. The imaging reveals bilateral reticulonodular interstitial opacification. The nodules vary in size, and the larger nodules show cavitation. What is the most likely diagnosis? a W egener's granulomatosis b Eosinophilic pneumonia C Systemic lupus erythematosus (SLE) d Multiple pulmonary emboli e Goodpasture's syndrome
Answer A: W egener's granulomatosis The clinical differential diagnosis is between W egener's granulomatosis and Goodpasture's syndrome and the imaging findings are classical for W egener's granulomatosis. Findings in Goodpasture's syndrome include consolidation with relative apical sparing in the acute stage, followed by an interstitial pattern of opacification in the later stages of the disease.
88
A 32-year-old artist presented with chest pain and dyspnoea. Physical examination was unremarkable but blood gas analysis showed her to be markedly hypoxic and a pulmonary embolus was suspected. A CT pulmonary angiogram (CTP A) was performed with bolus tracking (threshold triggering); however, the pulmonary arterial contrast opacification was sub-optimal. The intravenous cannula is well sited and there is no overt sign of swelling around the cannula site. The patient took a deep breath just prior to the scan and the scan appeared to trigger appropriately with the region of interest (ROI) sited over the main pulmonary artery . What is the most likely cause for the sub- optimal opacification of the pulmonary vessels? a Left to right shunt b Contrast extravasation at the injection site C Right to left shunt d Dilution of opacified blood with unopacified blood e Hypodynamic circulation
D This is a common problem with the CTP A, especially with younger patients. As they take in a deep breath just prior to the scan their intrathoracic pressure is reduced and unopacified blood is drawn up from the IVC. This effectively dilutes the opacified blood entering from the SVC. One way to avoid this pitfall is to ask the patient to only take a modest breath prior to scanning.
89
Y ou have been asked to review a chest radiograph by a junior doctor. The image demonstrates subtle hazy opacification of the upper part of the lower zone of the right lung. The right atrial border is indistinct and the horizontal fissure runs from the right hilum to the eighth rib in the mid axillary line. What is the most plausible explanation for these findings? a Middle lobe collapse b Middle lobe consolidation C Pectus excavatum d Right lower lobe mediobasal segment consolidation e Right lower lobe anteriobasal segment consolidation
Answer A: Middle lobe collapse The findings are those of middle lobe collapse. Signs on the frontal radiograph can be subtle, and it is more easily seen on the lateral radiograph. In this case the loss of clarity of the right atrial border indicates the pathology is located in the middle lobe. There is loss of volume (the normal horizontal fissure runs from the hilum to the sixth rib in the mid axillary line), therefore collapse of the middle lobe, rather than consolidation, is the likely cause for these appearances.
90
A 62-year-old widow presented to the Neurology Clinic with fatigue and weakness, particularly after minimal exertion. Cranial nerve examination revealed rapid fatigability of the facial muscles and her chest radiograph showed mediastinal widening. The patient then underwent contrast enhanced CT of the chest. Which of the following findings best fits the given clinical scenario? a Diffuse mediastinal adenopathy and a widespread interstitial thickening b Retrosternal goitre demonstrating areas of necrosis and haemorrhage with avid contrast enhancement C Isolated homogeneous soft tissue mass within the anterior mediastinum outlined by fat d Diffuse, invasive mass containing areas of haemorrhage and calcification encasing the major vessels e Large, eccentric aortic arch aneurysm
Answer C: Isolated homogeneous soft tissue mass within the anterior mediastinum outlined by fat The neurological findings are classic of the myasthenia gravis - an autoimmune disorder characterised by antibodies against postjunctional acetylcholine receptors. The condition is often associated with thymoma.
91
A retired railway worker underwent plain chest radiography and CT scanning. What appearance would be most in keeping with benign asbestos-related pleural plaque disease? a Asymmetric pleural thickening of the right lung, predominantly involving the mediastinal pleural surface. No calcification is evident; however, there is a moderately large pleural effusion. b Bilateral focal pleural thickening involving the apices and costophrenic angles, but sparing the diaphragms. Most of the plaques are calcified. c Bilateral focal pleural thickening predominantly affecting the diaphragms and lower thorax, sparing the apices, costophrenic angles and mediastinal pleura. Only a few of the plaques are calcified. d Bilateral focal pleural thickening involving the lower thorax and sparing the costophrenic angles, diaphragms, mediastinal pleura and apices. Most of the plaques are calcified. e Bilateral focal pleural thickening involving the costophrenic angles and lower thorax, but sparing the diaphragms, mediastinal pleura and apices. Only a few of the plaques are calcified.
Answer C: Bilateral focal pleural thickening predominantly affecting the diaphragms and lower thorax, sparing the apices, costophrenic angles and mediastinal pleura. Only a few of the plaques are calcified. The typical appearances of benign asbestos-related pleural plaques are bilateral focal pleural thickening of the chest wall between the seventh and tenth ribs and diaphragms with sparing of the costophrenic angles, apices and mediastinal pleura. Plaques show a predilection for the posterior-lateral portion of the chest wall and may or may not be calcified. Appearances raising the possibility of malignant mesothelioma include concentric pleural thickening involving the mediastinal pleura, pleural effusion and effacement of the subpleural fat plane.
92
A 60-year-old man has chronic obstructive pulmonary disease (COPD). He has a long smoking history and is being considered for lung volume reduction surgery (L VRS). What pattern of disease would give the best chance of a successful outcome following LR VS? a Mild, predominantly upper and mid zone paraseptal emphysema b Severe lower zone bullous emphysema and mild upper zone paraseptal emphysema C Severe upper zone bullous emphysema with relatively spared lower zones d Severe upper zone bullous emphysema with moderate lower zone centrilobular emphysema e Severe centrilobular emphysema affecting all zones
Answer C: Severe upper zone bullous emphysema with relatively spared lower zones. Lung volume reduction surgery (L VRS) is a palliative procedure for patients with advanced disease. It has a number of clinical exclusion criteria because of relatively high operative risk. There are still some controversies around this form of surgery , but it is likely to continue to have a place in the treatment of emphysema. It comprises wedge resection of the areas of greatest disease, mainly the upper lobes, thus improving the performance of the remaining lung. Best candidates for surgery have upper lobe predominant emphysema, a good amount of normal or mildly emphysematous lung and significant regional heterogeneity on perfusion scintigraphy .
93
A patient underwent endovascular repair of an abdominal aortic aneurysm. After the endograft had been successfully sited a check angiogram showed opacification of the aneurysm sac by retrograde flow through the inferior mesenteric artery . What type of endoleak is this? a Type I endoleak b Type II endoleak C Type III endoleak d Type IV endoleak e Type V endoleak (endotension)
Answer B: Type II endoleak Endoleaks are common immediately post EV AR and most resolve spontaneously . They are classified as follows: I Leak from proximal or distal graft attachment site II Retrograde filling of sac from persistent collateral vessel III Leak from midgraft/component junction IV Leak through porous graft material
94
A 17-year-old student presented with an acutely ischaemic left calf and foot and on further questioning he described several months of claudication in his left calf which was worse during periods of prolonged standing. He also reluctantly admitted to a single episode of intravenous drug use through the antecubital fossa. Lower limb arteriography demonstrated stenosis and post- stenotic dilatation of the popliteal artery , which was noted to be deviated medially within the popliteal fossa. What is the most likely diagnosis? a Atherosclerosis b Baker's cyst c Buerger's disease d Mycotic aneurysm e Popliteal entrapment
Answer E: Popliteal entrapment In popliteal entrapment the popliteal artery is displaced medially around the medial head of gastrocnemius. It is common in young athletes, and should be considered in leg ischaemia without trauma in this age group.
95
An elderly former smoker with worsening ischaemic symptoms in both legs underwent diagnostic lower limb angiography . Independent of the signs and symptoms of his disease, what is the most likely site of atherosclerotic disease in the lower limb? a Iliac artery b Common femoral artery c Superficial femoral artery d Popliteal artery e Tibial artery
Answer C: Superficial femoral artery Atherosclerotic disease typically has a symmetrical pattern and develops at points of turbulent flow (e.g. bifurcations). In the lower limb the commonly affected sites are: SFA > Iliac artery > Tibial artery > Popliteal artery > CFA.
96
46-year-old male who was in a high-speed road traffic accident presents acutely to the Emergency Department. He has severe chest pain radiating to his back and is haemodynamically unstable. What finding on an anterior posterior chest radiograph is most specific for acute thoracic aortic injury? a Widening of the mediastinum b Widened right paratracheal stripe c Indistinct aortic arch contour d Obscuration of the aortopulmonary window e Right-sided haemothorax
Answer C: Indistinct aortic arch contour
97
A three-week-old baby had a chest radiograph to investigate tachypnoea and mild cyanosis which showed cardiomegaly . An echocardiogram revealed a dilated right atrium and abnormal tricuspid valve with a small, dysplastic but functioning right ventricle. What is the most likely diagnosis? a Tricuspid atresia b Ebstein's anomaly C Myocarditis d An atrial septal defect e Cor triatriatum
Answer B: Ebstein's anomaly The description is that of Ebstein's anomaly . Tricuspid atresia would not have a functioning right ventricle.
98
A patient with extensive, multi-system arterial disease was scheduled for a lower limb vascular study and intervention. Due to their comorbidities carbon dioxide was considered as a contrast agent rather than iodinated contrast. When should carbon dioxide not be used? a Intra-arterially below the diaphragm b Intra-arterially in suspected arteritis c Intravenously in the presence of an inferior vena cava filter d Intravenously in Eisenmenger's syndrome e Intravenously in the presence of a deep venous thrombosis
Answer D: Intravenously in Eisenmenger's syndrome Carbon dioxide is a useful negative vascular contrast agent in situations where iodinated contrast is contraindicated. The main risk is cerebral toxicity and it should therefore be avoided intra-arterially above the diaphragm and intravenously in patients with a right-to-left shunt.
99
A 43-year-old male presented to the Cardiology team with a long history of coronary heart disease. His chest radiograph demonstrated enlargement of the left ventricular apex. What characteristic would make a diagnosis of a true ventricular aneurysm more likely than a false aneurysm? a A mouth considerably smaller than the maximal diameter b No myocardial fibres in the wall C An aneurysm that protruded only in systole d A previous history of myocardial infarction e Thrombus within the aneurysm
Answer C: An aneurysm that protruded only in systole Although most true aneurysms protrude in diastole and systole, a functional aneurysm protrudes only in systole. All false aneurysms protrude in diastole and systole.
100
A 57-year-old man with diabetes mellitus presented with anterior chest pain on minimal exertion and an exercise tolerance test was positive. Cardiac angiography demonstrated 70% stenosis of the circumflex, 90% stenosis of the left anterior descending and complete occlusion of the right coronary arteries. It was agreed with the patient that coronary artery bypass surgery was appropriate. Which of the following native grafts is most appropriate for bypassing the left anterior descending artery? a Saphenous vein b Left internal mammary artery C Left superior epigastric artery d Radial vein e An intercostal artery
Answer B: Left internal mammary artery The distal left anterior descending (LAD) artery lies anteriorly in the chest. The left internal mammary artery (LIMA) lies along the anterior chest wall and passes close to the LAD in the interventricular groove, which makes it an ideal graft. Additionally , arteries make more sustainable graft material.
101
A 57-year-old woman presented with reduced exercise tolerance and shortness of breath. No specific abnormality was found on clinical examination. Chest radiography showed enlarged central pulmonary arteries and subsequent chest CT confirmed pulmonary artery enlargement and also showed right ventricular dilation. What additional feature would make chronic thromboembolism a more likely diagnosis than systemic to pulmonary circulation shunting? a Pleural effusion b Interstitial septal lines C Mosaic attenuation d Flattening of the interventricular septum e Reflux of contrast into the inferior vena cava
Answer C: Mosaic attenuation Mosaic attenuation is a common feature of chronic thromboembolism; it can be seen in left to right shunts but this is much less common and tends to be more diffuse.
102
A 55-year-old female presented to the Emergency Department with acute central chest pain and shortness of breath. Her daughter had died recently following a post-partum haemorrhage. She was hypotensive with signs of left cardiac failure and her troponin T was elevated. Coronary angiography demonstrated normal coronary arteries and cardiac MRI was performed. This revealed apical hypokinesis and dilatation with normal basal function. There was no myocardial delayed hyperenhancement. Subsequent echocardiography 12 weeks later showed improved left ventricular function. What is the most likely diagnosis? a Myocardial infarction involving the left anterior descending artery b Myocarditis C Hypertrophic cardiomyopathy d Coronary artery spasm e Tako-tsubo cardiomyopathy
Answer E: Tako-tsubo cardiomyopathy This description is classic of Tako-tsubo cardiomyopathy , which is also known as transient catecholaminergic myocardial stunning. It often occurs following a stressful event and most patients recover completely .
103
A 47-year-old lady presented with sudden onset right hemiparesis after lifting a heavy shopping bag. CT of her brain demonstrated two foci of low attenuation within the posterior frontal lobe and anterior frontal lobe adjacent to the interhemispheric fissure on the left which involved both white and grey matter. In addition she was also noted to have an erythematous, swollen left calf which was confirmed with a Doppler study to be due to thrombus within the superficial femoral vein. What is the next most appropriate radiological investigation? a CT venography b CT arterography C Echocardiography d Abdominal ultrasound e MRI brain
Answer C: Echocardiography In the presence of a deep-vein thrombosis (DVT) an embolic stroke raised through a right to left shunt should be investigated. Under certain circumstances, for example during temporary raised intrathoracic pressure when lifting a heavy shopping bag, in the presence of an atrial septal defect (ASD) or ventricular septal defect (VSD), emboli may cross from the venous to arterial circulation. Therefore, in this case the search for a cardiac septal defect (most likely to be an ASD) with echocardiography is indicated.
104
A patient undergoing echocardiography for an acyanotic shunt had the following findings on imaging: dilated left atrium and ventricle, dilated right ventricle, undilated right atrium and undilated aorta. What is the most likely cause of the shunt? a Ostium primum atrial septal defect (ASD) b V entricular septal defect (VSD) c Patent foramen ovale d Ostium secundum ASD e Patent ductus arteriosus (PDA)
Answer B: Ventricular septal defect (VSD) The following features can differentiate between the position of acyanotic shunts: ASD - Right atrium + Ventricle Enlargement/No dilated aorta VSD - Left atrium + ventricle + Right ventricle/No dilated aorta PDA - Left atrium + Ventricle/Aorta dilated
105
A 25-year-old presented with shortness of breath after minimal exercise. His general practitioner (GP) examined his cardiovascular system and noted a harsh ejection systolic murmur in the left parasternal region. No other murmurs were detected and there were no other positive clinical findings. His chest radiograph showed calcification in the region of the aortic valve. Which of the following is the most likely aetiology? a Congenital bicuspid aortic valve b Aortic valve atherosclerosis c Rheumatic valve disease d Previous endocarditis e Patent ductus arteriosus
Answer A: Congenital bicuspid aortic valve The murmur description is consistent with aortic stenosis. In an otherwise young and healthy individual, a congenital bicuspid aortic valve is most likely . These individuals can become clinically symptomatic under the age of 30. Aortic valve calcification is a not uncommon finding on the chest radiograph. Atherosclerotic disease of the aortic valve presents later in life. Rheumatic aortic valve disease is rare in the absence of mitral valve disease.
106
A 23-year-old intravenous drug abuser presented to the Emergency Department with fever and swelling in his left groin. He was also noted to be short of breath at rest with peripheral cyanosis. A chest radiograph demonstrated widespread bilateral foci of consolidation. Ultrasound of his left groin demonstrated a superficial abscess with non-occlusive thrombus in the left common femoral vein. An echocardiogram performed by a cardiologist showed an echogenic intracardiac mass. What is the best explanation for these findings? a Tricuspid valve endocarditis and septic pulmonary emboli b Intracardiac bland thrombus and Mycoplasma pneumonia C Intracardiac bland thrombus and bland pulmonary emboli d Pulmonary valve endocarditis and septic pulmonary emboli e Mitral valve endocarditis and Mycoplasma pneumonia
Answer A: Tricuspid valve endocarditis and septic pulmonary emboli Intravenous drug abusers are prone to right-sided valvular endocarditis from organisms introduced to the venous system while injecting. These vegetations, seen most commonly on the tricuspid valve, often throw off emboli to the lungs.
107
A 33 year old HIV positive woman presents with increasing headache and confusion. O/E she is pyrexial and has left leg and right facial weakness. A CT Head demonstrates multiple lesions measuring between 2-4 cm which are predominantly situated at the corticomedullary junction. The lesions have a thin enhancing rim as well as associated oedema and local mass effect. Which one of the following is the most likely diagnosis? A Cryptococcosis B Histiocytosis C HIV Encephalopathy D Multiple cerebral metastases E Toxoplasmosis
E - Toxoplasmosis
108
A 28 year old woman suffers a 1 week episode of diarrhoea and vomiting due to the Norwalk virus. She is noted by her boyfriend to be increasingly lethargic at home and is unable to tolerate oral fluids. She then becomes confused and agitated and suffers a generalised tonic clonic seizure. Which one of the following radiological findings is most likely. A Focal high signal on FLAIR images within the right cerebellar hemisphere B Focal ovoid lesions of high FLAIR signal in the periventricular white matter C High signal on FLAIR images in the occipital lobes D High signal on FLAIR images in the parasaggital cortex bilaterally E Loss of grey-white matter differentiation in the region of the basal ganglia and insular bilaterally
D High signal on FLAIR images in the parasaggital cortex bilaterally Hx of venous sinus thrombosis. Most common site is the superior saggital sinus - can lead to venous infarcts affecting the parasaggital areas and dont conform to arterial territories
109
A 44yr old woman has a history of pain and swelling in her left cheek, particularly after eating. Her GP is suspicious that she has a parotid duct calculus and refers her for sialography. Which one of the following statements is true regarding sialography? A Approximately 10 mL contrast is usually required to fill the parotid duct and branches B High osmolar contrast media are contraindicated. C It is contraindicated in acute infection D Pain post procedure warrants further investigation E The orifice of the parotid duct is adjacent to the 2nd upper pre molar
C It is contraindicated in acute infection A control film is performed initially. Up to 2 ml contrast (High or low osmolar is injected before further images are taken. The procedure is contraindicated in acute infection or inflammation. Pain, duct rupture and infection are recognised complications.
110
A young patient undergoes CT of the paranasal sinuses. The main finding is an enhancing nasal mass with widening of the left pterygopalatine fissure. What is the most likely diagnosis? A Adenoid cystic carcinoma B Angiofibroma C Angiosarcoma D Inverting papilloma E Lymphoma
B Angiofibroma A nasal mass with widening of the pterygopalatine fissure is pathognomonic of juvenile angiofibroma
111
A 5 year old boy presents with rapidly progressive right sided proptosis. On examination, he is noted to have lateral deviation of the right eye but visual acuity is normal. Ct of the orbits reveals a large, isodense mass in the superomedial right orbit. The extra-ocular muscles cannot be seen separately and there is destruction of the medial wall of the bony orbit. The mass displays uniform enhancement post contrast. What is the most likely diagnosis? A Capillary haemangioma B dermoid cyst C Intraconal schwannoma D Retinoblastoma E Rhabdomyosarcoma
E Rhabdomyosarcoma High malignant tumour most common in 2-5 year olds
112
A 46yr old woman presents with a painful left eye. She has enopthalmos on clinical examination . CT reveals a mass arising from the greater wing of the left sphenoid with some underlying bone destruction. The mass is poorly mariginated and infiltrating the intraconal compartment. What is the most likely diagnosis? A Caroticocavernous fistula B Lymphoma C Metastatic breast carcinoma D Orbital dermoid E Orbital varix
C Metastatic breast carcinoma Characteristic appearance of metastatic scirrhous breast carcinoma
113
A 33 year old woman presents to the ED with a reduced conscious level. She has been generally unwell with fever, malaise and a dry cough for several weeks and more recently has developed a left facial nerve palsy. An MRI brain reveals nodular thickening and enhancement of the dura and leptomeninges. There is also enhancement of the optic tracts and optic chiasm, as well as the pituitary infundibulum. A few small foci of high T2 signal are demonstrated in the periventricular white matter. What is the most likely diagnosis? A MS B LCH C PML D Sarcoidosis E Wilsons disease
D Sarcoidosis Enhancement involving the optic apparatus, floor of 3rd ventricles and pituitary infundibulum is suggestive of sarcoidosis
114
A 21 year old man has facial and mandibular radiographs following minor trauma. These show no evidence of fracture, however there are multiple dense bony lesions arising from the paranasal sinuses and the angle and ramus of the mandible. These lesions are entirely asymptomatic. Which one of the following conditions may be associated with these findings? A Gardners syndrome B Gorlin-Goltz syndrome C Juvenile polyposis D Klippel Feil syndrome E Turners syndrome
A Gardners syndrome Multiple maxillofacial osteoma's are a feature of FAP (Gardners syndrome. They precede the colonic polyposis.
115
A 77-year-old gentleman suffers a ruptured abdominal aortic aneurysm for which he undergoes emergency surgery. Early in the postoperative period he develops acute lower back pain and is incontinent. An urgent MRI is performed. What are the most likely radiological findings? A Enhancing, heterogeneous intramedullary mass lesion B Extradural collection with peripheral enhancement C Focal high T2 signal within the spinal cord with mild cord swelling D Large central disc herniation at the affected level with associated high T2 signal within the cord E Marked diffuse swelling of the spinal cord at the affected level
C These are the expected findings in spinal cord infarction and this clinical history is typical.
116
An 82-year-old woman who was previously well has had several falls in the past 2—3 months. She has become increasingly confused but there are no localising neurological signs on physical examination. Which one of the following is the most likely finding on CT? A High attenuation in both Sylvian fissures, the basal cisterns and the lateral ventricles. B High attenuation overlying the right frontoparietal cortex and extending into the interhemispheric space with midline shift to the left. C Mixed attenuation areas overlying both hemispheres with normal ventricles and no midline shift. D High attenuation lentiform collection overlying the left temporal region with effacement of the left lateral ventricle. E Superficial areas of low density in the right frontotemporal region, containing small foci of high attenuation.
C This is a typical history of acute on chronic subdural haemorrhage. Option A describes subarachnoid blood, B describes an acute subdural haemorrhage, D describes an extradural haemorrhage and E describes cerebral contusions.
117
A 7-year-old boy is referred from the paediatric clinic for an MRI brain to investigate learning difficulties and abnormal gait. The MRI reveals high T2 signal in the splenium and posterior body of the corpus callosum as well as in the peritrigonal white matter. There is marginal enhancement at the anterior edge of the abnormal area. Which one of the following is the most likely diagnosis? A Adrenoleukodystrophy B Alexander disease C Kearns-Sayer syndrome D Krabbe’s leukodystrophy E Maple syrup urine disease
A This is the most common leukodystrophy of children. Demyelination begins in the posterior central white matter and progresses to the corticospinal tracts and visual and auditory pathways. The leading edge shows enhancement due to active inflammation.
118
A 19-year-old HIV-positive man is admitted with headaches, confusion and disorientation. He is mildly pyrexial. A CT brain reveals multiple hypodensities, particularly in the brainstem and in the periventricular white matter. There is some ependymal enhancement postcontrast. What is the most likely cause for these findings? A CMV encephalitis B Cryptococcosis C HIV encephalitis D Toxoplasmosis E Tuberculosis
A Cerebral CMV infection usually presents as encephalitis, ventriculitis, infarcts or meningitis. The typical sites for encephalitis are the brainstem and periventricular white matter. Cryptococcus usually causes a meningitis which is poorly seen on imaging. HIV encephalitis manifests as demyelination and gliosis characteristically in the centrum semiovale. Toxoplasmosis is characterised by ring-enhancing lesions at the corticomedullary junction and in the basal ganglia and thalamus. Lastly, tuberculosis causes multiple granulomata (initially hypodense on CT with little enhancement but subsequently calcify following treatment) and leptomeningeal disease.
119
A 35-year-old man is admitted from outpatient clinic with a history of worsening lower back pain and leg weakness. Plain lumbar radiographs reveal expansion of the spinal canal at L3—4 level. He has an urgent MRI which shows a lobulated extramedullary mass at this level, causing nerve root compression. The mass is hyperintense to the spinal cord on T2w images and there is an associated paravertebral mass. What is the most likely diagnosis? A Astrocytoma B Disc extrusion C Ependymoma D Meningioma E Neurinoma
E Neurinomas and meningiomas are the commonest intradural extramedullary tumours. Neurinomas may occur at any level whereas meningiomas tend to be thoracic and are very rare in the lumbar spine.
120
A 67-year-old woman with known osteoarthritis presents with lower back pain radiating down her left leg. She has an MRI of the lumbar spine which shows a lesion at the L4—5 facet joint with compression of the thecal sac at this level. The lesion is of intermediate signal on T2w images and is displacing the ligamentum flavum. What is the most likely diagnosis? A Astrocytoma B Disc protrusion C Ependymoma D Osteophyte E Synovial cyst
E Synovial cysts may be solid with cartilaginous or myxomatous components.
121
A previously well 70-year-old man is admitted with an acute onset of left-sided weakness and dysphasia. CT shows low attenuation in the right temporoparietal region with loss of normal grey—white matter differentiation. He undergoes an MRI brain one week later. What are the most likely radiological findings? A High FLAIR signal in the right temporoparietal region with no enhancement seen postcontrast B High T2 signal in the right temporoparietal region with gyriform enhancement seen postcontrast C High T2 signal in the right temporoparietal region with no enhancement seen postcontrast D Intermediate T2 signal in the right temporoparietal region with no enhancement seen postcontrast E Uniformly high signal on gradient echo images in the right temporoparietal region
B In the subacute phase of cerebral infarction there is disruption of the bloodbrain barrier and structural breakdown leading to oedema. Contrast enhancement is seen on MRI in almost all cases by the end of the first week and a gyriform pattern is most characteristic.
122
A 22-year-old woman presents with visual loss and headaches. On examination, she has bilateral visual field defects and decreased visual acuity. CT reveals foci of calcification at both optic nerve heads. What is the most likely diagnosis? A Choroidal haemangioma B Drusen C Leukaemia D Optic neuritis E Sclerosing endophthalmitis
B
123
A 14-year-old girl with a kyphoscoliosis has multiple skin lesions which have been characterised as basal cell tumours. In addition she has had investigations which have revealed calcification of the falx and several bifid ribs. Which of the following findings would be most likely on a dental panoramic radiograph? A A large well-defined lucency inferior to the inferior alveolar canal B Multiple ill-defined lucent lesions C Multiple sclerotic lesions, particularly around the angle of the mandible D Multiple small periapical lucencies E Multiple well-defined multiloculated lucencies
E This is Gorlin-Goltz syndrome, in which there are multiple odontogenic keratocysts.
124
A 32-year-old woman with a known history of excessive alcohol intake presents with a lump on the left side of her neck. She has an ultrasound scan which demonstrates a solitary nodule of mixed reflectivity in the left lobe of her thyroid which measures 3 cm in diameter. She also has several enlarged, uniformly hypoechoic cervical lymph nodes. On thyroid scintigraphy a low uptake region is seen corresponding to the site of the nodule. What is the most likely diagnosis? A Colloid nodule B De Quervain's thyroiditis C Follicular carcinoma of thyroid D Graves' disease E Papillary carcinoma of thyroid
E Thyroid cancers tend to be cold on scintigraphy. Papillary carcinoma is the commonest thyroid tumour (50-80%) and spreads early to local lymph nodes.
125
A 24-year-old woman presents to her GP with jaw stiffness and headaches.Her partner complains that she grinds her teeth during the night. She attends for an MRI scan to visualise the temporomandibular joints. Sequences are performed with the mouth open and closed. Which of the following radiological findings are most likely? A Anterolateral displacement of the biconcave articular disc on mouth closing B Anteromedial displacement of the biconcave articular disc on mouth opening C Anteromedial displacement of the biconvex articular disc on mouth opening D Posterior displacement of the biconcave articular disc on mouth opening E Posterior displacement of the biconvex articular disc on mouth opening
B
126
An 18-morith-old boy with multiple skin lesions, developmental delay and seizures has an MRI of the brain. This demonstrates several small subependymal nodules that are isointense to white matter on T2w images and project into the lateral ventricles. There is also a small, well-demarcated mass at the foramen of Monro which is hyperintense on T2w images and displays uniform enhancement. What is the most likely underlying diagnosis? A Neurocutaneous melanosis B Medulloblastoma C Neurofibromatosis type 1 D Neurofibromatosis type 2 E Tuberous sclerosis
E This patient has subependymal hamartomas (which can calcify) and a giant cell astrocytoma. Other CNS features include cortical tubers.
127
An 18-month-old child with a facial port wine stain presents witli reduced movement on the left side and developmental delay. Her mother also reports left-sided focal seizures. What are the most likely radiological findings? A Atrophy of the left cerebral hemisphere with enhancement overlying the left parietal cortex and enlargement of the right choroid plexus B Atrophy of the right cerebral hemisphere with enhancement overlying the right parietal cortex and enlargement of the right choroid plexus C Cystic dilatation of the fourth ventricle with hypoplasia of the vermis and hydrocephalus D Hydrocephalus, inferior displacement of the cerebellar tonsils and elongation of the fourth ventricle E Multiple small calcified subependymal nodules, a partly cystic mass at the foramen of Monro and several retinal lesions
A The clinical history describes Sturge-Weber syndrome. The imaging findings include leptomeningeal angiomas on the same side as the facial port wine
128
A 37-year-old man is brought to the Emergency Department with extensive facial injuries following a bicycle accident. Facial radiographs demonstrate multiple fractures. Which one of the following descriptions of Le Fort fractures is correct? A Le Fort I: bilateral fractures of the rami of the mandible B Le Fort I: fractures of the nasal bridge and medial orbital walls C Le Fort II: bilateral fractures of the rami of the mandible and both zygomatic arches D Le Fort II: fractures through the nasal bridge, lacrimal bones and medial orbital walls extending to the pterygoid plates E Le Fort III: fractures through the nasal bridge, lacrimal bones and medial orbital walls extending to the pterygoid plates
D The Le Fort classification applies to maxillary fractures and none of them involve the mandible.
129
A 65-year-old woman sees her GP with diarrhoea, palpitations and fatigue. Clinical examination of her neck is normal, but her thyroid function tests are consistent with hyperthyroidism and she is referred for thyroid scintigraphy. Which one of the following statements is true regarding radionuclide thyroid imaging? A Iodine 123 is taken up by the salivary glands. B If iodine 123 is used, imaging should be performed immediately after the injection. C If Tc-99m pertechnetate is used, imaging should be performed 4—6 hours after the injection. D Increased uptake may be seen in the pyramidal lobe in normal individuals. E It is contraindicated in patients with known parathyroid malignancy.
D
130
A 32-year-old woman presents with infertility and irregular periods. Investigations performed by her GP demonstrate a markedly elevated prolactin level. What would be the most likely MRI finding? A A 6-mm avidly enhancing mass in the posterior pituitary B A 6-mm poorly enhancing mass in the anterior pituitary C A 6-mm poorly enhancing mass in the posterior pituitary D A 20-mm avidly enhancing mass in the anterior pituitary E A 20-mm avidly enhancing mass in the posterior pituitary
B Prolactinomas are the most common functioning pituitary microadenoma. They typically arise laterally in the anterior lobe of the pituitary gland.
131
A 53-year-old woman is referred for an ultrasound by her GP as she has a lump in her right cheek which has grown slowly over a period of 8-10 months. Ultrasound demonstrates a multiloculated, predominantly hypoechoic mass in the right parotid gland. She goes on to have an MR1 which confirms a welldefined multiloculated mass. It is of low signal on Tlw images and high signal on T2w images. What is the most likely diagnosis? A Adenoid cystic carcinoma B Lipoma C Lymphoma D Pleomorphic adenoma E Warthin's tumour
D
132
An 18-year-old HIV-positive man is admitted to hospital with headaches, drowsiness and increasing confusion. A CT head reveals dilatation of the ventricles, hyperdensity of the basal cisterns and several small isodense and hypodense lesions at the corticomedullary junction. Postcontrast images show enhancement of the basal cisterns and ring enhancement of the corticomedullary lesions. What is the most likely diagnosis? A Cryptococcosis B Leptomeningeal metastases C Sarcoidosis D Toxoplasmosis E Tuberculosis
E The imaging appearance describes basal leptomeningeal involvement in CNS tuberculosis as well as the presence of tuberculomas. Sarcoidosis may have similar appearances but tuberculosis (especially with the presence of tuberculomas or abscesses) is more likely in HIV infection.
133
A 3-year-old child with scoliosis attends for an MRI of the spine. The sagittal images show apparent thinning of the lower thoracic spinal cord and abnormal signal within the cord. Axial images show that the cord returns normal signal and is split in two over the length of two vertebral bodies, with a cleft of CSF between the two hemicords. Which of the following may be associated with these findings? A Atrophy of the filum terminale B Bifid vertebrae C Descent of the cerebellar tonsils below the foramen magnum D High conus medullaris E Narrowing of the interpedicular distance
B This describes diastematomyelia which has several associations, including hemivertebrae and bifid vertebrae.
134
A 51-year-old man presents with painless swelling of the left eye and is found to have left-sided proptosis on clinical examination. MRI demonstrates low T1 signal and high T2 signal in the left lacrimal gland and superior rectus muscle. These areas display enhancement following contrast administration. The bony orbit appears normal. What is the most likely diagnosis? A Capillary haemangioma B Cavernous haemangioma C Graves' disease D Lymphoma E Metastasis
D Orbital lymphoma typically presents with painless orbital swelling. It moulds to the bony orbital contours rather titan causing bony destruction (although this can occur in very aggressive cases).
135
A 41-year-old HIV-positive man undergoes an MRI brain to investigate headaches, fever and confusion. This shows multiple foci in the basal ganglia and brainstem which are of low signal on Tlw and high signal on T2w images. There is no significant associated oedema and no enhancement is seen postcontrast. What is the most likely diagnosis? A Cryptococcosis B Cytomegalovirus infection C HIV encephalopathy D Lymphoma E Progressive multifocal leukoencephalopathy
A Cryptococcosis is the second commonest opportunistic CNS infection in AIDS. Early features include dilated perivascular spaces with the development of ciyptococcomas as the disease progresses.
136
A 36-year-old man with a history of asthma and hay fever presents with loss of smell and recurrent headaches. CT of the paranasal sinuses shows several rounded masses in the maxillary sinuses and nasal cavity with enlargement of the ostia of the maxillary antra bilaterally. The bones appear normal. What is the most likely diagnosis? A Inverting papillomas B Mucocoeles C Nasal granulomas D Nasal polyps E Squamous carcinoma
D Nasal polyposis is common in adults. The polyps may cause widening of the nasal airway and/or maxillary antra.
137
A 62-year-old man is admitted following a seizure. He gives a history of pulsatile left-sided tinnitus for 6 weeks, having undergone a left craniotomy for resection of a meningioma 4 months ago. The neurosurgeons are concerned he may have a dural arteriovenous fistula. Which one of the following investigations would be most appropriate to confirm or refute this? A Conventional MR angiography B Conventional MR venography C CT angiography D Intra-arterial catheter angiography E Skull radiograph
D Conventional MR angiography and venography may be completely normal in the setting of dural arteriovenous fistula. Dynamic MR subtraction angiography may be helpful but intra-arterial angiography remains the gold standard.
138
A 23-year-old man presents with right-sided proptosis. His visual acuity is normal. CT reveals a well-defined, hyperdense mass posterior to the globe which spares the orbital apex. The medial and lateral orbital walls are moulded around the mass but there is no evidence of bony destruction. What is the most likely diagnosis? A Capillary haemangioma B Cavernous haemangioma C Ocular metastasis D Optic nerve glioma E Orbital pseudotumour
B These are the most common intraorbital tumours in adults.
139
A 48-year-old woman presents to her GP with a midline neck mass which has been growing slowly over many months. On examination, she has a welldefined lump in the suprasternal notch and she is referred for an ultrasound. This shows a predominantly cystic lesion with some internal echoes. There is also a single echogenic focus within the lesion which has dense posterior acoustic shadowing. What is the most likely diagnosis? A Dermoid cyst B Epidermoid cyst C Haemorrhagic thyroid nodule D Ranula E Thymic cyst
A Dermoid cysts are the commonest teratoma in the head and neck. This scenario describes a cyst with cellular contents as well as an osseodental structure. CT and MRI may show globules of fat with fat and/or fluid levels.
140
A 59-year-old man presents with conductive hearing loss on the right side. CT reveals a non-enhancing mass in the middle ear which is suspicious for an acquired cholesteatoma. Which one of the following is a well recognised complication of this condition? A Ankylosis of the ossicular chain B Erosion of the lateral semicircular canal C Middle ear effusion D Opacification of the mastoid air cells E Osteomyelitis of the temporal bone
B An acquired cholesteatoma is a mass of epithelial debris within the middle ear, leading to conductive hearing loss. It can result in several complications: destruction of the ossicles, destruction of the tegmen tympani causing a cerebral abscess or meningitis, labyrinthine fistula due to erosion of the lateral semicircular canal and facial paralysis secondary to facial nerve involvement.
141
A 40-year-old woman has been admitted to the Intensive Therapy Unit (ITU) with severe pancreatitis. She is currently being ventilated but has worsening respiratory failure, refractory to oxygen therapy. In addition, she has a normal capillary wedge pressure. Which of the following is the most likely radiological feature on portable CXR? A Cardiomegaly B Mediastinal lymphadenopathy C Patchy peripheral airspace opacification D Pleural effusions E Well-defined lobar airspace opacification
C Direct and indirect insults to the lung can result in increased permeability of the pulmonary vasculature allowing protein-rich fluid to pass into the alveolar spaces at normal hydrostatic pressures. ARDS is the more severe form of this disease and the earliest radiographic findings are patchy ill-defined airspace opacities in both lungs.
142
A 25-year-old asthmatic man is referred to the chest outpatient clinic with a fever, cough and shortness of breath. A course of antibiotics has not improved his symptoms. Investigations performed in the clinic include a positive skin test for Aspergillus jumigatus and an elevated serum IgE. The patient is known to be immunocompetent with no previous history of sarcoidosis or tuberculosis. Which one of the following are the most likely high-resolution CT (HRCT) findings? A A lower lobe predominance B An air crescent sign C Central bronchiectasis D The halo sign E Wedge-shaped peripheral infarcts
C Allergic bronchopulmonary aspergillosis (ABPA) is part of a spectrum of disease caused by Aspergillus fumigatus. Hypersensitive individuals (commonly those with asthma) can present with ABPA and the key radiological features are central airway mucoid impaction leading to central bronchiectasis.
143
An 80-year-old lifelong male smoker presents with a cough and wheeze. A CXR demonstrates right middle lobe airspace opacification with bulging of the central oblique and horizontal fissures. The radiographic appearances fail to resolve 4 weeks later, after an appropriate course of antibiotics. CT evaluation demonstrates a large cavitating centrally placed mass. Which one of the following diagnoses is the most likely? A Adenocarcinoma B Large cell carcinoma C Lymphoma D Small cell carcinoma E Squamous cell carcinoma
E In lung cancer, the radiological pattern of disease varies with the cell type. Squamous cell tumours are the most common tumour to cavitate and those most frequently associated with collapse/consolidation of the lung due to their predominantly central location.
144
A 30-year-old male nonsmoker presents to his GP with a three-month history of intermittent episodes of cough and wheeze. Initially diagnosed as having asthma, the patient was found to be a 1-antitrypsin deficient after mentioning that several relatives have had similar problems in the past. As part of the subsequent investigations, an HRCT chest was performed. Which finding is most consistent with this clinical scenario? A Low attenuation regions with a lower lobe predominance B Low attenuation regions with an upper lobe predominance C Pleural effusion D Spontaneous pneumothorax E Subpleural low attenuation areas
A Panlobular emphysema is seen in al-antitrypsin deficiency. The disease tends to occur in a lower lobe distribution (unless there is a smoking history, where an upper lobe predominance can be seen).
145
A 16-year-old man has been sent for a CXR by his GP. He has had a chronic cough for 3 months and the GP is concerned that there may be an underlying pneumonia. Having reviewed the film and decided that this is not the case, you note the presence of a unilateral hypertransradiant hemithorax. Which of the following causes would not be in your differential diagnosis? A MacLeod's syndrome B Poland's syndrome C Poliomyelitis D Pulmonary agenesis and hypoplasia E Pulmonary embolus
D Pulmonary agenesis and hypoplasia is usually asymptomatic with mediastinal displacement towards a dense hemithorax. Poliomyelitis can cause atrophy of the overlying pectoral muscles. MacLeod's syndrome is a late sequel of childhood bronchiolitis with a small lung, small pulmonary arteries and expiratory air trapping on the affected side. If there is an embolus lodged in a major pulmonary artety, the vessels distal to the obstruction will be underperfused with associated loss of lung volume.
146
An 80-year-old man presents to the Emergency Department chronically short of breath. On examination, there is dullness at the left lung base and a CXR reveals loss of the left costophrenic angle. An ultrasound is performed and demonstrates a pleural effusion with no internal echoes. Diagnostic aspiration reveals that the pleural fluid has a protein concentration of 15 g/L. Which one of the following would be the most likely cause? A Cardiac failure B Collagen vascular diseases C Infection D Malignancy E Pulmonary infarction
A A transudate is defined as a fluid collection with a low protein concentration (< 30 g/L), whereas an exudate has a high protein concentration (> 30 g/L). Cardiac failure increases the capillary hydrostatic pressure which forces protein poor fluid across intact membranes. Any pathological process that leads to damage of cell membranes will allow the passage of protein macromolecules through the membrane and consequently an exudate.
147
A young woman with Turner's syndrome is found to be hypertensive. On examination, her femoral pulses are delayed, relative to the carotid pulses. In addition there is a mid to late systolic murmur. Which one of the following is the most likely radiological finding? A An ‘8’ sign due to modelling deformities of the major thoracic vessels B An enlarged external mammary artery on a lateral plain chest radiograph C Elevated left ventricular apex D Rib notching affecting all ribs E Superior rib notching
C Coarctation of the aorta is associated with inferior rib notching (which takes several years to develop) and the resultant hypertension often produces left ventricular hypertrophy. The first two ribs are generally spared as the intercostal arteries are supplied via the costocervical trunk proximal to the coarctation and therefore do not contribute to the collateral circulation.
148
A 50-year-old man undergoes a routine preoperative CXR. The reporting radiologist notes that the heart is shifted to the left, the right heart border is indistinct and there is a steep inferior slope of the anterior ribs. Which one of the following syndromes may be associated with these findings? A Churg Strauss syndrome B Eisenmenger's syndrome C MacLeod's syndrome D Marfan's syndrome E Swyer James syndrome
D Pectus excavatum is often an isolated abnormality but may be associated with Marfan's syndrome or congenital heart disease.
149
A 60-year-old man presents to his GP with a cough, fever, dyspnoea and some chest pain. He also complains of painful wrists and hands which are worse at night. A CXR demonstrated a pleural-based left chest wall mass with a welldemarcated and slightly lobulated contour. No rib destruction is evident. A subsequent CT confirmed these findings with the mass demonstrating slightly heterogeneous enhancement after contrast administration. No pleural effusion was seen. Which one of the following is the most likely diagnosis? A Empyema B Localised mesothelioma C Pleural extension of a lung tumour D Pleural metastasis E Subpleural lipoma
B Hypertrophic osteoarthropathy is a well-recognised complication seen with a localised mesothelioma.
150
A 40-year-old woman has a previous histoiy of histoplasmosis. She undergoes a chest CT which demonstrates confluent soft tissue infiltration throughout the mediastinum. Tissue biopsy determines a diagnosis of fibrosing mediastinitis subsequent to the histoplasmosis infection. Which one of the following complications would be the most common to occur? A Oesophageal obstruction B Pulmonary artery obstruction C Pulmonary venous obstruction D Superior vena cava obstruction E Tracheal obstruction
D Superior vena cava obstruction is the most common complication of fibrosing mediastinitis, but occasionally it can present with pulmonary arterial obstruction, pulmonary venous obstruction (peribronchial obstruction, septal lines etc), central airway narrowing (stridor) and oesophageal narrowing (dysphagia).
151
A 25-year-old man has a CXR (PA and lateral) performed for a chronic cough. This demonstrates a mass projected anterior to the ascending aorta and a contrast-enhanced CT chest is performed. There are no associated clinical syndromes (such as myasthenia gravis) and no CT features to suggest a thymic mass or germ cell tumour. What additional CT finding is most likely to suggest a diagnosis of lymphangioma? A Bone destruction B Multiple cystic spaces C Narrow contact with the ascending aorta D Retrosternal extension E Uniform fat attenuation
B Lymphangiomas (cystic hygromas) are congenital malformations of the lymphatic system presenting as prevascular masses and comprising complex cystic spaces with the attenuation of the contents close to water on CT.
152
A 27-year-old, previously fit and well man presents to his GP with a short history of pyrexia, cough and haemoptysis. He has never previously been admitted to hospital. Sputum culture has grown Streptococcus pneumoniae. What is the most likely chest radiograph finding? A Bronchopneumonia B Cavitation C Empyema D Large pleural effusion E Lobar consolidation
E Many community-acquired pneumonias are caused by Streptococcus pneumoniae with radiographic features of peripheral, homogeneous opacification. Air bronchograms may be present, but cavitation and empyema are uncommon.
153
A 7-year-old girl, who has recently migrated to this country from India, presents with a productive cough, fever, night sweats and weight loss. A CXR demonstrates marked consolidation in the right upper lobe. Sputum cytology reveals the presence of acid fast bacilli. What additional radiological finding is most likely to suggest a diagnosis of current primary tuberculosis as opposed to post-primary tuberculosis? A Cavitation B Mediastinal lymphadenopathy C Multifocal lesions D Ranke complex E Rasmussen aneutysm
B Primary tuberculosis causes a pneumonia that mimics Streptococcus pneumoniae in its radiographic appearance and, in children, lymphadenopathy is the most common manifestation.
154
A 30-year-old male engineer has recently returned from North America having inspected a number of construction sites. He develops flu-like symptoms and a CXR reveals the presence of a solitary well-defined nodule. What additional finding would make a diagnosis of Histoplasmosis infection more likely, rather than Cryptococcus infection? A Air bronchograms B Cavitation C Central calcification D Lymphadenopathy E Pleural effusion
C Histoplasma capsulatum is a fungus found in moist soil and bird/bat excreta and histoplasmosis occurs most commonly in areas of construction or regions near bat caves. A ‘target’ lesion describes a solitary, well-defined nodule (a histoplasmoma) with central calcification and is very specific for this condition.
155
A 30-year-old man is HIV positive with a most recent CD4 count — 100 cells/pL. He presents to the infectious diseases team with a cough, dyspnoea and general malaise. A CXR demonstrates bilateral, diffuse, medium-sized reticular opacities. An air-filled parenchymal cavity (pneumatocoele) is seen, but there is an absence of either mediastinal lymphadenopathy or a pleural effusion. What is the most likely underlying opportunistic infection? A Streptococcus pneumoniae B Ciyptococcus neoformans C Cytomegalovirus D Mycobacterium avium complex E Pneumocystis carinii
E Pneumatocoeles will generally disappear over time and the majority of radiological signs of PCP will resolve with treatment.
156
A 38-year-old man is referred to a chest physician for evaluation of a chronic productive cough. Over the past 10 years he has experienced increased expectoration of mucoid sputum that became purulent during infective exacerbations. On plain radiography the trachea had a corrugated outline. CT evaluation revealed dilatation of the trachea and mainstream bronchi. Which one of the following is the most likely diagnosis? A Amyloidosis B Mounier-Kuhn disease C Relapsing polychondritis D Tracheal leiomyoma E Wegener's granulomatosis
B Mounier-Kuhn disease (tracheobronchomegaly) describes patients with marked dilatation of the trachea and mainstream bronchi and is a radiological diagnosis.
157
A 20-year-old woman with cystic fibrosis undergoes an HRCT chest as a result of worsening respiratory function. Nontapering, thickened bronchi are easily seen within 1 cm of the costal pleura. The internal bronchia] diameter is greater than that of the adjacent pulmonary artery and the bronchial lumen has assumed a beaded configuration. Whilst there are V- and Y-shaped densities, the ‘tree in bud’ sign and mosaic perfusion are not seen. What is the most likely diagnosis? A Bronchiolitis B Cylindrical bronchiectasis C Cystic bronchiectasis D Infective consolidation E Varicose bronchiectasis
E Bronchiectasis (irreversible dilatation of the bronchi) is classified into three pathological subtypes of increasing severity: cylindrical (relatively uniform aiiway dilatation), varicose (the bronchial lumen assumes a beaded configuration) and cystic (a string or duster of cystic structures).
158
A 15-year-old girl has a follow-up CXR and ultrasound scan of her liver. She is known to have had meconium ileus at birth and has subsequently suffered with recurrent chest infections, poor weight gain, loose malodorous stools and multiple gallstones. Which of the following findings is most likely to be present on the CXR? A Bronchiectasis with a predominant lower lobe distribution B Ground glass opacity C In-dwelling venous catheter D Pleural effusion E Reduced lung volumes
C Recurrent, long courses of intravenous antibiotics often lead to medium/longterm venous access in patients with CF. Bronchiectasis is seen in a predominantly upper lobe distribution, with mucus plugging and hyperinflation.
159
A 60-year-old woman with rheumatoid arthritis presents with a flu-like illness and nonproductive cough. Her symptoms have not responded to an appropriate course of antibiotics. An HRCT is performed and demonstrates widespread mosaic perfusion. Which additional CT finding would suggest a diagnosis of obliterative bronchiolitis rather than diffuse pulmonary haemorrhage? A Increased calibre pulmonary vessels in the hyperattenuated area B Increased calibre pulmonary vessels in the hypoattenuated area C Normal calibre pulmonary vessels in the hyperattenuated area D Reduced calibre pulmonary vessels in the hypoattenuated area E Reduced calibre pulmonary vessels in the hyperattenuated area
D Obliterative bronchiolitis describes bronchiolar and peribronchiolar inflammation affecting the membranous and respiratory bronchioles. Affected areas display decreased attenuation as a result of air trapping and decreased perfusion relative to normal areas.
160
You are asked to review the CT scan of an elderly female patient who has evidence of left upper lobe collapse on a CXR. On CT, in which one of the following directions will the left upper lobe have collapsed? A Anteriorly and laterally B Anteriorly and medially C Inferiorly and medially D Posteriorly and medially E Superiorly and medially
B Left upper lobe collapse results in volume loss in an anterior and medial direction, as opposed to superior and medial collapse of the right upper lobe collapse.
161
A middle-aged man has recently had a CT abdomen performed for chronic lower left abdominal pain. Whilst mild sigmoid diverticular disease was present, it was also noted that there was significant para-aortic lymphadenopathy. Lymphoma was the suspected diagnosis and a chest CT was performed, prior to biopsy. "Which additional CT finding is most likely to suggest a diagnosis of non-Hodgkin’s lymphoma rather than Hodgkin's disease? A Isolated pulmonary consolidation B Paramediastinal interstitial fibrosis C Peripheral subpleural masses with a pleural effusion D Peripheral subpleural masses without a pleural effusion E Pulmonary consolidation with mediastinal lymphadenopathy
A At presentation, Hodgkin's disease most commonly has lung parenchymal disease accompanied by intrathoracic adenopathy, whereas isolated lung involvement is not uncommon in non-Hodgkin's lymphoma. Pleural effusions (unilateral) are common in both types of lymphoma, as are peripheral subpleural masses. Consolidation with air bronchograms can also be seen in both diseases.
162
A confused 70-year-old man with a history of cough and some shortness of breath attends your Radiology Department for a CXR. It is noted that there are multiple discrete, spherical and well-defined pulmonary nodules with a peripheral distribution. Some calcification is noted within some of these nodules but cavitation is not evident. The accompanying nurse from the care home tells you that he has a ‘growth’ somewhere but is not sure what this is. What is the most likely primaiy tumour? A Adenocarcinoma of the colon B Anaplastic thyroid carcinoma C Chondrosarcoma of the femur D Invasive ductal carcinoma of the breast E Squamous cell carcinoma of the oesophagus
C Calcified lung metastases rarely occur, except in osteosarcoma and chondrosarcoma. Even if a primary tumour displays calcification (eg breast or colonic carcinoma), the pulmonary metastases will rarely demonstrate calcification.
163
A 30-year-old man has had an HRCT reviewed at your local multidisciplinary team meeting. The HRCT demonstrates ground glass opacification throughout both lungs with a mosaic pattern on expiratory images. Which additional finding would make a diagnosis of extrinsic allergic alveolitis (EAI) more likely than respiratory bronchiolitis—interstitial lung disease (RB-ILD)? A A normal chest radiograph B A positive smoking histoiy C A stable clinical course D Exposure to paint sprays E Poorly defined centrilobular nodules
D Extrinsic allergic alveolitis is an immunologically mediated lung disease as a result of exposure to lung antigens, including those found in paint sprays.
164
A 30-year-old patient has presented with a nonproductive cough, fatigue, weight loss and a fever. As a result of an abnormality seen on a CXR, an HRCT chest is performed and demonstrates multiple thin-walled cysts within the lung parenchyma. Which additional finding is most likely to suggest a diagnosis of Langerhans cell histiocytosis (LCH) rather than lymphangioleiomyomatosis? A A smoking histoiy B Female sex C Increased lung volumes D No zonal predilection E Pleural effusion
A LCH is most commonly seen in men (4:1) and the vast majority of patients are cigarette smokers (the converse is true for lymphangioleiomyomatosis).
165
A 30-year-old woman complains of shortness of breath and a CXR is requested. The request form states ‘connective tissue disease’ but no further medical history is provided. The CXR demonstrates mild pleural thickening with small, bilateral pleural effusions. Interstitial fibrosis, with some honeycomb formation, is seen within the lower lung zones. In addition there is a single, well-circumscribed 2 cm lesion in the right upper zone, with an area of cavitation seen centrally. Which one of the following connective tissue diseases is the most likely diagnosis? A Ankylosing spondylitis B Dermatomyositis C Rheumatoid arthritis D Scleroderma E Systemic lupus erythematosus
C Rheumatoid arthritis is associated with pleural effusions/thickening, interstitial fibrosis, bronchiectasis and bronchiolitis. Necrobiotic nodules are uncommon and are usually associated with subcutaneous nodules.
166
A 30-year-old woman presents with a history of a low grade fever, malaise, anorexia and weight loss. She also reports pleuritic type chest pain. A CXR shows bilateral small pleural effusions with linear band atelectasis at both bases. No other chest abnormality is seen. Which one of the following is the most likely diagnosis? A Ankylosing spondylitis B Dermatomyositis C Rheumatoid arthritis D Scleroderma E Systemic lupus erythematosus
E Pleuro-pulmonary disease is very common in SLE, occurring in over 50% of patients at some stage during the course of their disease. This often manifests as relatively small bilateral pleural effusions, associated with pleuritic chest pain.
167
A 40-year-old man presents with rhinitis, sinusitis and otitis media. In addition he has dyspnoea and pleuritic chest pain and has had episodes of haemoptysis. His records indicate that he has had a recent renal biopsy which diagnosed the presence of focal necrotising glomerulonephritis. Which one of the following is the most likely radiological finding demonstrated on chest CT? A Lobular mass with feeding and draining vessels B Mediastinal lymphadenopathy C Multiple cavitating lung parenchymal nodules D Parenchymal mass lesion with lobar collapse E Upper lobe mycetoma
C Wegener's granulomatosis is characterised by necrotising granulomatous inflammation of the upper and lower respiratory tracts and is associated with glomerulonephritis. The most common radiological finding is cavitating lung masses measuring up to 10 cm in diameter.
168
A 63-year-old man attends the Radiology Department for a CXR. He presented to his GP with a chronic cough and breathlessness and an occupational history of having worked both in a quarry and, later on in his life, as a coal miner. Which one of the following radiographic findings is most likely to suggest a diagnosis of silicosis rather than coal worker's pneumoconiosis (CWP)? A A predominantly upper lobe distribution of disease B Eggshell calcification of mediastinal lymph nodes C Large (greater than 3 mm) lung nodules D Progressive massive fibrosis E Small (up to 3 mm) lung nodules
D Silicosis and CWP both result in small, well-defined nodules with an upper lobe distribution. As the nodules enlarge they can coalesce and form mass-like opacities with upper lobe contraction (progressive massive fibrosis). This is much more commonly seen in silicosis
169
A 40-year-old former construction worker presents with increasing dyspnoea and purulent sputum production. A CXR demonstrates lobar consolidation and he is treated for community-acquired pneumonia. However, incidental note is made of the presence of pleural plaques, which are assumed to be the result of asbestos exposure. Which one of the following radiological features would be most in keeping with asbestos related pleural plaques? A Distribution along the anterior chest wall B Involvement of the parietal pleura C Mediastinal lymphadenopathy D Sparing of the diaphragmatic dome E Sparing of the mediastinal pleura
B Asbestos-related pleural plaques involve the parietal pleura almost exclusively and the classic locations are along the posterolateral and lateral chest walls, the diaphragmatic dome and mediastinal pleura. Calcified diaphragmatic plaques are virtually pathognomonic of asbestos exposure.
170
A 25-year-old male pedestrian has been hit by a car and is currently being resuscitated in the Emergency Department. He complains of paraesthesia involving his left shoulder. Which one of the following radiological features is the most likely related cause? A Dislocated left sternoclavicular joint B Fractured left 2nd rib C Fractured left humerus D Left tension pneumothorax E Right anterior shoulder dislocation
B Fractures of the 1st to 3rd ribs imply severe trauma and can be associated with vascular, neural, spinal or tracheobronchial injuries.
171
A 40-year-old male window cleaner has fallen approximately 4 m from his ladder whilst at work. He is currently on a spinal board and being assessed in the resuscitation department of your Emergency Department. He complains of left-sided chest pain and shortness of breath. A CXR demonstrates fractures of the left 3rd, 4th and 5th lateral ribs and there is strong clinical concern of a pneumothorax. If there is a left pneumothorax, which one of the following radiographic signs is most likely to be present? A A left-sided haemothorax B An abnormally deep left costophrenic sulcus C Left upper lobe pulmonary contusion D Mediastinal shift towards the left E Visible visceral pleura with absent lung markings peripherally
B Known as the ‘deep sulcus sign’. A visible pleural line due to a pneumothorax is more commonly seen on an erect rather than supine CXR.
172
A 27-year-old man has been involved in a high-speed road traffic accident. There is significant diagonal bruising over the abdomen, due to the wearing of a seat belt. He is haemodynamically stable, but complains of severe abdominal pain and a CT of the chest and abdomen is performed. Which one of the following radiographic signs on a CXR would be most likely to suggest a right-sided diaphragmatic injury? A A nasogastric tube coiled within the left hemithorax B A right pleural effusion C Elevated left hemidiaphragm D Hollow viscera seen within the chest E Mediastinal shift towards the left
E Abdominal contents passing into the right hemithorax via a right diaphragmatic rupture will push the mediastinal structures towards the left. Hollow viscera within the chest are more commonly seen in a left diaphragmatic rupture as the liver will tend to obstruct the passage of abdominal contents into the chest (the same will apply for a nasogastric tube).
173
A 60-year-old female driver lost control of her car on black ice and hit a tree at approximately 60 miles per hour. Following resuscitation in the Emergency Department, she is found to have serious head and limb injuries and complains of upper thoracic back pain. Which one of the following radiographic signs would make you most suspicious for the presence of a traumatic aortic rupture? A A left apical pleural cap B A mediastinum more than 5 cm wide C A right apical pleural cap D Deviation of the trachea to the left E Widening of the left paratracheal stripe
A Signs of a mediastinal haematoma (and possible aortic rupture) include widening of the mediastinum (greater than 8 cm above the level of the carina or more than 25% of the width of the chest at this level), a left apical pleural cap or pleural effusion and deviation of the trachea to the right.
174
A 30-year-old man with a history of heavy alcohol abuse has been admitted to ITU with severe acute pancreatitis. He is currently ventilated and afebrile, but has worsening respiratory failure, diminished pulmonary compliance and a normal capillary wedge pressure. The clinical team suspect a diagnosis of acute respiratory distress syndrome. Which one of the following radiographic signs would you not expect to see in acute respiratory distress syndrome (ARDS)? A Complete resolution of the lung abnormality B Diffuse ground glass opacities with a bilateral distribution C Diffuse ground glass opacities with a peripheral distribution D Diffuse ground glass opacities with air bronchograms E Pleural effusions on plain supine radiography
E ARDS results from insults to the lung either from direct (eg pneumonia) or indirect (eg pancreatitis) causes and develops through three phases: exudative, proliferative and finally fibrotic.
175
An elderly alcoholic man has been admitted to hospital having been found unconscious at home by a neighbour. A CT head has found no cause for his collapse and he is admitted to a medical ward. Twelve hours later, he is found to be increasingly breathless and hypoxic and a CXR is performed. Which one of the following findings is most likely to suggest aspiration? A A 48-hour delay in the development of radiographic infiltrates following aspiration B Bilateral pleural effusions C Pulmonary infiltrates with an upper lobe predominance D Regression of radiographic pulmonary changes after 72 hours E Well-defined pulmonary infiltrates
D Aspiration is predisposed in patients with a reduced consciousness level and pulmonary changes tend to occur within hours following aspiration and regress 72 hours later.
176
A middle-aged woman is currently ventilated on ITU following a major episode of sepsis and multiorgan failure. Her respiratory function is declining and a mobile CXR shows patchy pulmonary infiltrates within both lungs. A diagnosis of pulmonary oedema is suspected. Which one of the following radiological findings is most likely to suggest a diagnosis of noncardiac rather than cardiogenic pulmonary oedema? A Cardiomegaly B Central peri hilar airspace opacification C Interstitial lines D Peribronchial cuffing E Pleural effusions
B The most common causes of pulmonary oedema in the ITU patient are cardiac failure and overhydration and may be radiologically indistinguishable. Overhydration will tend to cause a more central distribution of oedema and a wider vascular pedicle.
177
A 25-year-old man with severe cystic fibrosis has just undergone a double lung transplant. On day 9 postoperatively, he develops diffuse, bilateral airspace opacities, interstitial lines and pleural effusions. There are no other signs to suggest that this is due to cardiac failure. Which one of the following is the most likely diagnosis? A Acute rejection B Bronchial stenosis C Obliterative bronchiolitis D Post-transplantation lymphoproliferative disease (PTLD) E Reperfusion oedema
A Acute rejection has nonspecific radiological findings with persisting airspace opacities developing at day 5—10 postoperatively.
178
A 43-year-old man presents with a cough, fever and two episodes of haemoptysis. As a result of changes seen on a CXR, a CT chest has been performed. This demonstrates multiple nodules throughout both lungs with no zonal predilection. Cavitation is seen in some of the nodules and others have areas of ground glass opacification surrounding them. Which one of the following is the most likely diagnosis? A Metastatic lung disease B Multiple pulmonary infarcts C Rheumatoid lung nodules D Sarcoidosis E Wegener's granulomatosis
E Wegener's granulomatosis is a multisystem disease, commonly affecting the respiratory and renal tracts. In most patients there are multiple nodules which can cavitate and infarct, leading to surrounding areas of ground glass
179
A 70-year-old man has had a routine CXR prior to an elective total hip replacement. It is noted that the heart is enlarged (the cardiothoracic ratio exceeds 50%). What is the most likely underlying pathology? A Atrial enlargement B Cardiac myxoma C Constrictive pericarditis D Pericardial effusion E Ventricular enlargement
E Cardiomegaly is a useful sign in detecting structural heart disease. It is most commonly the result of ventricular enlargement, whereas atrial enlargement and pericardial effusion are far less common causes.
180
A 45-year-old man presents to the Emergency Department with increasing shortness of breath following a long-haul flight. A CTPA is performed and excludes the presence of a pulmonary embolus. However, cardiac abnormalities are noted and further questioning reveals that the patient had rheumatic fever as a child. Which additional radiological finding is most likely to suggest a diagnosis of rheumatic mitral regurgitation rather than rheumatic mitral stenosis? A Alveolar pulmonary oedema B Enlarged left atrial appendage C Enlarged left atrium D Left ventricular enlargement E Tricuspid regurgitation
D The cardinal feature of rheumatic mitral valve disease is left atrial enlargement (particularly affecting the left atrial appendage in rheumatic disease). Enlargement of the left ventricle is not a feature of mitral stenosis but can be seen with severe mitral regurgitation
181
A 52-year-old man presents with a long history of right iliac fossa pain, intermittent diarrhoea and weight loss. He has also experienced episodes of flushing, wheezing and dyspnoea. A CT abdomen reveals a calcified mesenteric mass with retraction of the surrounding tissues. There is evidence of multiple liver metastases and a diagnosis of metastatic carcinoid is made. On cardiac MRI, which one of the following is the most likely radiological finding? A Aortic regurgitation B Mitral regurgitation C Mitral stenosis D Tricuspid regurgitation E Tricuspid stenosis
D Carcinoid syndrome can cause subendocardial fibroelastosis with thickening and shortening of the tricuspid valve cusps. This usually leads to tricuspid regurgitation but can sometime cause tricuspid stenosis.
182
A 20-year-old professional footballer complains of dyspnoea and palpitations with occasional episodes of syncope. He has had an echocardiogram at another hospital which has raised the possibility of hypertrophic cardiomyopathy. Which one of the following is the most likely MRI finding? A Dilated left ventricle B Mitral regurgitation C Rapid early diastolic ventricular filling D Reduced cardiac contractility E Systolic anterior motion of the mitral valve
E Hypertrophic cardiomyopathy is characterised by marked hypertrophy of the left ventricular myocardium with good or even hyperdynamic contractility. Altered flow dynamics can cause the mitral valve to be compressed against the interventricular septum, partially obstructing the subaortic region—known as systolic anterior motion of the mitral valve.
183
A young man presents with generalised fatigue, dyspnoea and episodes of chest pain. A transthoracic echocardiogram has raised the possibility of a cardiac tumour. Which one of the following signs on CT is least likely to be seen with a malignant cardiac neoplasm? A Cardiac wall destruction B Involvement of more than one cardiac chamber C Narrow attachment to the cardiac wall D Pericardial invasion E Pulmonary vein extension
C Primary cardiac tumours are rare and occur less frequently than metastatic tumours to the heart. If a primary cardiac tumour has a narrow attachment to the cardiac wall, then it is more likely to be a benign lesion than a malignant primary cardiac tumour.
184
A 72-year-old woman is seen by her GP with a history of palpitations and congestive cardiac failure. A transthoracic echocardiogram is performed and raises the concern that a cardiac tumour may be present. She attends the radiology department for a cardiac MRI examination. From this list of diagnoses and MRI findings, which one is correct? A Cardiac angiosarcoma most commonly produces a left ventricular mass. B Cardiac fibroma most commonly produces right atrial wall enlargement. C Cardiac lymphoma most commonly produces a focal right atrial mass. D Cardiac myxoma most commonly produces a left atrial mass. E Cardiac rhabdomyoma most commonly produces interatrial septal
D Cardiac myxoma is the most common cardiac tumour with 75% occurring within the left atrium.
185
A 70-year-old woman complains of progressive dyspnoea. She undergoes an HRCT of the chest and this demonstrates interstitial thickening at the lung bases. Which additional radiological finding would suggest a diagnosis of pulmonary fibrosis rather than congestive heart failure? A Honeycomb destruction B Peribronchial cuffing C Pleural effusion D Rapid resolution on subsequent chest radiographs E Upper lobe blood diversion
A Idiopathic pulmonary fibrosis presents with peripheral reticular opacities predominantly at the lung bases within which there are areas of honeycomb destruction.
186
A 59-year-old woman complains of shortness of breath, palpitations and dizziness. An ECG demonstrates changes consistent with coronary artery disease and a CT coronary angiogram is performed. Which one of the following is the most likely associated radiological finding? A A haemodynamically significant stenosis defined as a reduction of at least 30% of luminal diameter B Absence of flow beyond a stenosis which is pathognomonic for acute arterial occlusion C An initial increase in the outer diameter of the coronary artery wall D Calcification of the main pulmonary arteries E Significant impairment of left ventricular function with an ejection fraction <70%
C The normal coronary arterial lumen is initially preserved with atherosclerosis with increasing wall thickness accompanied by an increasing outer vessel wall diameter. Calcification of the coronary arteries is proportional to the degree of coronary arterial stenosis present.
187
A 41-year-old man has previously had a large anterior myocardial infarction. He now presents with increasing shortness of breath on exertion and it is suspected that he has a degree of pulmonary venous hypertension (PVH) due to left ventricular failure. Which one of the following is the most likely radiological finding? A A fine nodular parenchymal lung pattern if chronic PVH develops B Kerley A septal lines radiating from the hilum to the pleural surface C Kerley C septal lines seen at right angles to the pleural surface within the peripheral lower zones D Lower lobe pulmonary venous blood diversion E Relative thinning of bronchial wall thickness compared with normal
A PVH occurs as a result of increased resistance in the pulmonary veins most commonly as a result of left-sided heart disease.
188
A 67-year-old woman presents with a sudden onset of dyspnoea and orthopnoea, following a recent myocardial infarction. Clinical examination reveals a pansystolic murmur with a thrusting apex beat. CXR demonstrates pulmonary oedema with a normal heart size. The suspected diagnosis is severe mitral regurgitation secondary to a ruptured chorda tendinea. Which one of the following signs is associated with this diagnosis? A Airspace opacification predominantly in a central distribution B Airspace opacification predominantly in a peripheral distribution C Airspace opacification predominantly in the left lower zone D Airspace opacification predominantly in the left upper zone E Airspace opacification predominantly in the right upper zone
E Certain patterns of opacification may suggest a certain diagnosis. Severe mitral regurgitation is associated with opacification within the right upper zone resulting from regurgitant blood flow in the right upper lobe pulmonary artery from the superoposteriorly positioned mitral valve.
189
A 33-year-old woman presents to the Emergency Department with gradually increasing shortness of breath, such that now it is interfering with her daily activities. She has previously been diagnosed with a pulmonary embolus and it is thought that she may have pulmonary arterial hypertension as a result of chronic pulmonary thromboembolic disease. Which one of the following radiological signs is the most likely to be demonstrated? A A right descending pulmonary artery measuring 10 mm in diameter B Calcification of the pulmonary arteries C Lobulated hilar masses D Peripheral pulmonary arterial branches seen beyond a segmental level E The diameter of the main pulmonary artery is less than the ascending
B Chronic pulmonary arterial hypertension is characterised by enlargement of the central pulmonary arteries (with a diameter greater than that of the adjacent ascending aorta) with associated tapering of the peripheral arterial branches at the segmental level (peripheral pruning).
190
Whilst reporting a CXR requested by a GP, you note that the left lung demonstrates reduced pulmonary vascularity relative to the right lung and are undecided as to whether this is an ‘apparent’ or ‘real’ finding. Which one of the following conditions is a ‘real’ cause of uneven pulmonary vascularity? A MacLeod's syndrome B Mastectomy C Patient rotation D Patient scoliosis E Poland's syndrome
A MacLeod's syndrome (Swyer James syndrome) is a manifestation of childhood postinfectious obliterative bronchiolitis, resulting in diminished vascularity and reduced growth of the affected lung.
191
A 37-year-old woman presents with episodes of dyspnoea, hypoxia and cyanosis. Initial laboratory investigations are unremarkable and a tuberculin test is negative. On her CXR, there is a lobulated opacity in the left lower lobe with an associated prominent vascular shadow. Inspiratory and expiratory films demonstrate a change in the size of the opacity. What is the most likely A Pulmonary arteriovenous malformation B Pulmonary artery pseudoaneurysm C Pulmonary embolus D Pulmonary plethora E Rasmussen aneurysm
A
192
A 71-year-old man has collapsed at home, having complained of pleuritic chest pain and shortness of breath. He has had similar episodes in the past caused by pulmonary emboli. What finding on review of his CTPA would be consistent with a diagnosis of acute rather than chronic pulmonary embolic disease? A Calcified pulmonary arterial thrombus B Crescentic thrombus adherent to the arterial wall C Enlarged pulmonary arteries D Mosaic attenuation of the lung parenchyma E ‘Tram track’ appearance of a pulmonary artery
E On CTPA, an acute PE is seen as an intravascular filling defect. If contrast medium flows around the thrombus, then a ‘tram track’ appearance can be seen.
193
A 35-year-old man presents with increasing shortness of breath, fever and a non productive cough. He is known to be HIV-positive and a recent CD4 count was 110 cells per cubic millimetre. What additional finding is likely to suggest a diagnosis of Pneumocystis jiroveci (carinii) rather than tuberculosis? A A lower lobe predominance B Bilateral, diffuse, coarse reticulonodular opacities C Bilateral, diffuse, fine reticular opacities D Bilateral hilar lymphadenopathy E Pleural effusions
C Pneumocystis pneumonia typically demonstrates bilateral, symmetrical, fine to medium reticular opacities as opposed to the coarse, reticulonodular pattern seen in patients with advanced HIV and tuberculosis.
194
A 56-year-old female smoker presents with increasing shortness of breath, fever and a productive cough. Her CXR demonstrates diffuse opacification the right lung base and treatment is commenced for community-acquired pneumonia. Which additional radiological finding is most likely to suggest diagnosis of Streptococcus pneumoniae rather than Staphylococcus aureus? A Air bronchograms B Cavitating nodules C Empyema D Pleural effusion E Scattered multifocal opacities
A Pleural effusions, cavitation and empyema formation are common with staphylococcal infection, whereas air bronchograms are unusual.
195
A 28-year-old woman has had a CXR following the development of a persistent cough. The PA and lateral views demonstrate a significant mediastinal mass. Which one of the following is the correct radiological consideration as you review these films? A Bronchogenic cysts are usually symptomatic. B Bronchogenic cysts commonly display calcification of the cyst wall. C Bronchogenic cysts commonly have a peripheral distribution. D Neurenteric cysts are usually found in the middle mediastinum. E Neurenteric cysts are frequently symptomatic.
E ‘Foregut duplication cysts’ describe those congenital cysts derived from the embryological foregut and includes bronchogenic, enteric and neurenteric cysts. Neurenteric cysts are found in the posterior mediastinum (with associated vertebral anomalies) and frequently produce pain.
196
A 21-year-old woman presents with fever, arthralgia and weight loss. A clavicular bruit is detected clinically as well as by diminished upper extremity pulses. Catheter pulmonary and aortic arch angiography is performed and the findings suggest a diagnosis of Takayasu's disease. Which one of the following angiographic findings is most likely to have been present? A Aortic dissection B Intercostal collateral development C Pseudoaneurysm development D Pulmonary arterial involvement E Sparing of brachiocephalic artery
D Granulomatous vasculitis (Takayasu's disease) is a chronic inflammatory disease involving the aorta (and its main branches) and pulmonary arteries leading to stenosis, occlusion or dilatation.
197
Whilst reporting plain radiographs from a respiratory outpatient clinic, you view a CXR that demonstrates bilateral hypertransradiant hemithoraces. The lung volumes are normal and, unfortunately, there is no clinical history accompanying the request card. Which diagnosis would best explain these findings? A Acute bronchiolitis B Asthma C COPD D Multiple pulmonary emboli E Tracheal stenosis
D Multiple pulmonary emboli can lead to pulmonary arterial pruning and increased transradiancy in normal volume lungs. The overinflation seen in asthma is secondary to bronchial constriction and mucus plugging, whereas bronchial inflammation can lead to overinflation seen in bronchiolitis. Tracheal stenosis will also impair the expiratory phase of ventilation, also potentially leading to overinflation.
198
A CXR is performed on a 62-year-old man with a chronic cough. This demonstrates multiple tiny nodules throughout both lungs, measuring up to 2 mm in size. These micronodules appear to be of greater density than soft tissue. Which one of the following is the most likely diagnosis? A Coal worker's pneumoconiosis B Miliary histoplasmosis C Miliary tuberculosis D Sarcoidosis E Silicosis
E Silicosis often demonstrates multiple well-defined nodules, which can appear veiy dense due to the pure silica deposits. The other conditions listed here can also demonstrate multiple opacities of varying morphology, but these are typically of soft tissue density.
199
A 51-year-old man presents to his GP with hypertension and intermittent headaches. A CXR is performed and demonstrates inferior rib notching. The initial working diagnosis is coarctation of the aorta, but which additional diagnosis should also be considered? A Marfan's syndrome B Neurofibromatosis Type I C Rheumatoid arthritis D Scleroderma E Systemic lupus erythematosus
B Neurofibromatosis type 1 is a well-recognised cause of inferior rib notching.
200
A 58-year-old woman presents to her GP with increasing shortness of breath and a dry cough. A CXR demonstrates a reticular interstitial pattern with a ‘honeycomb’ appearance of multiple cystic spaces in the lower zones. Which one of the following is the most likely diagnosis? A Extrinsic allergic alveolitis B Langerhans cell histiocytosis C Previous radiation therapy D Rheumatoid lung E Sarcoidosis
D Rheumatoid fibrosis demonstrates predominantly basal changes, whilst the other conditions listed here typically produce interstitial changes in different distributions.
201
A 50-year-old man has entered a drug trial which requires a CXR as part of the protocol. The reporting radiologist has noticed a solitary pulmonary nodule measuring 2 cm in diameter, within the right mid zone. There is no associated pulmonary, pleural or mediastinal lymphadenopathy. Which additional finding is likely to suggest that this is a malignant mass? A A doubling time of less than 2 weeks B Enhancement <15 HU post contrast medium administration C Laminated calcification D Lobulated margin E Well-defined margins
D The most important criteria distinguishing benign from malignant solitary pulmonary nodules are the nodule attenuation and growth over time. Lobulation of a mass suggests uneven growth rates in a tumour mass.
202
A 75-year-old man presents to his GP with a persistent cough, green sputum and fevers. A CXR demonstrates basal consolidation with an upwardly concave meniscus, travelling up the lateral chest wall. An ultrasound of the chest demonstrates a hypoechoic pleural collection with internal septations and debris. Pleural aspiration is remarkable for normal pH and protein concentration of 44g/L. What is the most likely cause for this effusion? A Cardiac failure B Empyema C Pancreatitis D Parapneumonic effusion E Pulmonary infarction
D The ultrasound and pleural fluid features are those of an exudative pleural effusion. The normal pH makes an empyema unlikely and a parapneumonic effusion is the most likely diagnosis.
203
A 45-year-old woman presents with pyrexia, cough, weight loss and night sweats of three months' duration. She is found to have a peripheral eosinophilia and a CXR reveals patchy, nonsegmental consolidation in the periphery of the mid and upper zones. The radiographic changes rapidly resolve with corticosteroid therapy. Which one of the following was the most likely diagnosis? A Chronic eosinophilic pneumonia B Churg Strauss syndrome C Invasive aspergillosis D Simple pulmonary eosinophilia (Loffler's syndrome) E Tuberculosis
A Patients with simple pulmonary eosinophilia typically have a self-limiting mild respiratory illness with a peripheral eosinophilia. Chronic eosinophilic pneumonia is associated with a more severe and prolonged illness with characteristic mid and upper zone peripheral infiltrates (‘reverse bats wing’ appearance) that resolve rapidly with corticosteroids.
204
A 27-year-old woman has severe asthma. She is admitted to ITU with a severe, life-threatening exacerbation requiring mechanical ventilation. Two days later, a supine CXR is performed. This demonstrates a lucent line around the left heart border and aortic arch with surgical emphysema at the root of the neck. The lungs are hyperinflated but appear clear. Which complication is likely to have occurred? A Alveolar rupture B Diaphragmatic rupture C Oesophageal perforation D Pneumothorax E Tracheobronchial rupture
A The clinical and radiographic appearances are those of a pneumomediastinum. Asthma and mechanical ventilation are risk factors for alveolar rupture, with gas tracking back through the peribronchovascular sheath to the mediastinum.
205
The radiograph of an 8 year old boy with dietary vitamin D deficiency reveals cupping and fraying of the distal tibial metaphysis. Which radiological finding is a recognised feature of this condition? A Cortical sclerosis involving the margin of the epiphysis B Expansion of the costochondral junctions C Exuberant periosteal reaction D Increased density of the end of the metaphases E Metaphyseal spurs
B Sclerosis of the margins of the epiphysis (Wimberger sign), metaphyseal spurs (Pelcan), dense metaphyseal lines and exuberant periosteal reactions (Secondary to recurrent subperiosteal bleeding) are features of scurvy. Expansion of the costochondral junctions results in characteristic rachitic rosary
206
A 60-year-old woman is assessed by the Emergency Department following a fall onto her right wrist. The initial radiograph shows an extra-articular fracture of the right distal radius, with volar subluxation of the distal fragment. Which eponymous fracture type best matches this description? A Barton's fracture B Colles’ fracture C Hutchinson's fracture D Reverse Barton's fracture E Smith's fracture
E Although eponymous classification of injuries is often criticised, many eponyms endure as succinct descriptions of fracture patterns, forcing the radiologist to remain aware of the more common eponymous injuries.
207
A 32-year-old builder is brought to the Emergency Department following a fall from scaffolding. He is believed to have fallen a considerable height I and witnesses report that he landed on his feet. On primary survey, he is tachycardic, hypotensive and extremely tender on palpation of the pelvis and left hip. During resuscitation, a radiographic trauma series is obtained. What is the most likely pattern of pelvic injury? A Bilateral fractures of the superior and inferior pubic rami B Bilateral fractures of the superior and inferior pubic rami with a fracture through the left sacral ala C Disruption of the sacroiliac joints and pubic symphysis D Localised fracture through the left iliac wing E Vertical fracture through the left ilium with fractures through the left superior and inferior pubic rami
E
208
A 43-year-old man is investigated for pain related to his left arm. Plain radiography demonstrates a well-defined, lytic lesion in the proximal humerus, with chondroid matrix mineralisation and a narrow zone of transition. There is deep endosteal cortical scalloping and the suggestion of bone expansion. What is the most likely diagnosis? A Chondroblastoma B Chondroma C Chondromyxofibroma D Chondrosarcoma E Osteochondroma
D
209
A 19-year-old man, the unrestrained driver in a high-energy road traffic accident, has been brought by ambulance to the Emergency Department. A lateral cervical spine radiograph shows an anterior wedge fracture of C5 with a retropulsed bony fragment. What was the likely predominant force acting on the cervical spine at the time of injury? A Compression B Distraction C Extension D Flexion E Shearing
A
210
A 14-year-old boy attends the Emergency Department following an injury to his right ankle in a rugby match. The radiograph shows a triplane fracture. Which fracture is likely to form part of this complex injury? A Coronal fracture through the physis B Coronal fracture through the epiphysis C Horizontal fracture through the metaphysis. D Sagittal fracture through the epiphysis E Sagittal fracture through the metaphysis
D Triplane fractures are injuries of adolescence, occurring around the time of epiphyseal fusion. Partial fusion of the growth plate results in complex fracture geometry following injury; typically, this includes a coronal metaphyseal fracture, a horizontal fracture through the physis and a sagittal epiphyseal fracture (eg Salter-Harris IV injury). Tillaux fractures occur at a similar age, and are characterised by a horizontal fracture through the growth plate and a vertical epiphyseal fracture (Salter-Harris III injury).
211
A solitary, lytic lesion with aggressive features is an unexpected incidental finding on radiography of the left knee. Which radiological feature would favour a diagnosis of metastasis rather than primary bone tumour? A Bone expansion B Diaphyseal location C Florid periosteal reaction D Tumour bone formation E Soft tissue mass
B
212
A pelvic radiograph reveals a symmetrical abnormality of the proximal femora characterised by thin lucent lines perpendicular to the medial femoral cortex, with a faint sclerotic margin. These linear lucent areas do not extend across the full width of the femur, and the visualised bones are otherwise of normal appearance. Which is the most likely diagnosis? A Hyperparathyroidism B Multiple myeloma C Osteomalacia D Osteoporosis E Paget's disease
C Looser's zones represent areas of unmineralised osteoid and are pathognomonic of osteomalacia. The medial aspects of the proximal femora are typical sites for Looser's zones; other common sites include the pubic rami, ribs and the lateral border of the scapula.
213
A 32-year-old man attends hospital following a fall onto his flexed left arm. He is referred to the duty orthopaedic team with a ‘Monteggia injury’. What are the most likely radiological findings? A A fracture of the distal radius with an associated dislocation of the radial B A fracture of the distal radius with an associated disruption of the distal radioulnar joint C A fracture of the distal ulna with an associated dislocation of the radial D A fracture of the proximal ulna with an associated dislocation of the radial head E A fracture of the proximal radius with an associated disruption of the distal radioulnar joint
D
214
On an otherwise normal lateral radiograph of the knee, the patella is noted to be inferiorly situated, in keeping with patella baja. What is a possible association of this condition? A Cerebral palsy B Chondromalacia patella C Juvenile idiopathic arthritis D Quadriceps atrophy E Recurrent patellar subluxation
C On a lateral radiograph, the length of the patellar tendon should equal the height of the patella, plus or minus 20%. Patella baja is associated with polio, juvenile chronic arthritis and achondroplasia.
215
A 27-year-old man is referred by his GP with progressively painful swelling of his left knee following a minor football injury some weeks ago. The radiograph shows a 5-cm ill-defined lytic lesion within the left distal femoral metaphysis, with a permeative pattern of bone loss and areas of cloud-like ossification. There is an extensive periosteal reaction, predominantly orientated perpendicular to the cortex. What is the most likely diagnosis? A Aneurysmal bone cyst B Chondrosarcoma C Ewing's sarcoma D Metastasis E Osteosarcoma
E The appearances are highly aggressive, and characteristic of osteosarcoma. The tumour matrix indicates a tumour of osseous rather than cartilaginous origin, malting chondrosarcoma highly unlikely.
216
A 25-year-old doctor injures her left wrist whilst snowboarding. Initial radiographs are reported as showing no fracture, but there is clinical suspicion of a scapholunate ligament disruption. Further views are obtained. Which radiological feature would support the diagnosis? A Scapholunate angle less than 30° B Scapholunate distance of 2 mm C ‘Signet ring’ appearance of the scaphoid D Rotatory subluxation of the lunate E Wedge-shaped appearance of the lunate
C The ‘signet ring’ appearance is clue to rotatory subluxation of the scaphoid as a result of disruption of the scapholunate ligament.
217
Regarding MRI examination of the shoulder, what are the signal characteristics of the normal supraspinatus tendon? A High signal intensity on all sequences B High signal on Tlw, low signal on T2w C Intermediate signal on all sequences D Low signal on all sequences E Low signal on Tlw, high signal on T2w
D
218
In a 40-year-old woman complaining of wrist pain, radiographs reveal sclerosis and collapse of the lunate, with rotatory subluxation of the scaphoid. What is the most likely diagnosis? A Freiberg's disease B Kienboeck's disease C Kohler's disease D Sever's disease E Sinding-Larsen disease
B Kienboeck's disease describes lunate collapse as a result of vascular insufficiency and avascular necrosis. Avascular necrosis of the navicular is Kohler's disease whilst Freiberg's disease describes avascular necrosis of the second metatarsal head. Sever's disease relates to calcaneal apophysitis whilst Sinding-Larsen's disease is a cause of anterior knee pain in adolescents.
219
A 30-year-old women experiences, amongst other symptoms, recurrent episodes of painful swelling and stiffness of both hands. Antibodies against antinuclear antigens and double-stranded DNA are detected in peripheral blood samples. Radiographs of both hands are obtained during her assessment. Which radiographic findings are most likely? A Atrophic soft tissues, resorption of the terminal phalanges and soft tissue calcinosis B Punched-out marginal erosions in an asymmetric distribution, with preservation of bone density C Symmetrical abnormality of the MCPs of the index and middle finger, characterised by joint space narrowing, subarticular cysts and hook-like osteophytes D Symmetrical soft tissue swelling, marginal erosions and juxta-articular osteopaenia E Ulnar deviation at the MCPJs, without evidence of erosions
E
220
A 15-year-old male haemophiliac patient presents to the Emergency Department with painful swelling of the right knee. There have been similar presentations in the past. Following radiographs, he is referred for an MRI of the right knee. Which of the following imaging features would be regarded as unusual or atypical given the history? A Accelerated maturation of the epiphysis B Epiphyseal enlargement C Irregular synovial thickening D Juxta-articular sclerosis E Blooming of the synovium on gradient echo sequences
D Juxta-articular osteoporosis is a typical feature of haemophilic arthropathy, as a result of the periarticular hyperaemia associated with recurrent haemorrhage.
221
An asymptomatic 65-year-old woman on long-term steroids for rheumatoid disease undergoes dual energy X-ray absorptiometry (DXA). Her Z score is —2 and her T score is —2.7. What is the WHO definition of osteoporosis? A T score less than —1 B T score less than —2.5 C Z score less than —1 D Z score less than —2.5 E Mean of T and Z score less than —2
B Bone density can be measured in relation to an age and sex-matched population (Z score) or in relation to a population of young adults of the same sex (T score). The WHO defines osteoporosis as a T score less than —2.5, therefore relating bone mineral density to sex-matched peak bone mass.
222
A 13-year-old boy is referred to the orthopaedic surgeons with a short history of pain and swelling around his left elbow. The radiograph reveals a 4-cm area of permeative bone destruction within the distal diaphysis of the left humerus, with a wide zone of transition. There is an extensive associated soft tissue component and evidence of a ‘hair-on-end’ pattern of periosteal reaction. What is most likely diagnosis? A Askin tumour B Chondroblastoma C Chondromyxoid fibroma D Ewing's sarcoma E Malignant fibrous histiocytoma
D Infection and Langerhans cell histiocytosis (LCH) should be considered in the differential diagnosis of a permeative bone lesion in a child. Askin tumour is a rare primitive neuroectodermal tumour of the chest wall in children.
223
A 2-cm, well-defined lytic bone lesion in the proximal tibial metaphysis is an incidental finding in a 25-year-old woman. The lesion has a thick sclerotic margin and there is a ground glass appearance to the matrix. There is a history of endocrine disturbance and several cafe-au-lait spots are evident on examination. Skeletal scintigraphy subsequently reveals multiple areas of increased activity within the skeleton. What is the most likely diagnosis? A Gardner's syndrome B Mazabraud's syndrome C Maffucci's syndrome D McCune-Albright E Ollier's syndrome
D McCune-Albright syndrome is characterised by polyostotic fibrous dysplasia, cafe-au-lait spots and endocrine disturbance (most commonly precociou puberty in girls). The rare association of polyostotic fibrous dysplasia and soft tissue mxyomas is Mazabraud’s syndrome.
224
A 19-year-old student returns to the UK following 4 months' travelling around the world. Radiographs reveal multiple oval areas of calcification, up to 1 cm in long axis, aligned in the direction of muscle fibres. What is the most likely diagnosis? A Cysticercosis B Dracunculus (guinea worm) infection C Hydatid disease D Loiasis E Schistosomiasis
A
225
A 20-year-old man is brought to the Emergency Department after diving into the shallow end of a swimming pool at a party. Witnesses describe a hyperflexion injury to the cervical spine. A fracture is identified on the lateral cervical radiograph. What is the most likely fracture configuration given the mechanism of injury? A Anterior wedge fracture B Burst fracture, with an anterior wedge fracture and a retropulsed fragment C Fracture dislocation, with anterior dislocation of the more cranial vertebra and an avulsion fracture of the superoanterior margin of the more inferior vertebra D Fracture of the pars interarticularis, in association with an avulsion fracture through the anteroinferior margin of the vertebra above E Posterior elements fracture with anterior vertebral wedging
A
226
A young patient is newly diagnosed with diaphyseal aclasis. What would be the expected imaging findings? A Multiple enchondromas B Multiple enostoses C Multiple osteochondromas D Multiple osteomas E Multiple osteoid osteomas
C
227
The unrestrained passenger of a vehicle involved in a high-energy road traffic accident is admitted with a ‘hangman's fracture’. What is the most likely appearance on plain film? A Fractures through the neural arch of Cl B Fractures through the neural arch of C2 C Fracture of the spinous process of C7 D Transverse fracture through the base of the dens E Wedge compression fracture of an upper cervical vertebra
B
228
A 6-year-old boy is referred to the orthopaedic team with a limp. A pelvic radiograph reveals loss of height of the right femoral head, with fragmentation and sclerosis of the epiphysis. What is the most likely diagnosis? A Developmental dysplasia of the hip B Perthes' disease C Septic arthritis D Slipped upper femoral epiphysis E Transient synovitis of the hip
B
229
A 65-year-old woman is referred for a pelvic radiograph to investigate intermittent right hip pain. The radiograph shows thin lucent lines within both inferior pubic rami. Which radiographic feature would support a diagnosis of osteoporotic fracture rather than osteomalacia? A Callus formation B Failure to extend across the entire width of the bone C Sclerotic margin to lucencies D Similar appearances within the proximal femora E Symmetrical appearance
A
230
A young footballer has an MRI of the left knee following a recent injury. There is amorphous intermediate signal in the region of the anterior cruciate ligament and a bone contusion involving the articular surface of the lateral femoral condyle. In which other location is a bone contusion most likely? A The anterolateral aspect of the tibia B The anteromedial aspect of the tibia C The central articular surface of the tibia D The posterolateral aspect of the tibia E The posteromedial aspect of the tibia
D
231
A 50-year-old man has an MRI examination of his right shoulder. Which pattern of imaging features is compatible with a partial thickness supraspinatus A A gap between the distal and proximal portions of the tendon, with retraction of the proximal tendon B Areas of increased signal on T1 and T2 images C Areas of increased signal on T1 and T2 images, extending across the full thickness of the tendon D Areas of intermediate signal on Tl- and PD-weighted images, with low signal on T2w images E Low signal on all sequences
B
232
A 30-year-old man complains of intermittent painful swelling of his left knee over the past year. Radiographs show several small articular erosions, whilst subsequent MRI reveals foci of low T2/T2* signal intensity within the synovium. Which is the most likely diagnosis? A Alkaptonuria B Calcium pyrophosphate arthropathy C Pigmented villonodular synovitis D Psoriatic arthropathy E Synovial chondromatosis
C
233
A 45-year-old man with hyperuricaemia is referred by the rheumatologists for an image-guided aspiration of a right ankle effusion. There is no previous history of note. A sample of the aspirate is sent for polarising light microscopy. What findings would confirm the clinical suspicion of gout? A Rhomboid crystals B Negatively birefringent needle shaped crystals C Negatively birefringent rhomboid crystals D Positively birefringent needle shaped crystals E Positively birefringent rhomboid crystals
B Monosodium urate crystals are needle-shaped and strongly negatively birefringent under polarising light. Weakly positively birefringent rhomboid crystals are characteristic of calcium pyrophosphate dihydate deposition.
234
Which one of the following conditions is NOT a recognised component of the SAPHO spectrum? A Arthritis B Hyperostosis C Osteomyelitis D Pustulosis E Synovitis
A
235
A 75-year-old diabetic man underwent a left below knee amputation 3 months ago for osteomyelitis of the distal tibia. Since then, he has experienced recurrent episodes of fever and malaise. MRI is contraindicated due to a metallic aortic valve. Which is the best investigation to exclude an occult focus of osteomyelitis? A CT B US C Scintigraphy using gallium D Scintigraphy using indium-labelled white cells E Scintigraphy using technetium (Tc-99m) monodiphosphonate
E Although an indium-labelled white cell study is more specific, a bone scintigram using Tc-99m monodiphosphonate is a more sensitive test to exclude osteomyelitis.
236
During an MRI examination of the shoulder, a 4-cm, well-defined structure is noted within the spinoglenoid notch, exhibiting high signal on T2w and low signal on Tlw images. Which muscles should be carefully scrutinised for evidence of swelling or atrophy? A Infraspinatus and supraspinatus B Subscapularis and trapezius C Supraspinatus and subscapularis D Teres minor and infraspinatus E Trapezius and teres minor
A
237
In relation to bone formation and turnover, defective osteoclastic function is a predominant feature of which disease? A Osteomalacia B Osteopetrosis C Osteoporosis D Paget's disease E Rickets
B
238
A 40-year-old builder is admitted unconscious to the Emergency Department following an accident at work. Details of the accident are unclear, but one witness describes scaffolding collapsing. He undergoes an emergency CT head and cervical spine, which reveals lateral displacement of both the lateral masses of Cl. How may such an injury be described? A Atlanto-axial subluxation B Clay shoveller's fracture C Dens fracture D Hangman's fracture E Jefferson's fracture
E
239
A 20-year-old man complains of a 3-month history of pain from his left femur. The pain is of insidious onset and is worse at night. As part of his assessment, a bone scintigram is performed, which shows a corresponding area of abnormality in the left femoral shaft, characterised by a focus of very high activity surrounded by a diffuse area of modestly increased activity. What is the most likely diagnosis? A Aneurysmal bone cyst B Enostosis C Enchondroma D Osteoid osteoma E Osteoma
D
240
An 18-year-old man experiences persistent symptoms following a fracture through the waist of the right scaphoid. Radiographs of the right scaphoid indicate non-union. An MRI is performed to assess the vascularity of the proximal pole. Which imaging features are consistent with a diagnosis of avascular necrosis? A Bone marrow enhancement following administration of gadolinium B High signal surrounding the fracture on T2w images C High signal within the proximal pole on Tlw images D High signal within the proximal pole on STIR images E Low signal within the proximal pole on Tlw images
E Low signal on T1 reflects death of the adipocytes. The combination of low signal on Tlw images and low or intermediate signal on T2w images accurately predicts avascular necrosis.
241
A 35-year-old man is brought to the Emergency Department following a fall from a motorcycle. The lateral cervical radiograph shows a well-marginated triangular area of bone at the anterosuperior margin of C5. The cortical margin of the adjacent vertebral body is smooth. The rest of the spine is normal. What is the most likely diagnosis? A Avulsion of the anterior longitudinal ligament B Limbus vertebra C Schmorl's node D Teardrop fracture of C5 E Unfused ring epiphysis
B
242
A 55-year-old lady, complaining of recent flattening of the longitudinal arch of the foot, is referred for an ultrasound examination of the left ankle. Which tendon should be the subject of particular scrutiny? A Achilles tendon B Flexor hallucis longus C Peroneus longus D Tibialis anterior E Tibialis posterior
E
243
A 13-year-old boy is referred for radiographs of his hips following three weeks of right hip pain. What imaging features would support the diagnosis of slipped upper femoral epiphysis? A Disruption of Klein's line B Fragmentation of the femoral epiphysis C Increased epiphyseal height D Radiolucent subchondral fissure E Sclerosis of the femoral head
A Klein's line is drawn along the lateral border of the femoral neck and normally intersects roughly a sixth of the femoral epiphysis. Subchondral Assuring (crescent sign), epiphyseal collapse, fragmentation and sclerosis are all features of Perthes' disease.
244
A 55-year-old woman undergoes arthrography to investigate a 3-month history of pain and stiffness in her left shoulder, with particular limitation of external rotation. Only a few millilitres of contrast medium could be injected into the joint, before provoking discomfort. What other finding would be supportive of the diagnosis of adhesive capsulitis? A Contrast medium opacifies the subacromial/subdeltoid bursa B Decreased resistance to contrast injection C Distended axillary recess D Lymphatic filling E Venous filling
D
245
The radiograph of a 40-year-old man with a painful knee shows multiple calcified loose bodies, each of similar size, within the joint. The joint space is preserved. What diagnosis is most likely? A Calcium pyrophosphate arthropathy B Gout C Pigmented villonodular synovitis D Rheumatoid arthritis E Synovial osteochondromatosis
E
246
A 50-year-old man presents to the Emergency Department following an injury to his right hand. No fracture is detected on plain radiographs, out the second and third MCPJs appear abnormal, with joint space narrowing, iubarticular cysts, hook-like osteophytes and flattening of the metacarpal reads. What is the most likely diagnosis? A Alkaptonuria B Gout C Haemochromatosis D Osteoarthrids E Psoriatic arthritis
C
247
A 34-year-old man with chronic back pain is referred by his GP for thoracic and lumbar spine radiographs. The GP is concerned about the possibility of ankylosing spondylitis. Which radiological feature is atypical for ankylosing spondylitis, and might suggest an alternative diagnosis? A Ankylosis of the apophyseal joints B Anterior longitudinal ligament calcification C Osteophyte formation D Sclerosis of the anterior corners of the vertebrae E Vertebral body squaring
C Syndesmophytes, rather than osteophytes, are characteristic features of ankylosing spondylitis. They are differentiated from osteophytes by their vertical orientation, as they represent ossification of the outer border of the annulus fibrosus. Progression and maturation of the syndesmophytes result in a ‘bamboo spine’.
248
A 70-year-old patient undergoes a staging CT for renal cell carcinoma, which shows ligamentous ossification extending from the fifth to ninth thoracic vertebrae. There is relative sparing of the left side of the vertebrae and disc space height is preserved. The apophyseal and sacroiliac joints appear normal. What is the most likely diagnosis? A Ankylosing spondylitis B Degenerative disc disease C Diffuse idiopathic skeletal hyperostosis D Metastatic disease E Ossification of the posterior longitudinal ligament
C
249
A 40-year-old tennis player undergoes an MRI following a 3-month history of left ankle pain. The Achilles tendon has a convex anterior margin and exhibits a small linear area of increased signal within the tendon on T2- and T2*-weighted images. What is the most likely diagnosis? A Achilles paratendonitis B Achilles tendinosis C Achilles tendinosis with complete tear D Achilles tendinosis with cystic degeneration E Achilles tendinosis with partial tear
E
250
A 30-year-old man undergoes an MRI examination of his left ankle, which shows a rounded mass within the pre-Achilles fat pad, with signal characteristics identical to adjacent muscle. Which anatomical variant could best account for these appearances? A Accessory popliteus muscle B Accessory soleus muscle C Anomalous insertion of the gastrocnemius tendon D Anomalous insertion of plantaris tendon E Presence of peroneus quartus
E
251
A 9-year-oid boy injures his right wrist playing football. The radiograph reveals a fracture extending through the epiphysis and into the metaphysis. How would this injury be classified in the Salter-Harris classification? A Type I B Type II C Type III D Type IV E Type V
D
252
An 18-year-old man attends his general practitioner with a painful right knee. His radiograph shows a well-defined, lobular, lytic lesion within the proximal tibia! epiphysis, extending into the metaphysis. There is a faintly sclerotic margin and no matrix calcification. What is the most likely diagnosis? A Chondroblastoma B Chondromyxoid fibroma C Enchondroma D Giant cell tumour E Osteoid osteoma
A
253
A 50-year-old man has an MRI of his right shoulder for chronic shoulder pain. The distal supraspinatus tendon displays intermediate signal intensity on Tlw images and low signal intensity on T2w images. What is a possible explanation for these appearances? A Chemical shift artefact B Magic angle phenomenon C Movement artefact in the frequency-encoding direction D Movement artefact in the phase-encoding direction E Susceptibility artefact from calcification
B The magic angle phenomenon refers to the increased signal observed on sequences with a short echo time (eg Tlw, proton density (PD)) within tissues containing parallel unidirectional collagen fibres, when such fibres are at an angle of 55° to the main magnetic field. It is of particular relevance in shoulder MR, where it may mimic supraspinatus tendinosis.
254
A series of neonatal radiographs reveal a narrow thorax with short ribs, square iliac wings with horizontal acetabular roofs, short sacrosciatic notches, progressive narrowing of the interpedicular distance and posterior scalloping of the vertebral bodies. What is the most likely diagnosis? A Achondroplasia B Campomelic dysplasia C Cleidocranial dysplasia D Ellis-van Creveld syndrome E Morquio's syndrome
A The iliac wings in Morquio's syndrome are characteristically flaredTather than square.
255
A 15-month-old boy is referred from his GP with a limp. Which radiological finding would be consistent with DDH? A Accelerated epiphyseal ossification B Decreased distance between the medial portion of the proximal femoral metaphysis and the pelvis C Increased acetabular angle D Inferolateral displacement of the femoral head in relation to Perkin's line E Preservation of Shenton's line
C
256
A 35-year-old woman with back pain has radiographs taken of her lumbosacral spine. The frontal radiograph reveals narrowing of the right sacroiliac joint with significant periarticular sclerosis. The contralateral sacroiliac joint is normal. The lumbar spine is within normal limits for age. What is the most likely diagnosis? A Ankylosing spondylitis B Infection C Osteitis condensans ilii D Psoriatic arthropathy E Reiter's syndrome
B Infection is the commonest cause of unilateral sacroiliitis, and TB should be considered as a possible organism. Ankylosing spondylitis and osteitis condensans ilii have symmetrical appearances, whilst Reiter's syndrome is bilateral (but asymmetric). Psoriatic arthropathy produces bilateral disease, which is symmetrical in most cases.
257
The skull radiograph of a 75-year-old man reveals a well-defined lytic area involving the frontal bone. A radiograph of the femur in the same patient shows a well-defined lucency extending from the articular surface to the diaphysis. The transition between lytic and normal bone is well defined and appears flame shaped. What is the most likely diagnosis? A Acromegaly B Fibrous dysplasia C Myeloma D Paget's disease E Skeletal metastases
D Paget's disease is a condition of uncertain aetiology characterised by increased turnover and excessive remodelling of bone. Osteoporosis circumscripta and advancing flame-shaped lucencies within long bones are features characteristic of active osteolytic disease. Inactive disease is characterised by widespread sclerosis: cotton wool sclerosis in the skull, enlarged ivory vertebrae with cortical thickening and coarsened thick trabeculae within the long bones. Paget's disease may be complicated by fractures, accelerated osteoarthritis.
258
A 24-year-old male presented after an inversion injury to right foot while playing football. On examination, there is swelling and bruise over the lateral aspect of the right ankle. ANP at ED performed a plain radiograph which showed soft tissue swelling over the lateral malleolus. A week later he was seen at the fracture clinic and the clinician requested for an MRI ankle. Given the likelihood of the injury, what MRI sequence is the most useful to identify the abnormality ? a. Axial PD-weighted with Fat suppression B Coronal PD-weighted with Fat suppression C Sagittal PD-weighted with Fat suppression d. Axial T1- weighted e. Coronal T1-weighted
a. Axial PD-weighted with Fat suppression
259
Which MR plane is best to identify the following ligaments? ATFL PTFL Anterior and posterior tibiofibular ligaments
Axial PD Fat sat
260
Which MR plane is best to identify the following ligaments? Deltoid ligament Calcaneofibular ligament
Coronal PD Fat sat
261
A 26-year-old man was seen in ED with an injury to his left foot after a motorbike injury. On examination, there was tenderness of the left forefoot. Plain radiograph demonstrates dorsal step off sign on lateral view. On AP view, you notice 3mm diastasis between 1st and 2nd metatarsal bases. Based on the appearances, you suspect an important ligamental injury. Where does the ligament attach to ? Middle cuneiform and 2nd metatarsal base Medial cuneiform and 1st metatarsal base Medial cuneiform and 2nd metatarsal base Middle cuneiform and 3rd metatarsal base Base of the 1st and 2nd metatarsal base
Medial cuneiform and 2nd metatarsal base
262
What is the most sensitive imaging for a Lisfranc injury?
Weight bearing radiograph
263
A 22-year-old lady presented after a traumatic injury to her right ankle while having a party. Plain radiograph of ankle shows fracture of the medial malleolus and widened distal tibio-fibular syndesmosis. You suspect further injury and requested for another radiograph. The subsequent radiograph shows fracture of the proximal fibula. What is the injury being described ? a. Triplane fracture b. Pilon fracture c. Maisonneuve fracture d. March fracture e. Pilon fracture
c. Maisonneuve fracture
264
A 32-year-old man presented with knee injury after playing rugby. MRI of the knee shows tear of ACL, lateral meniscus and medial collateral ligament. You recognized this as O’Donoghue’s unhappy triad due to pivot shift mechanism. You also noticed high signal intensity in PCL in PD sequence but normal signal intensity in T2 weighted images. What statement is false regarding magic angle artefact ? a. It is seen in T1 and GRE sequences b. It is seen in short TE sequences c. It is seen if the structure is angled at 45 degree to the main magnetic field. d. It is more seen in 1.5T machine than 3T machine e. It can be seen at TFCC ( Triangular fibrocartilage complex)
c. It is seen if the structure is angled at 45 degree to the main magnetic field. Magic angle artefact : MRI - Areas of tightly bound collagen, angled at 54.74 degree with Bo - Hyperintensity seen in short TE sequences (less than 32ms) e.g T1, PD, GRE. Therefore, can be mistaken for tendinopathy - Not seen in T2 WI - Areas affected : PCL, Supraspinatus, TFCC, Infrapetellar tendon at tibial insertion, Peroneal tendon as they hook around lat malleolus, Cartilage e.g femoral condyle
265
A 45-year-old gentleman presented with pain over the right thigh. He gives history of motorbike injury a month ago. Plain radiograph of right thigh did not show any fractures. On MRI, there is 20cm fusiform well defined area of fluid collection within the deep subcutaneous tissue on the lateral aspect of right thigh. What is the likely diagnosis ? a. Iliotibial band syndrome b. Trochanteric bursitis c. Morel-Lavellée lesion d. Hemangioma e. Parsonage-Turner syndrome
c. Morel-Lavellée lesion
266
A 19-year-old girl presented after twisting injury to right knee. Plain radiograph showed joint effusion. On MRI, there is characteristic bone edema in the medial aspect of the patella and in the lateral femoral condyle. Which statement is true regarding this condition ? A High Insall-Salvati ratio increases the risk for recurrent patellar dislocation B Patella baja increases the risk for recurrent patellar dislocation C Lateral patellar retinaculum abnormalities are commonly seen in MRI D Reduced TT-TG distance is often associated E Medial patellar retinaculum is rarely torn during patellar dislocation
A High Insall-Salvati ratio increases the risk for recurrent patellar dislocation
267
A 21-year-old boy of Ashkenazi Jews descent presented with 2 week history of right hip pain in rheumatology clinic. Initial plain radiograph demonstrated a curvilinear lucent line in the femoral head. There was palpable mass over the left hypochondriac region. You suspected Gaucher’s disease. MRI both hips were performed. Which of the following MR features would make you think an alternative diagnosis ? a. Geographic subchondral lesion outlined by serpentine low signal line in T1WI b. Peripheral low signal intensity with inner high signal intensity area in T2WI c. Diffuse bone marrow edema d. Cortical bone destruction on T1WI e. High T2/intermediate T1 signal line sandwiched between two low signal lines
d. Cortical bone destruction on T1WI
268
A middle aged female who does cross country running presented with pain in her right hip. Plain radiograph showed an osseous bump over head neck junction and over-coverage of right femoral head by acetabulum. A diagnosis of femoro-acetabular impingement was made and MRI hip arthrogram with Gd were performed to assess chondro-labral injury. Which MRI plane would you use to assess ’Alpha’ angle ? a. Coronal T1WI b. Sagittal T2 STIR c. Axial-oblique T1WI d. None of the above e. All of the above
c. Axial-oblique T1WI
269
A 60-year-old gentleman presented with pain in his right shoulder. He denies any significant trauma. On examination, there is mild tenderness over the superior aspect of the joint and limited initial abduction. Plain radiograph was normal. He was referred for US of the right shoulder for suspected supraspinatus tendinopathy. What is the ideal technique for assessing the tendon in question ? A Arm in neutral position, elbow flexed to 90 degrees, forearm supinated B Arm externally rotated with elbow flexed to 90 degrees, forearm supinated C Shoulder internally rotated and extended D Patient placing hand on the contralateral shoulder E Shoulder abducted to 120 degrees
C Shoulder internally rotated and extended
270
A 42-year-old was referred from rheumatology clinic for MRI shoulder arthrogram. When injecting the solution, you have noticed contrast in the subacromial/subdeltoid bursa. Patient then proceeded for MRI shoulder which showed a communication between glenohumeral joint and subacromial/subdeltoid bursa. You diagnosed full thickness rotator cuff tear. Which one of the following is the most commonly torn tendon? a. Tendon of deltoid muscle b. Subscapularis tendon c. Supraspinatus tendon d. Infraspinatus tendon e. Teres minor
c. Supraspinatus tendon
271
A 28-year-old rugby player presented with recurrent dislocation of left shoulder. MRI shoulder arthrogram was performed. You have noticed a tear in antero-inferior labrum in keeping with Bankart lesion. You are looking for further associated injury. What is the typical location for Hill- Sachs lesion ? Antero-inferior aspect of the humeral head Posteroinferior aspect of the humeral head Posterolateral aspect of the humeral head Posterolateral aspect of the glenoid rim Antero-medial aspect of the humeral head
Posterolateral aspect of the humeral head
272
A 24-year-old man presents with pain in his right wrist. He is a body builder and uses anabolic steroid. Plain radiograph of the wrist revealed sclerosis in the lunate bone. MRI wrist shows low T1 and low T2 signal in the lunate bone and confirms your diagnosis of Kienböck disease. Which of the following is most likely to be associated with this condition ? a. Dorsal intercalated segmental instability b. Volar intercalated segmental instability c. Negative Ulnar variance d. Positive ulnar variance e. Fracture of the triquetrum
c. Negative Ulnar variance
273
A 30-year-old lady was referred from orthopaedic clinic for wrist arthrogram. The lady initially presented with pain over ulnar side of wrist following a fall on outstretched hand. Plain radiograph demonstrated no fractures. During the contrast injection, you have noticed contrast flowing into distal radio-ulnar joint. What is the most likely explanation ? a. Tear of scapholunate ligament b. Tear of Triangular fibrocartilage complex c. Tear of lunotriquetral ligament d. Injury of distal radioulnar joint e. Normal findings
b. Tear of Triangular fibrocartilage complex
274
A 35-year-old normally fit and well lady presented with pain over wrist and pins/needles sensation in her lateral three fingers for about a year. On USS, there was a well defined fusiform hypoechoic mass with some internal heterogeneity measuring 3.5cm x 1.5cm. On MRI, the lesion is isoechoic to muscle in T1WI. It has hyperintense T2 signal with central area of low signal intensity. What is the most likely diagnosis ? a. Malignant peripheral nerve sheath tumour b. Benign peripheral nerve sheath tumour c. Intramuscular myxoma d. Carpal tunnel syndrome e. Subcutaneous lipoma
b. Benign peripheral nerve sheath tumour
275
A 26-year-old lady presented with a painful growing mass over the left upper arm for last 4 weeks. She does not recall any specific injury to her arm. Plain radiograph demonstrates a globular mass abutting the cortex of the humerus. It has osteoid matrix with dense peripheral calcification. On close inspection, you have noticed subtle periosteal reaction. Based on the likely diagnosis, which one of the following is the most appropriate next step ? Refer to regional sarcoma centre for biopsy Close interval follow up imaging c. No follow up needed Arrange an urgent CT scan Arrange an urgent MRI scan
Close interval follow up imaging
276
A 76-year-old lady presented with chronic right shoulder pain with stiffness and reduced range of motion. She gives a history of fall onto her right shoulder few months ago. Plain radiograph of the shoulder was performed which showed destruction and remodeling of the humeral head with intra-articular mineralization. MRI demonstrated extensive rotator cuff tearing. Which one of the following statements is true regarding this condition ? It occurs due to deposition of monosodium urate crystals It occurs due to deposition of calcium hydroxyapatite crystals It occurs due to deposition of calcium pyrophosphate crystals It occurs due to Iron deposition Foot is the most commonly affected joint
It occurs due to deposition of calcium hydroxyapatite crystals
277
A 75-year-old retired bus driver presented with discomfort in his right hand. He is an ex-smoker and takes medications for hypertension. GP requested plain radiograph of both hands to look for any osteoarthritis. While reading the film, you have noted an aggressive lytic lesion in the right thumb. There were no previous films to compare. Based on the patient demographic and the appearance of the lesion, you suspect a metastatic lesion. Which of the following is the most likely primary site ? a. Kidney b. Prostate c. Lung d. Breast e. Colorectal cancer
c. Lung
278
A 16-year-old boy presented with discomfort around left shoulder joint. No history of prior trauma. Plain radiograph demonstrated a well defined centrally located lucent lesion in the proximal metaphysis of the humerus. There is mild expansion of the bone with endosteal scalloping. In addition, there is smooth periosteal reaction but no fractures or cortical breach. What is the most likely diagnosis ? a. Aneurysmal bone cyst b. Simple bone cyst c. Giant cell tumour d. Intraosseous lipoma e. Telangiectatic osteosarcoma
a. Aneurysmal bone cyst
279
A 70-year-old gentleman presented to his GP with low back pain following a long haul road trip. He is known to have DM, HTN and hypercholesterolemia. No focal neurology on examination. GP referred him for plain radiograph of spine to assess for any vertebral compression fracture. Plain radiograph revealed flowing bridging anterior vertebral osteophytes with no significant disc space narrowing. You also noted ossification of the posterior longitudinal ligament on CT abdomen Pelvis performed 3 weeks ago. Based on your diagnosis, which one of the following features may also be present ? a. Pelvic enthesophytes b. Sacroiliitis c. d. Hook like osteophyte in the metacarpals Aneurysmal bone cyst in the vertebra e. All of the above
a. Pelvic enthesophytes
280
A 53-year-old woman is seen in the general surgical outpatient clinic. She attended her GP with a 1-month history of upper abdominal pain and was found to have a palpable, firm mass in the epigastrium. An upper gastrointestinal (GI) endoscopy is normal and the surgical team request a contrast-enhanced CT of the abdomen. This demonstrates a multicystic mass in the pancreas. Which findings would make a mucinous cystic tumour more likely than a serous cystadenoma? A Central stellate calcification is present within the lesion. B The mass contains 12 separate cysts. C The smallest cystic component measures 28 mm in diameter. D The patient has a known diagnosis of von Hippel-Lindau disease. E The tumour is located in the head of the pancreas.
C Mucinous cystic pancreatic tumours (cystadenomas and cystadenocarcinomas) typically contain a few large cysts, each measuring more than 20 mm diameter.
281
A 42-year-old man presents to the Emergency Department with a 7-day history of severe bloody diarrhoea and abdominal pain. He has previously been fit and well with no significant medical history. On examination, the patient is dehydrated with generalized abdominal tenderness but no clinical evidence of peritonism. An abdominal radiograph is performed. Which radiographic finding would be most suggestive of a toxic megacolon? A Caecum measuring 4.5 cm in diameter B Multiple mucosal islands in a dilated transverse colon C Pseudodiverticulae in the descending colon D Thickened haustrae throughout the entire colon E ‘Thumbprinting’ of the transverse and descending colon
B The presence of severe ulceration leading to mucosal islands is a major sign of toxic megacolon (the other key finding is colonic dilatation > 5 cm).
282
A 26-year-old man presents to the Emergency Department with acute epigastric pain and vomiting. The serum amylase is found to be markedly elevated and the patient is treated for acute pancreatitis. A contrastenhanced CT of the abdomen is subsequently performed and demonstrates calcification throughout the pancreas. Bilateral renal calculi are also noted. What is the most likely underlying diagnosis? A Hereditary' pancreatitis B Hyperparathyroidism C Hypoparathyroidism D Mucinous cystadenocarcinoma E Multiple pancreatic pseudocysts
B A significant minority of patients with hyperparathyroidism will develop acute pancreatitis and around 30% of these patients develop pancreatic calcification. Hypoparathyroidism is associated with calcification in the soft tissues but pancreatic calcification is not a recognised feature. Hereditary pancreatitis is an autosomal dominant condition with 60% of patients demonstrating round, coarse calcification of the pancreas.
283
A 35-year-old pregnant woman (28 weeks gestation) presents to her GP with right upper quadrant abdominal pain and is found to have abnormal liver function tests. An abdominal ultrasound is performed and demonstrates a diffusely hyperechoic liver with a discrete 4-cm hypoechoic lesion in the right lobe. An MRI is performed 3 weeks later, showing that the liver lesion has increased in size, now measuring 7 cm diameter. The lesion is isointense on in-phase Tlw images, losing signal on out-of-phase images. Following intravenous gadolinium there is immediate and intense enhancement with early washout. What is the most likely diagnosis? A Choriocarcinoma B Focal nodular hyperplasia C Hepatic adenoma D Liver haemangioma E Metastatic breast cancer
C The MRI findings are highly suggestive of a hepatic adenoma and the history of rapid growth during pregnancy is supportive of this diagnosis.
284
A 59-year-old man undergoes surgical resection of a rectal tumour. A contrast-enhanced CT of the abdomen is performed 3 months later and demonstrates a new, solitary 3-cm liver metastasis. The lesion lies inferior to the level of the left and right portal veins and posterior to the right hepatic vein. The remainder of the CT examination is unremarkable and the patient is assessed for surgical resection of the liver lesion. Which segment of the liver does the liver metastasis lie in? A Segment 4b B Segment 5 C Segment 6 D Segment 7 E Segment 8
C The Couinaud classification divides the liver into 8 independent segments, each of which has its own vascular supply and biliary drainage. The portal vein separates the superior and inferior segments, with further division based on the relationship to the nearest hepatic veins. Because of this division into selfcontained units, the Couinaud classification carries particular importance in the setting of resectable liver lesions, such as a solitary colorectal metastasis
285
A 38-year-old woman receives an orthotopic liver transplant for chronic liver failure due to primary biliary cirrhosis. The patient's liver enzyme levels become markedly elevated after 24 hours and her clinical condition deteriorates. An abdominal ultrasound is performed with Doppler evaluation of the hepatic vessels. Given the clinical history, which vascular complication is most likely to have occurred? A Arterioportal fistula B I VC thrombosis C Hepatic artery stenosis D Hepatic artery thrombosis E Portal vein thrombosis
D Hepatic artery thrombosis is the most common and serious early vascular complication post liver transplant. Thrombolysis is an option in this setting, but many patients will ultimately require retransplantation. Prompt and accurate diagnosis is therefore essential. A parvus et tardus waveform may be seen distal to stenosis of the hepatic artery, with associated elevated systolic velocity at the stenotic segment. Porta! vein thrombosis is less common and can result in the acute development of ascites and varices. IVC thrombosis is rare and usually related to surgical technique, whilst an arterioportal fistula is an infrequent complication of liver biopsy.
286
A 68-year-old man presents to his GP with a 1-month history of epigastric pain, vomiting and mild weight loss. Examination is unremarkable and the patient is referred for an upper gastrointestinal endoscopy. This demonstrates mild gastritis with biopsies positive for Helicobacter pylori and he is commenced on eradication therapy. Three months later, the symptoms have persisted and the patient has lost 5 kg in weight. A double contrast barium meal is performed and reveals a shallow ulcer on the lesser curve of the stomach. Which additional finding would make the ulcer more likely to be benign than malignant? A Hampton's line is present. B Nodular mucosal folds stop at the edge of the lesion. C The ulcer does not extend beyond the gastric wall. D The ulcer has an irregular margin. E The ulcer measures 40 mm in size.
A Hampton’s line refers to a lucent line crossing the ulcer base: its presence is highly suggestive of a benign ulcer.
287
A 42-year-old man has type 1 diabetes mellitus. Despite intensive medical management, the patient's glycacmic control remains problematic and he receives a cadaveric pancreatic transplant with the pancreatic graft anastomosed to the right common iliac artery. Four days following surgery, the clinical team are concerned about the pancreatic graft function and request radiological assessment for post-transplant complications. Which one of the following statements is true regarding pancreatic transplant imaging? A In acute rejection, the pancreatic graft is small and hyperechoic on ultrasound. B Pancreatic exocrine secretions often drain into the urinary bladder. C Radionuclide imaging with Tc-99m-pertechnetate is the most sensitive way of detecting acute pancreatic rejection. D Surgical complications are more common following renal transplantation than pancreatic transplantation. E Transplant pancreatitis is very rare in the first 48 hours post surgery.
B Formation of a cystoduodenostomy drains the exocrine pancreatic secretions into a duodenal loop, anastomosed direedy with the urinary bladder. Enteric drainage into small bowel can also be performed depending on surgical technique.
288
A 52-year-old female patient is under the care of a rheumatologist with a diagnosis of diffuse scleroderma. She presents to her GP with vomiting, intermittent abdominal pain and reduced bowel habit. An abdominal radiograph demonstrates several loops of gas-filled bowel but there is no evidence of mechanical obstruction. A barium follow-through examination is performed. In view of the clinical history, what are the most likely findings? A Dilated small bowel with increased number of valvulae conniventes B Extraluminal mass in the ileum, causing ulceration and a shouldered stricture C Long irregular ileal stricture with antimesenteric mucosal thickening D Nodular thickening of the valvulae conniventes of the duodenum only E Short stricture in the terminal ileum with ‘cobblestoning’ of the mucosa
A This describes the characteristic ‘hide bound’ appearance of the small bowel in scleroderma.
289
A 23-year-old woman complains of episodes of diarrhoea and rectal bleeding. Her father died of colorectal cancer aged 39. A double contrast barium enema is performed and demonstrates more than one hundred small polyps, measuring up to 5 mm in size, throughout the colon. An upper GI endoscopy demonstrates multiple polypoid lesions in the stomach and duodenum. What is the most likely diagnosis? A Carcinoid syndrome B Familial adenomatous polyposis C Hereditary non-polyposis colorectal cancer D Juvenile polyposis E Peutz-Jeghers syndrome
B Autosomal dominant condition with multiple colonic adenomas and 100% risk of colorectal carcinoma 20 years after diagnosis. Associated with hamartomas in the stomach, gastric and duodenal adenomas and periampullary carcinoma.
290
A 78-year-old man has myelodysplastic syndrome and requires frequent blood transfusions. He develops progressively abnormal liver function tests and a grossly elevated ferritin level. An MRI of the liver is performed using breath hold half Fourier single shot spin echo T2w images. Which finding would make a diagnosis of haemosiderosis (iron overload from recurrent blood transfusion) more likely than haemochromatosis? A Increased T2 signal in the liver only B Increased T2 signal in the liver and spleen C Reduced T2 signal in the liver only D Reduced T2 signal in the liver and spleen E Reduced T2 signal in the spleen only
D In iron overload due to recurrent transfusions, there is increased iron deposition in the reticuloendothelial system. This leads to reduced Tl, T2 and T2* signal intensity in the liver and spleen. Haemochromatosis causes diffusely reduced T2 signal in the liver and may lead to cirrhosis, but the splenic signal intensity should remain normal. Diffuse fatty liver will lead to increased T2 signal in the liver with signal loss during out-of-phase images.
291
A 31-year-old woman develops mild acute pancreatitis and is managed conservatively. It is her third episode of pancreatitis but there is no history of excess alcohol consumption and an abdominal ultrasound is normal. Magnetic resonance cholangiopancreatography (MRCP) is performed and is reported as showing evidence ofpancreas 4bdw,rn- Which one of the following findings is likely to have been present on MRGP? A A 3-cm cystic structure in the head of the pancreas B An accessoiy pancreatic duct passing around the duodenum C The common bile duct draining into the minor papilla D The dorsal pancreatic duct (duct of Santorini) draining into the minor papilla E The ventral pancreatic duct draining into the minor papilla
D It appears that relative stenosis of the cranially sited minor papilla results in increased risk of pancreatitis in these patients.
292
A 22-year-old woman presents to her GP with a 4-month history of increasing right upper quadrant pain. An abdominal ultrasound is performed and demonstrates a 6-cm solid lesion of increased reflectivity in segment 6 of the liver. A contrast-enhanced CT of the liver is performed and demonstrates that the lesion enhances moderately and has a lobulated margin. Which additional finding would make a diagnosis of fibrolamellar carcinoma more likely than that of focal nodular hyperplasia (FNH)? A A hyperechoic central scar B A preexisting history of chronic liver disease C Delayed enhancement of a central scar D Punctuate calcification in the lesion E The patient is talcing the combined oral contraceptive pill
D There is considerable overlap in the imaging appearances of these two conditions, but punctate calcification occurs in over half of patients with fibrolamellar carcinoma and is extremely unusual in FNH.
293
A 33-year-old woman presents to her GP with a one year history of intermittent rectal bleeding. She experiences regular episodes of fresh blood per rectum with associated lower abdominal pain, lasting several days at a time. A flexible sigmoidoscopy is normal. A double contrast barium enema is performed and demonstrates an irregular appearance of the anterior wall of the sigmoid colon with mild extrinsic mass effect. What is the most likely diagnosis? A Carcinoma of the sigmoid colon B Endometriosis C Pelvic lipomatosis D Radiation enteritis E Solitary rectal ulcer syndrome
B Involvement of the GI- tract is not uncommon in endometriosis and the sigmoid colon and pelvic small bowel loops are typical sites of involvement.
294
A 59-year-old man presents to his GP with a 3-day history of right upper quadrant pain and vomiting. There is a past medical history of ischaemic heart disease and type 2 diabetes mellitus. An abdominal ultrasound demonstrates thickening of the gallbladder wall and pericholecystic fluid, but no gallstones. The patient deteriorates clinically with elevation of white cell count and CRP levels. A repeat ultrasound 3 days later demonstrates a bright echogenic area in the gallbladder fundus with acoustic shadowing. What is the most likely diagnosis? A Adenomyomatosis B Emphysematous cholecystitis C Gallbladder carcinoma D Mirizzi syndrome E Porcelain gallbladder
B The development of gas in the gallbladder of an unwell diabetic patient is suggestive of emphysematous cholecystitis.
295
A 48-year-old man presents to his GP with epigastric pain, diarrhoea and weight loss over a period of 6 months. Laboratory investigations reveal a reduced serum albumin, and a contrast-enhanced CT of the abdomen demonstrates diffuse thickening of the gastric mucosa. A double contrast barium meal examination is performed and shows markedly thickened mucosal folds in the gastric body with sparing of the gastric antrum. The mucosal folds alter in size and position during the examination. What is the most likely diagnosis? A Eosinophilic gastritis B Gastric lymphoma C Infiltrative gastric adenocarcinoma (linitis plastica) D Menetrier's disease E Organoaxial gastric volvulus
D Menetrier’s disease characteristically produces thickened hyperplastic mucosa (sparing the gastric antrum) but the stomach remains pliable.
296
A 46-year-old woman from Bangladesh is being treated for pulmonary tuberculosis. Despite anti-tuberculosis chemotherapy, she develops increasing fevers with abdominal discomfort and distension. An abdominal and pelvic ultrasound demonstrates a moderate volume of peritoneal free fluid, and a contrast-enhanced CT of the abdomen and pelvis is performed. What are the lilcely findings on CT? A A mixed solid:cystic ovarian mass with serosal deposits on the liver and spleen B Ascites with enlarged mesenteric lymph nodes containing high attenuation C Gastric wall thickening extending into the spleen with enlarged coeliac axis lymph nodes and ascites D Peritoneal nodularity with high density ascites E Portal vein thrombosis with ascites
D In peritoneal TB, the presence of dense ascites, peritoneal nodularity and lymph nodes with low attenuation centres are characteristic findings.
297
A 49-year-old man develops weight loss, upper abdominal pain and three episodes of vomiting fresh red blood. Subsequent upper gastrointestinal endoscopy reveals a distal gastric adenocarcinoma. The patient undergoes a surgical procedure to resect the tumour, but develops increasing epigastric pain and fever 4 days later. An upper GI contrast study is performed. Which one of the following statements is true regarding this examination? A A partial distal gastrectomy with gastrojejunostomy (Billroth II procedure) involves an end-to-end anastomosis. B Control images prior to contrast administration are not indicated in this C If a water-soluble contrast examination appears normal, barium can be used as it has a higher sensitivity in identifying anastomotic leaks. D The oesophago-gastric junction is the most common site for perforation and contrast leaks. E Thickening of the mucosa at the surgical anastomosis with delayed gastric emptying is most likely due to residual gastric tumour.
C Anastomotic leakage is one of the most serious complications following gastric surgery and may occur in the acute or chronic phase. In a contrast study, water-soluble contrast should be used initially, but if no leak is detected then barium can be used as it is more sensitive for the detection of subtle postoperative leaks (outweighing the risk of barium spilling into the peritoneal cavity). The most common site for leakage is at surgical anastomoses and suture lines; therefore control images are invaluable to note the location of these sites and look for extraluminal gas
298
A 56-year-old woman presents with a 4-day history of right upper quadrant pain and vomiting. She describes a previous episode one year ago that resolved after a few days. On examination, she is very tender in the right upper quadrant with guarding on deep palpation during inspiration. Laboratory investigations reveal elevated white cell count and CRP but normal liver function tests and an abdominal ultrasound is performed. What are the most likely ultrasound findings? A Hypoechoic mass in the pancreatic head with common bile duct measuring 14 mm and pancreatic duct measuring 6 mm in diameter B Nodular liver surface, mixed reflectivity liver texture and ascites C Severe intrahepatic duct dilatation with no cause identified D Several large gallstones with gallbladder wall measuring 5 mm and a rim of pericholecystic fluid E Several small gallstones with gallbladder wall thickness of 2 mm
D Ultrasound findings of acute cholecystitis also include gallbladder distension and the sonographic Murphy's sign (the patient is unable to breathe in deeply when the probe is pressed firmly over the gallbladder). The normal gallbladder wall can measure up to 3 mm in thickness and wall thickening may be due to nonbiliary causes (hypoalbuminaemia, heart failure, etc).
299
A 42-year-old man is admitted to hospital with acute abdominal pain. There is a significant medical history of polycythaemia rubra vera, for which the patient undergoes regular venesection. On examination, there is right upper quadrant tenderness and hepatomegaly. Liver function tests are acutely elevated and the patient's condition deteriorates. A catheter angiogram is performed to assess the major hepatic vessels and shows a ‘spider's web’ appearance within the liver. What is the diagnosis? A Budd Chiari syndrome B Capillary haemangioma C Hereditary haemorrhagic telangiectasia (HHT) D Portal vein thrombosis E Spontaneous hepatic haematoma
A When contrast is injected into the hepatic veins, a ‘spider's web’ appearance of collateral vessels is diagnostic of Budd Chiari syndrome.
300
A 74-year-old woman is referred to the hepatology outpatient clinic with persistently abnormal liver function tests. There is a past medical history of myocardial infarction, atrial fibrillation and hypertension, but no previous history of liver disease. On abdominal ultrasound, the liver appears normal with antegrade portal venous flow demonstrated. A CT of the abdomen is performed and the mean density of the liver is 86 Hounsfield Units (HU) precontrast. What is the most likely diagnosis? A Amiodarone therapy B Chronic Budd Chiari syndrome C Chronic hepatitis B D Diffuse fatty infiltration E Wilson's disease
A Amiodarone contains iodine; therefore deposition in the liver leads to increased density on CT.
301
A 68-year-old man presents to his GP with weighc loss and jaundice. Liver function tests demonstrate obstructive jaundice and an abdominal ultrasound shows mild intrahepatic biliary dilatation with a common bile duct measuring 12 mm in diameter. In the pancreatic head, a 3-cm hypoechoic mass is present. An ERCP is performed with insertion of a plastic stent and brushings confirm a pancreatic ductal adenocarcinoma. A triple-phase (precontrast, arterial and portal venous) multidetector CT of the pancreas is performed. Which finding would indicate a nonresectable pancreatic tumour? A Enhancing pancreatic parenchyma between the tumour and superior B The pancreatic duct dilated to 6 mm C The presence of a 5-mm coeliac axis lymph node D The tumour has invaded the duodenum E The tumour in contact with 75% of the superior mesenteric arteiy
E If the tumour is in contact with more than half of the vessel circumference, it is very unlikely to be resectable.
302
An 82-year-old woman is referred to the on-call surgical team as an emergency admission. The patient lives in a residential care home and has a 48-hour history of generalised abdominal pain and vomiting. On examination, she is dehydrated and tachycardic and an abdominal radiograph demonstrates multiple dilated small bowel loops measuring up to 4.8 cm in diameter. A linear gas-filled structure is present in the right upper quadrant with short branches extending from it. What is the most likely diagnosis? A Acute mesenteric ischaemia B Emphysematous cholecystitis C Gallstone ileus D Obstructed right inguinal hernia E Small bowel obstruction due to adhesions
C A gallstone causing a cholecystoduodenal fistula and aerobilia can also obstruct the distal small bowel.
303
A 47-year-old man is knocked off his motorcycle and brought to the Emergency Department. On examination, he is haemodynamically stable but has left upper quadrant tenderness. A contrast-enhanced CT of the abdomen is performed and shows no evidence of visceral injury. The reporting radiologist notices a solitary, well-defined lesion in the large bowel that is of lower attenuation than the surrounding colonic wall. Which single additional finding would be most consistent with a colonic lipoma? A There is a mean density of —10 HU on CT. B Mucosal ulceration is seen on colonoscopy. C On ultrasound, the lesion changes shape when compressed. D The lesion lies in the sigmoid colon. E There is a one-month history of rectal bleeding and weight loss.
C Known as the 'squeeze sign’.
304
A 42-year-old man has a history of alcohol excess and a previous duodenal ulcer. He presents to the Emergency Department with a 1-day history of epigastric pain and vomiting. Initial laboratory investigations are remarkable for a grossly elevated serum amylase and the patient is treated with intravenous fluids and analgesia. Six days later, his condition deteriorates and he develops a temperature of 39°C. A contrast-enhanced CT of the pancreas is performed. Which one of the following findings would be most indicative of infected pancreatic necrosis? A An area of nonenhancement in the pancreatic body containing a locule of B Diffuse enlargement of the pancreas with peripancreatic fat stranding C Focal enlargement of the pancreatic head with reduced enhancement D Splenic artery pseudoaneurysm formation adjacent to the pancreatic tail E Splenic vein thrombosis extending to the superior mesenteric vein
A Necrotic pancreatic tissue will demonstrate reduced or absent enhancement on contrast-enhanced CT. It may not be possible to differentiate sterile from infected necrosis, but the presence of gas is a strong predictor of infection. Local vascular complications are well recognised to occur in severe acute pancreatitis but may occur without pancreatic necrosis.
305
All 80-year-old man is referred to the gastroenterology outpatient clinic with a 1-year history of dysphagia. He describes worsening difficulty swallowing solids and liquids with associated loss of 3 kg in weight. The past medical history includes Parkinson’s disease and right lower lobe pneumonia 6 months ago. An upper gastrointestinal endoscopy is normal and the patient is referred for a contrast swallow examination for suspected oesophageal dysmotility. Which statement is true regarding this examination? A If aspiration is suspected, water-soluble meglumine diatrizoate (Gastrografin) should be used initially. B In suspected oesophageal dysmotility, an antispasmodic (eg Buscopan) should be administered prior to prone swallow. C Motility of the mid- and lower oesophagus is best assessed with the patient standing erect in the left anterior oblique position. D Repeated swallowing should be avoided and only single boluses of barium be used to assess for oesophageal dysmotility. E Secondary oesophageal contractions are chaotic and do not propel the barium bolus.
D Repeated swallowing can interrupt normal peristalsis and produce a falsely abnormal appearance.
306
A 72-year-old man is referred to hospital as an emergency admission by his GP. He has experienced vomiting and abdominal pain for 24 hours following a takeaway meal. There is a past medical history of ischaemic heart disease, chronic obstructive pulmonary disease and hypertension. An abdominal radiograph is performed and demonstrates several gas-filled loops of small bowel centrally measuring up to 2.5 cm diameter. In the left side of the abdomen, multiple round foci of gas are projected over the wall of a loop of large bowel. No free gas or mucosal thickening is identified. What is the most likely explanation for the clinical and radiographic findings? A Gastroenteritis with incidental pneumatosis coli B Emphysematous pyelonephritis with a paralytic ileus C Ischaemic colitis causing intramural bowel gas D Perforated sigmoid diverticulitis with gas in the retroperitoneum E Small bowel obstruction due to a gallstone ileus
A The presence of cyst-like gas pockets in the left hemicolon of a patient with COPD is suggestive of pneumatosis cystoides intestinalis (pneumatosis coli).
307
A 68-year-old woman presents with a 2-month history of generalised abdominal bloating and two episodes of vaginal bleeding. On examination, the abdomen is distended with clinical evidence of ascites. Tumour markers are performed; CA 15-3 is normal, CA 125 and CEA are slightly elevated and CA 19-9 is markedly elevated. An abdominopelvic ultrasound demonstrates a moderate volume of ascites, multiple liver metastases and bilateral mixed solid/cystic adnexal masses. What is the most likely underlying primary tumour? A Breast cancer B Gastric adenocarcinoma C Melanoma D Ovarian cancer E Primary peritoneal carcinoma
B This clinical history is classic for a Krukenberg tumour—ovarian metastases from a GI tumour (most frequently a gastric adenocarcinoma). Colorectal cancer is the second most common cause of this type of metastatic tumour presentation. Although melanoma may spread anywhere in the body, it is not a common cause of ovarian metastases. Metastatic ovarian cancer is made less likely by the minimally elevated CA-125 (would expect very high levels) and the presence of liver metastases.
308
A 53-ycar-old man is seen in the liver transplant outpatient clinic. Two years ago, he underwent an orthotopic liver transplant for alcoholic liver disease and currently takes oral cyclosporin. He reports a 3-month history of weight loss and his liver function tests are found to be abnormal. A contrastenhanced CT demonstrates multiple new low attenuation lesions within the liver. There is also marked thickening of several small bowel loops. What is the most likely diagnosis? A Chronic graft ischaemia with portal hypertension B Cyclosporin hepatotoxicity C Multifocal hepatocellular carcinoma D Post transplant lymphoproliferative disorder (PTLD) E Secondary amyloidosis
D Up to 5% of liver transplants develop PTLD: extranodal disease is the most common pattern.
309
A 29-year-old woman received a living related bone marrow transplant for chronic myeloid leukaemia 13 days ago. She has experienced bloody diarrhoea and severe lower abdominal pain for the past 4 days and an abdominal radiograph demonstrates prominent loops of gas-filled large bowel. A contrast-enhanced CT of the abdomen is performed and shows moderate wall thickening of the right hemicolon and terminal ileum with mesenteric fat stranding. There is no abdominal lymphadenopathy and the rectum and sigmoid colon appear normal. What is the most likely diagnosis? A Crohn's disease B Cytomegalovirus (CMV) colitis C Neutropenic colitis D PTLD E Pseudomembranous colitis
C The patient is highly likely to be severely neutropenic at this stage and the CT findings are typical of neutropenic colitis.
310
A 32-year-old man with Crohn's disease reports increased perianal pain and swelling over a 2-month period. On examination, there is a small perineal sinus lying at die 3 o'clock position in relation to the anus. On MRI, a fistulous track of high T2 signal is seen to pass from the anal canal, through the internal sphincter and then runs medial to the external sphincter. The track reaches the skin surface of the perineum and correlates with the sinus opening on physical examination. Which description best describes this anal fistula? A Extrasphincteric B Infrasphincteric C Intersphincteric D Suprasphincteric E Trans-sphincteric
C The Parks' classification defines anal fistulae by the structures involved. The intersphincteric fistula is the most common of anal fistulae (around 70%) and does not pass through the external sphincter. A trans-sphincteric fistula will cross both the internal and external sphincters to reach the skin surface while a suprasphincteric fistula passes above the puborectalis muscle to involve the ischiorectal fossa. The least common fistula is the extrasphincteric fistula as this arises from the rectum and passes through the levator ani muscles to reach the skin surface without involving the anal sphincter mechanism at all.
311
A 32-year-old woman undergoes a laparoscopic cholecystectomy for gallstones. even days later, she presents to the Emergency Department with increasing abdominal pain and fevers. On examination, her temperature is 39.6°C, HR =100 bpm and BP = 110/60 mmHg with tenderness and guarding in the right upper and lower quadrants of the abdomen. Laboratory investigations reveal a grossly elevated CRP level and white cell count. The clinical team request a contrast-enhanced CT for suspected intra-abdominal sepsis. Which statement is true regarding intra-abdominal fluid collections? A Fluid in the lesser sac communicates freely with the left subphrenic space. B Fluid in the right paracolic gutter communicates freely into the pelvis and superiorly to the right subdiaphragmatic space. C Fluid in the right paracolic gutter will be bounded superiorly by the phrenicocolic ligament. D Postoperative gallbladder collections usually lie in the right infracolic space. E The right subphrenic space is also called ‘Morrison's pouch’.
B The right paracolic gutter communicates freely with the right perihepatic space (bounded by the falciform ligament). Postoperative gallbladder collections will tend to lie in the gallbladder fossa and the hepatorenal recess (also known as Morrison's pouch).
312
A 79-year-old man is brought to the Emergency Department widi generalised abdominal pain and vomiting for 5 days. He has not opened his bowels or passed flatus during this period and has been immobile for the past 48 hours. On examination, he is dehydrated with a distended abdomen and increased bowel sounds. An abdominal radiograph is performed and shows dilated loops of large bowel, measuring up to 5 cm in diameter. Dilated small bowel is present centrally but there is no evidence of perforation. Which statement is true regarding this clinical setting? A Colonic pseudo-obstruction is a recognised cause of these radiographic findings. B Diverticulitis is the most common cause of large bowel obstruction in the UK. C Obstruction of the large bowel occurs more commonly on the right side of the colon than the left. D Paralytic ileus is excluded by these radiographic findings. E The ileocaecal valve is not competent in this patient.
A Colonic pseudo-obstruction can produce the same radiographic findings as large bowel obstruction: an instant contrast enema or CT should differentiate mechanical obstruction from pseudo-obstruction.
313
A 27-year-old man is referred to the hepatology outpatient clinic with a 3-week history of malaise, lethargy and mild upper abdominal pain. Liver function tests performed by his GP are signiflcandy abnormal. The results of hepatitis serology performed in the clinic are consistent with an acute hepatitis B infection. An abdominal ultrasound is performed. What is the most likely finding on ultrasound? A Decreased reflectivity of the liver parenchyma B Increased reflectivity of the liver parenchyma C Nodular liver surface D Normal ultrasound appearances E Retrograde portal venous flow
D In acute viral hepatitis, there can be diffusely reduced reflectivity of the liver but the majority of patients have a normal ultrasound examination.
314
A 49-year-old woman is an emergency admission to the surgical admissions unit with a 5-day history of upper abdominal pain. On clinical examination, there is right upper quadrant tenderness and laboratory investigations show an elevated white cell count and CRP. An abdominal ultrasound is performed, but is of limited value due to the patient's body habitus and the gallbladder is poorly visualised. The patient undergoes dynamic radioisotope hepatobiliary scintigraphy with an intravenous injection of a Tc-99 m-labelled pharmaceutical. Which one of the following statements is true regarding radioisotope hepatobiliary scintigraphy? A Increased isotope activity in the region of the gallbladder is consistent with acute cholecystitis. B Nonvisualisation of the gallbladder after 2 hours is consistent with acute cholecystitis. C Sulphur colloid is the most commonly used pharmaceutical in this D The administration of intravenous morphine causes sphincter of Oddi relaxation. E Visualisation of isotope activity in the duodenum is abnormal.
B The normal gallbladder will appear after approximately 20 minutes. In acute cholecystitis, the gallbladder is typically not seen due to cystic duct obstruction.
315
A 79-year-old woman trips and falls whilst stepping off a bus. She suffers a fractured right neck of femur and undergoes a hemiarthroplasty the following day. Her early recovery is complicated by bronchopneumonia which resolves after 5 days of broad spectrum antibiotics. On her tenth day in hospital she develops abdominal pain and diarrhoea and pseudomembranous colitis is suspected clinically. Which one of the following statements is true regarding pseudomembranous colitis? A A normal abdominal CT effectively excludes pseudomembranous colitis. B Ascites is present in up to 40% of patients. C CT carries a low positive predictive value for pseudomembranous colitis. D Extensive pericolonic stranding is a typical feature on CT. E The rectum is not involved in 40—50% of patients.
B Ascites can occur with other colitides, but is often seen in pseudomembranous colitis. CT typically demonstrates mucosal enhancement and marked colonic wall thickening but only mild pericolonic stranding, in patients with pseudomembranous colitis. These findings have a high positive predictive value but a normal CT does not exclude pseudomembranous colitis. Rectal sparing occurs in around 10% of patients.
316
A 62-year-old woman presents to the Emergency Department with a 2-day history of excruciating abdominal pain and is found to have an elevated serum amylase. An abdominal ultrasound demonstrates multiple stones in the gallbladder but there is no biliary dilatation and the pancreas is obscured by bowel gas. The patient's clinical condition deteriorates and a contrastenhanced CT is performed. This demonstrates ill-defined enlargement of the pancreas with infiltration of the peripancreatic fat. The peripancreatic fluid is localised only to the anterior pararenal space. Which one other structure also lies in the anterior pararenal space? A Descending colon B Gallbladder C Kidneys D Spleen E Stomach
A The anterior pararenal space is the most anterior of the three retroperitoneal compartments.
317
A 40-year-old woman has a 15-year history of ulcerative colitis (UC). After the initial diagnosis, she suffered frequent exacerbations of colitis requiring several hospital admissions. She declined surgical intervention at that stage and has subsequently been well controlled on medical management. Recently, she has developed a change in bowel habit and a double contrast barium enema is performed. This shows a stricture in the descending colon. Which one statement is true regarding strictures in ulcerative colitis? A Abrupt shouldering is typical of a benign stricture in UC. B In patients with UC, colorectal carcinomas typically arise from tubular adenomas. C The majority of strictures in UC are benign. D There is no increased risk of colorectal carcinoma in patients with UC. E Widening of the presacral space is pathognomonic of a rectal carcinoma.
C Benign strictures in UC are typically smooth and symmetrical and are due to chronic smooth muscle hypertrophy. These occur in 10—20% of patients with UC and are most common in the left colon. Carcinomas arise from dysplastic changes within the diseased epithelium and not from adenomas as in the general population.
318
A 17-year-old man is referred to the gastroenterology outpatient clinic with iron deficiency anaemia. The patient is otherwise well with no gastrointestinal symptoms and a normal physical examination. Endoscopic examination of the upper and lower gastrointestinal tract is normal. A mesenteric catheter angiogram is performed and demonstrates a persistent vitelline artery. What is the diagnosis? A Behfet's disease B Colonic arteriovenous malformation C Intestinal lymphangiectasia D Meckel's diverticulum E Small bowel angiodysplasia
D This angiographic finding is diagnostic of a Meckel’s diverticulum as it indicates that a remnant of the omphalomesenteric (vitelline) duct is present.
319
A 48-year-old man has a strong family history of colorectal cancer. He is found to have a mild microcytic anaemia and a stool sample for faecal occult blood testing is positive. A CT colonography is performed and, on 3D images, a 1-cm focal polypoid mass is seen in the wall of the sigmoid colon. The reporting radiologist is unsure whether this lesion is significant and reviews the 2D supine and prone axial images. Which additional feature would be most consistent with a polyp? A The lesion contains a locule of gas at its base. B The lesion has a mean density of — 150 HU. C The lesion is of homogeneous attenuation. D The lesion lies on the dependent surface of the bowel on prone and supine images. E There are diverticulae seen in the sigmoid colon.
C A polyp will usually demonstrate uniform soft tissue density, similar to the surrounding bowel wall.
320
A 23-year-old man presents with a 2-day history of vomiting and generalised abdominal pain. Two years ago, he underwent a small bowel resection for an ileal stricture due to Crohn's disease. Initial blood tests reveal a raised CRP and white cell count and an abdominal radiograph demonstrates dilated loops of small bowel. Small bowel obstruction is suspected and a contrast-enhanced CT of the abdomen is performed. Which one of the following statements is true regarding the role of multidetector CT in small bowel obstruction? A Five to 15% of small bowel obstructions are due to hernias. B Twenty to 30% of small bowel obstructions arc due to adhesions. C Bowel wall thickening and intramural gas indicate the presence of pneumatosis coli. D Closed loop obstruction is less likely to result in bowel ischaemia than simple obstruction. E In small bowel obstruction due to adhesions, a transition point will not be seen.
A Bowel wall thickening, lack of enhancement, adjacent fluid and pneumatosis ntestinalis are all CT signs of ischaemia (strangulation) in small bowel obstruction. Fifty to 80% of small bowel obstruction is attributable to idhesions while 10% is due to hernias. In adhesions, there will usually be a listory of previous abdominal surgery with CT demonstrating small bowel ibstruction. The transition point may be identified, but the actual adhesive land is usually not visualised.
321
A 67-year-old woman undergoes surgical resection of a distal sigmoid adenocarcinoma. The surgeon performs a primary anastomosis between the descending colon and rectum and leaves a defunctioning loop colostomy. Nine days later, the patient is experiencing fevers and low abdominal pain. A contrast-enhanced CT shows a small fluid collection around the anastomosis with no definite abscess identified. The surgical team are concerned about the integrity of the anastomosis. Which investigation would you choose to look for an anastomotic leak? A Barium enema B Barium follow-through C MRI pelvis with intravenous gadolinium D Water-soluble contrast cystogram E Water-soluble contrast enema
E The combination of water-soluble contrast enema and CT is used to look for anastomotic leakage and abscess formation.
322
A 64-year-old woman presents to her GP with increasing discomfort in her upper abdomen and anorexia. There is a past medical history of gallstones. The GP requests an abdominal ultrasound and this demonstrates a 6 x 4 cm mixed echogenicity lesion in the gallbladder fossa, with the gallbladder not separately visualised. On CT, the gallbladder fossa mass demonstrates central low attenuation with peripheral enhancement and mild intrahepatic biliary dilatation. Low attenuation lymph nodes are present at the porta hepatis (measuring up to 1.5 cm short axis). Which diagnosis is most likely? A Adenomyomatosis B Gallbladder carcinoma C Hepatocellular carcinoma D Porcelain gallbladder E Xanthogranulomatous cholecystitis
B A gallbladder fossa mass with little/no visible normal gallbladder and hilar biliary obstruction is highly suggestive of gallbladder carcinoma.
323
A 52-year-old man is investigated for weight loss and dyspepsia. At endoscopy, an adenocarcinoma of the posterior wall of the gastric body is visualised and confirmed on histology. A contrast-enhanced CT of the abdomen is performed (with intravenous Buscopan and water as oral contrast) to stage the tumour. The primary tumour is seen as focal gastric mucosal thickening with a small amount of free fluid noted in the left paracolic gutter and pelvis. An endoscopic ultrasound is performed and shows that the tumour extends beyond the serosal surface of the posterior gastric wall. Which structure is most at risk of direct invasion by this tumour? A Abdominal aorta B Left lobe of liver C Pancreas D Right diaphragmatic crus E Transverse colon
C The body and tail of the pancreas lie posterior to the stomach and can be infiltrated by direct extension of a gastric tumour.
324
A 22-year-old woman attends the Emergency Department with a 10-day history of vomiting and diarrhoea. The symptoms have worsened and are now associated with severe abdominal pain. Initial investigations reveal an elevated neutrophil count and CRP and she is treated with intravenous fluids and antiemetics. In view of increased pain and fever, a contrast-enhanced CT of the abdomen and pelvis is performed and shows that a segment of bowel is significandy thickened. The microbiology laboratory telephones the clinical team and states that Yersinia enterocolitica has been isolated from the patient's stool samples. Which segment of the bowel is likely to be abnormal? A Duodenum B Gastric antrum C Proximal jejenum D Sigmoid colon E Terminal ileum
E An increased number of thickened valvulae conniventes are seen in the distal ileum with nodular filling defects due to lymphoid hyperplasia.
325
A 74-year-old woman is referred to hospital by her GP as an emergency medical admission. The referral letter indicates that the patient is in residential care and has Alzheimer's disease. Her carers have noticed generalised malaise and significant weight loss over the past 6 weeks. A contrast-enhanced CT is performed and demonstrates multiple low attenuation liver metastases. These lesions contain foci of amorphous calcification and show rim enhancement in the portal venous phase. What is the most likely underlying malignancy in this patient? A Carcinoid tumour of ileum B Endometrial carcinoma C Mucinous adenocarcinoma of colon D Multifocal hepatocellular carcinoma E Papillary carcinoma of thyroid
C Calcification occurs in 2—3% of liver metastases. Mucinous adenocarcinoma of the GI tract is the most frequent underlying primary tumour.
326
A 79-year-old woman is admitted to hospital with a 2-day history of diarrhoea and abdominal pain. A contrast-enhanced CT of the abdomen demonstrates mucosal thickening of the proximal descending colon with a low attenuation ‘target sign’ appearance. The rectosigmoid and right hemicolon are normal in appearance. The patient is managed conservatively and the symptoms resolve. Six months later, a double contrast barium enema is performed and shows an irregular stricture of the descending colon with barium sacculation. What was the original diagnosis? A Acute diverticulitis B Giardiasis C Ischaemic colitis D Pseudomembranous colitis E Ulcerative colitis
C Stricture formation (with barium sacculation on double contrast enema) can occur in the splenic flexure due to fibrosis of the ischaemic bowel.
327
A 32-year-old man presents to his GP with increasing pain on swallowing solids and liquids. He has lost 15 kg in weight over the preceding 2 months. After a full history and examination, he is found to be HIV positive with a very low CD4 count. The GP refers him for a barium swallow examination and this demonstrates a single ulcer in the mid-oesophagus. The ulcer has a smooth margin, measures 4 cm in length and is oval in shape. There is no stricture identified. Which diagnosis is most likely? A Candida oesophagitis B CMV oesophagitis C Intramural pseudodiverticulosis D Oesophageal lymphoma E Squamous cell carcinoma of the oesophagus
B A single ‘giant’ ulcer in an immunocompromised host is highly suggestive of a viral oesophagitis (eg CMV or herpes simplex).
328
A 46-year-old woman was diagnosed with breast cancer 3 months ago. A recent abdominal ultrasound identified a solitary liver lesion and an MRI of the liver is performed. This demonstrates a 2.5-cm diameter mass in liver segment 8. This lesion has a well-defined, lobulated contour and yields high T2 signal. An extended echo time of 180 ms is used and the lesion remains of high T2 signal (greater than the spleen, less than CSF). What is the most likely diagnosis? A Breast cancer metastasis B Focal nodular hyperplasia C Haemangioma D Hepatic adenoma E Simple liver cyst
C Extended echo times will emphasise the high T2 signal intensity of liver haemangiomas, in comparison with surrounding structures.
329
A 27-yeat-old woman presents to the Emergency Department with a 3-day history of sharp pain in the left iliac fossa. A transvaginal pelvic ultrasound is performed and shows a 5-cm unilocular cyst in the left ovary. The radiologist then performs a transabdominal ultrasound to assess the kidneys. It is noted that the liver parenchyma extends significantly below the right costal margin and passes anterior to the right kidney. The liver texture appears uniformly normal. What is the most likely explanation for the appearance of the liver? A Biliary hamartoma B Fitz-Hugh-Curtis syndrome C Focal fatty infiltration D Focal nodular hyperplasia (FNH) E Riedel's lobe
E Riedel's lobe is described as a ‘tongue-like’ projection of the anterior tip of the right lobe of liver. It is a variant of normal and is more common in women.
330
A 56-year-old woman is found to have a 2.5-cm renal cell carcinoma in the upper pole of the right kidney. A contrast-enhanced CT of the chest and abdomen is performed and shows no evidence of local lymphadenopathy or distant metastases. The reporting radiologist notes a 2-cm cystic lesion in the pancreatic body. When assessing this cystic pancreatic lesion, which one of the following statements is true? A Eighty to 90% of symptomatic cystic lesions are pseudocysts. B Ninety to 95% of serous cystadenomas (microcystadenomas) contain calcification. C An asymptomatic solitary 8-mm simple cyst requires mandatory followup. D In a mucinous cystadenoma (macrocystadenoma), multiple small cysts typically measure up to 20 mm each. E The majority of mucinous cystadenomas occur in the pancreatic head.
A The majority of symptomatic cystic pancreatic lesions are pseudocysts and many will resolve spontaneously. Serous cystadenomas can occur anywhere in the pancreas and contain multiple small cysts measuring up to 20 mm each while up to one-third contain calcification. Ninety per cent of mucinous (macrocystic) cystadenomas occur in the pancreatic body and tail and the cysts typically measure greater than 20 mm. Long-term follow-up of cystic pancreatic lesions indicates that an asymptomatic simple cyst measuring less than 20 mm is unlikely to become clinically significant.
331
A 30-year-old man attends the Emergency Department with a 2-day history of abdominal pain and vomiting. On examination, he is afebrile with a firm mass palpable in the right lower quadrant of the abdomen. A supine abdominal radiograph is performed and demonstrates dilated loops of small bowel with a large soft tissue mass in the right lower quadrant. On ultrasound, the mass has a ‘pseudotumour’ appearance. What is the most likely diagnosis? A Colonic carcinoma B Gallstone ileus C Intussusception D Psoas abscess E Strangulated femoral hernia
C The ‘pseudotumour’, ‘pseudokidney’ and ‘target’ signs all describe the characteristic sonographic appearance of intussusception.
332
An 83-year-old man undergoes an emergency left hip hemiarthroplasty following a fractured neck of femur. Six days after surgery, he develops increasing abdominal distension with nausea and vomiting. An abdominal radiograph is performed and demonstrates dilatation of the ascending and transverse colon with the caecum measuring 7.0 cm in diameter. The clinical team believe that the patient may have colonic pseudo-obstruction and a single contrast (instant) enema is performed using water-soluble contrast. What are the likely findings in colonic pseudo-obstruction? A Extrinsic compression of the sigmoid colon B Long, irregular stricture of the sigmoid colon C Long, smooth stricture at the splenic flexure D No stricture demonstrated E Short ‘apple core’ stricture of the descending colon
D An instant enema can exclude mechanical obstruction in patients with colonic pseudo-obstruction.
333
A 49-year-old man is involved in a road traffic accident and sustains serious head and chest injuries. He is ventilated on the intensive care unit and his injuries are managed conservatively. Ten days later, he develops a temperature of 39.5°, becomes tachycardic and requires inotropic support to maintain his blood pressure. An abdominal ultrasound is performed and shows a cystic structure in the right upper quadrant measuring 1 2 x 8 cm in size. The mass has a 6-mm thick wall, contains a layer of echogenic material and is surrounded by a rim of fluid. What is the most likely diagnosis? A Acalculous cholecystitis B Acute cholangitis C Gallbladder haematoma D Traumatic hepatic arteiy pseudoaneurysm E Xanthogranulomatous cholecystitis
A Acalculous cholecystitis should always be considered in the seriously ill patient who develops unexplained sepsis.
334
A 48-year-old man presents with a painless swelling in the right scrotum. He has a past medical history of bilateral undescended testes and subsequent orchidopexy. On examination, there is a firm right testicular lump but no inguinal lymphadenopathy. On ultrasound, a well-defined, homogeneous hyporeflective mass was found within the right testicle. The right epididymis and contralateral testicle appeared normal. What is the most likely diagnosis? A Leukaemic testicular infiltrate B Testicular epidermoid cyst C Testicular metastasis D Testicular seminoma E Testicular teratoma
D The clinical history and ultrasound appearances are highly suggestive of testicular seminoma.
335
A 35-year-oid man is discovered to have a right testicular mass on ultrasound. Which additional ultrasound finding would suggest a diagnosis of teratoma rather than seminoma? A A testicular mass that contains areas of calcification B A testicular mass that demonstrates increased colour Doppler flow C A testicular mass that is homogeneously anechoic with posterior acoustic enhancement D A testicular mass that is hypoechoic compared with the surrounding testicular parenchyma E A testicular mass that has well-defined margins
A
336
A 32-year-old man received a cadaveric renal transplant 3 days ago. He now presents with increasing right iliac fossa pain and deteriorating renal function. On ultrasound, there is mild dilatation of the pelvicalyceal system with prominent renal pyramids. On Doppler ultrasound, colour Doppler flow is present within the renal artery and the interlobar arteries. The interlobar arterial waveform has tall systolic peaks with diastolic flow below the baseline. What is the most likely diagnosis? A Acute tubular necrosis B Acute rejection C Ar teriovenous fistula D Renal artery stenosis at the site of anastomosis E Renal vein thrombosis
E Diastolic flow reversal in the renal arcuate arteries on spectral Doppler ultrasound is a feature of renal vein thrombosis. Acute tubular necrosis (ATN) and acute rejection are difficult to distinguish on a single Doppler examination because they both cause an increase in the resistive index (> 0.8). Renal artery stenosis does not cause diastolic flow reversal and classically produces a slow rising parvus et tardus waveform. An arteriovenous fistula is usually the result of renal biopsies and presents as an area of turbulent high flow within the renal parenchyma.
337
A 40-year-old female diabetic patient has right loin pain, vomiting and a fever. An ultrasound examination is requested to exclude urinary obstruction. This demonstrates no evidence of upper tract dilatation, but features of acute pyelonephritis are present. What are the most likely sonographic findings within the right kidney? A Focal areas of reduced reflectivity in the renal parenchyma B Focal atrophy of segments of the right kidney C Increased echogenicity of the renal calyces D Enlarged right kidney and diffusely hyperechoic parenchyma E Shrunken right kidney and diffusely hyperechoic parenchyma
A
338
A 58-year-old woman suffers a left ureteric injury during a total abdominal hysterectomy. Postoperatively, she develops left loin pain and a fever, and ultrasound demonstrates a moderate left hydronephrosis. The clinical team are concerned that she has an infected, obstructed left kidney and request a nephrostomy. Which one of the following statements is correct regarding percutaneous nephrostomy? A A 4 French nephrostomy catheter is adequate to drain an infected collecting system likely to contain pus. B If the renal pelvis is punctured with the wire during a successful procedure, the patient will always require surgical repair. C It is best to dilate the tract 1 French size bigger than the size of the intended nephrostomy catheter (eg 8F for a 7F catheter). D It is best to directly puncture the renal pelvis. E It is usually best to aim to puncture an upper pole calyx.
C Puncture of posterior calyces in the mid- and lower poles is optimal. Upper pole puncture increases the risk of pneumothorax whilst direct puncture of the pelvis increases the risk of major vascular injury and persistent urinaty leak. If the renal pelvis is inadvertently punctured with the guidewire, once adequate drainage is obtained, this usually settles with observation
339
A 58-year-old man recently migrated to the UK from Kenya. He has been experiencing haematuria, weight loss and dysuria for several months. A series of imaging investigations are requested by the urologists and reveal evidence of renal tract TB. Which one of the following statements best describes the likely radiological findings in renal tract tuberculosis? A A chest radiograph is normal in 75—80% of cases. B Bladder calcification is more commonly seen than renal or ureteric calcification. C Free vesicoureteric reflux into a widely dilated upper renal tract is frequently seen. D Tramline calcification is seen within the seminal vesicles. E Findings usually present as bilateral renal tract disease.
C
340
A 43-year-old female diabetic patient with right-sided renal colic has a CT urogram (CTU). This demonstrates a calculus within the right renal pelvis and the right renal cortex is almost entirely replaced by a heterogeneous illdefined mass that extends to involve Gerota's fascia. This right renal mass contains rounded areas of low attenuation (—15 to —20 HU) which don't enhance postcontrast. On a subsequent MRI, the right renal mass appears heterogeneous with rounded areas of high signal on Tlw and low signal on STIR images. What is the most likely diagnosis? A Emphysematous pyelonephritis B Renal angiomyolipoma C Renal cell carcinoma D Staghorn calculus with coexistent malakoplakia E Xanthogranulomatous pyelonephritis.
E Xanthogranulomatous pyelonephritis (XGP) is a rare form of low-grade chronic renal infection, characterised by progressive destruction of the renal parenchyma. The histological hallmark is replacement of the renal parenchyma by lipid-laden foamy histiocytes. Women are predominantly affected (in a 3:1 ratio), usually in mid-life (50—60 years) and after a long history of recurrent urinary tract infection or urinary stones. Involvement of both kidneys is exceedingly rare. XGP is diffuse in 90% of cases, but focal XGP can simulate a renal tumour.
341
A 55-year-old HIV-positive man presents with macroscopic haematuria and right-sided renal colic. An IVU does not demonstrate any renal tract calcification, but there is a dense right nephrogram with no excretion of contrast on a delayed film. The urologist performs a retrograde ureteroscopy and retrieves a 9-mm right ureteric calculus. What is the likely composition of the calculus? A Calcium oxalate B Cysteine C Indinavir phosphate D Struvite E Uric acid
C
342
A 29-year-old man has an IVU performed following an episode of haematuria. This demonstrates complete right-sided ureteric duplication. Which one of the following statements is true? A If present, an ectopic ureterocoele is usually related to the lower moiety B The lower moiety ureter usually obstructs at the vesicoureteric junction. C The upper moiety calyces are prone to vesicoureteric reflux. D The upper moiety ureter is prone to ureteric obstruction. E The upper moiety ureter usually inserts into the bladder superior to the lower moiety ureter.
D
343
A patient with normal renal function and suspected right renal artery stenosis undergoes a dynamic angiotensin converting enzyme inhibitor (ACE1) MAG3 renogram. Which one of the following statements best describes the findings of the ACEI renogram in right renal artery stenosis? A After administration of the ACEI, there is increased blood flow to the right kidney. B On a time activity graph, the mean transit time of the right kidney is reduced following ACEI. C On a time activity graph, the time to peak of the right kidney is increased following ACEI. D The ACEI renogram will be normal because the renal function is preserved. E The ACEI renogram will show an increase in total and relative renal function of the right kidney when compared with the left kidney.
C
344
A 27-year-old man with membranous glomerulonephritis presents with a 1-day history of right-sided flank pain and haematuria. An abdominal radiograph did not reveal any renal calcification but his renal function has significantly deteriorated over the past 24 hours. On ultrasound there is a large, oedematous right kidney with loss of the corticomedullary differentiation. On a subsequent IVU, there is a faint nephrogram with absent pelvicalyceal filling after 15 minutes. What is the most likely diagnosis? A Acute hydronephrosis B Acute pyelonephritis C Acute renal infarction D Acute renal vein thrombosis E Chronic pyelonephritis
D
345
A 41-year-old woman is diagnosed with significant right renal arteiy stenosis and referred for angioplasty. Regarding this procedure, which one of the following statements is correct? A Angioplasty of ostial lesions has a poorer prognosis than angioplasty of more distal lesions. B Intra-arterial GTN to treat vasospasm is contraindicated during the procedure. C Stenoses due to fibromuscular dysplasia don't respond well to angioplasty D Stenoses due to fibromuscular dysplasia tend to involve the renal artery E The majority of renal artery stenoses are due to fibromuscular dysplasia.
A Ostial and proximal lesions are usually due to atherosclerotic disease. Ostial stenoses are due to aortic wall atheroma and are prone to elastic recoil; therefore they have poor results with angioplasty alone. As a result, many radiologists will opt for angioplasty and primary stenting in patients with ostial stenoses. Fibromuscular dysplasia typically affects the mid-distal renal artery and responds well to angioplasty alone.
346
An 81-year-old woman presents to the Emergency Department with sepsis and left iliac fossa pain. She has grossly elevated inflammatory markers and serum creatinine = 212jlmo!/L. A contrast-enhanced CT is requested for suspected acute diverticulitis, but the on-call radiology SpR is concerned about the possibility of contrast-mediated nephrotoxicity (CMN). Which one of the following statements is true regarding CMN? A Atrial fibrillation is an independent risk factor for developing CMN. B CMN is defined as renal impairment (an increase in serum creatinine by more than 50% or by 44 pmol/L above baseline) within 24 hours of contrast injection. C Low osmolar contrast media is more nephrotoxic than high osmolar contrast media, in patients with pre-existing renal impairment. D Pre-hydration with 1 mL/kg body weight/h of 0.9% NaCl for 4 hours prior to the contrast injection may increase the incidence of CMN. E Prophylactic haemodialysis in patients with renal impairment does not reduce the risk of CMN.
E CMN is defined as an increase in serum creatinine by more than 25% or by 44[lmol/L above baseline within 3 days of contrast administration. Prophylactic haemodialysis does not reduce the incidence of CMN but haemofiltration does.
347
A 35-year-old patient received a cadaveric renal transplant 5 days ago and now presents with worsening renal function and decreasing urine output. Which one of the following findings on a Tc-99m DTPA radionuclide scan would favour a diagnosis of acute tubular necrosis (ATN) over acute rejection? A Delayed renal excretion B Elevated resistive index greater than 0.7 C Increased renal perfusion after administration of an ACEI (eg Captopril) D Poor/impaired graft perfusion E Preserved renal transplant perfusion
E ATN is an early complication in cadaveric allografts and frequently resolves spontaneously in 1—3 weeks. The radionuclide imaging findings of ATN are of preserved perfusion but poor renal function and urine excretion. In acute rejection however, there is both impaired renal function and reduced perfusion on radionuclide imaging.
348
A 34-year-old man is knocked off his bicycle by a car and presents to the Emergency Department with bruising over the right flank and gross haematuria. The A&E SpR suspects renal injury and requests a CT abdomen. Which one of the following findings is most likely to be seen in uncomplicated renal contusion (Grade 1 renal injury)? A Ill-defined areas of low attenuation with irregular margins B Subcapsular high attenuation collection C Wedge-shaped areas of high attenuation, typically involving the renal D Well-defined areas of low attenuation within the renal parenchyma E Urinoma formation
A
349
A 68-year-old man is involved in a traffic accident and sustains a pelvic fracture, head and limb injuries. Attempted urethral catheterisation in the Emergency Department is unsuccessful and a cystourethrogram is requested to exclude urethral injuries. Regarding urethral injuries, which one of the following statements is correct? A Anterior urethral injury is more commonly due to iatrogenic or penetrating trauma than to blunt traiima. B Cystography should precede a retrograde urethrogram in a patient with suspected urethral injury. C In men, on digital rectal examination the prostate is lower than normal in patients with urethral trauma. D Urethral injuries occur in 50% of major pelvic fractures. E Urethral injury due to blunt trauma more commonly affects the penile urethra.
A
350
A 54-year-old man attends the Radiology Department for an ascending urethrogram. He has a past histoiy of previous urethral stricture and urethroplasty at a different hospital and has now developed recurrent lower urinary tract symptoms. Which one of the following statements regarding urethral strictures is true? A Inflammatory strictures most commonly occur within the penile urethra, the site of the periurethral glands. B Most inflammatory strictures occur in the prostatic urethra. C The urethral strictures due to transurethral retrograde prostatectomy (TURP) are typically long segment, irregular strictures. D Traumatic strictures usually take longer to develop than inflammatory strictures. E Ultrasound is more accurate than conventional urethrography in the assessment of urethral strictures.
E
351
A 42-year-old man is referred for investigation of painless microscopic haematuria. An IVU is performed and demonstrates bilateral small areas of calcification within the kidneys on the control image. On the 5-min postcontrast IVU film, the calcification appears to lie within the collecting system. On ultrasound, there are numerous small hyperechoic rounded areas within the medullary pyramids, many of which cast an acoustic shadow. What is the most likely diagnosis? A Adult polycystic kidney disease B Hyperparathyroidism C Medullary sponge kidney D Primary hyperoxaluria E Sarcoidosis
C
352
A 32-year-old man involved in a high-speed traffic accident is found to have blood at the urethral meatus and a high riding prostate during the secondary clinical survey. The examining doctor suspects a urethral injury. Which part of the urethra is most likely to be involved? A Bulbar urethra B Membranous urethra C Penile urethra D Penoscrotal urethra E Prostatic urethra
B
353
You are the radiologist reviewing the mammograms of a 56-year-old woman. When compared with her previous mammograms, areas of calcification previously seen within the left upper outer quadrant have now disappeared. Which of the following is not a possible explanation? A Breast surgery B Chemotherapy C Postmenopausal changes D Radiotherapy E Spontaneous resolution
C
354
Which one of the following statements best describes the CT appearances of a renal oncocytoma (tubular adenoma)? A It appears as a small, ill-defined renal mass in the majority of cases. B It is bilateral in 60-80% of cases. C It characteristically consists of multiple renal lesions. D CT shows punctuate calcification in the majority of patients. E Low attenuation (—100 to —50 HU) areas within a large lesion are consistent with an oncocytoma.
E Large lesions can extend into and engulf the perinephric fat, and can therefore be mistaken for angiomyolipomas (due to fat content).
355
A 42-year-old man with known Wegener's granulomatosis develops haematuria. He has an abdominal ultrasound which reveals small, smooth kidneys with diffuse thinning of the renal parenchyma. The pelvicalyceai systems appear normal but there is an increased amount of renal sinus fat. What is the most likely diagnosis? A Bilateral vesicoureteric reflux B Chronic glomerulonephritis C Medullary sponge kidney D Pyelonephritis E Renal tuberculosis
B
356
A 22-year-old pregnant woman (30 weeks' gestation) presents with right flank pain. She has an abdominal ultrasound which shows dilatation of the right pelvicalyceai system. Which one of the following additional findings would suggest a diagnosis of mechanical ureteric obstruction rather than pregnancy-related dilatation? A An elevated resistive index (RI) B Decreased corticomedullary differentiation C Hyperechoic renal parenchyma D Renal parenchymal thinning E Ureteric and pelvicalyceai dilatation
A Mechanical obstruction is associated with elevation of the RI.
357
A 35-year-old woman presents with a painless lump in the outer upper quadrant of her left breast. She is referred for an ultrasound examination of the left breast. Which of the following ultrasound findings would suggest a malignant rather than a benign breast mass? A A larger transverse than anterior-to-posterior diameter B Ill-defined echogenic halo around the lesion C Less than 1 cm in greatest diameter D Posterior acoustic enhancement E Uniform hyperechogenicity
B On ultrasound, breast carcinomas are generally ill-defined, hypoechoic masses which can have a surrounding echogenic halo. They also tend to have larger anterior-to-posterior than transverse diameter.
358
Which one of the following statements best describes the course of the normal ureter within the pelvis? A Anterior to the inferior pubic ramus, the ureter runs posteromedially to enter the urinary bladder. B In females, the ureter lies within the broad ligament where it is intraperitoneal for a short portion of its length and runs inferomedially to enter the urinary bladder. C In males, the ureter runs anterior to the cremasteric artery and turns medially to enter the urinary bladder. D In the region of the ischial spine, the ureter turns medially, anteriorly and inferiorly to enter the bladder. E The ureter enters the pelvis by crossing the bifurcation of the common iliac artery and runs medially to enter the urinary bladder.
D
359
An immunosuppressed 24-year-old man presents with left renal colic. He is referred for an IVU. The control film shows a gas containing, round lamellated mass within the urinary bladder. Postcontrast, there are multiple filling defects within the urinary bladder. What is the most likely cause of these appearances? A Blood clot B Bladder calculi C Cystitis D Fungal ball E Schistosomiasis
D
360
A 67-year-old man with a history of transitional cell carcinoma of the bladder now presents with several episodes of haematuria. A cystoscopy and biopsies performed 4 weeks ago were negative and he now attends for an MRI pelvis. On sagittal T2w images, there is an area of thickening in the bladder wall with low signal change in the surrounding perivesical fat. Which one of the following findings would suggest a diagnosis of postbiopsy change, rather than recurrent transitional cell carcinoma? A On dynamic contrast-enhanced Tlw images, the area of low signal shows early and avid contrast enhancement. B On fat-suppressed proton density sequences, this area returns a high signal. C On post-gadolinium Tlw images, inflammatory change avidly enhances whilst transitional cell carcinoma does not. D The area shows delayed enhancement on Tlw post-gadolinium images. E There is signal loss on gradient echo images, due to tissue inhomogeneity.
D On dynamic contrast-enhanced MRI, bladder cancer shows earlier enhancement than post-biopsy/surgical tissue and is therefore helpful in distinguishing between bladder cancer and post biopsy change.
361
A 53-year-old woman is recalled to the screening breast outpatient clinic as her initial mammograms have revealed an area of suspected microcalcification. Compared with the standard mammographic projections, which one of the following statements best describes the technique needed to provide magnification views? A A double-coated film is necessary to avoid parallax and crossover. B A molybdenum target is used because it provides a low energy spectrum. C A smaller focal spot of 0.1 mm is used. D An air gap is avoided as it reduces signal to noise ratio. E Tube current should be as high as possible to keep exposure times short.
C To provide magnification views, a focal spot of 0.1 mm should be used (smaller than the 0.3 mm focal spot used for standard mammographic projections) with an air gap of 1 5—30 cm. Options D & E are correct regarding mammography in general and hold true for both standard and magnification views.
362
Which of the following best describes the radiological findings of urinary tract malakoplakia? A Intramural bladder wall gas B Multiple filling defects in the pelvicalyceal systems and proximal ureters on IVU, with sparing of the urinary bladder C Multiple small oval filling defects at the bladder base D Plaque-like thickening of the pelvicalyceal urothelium E Tram-track calcification within the bladder wall
C Malakoplakia is a benign, inflammatory condition that predominantly affects the bladder and lower urinary tract.
363
A 70-year-old man with prostate cancer has an MRI examination to locally stage the disease. In which part of the prostate gland is a carcinoma most likely A Central zone B Peripheral zone C Peri-urethral zone D Transitional zone E Within the verumontanum
B Seventy per cent of prostate cancers arise from the peripheral zone.
364
A 53-year-old man has an MRI of his pelvis as a staging investigation for bladder cancer. The request card also states that the prostate is mildly enlarged on digital rectal examination and the serum prostate specific antigen (PSA) level is borderline elevated. The reporting radiologist reviews the prostate in detail. Which one of the following statements best describes the MRI findings of a normal prostate gland? A On Tlw images, the central zone is of higher signal intensity than the peripheral zone. B On Tlw images, the central zone is of lower signal intensity than the peripheral zone. C On T2w images, the peripheral zone is of lower signal intensity than the central and transitional zones D The peripheral zone is of higher signal intensity than the central zone on T2w images. E The seminal vesicles are hypointense on T2w images.
D Zonal anatomy of the prostate is best demonstrated on T2w sequences.
365
Regarding the radiological anatomy of the normal prostate gland, which of the following statements is correct? A The anterior fibromuscular band separates the prostate from the rectum, B The central zone atrophies with advancing age. C The neurovascular bundles lie anterolateral to the prostate gland. D The prostate gland is a flattened conical structure with its apex pointed superiorly. E The zonal anatomy of the adult prostate gland is seen on transabdominal ultrasound.
B The central zone atrophies with age whilst the transitional zone enlarges by developing benign prostatic hypertrophy (BPH).
366
A 64-year-old woman with bladder cancer is discussed in the urology multidisciplinary meeting. A recent CT suggested the possibility of tumour extension into the perivesical fat and the urologists request an MRI pelvis to assess whether the bladder wall is breached (as this would upstage the tumour). Which one of the following MRI sequences is best for assessing bladder wall integrity? A Gradient echo sequences with a long flip angle B Postcontrast STIR C Proton density fat saturation D Tlw (precontrast) E T2w
E Postcontrast STIR sequences are pointless as gadolinium shortens T1 and STIR removes signal from tissue with a short Tl.
367
A final year medical student attends a breast screening outpatient clinic as part of her clinical attachment. She wishes to know more about the UK NHS Breast Screening programme and asks several questions of the radiology SpR in the breast clinic. Which one of the following statements is true? A Double reading of all screening mammograms must be performed. B Incident screens may be performed using only the MLO view. C The acceptance rate of women invited to breast screening is over 90%. D Up to 20% of women are recalled from screening for further-assessment. E Women over 70 years of age may stay in the Breast Screening Programme by choice.
E Women between the ages of 50 and 70 are invited for screening every 3 years. Women over 70 are encouraged to attend by self-referral but are not invited.
368
A 33-year-old man presents with left renal colic. An abdominal radiograph is normal but a subsequent CT urogram demonstrates an obstructing, opaque 10-mm distal left ureteric calculus. What is the renal calculus most likely to be composed of? A Calcium oxalate B Calcium phosphate C Cysteine D Pure matrix E Uric acid
E Uric acid stones are not visible on plain radiographs but are seen on CT.
369
A 74-year-old man with increased urinary frequency and hesitancy is found to have an enlarged prostate on digital rectal examination. He is referred for a TRUS and biopsy. Which one of the following statements best describes the TRUS findings of benign prostatic hypertrophy (BPH)? A Dense echogenic foci are seen at the margin of the peripheral and transitional zones. B The central zone is enlarged. C The peripheral zone is enlarged and appears homogeneously hypoechoic. D The peripheral zone is enlarged and is of mixed echogenicity. E The transitional zone is enlarged.
E The central zone atrophies with age while the transitional zone increases in size as it develops BPH. Peripheral zone enlargement is not a feature of BPH.
370
A 64-year-old woman presents with bloating and vague pelvic pain and is referred for a pelvic ultrasound. On transabdominal ultrasound, she is found to have a large right adnexal mass. Which one of the following sonographic findings would indicate that this mass is more likely to be malignant than benign? A Doppler waveform with a high resistive index (> 0.8) B Homogeneously hypoechoic mass with posterior acoustic enhancement C Multiple septations that are approximately 1 mm thick D Papillary projections E Size > 4 cm
C
371
A 67-year-old man attends the Emergency Department with acute abdminal pain. A CT abdomen is performed and demonstrates an uncomplicated acute appendicitis. The reporting radiologist notes an incidental finding of a bulky prostate gland. On axial CT images, which one of the following CT findings is an unequivocal feature of prostatic enlargement? A The diaphragmatic urethra is dilated. (Dilatation) B The prostate is identified 1 cm above the symphysis pubis. C The prostate is identified 1 cm or less below the symphysis pubis. D The prostate is identified 2—3 cm above the symphysis pubis. E The prostate is identified less than 2 cm from the posterior aspect of the symphysis pubis.
D On CT, as a general rule, the prostate is not considered to be enlarged if an image obtained 1 cm above the symphysis does not include the prostate. Unequivocal enlargement of the prostate is diagnosed if the prostate is seen in images 2—3 cm or more above the symphysis pubis.
372
A 65-year-old man is referred by his GP ro the urologists for investigation of chronic macroscopic haematuria. The patient is a teacher and has recently returned to the UI< after 15 years working in Malawi. He suffered an episode of dysuria and haematuria several years ago and a local doctor diagnosed schistosomiasis. You are asked to supervise and report a CT urogram for this patient. Which one of the following statements is true regarding the radiological features of schistosomiasis? A Cortical nephrocalcinosis is characteristic. B In late stage disease, a 1- to 3-mm band of calcification surrounding the bladder wall is seen. C In the late stage, the bladder is dilated, thin walled and calcified, giving an egg-shell appearance. D Narrowed ‘pipe-stem’ ureters are evident. E ‘Tram-line’ calcification within the seminal vesicles is found in the majority of cases.
B In schistosomiasis, the ureters become grossly dilated and tortuous and may have multiple filling defects due to either granulomata or ureteritis cystica. The urinary bladder becomes small and fibrosed.
373
A 25-year-old man is seen in the Emergency Department following a fall from a height of 20 feet. Plain radiographs reveal a fractured pelvis and a cystogram is performed by the on-call radiology SpR. This demonstrates contrast extravasation from the urinary bladder in an irregular streaky fashion. Which one of the following is the most likely diagnosis? A Bladder wall contusion B Extraperitoneal bladder rupture C Intraperitoneal bladder rupture D Mixed intra- and extraperitoneal bladder rupture E Subserosal bladder rupture
B Extraperitoneal bladder rupture is more common (50—85%) than either intraperitoneal (15—45%) or mixed intra- and extraperitoneal bladder rupture (0—12%). Extraperitoneal bladder rupture is frequently associated (89—100%) with pelvic fractures.
374
A 28-year-old woman has a strong family history of breast cancer and is referred for an MRI examination of the breasts. Regarding MRI of the breast, which one of the following statements is correct? A Breast MRI shotdd be performed during the middle of the menstrual cycle to improve sensitivity. B Malignant lesions tend to show poor enhancement following intravenous contrast, compared with surrounding breast tissue. C MRI has a high sensitivity and specificity for the detection of invasive breast cancer. D Post radiotherapy, abnormal enhancement patterns return to normal within 3—6 months. E The patient is imaged in a supine position with the breasts placed in a dedicated breast coil to improve signal to noise ratio.
D Malignant breast lesions enhance postcontrast; however, normal hormonally active breast tissue can also enhance, particularly during the middle of the menstrual cycle (6th—17th days). In younger patients it may be helpful to repeat the scan earlier or later in the menstrual cycle to improve specificity.
375
Which one of the following statements best describes the characteristic radiological features of retroperitoneal fibrosis? A A plaque-like mass that encases the aorta and displaces it laterally, most commonly to the left B A plaque-like mass that displaces the kidneys and ureters laterally at the Ll-2 level C A plaque-like mass that displaces the aorta and iliac arteries anteriorly D A plaque-like mass that narrows and displaces the ureters laterally at the L4-5 level E A plaque-like mass that narrows and medially displaces the ureters at the L4-5 level
E
376
A mother is concerned about the shape of her 2-year-old son's head. The GP agrees that it appears elongated and he is referred for skull radiographs and a CT head to look for evidence of craniosynostosis. Which one of the following statements is true regarding congenital skull abnormalities? A Apert’s syndrome is associated with sagittal synostosis. B Brachycephaly is associated with a higher incidence of neurological abnormalities compared with scaphocephaly. C Crouzon syndrome affects only the coronal sutures. D Sagittal synostosis is often seen with hydrocephalus. E Synostosis of the lamboid suture is more common than the sagittal
B The sagittal suture is the most commonly affected in primary craniosynostosis. Involvement of the coronal suture (in brachycephaly) is often associated with clinical syndromes.
377
A 2-year-old boy presents with repeated urinary tract infections. Which one of the following investigations would be the most appropriate investigation? A Intravenous urogram at 6 weeks B Micturating cystogram at 6 weeks C Tc-99m DMSA renal scintigraphy at 4 months D Tc-99m DTPA indirect cystography at 4 months E An ultrasound at 4 months
C A micturating cystogram is indicated if the infant is under the age of 6 months, but is not a requirement if the infant is > 6 months unless there is an abnormal US or DMSA.
378
A newborn delivered by caesarean section shows signs of respiratoiy distresssoon after birth. A chest radiograph is performed. Which one of the following features favours the diagnosis of transient tachypnoea of the newborn (TTN)? A A ground glass appearance throughout both lungs B Hyper inflated lungs C Loss of lung volume D Radiographic resolution after 2 weeks E The presence of a pleural effusion
B The radiographic features of TTN include hyperaeration of the lungs with an increase in pulmonary interstitial markings. Resolution should be seen clinically and radiographically within 48—72 hours.
379
Aii 18-month-old infant presents with failure to thrive and anorexia. On examination an upper abdominal mass is palpable. Blood tests reveal an iron deficiency anaemia and a raised alpha fetal protein. The clinicians wish to exclude a hepatoblastoma. When imaging this child, which one of the following statements holds true? A Angiography should be performed to establish vascular anatomy. B Calcification will be seen on a plain abdominal radiograph in 5—10% of C Following contrast administration, there is centripetal and heterogeneous enhancement on CT. D Hepatoblastoma demonstrates increased activity during the delayed phase of Tc-99m sulphur colloid scintigraphy. E Ultrasound characteristically demonstrates a focal echogenic mass.
C Calcification is common and present in 50% of plain abdominal radiographs. Hepatoblastoma tends not to be multifocal; this would make hepatocellular carcinoma a more likely diagnosis.
380
A 4-year-old child presents with upper back pain. General examination reveals hepatomegaly and blood tests demonstrate an iron deficiency anaemia. The child's chest radiograph demonstrates an abnormal mediastinal contour and subsequent CT confirms an 8-cm posterior mediastinal mass which contains calcification. The lungs are clear. Which one of the following is most likely the diagnosis? A Extramedullary haemopoiesis B Lymphoma C Neuroblastoma D Neurofibroma E Teratoma
C If a paediatric posterior mediastinal mass contains calcification, it is most likely to be a sympathetic chain tumour.
381
A baby boy is born prematurely at 30 weeks gestation. Cranial ultrasound demonstrates bilateral multiseptate cystic lesions within the frontal lobe white matter with associated ex vacuo dilatation of the ventricles. Which of the following is the most likely diagnosis? A. Periventricular leucomalacia. B. Porencephaly. C. Supratentorial arachnoid cysts. D. Vein of Galen malformation. E. Subependymal cysts.
A. Periventricular leucomalacia. This refers to white matter necrosis, typically involving the centrum semiovale, and is seen in premature infants. This results from hypoxic-ischaemic injury at the watershed areas, which in premature infants are present in a periventricular location. Porencephaly refers to an area of encephalomalacia, which may or may not communicate with the ventricular system and develops postnatally or in the third trimester. This is the end result of a destructive process, such as an intraparenchymal haemorrhage. Sylvian fi ssure cysts are the most common site for supratentorial arachnoid cysts. Vein of Galen malformations occur in the midline and exhibit Doppler fl ow. Subependymal cysts are detected in the caudothalamic groove.
382
A 10-year-old girl presents with pain in the left hip which she fi rst noticed when playing sports at school. Apart from pain on movement and several prominent café-au-lait spots, there is nothing else to fi nd on examination. The patient is apyrexic and her WCC and CRP are normal. A plain fi lm of the pelvis reveals a lucent lesion in the proximal femoral metaphysis. The margins of the lesion are well defi ned and the metaphysis is expanded with adjacent cortical thinning. The lesion extends as far as, but does not involve, the physis. There is GGO in the centre of the lesion. There is no periosteal reaction. What is the most likely diagnosis? A. Osteomyelitis. B. Chondroblastoma. C. Aneurysmal bone cyst. D. GCT. E. Fibrous dysplasia.
E. Fibrous dysplasia. The clue here is the café-au-lait spots, which are seen in McCune–Albright syndrome (unilateral polyostotic fi brous dysplasia, precocious puberty, and café-au-lait spots). Osteomyelitis is unlikely because of the normal infl ammatory markers, lack of systemic symptoms, and lack of periosteal reaction. Aneurysmal bone cyst and GCTs are possibilities (the former may even complicate fi brous dysplasia). Chondroblastoma is an epiphyseal lesion.
383
A 2-day-old term neonate with an antenatal history of enlarged kidneys undergoes ultrasound of the renal tracts, which reveals bilateral enlarged and diffusely echogenic kidneys, with loss of cortico-medullary differentiation. Further assessment with a high-resolution linear probe reveals multiple small radially oriented cysts. Which of the following statements regarding this condition is false? A. It is associated with congenital hepatic fi brosis. B. The severities of renal and hepatic involvement are inversely proportional to each other. C. Severe renal compromise is the immediate cause of death in the perinatal group. D. Potter facies may be found in severely affected neonates. E. It is associated with clubfoot deformity.
C. Severe renal compromise is the immediate cause of death in the perinatal group. The incidence of autosomal recessive polycystic kidney disease (ARPKD) is approximately 1:20,000 births. It is a disease of tubular ectasia and fi brosis that results in bilateral enlarged kidneys, with loss of cortico-medullary differentiation and multiple small radially arranged cysts. ARPKD is associated with congenital hepatic fi brosis, the severity of which is inversely proportional to the renal abnormality. Four distinct groups of ARPKD are recognized based on age at presentation: perinatal, neonatal, infantile, and juvenile. The most severe renal involvement is seen in the perinatal group. In this group, severe renal impairment results in reduced urine output, oligohydramnios, and pulmonary hypoplasia. Severe respiratory compromise is the immediate cause of death in these patients. Oligohydramnios is also associated with Potter facies (low set and fl attened ears, short and snubbed nose, deep eye creases, and micrognathia) and clubfoot deformity.
384
A 2-year-old girl is investigated for slow motor development via MRI. Which of the following radiological features would suggest a diagnosis of Dandy– Walker malformation, as opposed to Dandy–Walker variant? A. Cerebellar dysgenesis. B. Enlargement of the posterior fossa. C. Agenesis of the corpus callosum. D. Holoprosencephaly. E. Cystic dilatation of the fourth ventricle.
7. B. Enlargement of the posterior fossa. Dandy–Walker variant is more common, accounting for a third of all posterior fossa malformations, but less severe than the malformation. Enlargement of the posterior fossa is not a feature. Cystic dilatation of the fourth ventricle with vermian dysgenesis is characteristic of both. Associated CNS anomalies, usually of the midline, are also seen, as is ventriculomegaly, although both are more common with Dandy–Walker malformation.
385
A 7-year-old girl presents with a history of continuous dribbling incontinence. On imaging she is found to have bilateral duplex kidney, complete ureteral duplication, and ectopic ureter. Which of the following statements regarding ectopic ureters and ureteral duplication is true? A. Ectopic insertion is more commonly associated with solitary ureter than complete ureteral duplication. B. C. Urinary incontinence with ectopic ureter is more common in boys. In complete ureteral duplication, the ureter of the upper pole moiety inserts into the bladder superior to the ureter of lower pole moiety. D. The upper moiety ureter is associated with ureterocele whereas the lower moiety ureter is associated with vesicoureteric refl ux. E. The obstructed lower moiety may not be visualized on IVU.
D. The upper moiety ureter is associated with ureterocele whereas the lower moiety ureter is associated with vesicoureteric refl ux. Ectopic ureter results when the ureteral bud fails to separate from the Wolffi an duct and as a consequence is carried more caudally than normal. In females, the ectopic ureter can insert distal to the external sphincter, resulting in incontinence. Approximately 70% of ectopic insertion is associated with complete ureteral duplication. According to the Weigert–Meyer rule of complete ureteral duplication, the ureteric orifi ce of the upper moiety inserts into the bladder medial and inferior to the lower moiety. The ureter draining the Upper moiety is associated with Ureterocele and Obstruction (note the vowels), whereas the lower pole moiety is associated with vesico-ureteric refl ux. The obstructed upper moiety may not be visualized on IVU. The absence of upper pole calyx and displaced (‘drooping lily’) lower moiety calyces help in making the diagnosis.
386
A 14-year-old girl has an episode of pancreatitis. An MRCP examination is subsequently performed to assess for any biliary disease or pancreatic duct anomaly. You notice failure of fusion of the ventral and dorsal ducts, and suspect pancreas divisum. What other appropriate fi nding do you notice on the scan? A. Longer dorsal duct draining via major papilla/shorter ventral duct draining via minor B. C. D. E. papilla. Longer dorsal duct draining via minor papilla/shorter ventral duct draining via major papilla. Longer dorsal duct and shorter ventral duct both draining via the major papilla. Shorter dorsal duct draining via major papilla/longer ventral duct draining via minor papilla. Shorter dorsal duct draining via minor papilla/longer ventral duct draining via major papilla. F. Shorter dorsal duct and longer ventral duct both draining via the minor papilla.
B. Longer dorsal duct draining via the minor papilla/shorter ventral duct draining via the major papilla. Pancreas divisum is the most common congenital anomaly of the pancreatic duct and has a prevalence of 4–10% of the general population. Although it is usually asymptomatic and an incidental fi nding, it can lead to recurrent episodes of pancreatitis in children and adults. By defi nition, the dorsal and ventral ducts fail to fuse during embryological development, which results in the following features seen at MRCP: (a) a prominent dorsal pancreatic duct, which drains directly into the minor papilla, and (b) a ventral duct, which does not communicate with the dorsal duct, but joins with the distal bile duct to enter the major papilla (ampulla of Vater). Typically, the ventral duct is short and narrow, while the dorsal duct normally has a larger caliber.
387
A 5-year-old boy is admitted for investigation of headache and vomiting. Unenhanced CT demonstrates a hyperdense mass centred on the cerebellar vermis and effacing the fourth ventricle. Homogenous enhancement is demonstrated on contrast administration. What is the most likely diagnosis? A. Ependymoma. B. Pilocytic astrocytoma. C. Haemangioblastoma. D. Brainstem glioma. E. Medulloblastoma.
. E. Medulloblastoma. This is the most common malignant posterior fossa tumour in children, generally occurring before 10 years of age. The vast majority (85%) arise in the cerebellar vermis. They are hyperdense due to their high cellular content. Calcifi cation is uncommon, occurring in up to 20% of cases. Pilocytic astrocytoma typically present as a cystic mass with an enhancing nodule. The most common location is the cerebellum, but they usually occur in other sites when associated with neurofi bromatosis type 1. Haemangioblastomas are rare in children and even in the setting of von Hippel–Lindau syndrome typically manifest in early adulthood. Ependymomas are usually more heterogenous owing to calcifi cation, cystic change, and haemorrhage. The tumour arises from ependymal cells that line the ventricular system and central canal of the spinal cord. Brainstem gliomas are hypodense on CT and may show exophytic growth into the adjacent cisternal spaces.
388
A child is diagnosed with neuroblastoma. He is referred for staging and you are asked to advise on the standard radiological investigation of bony metastases. What do you advise? A. Whole body MRI. B. Whole body 18-FDG PET-CT. C. 123I-metaiodobenzylguanidine (MIBG) scan. D. 99mTc methylene-diphosphonate (MDP) isotope bone scan. E. MIBG and isotope bone scan.
C. 123I-metaiodobenzylguanidine (MIBG) scan. Owing to the high specifi city and sensitivity in neuroblastoma, MIBG imaging has superseded the use of 99mTc bone scans for the detection of skeletal metastases in the majority of children with neuroblastoma, which take up the tracer in >90% of cases, and has been recommended by the last international consensus conference as a standard element of staging and response evaluation. False-negative scans may be observed in approximately 10% of neuroblastomas that do not concentrate MIBG. In addition, very small amounts of bone marrow tumour will often not be detected and therefore the MIBG scan must be supplemented with bilateral bone marrow biopsy. For those patients whose tumours are negative for MIBG uptake at diagnosis, the 99mTc MDP bone scan is the standard test recommended to evaluate skeletal metastases. However, the low specifi city of this test and the diffi culty in interpreting uptake in young children with actively growing bones make investigation of alternative methods preferable. Whole-body MRI is also a sensitive test for neuroblastoma tumours, including bone and bone marrow metastases, although the specifi city is much lower than for MIBG. MRI and CT are appropriate in the staging of neuroblastoma, but for the assessment of local invasion rather than bony involvement in particular. There currently is no consensus about the optimal imaging modality for assessing local disease. Both MRI and CT are routinely used, depending on local availability and the radiologist’s preference. 18-FDG PET-CT imaging is not as sensitive as MIBG imaging for bony metastases, although it may be more sensitive for small soft tissue tumours and nodal metastases. The use of PET-CT is not at present well defi ned and delivers a high radiation dose.
389
A cardiac MRI is being carried out on an infant for a conotruncal rotational abnormality. It is clear that this infant has 150 clockwise rotation of the great vessels. What conotruncal rotation abnormality does this infant have? A. Normal rotation. B. Situs inversus. C. L-transposition. D. D-transposition. E. Double-outlet right ventricle.
20. B. Situs inversus. The primitive truncus is a midline structure that persists to a degree in truncus arteriosus abnormality. During normal development the truncus divides into the aortic and pulmonary trunks, which then undergo 150° anticlockwise rotation. In situs inversus there is 150° clockwise rotation. Transposition of the great arteries (TGA) is characterized by 30° rotation, anticlockwise in D-TGA and clockwise in L-TGA. Double-outlet right ventricle displays 90° anticlockwise rotation.
390
A 12-year-old boy is investigated via MRI brain for headache, nystagmus, and ataxia. Which of the following radiological fi ndings would suggest a diagnosis of Chiari I malformation as opposed to Chiari II? A. Lacunar skull. B. Myelomeningocoele. C. Elongation of the fourth ventricle. D. Caudal displacement of the cerebellar tonsils. E. Cervicomedullary kinking.
D Caudal displacement of the cerebellar tonsils. Chiari II is seen in all patients with open spinal dysraphisms, such myelomeningocoele. Lacunar skull (luckenshadel) is also associated with Chiari II. Cervicomedullary kinking is common to both, although more so with Chiari II. Caudal displacement of the cerebellar tonsils is a feature of Chiari I, whereas in Chiari II the vermis herniates into the foramen magnum and the tonsils are lateral to the medulla.
391
A 12–year-old who is a keen athlete presents with left groin pain. A plain fi lm of the pelvis reveals avulsion of the apophysis of the left ischial tuberosity. Which muscle attachment has he injured? A. Sartorius. B. Hamstrings. C. Adductors. D. Rectus femoris. E. Iliopsoas.
B. Hamstrings. Avulsion fractures in the pelvis are generally uncommon injuries and are seen almost exclusively in adolescent athletes. They occur at the apophyses, which while growing are more prone to injury than the adjacent tendons. The hamstrings attach at the ischial tuberosity. Sartorius attaches at the anterior superior iliac spine, rectus femoris at the anterior inferior iliac spine, the adductors at the symphysis pubis, and iliopsoas at the lesser trochanter. Care should be taken not to mistake an old avulsion, which can produce irregularity, marked periostitis, and adjacent soft-tissue mineralization, for a more sinister lesion.
392
A 6-day-old neonate presents with persistent vomiting. A plain x-ray of abdomen shows a dilated stomach and proximal duodenum, suggesting a high-grade duodenal obstruction. A subsequent upper GI contrast study confi rms obstruction in the second part of the duodenum with a ‘windsock’ type deformity evident. What is the most likely diagnosis? A. Duodenal atresia. B. Midgut volvulus. C. Annular pancreas. D. Preduodenal portal vein. E. Duodenal web.
E. Duodenal web. A duodenal web is classically associated with the ‘windsock sign’ seen on an upper GI contrast study. Over time, the web or diaphragm passively elongates as a result of continual peristalsis, to form the windsock confi guration of an intraluminal duodenal diverticulum. Duodenal atresia is commonly associated with the ‘double bubble’ appearance on AXR. The atresia is most commonly just distal to the ampulla of Vater. Associated anomalies are common, such as congenital heart disease, Down’s syndrome, and malrotation. Midgut volvulus is usually a complication of malrotation. The AXR may be normal, gasless, or show signs of duodenal/high small bowel obstruction. The upper GI contrast study shows an abnormal position of the duodenal-jejunal fl exure (to the right of midline) or duodenal obstruction in severe cases. More often the volvulus is intermittent, when the ‘corkscrew’ appearance is classical, due to clockwise twisting of the jejunum around the superior mesenteric artery. Annular pancreas and preduodenal portal vein are exceedingly rare. In preduodenal portal vein (persistent left vitelline vein), the portal vein lies in an abnormal position anterior to the duodenum and may cause compression. However, in this entity, the primary obstruction is more usually due to an associated obstructing duodenal lesion, such as an intraluminal membrane or web, and not to the abnormal position of the vein.
393
A male neonate born at 26 weeks gestation is currently being treated in your neonatal ICU. The patient’s mother received corticosteroids prior to delivery and prophylactic surfactant administration as per your department’s standard practice. The CXR was clear for the fi rst 7 days. Despite this the child developed streaky perihilar granular opacities and respiratory diffi culties. Further surfactant administration has been carried out, but the CXR carried out today (day 28 postpartum) shows small streaky linear densities along with cystic bubbly lucencies, which have been becoming increasingly prominent over the last 7 days and are distributed in an irregular pattern bilaterally. What is the most likely explanation for this appearance? A. Surfactant defi ciency. B. Meconium aspiration. C. BPD. D. Beta haemolytic streptococcal pneumonia. E. PIE.
C. BPD. Whilst surfactant defi ciency is undoubtedly a feature of this case, the evolution of the clinical scenario indicates that a further condition is evolving to explain the fi ndings and clinical condition. In this case the two likeliest conditions are BPD and PIE, both most commonly associated with immature lungs and both of which give bubbly lucencies on radiography. PIE is a feature of air leak phenomena which occur in stiff lungs and is due to either high airway pressure or alveolar overdistention causing passage of gas into the interstitial spaces. It is associated with other airleak phenomena such as pneumopericardium. BPD was originally described to occur in four stages, but the advent of refi ned ventilation, surfactant, and prophylactic administration of corticosteroids, have changed the typical progression. A complete discussion of these diseases is found in the article referenced below. BPD tends to develop more gradually than PIE (as described in the clinical vignette) and tends to occur later than PIE.
394
A 5-year-old girl with a history of precocious puberty and increased serum inhibin levels is referred for ultrasound of the pelvis. On ultrasound, there is a complex solid/cystic mass in the adnexa. MRI of the pelvis confi rms a solid/cystic ovarian mass with a ‘sponge-like appearance’ on T2WI. What is the likely diagnosis? A. Sertoli–Leydi cell tumour. B. Juvenile granulosa cell tumour. C. Mature cystic teratoma. D. Fibroma. E. Mucinous cystadenocarcinoma.
B. Juvenile granulosa cell tumour. This is a sex cord stromal tumour arising from the granulose-thecal cells. They typically affect prepubescent girls and are unilateral. They are usually hormonally active (secrete oestrogen), resulting in precocious puberty. Granulosa cell tumours have a varied appearance on imaging. They may be completely solid with or without haemorrhagic or fi brotic changes, multilocular solid/cystic, or a completely cystic neoplasm. On T2WI they have a characteristic sponge-like appearance with innumerable cystic areas in a solid lesion of intermediate signal. Inhibin is the tumour marker used to monitor these lesions. Sertoli–Leydig cell tumour is also hormonally active, but it results in virilization and hirsutism. Malignant epithelial neoplasms of the ovaries are extremely rare in prepubertal girls. Mature cystic teratoma is the most common type of ovarian neoplasm in the paediatric age group, containing derivatives of at least two of the three germ cell layers: ectoderm, mesoderm, or endoderm. Fibroma is rare in children.
395
An antenatal ultrasound of foetus at 20 weeks gestation reveals an occipital encephalocele. Foetal MRI demonstrates bilateral enlarged kidneys with cystic dysplasia and polydactyly. What is the diagnosis? A. Autosomal recessive polycystic kidney disease. B. Bardet–Biedl syndrome. C. Meckel Gruber syndrome. D. Tuberous sclerosis. E. Zellweger syndrome.
C. Meckel Gruber syndrome. All of the mentioned conditions are associated with multiple renal cysts, but the triad of bilateral enlarged cystic kidneys, occipital encephalocele, and polydactyly is diagnostic of Meckel Gruber syndrome. Bardet–Biedl syndrome is associated with enlarged cystic dysplasia of the kidneys with polydactyly.
396
A 7–day-old neonate presents with delayed passage of meconium and abdominal distension. There is no vomiting. An AXR shows non-specifi c gas- fi lled loops of bowel extending distally with air/fl uid levels. A water-soluble contrast enema is performed and this shows a large fi lling defect in the proximal descending colon. There is a calibre change at this site between a small distal colon and mildly distended proximal colon. The colon wall is otherwise smooth, with no abnormal contractions or wall irregularity. What is the most likely diagnosis? A. Hirschsprung’s disease. B. Meconium plug syndrome. C. Meconium ileus syndrome. D. Small left colon syndrome. E. Colonic atresia.
B. Meconium plug syndrome. This is a transient disorder of the neonatal colon characterized by delayed passage of meconium and proximal dilatation of the bowel. The obstruction is due to a colonic dysmotility and associated with a large meconium plug in the left colon. The colon may appear normal or distended proximal to the plug and small caliber distally. A water-soluble contrast enema is both diagnostic and therapeutic. If there is persistence of abdominal distension and/or failure to evacuate, then rectal biopsy may be needed to exclude Hirschsprung’s disease. Hirschsprung’s disease is a disorder of the bowel resulting from absence of normal ganglionic cells in the Auerbach’s and Meissner’s plexuses. Aganglionosis always affects the rectum and extends proximally for a variable distance. Short segment disease occurs in the vast majority of cases, with the transition zone (the junction between the normal-sized distal aganglionic segment and the proximal dilated bowel) occurring in the region of the rectosigmoid. Contrast enema is used to delineate the level of the transition zone and care must be taken not to overfi ll the recto-sigmoid and thus mask the transition zone. Irregular, uncoordinated contractions in the aganglionic segment would more commonly be seen in the distal colon in Hirschsprung’s disease than in a meconium plug syndrome. Meconium ileus syndrome is a distal small bowel obstruction secondary to thick inspissated meconium in the ileum, usually associated with CF. Water-soluble contrast enema may be both diagnostic and therapeutic. A small left colon syndrome is a subtype of meconium plug syndrome. Contrast enema shows an apparent transition zone at the splenic fl exure. There is a normal-sized sigmoid colon, but a small descending colon, mimicking a microcolon, which becomes normal sized again around the splenic fl exure. The caliber abnormality is therefore limited to the descending colon, which was not described in this case. Colonic atresia is secondary to a vascular insult in utero and is extremely uncommon. A microcolon exists distal to the atretic segment.
397
An 18-month-old girl presents with increasing incoordination and developmental regression. T2WI demonstrates confl uent high signal within the periventricular white matter and centrum semiovale, with radiating linear low signal intensity, giving a ‘tigroid’ pattern. Sparing of subcortical U fi bres is also noted. What is the most likely diagnosis? A. Krabbe disease. B. Metachromatic leucodystrophy. C. X-linked adrenoleucodystrophy. D. Alexander disease. E. Canavan disease.
B. Metachromatic leucodystrophy. Leucodystrophies are dysmyelinating inherited white matter diseases, which are secondary to lysosomal, peroxisomal, or mitochondrial dysfunction. Metachromatic leucodystrophy is caused by defi ciency of the lysosomal enzyme ayrlsulfatase A. The ‘tigroid’ pattern relates to sparing of perivascular white matter.
398
An 8-month-old child presents with gradually increasing abdominal swelling. An ultrasound scan of abdomen is performed and this shows a large liver mass, which is well-defi ned and slightly hyperechoic to surrounding hepatic parenchyma. A subsequent biphasic CT scan confi rms a solitary heterogeneous, but predominantly hypodense, liver mass on both phases with some calcifi cation. The serum alphafetoprotein is elevated. What is the most likely diagnosis? A. Hepatic angiosarcoma. B. Mesenchymal hamartoma. C. Undifferentiated embryonal sarcoma. D. Hepatoblastoma. E. Hepatocellular carcinoma.
D. Hepatoblastoma. This is the most common primary malignant hepatic tumour in children (approximately 80%). Most cases occur below the age of 3 years. Serum alphafetoprotein is elevated in 70–90%. The imaging features are as described. Calcification is seen in approximately 50%. HCC is the next most common malignant liver tumour and has two peaks in childhood, at 2–4 years and 12–14 years. Serum alphafetoprotein is also elevated in approximately 80%. Imaging features are similar to those seen in an adult population, i.e. larger tumours are of heterogeneous echogenicity on ultrasound and on biphasic CT they show arterial enhancement with rapid washout on the portal venous phase. Undifferentiated embryonal sarcoma (UES) is the third most common liver malignancy and is most common between the ages of 6 and 10 years. Serum alphafetoprotein levels are normal. The imaging hallmark of this tumour is the discrepancy between its appearance on ultrasound and that on CT. Ultrasound is used to confi rm the solid nature of this tumour, which typically is isoechoic or hyperechoic. On CT, UES is seen as a well-circumscribed, multiseptate, fl uid- attenuating lesion. Enhancement of solid-appearing septa and a pseudocapsule may be seen. Mesenchymal hamartoma is the second most common benign hepatic lesion (to haemangioendothelioma). It is usually seen under the age of 2 years. Serum alphafetoprotein is not elevated. The typical ultrasound appearance is a multiseptate, cystic mass. Less commonly, the solid component of the lesion can be more predominant, with multiple smaller cysts giving the lesion a Swiss-cheese appearance. The usual CT fi nding is of a multilocular cystic mass with enhancing septa of varying thickness. Angiosarcoma is not usually seen in children, but is a rare, highly vascular and highly malignant, hepatic tumour of adults.
399
A 6-week-old male child presents with non-bilious vomiting after feeds, which has more recently become projectile in nature. You clinically suspect hypertrophic pyloric stenosis. Which of the following ultrasound fi ndings would be inconsistent with this diagnosis? A. A pyloric muscle thickness of 4 mm. B The presence of retrograde gastric contractions. C A pyloric thickness (serosa to serosa) of 10 mm. D The presence of hyperperistaltic gastric contractions. E A pyloric channel length of 12 mm.
E. A pyloric channel length of 12 mm. Ultrasound is the primary imaging modality used for the diagnosis of hypertrophic pyloric stenosis, although the criteria are generally applied to infants of 6 weeks and older. These include muscle thickness >3 mm, pyloric channel length >14 mm, pyloric thickness (serosa to serosa) >10 mm, failure of the pylorus to open with pyloric elongation and displacement, as well as retrograde or hyperperistaltic gastric contractions. A pyloric ratio, which is the ratio of the wall thickness to the pyloric diameter, above 0.27 is stated to be 96% sensitive for the diagnosis. The standard measurements above are not always applicable to neonates and real-time assessment to look for passage of fl uid from the stomach into the duodenum becomes more important
400
A CT chest has been requested for a neonate in the neonatal ICU. This infant was born at 27 weeks gestation and developed right-sided PIE during the fi rst week of life. The neonatologists practiced selective left bronchial intubation and no further air leak sequelae occurred. Also present on the CXR is a hyperlucent lesion in the right lower lobe. This is not clearly seen on the initial radiographs due to the generalized haziness present due to the surfactant defi ciency. This lesion is not increasing in size and is not causing any signifi cant respiratory embarrassment, but requires further assessment to defi ne treatment. On CT a focal lesion is present confi ned to the right lower lobe, which consists of multiple cystic structures with central linear densities. This area demonstrates mild expansion. What is the diagnosis? A. Congenital cystic adenomatoid malformation. B. Persistent PIE. C. Congenital diaphragmatic hernia. D. Congenital lobar emphysema. E. Bronchogenic cyst.
B. Persistent PIE. Although alluded to in the clinical scenario, this should not be assumed to be the most likely diagnosis in the absence of the CT fi ndings, as this is an extremely uncommon condition. The CT findings provide the diagnosis due to the linear densities within the cystic cavities representing the bronchopulmonary bundle surrounded by air within the interstitial space. This appearance is seen in over 80% of cases. The abnormality is often confi ned to a single lobe, but can be more widespread. Current optimal management is debated. Lesions increasing in size are thought to be best treated with surgical resection, with stable lesions often resolving over time with conservative management.
401
A 10-year-old boy of Japanese origin presents with episodes of right transient hemiparesis and declining intellect. MRI brain is performed. Which of the following are the most likely radiological fi ndings? A Multiple fl ow voids within the basal ganglia bilaterally. B Irregular beading of the left extracranial internal carotid artery. C Hypoplasia of the left internal carotid artery. D Distal left middle cerebral artery aneurysm. E. Normal study.
A. Multiple fl ow voids within the basal ganglia bilaterally. The history and ethnic origin of the patient suggest moyamoya syndrome. This is an idiopathic progressive arteriopathy of childhood resulting in narrowing of the distal internal carotid arteries and lenticulostriate collateralization. This collateralization causes the multiple basal ganglia fl ow voids, likened to a ‘puff of smoke’ (moyamoya in Japanese) on angiography. Secondary causes of moyamoya collateralization include neurofi bromatosis type 1, post-radiation therapy and sickle cell anaemia. Irregular beading of the extracranial internal carotid arteries is seen in fi bromuscular dysplasia.
402
A 4-day-old cyanosed infant is admitted as an emergency to SCBU following a home birth. The history is of episodes of severe cyanosis developing when the infant is distressed. A CXR is carried out and shows a normal mediastinal contour with slightly decreased pulmonary vascularity. What do you think the most likely cause of this infant’s cyanosis is based on these fi ndings? A. Truncus arteriosus. B. Pulmonary valve atresia with an intact ventricular septum. C. Pulmonary valve hypoplasia with a VSD and overriding aorta. D. D-transposition of the great arteries. E. Coarctation of the aorta.
C. Pulmonary valve hypoplasia with a VSD and overriding aorta. These are the features of tetralogy of Fallot. Even in the absence of radiology this represents the most common form of congenital cyanotic heart disease. To help characterize congenital heart disease there are a number of features to note on the radiograph as well as clinically. The presence of cyanosis excludes coarctation, which does not cause cyanosis. Truncus arteriosus (and total anomalous pulmonary venous return (TAPVR)) are associated with increased pulmonary fl ow, not reduced, as present in this case. Pulmonary valve atresia with an intact ventricular septum does have reduced pulmonary fl ow, but would present earlier than four days and the heart is typically grossly enlarged, as in this anomaly there is no forward fl ow of blood out of the right ventricle. D-transposition of the great arteries is classically described as giving an egg-on-a-string appearance due to the narrowed superior mediastinum caused by the abnormal rotation of the great vessels. This appearance is seldom seen as D-transposition presents early with cyanosis and is surgically corrected. The most common CXR appearance in an infant is a normal CXR. The pulmonary vascularity on the radiograph is either normal or increased, not decreased.
403
An 11-year-old boy falls and injures his elbow. Which of the following combination of injuries is most likely? Posterior dislocation and capitellum fracture. Anterior dislocation and lateral epicondyle fracture. Posterior dislocation and medial epicondyle fracture. Anterior dislocation and trochlea fracture. Posterior dislocation and radial head fracture.
C. Posterior dislocation and medial epicondyle fracture. The elbow is the most commonly dislocated joint in children (in adults the shoulder and interphalangeal joints are more common). A supracondylar fracture of the elbow is more common in children (65% of elbow fractures) than in adults, who generally suffer a radial head injury (50%). However, in both groups of patients, if dislocation occurs it tends to be posterior dislocation of the radial head and ulna with respect to the distal humerus (85–90%). The most common fracture seen in association with elbow dislocation in children is a fracture of the medial epicondyle or separation of the medial epicondyle. The second most common fracture involves the radial head and neck. In terms of total paediatric elbow fractures, those involving the lateral epicondyle (15%) are next most common after the supracondylar fracture; medial epicondyle fractures comprise approximately 10%.
404
A 6-year-old boy undergoes an ultrasound examination of the renal tracts that shows dilated, polygonal, multifaceted calcyces in the right kidney. The infundibula, renal pelvis, and ureter are normal in calibre. IVU confi rms the ultrasound fi ndings and there is normal contrast excretion. What is the diagnosis? A. Congenital megacalcyces. B. Obstructive hydronephrosis. C. Multicystic dysplastic kidney. D. Congenital megacystis-microcolon syndrome. E. Vesico-ureteric refl ux.
A. Congenital megacalyces This is a rare developmental abnormality of the renal medulla. Imaging demonstrates dilated, multifaceted, or polygonal calyces as opposed to spherical calyces of obstructive hydronephrosis. In addition, the infundibula, renal pelvis, and ureter are normal in calibre in congenital hydronephrosis. The affected kidneys may demonstrate cortical thinning but the renal function is preserved.
405
A 10-year-old boy presents with a 1-day history of severe epigastric pain radiating to the back. Serum amylase is elevated. The patient has an ultrasound scan of abdomen performed which does not show evidence of gallbladder or biliary disease. Pancreatitis is suspected. What is the most useful sign with regard to the pancreas on ultrasound that would support this diagnosis? A. Increased pancreatic echogenicity. B. Decreased pancreatic echogenicity. C. Pancreatic duct dilatation. D. Pancreatic atrophy. E. Pancreatic calcifi cation
C. Pancreatic duct dilatation. In cases of acute pancreatitis in children, the echogenicity of the pancreas on ultrasound is not a helpful diagnostic feature and pancreatic enlargement is absent in 50% of the patients. The most useful diagnostic feature is dilatation of the pancreatic duct (>1.5 mm at 1–6 years of age, >1.9 mm at 7–12 years, and >2.2 mm at 13–18 years). Pancreatic ductal dilatation may also be seen in chronic pancreatitis. Pancreatic atrophy and parenchymal calcifi cations are more typically associated with chronic disease.
406
A male neonate is born at 40 weeks gestation. There is a history of polyhydramnios during pregnancy. During baby checks, the baby is noted to have a sacral dimple. At this time the mother reports that the baby has been unable to feed properly during the initial 24 hours, during which it dribbles and spits out milk. She also reports the baby has had recurrent coughing episodes. The paediatrician is unable to pass an nasogastric tube and suspects a tracheoesophageal fi stula. A contrast swallow is requested. This demonstrates termination of the proximal oesophagus, with a fi stulous connection between the proximal oesophagus and the trachea. Contrast does not pass into the distal oesophagus. What class of tracheoesophageal fi stula is this likely to be? A. Type A. B. Type B. C. Type C. D. Type D. E. Type E.
Type B. Tracheoesophageal fi stulae are classed depending on the communications present between the trachea and the oesophagus. In types A–D the mid-portion of the oesophagus is absent. Type A are not truly fi stulae, but consist of an absence of the mid-portion of the oesophagus. In type B the proximal oesophageal bud communicates with the trachea and in type C it is the distal bud that has a tracheal fi stula. Type C is the most common subtype. Both proximal and distal buds have fi stulous connections with the trachea in type D. Type E (or H type) tracheoesophageal fi stulae have complete trachea and oesophagus with a fi stulous connection between them resembling the letter H.
407
A 15-year-old male undergoes a pelvic MRI on which an incidental note is made of absent bilateral seminal vesicles. Which of the following is commonly associated with bilateral seminal vesicle agenesis? A. Renal agenesis. B. CF. C. Calcifi ed vas deferens. D. Ectopic ureter. E. Rotation anomaly of one kidney
B. CF. Bilateral seminal vesicle agenesis is frequently associated with bilateral agenesis of the vas deferens and mutations in the CF transmembrane conductance regulator gene (64–73%). It is thought to be related to luminal obstruction by thick secretions. Bilateral agenesis of the vas deferens is seen in 99% of CF in males. The affected patients usually have normal kidneys. Unilateral seminal vesicle agenesis and seminal vesicle cyst are commonly associated with ipsilateral renal agenesis. Bilateral calcifi ed vas deferens is commonly seen in diabetics.
408
A 7-year-old boy is being investigated for a history of mild exertional dyspnoea. A CXR has been carried out that shows normal lungs but does demonstrate a linear density passing inferiorly from the right lower hemithorax to the level of the diaphragm. A cardiac MRI has been requested, which shows normal pulmonary venous drainage on the left side. On the right side, the lower pulmonary vein drains into the right atrium, the superior pulmonary vein drains into the left atrium, and the arterial supply for the entire right lung arises from the pulmonary artery. No cardiac abnormality is identifi ed. What form of abnormality does this child have? A. Total anomalous pulmonary venous return. B. Partial anomalous pulmonary venous return (PAPVR). C. Extralobar sequestration. D. Cor triatriatum. E. Scimitar syndrome.
B. Partial anomalous pulmonary venous return (PAPVR). This most commonly involves the right superior pulmonary vein, but in this case involves the inferior vein. This condition may remain asymptomatic for a number of years, depending on the amount of blood that returns anomalously. PAPVR can drain into the superior vena cava (SVC), right atrium, or IVC on the right and into the brachocephalic vein or coronary sinus on the left. Scimitar syndrome is a subtype of PAPVR that is associated with ipsilateral lung hypoplasia (hence alternative name hypogenetic lung syndrome) and occasionally dextrocardia. As noted in the history, the lungs were normal. Extralobar sequestrations have aberrant pulmonary arterial supply, not present in this case. Total anomalous pulmonary venous return presents in the neonatal period, commonly with cyanosis and plethora on CXR. Patient survival depends on the presence of an ASD or PDA. No atrial abnormalities were noted to indicate cor triatriatum, although this is another disorder in the spectrum of pulmonary venous developmental anomalies.
409
A 14-year-old boy presents with a slow-growing painless mass at the angle of the mandible on the left. Ultrasound demonstrates a hypoechoic left parotid mass containing echogenic calcifi c foci. On follow-up contrast- enhanced MRI, the mass demonstrates mild increased enhancement. Which of the following is the most likely diagnosis? A. Warthin tumour. B. Primary lymphoma. C. Parotitis. D. Pleomorphic adenoma. E. Haemangioma.
D. Pleomorphic adenoma. This is the most common benign salivary gland tumour in children and usually appears in later childhood or adolescence. The tumour originates in the parotid gland in up to 90% of cases. Haemangiomas are the next most common benign lesion. They are usually seen in the fi rst 6 months of life and have a female predilection. They are hypoechoic and display a variable degree of abnormal vasculature. Parotitis is usually due to mumps and results in a tender gland, which is diffusely enlarged with a heterogenous echotexture on ultrasound. Warthin tumour is a well-circumscribed cystic solid lesion, usually towards the tail of the parotid gland. It is the most common lesion to manifest as multifocal or bilateral masses. Primary lymphoma of the salivary glands is rare, but most often involves the parotid. Ultrasound will show an enlarged, diffusely infi ltrated gland.
410
A 5-year-old boy with bilateral wrist pain undergoes a plain fi lm which reveals several peduncuated bony outgrowths from the metaphyses of both radii, which point away from the adjacent joints. What is the most likely diagnosis? A. Ollier disease. B. Maffucci syndrome. C. Morquio syndrome. D. Diaphyseal aclasia. E. Hunter syndrome.
D. Diaphyseal aclasia. The description is classic for multiple hereditary osteochondromas/exostoses, also known as diaphyseal aclasia. Osteochondromas are the result of displaced growth plate cartilage, which causes lateral bone growth from the metaphysis. They typically point away from the epiphysis. There is continuity of the normal marrow, cortex, and periosteum between the exostosis and the host bone. The cartilage cap, which is the source of growth, may have some chondoid matrix, but the appearance is otherwise of a deformed but normal bone. They are normally found in the extremities, with 36% around the knee. Their growth normally ceases at skeletal maturity. Symptoms are related to pressure effects on adjacent neural or vascular structures. Less than 1% of solitary osteochondromas undergo malignant transformation to chondrosarcoma. Findings that should alert to this are destruction of exostosis bone, destruction of matrix in the cartilage cap, irregular or thick (>2 cm in adults, >3 cm in children) cap, or growth of the cap after skeletal maturity. Multiple hereditary osteochondromatosis is an uncommon autosomal dominant condition. Patients present with multiple osteochondromas, which cause short stature. The elbow and wrist joints are often deformed. There is a higher risk of malignant transformation than in solitary osteochondromas, probably 2–5%. Ollier disease is the presence of multiple enchondromas and Mafucci syndrome requires, in addition, multiple soft-tissue haemangiomas. Morquio and Hunter syndromes are mucopolysaccharidoses, with their own musculoskeletal abnormalities, which often make an appearance in exams.
411
A 15-year-old female undergoes an ultrasound of pelvis that demonstrates uterine abnormality with a differential diagnosis of bicornuate or septate uterus. An MRI of the pelvis is requested for further assessment. Which of the following fi ndings on MRI is suggestive of a bicornuate uterus? A. Two uterine cavities. B. Fundal concavity of less than 1 cm. C. Intercornual distance of more than 4 cm. D. Convex external fundal contour. E. Thin fi brous low-intensity septum separating the uterine cavities.
C. Intercornual distance of more than 4 cm. Incomplete fusion of the Mullerian ducts results in bicornuate uterus. MRI demonstrates divergent uterine horns with an external fundal cleft that is more than 1 cm deep and an intercornual distance of more than 4 cm. In bicornuate bicollis uterus, two separate cervical canals are seen. This is distinguished from uterus didelphys by a greater degree of fusion between the horns in the lower uterine segment. Septate uterus results from incomplete septal resorption following Mullerian duct fusion. It is the most common uterine anomaly (55%). At MRI, the external fundal countour may be convex, fl at, or minimally concave (less than 1 cm deep). MRI is also useful in characterizing the type (fi brous/ muscular) and extent of the septum. Differentiation between bicornuate and septate uterus is important, as septate uterus is amenable to surgical management.
412
Follow-up MRI is performed on a foetus of 26 weeks gestational age after ultrasound raised the suspicion of agenesis of the corpus callosum (ACC). This subsequently confi rms that the callosum is absent. What is the most likely additional radiological fi nding? A. None, isolated abnormality. B. Parenchymal T2WI signal hypointensity. C. Periventricular nodular heterotopia. D. Dysplastic brainstem. E. Delayed sulcation.
E. Delayed sulcation. Signs of ACC include absence of the cavum septum pellucidum, colpocephaly, high-riding third ventricle, and widening of the inter-hemispheric fi ssure. ACC is reported to be isolated in <10% on foetal MR imaging. Sulcation delay is present in most foetuses with ACC (particularly those imaged at <30 weeks gestation), including those with good neurodevelopmental outcome, implying a global white matter dysgenesis. Periventricular nodular heterotopia and parenchymal T2WI signal hypointensity are usually seen in association with abnormal sulcal morphology. Associated posterior fossa abnormalities are also common, with cerebellar hemispheric abnormalities seen more than abnormalities of the vermis. Brainstem abnormalities typically occur in association with a cerebellar abnormality.
413
An infant is of short stature and undergoes a skeletal survey. This reveals markedly shortened femora and humeri, although the other long bones are also greatly affected. The vertebral bodies are moderately fl attened and there is a reduction in the interpedicular distance in a caudal direction in the spine. What is the diagnosis? A. Thanatophoric dysplasia. B. Achondroplasia. C. Chondrodysplasia punctata. D. Jeune syndrome. E. Cleidocranial dysplasia.
B. Achondroplasia. This question tests knowledge of the severe skeletal dysplasias. Achondroplasia is the most common cause of dwarfi sm and is hereditary. The humeri and femora are more profoundly affected than the other long bones, although the entire skeleton is abnormal. Narrowing of the interpedicular distance as one progresses caudally down the spine is unique to this condition. Other features include a large skull with a small skull base, square iliac wings with a ‘champagne- glass’ pelvic cavity, and widened metaphyses. Thanatophoric dysplasia results in early death. Features include severe platyspondyly and ‘telephone receiver’ femora. Chondrodysplasia punctata results in stippled epiphysis and rhizomelic dwarfi sm. Jeune syndrome (asphyxiating thoracic dystrophy) results in respiratory distress, very short ribs, metaphyseal irregularity/beaking, and trident acetabulum. Cleidocranial dysplasia is characterized by lack of development of the pubic bones bilaterally. There is also absence or hypoplasia of the clavicles and the presence of wormian bones in the skull.
414
With appropriate clinical suspicion, which of the following radiologically depicted injuries would most raise suspicion of NAI? A. Posterior rib fracture. B. Vertebral compression fracture. C. Duodenal haematoma. D. Spiral fracture of long bone. E. Jejunal laceration
A. Posterior rib fracture. Metaphyseal corner fractures (or bucket handle fractures) and posterior rib fractures are the most specifi c for NAI. The other injuries given in the stem are also worrisome for NAI, particularly when observed in infants. Posterior rib fractures are sometimes relatively occult on plain fi lm and scintigraphy is more sensitive. They occur when the chest is squeezed tightly, e.g. when a child is shaken. Shaking can also result in a thoraco-lumbar compression fracture. Spiral fractures of long bones are suspicious in infants, but in ambulatory children the ‘toddler’s’ spiral fracture of the tibia is common and often has no memorable preceding trauma. Duodenal haematoma and jejunal perforation are both recognized sequelae of NAI: the small bowel is the most injured organ in child abuse.
415
A patient presents with recent onset neurological symptoms suspicious of an acute presentation of multiple sclerosis (MS). Which of the following anatomical sites of plaque involvement is least consistent with this? A. Corpus callosum. B. Spine involvement in the absence of brain involvement. C. Cerebral cortex. D. Symmetrical involvement of cerebral white matter. E. Floor of the fourth ventricle.
D. Symmetrical involvement of cerebral white matter. Symmetrical involvement of the cerebral hemispheres or cerebellar peduncles is unusual in MS and is occasionally seen in acute disseminated encephalomyelitis (ADEM). ADEM can mimic an acute presentation of MS both clinically and in terms of imaging fi ndings. The monophasic nature of ADEM can be deduced both from the uniformity of lesional oedema and contrast enhancement in the acute phase. Lesions in ADEM resolve on follow-up, and although enhancing and non-enhancing lesions can coexist for a period, new lesions should not appear. MS plaques are classically seen in the periventricular and juxtacortical white matter. The other options in the question are all common plaque locations. Involvement of the corpus callosum is characteristic. Other common supratentorial sites include the white matter abutting the temporal horns and trigones of the lateral ventricles. Cortical lesions are less conspicuous on MRI than white matter lesions, but their detection is improved by the inclusion of a FLAIR sequence. Juxtacortical white matter lesions are highly suggestive of MS, as lesions are not commonly seen in this region in normal ageing. Twelve per cent of patients have lesions on MRI limited to the spine without brain involvement.
416
A 52-year-old man is investigated by MRI of brain for a possible transient ischaemic attack (TIA). Focal lesions of CSF signal intensity are identifi ed adjacent to the anterior commissures. The referring neurologist suspects that these lesions are chronic lacunar infarcts. What MRI fi nding suggests that these are in fact prominent perivascular spaces? A. No restricted diffusion on DWI. B. Hypointense on T1WI. C. Hyperintense on T2WI. D. Suppress on FLAIR. E. Normal surrounding brain parenchyma.
E. Normal surrounding brain parenchyma. Perivascular spaces (Virchow–Robin spaces) are pial lined interstitial fl uid structures that accompany penetrating arteries, but do not communicate directly with the subarachnoid space. They can occur anywhere, but typically cluster around the anterior commissure. They follow CSF signal intensity, suppress on FLAIR, and do not exhibit restricted diffusion. They can occasionally be giant when located within the midbrain. Lacunar infarcts will also be hypointense on T1WI and hyperintense on T2WI. Restricted diffusion will be seen when acute/subacute. They are typically of increased signal on FLAIR, although will suppress if there is central encephalomalacia. A halo of surrounding high signal on T2WI and FLAIR is typical of lacunar infarction, although up to 25% of prominent perivascular spaces can also demonstrate a slight halo of increased signal. Lacunar infarcts are also more typically seen in the setting of more extensive white matter disease.
417
A patient is referred to your neurointerventional team for embolization of a meningioma prior to surgical resection. The lesion is based on the tentorium. What is the likely feeding vessel (parent vessel is named in brackets)? A. Anterior meningeal artery (vertebral). B. Middle meningeal artery (external carotid artery (ECA)). C. Posterior meningeal artery (variable). D. Bernasconi–Casanari artery (Internal carotid artery (ICA)). E. Dorsal meningeal artery (ICA).
D. Bernasconi–Casanari artery (ICA). The majority of meningiomas occur in the parafalcine region, along the convexity or around the sphenoid. These all derive their supply from the ECA, although parafalcine meningiomas can also receive supply from a branch of the ophthalmic artery. Tentorial or cerebellopontine angle (CPA) tumours are classically fed by the Bernasconi–Casanari artery, a branch of the meningohypophyseal trunk of the ICA. Lesions around the foramen magnum, clivus, and posterior fossa are fed by branches of the vertebral artery (anterior and posterior meningeal) and meningohypophyseal trunk of the ICA.
418
A 13-year-old boy is brought to your paediatric hospital with a recent history of headache and high fever. The child is becoming progressively drowsy and demonstrates rigidity and tremor on neurological examination. An MRI is requested. Relevant past medical history on the request form is of recent travel to Asia, history of measles as a 6-year-old and recent viral infection. The MRI scan shows increased signal on T2WI and FLAIR sequences in the hippocampal regions of the temporal lobes. There is also increased FLAIR signal in the thalami and putamina bilaterally. Small foci of increased T1WI signal within these regions are felt to represent haemorrhage. What diagnosis would you place at the top of your differential list? A. Herpes simplex type 1 encephalitis. B. Herpes simplex type 2 encephalitis. C. Japanese encephalitis. D. Varicella Zoster encephalitis. E. Subacute sclerosing panencephalitis (SSPE).
C. Japanese encephalitis. Japanese encephalitis and herpes simplex virus (HSV) encephalitis both present with similar acute and rapidly progressive neurological symptoms. The key differentiator is involvement of the basal ganglia, which is typical in Japanese encephalitis but rare in HSV. Both commonly involve the hippocampi, this being the classical appearance of HSV encephalitis. HSV type 1 is the subtype that affects adult and older children. HSV type 2 causes neonatal and in utero infection. Herpes varicella zoster virus (VZV) infection can be seen in immunocompromised children, but in the immunocompetent population is more typically seen in elderly patients, often, but not exclusively, in the presence of cutaneous shingles. It causes a vasculitis, which can be seen angiographically and causes bilateral increased T2WI/FLAIR foci and gyriform enhancement in the distribution of the vasculitis. SSPE presents with a more protracted history of neurological decline.
419
An 8-year-old female patient presents to your paediatric neurology service with a history of increasing ataxia, repeated headaches, and vomiting, increasing in severity over the last 5 months. Clinical examination reveals marked cerebellar signs of past pointing and dysdiadochokinesis. An MRI is requested, which shows a solid mass in the posterior fossa measuring 2 cm in size. This mass arises in the left cerebellar hemisphere and displaces the fourth ventricle. It is of low intensity on T1WI and high signal on T2WI. There is only a small rim of surrounding oedema. The lesion demonstrates relatively homogeneous moderate enhancement. There is no evidence of subarachnoid seeding. What is the most likely diagnosis? A. Pilocytic astrocytoma. B. Ependymoma. C. Medulloblastoma. D. Metastasis. E. Lhermitte–Duclos syndrom
A. Pilocytic astrocytoma. This question deals with the classical neurological differential diagnosis of a posterior fossa mass in a child. While there are many causes, pilocytic astrocytoma and medulloblastoma account for over 60% of all childhood posterior fossa masses. Pilocytic astrocytomas have a classical appearance of being cystic lesions with an avidly enhancing mural nodule. However, 30% of pilocytic astrocytomas are solid tumours. In differentiating them from medulloblastomas, pilocytic astrocytomas often arise more peripherally and displace the fourth ventricle, whereas medulloblastomas usually arise centrally from the vermis. Subarachnoid seeding is seen in up to 50% of cases of medulloblastoma. Ependymomas are also included in the differential. As these arise from the ependyma lining the ventricle, they tend to be centred on the fourth ventricle in children. Metastases are the most common cause of a posterior fossa mass in adults, but are less common in children
420
An MRI is carried out for your neurology service on a 30-year-old male patient. The most pertinent abnormality is of thick smooth meningeal enhancement following the dural-arachnoid around the convexity, falx, and tentorium without extension into the basal ganglia or ventricles. The referring clinician arrives to discuss the fi ndings, but has mislaid the request form and clinical information. While he is looking for it, what do you think the most likely clinical presentation is? A. Neck stiffness, photophobia, raised WCC, petechial rash. B. Neck stiffness, drowsiness, with history of breast cancer. C. Acute limb weakness on right side 3 days previously, not resolving. D. Recent back surgery complicated by ongoing CSF leak. E. Possible fungal meningitis in immunocompromised patient.
D. Recent back surgery complicated by ongoing CSF leak. The key feature is the pattern of meningeal enhancement. It is important to recognize that the imaging fi ndings describe pachymeningeal, not leptomeningeal, enhancement and as such the causes of leptomeningeal enhancement (bacterial or fungal meningitis) can be discounted. Similarly a recent stroke can cause gyriform enhancement, not pachymeningeal enhancement. Breast cancer is the most common malignancy to cause pachymeningeal enhancement, but this would usually be more nodular than smooth. CSF leak or any cause of intracranial hypotension will be associated with smooth pachymeningeal thickening, primarily over the convexities and falx. This enhances, not because of any infl ammatory process, but because this part of the meninges has no blood–brain barrier, unlike the leptomeninges.
421
A patient is having an MRI scan carried out to investigate a possible right frontal astrocytoma, incidentally detected on CT following a head injury. The MRI features are typical of an astrocytoma, with no evidence of necrosis or callosal involvement to indicate glioblastoma multiforme (GBM). MRS has been carried out to help assess the grade of this tumour. What MRS features would indicate a high grade lesion? A. Elevated choline, reduced N-acetyl aspartate (NAA), choline/creatine (Cho/Cr) ratio of 1. B. Elevated choline, reduced NAA, Cho/Cr ratio of 2. C. Normal choline, elevated NAA. D. Reduced choline, reduced NAA. Cho/Cr ratio of 1.2. E. All normal, these are unaffected by tumour grade.
B. Elevated choline, reduced NAA, Cho/Cr ratio of 2. NAA is thought to be a marker of neuronal integrity, choline indicates cell turnover, and creatine indicates cell metabolism. Lactate is not detectable in normal brain spectra but is elevated in infl ammation, infarction, and some neoplasms. Most brain conditions, whether neoplastic, vascular, or demyelinating, are associated with a reduction in NAA. A notable exception is Canavan’s disease, which causes a rise in NAA. Choline is elevated in many disorders, but is markedly increased in high-grade neoplasms. It has been reported that the ratio of choline to creatine can be used to help grade tumours, with a ratio over 1.5 indicating high grade in the majority of cases. A reduced choline and NAA in an area of tumour can indicate necrosis.
422
Which one of the following orbital pathologies typically arises from the intraconal compartment? A. Cavernous haemangioma. B. Adenocystic carcinoma. C. Rhabdomyosarcoma. D. Dermoid. E. Orbital pseudotumour.
A. Cavernous haemangioma. The orbital compartments are split up into extraconal, conal, intraconal, and globe. The extraconal compartment consists of fat, lacrimal gland, and bony orbit. Pathology in this region includes infection, neurofi broma, adenocarcinoma, mucoepidermoid and adenoid cystic carcinoma, neoplasia of bone, and lymphoma. Conal pathology (muscle) includes rhabdomyosarcoma, thyroid eye disease, and idiopathic orbital infl ammation (pseudotumour). The intraconal compartment consists of fat, lymph nodes, vessels, nerves, and the optic nerve sheath complex. Pathology in this region includes venolymphatic malformation, haemangioma, arteriovenous malformation, optic nerve meningioma/glioma, and lymphoma. Pathology in the globe includes retinoblastoma, metastasis, and melanoma.
423
A 20-year-old female with a history of neurofi bromatosis presents with reduced visual acuity in the right eye. She subsequently has CT and MR imaging of the orbits to assess for a tumour relating to the right optic nerve. Which of the following fi ndings on imaging would be more suggestive of the presence of an optic nerve glioma, rather than a meningioma arising from the optic nerve sheath? A. Presence of the ‘tram-track’ sign. B. Presence of optic canal widening. C. Presence of marked intense tumour enhancement. D. Presence of calcifi cation. E. Presence of bony hyperostosis.
B. Presence of optic canal widening. The presence of a widened optic nerve canal occurs in up to 90% of cases of optic nerve glioma. While it can also occur in meningioma, it is more common in glioma and some cases of meningioma may even have a narrowed canal secondary to bony hyperostosis. The ‘tram-track’ sign is typically associated with meningioma and refers to the more avidly enhancing meningioma surrounding the non-enhancing optic nerve on axial CT and MR imaging of the orbit. Although both meningioma and glioma enhance following intravenous contrast, it is meningioma that is more typically associated with marked intense enhancement. Calcifi cation and bony hyperostosis are features associated with meningioma. Calcifi cation is rare in gliomas, unless they have previously undergone radiotherapy.
424
A 45-year-old man has a severe head injury and is noted to have a left facial nerve palsy. Following stabilization, a subsequent HRCT scan of the temporal bone is performed. This demonstrates a fracture of the left temporal bone, involving the course of the left facial nerve. Which orientation of fracture and which segment of the facial nerve are most likely to be involved? A. Transverse/internal auditory canal. B. Longitudinal/internal auditory canal. C. Transverse/labyrinthine. D. Longitudinal/labyrinthine. E. Transverse/mastoid. F. Longitudinal/mastoid
C. Transverse/labyrinthine. Transverse temporal bone fractures are more commonly associated with facial nerve paralysis (approximately up to 50%) than longitudinal temporal bone fractures (approximately up to 20%). The labyrinthine segment is the most likely segment of the facial nerve to be associated with facial paralysis.
425
A 45-year-old woman presents with a several month history of neck pain and gradually progressive weakness and paraesthesia in the upper limbs. An MRI scan of the cervical spine is performed and this shows a well-defi ned central intramedullary mass in the mid-cervical spinal cord. The mass is generally slightly hyperintense on T2WI, but also has a few low signal peripheral areas. It enhances homogeneously with gadolinium. What is the most likely diagnosis? A. Astrocytoma. B. Metastasis. C. Haemangioblastoma. D. Ganglioglioma. E. Ependymoma.
E. Ependymoma. Ependymoma is the most common intramedullary neoplasm in adults. It tends to be centrally located within the cord, unlike astrocytoma, which can be eccentric. Astrocytoma can have a longer segment of cord involvement than ependymoma and may have a more infi ltrative margin. The peripheral low signal areas seen on T2WI in ependymoma are related to haemosiderin deposition from prior haemorrhage. Haemangioblastoma is more often seen in the dorsal cord than the cervical cord and is typically a small well-defi ned lesion. It may have an associated cord cyst or syrinx. Flow voids may be seen within the lesion, from dilated vascular channels. Ganglioglioma is a very rare, slow-growing tumour of low malignant potential. The imaging appearance is non-specifi c, but there are some fi ndings that may suggest the diagnosis. Compared with other spinal cord tumors, gangliogliomas are more likely to involve long segments of the cord (greater than four levels, up to the whole cord), to be associated with bone erosion or scalloping, to have tumoral cysts, and to have areas of mixed high signal on precontrast T1WI. Intramedullary metastasis represents less than 5% of intramedullary lesions. They usually occur in the setting of advanced malignant disease, typically from a lung or breast primary. The spinal cord oedema can seem out of keeping with the small size of the metastatic lesion.
426
A 58-year-old patient is found at home with a reduced GCS. CT brain reveals atrophy only. MRI brain reveals hyperintensity in the tegmentum (except for the red nucleus) and hypointensity of the superior colliculus on T2WI, as well as hyperintensity in the basal ganglia. What is the most likely cause? A. Cocaine abuse. B. Methanol poisoning. C. Primary basal ganglia haemorrhage. D. Wilson’s disease. E. Carbon monoxide poisoning.
D. Wilson’s disease. Hyperintensity in the tegmentum (except for the red nucleus) and hypointensity of the superior colliculus are described as the ‘face of the giant panda sign’ and are seen in axial T2WI sections of the midbrain in Wilson’s disease. A ‘double panda sign’ has also been described, with a second ‘panda cub face’ in the pons. Abnormal signal can also be seen in the basal ganglia and thalamus in Wilson’s disease (putamen most commonly). The signal abnormalities are due to copper deposition. Signal is generally reduced on T1WI sequences, although it may be increased due to the paramagnetic effects of copper and also due to the hepatic component of Wilson’s disease (a portocaval shunt can produce this latter fi nding). Signal is generally increased on T2WI sequences, but it can be of mixed or reduced intensity. Similarly carbon monoxide poisoning and methanol poisoning can cause increased or reduced signal on T1WI. Methanol poisoning typically causes abnormal signal in the putamen, with haemorrhagic necrosis being more typical, whereas carbon monoxide poisoning typically affects the globus pallidus. The latter would be expected to cause low attenuation in the basal ganglia on CT. The fi ndings on CT exclude basal ganglia haemorrhage. Amphetamine and cocaine abuse can cause high T2WI signal in the basal ganglia due to small areas of infarction, but are not associated with the midbrain changes.
427
A 27-year-old man suffers a head injury. A CT brain is performed. Which of the following features favours a subdural haematoma (SDH) over an extradural haematoma (EDH)? A. B. C. D. The haematoma measures 50 HU. The presence of a temporal skull fracture. The haematoma crosses the midline over the falx. The collection has a biconvex confi guration. E. The haematoma crosses sutures.
E. The haematoma crosses sutures. An SDH will cross suture lines and may extend over the whole cerebral hemisphere; an EDH will not. Only an EDH can cross the midline, and this usually occurs at the vertex in the setting of a venous EDH. EDHs are more usually arterial bleeds and associated with temporal skull fractures, which disrupt the adjacent middle meningeal artery. The latter are typically biconvex; SDHs are typically lentiform in shape. Both measure 50–60 HU if acute. Chronic SDHs may be iso- or hypodense to brain, although anaemia or clotting disorders can produce a similar appearance. EDH is a neurosurgical emergency more so than SDH because of the potential lucent period followed by sudden deterioration as arterial bleeding continues after having stripped the dura from the inner table of the skull.
428
A 34-year-old woman presents with a seizure. She has a history of migraine and low mood over the preceding year, and reported occasional episodes of confusion. On examination there is slight left-sided motor weakness. An MRI of brain reveals small multifocal frontal and parietal subcortical white matter T2WI hyperintensities, and a few areas of restricted diffusion in the right cerebral hemisphere on DWI. What is the most likely diagnosis? A. Multiple sclerosis (MS). B. SLE. C. Small vessel ischaemia. D. Susac syndrome. E. Lyme disease.
B. SLE. The combination of white matter hyperintensities and focal infarcts (indicated by the areas of restricted diffusion) in combination with established neurology and neuropsychiatric symptoms, in a woman of child-bearing age, strongly suggests the diagnosis of cerebral lupus. SLE is an autoimmune disorder that affects many organ systems, including the CNS. Clinical presentation can include migraine, seizures, stroke, chorea, psychosis, mood disorder, acute confusional state, cognitive dysfunction, transverse myelopathy, cranial neuropathies, and aseptic meningitis. The CNS pathology includes vasculitis, dural venous sinus thrombosis, cerebritis, intracranial haemorrhage, infarction, and infection. Infarction can occur as a result of direct thrombosis as well as embolism from Libman–Sacks endocarditis. Diffuse neuropsychiatric symptoms are attributed to direct neuronal damage mediated by antibodies. The most common imaging fi nding is small multifocal T2WI or FLAIR hyperintense white matter lesions. Focal infarcts and grey matter lesions may also be seen, as well as diffuse steroid responsive subcortical lesions. Acute lesions may enhance. MR angiography and venography may show thrombotic lesions of vessels and dural venous sinus thrombosis, respectively. When a patient with SLE presents with an acute neurological deterioration, it is crucial to image them promptly: initially with unenhanced CT to exclude haemorrhage and then with MRI (including post-contrast sequences) to evaluate for stroke or abscess. Note that a negative brain MRI cannot exclude cerebral lupus. Treatment consists of immunosuppression, with anticoagulation for thrombotic events. Stroke is atypical for MS. Small vessel ischaemia should not occur in such a young age without an underlying cause and is not a satisfactory diagnosis. Susac syndrome is a triad of encephalopathy, branch retinal artery occlusions, and hearing loss. It is a microangiopathy of unknown aetiology and results in multiple T2WI hyperintense deep white matter and corpus callosum lesions. Lyme disease results in T2WI hyperintense periventricular white matter lesions, which may enhance, and it may resemble ADEM or MS.
429
A 12-year-old male with a history of gelastic seizures is referred for MRI of the brain. Which of the following statements regarding hamartomas of the tuber cinereum is true? A. No change in size, shape, or signal intensity on follow-up MRI. B. Demonstrate homogenous contrast enhancement. C. Calcifi cation is a common fi nding. D. Hyperintense on T1WI and T2WI, and hypointense on fat suppressed sequences. E. Located in the sella turcica.
A. No change in size, shape, or signal intensity on follow-up MRI. Hypothalamic hamartomas are developmental malformations located in the tuber cinereum of the hypothalamus. The typical patient is male, in the fi rst or second decade of life, presenting with precocious puberty or gelastic seizures. On MRI, they appear as well-defi ned pedunculated or sessile lesions that are iso/mildly hypointense on T1WI and iso/hyperintense on T2WI, with no contrast enhancement or calcifi cation. Lack of interval change strongly supports the diagnosis.
430
A 50-year-old male undergoes an MR carotid angiogram on which an incidental soft-tissue mass is noted in right parapharyngeal soft tissue. The mass displaces the right parapharyngeal space anteromedially. What is the location of the soft-tissue mass? A. Masticator space. B. Carotid space. C. Retropharyngeal space. D. Mucosal space. E. Parotid space.
E. Parotid space. Loss of symmetry and displacement of the parapharyngeal space are useful for lesion identifi cation and localization in the parapharyngeal soft tissues. A thorough knowledge of the anatomical relationship between the spaces is essential. The parapharyngeal space is shaped like an inverted pyramid with the apex pointing inferiorly toward the greater cornu of the hyoid bone and the skull base demarcates the base superiorly. A lesion arising from the parotid space displaces the fat in the parapharyngeal space anteromedially. A lesion in the masticator space displaces the parapharyngeal fat posteromedially. Carotid space lesions displace it anteriorly, mucosal space lesions displace it posterolaterally, and retropharyngeal space lesions displace it anterolaterally. Posterior displacement of the carotid space or parapharyngeal fat completely surrounding a lesion localizes it to the parapharyngeal space.
431
A 52-year-old woman presents with gradually increasing gait disturbance and lower limb sensory symptoms. An MRI of her spine is performed and this shows an anteriorly placed intradural, but extramedullary spinal mass. It is fairly markedly low signal on T1WI and T2WI, and shows only miminal patchy enhancement post administration of intravenous gadolinium. What is the most likely diagnosis? A. Neurofi broma. B. Schwannoma. C. Lymphoma. D. Metastasis. E. Meningioma.
E. Meningioma. Spinal meningiomas are typically iso- to hypointense on T1WI and slightly hyperintense on T2WI. There is usually strong and homogeneous enhancement with gadolinium. However, some meningiomas may contain calcifi cation and are typically the only intradural extramedullary tumours to do so. Some meningiomas can be heavily calcifi ed and such a meningioma is being described in the question. These will remain dark on all MRI sequences and demonstrate only little contrast uptake (in the non-calcifi ed areas). Schwannomas, neurofi bromas, and metastases would not typically be hypointense on T2WI. Meningeal lymphomas are very rare and usually manifest as diffuse thickening of nerve roots and/ or multiple enhancing nodules.
432
You are asked to protocol an MRI scan that is specifi cally being performed to look for vertebral metastatic disease. The radiographer complains that you have asked for too many sequences. Which of the following sagittal sequences is likely to be least helpful for the purposes of your examination? A. STIR. B. T2 fast SE with fat saturation. C. T2 fast SE. D. T1 fast SE. E. T1 GE out of phase.
C. T2 fast SE. T2 fast SE is probably the least useful sequence when specifi cally looking for vertebral marrow deposits because the metastases are less conspicuous, typically being high signal on a background of high-signal fatty marrow. On STIR and T2 fast SE with fat saturation, the metastases typically stand out as being of increased signal on a background of dark marrow because of the fat saturation techniques. On T1 fast SE sequences, the metastases typically stand out as being low signal on a background of high-signal fatty marrow. Finally, T1 GE out–of-phase imaging is also good for looking for vertebral metastatic disease. This is a sequence with a specifi c echo time corresponding to the time it takes for water and fat protons to move exactly 180° out of phase. In the normal adult human, the medullary bone of the vertebral bodies contains approximately equal amounts of water and fat protons. In out-of- phase conditions, the signal of both will cancel out, leaving the vertebrae completely black. In the case of vertebral pathology, however, the signal will increase and, as such, vertebral metastases (or other lesions) will clearly stand out
433
A 44-year-old woman presents with severe facial injuries following an RTA. CT of the facial bones demonstrates multiple maxillofacial fractures in keeping with a Le Fort confi guration. Which of the following statements regarding Le Fort fractures is false? A. B. Any combination of Le Fort I, II, and III fractures can occur. Disruption of pterygoid plates from the posterior maxilla is an essential fi nding in Le Fort fractures. C. D. E. Le Fort fractures by defi nition refer to fractures involving the maxilla bilaterally. Craniofacial separation is noted in Le Fort III pattern. Le Fort fracture associated with a palate fracture will result in widening of the maxillary arch.
C. Le Fort fractures by defi nition refer to fractures involving the maxilla bilaterally. Separation of all or a portion of the maxilla from the skull base is described as a Le Fort fracture. This can be unilateral when it is associated with sagittal or parasagittal fractures of the palate. Le Fort fractures by defi nition involve the posterior maxillary buttress at the junction of the posterior maxillary sinus and the pterygoid plates of the sphenoid. This may be either through the pterygoid plates or through the posterior walls of the maxillary sinus. Once a pterygomaxillary disruption has been identifi ed, the remaining facial buttresses are inspected to identify the type of Le Fort fracture. In Le Fort I fracture, the maxillary arch will move in relation to the rest of the face and skull. In Le Fort II fracture, the entire maxilla will move in relation to the skull base. In Le Fort III, there is complete craniofacial separation. Any combination of Le Fort I, II, and III can occur. Posterior extension of Le Fort fracture into the hard palate results in widening of the maxillary arch and dental malocclusion.
434
A 45-year-old female undergoes an MRI of the pituitary that demonstrates a kidney-shaped lesion located centrally in the pituitary fossa in the axial plane. It is hyperintense on T1WI and hypointense on T2WI. There is no enhancement following gadolinium administration and no fl uid–fl uid level. What is the diagnosis? A. Rathke cleft cyst. B. Craniopharyngioma. C. Cholesterol granuloma. D. Haemorrhagic adenoma. E. Lipoma.
A. Rathke cleft cyst. These are benign cystic lesions of the pituitary fossa derived from the Rathke pouch. They are usually asymptomatic. Rathke cleft cysts are located in the midline between the anterior and posterior pituitary lobes and have a characteristic kidney shape on axial images. They are homogenously hyperintense on T1WI, due to high protein concentration, and hypointense on T2WI, due to low intracystic water content. They do not enhance following contrast administration. Absence of a fl uid–fl uid level is helpful in differentiating Rathke cleft cyst from hemorrhagic adenoma. Acute haemorrhage has heterogenous signal on T2WI and may demonstrate thin peripheral enhancement on T1WI. Craniopharyngiomas have variable solid, cystic, and calcifi ed components. They demonstrate heterogenous enhancement following contrast. They may be intrasellar or suprasellar. A pseudo-fl uid–fl uid level may occasionally be seen in craniopharyngioma. Lipoma and cholesterol granuloma are hyperintense on both T1WI and T2WI.
435
A 5-year-old girl with a clinical suspicion of retropharyngeal abscess is referred for MRI of the neck. Which of the following features on MRI is useful in differentiating retropharyngeal abscess from retropharyngeal suppurative lymph node? A. Enhancing wall. B. Rounded or ovoid confi guration. C. Mass effect. D. Filling of retropharyngeal space from side to side. E. Primary infection source such as otitis media or tonsillitis.
D. Filling of retropharyngeal space from side to side. Understanding the retropharyngeal space anatomy is crucial in differentiating retropharyngeal space abscess and retropharyngeal suppurative lymph node. The retropharyngeal space is bounded by visceral fascia covering the pharynx and oesophagus anteriorly, the prevertebral fascia covering the prevertebral muscles posteriorly, and the carotid sheaths laterally. A retropharyngeal suppurative lymph node is unilateral, whereas a retropharyngeal abscess fi lls the entire retropharyngeal space from side to side. The differentiation is important because many cases of suppurative lymph nodes do not have purulent material at surgery. The treatment for suppurative lymph nodes is a trial of antibiotics if the patient is stable. Surgical drainage is considered if there is progression or if the suppurative lymph node is large at presentation. The volume of central low density is a better predictor of purulence than the mere presence of rim enhancement and low-density centre.
436
A 62-year-old man presents with tremor and incontinence. Examination reveals bradykinesia and gait ataxia. He is also noted to have a reduced mini mental state examination (MMSE) and postural hypotension. He has an MRI scan of brain as part of the diagnostic workup. On the axial T2WI a cruciform hyperintensity in the pons is noted. Which of the following is the most likely diagnosis? A. Parkinson’s disease. B. Multisystem atrophy (MSA). C. Progressive supranuclear palsy (PSP). D. Cryptobasal degeneration. E. Lewy body dementia.
B. Multisystem atrophy (MSA). T2WI cruciform hyperintensity or the ‘hot cross bun’ sign within the pons, is suggestive of MSA, although it is not specifi c, as it also occurs in spinocerebellar ataxia. Other MRI features include hyperintensity within the putamen. Patients with MSA typically have parkinsonian features poorly responsive to levodopa therapy and autonomic disturbance. MRI features of Parkinson’s disease are generally non-specifi c, but narrowing of the pars compacta of the substantia nigra may be seen on T2WI. Substantia nigra atrophy is also a feature of Lewy body dementia. T2WI hypointensity of the putamen due to iron deposition is a feature of progressive supranuclear palsy.
437
A 56-year-old man with chronic alcohol dependence presents with progressive cognitive impairment, gait disturbance, and signs of interhemispheric disconnection. An MRI scan of brain demonstrates increased T2WI signal lesions without mass effect within the corpus callosum and dorsal part of the external capsule. Which of the following is the most likely diagnosis? A. Wernicke’s encephalopathy. B. Osmotic myelinolysis. C. Marchifava–Bignami disease. D. Alcohol withdrawal syndrome. E. Chronic hepatic encephalopathy.
C. Marchifava–Bignami disease. This is a rare complication of chronic alcohol consumption characterized by demyelination and necrosis of the corpus callosum, although other white matter tracts (such as the external capsule) may be involved. Bilateral and symmetrical T2WI hyperintensities within the thalami, periaqueductal grey, and mammillary bodies are seen in Wernicke’s encephalopathy. Osmotic myelinolyis is usually secondary to rapid changes in serum sodium and results in increased T2WI signal within the central pons. Extrapontine myelinolysis is rare. In alcohol withdrawal syndrome, patients present with seizures and delirium tremens. MRI may show volume loss in the temporal cortex and anterior hippocampus. Hepatic encephalopathy secondary to hepatocellular failure may produce T1WI hyperintensity within the caudate nucleus, globus pallidus, putamen, and anterior midbrain due to increased concentration of manganese.
438
A 74-year-old woman is admitted with acute left-sided hemiplegia of 2 hours onset. On assessment by the stroke team she is deemed suitable for thrombolysis and referred for CT. As part of your institution’s work-up, CT perfusion (CTP) is performed following the unenhanced study. Which of the following CTP fi ndings with regards to cerebral blood fl ow (CBF), cerebral blood volume (CBV) and mean transit time (MTT) are consistent with infarction? A. Decreased CBF/decreased CBV/increased MTT. B. Decreased CBF/increased CBV/increased MTT. C. Decreased CBF/decreased CBV/decreased MTT. D. Increased CBF/increased CBV/decreased MTT. E. Increased CBF/decreased CBV/decreased MTT.
Decreased CBF/decreased CBV/increased MTT. In identifying the ischaemic penumbra, CT perfusion offers promise in improved patient selection for thrombolysis beyond a rigid time window. In the infarct core (tissue which is not salvageable) both CBF and CBV are decreased with a corresponding increase in MTT. Penumbral tissue (which is potentially recoverable by thrombolysis) exhibits a CBF/CBV mismatch with an increased CBV, but decreased CBF (and increased MTT). An increase in CBF and CBV with decreased MTT is a feature noticed in tumours secondary to angiogenesis and microvascular permeability.
439
A 36-year-old man is admitted following a seizure. Unenhanced CT demonstrates a right frontal mixed attenuation lesion, which avidly enhances post contrast. Multiple fl ow voids are seen on MRI of brain. What fi nding on catheter angiography differentiates arteriovenous malformation (AVM) from other vascular lesions of the brain? A. Early venous drainage. B. Arterial stenoses of feeder vessels. C. External carotid transdural supply. D. Dilated medullary veins (caput medusa). E. Dilated cortical veins.
A. Early venous drainage. Brain AVMs are abnormal connections between arteries that would normally supply the brain tissue and veins that normally drain the brain, resulting in shunting with an intervening network of vessels within the brain parenchyma. The fi nding of early venous drainage is important in differentiating brain AVMs from other vascular lesions. Cortical venous drainage may be seen in superfi cial lesions. Recruitment of transdural supply is observed in large lesions, although this is a more typical feature of proliferative angiopathy, which is a diffuse type of AVM. Classically, in this condition, normal brain parenchyma is interspersed between the abnormal vessels. Stenoses of feeder vessels are also often identifi ed in proliferative angiopathy. The caput medusa of dilated medullary veins is a feature of DVAs, a normal variant. Dilated cortical veins are seen in dural arteriovenous fi stulas (DAVFs), which are abnormal connections between arteries that normally supply the meninges (but not the brain) and small venules within the dura.
440
A 16-year-old male with a history of epilepsy is investigated via MRI. Axial T2WI demonstrates a cystic space within the left frontal lobe isointense to CSF. This is causing local mass effect and there is adjacent enlargement of the left lateral ventricle. What is the most likely diagnosis? A. Porencephalic cyst. B. Arachnoid cyst. C. Schizencephaly. D. Hydranencephaly. E. Ependymal cyst.
A. Porencephalic cyst. Porencephaly is a congenital/acquired cystic cavity within the brain parenchyma with adjacent enlargement of the lateral ventricle. They develop in utero or early infancy. Arachnoid cysts are also CSF isointense, but are extra-axial, displacing the brain away from the adjacent skull. Ependymal cysts are intraventricular and the surrounding brain is usually normal. Schizencephaly is characterized by an intraparenchymal cleft extending from the ventricular surface to the brain surface lined by gray matter. Hydranencephaly results from an early destructive process of the developing brain. The cranial vault is CSF fi lled with absence of the cortical mantle and ventricles (water-bag brain). Death in infancy is typical.
441
A 32-year-old man presents with schizophrenic-like psychosis and parkinsonian-type movement disorder. There is a family history of neuropsychiatric disturbance. An initial CT is requested which demonstrates heavy bilateral, symmetric calcifi cations within the globus pallidus, thalami, putamen, and cerebellum. There is no enhancement post contrast. Which of the following suggests a diagnosis of Fahr disease over pseudohypoparathyroidism? A. Involvement of the globus pallidus. B. Involvement of the thalami. C. Involvement of the putamen. D. Involvement of the cerebellum. E. Normal calcium-phosphorus metabolism.
E. Normal calcium-phosphorus metabolism. Fahr disease is a rare degenerative neurological disorder characterized by extensive bilateral basal ganglia calcifi cations that can lead to progressive dystonia, parkinsonism, and neuropsychiatric manifestations. CT has higher diagnostic specifi city for basal ganglia calcifi cation over MRI. The distribution of calcifi cations is similar in endocrinological disorders such as hyperparathyroidism, hypoparathyroidism, and pseudohypoparathyroidism. The calcium– phosphorus metabolism is usually normal in Fahr disease. In pseudohypoparathyroidism the serum calcium is low with an appropriately high parathyroid hormone (PTH), due to PTH resistance.
442
A 32-year-old female is referred to neurology complaining of visual disturbance and headache. She is 4 months postpartum. On examination a bitemporal hemianopia is noted. Hormonal testing reveals hypoadrenalism and hypothyroidism. A dedicated MRI of her pituitary gland is requested. Which of the following features is suggestive of autoimmune hypophysitis over pituitary adenoma? A. Asymmetric pituitary enlargement. B. Heterogenous gadolinium enhancement. C. Loss of the posterior pituitary bright spot. D. Sphenoid sinus mucosal thickening. E. Age >30.
C. Loss of the posterior pituitary bright spot. Autoimmune hypophysitis (AH) and non-secreting pituitary adenomas can only be differentiated with certainty on histology. As a result, approximately 40% of patients with AH are misdiagnosed as having pituitary macroadenoma and undergo unnecessary surgery. Hormone production is compromised in both conditions, although a history of infertility is common with adenomas, whereas patients with AH typically achieve spontaneous pregnancy. In an attempt to develop a scoring system to differentiate the two conditions, a recent study found that features signifi cantly associated with AH over adenoma were age <30, relation to pregnancy, homogenous gadolinium enhancement, loss of the posterior pituitary bright spot, and enlarged stalk. Features consistent with adenoma were asymmetrically enlarged pituitary, size >6 cm3, and associated sinus mucosal thickening. The normal posterior pituitary gland is bright on T1WI due to the rich content of vasopressin neurosecretory granules. This is frequently lost in AH due to direct autoimmune involvement of the neurohypophysis, whereas it is conserved in the majority of adenomas, even when displaced by large tumour size.
443
E. coli, with or without an underlying diagnosis of diabetes mellitus, is the most common pathogen behind many urological conditions. All of the following conditions are most commonly due to E. coli infection except one, which one? A. Xanthogranulomatous pyelonephritis (XGP). B. Emphysematous pyelonephritis. C. Pyonephrosis. D. Pyeloureteritis cystica. E. Malakoplakia.
A. Xanthogranulomatous pyelonephritis (XGP). This is most commonly secondary to proteus infection.
444
A 50-year-old male patient on long-term analgesia presents with a history of macroscopic haematuria. An intravenous urogram is requested. The preliminary fi lm is unremarkable. Following intravenous contrast administration, there is a delay in excretion on the right side. Subsequent images demonstrate a fi lling defect in the distal ureter with proximal dilatation. The ureter immediately distal to the fi lling defect is also mildly dilated, but normal in calibre further down. What is the diagnosis? A. Non-radioopaque calculus. B. Transitional cell carcinoma (TCC). C. Blood clot. D. Sloughed papilla. E. Pyeloureteritis cystica.
B. Transitional cell carcinoma (TCC). The appearance of ureteral dilatation around and below an intraluminal fi lling defect is described as the ‘goblet’ sign. This sign indicates that the fi lling defect is caused by a chronic process, which allows dilatation of the ureter immediately below to accommodate the lesion. In addition to TCC, this appearance may rarely be seen with metastatic disease or endometriosis. Chronic analgesic use is a risk factor for TCC. Pyeloureteritis cystica appears as multiple small fi lling defects in the renal pelvis and ureter, typically seen in diabetics.
445
A 62-year-old woman presents with recurrent urinary tract infections (UTIs) and a pelvic/renal tract ultrasound is performed. This demonstrates normal kidneys and bladder, but there is a 5-cm solid, hypoechoic mass seen arising from the right ovary. There is acoustic shadowing caused by the mass, but a subsequent CT scan does not show any calcifi cation within the mass or any metastatic disease. What is the most likely cause of the ovarian mass? A. Sertoli–Leydig cell tumour of the ovary. B. Cystadenocarcinoma of the ovary. C. Fibrothecoma of the ovary. D. Granulosa cell tumour of the ovary. E. Embryonal cell carcinoma of the ovary.
C. Fibrothecoma of the ovary. Ovarian fi broma is the most common sex-cord stromal neoplasm and is almost always benign. Occasionally there will be histologic features of both fi broma and thecoma, giving rise to the term ‘fi brothecoma’. These are typically solid lesions on ultrasound and have marked acoustic shadowing in 18–52% of cases. This shadowing is not secondary to calcifi cation, but is related to marked attenuation of sound by the dense hypoechoic mass itself. This can be a useful sign for identifying fi brothecoma on ultrasound. Classifi cation of ovarian neoplasms is based on histologic features and typically includes the general categories of epithelial, germ cell, sex cord-stromal, and metastatic neoplasms. Cystadenomas/adenocarcinomas belong to the epithelial group of malignancies and make up over 90% of primary ovarian malignancies. Sertoli–Leydig cell tumours, granulosa cell tumours, and fi brothecomas belong to the sex cord- stromal group of neoplasms. Sertoli–Leydig cell tumours may be subtle on ultrasound, as many are small and solid. They can secrete androgens and cause virilization. Granulosa cell tumours often have a sponge-like consistency and may produce oestrogen and cause endometrial disease. Dysgerminomas and embryonal cell carcinomas are very rare germ cell neoplasms. The most common germ cell neoplasm is of course a mature cystic teratoma (dermoid cyst). A hyperechoic area (not usually as intensely echogenic as calcifi cation) with acoustic shadowing is highly predictive of this lesion. Hyperechoic lines and dots, sometimes known as the dermoid mesh, are also highly predictive of a mature teratoma. Less common associated signs include a fl uid–fl uid level and fl oating globules. Calcifi cation can be seen, but is not specifi c to dermoids, as other ovarian neoplasms can calcify. Brown DL, Dudiak KM, and Laing FC.
446
A 36-year-old woman undergoes MRI of the pelvis for assessment of pelvic pain. She had a previous hysterectomy due to post-partum haemorrhage and thus transvaginal ultrasound (TVUS) is not an option. Abdominal ultrasound is technically diffi cult due to body habitus and the ovaries cannot be visualized. MRI reveals a 7-cm left adnexal lesion of predominantly intermediate and high signal on T2WI, but with low-signal components within. On T1WI, there is layering of low and high signal, with suppression of the high signal on the T1WI with fat saturation. What do you advise in your report? A. Referral for chemoradiotherapy. B. Referral for laparoscopy for staging. C. Follow up in 3 months’ time. D. Referral for surgery. E. No follow-up required.
D. Referral for surgery. The fi ndings are consistent with a mature cystic teratoma or dermoid cyst. They are derived from all three germ cell layers and thus contain fat and may contain desquamated epithelium, skin, hair, and teeth. The classical fi nding is of high T1WI signal, which suppresses on fat saturation imaging. They can often have a fat–fl uid level. The low signal on T2WI may represent tooth or calcifi cation. Soft-tissue protuberances represent Rokintansky nodules or ‘dermoid plugs’ of sebaceous material. It is the most common ovarian tumour, accounting for 20% of all ovarian neoplasms. Dermoids can be bilateral in 8–25% of cases. The chance of malignant transformation is low but surgical resection is indicated, not just for relief of symptoms, but because of the risk of torsion (in 4–16% of cases) or rupture (the latter is rare but can lead to chemical peritonitis). Adjuvant treatment need not be considered unless the case is complicated by malignant transformation.
447
A 45-year-old male presents with severe epigastric pain radiating to the back. Blood tests reveal elevated serum amylase and calcium. A CT scan of abdomen demonstrates peripancreatic infl ammatory stranding, renal medullary nephrocalcinosis, and sacro-iliac joint erosions. What further investigation(s) would you recommend? A. Serum parathyroid hormone assay and 99mTc sestamibi scan. B. Serum parathyroid hormone assay and 111In pentetreotide scan. C. Serum parathyroid hormone assay and meta-iodobenzyl-guanidine (MIBG) scan. D. Serum parathyroid hormone and 99mTc pertechnetate scan. E. Serum parathyroid hormone assay and 201Tl scan.
A. Serum parathyroid hormone assay and 99mTc sestamibi scan. Clinical fi ndings, blood tests, and CT of the abdomen are diagnostic of pancreatitis. The most common causes for pancreatitis are alcohol and choledocholithiasis. Rarely, it may be caused by hyperparathyroidism. The associated fi ndings on CT are suggestive of hyperparathyroidism, therefore further assessment with serum parathyroid hormone assay and 99mTc sestamibi scintigraphy is indicated. 99mTc sestamibi washes out more rapidly from the thyroid gland than from hyperfunctioning parathyroid glands and therefore it can be used on its own. MIBG, a noradrenaline analogue, is used in the evaluation of neuroblastomas and paragangliomas. 99mTc pertechnetate is taken up by the thyroid gland only and is therefore not useful on its own in parathyroid imaging. However, it can be used in combination with 201Tl, which is taken up by both thyroid and parathyroid. Subtracting the two scintigrams allows parathyroid localization.
448
A 32-year-old asymptomatic woman who is BRCA1 positive undergoes breast cancer surveillance via MRI. A lesion within the left breast is identifi ed. Which of the following MRI features is the most predictive for malignancy? A. Irregular margin. B. T2WI signal hyperintensity. C. Progressive enhancement curve on dynamic T1WI post contrast. D. Plateau enhancement curve on dynamic T1WI post contrast. E. Washout enhancement curve on dynamic T1WI post contrast.
A. Irregular margin. A woman over the age of 30 years who is a BRCA1 or BRCA2 carrier should be offered MRI annually for breast cancer surveillance. The description of the margin of the mass is the most predictive feature of the breast MR image interpretation. Irregular or spiculated margins have a positive predictive value of 84–91%. T2W signal hyperintensity is suggestive of benign pathology, but not in the setting of an irregular or spiculated mass. There is overlap in enhancement kinetics between benign and malignant disease, and thus reliance on kinetic assessment alone is not recommended. Progressive enhancement (type I) is associated with benign pathology, whereas plateau (type II) and washout (type III) curves are suggestive of malignant disease. Due to the importance of lesion morphology, the MRI technique should focus on optimizing high spatial and temporal resolution.
449
A 73-year-old male diabetic patient, with poorly controlled hypertension, is referred for renal Doppler ultrasound due to an episode of fl ash pulmonary oedema. He has a history of stage 3 chronic kidney disease. The ultrasound shows a small left kidney, which measures 5 cm in bipolar diameter. The right kidney is also small, measuring 6 cm. The resistive indices measure 0.9 on both sides. The peak systolic velocity is 130 cm/s on the left and 150 cm/s on the right. Which interventional treatment would be recommended for this patient? A. Renal artery angioplasty on left side. B. Renal artery stenting on left side. C. Renal artery stenting on right side. D. Bilateral renal artery stenting. E. No intervention.
E. No intervention. The factors described are all indicators of poor outcome following renal artery intervention. Reduced renal size bilaterally indicates advanced bilateral renal disease, unlikely to respond to intervention. The renal ultrasound Doppler patterns are also not suggestive of renal artery stenosis, which is indicated by a peak systolic velocity of greater than 180 cm/s. Resistive indices of greater than 0.7 indicate a likelihood of improvement after intervention.
450
You are asked to perform an antenatal ultrasound examination and note that the placenta has an unusual morphology. You see an additional lobule, which is separate from the main bulk of the placenta. What is this variant of placental morphology known as? A. Circumvallate placenta. B. Bilobed placenta. C. Placenta membranacea. D. Succenturiate placenta. E. Placenta accreta.
D. Succenturiate placenta. This is an additional lobule separate from the main bulk of the placenta. The signifi cance of this variant is the rupture of vessels connecting the two components or retention of the accessory lobe with resultant post-partum haemorrhage. Circumvallate placenta has a chorionic plate smaller than the basal plate, with associated rolled placental edges. There is known to be an increased risk of placental abruption and haemorrhage with this type of placenta. A bilobed placenta is a placenta with two evenly sized lobes connected by a thin bridge of placental tissue. This has no known increased risk of morbidity. Placenta membranacea is a thin membranous structure circumferentially occupying the entire periphery of the chorion. There is an increased risk of placenta praevia, as a portion of the placenta completely covers the internal os. Placenta acreta is not a variant of placental morphology. It occurs when there is superfi cial invasion of the chorionic villi of the placenta into the basalis layer of the uterine wall. Deeper invasion of the myometrium is termed ‘placenta increta’. Even deeper invasion involving the serosa or adjacent pelvic organs is termed ‘placenta percreta’. The risk of this is catastrophic intrapartum haemorrhage at the time of placental delivery.
451
A 36-year-old male patient presents with abdominal pain. He has a history of hypertension and obesity. A CT of abdomen reveals a 6-cm right adrenal mass, which shows heterogenous but peripheral enhancement, necrosis, and some calcifi cation. There is early invasion of the IVC. The left adrenal gland is atrophied. What is the most likely diagnosis? A. Neuroblastoma. B. Adrenal cortical carcinoma. C. Myelolipoma. D. Adrenal adenoma. E. Phaeochromocytoma.
B. Adrenal cortical carcinoma. The clinical picture is one of undiagnosed Cushing’s syndrome with obesity and hypertension. In this case it is adrenocorticotropic hormone (ACTH) independent Cushing’s, as the negative feedback from the cortisol producing adrenal carcinoma causes reduction in ACTH levels and atrophy of the contra-lateral, normal adrenal gland. Adrenal adenoma can cause Cushing’s syndrome, but the features described point to adrenal carcinoma. They have a bimodal distribution (fi rst and fourth decades). On average 55% are functional, manifesting with Cushing’ syndrome, feminization, virilisation, or a mixture of these. Hypertension is common in all syndrome types. The majority of masses measure more than 6 cm. They are inhomogenous on unenhanced CT, owing to necrosis. They enhance heterogeneously, often peripherally, with a thin rim of enhancing capsule in some cases. In 19–33% of cases calcifi cation or microcalcifi cations have been identifi ed. The liver is the most common metastatic site, followed by the lung and lymph nodes. Direct extension and tumour thrombus can also occur. Compression of the IVC can lead to presentation with abdominal pain, lower extremity oedema or pulmonary embolism. Neuroblastoma is a disease of childhood. Myelolipoma is a relatively uncommon benign adrenal mass containing fat and haemopoeitic tissue. Phaeochromocytoma is classically brightly enhancing, but can have a variety of CT appearances. It would explain hypertension, but not atrophy of the contra-lateral adrenal gland. Phaeochromocytoma rarely invades the IVC.
452
A 24-year-old woman presents to the symptomatic breast clinic with a palpable left-sided breast lesion. There is no family history of breast cancer. Clinical examination reveals a smooth, relatively mobile 2-cm lesion within the left upper quadrant. Ultrasound depicts a well-defi ned oval hypoechoic lesion with an echogenic capsule following the tissue planes. No malignant features are present. The patient states that she has a phobia of needles. What should be the next step in this patient’s management? A. Reassurance and discharge with advice. B. Correlation with mammography. C. Ultrasound guided core biopsy. D. Ultrasound guided FNA. E. Referral for MRI.
A. Reassurance and discharge with advice. The clinical and radiological fi ndings in this case are typical for fi broadenoma. Standard practice for investigating breast lumps involves triple assessment with clinical examination, imaging with ultrasound, and tissue diagnosis (with either cytology or histology). However, in women under the age of 25 who present with a clinically and radiologically benign lump, biopsy is not needed unless there is overriding clinical concern. To be assessed as defi nitely benign on ultrasound, there should be no malignant features (spiculation, angular margins, acoustic shadowing, calcifi cation, and marked hypoechogenicity) and the lesion should follow tissue planes (wider than it is tall). The ultrasound should also be performed by an experienced operator. The patient should be advised to seek further assessment if there is any increase in size or change to the mass.
453
A 40-year-old male with a history of haematuria undergoes CT urography. Initial non-contrast scan demonstrates right-sided medullary nephrocalcinosis. Following intravenous contrast administration, a striated ‘paintbrush’ appearance of the renal medulla is noted. The left kidney is unremarkable. What is the diagnosis? A. Hyperparathyroidism. B. Renal tubular acidosis. C. Medullary sponge kidney. D. Sarcoidosis. E. Multiple myeloma.
C. Medullary sponge kidney. Hyperparathyroidism, renal tubular acidosis, and medullary sponge kidney are the three most common causes of medullary nephrocalcinosis. The former two conditions are associated with hypercalciuria that results in uniform medullary nephrocalcinosis. Sarcoidosis and multiple myeloma are associated with hypercalcemia resulting in bilateral nephrocalcinosis. Medullary sponge kidney can affect the kidney segmentally, unilaterally, or bilaterally, therefore unilateral nephocalcinosisis is suggestive of medullary sponge kidney. Medullary sponge kidney is characterized by cystic dilatation of collecting tubules. Urine stasis within the dilated tubules predisposes to infection and calculus formation within the dilated tubules or urinary tracts. On excretory urogram, contrast within the dilated tubules produces a striated ‘paintbrush’ appearance of the renal pyramids.
454
A 50-year-old man attends his GP feeling generally lethargic. The GP organizes blood tests and these reveal renal impairment. A subsequent ultrasound examination shows bilateral hydronephrosis without obvious cause. A CT scan of the abdomen then demonstrates that the hydronephrosis is secondary to bilateral ureteric obstruction from abnormal retroperitoneal soft tissue, intimately related to the aorta and IVC. Which of the following features on CT would suggest that the soft tissue is more likely due to retroperitoneal fi brosis, rather than a malignant cause? Nodular contour to the soft tissue. Contrast enhancement of the soft tissue. More severe hydronephrosis in the kidneys. Close application of the soft tissue to the adjacent vertebrae. Soft-tissue extension above and below the level of the renal hila.
D. Close application of the soft tissue to the adjacent vertebrae. Unfortunately attempts to consistently distinguish benign retroperitoneal fi brosis (RPF) from malignancy are fraught with danger, but there are certain CT fi ndings which are more commonly seen in one or other of these conditions. Malignancy tends to be larger and bulkier, displaying mass effect and displacing the aorta and IVC anteriorly from the spine and the ureters laterally. The purely fi brotic process of benign RPF tends to tether these structures to the adjacent vertebrae. Malignancy is more likely to extend cephalad to the renal hila, with benign RPF remaining caudal to the hila. Neoplasia also more typically has a nodular outline, whereas benign RPF usually manifests as a plaque-like density. There are, of course, exceptions to both these features. Contrast enhancement is not a reliable feature for distinguishing benign RPF from malignancy, as both malignancy and active RPF can enhance with contrast. Similarly, the degree of hydronephrosis caused is not a good distinguisher.
455
A 35-year-old female presents with a history of menorrhagia. MRI of pelvis demonstrates a fi broid uterus for which treatment with high-intensity focused ultrasound (HIFU) is proposed. What is the principle mechanism of action of HIFU? A. Coagulation necrosis. B. Apoptosis. C. Cavitation. D. Microstreaming. E. Radiation forces.
A. Coagulation necrosis. HIFU is a non-invasive method to treat solid tumours or haemorrhage. As HIFU is essentially ultrasound, it requires an acoustic window to transmit ultrasound energy and is subject to similar artefact. The principle effect of HIFU is heat generation from absorption of acoustic energy. This causes coagulation necrosis within seconds. Hyperthermia also induces apoptosis, which can be an important delayed effect in tissue exposed to lower energy HIFU. This mechanism is also a potential limitation of HIFU as adjacent tissue may be at risk. Mechanical effects such as cavitation and microstreaming are also seen with the use of higher ultrasound intensity
456
A 56-year-old male patient has presented to the urologists with a PSA of 17 ng/ml. He does not have a mass palpable on DRE. Two separate TRUS investigations, with a total of eight biopsies, have failed to yield a tissue diagnosis. The urologists have asked you to carry out an MRI to help guide their future biopsies. As is routine in these diffi cult cases, magnetic resonance spectroscopy (MRS) is carried out on suspicious areas to provide extra information. The MRI identifi es an area of low signal in the anterior peripheral zone. This reveals an elevated choline and creatine peak, and a reduced citrate peak. Which of these features is suggestive of carcinoma? A. Elevated choline. B. Reduced polyamine peak. C. Reduced citrate peak. D. None of them, MRS is only sensitive in the transition zone. E. All of them.
E. All of them. MRI is not a primary investigation in the diagnosis of prostate cancer but it is used in staging known disease. Occasionally, in a patient with a high risk of prostate carcinoma, as in this case, when the urologists have repeatedly failed biopsies, MRI can be used to help guide biopsy. In this setting MRS can also be used to increase confi dence in the diagnosis. As with all prostate imaging for cancer, the results are more reliable in the peripheral zone due to the variability of appearances in the transitional zone. Choline is elevated in prostate cancer, and is thought to refl ect increased cell membrane turnover. Creatine, also detected on MRS, is unchanged. Polyamine is reduced. Citrate, which is stored in normal prostatic cells, is reduced, presumably because of reduced normal function within cancerous cells. The (creatine + choline)/citrate ratio has been used to help discriminate prostate cancer from normal prostate. The role of MRS in the transitional zone is unclear
457
A 60-year-old man has an unenhanced CT scan of renal tracts for suspected right renal colic. The examination is normal apart from an exophytic rounded lesion at the midpole of the left kidney, which is denser than adjacent renal parenchyma. You elect to perform an intravenous contrast- enhanced examination in the nephrographic phase to further evaluate this lesion. Which of the following Hounsfi eld attenuation values would be most appropriate if this lesion is benign? A. 20 pre-contrast attenuation, 30 post-contrast attenuation. B. 30 pre-contrast attenuation, 60 post-contrast attenuation. C. 40 pre-contrast attenuation, 50 post-contrast attenuation. D. 50 pre-contrast attenuation, 80 post-contrast attenuation. E. 60 pre-contrast attenuation, 70 post-contrast attenuation. F. 70 pre-contrast attenuation, 100 post-contrast attenuation.
E. 60 pre-contrast attenuation, 70 post-contrast attenuation. The attenuation of the normal renal parenchyma typically ranges from 30 to 40 HU. That of hyperattenuating renal masses usually is at least 40 HU but no higher than 90 HU on CT without intravenous contrast. Benign cysts are overwhelmingly the most common type of hyperattenuating renal mass and are also known as hyperdense renal cysts. They are usually cysts containing haemorrhage or proteinaceous material. Hyperattenuating cysts should not enhance and therefore cannot be diagnosed with confi dence by using unenhanced CT alone. A proper CT examination includes scanning both before and after the administration of intravenous contrast material. Masses that increase in attenuation by 10 HU or less are considered non-enhancing. Masses that increase in attenuation by more than 10 HU are considered enhancing. However, because the standard deviation of attenuation measurements may be more than 10 HU, an attenuation difference of 20 HU or more is a more specifi c criterion of enhancement.
458
A 2-day-old male neonate with a right-sided abdominal mass is referred for ultrasound of abdomen. Ultrasound demonstrates an enlarged right kidney containing multiple non-communicating cysts of varying size with little normal parenchyma. What is the most common associated abnormality of the contra-lateral kidney? A. Ectopic ureter. B. Pelvi-ureteric junction obstruction. C. Vesico-ureteric refl ux. D. Renal hypoplasia. E. Renal aplasia.
C. Vesico-ureteric refl ux. The abnormality described on ultrasound is multicystic dysplastic kidney (MCDK). It is the most common form of cystic disease in infants and the second most common cause of an abdominal mass in a neonate (after hydronephrosis). Obstruction/atresia of the ureter during the developmental stage is thought to be the etiology. Bilateral MCDK is uncommon, but associated anomalies of the contra-lateral kidney are seen in up to 50% of cases. Vesico-ureteric refl ux (30–40%) is the most common associated anomaly, followed by pelvi-ureteric junction obstruction (10–20%).
459
A 54-year-old woman with no history of major illness is incidentally discovered to have a small, solid enhancing lesion on CT at the lower pole of the right kidney. The CT has been performed pre and post intravenous contrast. The lesion measures 9 mm in size. What is the most appropriate management for this lesion? A. Nephron sparing surgery. B. Percutaneous biopsy. C. Repeat CT in 3–6 months. D. Right nephrectomy. E. Lesion ablation.
C. Repeat CT in 3–6 months. Solid masses smaller than 1 cm are challenging. Firstly, there is a reasonable chance that a very small solid mass is benign. Secondly, it is often diffi cult to characterize a mass smaller than 1 cm as solid and enhancing, despite a meticulous technique using state-of-the-art CT and MR imaging. Thirdly, these masses are often too small to biopsy, therefore when encountering a mass that is believed to be solid and is less than 1 cm in size, it is reasonable to observe them with an initial examination with CT or MR at 3–6 months followed by yearly examinations. A full work-up could ensue when the mass reaches 1 cm in size.
460
A 43-year-old woman with a history of breast carcinoma undergoes a CT of abdomen for abdominal pain and menorrhagia. This reveals an enlarged uterus and she proceeds to MRI. The normal T2WI zonal anatomy of the uterus is preserved. The endometrial stripe is of high T2WI signal and measures 14 mm in diameter, and the myometrium is thickened. Lattice-like enhancement of the high-signal T2WI endometrial area is demonstrated on T1WI post contrast administration. There is no evidence of myometrial invasion. What is the diagnosis most consistent with these fi ndings? A. Intrauterine contraceptive device (IUCD). B. Tamoxifen therapy. C. Lymphoma of the uterus. D. Endometrial stromal sarcoma. E. Pelvic congestion syndrome.
B. Tamoxifen therapy. The normal endometrial stripe is of high T2WI signal and measures 3–6 mm in diameter in the follicular phase and 5–13 mm in the secretory phase. The description of the endometrium in this case is consistent with endometrial hyperplasia, but there is in addition myometrial enlargement. An enlarged uterus is frequently encountered in the presence of endogenous or exogenous hormonal abnormalities. In these cases the uterus usually has normal zonal anatomy, although the signal intensity of the endometrium and myometrium is abnormally increased. However, with tamoxifen, the uterus can display marked zonal anatomy distortion. It is a weak oestrogen agonist and can result in endometrial hyperplasia, polyps, and carcinoma. The fi ndings of multiple cysts or lattice-like enhancement of the endometrium post contrast are encountered frequently in relation to tamoxifen therapy and favour a benign diagnosis. IUCDs are widely used for contraception. A study has found that the IUCD-bearing uterus is enlarged. IUCD placement most likely results in myometrial hypertrophy. The associated fi ndings are symmetrical globular enlargement of the uterine corpus, cervical elongation and enlargement, and diffuse or localized myometrial thickening. The IUCD will usually be seen as a band of low signal intensity within the endometrium. The uterus is rarely the primary site for lymphoma, but when it does occur, the most common manifestation is diffuse symmetrical uterine enlargement with relatively high signal on T2WI and epithelial preservation: the endometrium is usually normal. The myometrium will also lack its standard zonal appearance. There will almost always be associated lymphadenopathy. Endometrial stromal sarcoma invariably exhibits myometrial involvement. Imaging will typically show a large mass that replaces the endometrial cavity and infi ltrates the myometrium: at the very least the endometrial-myometrial border will be obscured. Myometrial involvement is commonly so extensive that a myometrial component predominates. Bands of low signal may be present within the area of myometrial invasion and these correspond to preserved bundles of myometrium at histologic examination. Extension along the vessels, fallopian tubes, or ligaments is another characteristic of the tumour. Pelvic congestion syndrome involves symptoms, including chronic pelvic pain, that are associated with dilated ovarian veins and pelvic varices resulting from left renal vein refl ux. This results in a thickened myometrium containing multiple large signal voids, the latter corresponding to engorged arcuate vessels. Additional fi ndings include varicose veins around the uterus and ovaries with retrograde fi lling of ovarian veins.
461
A 22-year-old male front seat passenger is admitted following a RTA. On examination in A&E, blood is noted at the external urethral meatus and there is swelling of the penis. An urethrogram is performed which demonstrates contrast extravasation below the urogenital diaphragm only. What type of urethral injury does this represent? A. Anterior urethral injury. B. Disruption of the membranous urethra. C. Bladder neck injury extending into the proximal urethra. D. Bladder base injury. E. Penile fracture.
. A. Anterior urethral injury. Urethral injuries are rarely life-threatening but have signifi cant long-term morbidity. Complications include stricture, incontinence, and impotence. The male urethra extends from the bladder base to the external meatus and is divided into the posterior (prostatic and membranous) and anterior (bulbous and penile) urethra. The anterior and posterior urethra are separated by the urogenital diaphragm. The Goldman classifi cation of urethral injury emphasizes anatomic location
462
A 38-year-old patient is referred for an urgent IVU and ultrasound by their GP, who has picked up mild renal impairment on recent blood tests. The ultrasound shows two normal-sized kidneys with no evidence of cortical loss. The IVU shows a normally enhancing renal outline, but the pelvi- calyceal system is abnormal. Some calyces show a non-specifi c clubbed or blunted appearance. Thin tracks are noted extending from other calyces. In a number of calyces there is a fi lling defect noted within a rounded, blunted calyx. From these appearances you are able to make a diagnosis. The patient has a classical history for this condition. From the given options, what is the most typical history? A. Recent history of acute hypotension on a background of dehydration. B. Pyrexia, renal angle pain, and urine culture positive for E. coli. C. Chronic non-steroidal anti-infl ammatory drug (NSAID) analgesic overuse. D. Previously diagnosed refl ux nephropathy. E. History of TCC within bladder.
C. Chronic non-steroidal anti-infl ammatory drug (NSAID) analgesic overuse. The appearances described are those of renal papillary necrosis. This can be due to a number of causes, most commonly analgesic abuse, but also severe renal infection in diabetics, sickle cell disease, haemophilia, and renal vein thrombosis. The causes of chronic renal disease can be subdivided by the effects noted on the renal parenchyma and the papillary/pelvicalyceal system, and whether these are uni- or bilateral. In bilateral cases such as this, no radiological abnormality suggests possible glomerulonephritis, acute tubular necrosis (ATN), acute cortical necrosis, or pyelonephritis. Generalized infi ltration of the renal parenchyma suggests amyloid or malignant infi ltration (e.g. lymphoma or leukaemia). Papillary/pelvicalyceal abnormalities, in the presence of normal parenchyma, indicate papillary necrosis, medullary sponge kidney, or renal TB. Papillary calyceal abnormality with focal parenchymal loss indicates refl ux nephropathy, TB, or calculus. Both parenchymal loss and papillary calyceal abnormalities indicate obstructive nephropathy or severe refl ux nephropathy.
463
A 37-year-old woman presents with a watery vaginal discharge and attends for an MRI of pelvis. She becomes quite claustrophobic at the end of the scan and you are called to assess her as she has been hyperventilating and the radiographers have become concerned. As you reassure her, you notice some peri-oral pigmentation. The MRI reveals a multicystic lesion (high T2WI and low T1WI signal) in the uterine cervix with a solid (low signal T1WI and T2WI) component in the deep cervical stroma. You note from the picture archiving and communication system (PACS) system that a barium enema previously revealed several colonic polyps. What is the likely cause for the MRI fi ndings? A. Malignant melanoma of the cervix. B. Carcinoid tumour of the cervix. C. Cervical pregnancy. D. Minimal deviation adenocarcinoma of the cervix. E. Invasive cervical squamous cell carcinoma.
D. Minimal deviation adenocarcinoma of the cervix. This is also known as adenoma malignum and, as in this scenario, is often associated with Peutz– Jeghers syndrome (characterized by mucocutaneous pigmentation, multiple hamartomatous polyps of the GI tract, and mucinous tumours of the ovary). Adenoma malignum makes up about 3% of adenocarcinoma of the cervix. Its MRI appearances are as described in the question, but the differential diagnosis includes deep nabothian cysts, fl orid endocervical hyperplasia, and even well-differentiated adenocarcinoma. It disseminates into the peritoneal cavity even in the early stage of the disease and its response to radiation or chemotherapy is poor. Cervical squamous carcinoma makes up to 90% of cervical carcinoma. The tumour is of high signal compared to the hypointense cervical stroma, but not cystic as in our vignette. It advances predominantly by direct extension and local spread; haematogenous dissemination is only occasionally seen in the form of hepatic metastases. Carcinoid tumour of the cervix is a subgroup of small cell carcinoma of the cervix. It cannot be differentiated from squamous cell carcinoma of the cervix on MRI fi ndings. Malignant melanoma of the female genital tract accounts for 1–5% of all melanomas. It usually occurs in the vaginal mucosa and occasionally involves the cervix. Malignant melanoma arising in the cervix is very rare (only about 30 reported cases). There is usually high signal intensity on T1WI. The incidence of cervical pregnancy has been increasing, possibly due to the increased number of induced abortions. Reported risk factors include multiparity, prior cervical surgical manipulation, cervical or uterine leiomyomas, atrophic endometrium, and septate uterus. The major symptom is painless vaginal bleeding. At MR it is characterized by a mass with heterogenous signal intensity and a partial or complete dark ring on T2WI sequences. As it contains haematoma, it often consists of some high signal on T1WI.
464
A 48-year-old women presents with shortness of breath and undergoes an HRCT of the chest to assess interstitial changes seen on plain fi lm. She has emigrated from Eastern Europe and knows that she had a gynaecological cancer that was treated there, but is unsure of her treatment. The HRCT reveals unilateral thickened interlobular septa, perilymphatic nodules, and ipsilateral hilar adenopathy. What is the most likely underlying diagnosis? A. Cervical carcinoma. B. Ovarian epithelial carcinoma. C. Endometrial carcinoma. D. Leiomyosarcoma of the uterus. E. Vaginal carcinoma.
A. Cervical carcinoma. This patient has developed lymphangitis carcinomatosis. In 50% of cases the septal thickening is focal or unilateral and this is useful in distinguishing lymphangitis from other causes of septal thickening, such as pulmonary oedema or sarcoidosis. Hilar adenopathy is present in 50% and pleural effusion in 30–50%. The interlobular septal thickening can be smooth (as in pulmonary oedema and alveolar proteinosis) or nodular (also found in sarcoidosis and silicosis). Lymphangitis carcinomatosis usually occurs secondary to the spread of (adeno-) carcinoma, most commonly bronchogenic, breast, and stomach. The mnemonic Certain Cancers Spread By Plugging The Lymphatics (Cervix Colon Stomach Breast Pancreas Thyroid Larynx) is useful. Lymphangitis carcinomatosis is occasionally associated with cervical carcinoma and certainly more so than with the other options presented.
465
A 30-year-old female patient with a history of infertility is referred for an HSG. She has a past history of pelvic infl ammatory disease. HSG reveals multiple small outpouchings from the uterine cavity. What is the diagnosis? A. Salpingitis isthmica nodosa. B. Asherman syndrome. C. Adenomyosis. D. Endometritis. E. Multiple endometrial polyps.
C. Adenomyosis. This is a condition in which the endometrium extends into the myometrium in either a diffuse or a focal distribution. It generally manifests as pelvic pain or abnormal bleeding. It is more commonly detected on MR imaging as thickening of the junctional zone or on ultrasound as diffuse or focal heterogenous myometrium. On HSG, adenomyosis appears as small diverticula extending from the endometrial cavity into the myometrium. Salpingitis isthmica nodosa appears as small outpouchings from the isthmic portion of the fallopian tube. Multiple uterine synechiae (linear fi lling defects in the uterine cavity) associated with infertility is known as Asherman syndrome. Polyps would appear as fi lling defects on HSG.
466
A 54-year-old woman is noted to be hypercalcaemic after complaining of lethargy and abdominal pain. Subsequent biochemical testing reveals an elevated parathyroid hormone. She is referred for scintigraphy with 99mTc sestamibi. Which of the following radiological fi ndings would suggest a diagnosis of parathyroid adenoma? A. Focus of decreased radionuclide activity within the lower pole of the right lobe of thyroid on initial and delayed images. B. Focus of decreased radionuclide activity within the lower pole of the right lobe of thyroid on delayed images only. C. Focus of increased radionuclide activity within the lower pole of the right lobe of thyroid on initial and delayed images. D. Focus of increased radionuclide activity within the lower pole of the right lobe of thyroid on initial images only. E. Focus of increased radionuclide activity within the lower pole of the right lobe of thyroid on delayed images only.
E. Focus of increased radionuclide activity within the lower pole of the right lobe of thyroid on delayed images only. Solitary parathyroid adenoma accounts for 85% of cases of primary hyperparathyroidism, with parathyroid hyperplasia (10%), multiple adenomas (4%), and carcinoma (1%) making up the remainder. When 99mTc MIBI is used for parathyroid imaging, immediate and delayed images of the neck and mediastinum are performed. Parathyroid adenomas may or may not be visualized on initial imaging, but they retain radiopharmaceutical on delayed (1–2 hours) images, whereas the normal thyroid washes out.
467
A 50-year-old man is discovered to have a cystic abnormality at the lower pole of the left kidney during an ultrasound scan performed for right upper quadrant pain. You think this might be a complex cystic lesion and therefore recommend a CT scan of renal tracts. This shows a conglomeration of variable sized cysts at the lower pole of the left kidney, but no capsule or mural irregularities of the cysts. What is the most likely diagnosis? A. Localized cystic renal disease. B. Multilocular cystic nephroma. C. Renal lymphangiomatosis. D. Cystic clear cell carcinoma. E. Multicystic dysplastic kidney.
A. Localized cystic renal disease. This is an uncommon, non-familial, and non-progressive disorder of the kidney characterized by the replacement of all or localized areas of a kidney by multiple variably sized cysts. These cysts form clusters that are separated by thin areas of normal renal parenchyma. The aggregated cysts in localized cystic disease can frequently appear like a multi-septate mass, but they do not form a distinct encapsulated mass and do not show mural irregularities, which are characteristics of cystic neoplasms. Multilocular cystic nephroma in adults is much more common in females (female:male 9:1) and manifests as a multi-loculated cystic lesion with hair-like septa and minimal mural enhancement. It is distinguished from localized cystic disease by the presence of a capsule. Typically extension into the central renal sinus is found and multilocular cystic nephroma is often classifi ed on imaging as Bosniak III lesions. Lymphangioma of the kidney is a rare benign cystic tumour that most often arises from the peripelvic region or renal sinus. It may more rarely arise from the lymphatics of the capsule or cortex. In the diffuse form of lymphangiomatosis, the cystic changes occur diffusely in the renal sinus or perinephric region, with a relatively normal appearing renal parenchyma. Clear cell RCC presents as a solid or a cystic lesion, the cystic variant accounting for 4–15% of all RCCs. These lesions typically have nodular or septal enhancement, distinguishing them from benign cystic renal lesions. Multicystic dysplastic kidney is a congenital maldevelopment in which the kidney is completely replaced by cysts with no normal renal parenchyma remaining.
468
A 28-year-old para 0 + 0 female patient is referred from the gynaecology team for an MRI of pelvis, after presenting with pain in the RIF. On TVUS they have identifi ed enlargement of the right ovary and have raised the possibility of a mass. On MRI, a 2-cm ovoid lesion is demonstrated within the right ovary. It is of high signal on T1WI and T1WI fat saturation sequences, and low signal on T2WI sequences. There is no evidence of a mural nodule or ascites and the uterus is not enlarged. What is the most likely diagnosis? A. Fibrothecoma. B. Endometriotic cyst. C. Brenner tumour. D. Simple follicular cyst. E. Endometroid carcinoma.
B. Endometriotic cyst. Also known as a chocolate cyst, the methaemoglobin within the lesion causes T1 shortening, resulting in increased signal on the T1WI sequence. This high signal remains and becomes more conspicuous on the T1WI with fat saturation sequence. Endometriosis is characterized by the presence of tissue resembling endometrium outside the uterus. The ovaries are the most commonly involved site, and endometriotic cysts usually have a thick fi brotic wall with chocolate-coloured hemorrhagic material. An endometriotic cyst may be high or low signal on T2WI sequences. Chronic cyclical haemorrhage and increased viscosity of the cyst material will produce T2 shortening, leading to the low signal, or ‘T2 shading’. The patient usually presents with cyclical pain. The cysts have a propensity for multicentric growth and are often associated with fi brous adhesions. These latter features increase the MR sensitivity. Small peritoneal implants of endometriosis may be identifi ed elsewhere and their conspicuity is increased by the use of fat saturation techniques. Endometroid and clear cell carcinoma of the ovary are malignancies associated with endometriosis and these represent 17.5 and 7.4% of ovarian carcinomas, respectively. Chocolate cysts with multiple locules, mural foci, or nodules within the cyst are suspicious for malignancy and contrast should be administered if this appearance is seen. The fi ndings in this question are not consistent with a follicular cyst, which would show low T1WI and high T2WI signal. Brenner tumours show low T2WI signal due to their abundant fi brous content and calcifi cation, but also low T1WI signal. Fibrothecomas show intermediate T1WI signal and usually low (although sometimes it can be mixed high and low) T2WI signal. They sometimes show associated uterine enlargement, as they may be oestrogenic.
469
A 33-year-old driver is severely injured in a motor vehicle accident. He develops increasing dyspnoea and hypoxia, and requires intubation. A chest x-ray (CXR) was normal on admission and his pulmonary capillary wedge pressure is normal. A repeat CXR performed at over 24 hours after the trauma is not normal. He is re-imaged during his intensive care unit (ICU) stay and at one point undergoes a computed tomography pulmonary angiography (CTPA), which is negative for pulmonary embolism (PE). The clinical team suspect acute respiratory distress syndrome (ARDS). Which of the following radiographic features is inconsistent with this diagnosis? A. Bronchial dilatation on computed tomography (CT). B. Bilateral heterogenous air-space opacities. C. Diffuse reticular changes. D. Pneumothorax. E. Bilateral pleural effusions.
E. Bilateral pleural effusions. The underlying diagnosis is ARDS. The causes may be direct lung injury (e.g. pneumonia, toxic gas inhalation, aspiration) or indirect lung injury (e.g. trauma, sepsis, pancreatitis). Diagnostic guidelines require a partial pressure of arterial O2/fraction of inspired O2 (PaO2/FiO2) < 200 mmHg and no evidence of left heart failure, and thus the presence of a pleural effusion casts doubt on the diagnosis; the appearance of ARDS can otherwise mimic pulmonary oedema. Bronchial dilatation is frequently seen on CT. The alveolar changes are heterogenous, showing a density gradient in both the cranio-caudal and antero-posterior directions (the dorsal/dependent and lower lobes are denser than the ventral/non-dependent and upper lobes). Radiographic changes tend to be absent for the fi rst 24 hours (with the exception of direct lung injury), then increase to remain static for days or weeks, and then begin to resolve. Pneumothorax can occur secondary to ventilation. Reticular changes, with a predilection for non-dependent lung, may be secondary to the underlying process or to barotrauma (seen in 85% of survivors in one study; the mortality of ARDS is approximately 50%)
470
A previously well 42-year-old man is admitted with acute left-sided pleuritic chest pain. His SaO2 is recorded as 92%. D-Dimer assay is elevated. His mother had died suddenly at the age of 58 years. He is further investigated via CTPA, which is negative for PE. Based on his presenting symptoms, the referring consultant continues to be concerned that the patient has a PE. What advice do you offer regarding this patient’s management? A. Refer for V/Q scanning. B. Refer for catheter pulmonary angiography. C. Commence anticoagulation for 3 months given clinical suspicion. D. Commence anticoagulation for 6 months given clinical suspicion. E. No further investigation or anticoagulation required. F. Repeat CTPA.
E. No further investigation or anticoagulation required. In the setting of a low probability clinical assessment and positive D-dimer assay, a negative CTPA has a negative predictive value of 96% and further investigation and treatment are therefore not warranted. A repeat CTPA may be indicated if the images are of poor quality. The Prospective Investigation of Pulmonary Embolism Diagnosis (PIOPED) II investigators recommend that in the setting of a high pre-test probability, a negative CTPA should be followed with either venous ultrasound or MR venography.
471
A 73-year-old patient is involved in a road traffi c accident (RTA) and sustains a head injury. He is intubated at the scene due to a low Glasgow Coma Scale (GCS). The patient is transferred for a CT chest as he is hypoxic. On reviewing the CT scan you note widespread emphysema, consistent with the history of smoking. He has a narrowing of the trachea, immediately inferior to the distal margin of the endotracheal (ET) tube. This narrowing is caused by an endoluminal mass associated with a circumferential area of soft tissue that extends into the paratracheal space. There is no pneumomediastinum and no other lung injury is seen. What is the most likely cause? A. Post-intubation stenosis. B. Tracheal papilloma. C. Non small cell lung cancer. D. Adenoid cystic carcinoma. E. Squamous cell carcinoma.
E. Squamous cell carcinoma. Tracheal malignancies make up 1–2% of all adult intrathoracic tumours and as such are uncommon. Malignant lesions make up 90% of all tracheal malignancies. Of these, squamous cell carcinomas are the most common, presenting in elderly patients with a history of smoking. Adenoid cystic carcinoma is the next most common, presenting in a younger age group and associated with a better prognosis. Benign lesions account for less than 10%. Non small cell lung cancer (NSCLC) would be the leading differential diagnosis if this lesion was found endobronchially, but not in the trachea. NSCLC can cause tracheal narrowing, but as an extrinsic lesion. The history is too brief for post-intubation stenosis to be considered and this is not associated with a soft tissue mass.
472
You are carrying out a CT chest scan on a patient who is under the joint care of the respiratory physicians and the rheumatologists. The patient reports slowly progressing stridor. The patient has already been assessed by ear, nose and throat (ENT) due to collapse of the nasal turbinates, but this is felt to be unconnected to his stridor. His infl ammatory markers are elevated. A nasal biopsy showed an infl ammatory infi ltrate in the cartilage causing dissolution, but no granuloma formation or vasculitis. The CT shows smooth thickening of the anterior trachea, with early calcium deposition, with relative sparing of the posterior trachea. This pattern is most marked in the subglottic region. There is narrowing of the airway. This pattern is unaffected on the expiratory scan as compared to the inspiratory scan. The transverse diameter of the trachea is 60% of the sagittal diameter. What is the likely cause? A. Wegener’s granulomatosis. B. Amyloidosis. C. Relapsing polychondritis. D. Mounier–Kuhn disease. E. Tracheobronchomalacia.
C. Relapsing polychondritis. Wegener’s granulomatosis and amyloidosis can both give a similar appearance to that described. Amyloid can occur as an isolated condition or as a part of systemic amyloidosis. It gives smooth narrowing, but can also give multifocal stenoses or plaques, and is frequently associated with calcifi cation. Wegener’s commonly affects the subglottic region, giving an identical appearance, although it can cause a more irregular pattern of thickening and ulcer formation. Similarly Wegener’s commonly affects the cartilage in the nose. However, histologically, Wegener’s causes vasculitis and granuloma formation. Mounier–Kuhn disease is also known as tracheobronchomegaly. It can be associated with tracheobronchomalacia, which may give a similar CT appearance to that described. The key difference is that this condition is characterized by a reduction in calibre of >50% of the airway lumen during expiration, as compared to inspiration. Relapsing polychondritis is a systemic condition also affecting the cartilage of the nose, ears, and joints.
473
A 56-year-old man is admitted via the accident and emergency (A&E) department. He has a past medical history of mitral valve disease. He is complaining of shortness of breath and the clinical team believe he has pulmonary oedema, but ask for your opinion on his CXR to rule out infection. The presence of which of the follow features could not be attributed to cardiac failure and would make you doubt the diagnosis? A. Perihilar alveolar opacities. B. Sparing of the lung periphery. C. A unilateral pleural effusion. D. Unilateral regional oligaemia. E. Right upper lobe opacifi cation.
D. Unilateral regional oligaemia. This represents Westermark’s sign and is associated with PE, not pulmonary oedema. The other features are consistent with cardiac failure. In particular, focal right upper lobe oedema is associated with mitral regurgitation, where the regurgitant jet produces locally increased pressures in the right upper lobe pulmonary veins with a focal increase in oedema in that region. This can mimic consolidation on plain fi lm, but will be seen to resolve after diuresis. Pleural effusions may be unilateral in cardiac failure.
474
A 62-year-old man undergoes lung scintigraphy for investigation of PE. There is no prior history of PE. Which of the following scan patterns would be in keeping with a low probability for PE? A Triple matched defect in the lower lung zone. B Single moderate matched V/Q defect with a normal CXR. C Perfusion defect with a rim of surrounding normally perfused lung. D No defects present on perfusion scan. E. Four moderate segmental defects.
C. Perfusion defect with a rim of surrounding normally perfused lung. Multiple bilateral perfusion defects with a normal ventilation scan are the classic diagnostic fi ndings in PE. Occluding pulmonary emboli produce segmental perfusion defects that extend to the pleural surface. As other conditions may also produce perfusion defects, the ventilation scan improves specifi city. Non-embolic lung disease will typically have both perfusion and ventilation abnormalities, resulting in matched defects. V/Q scans are categorized as normal, low, intermediate, or high probability. A perfusion defect that matches ventilation and CXR abnormalities in size and location is a triple matched defect. A triple matched defect in the middle or upper lung zones is in keeping with low probability, but rises to intermediate probability when in the lower zones. A single moderate matched V/Q defect, but with a normal CXR, is also of intermediate probability. No perfusion defect is in keeping with a normal scan and four moderate segmental defects is a high probability scan. A perfusion defect with a rim of surrounding normally perfused lung is known as the stripe sign and corresponds to low probability for PE, as PE perfusion defects should extend to the pleural surface and have no overlying stripe of perfused lung.
475
A 65-year-old man has kept pigeons for over 20 years. He is complaining of gradually worsening shortness of breath. A CXR shows increased interstitial markings, with reduction in lung volumes. A subsequent HRCT of chest shows quite marked pulmonary fi brosis with areas of honeycomb formation. Which part of the lung is likely to be relatively spared by the fi brotic process? A. Upper zones. B. Mid zones. C. Posterior costophrenic sulci. D. Central peribronchovascular regions. E. Subpleural lung.
C. Posterior costophrenic sulci. The question is leading to chronic hypersensitivity pneumonitis as the most likely underlying diagnosis. The fi brotic process, in advanced stages, affects both the subpleural lung and the peribronchovascular interstitium. There may be honeycomb formation at the lung bases, but unlike usual interstitial pneumonia/idiopathic pulmonary fi brosis, the honeycombing typically spares the most extreme posterior costophrenic sulci. Classically, the fi brotic process is more pronounced in the mid and upper lung zones.
476
A 70-year-old male undergoes endovascular stent graft repair of an infra- renal abdominal aortic aneurysm. A follow-up CT at 1 year demonstrates increasing aneurysm sac diameter without any evidence of endoleak. What is the diagnosis? A. Type I endoleak. B. Type II endoleak. C. Type III endoleak. D. Type IV endoleak. E. Type V endoleak.
Type V endoleak. The main aim of endovascular or surgical treatment of abdominal aortic aneurysm (AAA) is exclusion of the aneurysm sac from the systemic high-pressure circulation. Ongoing leakage of blood into the excluded aneurysm sac after endovascular repair is termed ‘endoleak’. Identifi cation of the type of endoleak and its effect on the aneurysm sac is important for further management. Type I endoleak: Contrast/blood leak at the proximal or distal landing zones of the stent graft is described as type I endoleak. This is due to poor proximal or distal graft apposition, exposing the sac to systemic pressures with signifi cant risk of aneurysm rupture. This type is further sub- divided into type IA (proximal aortic attachment) and type IB (distal iliac attachment). These are most commonly seen at the time of the procedure or may develop subsequently due to graft migration. They require urgent treatment. Type II endoleak: This is due to retrograde fl ow into the aneurysm sac via the inferior mesenteric artery (type IIA) or lumbar arteries (type IIB). Many of these close spontaneously and are managed expectantly. Further treatment is indicated if the sac enlarges or the patient develops symptoms of sac pressurization. Type III endoleak: Leakage of blood through the body of the stent graft due to either poor apposition of graft components or a tear in the graft material is type III endoleak. This requires urgent management due to sac pressurization. Type IV endoleak: Aneurysm sac opacifi cation without an identifi able source intraprocedurally is described as type IV endoleak. These are transient and usually resolve after withdrawal of anticoagulation. Type V endoleak: Continued growth of the sac without radiological evidence of a leak is termed type V endoleak or endotension. Continued growth of the aneurysm sac will require surgical repair due to risk of rupture.
477
A 28-year-old man is being investigated for haemoptysis. He has a history of sinusitis. Full blood picture is normal. He is referred for a CT of chest during which intravenous (IV) contrast was withheld by the radiographer due to a reduction in estimated glomerular fi ltration rate (eGFR). It reveals bilateral nodules in a peribronchovascular distribution, some of which show cavitation. There are peripheral wedge-shaped areas of consolidation. There are also areas of bronchial stenosis and thickening. No mediastinal or hilar adenopathy is present. What is the most likely diagnosis? A. Goodpasture’s syndrome. B. Sarcoidosis. C. Churg–Strauss syndrome. D. Wegener’s granulomatosis. E. Pulmonary tuberculosis.
D. Wegener’s granulomatosis. The history of sinusitis, pulmonary haemorrhage, and renal involvement (reduced eGFR) point towards Wegener’s granulomatosis. This is a systemic autoimmune disease characterized by granulomatous vasculitis of the upper and lower respiratory tracts, glomerulonephritis, and small vessel vasculitis. It predominantly affects male patients. There is pulmonary involvement in most patients. The most common radiographic appearance is lung nodules or irregularly marginated masses with no zonal predilection. The nodules are solitary in up to 25% and cavitating in 50% of cases. The cavities usually have a thick, irregular wall. With treatment they may resolve or result in a scar. Peripheral areas of wedge-shaped consolidation representing infarction may occur. Pleural effusions occur in less than 10% and mediastinal/hilar adenopathy is uncommon. Tracheal and bronchial thickening can be smooth or nodular. Haemorrhage can result in focal areas of dense consolidation, patchy bilateral areas of consolidation, or diffuse consolidation (these may be diffi cult to distinguish from infection). The main differential diagnosis would be Churg–Strauss syndrome, which involves asthma, eosinophilia (which would be noticed in the full blood picture), and less severe renal and sinus disease. Patchy transient consolidation, which may be peripheral, is the norm and nodules may occur, although cavitation is rare. Wegener’s granulomatosis is associated with c-ANCA and Churg–Strauss with p-ANCA. Goodpasture’s syndrome is associated with glomerulonephritis and pulmonary haemorrhage, but the fi ndings are initially extensive perihilar and basal consolidation with sparing of the apices, which is subsequently replaced by an interstitial pattern. Cavitating nodules are not a feature. Haemoptysis is uncommon in sarcoidosis and cavitating nodules rare; one would typically expect perilymphatic nodules and in classic cases adenopathy. TB can produce cavitating nodules and haemoptysis, but the other features are more consistent with Wegener’s granulomatosis.
478
A 50-year-old woman presents with progressive exertional dyspnoea, fatigue and atypical chest pain. Her jugular venous pressure (JVP) is elevated on examination. Her CXR reveals prominence of the right side of the heart with asymmetric enlargement of the central pulmonary arteries. Patchy oligaemic vascularity is also evident. What is the most likely diagnosis? A. Atrial septal defect. B. Primary pulmonary hypertension. C. Chronic thromboembolic pulmonary hypertension. D. Cardiopulmonary schistosomiasis. E. Pulmonary veno-occlusive disease.
C. Chronic thromboembolic pulmonary hypertension. Pulmonary hypertension is the haemodynamic consequence of vascular changes within the precapillary (arterial) or postcapillary (venous) pulmonary circulation. The diagnosis of primary (idiopathic) pulmonary hypertension (PPH) can only be made after exclusion of known secondary causes. The classical fi ndings in advanced disease include prominent central pulmonary arteries with sharply tapering peripheral vessels and right ventricular enlargement. It typically affects younger women of childbearing age. The radiographic features of pulmonary hypertension caused by chronic shunting are similar to PPH, although a normal sized cardiac silhouette may refl ect diminished shunting due to a markedly elevated pulmonary vascular resistance. Most congenital cardiac lesions that may eventually cause pulmonary hypertension are now repaired at an early age. Chronic thromboembolic pulmonary hypertension may mimic PPH clinically, making diagnosis diffi cult. Radiographic fi ndings are more likely to be asymmetrical. A triangular opacity representing pulmonary infarction may also been seen. Schistosomiasis will demonstrate the radiographic features of pulmonary hypertension and may also exhibit tiny nodular granulomas. It is endemic in the Middle East, Africa, and the Atlantic coast of South America. Pulmonary veno-occlusive disease is the post-capillary counterpart of PPH. It is suggested radiographically when the features of pulmonary arterial hypertension are accompanied by evidence of diffuse pulmonary oedema and a normal sized left atrium.
479
A 34-year-old woman presents with a 4-month history of gradually increasing dyspnoea and cough. A CXR and subsequent CT scan show multiple cavitating lung lesions. On the CT scan, some of these lesions are noted to have surrounding ground-glass attenuation. No other abnormality is seen. Which of the following diagnoses are the fi ndings most compatible with? A. Rheumatoid lung. B. Lung abscesses. C. Eosinophilic granuloma. D. Churg–Strauss syndrome. E. Melanoma metastases.
E. Melanoma metastases. The GGO surrounding a nodule is known as the ‘halo’ sign and represents perilesional haemorrhage. The differential diagnosis given rests on the presence of this feature and cavitation. Melanoma metastases can both cavitate and produce perilesional haemorrhage. The halo sign may also be seen in other conditions with perilesional haemorrhage or cellular infi ltration and is usually best seen on HRCT. These diagnoses include bronchoalveolar carcinoma, haemorrhagic metastases, Wegener’s granulomatosis, and angio-invasive infections, such as invasive aspergillosis. Alternative correct answers would be Wegener’s granulomatosis, lymphoma, bronchoalveolar carcinoma, and squamous cell carcinoma as these can produce both cavitating nodules and the halo sign. Rheumatoid lung, eosinophilic granuloma, and lung abscesses are associated with cavitating nodules but not the halo sign. Churg–Strauss syndrome is not associated with the halo sign and cavitation is rare.
480
A 64-year-old smoker is referred by his GP for persisting consolidation which has failed to resolve despite multiple antibiotic therapies. Of note he has been apyrexic and infl ammatory markers have not been particularly raised. The respiratory team request a CT of chest, which shows GGO and consolidation of almost the entire left lower lobe, delineated by the major fi ssure, which is not displaced. Air bronchograms are present, but there is no signifi cant loss of volume or expansion of the lobe and no mediastinal or hilar adenopathy. No mass obstructing the left lower lobe bronchus (either endoluminal or extrinsic) is demonstrated and the bronchoscopy fi ndings corroborate this (results from washings not yet available). A PET-CT is normal. What is the most likely pathology? A. Carcinoid tumour. B. Bronchioloalveolar carcinoma. C. Small cell carcinoma. D. Tuberculosis (TB). E. Klebsiella pneumonia.
B. Bronchioloalveolar carcinoma. In this disease, the tumour spreads along the alveolar septa without invading alveolar walls. The air in the alveoli is replaced by tumour cells, producing consolidation and GGO. Mediastinal lymphadenopathy is rare; pleural effusion is common. Diagnosis is made by sputum/bronchial washing cytology or lung biopsy. It mimics other causes of air-space opacifi cation such as pneumonia, haemorrhage, oedema etc. Disseminated adenocarcinoma, choriocarcinoma, or lymphoma might produce identical CT fi ndings. PET-CT is often negative in the case of both bronchioloalveolar carcinoma and carcinoid tumours of the lung. Carcinoid is in the form of a focal mass, not diffuse consolidation. Klebsiella pneumonia classically produces enlargement of the involved lobe, bulging of the fi ssures with the propensity for cavitation, and abscess formation. One would expect pyrexia and raised infl ammatory markers, and consolidation usually produces some abnormality on PET-CT. Small cell carcinoma is usually positive on PET-CT. TB is usually PET positive and has a predilection for the upper lobes, or apical segments of the lower lobes if the latter are involved.
481
A 45-year-old male smoker has a history of fatigue and mild shortness of breath. He also keeps pigeons. A CXR shows mildly increased interstitial markings in the upper zones. An HRCT of chest demonstrates multiple small pulmonary nodules and reticulation, more marked in the upper lungs. What location of the nodularity is more likely to suggest a diagnosis of subacute extrinsic alveolitis or respiratory bronchiolitis interstitial lung disease as opposed to sarcoidosis? A. Bronchovascular bundle. B. Centrilobular region. C. Fissural. D. Subpleural region. E. Interlobular septa.
B. Centrilobular region. In sarcoid, the granulomatous nodules are typically distributed along the lymphatics and are therefore seen along the bronchovascular bundles, interlobular septa, major fi ssures, and subpleural regions. The centrilobular region of the secondary pulmonary lobule contains the bronchiole and therefore conditions that cause peribronchiolar infl ammation, such as subacute extrinsic allergic alveolitis (EAA) and respiratory bronchiolitis interstitial lung disease (RBILD), more often cause centrilobular nodules. These are often ground glass in attenuation and ill- defi ned, unlike the more solid appearing granulomatous nodules of sarcoid. Rarely sarcoid can cause centrilobular nodules because of the presence of peribronchiolar granulomas, but the other mentioned locations are much more typical.
482
A 43-year-old patient presents with cough, shortness of breath and fever which has lasted a month. An HRCT reveals bilateral areas of consolidation, predominantly in a peripheral distribution. There are also areas of GGO, predominantly in the middle and upper zones, with band-like subpleural attenuation. The plain fi lm fi ndings have remained unchanged for days. What is the most likely diagnosis? A. Chronic eosinophilic pneumonia. B. Allergic bronchopulmonary aspergillosis (ABPA). C. Acute eosinophilic pneumonia. D. Löffl er’s syndrome. E. Eosinophilic granuloma.
A. Chronic eosinophilic pneumonia. This has been described as the radiological ‘photographic negative’ of pulmonary oedema. It is a disease of middle age and affects females more commonly than males. The history is that described, with a common history of atopy. The predominant histologic fi nding is fi lling of the alveolar airspaces with an infl ammatory infi ltrate containing a high proportion of eosinophils. There is usually also a cellular infi ltration of the interstitium and peripheral blood eosinophilia. There is a dramatic response to steroid therapy within days. ABPA and acute eosinophilic pneumonia do produce blood eosinophilia, but the former is characterized by bronchiectasis and mucus plugging with the possibility of mosaic perfusion in addition to peripheral consolidation, while the latter is characterized by diffuse GGO, defi ned nodules, smooth interlobular septal thickening, and often the presence of pleural effusion. Löffl er’s syndrome (simple pulmonary eosinophilia) refers to predominantly peripheral transient parenchymal consolidation accompanied by eosinophilia. There are minimal or no pulmonary symptoms, the plain fi lm appearances change within one to several days and spontaneous resolution occurs within one month. Eosinophilic granuloma is nodular/cystic, a pulmonary form of Langerhan’s cell histiocytosis, and should not be confused with the eosinophilic pneumonias.
483
A 30-year-old caucasian man, recently treated with bone marrow transplantation for acute myeloid leukaemia, presents with fever and cough. HRCT chest demonstrates multiple, small centrilobular nodules of soft tissue attenuation connected to linear branching opacities. What is the most likely cause of this fi nding? A. Endobronchial tuberculosis. B. Primary pulmonary lymphoma. C. Invasive aspergillosis. D. Obliterative bronchiolitis. E. Diffuse panbronchiolitis.
A. Endobronchial tuberculosis. The CT fi ndings describe the ‘tree-in-bud’ pattern, which results from centrilobular bronchiolar dilatation and fi lling by mucus, pus, or fl uid that resembles a budding tree. It is usually most pronounced in the lung periphery. All of the options provided are differentials for ‘tree-in-bud’, although infective causes are most common, classically endobronchial spread of active TB. The patient in this case is also at risk of invasive aspergillosis, although typically the ‘tree-in-bud’ pattern occurs in combination with consolidation accompanied by a halo of GGO. Obliterative bronchiolitis occurs in bone marrow transplantation in the setting of chronic graft-versus-host disease. The most sensitive CT fi nding in this condition is air-trapping on expiratory CT. Diffuse panbronchiolitis is of unknown cause, but occurs almost exclusively in Eastern Asia. Primary pulmonary lymphoma is also a rare cause of ‘tree-in-bud’. Other potential differentials of this pattern include cytomegalovirus infection, cystic fi brosis, aspiration, connective tissue disease, and tumour emboli.
484
A 56-year-old woman with a history of Sjogren’s syndrome complains of gradually increasing shortness of breath. A CXR has identifi ed a mild generalized interstitial pattern, with maintained lung volumes. A subsequent HRCT of chest demonstrates a few scattered well-defi ned, regular lung cysts. Within the lung parenchyma there is also noted patchy ground- glass change and mild centrilobular nodularity. Mild mediastinal and hilar lymphadenopathy is present. What is the most likely diagnosis? A. Langerhans cell histiocytosis. B. Desquamative interstitial pneumonia. C. Lymphangioleiomyomatosis. D. Lymphocytic interstitial pneumonia. E. Birt–Hogg–Dube syndrome.
D. Lymphocytic interstitial pneumonia. The key to this question, in addition to the described imaging features, is the history of Sjogren’s syndrome, which has an association with LIP. LIP is a benign lymphoproliferative disorder that is also associated with AIDS, autoimmune thyroid disease, and Castleman’s syndrome. Lymphomas may arise in some cases. On HRCT, thin-walled cysts are seen in two-thirds and are randomly distributed, occupying less than 10% of the lung parenchyma. Other features include GGO, centrilobular/subpleural nodules, and septal thickening. Eventually larger nodules (>2 cm), consolidation, and architectural distortion may develop. The presence of mediastinal and hilar lymphadenopathy (two-thirds) and septal thickening help distinguish from Langerhans cell histiocytosis. The presence of centrilobular nodules assist in the differentiation from lymphangioleiomyomatosis. Birt–Hogg–Dube syndrome is a rare autosomal dominant condition characterized by facial fi brofolliculomas, malignant renal tumours, and the development of thin-walled pulmonary cysts and spontaneous pneumothorax.
485
A 67-year-old man who was previously a manual worker presents with chest pain, which subsequently turns out to be due to myocardial ischaemia. He has a CXR performed which shows numerous small nodular densities and you suspect he has an occupational lung disease, as these densities are unchanged from previous radiographs. A subsequent HRCT of chest shows no evidence of linear interstitial change or fi brosis. Pulmonary function tests are normal. Which of the following possible causes is least likely to result in functional lung impairment? A. Coal workers’ pneumoconiosis. B. Silicosis. C. Berylliosis. D. Siderosis. E. Asbestosis.
D. Siderosis. Siderosis is due to the inhalation of iron oxide particles and usually occurs in welders. It causes mutiple small centrilobular nodules, but is not usually associated with any symptoms or fi brosis. If combined with silica dust it can cause silicosiderosis, which can be associated with fi brosis. Silicosis and coal workers pneumoconiosis, secondary to inhalation of silica dust and washed coal dust, respectively, show similar features on CT. This is usually the presence of 2–5 mm nodules, mainly involving the upper and posterior lung zones. Large opacities (>1 cm) indicate progressive massive fi brosis. Calcifi cation in lymph nodes can occur and egg shell calcifi cation is more typical in silicosis. Berylliosis is a chronic granulomatous lung disease caused by exposure to beryllium dust or fumes. CT fi ndings are similar to other granulomatous lung diseases, such as sarcoid. Fibrosis may therefore occur. Asbestosis is pulmonary fi brosis secondary to inhalation of asbestos fi bres.
486
You are attending a lecture on lung cancer, but unfortunately you arrive late so you have missed the introduction. The lecturer is describing a subtype of lung cancer. The description is of a tumour that comprises 30% of all lung cancers. It typically occurs peripherally, but can be central. This tumour can cavitate, but this occurs in only 4% of cases. Hilar and/or mediastinal involvement is seen in over half of cases on plain fi lm radiography. What subtype of lung cancer is being described? A. Adenocarcinoma. B. Bronchoalveolar carcinoma. C. Squamous cell carcinoma. D. Small cell carcinoma. E. Giant cell carcinoma.
A. Adenocarcinoma. These are all classical features of adenocarcinoma. Bronchoalveolar carcinoma is a subtype of adenocarcinoma. This comprises 2–10% of lung cancers. There are three subtypes: a solitary nodule (41%), multifocal nodules (36%), and peripheral consolidation (23%). Squamous cell carcinoma is only slightly less prevalent than adenocarcinoma. It cavitates in 86% of cases and typically occurs centrally. Small cell carcinoma comprises 18% of lung cancers. It usually presents on plain fi lm radiography as hilar and/or mediastinal adenopathy. CT often detects lung opacities. Giant cell carcinoma is a poorly differentiated subtype of NSCLC that is capable of rapid growth and early metastasis.
487
A 24-year-old serviceman presents with insidious onset of fever, headache and worsening non-productive cough. His white cell count and erythrocyte sedimentation rate (ESR) are elevated and serum cold agglutination is positive. He had failed to improve with initial antibiotic therapy. HRCT of chest demonstrates areas of ground-glass opacity, air-space consolidation, centrilobular nodules and thickening of bronchovascular bundles. What is the most likely diagnosis? A. Chlamydia pneumonia. B. Mycoplasma pneumonia. C. Pneumococcal pneumonia. D. Legionella pneumonia. E. Staphylococcal pneumonia.
Mycoplasma pneumonia. The given clinical history is classical of mycoplasma pneumonia, which usually affects younger adults in closed populations such as prisons or the military. It is one of the most common causes of community acquired pneumonia in otherwise healthy individuals. Serum cold agglutination is positive in up to 70%. On HRCT areas of ground-glass attenuation tend to be around areas of consolidation. Centrilobular nodules and peribronchovascular thickening are common associated fi ndings.
488
A 42-year-old male presents with chest pain, dyspnoea and palpitations. He undergoes cardiac MRI, which reveals extensive scattered delayed enhancement in the anterior, lateral and inferior wall and apex of the left ventricle. This enhancement occurs in the midwall with relative sparing of the subendocardial region. T2WI is unremarkable. What is the most likely diagnosis? A. Acute myocardial infarction. B. Sarcoidosis. C. Myocarditis. D. Hypertrophic cardiomyopathy. E. Amyloidosis.
C. Myocarditis. This is defi ned as infl ammation of the heart muscle. A large variety of infections, systemic diseases, drugs, and toxins have been associated with this condition. The diagnosis is based on a combination of clinical and imaging features. The presence of focal delayed enhancement on cardiac MRI in a non-coronary artery distribution, together with wall motion abnormalities, correlates strongly with myocarditis in the correct clinical setting. Many patients present with a non-specifi c illness characterized by fatigue, dyspnoea, and myalgia. An antecedent viral syndrome is present in more than 50% of patients. Myocarditis lesions occur typically in the lateral free wall and originate from the epicardial quartile of the ventricular wall. The subendocardial area is spared, a pattern that is otherwise typical for myocardial infarction (in the latter case the lesion would also correspond to a coronary artery territory). In myocarditis the enhancement pattern has been described as becoming less intense and more diffuse over weeks and months. In acute myocardial sarcoidosis, increased focal signal intensity can be observed on T2WI (secondary to oedema due to infl ammation) and both early and delayed post-contrast T1 weighted imaging (T1WI). Focal myocardial thickening is often seen due to the oedema and can mimic hypertrophic cardiomyopathy (HCM). HCM will reveal marked hypertrophy of the interventricular septum and left ventricular wall, with associated transmural delayed enhancement in the hypertrophied areas. The latter fi nding corresponds to the scattered fi brosis present and the amount of enhancement will inversely correlate with regional contractivity. Cardiac amyloidosis leads to a restrictive cardimyopathy. MR imaging shows functional impairment, biventricular hypertrophy, and non-specifi c inhomogenous gadolinium enhancement.
489
A 28-year-old woman presents with fever, myalgia and cough. Due to a current community outbreak, the clinical team suspect that she has H1N1 infl uenza (swine fl u). Which fi nding on her admission CXR is most strongly predictive of an adverse outcome? A. Upper lobe consolidation. B. Bilateral central opacity. C. Multizonal peripheral opacity. D. Air bronchogram. E. Pleural effusion.
C. Multizonal peripheral opacity. The majority of H1N1 infl uenza cases have been mild, but the 2009 strain can cause severe illness, including in young previously healthy persons. Radiological fi ndings in four or more lung zones distributed bilaterally and peripherally, are signifi cantly more often seen on the CXR obtained at admission in patients with poor outcome (requiring mechanical ventilation) compared to those with good clinical outcome. Central GGO is the most common radiographic abnormality, but is not signifi cantly associated with poor outcome. Pleural effusions are uncommon, although bilateral effusions are an independent predictor of short-term mortality in community acquired pneumonia. It should be noted that an initial normal CXR does predict against a poor outcome
490
A 56-year-old man presents with shortness of breath. He subsequently has an HRCT of chest performed. This shows a mosaic attenuation pattern throughout the lung parenchyma, but you are having some diffi culty determining if the more lucent areas are normal or abnormal. Which of the following fi ndings is most likely to be helpful in confi rming that the lucent areas are the abnormal areas and you are not dealing with multifocal GGO? A. Increased calibre of vessels in denser areas. B. Decreased calibre of vessels in denser areas. C. Increased calibre of vessels in lucent areas. D. Decreased caliber of vessels in lucent areas. E. Calibre of vessels is unhelpful and expiratory scans must be used.
D. Decreased calibre of vessels in lucent areas. When mosaic lung attenuation is observed, it often is in an extensive, but patchy, distribution and it is important to determine if it is the lucent or denser areas of lung that are abnormal. If the blood vessels in the lucent areas are smaller, then the lucent areas are probably abnormal. The paucity of vessels in these regions may be secondary to focal air-trapping or poor ventilation and subsequent refl ex vasoconstriction. If areas of lucency are exaggerated on expiratory scans (air- trapping), then this is the hallmark of small airways disease. Alternatively, if the areas of lucency do not become more prominent on the expiratory scans, small airways disease is not the likely cause. In this situation, the inhomogeneous lung attenuation is probably secondary to changes in vessel calibre, and secondary to pulmonary hypertension, including chronic PE, emphysema, or infl ammatory vasculopathies. If the blood vessels in the regions of relative lucency are equal in size to vessels in surrounding areas, the regions of relative opacity are most likely abnormal, e.g. areas of GGO.
491
A 65-year-old man presents to the A&E department with acute shortness of breath. He has a CXR performed and this demonstrates a ‘bat-wing’ pattern of pulmonary oedema. Which of the following is the most likely cause? A. Fat embolism. B. Diffuse alveolar damage. C. Adult respiratory distress syndrome. D. Acute mitral valve insuffi ciency. E. Left ventricular failure.
D. Acute mitral valve insuffi ciency. ‘Bat-wing’ oedema refers to a central, non-gravitational alveolar oedema, which is seen in less than 10% of cases of pulmonary oedema. It generally occurs with rapidly developing severe cardiac failure, such as that seen with acute mitral valve insuffi ciency or renal failure. It develops so rapidly that it is initially observed as an alveolar infi ltrate and the preceding interstitial phase of pulmonary oedema goes undetected radiologically. ARDS and diffuse alveolar damage may overlap pathophysiologically, and along with fat embolism show radiographic changes of a non-cardiogenic pulmonary oedema. These are similar to the standard radiographic features of cardiogenic pulmonary oedema affecting the lung parenchyma, except that GGO tends to be more confl uent and consolidative, the changes tend to be less dependent, and subfi ssural thickening/septal lines are uncommon.
492
A 57-year-old patient has a CXR carried out. This shows a mass in the left apex, adjacent to the spine. Numerous soft tissue densities are noted projected across the lungs. A lateral radiograph is carried out which projects the mass over the vertebral bodies and indicates that the smaller densities are cutaneous. You request additional information from the referring clinician. This patient has a complex history. He has type 1 neurofi bromatosis, but has also recently been diagnosed with myelofi brosis. You perform a CT scan, which shows widening of the neural foramen on the left side, which is in continuity with the left apex mass. This mass measures 2 cm in diameter and has an attenuation value of 4 HU. What is the most likely diagnosis? A. Neuroblastoma. B. Neurofi broma. C. Extramedullary haematopoesis. D. Lateral meningocele. E. Neuro-enteric cyst.
D. Lateral meningocele. Neuroblastoma is a tumour of childhood and would be extremely rare in a 57-year-old. Whilst the history of neurofi bromatosis type 1 (NF-1) would raise the possibility of this lesion being a neurofi broma, the CT fi ndings of a cyst discount this. Extramedullary haematopoesis is a rare feature of myelofi brosis and commonly gives bilateral soft tissue masses. Lateral meningoceles are herniations of CSF through a dilated neural foramen, most commonly in patients with a history of NF-1. As they contain cerebrospinal fl uid (CSF), the attenuation value would be comparable to water. A neuro-enteric cyst can have a similar appearance. These are often symptomatic and detected in childhood, unlike lateral meningoceles, which are asymptomatic. They are associated with congenital spinal abnormalities rather than widening of the neural exit foramen as described in this case.
493
A 64-year-old woman presents with a 3-week history of dry cough. A CXR is performed and shows multifocal bilateral peripheral areas of consolidation. An HRCT of chest is recommended and this demonstrates bilateral peripheral areas of consolidation and GGO. There is no fi brosis. There is peripheral eosinophilia detected on routine blood tests. A careful drug history is obtained. Which of the following medications is the patient most likely to be on? A. Nitrofurantoin. B. Amiodarone. C. Methotrexate. D. Bleomycin. E. Cyclophosphamide.
A. Nitrofurantoin. The fi ndings described on the CXR and HRCT scan are those of pulmonary eosinophilia. Drugs known to cause pulmonary eosinophilia include nitrofurantoin, penicillamine, sulphasalazine, non- steroidal anti-infl ammatory drugs, and para-aminosalicylic acid. Bleomycin and cyclophosphamide are more commonly associated with DAD. Other drugs that can cause this type of lung injury include busulphan, carmustine, gold salts, and mitomycin. In early DAD, HRCT typically shows scattered or diffuse areas of GGO. Fibrosis typically develops within 1 week and if progressive can cause marked architectural distortion and honeycombing. Amiodarone and methotrexate are associated with an NSIP pattern on HRCT. Carmustine and chlorambucil can also cause this appearance. With early disease, HRCT scans may show only scattered or diffuse areas of GGO. Later, fi ndings of fi brosis (traction bronchiectasis, honeycombing) predominate in a basal distribution.
494
A 70-year-old man presents with a 6-month history of cramping pain in the left calf brought on by walking and settling with rest. For the past 3 weeks he has been experiencing pain at rest, which is relieved by dependency of the foot. On examination, the popliteal and tibial pulses are absent. There is no ulceration or gangrene. What is the diagnosis? A. Intermittent claudication. B. Critical limb ischaemia. C. Acute limb ischaemia. D. Nerve root compression. E. Diabetic neuropathy.
B. Critical limb ischaemia. Muscle pain or discomfort in the lower limb brought on by exercise and relieved by rest within 10 minutes is termed intermittent claudication. This is secondary to reduced perfusion, which may be enough during periods of rest, but on exercise is insuffi cient to meet the metabolic demand of the muscles. With progression of disease, these symptoms may be noticed at rest. Typical ischaemic rest pain is relieved by dependency of the foot secondary to gravity-aided blood fl ow. Critical limb ischaemia is the presence of rest pain or tissue loss with ulcers or gangrene. This should be distinguished from acute limb ischaemia. The term ‘critical limb ischaemia’ implies chronicity with symptoms being present for at least 2 weeks. Diabetic neuropathy is associated with burning or shooting pain in the feet, which may sometimes be diffi cult to differentiate from atypical ischaemic rest pain. Distinguishing features include bilateral symmetric distribution, cutaneous hypersensitivity, and failure to relieve by dependency of the foot. Reduced vibration sensation and refl exes are also seen in diabetic neuropathy. Nerve root compression may sometimes result in continuous pain but it has a dermatomal distribution and is associated with backache.
495
A 24-year-old woman who is 28 weeks pregnant is admitted with suspected pulmonary embolism. As the on-call radiologist, her obstetrician contacts you seeking advice regarding further management. An admission CXR is normal. What investigation do you advise initially? A. Venous ultrasound. B. Low-dose CTPA. C. Reduced dose lung scintigraphy. D. MRA. E. Catheter pulmonary angiography.
A. Venous ultrasound. For pregnant patients, venous ultrasound is recommended before imaging tests with ionizing radiation are performed. Up to 29% of pregnant patients with PE will have a positive venous ultrasound, obviating the need for further imaging. The majority of the PIOPED II investigators currently recommend V/Q scanning over CTPA in the evaluation of PE in pregnant patients. The foetal dose with V/Q is similar to that with CTPA, although the effective dose per breast is much greater with CTPA. MRI requires further evaluation and gadolinium-based contrast agents have not been proven to be safe in pregnancy. The role of catheter angiography is probably limited to those patients requiring mechanical thrombectomy. It should be noted that even a combination of CXR, lung scintigraphy, CTPA, and pulmonary angiography exposes the foetus to approximately 1.5mGy of radiation, which is well below the accepted limit of 50 mGy for the induction of deterministic effects in the foetus
496
A 25-year-old male presents with a history of dislocation and spontaneous relocation of the patella while playing football. An MRI of the knee is requested. Which of the following fi ndings is consistent with the clinical history of patellar dislocation? Bone oedema involving medial facet of patella and medial femoral condyle. Bone oedema involving posterior patella and anterior aspect of the tibial plateau. Bone oedema involving the lateral facet of patella and lateral femoral condyle. Bone oedema involving the lateral facet of patella and medial femoral condyle. Bone oedema involving the medial facet of patella and lateral femoral condyle.
E. Bone oedema involving the medial facet of patella and lateral femoral condyle. Transient dislocation of the patella typically occurs laterally as a result of a twisting injury in a fi xed and fl exed knee. The medial facet of the dislocated patella impacts against the lateral femoral condyle, resulting in the classic bone contusion pattern. Rarely oedema may also be seen in the adductor tubercle of the medial femoral condyle due to avulsion of the medial patello-femoral ligament.
497
A 34-year-old female presents to the A&E department after falling on an outstretched hand. Examination reveals tenderness at the anatomic snuff box. A scaphoid radiograph series confi rms scaphoid fracture. Which of the following features is most associated with a poor prognosis? Fracture of the distal third. Fracture of the middle third. Fracture of the proximal third. D. Horizontal oblique fracture orientation. Displacement of the scaphoid fat stripe.
C. Fracture of the proximal third. Scaphoid fracture is the most common of all carpal bone fractures and also potentially serious due to the high rate of avascular necrosis. This fracture can be diffi cult to detect on initial radiographs. Wrist casting and repeat radiography after 1 week are typically advised if there is ongoing suspicion. Fractures of the proximal third account for 20% of injuries, but are associated with failure to unite in 90%. Middle third fractures make up the majority (70%), with up to 30% failing to reunite. Distal third fractures usually reunite. A vertical oblique fracture is more unstable than a horizontal oblique fracture. Fracture displacement of greater than 1mm is also a poor prognostic feature.
498
A 44-year-old female patient presents to the rheumatologists with a history of multiple painful joints for 2 years. She has synovitis clinically, confi rmed on ultrasound, which involves the MCP joints bilaterally. PA and Norgaard views of the hands are requested and show small erosions in the distal radio-ulnar joint and the piso-triquetral joint, but no erosions at the MCP joints. There is widening of the scapholunate interval on the right side. There is anklyosis of the capitate to the hamate on the left. There is periarticular osteoporosis. Which of these features is atypical of RA? A. Symmetrical disease. B. Synovitis on ultrasound but no erosions radiographically. C. Erosions noted in the radio-ulnar joint and radio-carpal joint preceding MCP erosions. D. Bony ankylosis of the carpal bones. E. Periarticular osteoporosis.
D. Bony ankylosis of the carpal bones. Whilst fi brous ankylosis of the carpal and tarsal bones does occur, bony ankylosis is extremely rare in RA. It is, however, common in JRA. There are a number of unusual fi ndings, which if present should indicate a diagnosis other than RA. Productive bone change (e.g. periostitis or enthesopathy) is extremely unusual. Osteophytes are also uncommon in the absence of advanced associated osteoarthritic change. The exception to this is the distal ulna, a feature known as ulnar capping. The other features are all typical of RA.
499
A 26-year-old woman presents with a 2-year history of an enlarging soft tissue mass in her left thumb adjacent to the interphalangeal joint. An x-ray of the left thumb shows a soft tissue swelling with a large well- defi ned erosion seen affecting the distal metaphysis of the proximal phalanx. There is no soft tissue calcifi cation or evidence of arthropathy at the interphalangeal joint. A subsequent MRI scan shows a 3.5-cm well- defi ned soft-tissue mass, which is low signal on T1WI and enhances post administration of gadolinium. The lesion is low signal on T2WI and gradient echo (GE) imaging. What is the most likely diagnosis? A. Ganglion cyst. B. Peripheral nerve sheath tumour. C. Lipoma. D. GCT of the tendon sheath. E. Soft tissue haemangioma.
D. GCT of the tendon sheath. A GCT of the tendon sheath is a nodular form of PVNS. These tumours are intimately associated with a tendon sheath and are most commonly located in the hand. They usually manifest as a small slow-growing mass, with or without pain. Radiographs may show no abnormality or non-aggressive remodelling of the adjacent bone. These lesions are typically hypo- or iso- intense to muscle on T1WI and T2WI, owing to abundant collagen and haemosiderin, often with enhancement. This is similar to the fi ndings of diffuse intra-articular PVNS, when the extent of haemosiderin deposition may cause hypointense nodules on T2WI and blooming artifact on gradient echo (GE) sequences. It must be stated that the degree of haemosiderin content may not always be enough to cause marked hypointensity on T2WI in GCT of the tendon sheath. A ganglion cyst could occur in this location and be related to a tendon sheath, but on MRI it is typically hyperintense on T2WI secondary to its fl uid component. There may be thin rim enhancement of the wall post administration of gadolinium. Peripheral nerve sheath tumours are typically hyperintense on T2WI with variable contrast enhancement. Lipomas are similar in signal characteristic to subcutaneous fat on MRI, i.e. hyperintense on both T1WI and T2WI. A soft-tissue haemangioma may contain phleboliths on plain radiographic imaging. On MRI, haemangiomas may be well circumscribed or have poorly defi ned margins, with varying amounts of increased T1WI signal owing to either reactive fat overgrowth or haemorrhage. Areas of slow fl ow are typically hyperintense on T2WI, while rapid fl ow can demonstrate a signal void on images obtained with a non-fl ow-sensitive sequence.
500
A 56-year-old man has a 6-week history of dull discomfort just above his right ankle. A plain ankle radiograph is performed and this demonstrates a relatively ill-defi ned area of lucency in the distal tibial metaphysis. An underlying aggressive lesion is suspected and the patient is referred for an MRI of the distal right leg. This shows a rather serpiginous-shaped lesion in the distal right tibia. A parallel rim of hypo- and hyperintensity is seen on one of the imaging sequences, which is very helpful in confi rming that the lesion is secondary to metadiaphyseal osteonecrosis rather than a neoplasm. On which imaging sequence is this parallel rim most likely to be seen? A. GE T2*WI. B. Fast spin echo (SE) T1WI. C. Fast SE T2WI. D. Fast SE STIR. E. Fast SE T1WI post gadolinium.
C. Fast SE T2WI. The parallel rim of hypo- and hyperintensity seen on T2WI refers to the ‘double line’ sign, which is almost pathognomic of osteonecrosis. It is most commonly associated with avascular necrosis of the femoral head, but can be seen in osteonecrosis at other sites on MRI. The ‘double line’ sign constitutes a hyperintense inner border (infl ammatory response of bone with granulation tissue), with a hypointense periphery (reactive bone interface). The characteristic plain radiographic pattern of metadiaphyseal osteonecrosis is that of a serpentine ring-like band of sclerosis that separates a central necrotic zone of variable lucency from surrounding normal marrow, although this pattern is a relatively late manifestation of osteonecrosis. Earlier in the course of the disease, osteonecrosis may result in a poorly defi ned region of lucency within the medullary space, a feature that may be indistinguishable from a lytic neoplastic process on x-ray. MRI is then very useful in these cases by showing the serpentine low signal rim of the lesion on T1WI. On T2WI, the rim of the lesion may have low signal, high signal or both (the ‘double line’ sign), the latter being the most specifi c sign for osteonecrosis.
501
A 39-year-old male presents with tenderness and decreased range of movement of the right elbow after falling on an outstretched arm while playing indoor football. A radial head fracture is noted on his radiographs, but the A&E doctor asks for your opinion, suspecting an additional injury. What is the most common associated fracture with this injury? A. Olecranon fracture. B. Coronoid process fracture. C. Scaphoid fracture. D. Proximal ulna fracture. E. Capitellum fracture.
B. Coronoid process fracture. Radial head fractures are common, accounting for approximately one-third of all elbow fractures and up to 5% of all fractures in adults. A recent retrospective study found that associated fracture of the upper extremity was seen in 10.2% of patients, with fractures of the coronoid process the most common (4.1%). Radial head fracture, coronoid fracture, and medial collateral ligament tear form the ‘terrible triad’ of the elbow, which requires operative fi xation.
502
You are looking at an MRI of the knees of a 16-year-old male. There is widening of the distal femoral metaphyses, with a widened intercondylar notch bilaterally. There is mild loss of joint space height in the medial tibio- femoral compartment, with subchondral cyst formation on the left, with preserved joint space but subchondral erosions on the right. The ligaments are intact. GE sequences reveal blooming artefact. Synovial enhancement causing joint erosion is noted on the enhanced T1WI sequence. What is the likely diagnosis? A. Juvenile arthritis. B. Pigmented villonodular synovitis (PVNS). C. Amyloid. D. Haemophilia. E. Tuberculous arthritis.
D. Haemophilia. There are three salient features to this question. Firstly, widening of the intercondylar notch—this is typically caused by haemophilia and JRA, but can also be caused by tuberculous arthritis. Secondly, causes of arthritis with variable loss of joint space. This is seen in tuberculous arthritis, amyloid, and PVNS, and may be present in haemophilia, although this can also cause severe arthropathy. Thirdly, causes of blooming artefact on GE sequences. This is usually caused by haemosiderin and is found in PVNS and haemophilia. Thus, when the whole picture is considered, the diagnosis is haemophilia.
503
A 75-year-old man has a cemented right total hip replacement. On routine follow-up imaging he is noted to have a progressive well-delineated, rounded, focal area of lucency at the cement bone interface adjacent to the tip of the femoral stem. Which of the following given reasons is the most appropriate for this progressive lucency? A. Aggressive granulomatous disease. B. Primary loosening. C. Cement fracture. D. Normal fi nding. E. Metal bead shedding.
A. Aggressive granulomatous disease. Well-delineated, rounded, focal areas of lucency at the cement bone interface, which are progressive, are suggestive of either infection or aggressive granulomatous disease. It can occur with both cemented and non-cemented components. Its origin is thought to be multifactorial. Metal, cement, or polyethylene fragments may penetrate the cement bone interface and induce a focal infl ammatory foreign-body reaction, leading to osteolysis. Primary loosening usually manifests as a wide (>2 mm) radiolucent zone at the cement–bone or metal–bone interface or a progressive radiolucent zone at the metal–cement interface. The radiolucent zones are not typically rounded. Cement fractures are thin linear lucent areas within the cement. They may be asymptomatic, but are important to identify as they may lead to component failure. Metal bead shedding is defi ned as opaque microfragments separated from the porous- coated femoral stem. Metal beads can be seen on immediate postoperative radiographs, as a consequence of the stem insertion. Bead shedding might later occur with loose non-cemented components, refl ecting micro-motion of the stem. These metal beads are seen in the soft tissue adjacent to the hip replacement and their increase in number on follow-up indicates loosening
504
A 30-year-old female runner presents with a history of pain in the legs on running. Plain radiographs are unremarkable. An isotope bone scan reveals subtle, longitudinal, linear uptake on the delayed bone scan images, with normal angiogram and blood pool images. What is the diagnosis? A. Stress fracture. B. Shin splints. C. Osteoid osteoma. D. Osteomyelitis. E. Hypertrophic osteoarthropathy.
B. Shin splints. Excessive exertion of tibialis and soleus muscles of the legs causes periostitis along the muscular attachments. This results in longitudinal linear uptake on delayed bone scan images. The angiogram and blood pool images are usually normal compared to stress fracture, which is associated with hyperperfusion and hyperaemia. On delayed images focal fusiform uptake is seen with stress fracture. Infection is associated with hyperperfusion, hyperaemia, and focal increased uptake. Osteoid osteoma demonstrates hyperperfusion, hyperaemia, and focal double density due to nidus and reactive osteosclerosis. Paget’s disease is associated with increased uptake in an enlarged and deformed bone. Age and clinical presentation are also against this diagnosis. Hypertrophic osteoarthropathy is associated with irregular cortical uptake producing the ‘tramline’ sign.
505
A 25-year-old man presents with a painful knee. A plain fi lm reveals a lucent area with a wide zone of transition in the distal femoral metaphysis. MRI reveals fl uid–fl uid levels. What is the most likely diagnosis? A. Aneurysmal bone cyst. B. GCT. C. Osteosarcoma. D. Chondroblastoma. E. Osteoblastoma.
C. Osteosarcoma. The telangiectatic variety of osteosarcoma does show fl uid–fl uid levels, as does malignant fi brous histiocytoma or any necrotic bone tumour. Telangiectatic osteosarcoma is highly vascular and contains necrotic tissue and blood, with tumour located only along the periphery and septa. MRI will thus reveal enhancing nodularity in the latter locations; this fi nding will be absent in the case of ABC or GCT. In addition to those mentioned in the stem, the plain fi lm fi ndings include bone expansion and cortical breakthrough. Unlike the other lesions, osteoblastoma does not demonstrate fl uid–fl uid levels on MRI; it is more common in the posterior elements of the spine than in the long bones. ABC, GCT, and chondroblastoma have a narrower zone of transition on plain fi lm than telangiectatic osteosarcoma. GCT is subarticular. Chondroblastoma is epiphyseal. Other benign causes of fl uid–fl uid levels include simple bone cysts and fi brous dysplasia
506
A 39-year-old man presents with a gradually enlarging swelling in the upper lateral aspect of the right calf. He is also experiencing some numbness affecting the dorsum of his right foot. An ultrasound scan and subsequently an MRI scan demonstrate a well-defi ned, thinly septated cystic lesion intimately related to the proximal tibio-fi bular joint and extending into the adjacent soft tissues. It measures approximately 4cm in maximum diameter. There is no enhancement of the soft tissue component post injection of gadolinium. What is the most likely diagnosis? A. Parameniscal cyst. B. Bursitis. C. Focal tenosynovitis. D. Ganglion cyst. E. Chronic seroma.
D. Ganglion cyst. What is being described is a ganglion cyst adjacent to the proximal tibiofi bular joint that is causing a common peroneal nerve palsy. This is a well-recognized entity. Ganglion cysts can be uni- or multilocular. They occur predominately in peri-articular locations and may arise from tendon sheaths, joint capsules, bursae, or ligaments. Although parameniscal cysts can extend inferiorly from the lateral knee joint margin, they typically show a communication with a meniscal tear. This is not described in the radiological fi ndings and they are not typically centred at the level of the proximal tibiofi bular joint. Bursal distension can cause a multiloculated fl uid collection. It can occur in typical locations around the knee joint, but not usually adjacent to the proximal tibiofi bular joint. Examples include pes anserine bursitis, semi-membranosis-tibial collateral ligament bursitis, and pre-, supra-, and infrapatellar bursitis. Focal tenosynovitis and chronic seroma do not particularly fi t with the clinical and radiological fi ndings.
507
A 56-year-old woman is referred for MR arthrography of her right shoulder for query rotator cuff tear. You are asked to explain the procedure to a group of medical students attached to the department. What is the advantage of using a fat-suppressed T1WI sequence? Differentiating partial from full thickness tear. Identify bursal fl uid collections. Differentiating inadvertent air injection from intra-articular loose body. Diagnosing capsular laxity. Detecting incidental bone marrow lesions.
A. Differentiating partial from full thickness tear. MR arthrography is most helpful for outlining labral-ligamentous abnormalities in the shoulder and distinguishing partial thickness from full thickness tears in the rotator cuff. The technique involves injection of diluted gadolinium mixed with iodinated contrast, which allows fl uoroscopic confi rmation of intra-articular needle placement. Partial and full thickness tears may not be distinguishable on standard T1WI because fat and gadolinium have similar signal intensities. This is especially the case when cuff tendons show contrast solution extending to the bursal surface but not defi nitively through it. This problem can be overcome with use of fat suppression. MR arthrography should include a T2WI sequence to identify bursal fl uid collections and tears. T2WI is also helpful in characterizing incidental bone marrow lesions. Inadvertent injection of gas may lead to a false-positive diagnosis of intra-articular loose bodies, but gas bubbles will rise to non-dependent regions, whereas loose bodies will gravitate to dependent locations. No accurate MR imaging criteria are recognized in the diagnosis of capsular laxity.
508
A 40-year-old female presents with a small lump in her foot. An MRI of the foot demonstrates a small soft tissue mass, which has homogenous low signal on T1WI and T2WI. The mass enhances with gadolinium. What is the most likely diagnosis? A. Morton’s neuroma. B. Lipoma. C. Ganglion cyst. D. Plantar fi bromatosis. E. Hemangioma.
D. Plantar fi bromatosis. Fibrous masses containing mature collagen are homogenously low in signal on T1WI and T2WI sequences, and demonstrate enhancement with gadolinium. Common fi brous masses in the foot are plantar fi bromatosis and fi broma of the tendon sheath. Morton’s neuroma is typically intermediate in signal on T1WI and low on T2WI with variable contrast enhancement. Lipomas follow fat signal intensity. They are high on T1WI and T2WI, and low on fat-suppressed sequences. A ganglion cyst follows fl uid signal. Ganglion cysts are low on T1WI and high on T2WI with rim enhancement. Haemangiomas are of mixed signal on T1WI and T2WI due to the presence of vessels, fat, and fi brous tissue. The vascular portions of hemangiomas enhance homogenously.
509
A 56-year-old woman is referred for MR arthrography of her right shoulder for query rotator cuff tear. You are asked to explain the procedure to a group of medical students attached to the department. What is the advantage of using a fat-suppressed T1WI sequence? A. B. C. E. Differentiating partial from full thickness tear. Identify bursal fl uid collections. Differentiating inadvertent air injection from intra-articular loose body. D. Diagnosing capsular laxity. Detecting incidental bone marrow lesions.
A. Differentiating partial from full thickness tear. MR arthrography is most helpful for outlining labral-ligamentous abnormalities in the shoulder and distinguishing partial thickness from full thickness tears in the rotator cuff. The technique involves injection of diluted gadolinium mixed with iodinated contrast, which allows fl uoroscopic confi rmation of intra-articular needle placement. Partial and full thickness tears may not be distinguishable on standard T1WI because fat and gadolinium have similar signal intensities. This is especially the case when cuff tendons show contrast solution extending to the bursal surface but not defi nitively through it. This problem can be overcome with use of fat suppression. MR arthrography should include a T2WI sequence to identify bursal fl uid collections and tears. T2WI is also helpful in characterizing incidental bone marrow lesions. Inadvertent injection of gas may lead to a false-positive diagnosis of intra-articular loose bodies, but gas bubbles will rise to non-dependent regions, whereas loose bodies will gravitate to dependent locations. No accurate MR imaging criteria are recognized in the diagnosis of capsular laxity.
510
A patient is referred from the dialysis unit with a history of joint and muscular pain. In particular they complain of bilateral hand pain and hip pain. The plain fi lms of both hands show a loss of distinction of the radial aspect of the phalanges of the index and middle fi ngers. There is an area of para-articular soft tissue calcifi cation noted adjacent to the middle fi nger metacarpal of the right hand. The pelvic x-ray is distinctly abnormal. There is a large expansile lucent lesion in the right iliac bone, which has a narrow zone of transition and no evidence of internal matrix. The bony defi nition of the rest of the pelvic bone reveals a coarsened trabecular pattern, but no evidence of expansion of the bones. There are multiple small linear lucencies noted along the medial aspect of the femurs bilaterally, which demonstrate a periosteal reaction. What condition do you think this patient has? A. Primary hyperparathyroidism. B. Secondary hyperparathyroidism. C. Paget’s disease. D. Osteomalacia. E. Renal osteodystrophy.
E. Renal osteodystrophy. The fi rst important observation to note is the referral route. A patient from the dialysis unit is going to have renal failure, thereby excluding tertiary hyperparathyroidism and making primary less likely. This leaves Paget’s disease, osteomalacia, and renal osteodystrophy. The patient clearly has features of both hyperparathyroidism (subperiosteal resorption, brown tumour in iliac bone) and osteomalacia (Looser’s zones), giving the diagnosis of renal osteodystrophy, which has features of both. Another feature of renal osteodystrophy is the soft-tissue calcifi cation noted in the hand. The Looser’s zones are small stress fractures on the load-bearing aspect of a bone, caused by osteomalacia. This contrasts with the stress fractures seen on the tensile aspect of bones (i.e. lateral aspect of femur) seen in Paget’s disease and fi brous dysplasia. Whilst Brown tumours are more closely associated with primary hyperparathyroidism, the majority actually occur in secondary hyperparathyroidism as this disease is much more prevalent
511
A 62-year-old male with a known diagnosis of bronchogenic carcinoma presents with pain and swelling of his wrists. What radiographic features are consistent with hypertrophic pulmonary osteoarthropathy? A. Metaphyseal lamellar periosteal reaction. B. Irregular epiphyseal periosteal proliferation. C. Asymmetrical, thick ‘feathery’ periosteal reaction. D. Cortical thickening and trabecular coarsening. E. Symmetrical, solid periosteal new bone formation.
A. Metaphyseal lamellar periosteal reaction. Hypertrophic pulmonary osteoarthropathy is a paraneoplastic syndrome secondary to the release of vasodilators. It typically causes burning pain and swelling, with the ankles and wrists being most commonly affected. Pulmonary causes include bronchogenic carcinoma, mesothelioma, and pleural fi broma. Radiographs demonstrate cortical thickening and lamellar periosteal proliferation in a diametaphyseal location. Bone scintigraphy will demonstrate patchy linear increased uptake along the cortical margins. Option B describes pachydermoperiostosis, a self-limited condition in adolescents. Option C is typical of thyroid acropachy. Cortical thickening and trabecular coarsening is a feature of Paget’s disease and symmetrical, solid periosteal new bone formation is described in hypervitaminosis A.
512
A 57-year-old female patient with a history of multiple myeloma is referred for imaging due to a history of arthralgia primarily affecting the hands. The patient describes early morning stiffness that eases through the day. The clinicians report a fi nding of synovitis clinically. Blood results have revealed a raised ESR. Hand x-rays are carried out which reveal sharply defi ned intra-articular marginal erosions at the MCP joints of the index and middle fi ngers bilaterally. The joint spaces are well preserved. There are also well- marginated subchondral cysts noted in the carpal bones, again with joint space preservation. Soft tissue nodules are noted around the wrist joints, which are not calcifi ed. There is no evidence of juxta-articular osteopenia. No osteophytes are noted. What diagnosis is most strongly suggested by these fi ndings? A. Gout. B. CPPD. C. RA. D. Amyloidosis. E. Wilson’s disease.
D. Amyloidosis. There are a lot of conditions that are capable of mimicking RA. In these cases a few key features can help reach a diagnosis. The classic fi nding in gout is of non-marginal erosions, as opposed to those described. Nevertheless, marginal erosions can occur with gout. An RA-type picture in the presence of non-marginal erosions or calcifi ed soft-tissue nodules (tophi) should suggest this diagnosis. CPPD gives a more productive pattern of arthritis, such as seen with OA, affecting the radio-carpal joint. Thus, it is often suspected when the appearance is of OA with a ‘funny distribution’. Amyloidosis is suggested fi rst by the history of MM. Involvement of the hands is more commonly seen in amyloid secondary to prolonged dialysis, but can be seen when the amyloid is secondary to MM, when the wrists are often affected. Amyloid can closely resemble RA in its distribution and the pattern of erosions. However, three important features can help differentiate: amyloidosis classically preserves the joint space, is not usually associated with periarticular osteopenia, and amyloidosis causes well-demarcated subchondral cyst formation in excess to that expected from the degree of joint disease
513
A 24-year-old man undergoes acute trauma to his right knee playing football. He is unable to weight bear. An x-ray of the right knee is performed and this demonstrates a large joint effusion and a small, avulsed elliptical fragment of bone at the medial aspect of the proximal tibia at the joint margin. Which knee structure is likely to be deranged in association with this injury at a subsequent MRI? A. Anterior cruciate ligament. B. Posterior cruciate ligament. C. Lateral collateral ligament. D. Patellar tendon. E. Lateral meniscus.
B. Posterior cruciate ligament. The avulsion injury described is a reverse Segond fracture. This injury is known to be associated with both mid-substance tears of the posterior cruciate ligament and avulsions of the PCL from the posterior tibial plateau. They can also be associated with medial meniscus injuries. They are not to be confused with a Segond fracture, which is a small elliptical fragment of bone avulsed from the lateral tibial plateau at the lateral joint margin, best seen on the AP view of the knee. They have a strong association with tears of the anterior cruciate ligament and also meniscal tears.
514
A 35-year-old man presents with pain, swelling, and reduced movement of his knee. A plain fi lm reveals a joint effusion, well-defi ned erosions with preservation of joint space, and normal bone mineralization. An MRI reveals, in addition, a mass in the region of the femoro-tibial joint space with low signal on T1WI and T2WI, and blooming artefact on GE imaging. What is the most likely diagnosis? A. Synovial cell sarcoma. B. Regional migratory osteoporosis. C. Gout. D. Synovial chondromatosis. E. PVNS.
E. PVNS. This is a monoarticular tumour-like proliferation of synovium that occurs in joints, bursae, and tendon sheaths. It may be focal or diffuse. It occurs most frequently in the knee (80% of cases), then the hip, ankle, shoulder, and elbow. The abnormal synovium is prone to haemorrhage, thus producing blooming artefact on GE sequences, secondary to haemosiderin deposition. In general the classic MRI appearance is variable low signal intensity on all sequences (T2WI signal being more variable due to fat, oedema, and blood products). Early changes involve a focal mass and joint effusion. Subsequently large erosions, synovial hypertrophy, and subchondral cysts may occur. Joint space is preserved until advanced disease is present and bone density is normal. After IV contrast at CT, PVNS shows variable enhancement, which can be striking. The differential diagnosis includes diseases causing recurrent haemarthroses, e.g. haemophilia and haemochromatosis (PVNS is monoarticular) as well as gout, amyloid, synovial chondromatosis, and tuberculosis. Some 90% of synovial cell sarcomas do not originate from a joint. They are usually isointense to muscle on T1WI, with heterogeneous high-signal intensity on T2WI. Regional migratory osteoporosis would obviously involve loss of bone mineralization, as well as marrow oedema. Gout demonstrates typically ‘rat’s bite’ para-articular erosions and soft-tissue calcifi cation; when it involves the knee it tends to affect the patello-femoral compartment.
515
You are reviewing the x-rays of a child that are stored in your department’s museum. Sequential radiographs have been taken as the child has aged and the appearances have become more pronounced with time. The child has a form of dwarfi sm. On the CXR you notice ‘oar-shaped’ ribs. The metacarpals are short and wide, but narrow proximally, giving a fan-like appearance. The patient has a J-shaped sella turcica. The iliac wings are wide, but the iliac bones narrow inferiorly. On the lateral lumbar spine, the vertebra have central anterior beaks. A clinical vignette mentions that the patient was not intellectually impaired. What condition does the patient probably have? A. Campomelic dysplasia. B. Niemann–Pick disease. C. Morquio syndrome. D. Achondroplasia. E. Hurler’s syndrome.
C. Morquio syndrome. The constellation of skeletal manifestations describes the characteristic appearance of dyostosis multiplex. This pattern of skeletal abnormalities is seen with the mucopolysaccharidoses (MPS), although it can also be seen with other storage disorders. With the exception of Hurler’s syndrome, where the manifestations are present at 1 year of age, the skeletal manifestations progress as the patients get older. Hurler’s and Morquio’s are the most common of the MPS conditions. Amongst the MPS conditions, Morquio’s stands out as a favourite for single best answer (SBA) and viva questions as it is the only MPS where the patient is not intellectually impaired. It also displays a central anterior vertebral body beak, whereas the other conditions have an anterior beak in the lower third of the vertebral body. The differences with achondroplasia are the progression with age and the pelvic shape. The pelvis in achondroplasia has widened iliac wings, with horizontal acetabular roofs and a narrow inlet, giving the classic ‘champagne glass’ appearance. The anterior beak in achondroplasia is also in the lower third of the vertebral body.
516
A 21-year-old patient attends the A&E department following a minor injury with a suspected fracture. The request form states that the patient has osteogenesis imperfecta. It is noted that the patient is of reduced stature and does not display any evidence of blue sclera, but that the colouration of his sclera has faded over time. He has normal hearing. What subtype of osteogenesis imperfecta does he likely have? A. Type I. B. Type II. C. Type III. D. Type IV. E. Type V.
72. D. Type IV. Osteogenesis imperfecta in an adult is almost always type I or IV. Type I is the most common. Patients have can have normal stature and the characteristic blue sclera are seen in 90%. Patients also often have hearing impairment. Type IV has variable bone fragility, from mild to severe. Hearing impairment is less common, as is reduced stature. Blue sclera are present in children, but are often absent after adolescence. Type II is universally fatal in the neonatal period. Type III is also severe and often associated with reduced lifespan. Stature is signifi cantly reduced. In patients who survive to adolescence the blue sclera are also often absent. Type V is not universally recognized, but is similar to type IV.
517
A small bowel series is requested for a patient who has a history of systemic sclerosis. Which of the following is a feature of small bowel systemic sclerosis? A. Stacked coin appearance due to infi ltration of small bowel loops. B Pseudo-diverticula affecting the anti-mesenteric side of the bowel. C. Decreased intestinal transit time. D. Small bowel systemic sclerosis is only seen in 10% of patients with systemic sclerosis, but the disease is rapidly progressive when it is present. E. Pneumatosis intestinalis.
E. Pneumatosis intestinalis. The stacked coin appearance is seen secondary to intramural haemorrhage—the appearances of systemic sclerosis are of tightly packed folds of normal thickness in a dilated portion of bowel, which has been given the title ‘accordion’ or ‘hidebound’ bowel. The pseudo-diverticula (10–40%) are seen on the mesenteric side of the bowel, unlike colonic diverticula. The transit time is prolonged, as there is reduced intestinal motility. Another classical feature is of a markedly dilated duodenum, due to the loss of the enteric innervations—mega duodenum. This classically terminates abruptly at the level of the superior mesenteric artery (SMA). Pneumatosis cystoides can occur in systemic sclerosis of the small bowel. Small bowel disease is seen in up to 40% of patients with systemic sclerosis and indicates rapidly progressing disease.
518
A 26-year-old female presents with a 1-day history of right iliac fossa (RIF) pain. She is mid-cycle and prone to mittleschmerz-type pain, but reports that this pain is more severe than previously. Serum infl ammatory markers are elevated. Clinical examination reveals tenderness in the RIF, but no rebound. Due to the compounding gynaecological history, a CT is requested. This reveals a thickened caecum and thickened appendix, which appears to have a defect in the wall on the multiplanar reformatted images. There is a calcifi ed density present in the orifi ce of the appendix. There is a loculated fl uid collection adjacent to the appendix, which has air bubbles within it. There is also fl uid in the pelvis. A perforated appendix is removed at surgery. Which of the CT fi ndings is most specifi c for detecting a perforated appendix? A. Presence of a faecolith. B. Identifi cation of a wall defect. C. Fluid in the pelvis. D. Adjacent abscess formation. E. Enlarged regional lymph nodes.
D. Adjacent abscess formation. Abscess formation has been found to be the most specifi c fi nding in appendiceal perforation, along with extraluminal gas and small bowel ileus. Abscess formation is also one of the least sensitive fi ndings. Regional mesenteric lymph nodes are the most sensitive, but are reasonably non-specifi c. A focal wall defect, if seen, is reasonably sensitive and specifi c. Appendicolith is only found in 50% on CT and has a specifi city of 70%.
519
A patient is being worked up for a pancreatic neoplasm to assess potential resectability. Which one of the following does not rule out surgery? Extension of the tumour beyond the margins of the pancreas into duodenum. Tumour involvement of adjacent organs. Enlarged peripancreatic lymph nodes (>15 mm). Encasement or obstruction of superior mesenteric vessels. E. Peritoneal carcinomatosis.
C. Enlarged peripancreatic lymph nodes (>15mm). Enlarged regional nodes are a sign of unresectability, but nodes adjacent to the pancreas are resected as part of Whipple’s procedure. The other factors all indicate that a pancreatic lesion is unresectable. Other features of unresectable pancreatic carcinoma are liver metastases. Only 10–15% of pancreatic neoplasms are resectable at presentation.
520
A 68-year-old male patient has a 20-year history of RA. During a recent fl are he was commenced on steroid therapy, although this has now been discontinued. The patient is now complaining of mild abdominal discomfort, diarrhoea, and mild weight loss. A barium meal is performed, but is suboptimal, as the patient is poorly mobile. Within the limitations of the study, there is reduced peristalsis in the oesophagus and mild refl ux. The antrum of the stomach is felt to be mildly narrowed and rigid. Thickened rugal folds are noted. A subsequent small bowel series is carried out. The jejunal folds measure 4 mm and the ileal folds appear more plentiful and measure 3 mm. Contrast is present in the caecum at 4 hours. Spot screening of the terminal ileum reveals the same fi ndings as those described above. What is the most likely diagnosis? A. Gastric erosions. B. Whipple’s disease. C. Mastocytosis. D. Amyloidosis. E. Crohn’s disease
D. Amyloidosis. This patient probably has amyloidosis secondary to prolonged RA. GI involvement is more common in primary (70%) than secondary (13%) amyloidosis. Nevertheless, the small bowel is involved in 74% of cases of GI amyloidosis and secondary amyloidosis is the most common type of amyloid disease. Amyloidosis is secondary to the deposition of insoluble amlyoid protein in soft tissues and organs. In primary amyloidosis the heart (90%), followed by the small bowel and the lungs (70%), are the most commonly affected organs. The kidneys are affected in 90% of cases of secondary amyloidosis. Amyloidosis classically causes a diffuse thickening of bowel folds. It may cause dilated bowel folds, if the myenteric plexus is involved. The main differential for amyloid is Whipple’s disease and intestinal lymphangiectasia. Whipple’s disease does not cause bowel dilatation or rigidity, as described in the antrum in this patient. Crohn’s disease can also present with thickened folds, but it is more commonly focal with the most pronounced abnormality in the terminal ileum. Ulceration is also commonly seen in Crohn’s, but 68 years old would be a late fi rst presentation for Crohn’s. Whilst option A is true, this is not what the question asked. Patients with mastocytosis most commonly present in infancy.
521
A 35-year-old male patient from the Indian subcontinent presents with a 2-month history of lower abdominal pain, per rectum (PR) bleeding, and weight loss. His haemoglobin is 9.4 and C-reactive protein (CRP) is 123. The patient is tender in the RIF. A CT scan is performed due the suspicion of appendiceal pathology, but unusual history. This shows bowel wall thickening of the terminal ileum with mild proximal bowel dilatation. The inner bowel wall is hypodense with enhancement of the outer bowel wall. There is stranding in the fat, which causes mass effect displacing other loops of bowel. Mild regional adenopathy is noted. The appendix is not visualized, but the caecum appears normal. There is a similar area of bowel wall thickening in the sigmoid colon. What is the most likely diagnosis? A. Yersinia. B. Tuberculosis. C. Lymphoma. D. Crohn’s disease. E. Carcinoid.
D. Crohn’s disease. The fi ndings described are classical for Crohn’s disease and lymphadenopathy is seen in up to 30% of cases. Tuberculosis more typically involves the caecum. Lymphoma usually causes a nodular appearance to the bowel. It is not associated with stricturing of the affected segment and is more classically associated with dilatation of the affected segment due to destruction of the myenteric plexus.
522
A 40-year-old female undergoes MRI of the liver, which demonstrates a 5-cm lesion that is isointense to liver on T1WI and slightly hyperintense on T2WI. It has a central scar that is hypointense on T1WI and hyperintense on T2WI. On contrast-enhanced dynamic MRI, the lesion is hyperintense in the arterial phase, and isointense to liver in the portal venous phase with delayed fi lling in of the central scar. What is the diagnosis? A. Hepatic adenoma. B. Fibrolamellar hepatoma. C. Hypervascular metastasis. D. Focal nodular hyperplasia (FNH). E. Giant haemangioma.
D. Focal nodular hyperplasia (FNH). This is the second most common benign liver tumour. It is thought to represent a hyperplastic response of hepatocytes to an underlying vascular malformation. It is most common in young adult females and is usually an asymptomatic solitary lesion. On histology, FNH consists of hyperplastic hepatocytes and small bile ductules around a central scar. The bile ductules of FNH do not communicate with the adjacent biliary tree. At ultrasound, FNH is isoechoic or hypoechoic. Colour Doppler may show prominent central vascularity. At CT, FNH is typically slightly hyperattenuating or isoattenuating to surrounding liver on precontrast images. On post contrast images, FNH is hyperattenuating in the arterial phase and isoattenuating in the portal venous phase with hypoattenuating central scar. The scar shows delayed enhancement. At MRI, FNH is iso- to hypointense on T1WI and slightly hyper- to isointense on T2WI. The central scar is hypointense on T1WI and hyperintense on T2WI. The enhancement pattern is similar to that on CT. If the appearances are atypical, MRI with hepatocyte-specifi c contrast agent (gadobenate dimeglumine) may be useful in confi rming the hepatocellular origin of the mass. With gadobenate dimeglumine, FNH is iso- to hyperintense on the 1–3 hour delayed images in over 96% of cases.
523
A 45-year-old male presents with a history of jaundice and RUQ pain. An ultrasound of the abdomen demonstrates an impacted calculus in the gallbladder neck with dilatation of the intrahepatic ducts. An MRCP is requested to exclude Mirizzi syndrome. What additional features on MRCP confi rm the diagnosis of Mirizzi syndrome? A. Dilated common hepatic duct. B. Dilated common hepatic and common bile ducts. C. Dilated common hepatic duct with normal common bile duct. D. Double duct sign. E. Normal ducts.
C. Dilated common hepatic duct with normal common bile duct. Mirizzi syndrome is a functional hepatic syndrome caused by extrinsic compression of the CHD by a calculus impacted in the gallbladder neck or cystic duct. Low insertion of the cystic duct into the CHD is a predisposing factor. Typical features at imaging include extrinsic compression of the CHD, a gallstone in the gallbladder neck or cystic duct, dilatation of the intrahepatic ducts and CHD proximally, and a normal CBD. Rarely, infl ammation around an impacted calculus leads to a stricture formation mimicking a periductal infi ltrating cholangiocarcinoma.
524
A 55-year-old female with cirrhosis undergoes MRI of the liver, which demonstrates multiple small nodules that are hypointense on T2WI and enhance following administration of gadolinium in the arterial and portal venous phase. The nodules demonstrate uptake of hepatocellular agent and super paramagnetic iron oxide (SPIO) particles. What is your diagnosis? A. Multifocal hepatocellular carcinoma (HCC). B. Siderotic nodules. C. Dysplastic nodules. D. Regenerative nodules. E. Multiple arterio-venous shunts.
D. Regenerative nodules. Regenerative nodules are formed in response to necrosis and altered circulation. They remain enhanced in the portal venous phase as opposed to HCC, which typically demonstrates contrast washout in the portal venous phase. Regenerative nodules have normal hepatocellular function and Kupffer cell density and therefore demonstrate uptake of both hepatocellular agents and SPIO particles. As dedifferentiation proceeds, the hepatocellular function and Kupffer cell density reduce.
525
A 62-year-old male with acute myocardial infarction develops abdominal discomfort and deranged liver function tests. A CT scan of the abdomen demonstrates heterogeneous liver enhancement, poor enhancement of the hepatic veins and inferior vena cava (IVC), ascites and bibasal pleural effusions. What additional feature would favour a diagnosis of passive hepatic congestion instead of acute Budd–Chiari syndrome? A. Flip-fl op enhancement pattern of the liver. B. Absent fl ow in hepatic veins. C. Dilated hepatic veins and IVC. D. Enlarged caudate lobe. E. Hepatomegaly.
C. Dilated hepatic veins and IVC. Elevated right atrial/central venous pressure due to cardiac decompensation results in impaired venous drainage from the liver, producing passive hepatic congestion. If prolonged, passive hepatic congestion can result in cardiac cirrhosis. On CT imaging, retrograde enhancement of dilated IVC and hepatic veins is seen in the arterial phase. In the portal venous phase, there is delayed/reduced enhancement of the hepatic veins due to impaired venous drainage. There is heterogenous enhancement of the liver parenchyma due to venous stasis. Other features of cardiac failure may be evident. Acute Budd–Chiari syndrome is characterized by narrowed hepatic veins and intrahepatic IVC (secondary to compression by the enlarged liver) and by fl ip-fl op pattern of enhancement between the arterial and the portal venous phases.
526
A 47-year-old male patient undergoes an MRI examination for further characterization of an adrenal lesion. Axial gradient T1 in- and out-of-phase sequences confi rm the benign nature of the adrenal lesion. Incidentally, the liver and pancreas demonstrate a signal drop on the in-phase images compared to out-of-phase images. What is your diagnosis and what additional sequence would confi rm the diagnosis? A. Diffuse fatty infi ltration. GE T2WI. B. Diffuse fatty infi ltration. SE T2WI. C. Haemochromatosis. SE T2WI. D. Haemochromatosis. GE T2WI. E. Haemosiderosis. GE T2WI. F. Haemosiderosis. SE T2WI.
D. Haemochromatosis. GE T2WI. A dual GE T1 in- and out-of-phase sequence is routinely used in identifying lipid content within an adrenal lesion. It is based on the phase interference effect. When the fat and water signals are in-phase, there is constructive interference and when they are out-of-phase there is destructive interference. This results in signal drop-off on the out-of-phase sequence. The reverse effect of decreased signal intensity on in-phase images compared to out-of- phase ones is seen in iron deposition diseases. This is because the echo time for the in-phase sequence is longer than for the out-of-phase sequence, therefore the in-phase sequence is more susceptible to the paramagnetic (dephasing) effects of iron. Haemochromatosis is an autosomal recessive genetic disorder. There is abnormal deposition of iron in parenchymal organs such as the liver, pancreas, heart, etc. In haemosiderosis or secondary haemochromatosis, iron deposition is seen in the reticuloendothelial system of the liver, spleen, and bone marrow. This type of deposition is not associated with tissue damage. A GE T2 sequence demonstrates signal loss due to the magnetic fi eld inhomogeneity produced by the paramagnetic effects of iron. GE sequences are more susceptible to the paramagnetic effects than SE sequences, as there is no 180° rephasing pulse in gradient sequences.
527
A 55-year-old man presents with dysphagia. He gives no history of weight loss and investigations reveal a normal full blood picture. He is referred for a barium swallow, which reveals a long stricture (several centimetres) in the mid to distal oesophagus with a fi ne reticular pattern adjacent to the distal aspect of the stricture and distal oesophageal widening. What is the most likely diagnosis? A. Refl ux oesophagitis. B. Candidiasis. C. Barrett’s oesophagus. D. Oesophageal adenocarcinoma. E. Hiatus hernia.
C. Barrett’s oesophagus. This represents progressive columnar metaplasia of the distal oesophagus secondary to refl ux oesophagitis. It is a premalignant condition associated with an increased risk of adenocarcinoma, 40-fold that of the general population. Strictures are more common in the distal, then mid oesophagus, rather than the classically described proximal third. The typical fi nding is of 1-cm- long strictures or ulceration with associated gastro-oesophageal refl ux and hiatus hernia. These fi ndings are non-specifi c and may result from a variety of other causes such as corrosive ingestion, nasogastric intubation, Crohn’s disease, or neoplasm (primary or secondary). However, the presence of a fi ne reticular pattern extending distally from the stricture appears to be specifi c for Barrett’s. A reticulonodular pattern has been described in patients with a superfi cial spreading adenocarcinoma, but this is rare and not classically associated with a stricture.
528
A patient presents to A&E with severe upper abdominal pain 4 days following a barium enema. There is no free air under the diaphragm on the erect CXR. There is mild elevation of the infl ammatory markers, but the surgeon is concerned with the degree of peritonism and requests a CT scan of abdomen. On this, the small bowel is dilated to 5 cm, but is not thick walled. The vascular structures enhance normally. There is infl ammatory change noted around the duodenum. Linear areas of low attenuation are noted extending from the porta hepatis into the liver parenchyma. These do not extend to the margin of the liver and are in general central in their location. The Hounsfi eld attenuation value of these areas is approximately –1500 HU. Barium in the rectum obscures the images of the pelvis. What is the most likely pathology? A. Cholecystoduodenal fi stula. B. Mesenteric infarction. C. Acute bowel obstruction. D. Perforated duodenal ulcer. E. Complication of barium enema
A. Cholecystoduodenal fi stula. The other answers are all causes of portal air, whereas the salient description is for air in the biliary tree.
529
A 50-year-old woman presents with dysphagia. At barium swallow, contrast passes sluggishly into the oropharynx. No peristaltic waves are seen in the upper oesophagus. After swallowing, the lumen of the hypopharynx and upper oesophagus remain patent and distended. The lower oesophagus outlines normally. What is the most likely diagnosis? A. Achalasia. B. Scleroderma. C. Polymyositis. D. Chagas disease. E. SLE.
C. Polymyositis. This condition and dermatomyositis affect skeletal muscle, which is found at the upper third of the oesophagus. These conditions begin in the upper oesophagus and extend caudally. Other fi ndings at fl uoroscopy include retention of barium in the valleculae and wide atonic pyriform fossae, regurgitation and nasal refl ux, aspiration, and failure of contrast to progress in the upper oesophagus without the aid of gravity. Polymyositis and dermatomyositis are associated with underlying malignancy. The latter also involves a heliotrope rash and Gottrons papules on fl exor surfaces. The lower oesophagus is composed of smooth muscle and is affected by conditions such as scleroderma and SLE, which result in atony and lack of peristalsis in the lower two-thirds, beginning caudally and moving cranially. Achalasia and Chagas disease result in dilatation of the whole oesophagus, with a ‘rat-tail’ deformity at the lower end.
530
A 58-year-old man with a history of alcohol abuse and diabetes presents with painless jaundice. Liver function tests reveal an obstructive picture and he undergoes an ultrasound of abdomen, which reveals dilatation of the CBD and a hypoechoic region in the head of the pancreas. He has a history of iodine allergy and undergoes MRI with dynamic gadolinium enhancement, as an alternative to contrast-enhanced CT. Which fi nding in the pancreatic head is most in keeping with the diagnosis of pancreatic adenocarcinoma? A. Hypointensity on T1WI. B. Hyperintensity on T2WI. C. Hyperintensity on a STIR sequence. D. Hypointensity during arterial phase enhancement. E. Hypointensity during portal venous phase enhancement.
D. Hypointensity during arterial phase enhancement. Pancreatic adenocarcinoma is generally a hypovascular tumour at CT as well as MRI. Dynamic contrast-enhanced CT has been the gold standard for the diagnosis of pancreatic adenocarcinoma, but MRI is of value in those with renal failure or sensitivity to iodine-based contrast media. Care should be taken in patients with a very low GFR (typically less than 30ml/ min) because of the risk of nephrogenic systemic fi brosis. Contrast-enhanced MRI may have a lower false negative rate than CT, as approximately 10% of pancreatic adenocarcinomas have been shown to be iso- rather than hypoattenuating, on both the pancreatic and portal venous phases. Chandarana et al. have reported that 25 of 25 neoplasms showed hypointensity during arterial phase enhancement (and 20 remained hypointense in venous phase), whereas only 12 of 25 were hypointense on unenhanced T1WI, and only 11 of 25 were hyperintense on STIR/T2WI.
531
A 50-year-old male patient is admitted with congestive cardiac failure and undergoes a CT scan of the abdomen, which shows tortuous and prominent intrahepatic and extrahepatic arterial branches with early fi lling of dilated hepatic veins and IVC. The arterial phase scan shows mosaic perfusion with multiple enhancing foci. In the portal venous phase there is homogenous enhancement of the liver, with the prominent hepatic veins and IVC noted. What is the diagnosis? A. Passive hepatic congestion. B. Budd–Chiari syndrome. C. Osler–Weber–Rendu syndrome. D. Multifocal transient hepatic attenuation differences. E. Von Meyerburg complex.
C. Osler–Weber–Rendu syndrome. Osler–Weber–Rendu syndrome or hereditary haemorrhagic telangiectasia (HHT) is a rare autosomal dominant multisystem vascular disorder characterized by angiodysplastic lesions in which there is communication between arteries and veins of varying sizes. It commonly affects the skin, lungs, and mucous membranes but any organ system may be involved. The liver is the most common site of abdominal HHT. Lesions range from tiny telangiectases to transient perfusion abnormalities and large confl uent vascular masses. Coronal maximum intensity projection (MIP) images are useful in appreciating telangiectases. Liver involvement is associated with arterio-venous shunting, porto-venous shunting, or both, resulting in hyperdynamic circulation, which may lead to high-output cardiac failure. Budd–Chiari syndrome is hepatic vein thrombosis and Von Meyerburg complex is multiple biliary hamartomas.
532
A 39-year-old male complains of severe, colicky left lower abdominal pain and rectal bleeding. He has experienced intermittent abdominal pain for the last 3–4 months. There is no previous history of medical problems. On examination he has left lower abdominal tenderness without signs of peritonism. A CT examination is performed which reveals a focal intraluminal abnormality, with the appearance of a mass within the sigmoid colon. There are concentric rings of soft tissue and fatty attenuation giving a ‘target’ like appearance. Mesenteric vessels are seen to course into the lesion. At the most distal point of the abnormality there is a more discrete low attenuation mass measuring approximately 3 cm in size of fatty attenuation. The large bowel distal to the sigmoid lesion is collapsed and proximal to the lesion there are multiple loops of small bowel and a dilated colon. What is the most likely underlying pathology for this condition? A. Benign tumour. B. Malignant tumour. C. Inverted diverticulum. D. Idiopathic. E. Infl ammatory bowel disease.
A. Benign tumour. The patient has presented with a colo-colic intussusception secondary to a lipoma of the colonic wall. The intussusception has been complicated by large bowel obstruction. Intussusception is caused by prolapse of a portion of the bowel into the lumen of the adjoining bowel lumen segment. Intussusception is most commonly a disease of young children and most commonly occurs in an ileocolic location. In children, over 90% of intussusceptions are idiopathic and no lead point is identifi ed. In adult patients approximately 80% of intussusceptions are caused by lead point lesions. The CT appearances described are typical of an intussusception. The low attenuation distal lead point represents the lipoma. Ultrasound is often used to diagnose intussusception in children and will show a mass with echogenic rings representing the fat in the invaginated mesentery.
533
A 45-year-old man has a long history of intermittent diarrhoea, abdominal bloating, and cramps, but has neglected to seek medical advice until now. His GP is worried about undiagnosed Crohn’s disease and sends him for a small bowel series. This shows some dilatation of the proximal small bowel, with segmentation and fl occulation of the barium and an increased number of normal thickness folds seen in the ileum. There is no evidence of stricture formation or ulceration. What is the most likely underlying diagnosis? A. Amyloidosis. B. Chronic ischaemic enteritis. C. Whipple’s disease. D. Coeliac disease. E. Lymphoma.
D. Coeliac disease. The segmentation and fl occulation of barium are fi ndings on a small bowel series that are typical of malabsorption and therefore the most likely diagnosis is coeliac disease. Other fi ndings in coeliac disease include dilatation, a granular appearance to the barium secondary to hypersecretion, jejunization of the ileum, and the ‘moulage’ sign. The latter refers to a smooth, tubular appearance to the jejunum in longstanding coeliac disease, secondary to atrophy and effacement of the jejunal mucosal folds. Lymphoma can be a complication of coeliac disease and generally causes shallow, ulcerated masses or the development of thickened, nodular small bowel folds. Whipple’s disease, amyloidosis, and chronic ischaemic enteritis all cause thickening of the small bowel folds
534
A 20-year-old male with a recent history of medulloblastoma now presents with vague abdominal pain, PR bleeding, and weight loss. Innumerable colonic polyps are demonstrated on colonoscopy. What is the most likely unifying diagnosis? A. Familial adenomatous polyposis. B. Turcot syndrome. C. Gardner syndrome. D. Lynch syndrome. E. Chronic infl ammatory bowel disease.
B. Turcot syndrome. FAP is a rare autosomal dominant condition resulting in the growth of hundreds of adenomatous polyps in the large bowel. Clinical symptoms commence from the third decade and include abdominal pain and PR bleeding. Colorectal cancer develops in almost all before the age of 40 years. Turcot syndrome is characterized by the association of colonic polyps similar to FAP and central nervous system tumours, typically medulloblastoma and glioblastoma multiforme. The combination of intestinal polyposis (identical to FAP) and numerous osteomas and epidermal cysts is typical of Gardner syndrome. Lynch syndrome is hereditray non-polyposis colorectal cancer. Colorectal carcinoma occurs earlier than in the average population. There is also an association with ovarian and endometrial malignancy. Chronic infl ammatory bowel disease is not associated with CNS malignancy.
535
A 50-year–old male is admitted under the surgical team having presented with upper abdominal pain and raised infl ammatory markers. Suspecting acute cholecystitis, an ultrasound is requested, but due to large body habitus there is poor visualization of his gallbladder. To further evaluate hepatobiliary scintigraphy using 99mTc-labelled iminodiacetic acid is arranged. Which of the following fi ndings are consistent with acute cholecystitis? A. Non-visualization of the gallbladder at 1 and 4 hours. B. Non-visualization of the gallbladder at 1 hour but seen at 4 hours. C. Visualization of the gallbladder at 1 hour. D. Visualization of the gallbladder at 30 minutes after morphine administration. E. Hepatobiliary scintigraphy is not appropriate for investigation of acute cholecystitis.
A. Non-visualization of the gallbladder at 1 and 4 hours. Hepatobiliary scintigraphy is most commonly used to evaluate suspected acute cholecystitis. A minimum of 2 hours fasting is required. Following prompt uptake by the liver, the radiotracer is excreted into the biliary system and drains into the small bowel. Activity should be demonstrated within the gallbladder by 1 hour. Morphine can be used during the scan to relax the sphincter of Oddi, thus pushing radiolabelled bile into the gallbladder. Acute cholecystitis is characterized by non-visualization of the gallbladder at both 1 and 4 hours or at 30 minutes following morphine administration. Non-visualization of the gallbladder at 1 hour, but seen at 4 hours, is indicative of chronic cholecystitis. A false-positive diagnosis of acute cholecystitis can occur with previous cholecystectomy, gallbladder agenesis, and tumour obstructing the cystic duct.
536
A 42 year old fit and well male has a CXR for emigration purposes. There is a rounded opacity in the right mediastinum. Subsequent CT Chest reveals a 4.5 cm nodal mass with central calcification and marked enhancement following contrast administration. There are no focal lung lesions. Which of the following is the most likely diagnosis? A Castlemans disease B Colorectal cancer metastasis C LCH D NHL E Silicosis
A Castlemans disease Low grade B cell lymphoid proliferation of unknown aetiology. Localised or multi centric (multiple LN, typically with systemic symptoms). Enhance avidly, may have central calcification and are as large as 16 cm.
537
1) A 30 year old fit and well man presents with chronic sinus symptoms. On CT there is generalised opacification of the maxillary sinuses and ethmoid air cells. High density material is noted centrally in the maxillary sinuses. What is the most likely diagnosis? A- Mucocele B- Allergic fungal sinusitis C- GPA- granulomatous polyangiitis D- Invasive Fungal Sinusitis E- Antrochoanal polyp
B Allergic fungal sinusitis
538
A 40 year old female presents with an enlarging mass on her face. She is systemically well. CT reveals a completely opacified right maxillary sinus. The HU measure 10-20. The sinus is expanded with bony thinning. A post-contrast CT is performed which reveals peripheral enhancement only. What is the most likely diagnosis? A- Antrochoanal polyp B- Inverted papilloma​ C- Invasive Fungal Sinusitis D- Mucocele​ E- Paranasal sinus lymphoma
D Mucocele
539
You are eagerly attending your regional skull base MDT. The next case is a patient diagnosed with Enthesioneuroblastoma. In front of the whole surgical team the consultant asks what would you expect to see on imaging? A- Relatively well defined T2 isointense mass with avid homogenous post contrast enhancement and restricted diffusion involving the left maxillary sinus and ethmoid air cells​ B- Ill-defined T2 hypointense mass with heterogenous post contrast enhancement in the nasal cavity with irregular surrounding invasion and bone destruction​ C- T1/2 isointense mass extending from the ethmoid air cells into the anterior cranial fossa with an area of narrowing at the cribriform plate. Some well-defined t2 hyperintense areas are seen adjacent to it in the anterior cranial fossa​ D- T1/T2 hypointensity of the left maxillary sinus with irregular bony destruction and high STIR signal in the pterygopalatine fossa. ​ E- T2 hyperintense mass in the right maxillary sinus with alternating hypointense lines
C- T1/2 isointense mass extending from the ethmoid air cells into the anterior cranial fossa with an area of narrowing at the cribriform plate. Some well-defined t2 hyperintense areas are seen adjacent to it in the anterior cranial fossa​
540
Following your perfect answer in the skull base MDT word has spread around the department that you are a genius. In morning teaching a consultant shows you a case of Juvenile Nasal Angiofibroma. You subsequently describe this lesion perfectly as... * A- T2 hyperintense mass in the right maxillary sinus with peripheral enhancement. The mass passes through the sinus ostium which is widened with smooth bony scalloping. It passes into the nasopharynx. ​ B- T2 hyperintense enhancing mass in the right maxillary sinus with alternating hypointense lines. It passes posteriorly into the nasopharynx. There is a focal area of bony hyperostosis in the maxillary sinus antrum​ C- T2 hyperintense mass involving the nasopharyngeal and oropharyngeal mucosa with enhancement tracking between with pterygoid muscles into foramen ovale.​ D- T2 hyperintense avidly enhancing lesion adjacent to the sphenoid sinus involving the pteryogopalatine fossa. The sphenopalatine foramen is widened with adjacent bony remodelling.​ E-T1/2 isointense mass extending from the ethmoid air cells into the anterior cranial fossa with an area of narrowing at the cribriform plate. Some well-defined t2 hyperintense areas are seen adjacent to it in the anterior cranial fossa
D- T2 hyperintense avidly enhancing lesion adjacent to the sphenoid sinus involving the pteryogopalatine fossa. The sphenopalatine foramen is widened with adjacent bony remodelling.​
541
A 56-year-old female patient presents with shortness of breath. A chest radiograph is unremarkable. A high-resolution CT scan is per- formed which shows mosaic perfusion with no air trapping on expiratory scan. What is the most likely diagnosis? a. bronchiolitis obliterans b. cystic fibrosis c. hypersensitivity pneumonitis d. chronic thromboembolic disease e. asthma
d. Mosaic perfusion is caused by abnormalities of ventilation, or vascular obstruction. Expiratory scans help to distinguish causes by establishing whether there is air trapping. With no air trapping present, pulmonary emboli of any cause are most likely. Air trapping would suggest airway disease such as bronchiolitis obliterans, or other causes of small airway obstruction such as bronchiectasis or cystic fibrosis.
542
In acute respiratory distress syndrome what is the first change usually seen on the chest radiograph? a. confluent consolidation b. pleural effusions c. increased heart size with globular shape d. volume loss with atelectasis e. patchy ill-defined opacities
e. Acute respiratory distress syndrome (ARDS) commences with interstitial oedema, progressing to congestion and extensive alveolar, and interstitial oedema and haemorrhage. The chest radiograph is often normal for the first 24 hours, before patchy opacities appear in both lungs. These progress to massive airspace consolidation over the following 24 – 48 hours. True volume loss, atelectasis, cardiomegaly and effusions are not seen in ARDS.
543
A 28-year-old male with known Marfan’s syndrome presents with chest pain and shortness of breath. An echocardiogram is performed. What are the most likely findings? a. aortic regurgitation and dilatation b. pulmonary stenosis c. aortic stenosis and post-stenotic dilatation d. global myocardial wall thickening e. ventricular septal defect
a. Marfan’s syndrome is an autosomal dominant connective tissue disorder. It predominantly affects the musculoskeletal system but 60 –98% of patients have cardiovascular manifestations. There is myxomatous degeneration of the aortic wall, leading to dilatation of the aortic root and aortic regurgitation. There is an association with congenital heart disease, incomplete coarctation and atrial septal defects.
544
Into which structure does the thoracic duct normally drain? a. left brachiocephalic vein b. left internal jugular vein c. left subclavian vein d. superior vena cava e. junction of left subclavian and internal jugular veins
e. The thoracic duct starts at the cisterna chyli at the level of T12. It passes behind the right diaphragmatic crus and crosses right to left in the thorax behind the oesophagus. It terminates by draining into the junction between the left subclavian and internal jugular veins, usually as two or three branches.
545
A 28-year-old female presents with menorrhagia and dysmenor- rhoea. An ultrasound scan shows a large fibroid in the uterus measuring 7 cm. An MR scan confirms an intramural fibroid which enhances vividly. The patient undergoes uterine artery embo- lization. What is the commonest complication occurring in the first 12 months following this procedure? a. premature ovarian failure b. failure of therapy with need for re-embolization or hysterectomy c. hysterectomy for uterine infection or pain d. persistent non-offensive vaginal discharge e. post-embolization syndrome
e. Post-embolization syndrome occurs in up to 52% of patients and is characterized by a flu-like illness with malaise and fever. This is self- limiting and lasts for up to 10 days. Failure of therapy with the need for re-embolization or hysterectomy occurs in 10% within the first 12 months, but increases to 20 – 25% at 5 years. A persistent non-offensive discharge per vaginum, which is negative on bacterial culture, occurs in 7 – 14% of patients and appears to be more common with submucosal fibroids. Hysterectomy for uterine sepsis or intractable pain is required in 2.9% in the first year. Premature ovarian failure is seen in approximately 1– 2% of patients, but this increases with age, rising to 25% in the over-45 age group.
546
A 55 year old presents with chest pain. His blood tests show a mild troponin rise, but an ECG is normal. A chest radiograph shows bilateral, symmetrical, hilar adenopathy but no other abnormality. Which feature on cardiac MRI would make cardiac sarcoid a more likely diagnosis than ischaemia secondary to coronary artery disease? a. delayed hyperenhancement of lateral wall b. full-thickness, abnormal, high T2 signal in lateral wall c. reduced inferior wall motility d. partial-thickness, abnormal, high T2 signal with subendocardial sparing at base of septum e. segmental area of reduced enhancement in lateral wall on early phase study
d. Acute cardiac sarcoid is seen on MRI as high T2 signal in the myocardium, which may be associated with wall thickening secondary to oedema. Early enhancement may also be seen with sarcoid on post- contrast scans. Delayed hyperenhancement can occur in acute sarcoid, but is also seen in non-viable myocardium secondary to ischaemia. However, in non-ischaemic conditions, it is often seen only involving the central section of the wall with subendocardial sparing. Acute sarcoid may also cause nodular high-signal areas of high T2 signal. It usually involves the base of the septum and left ventricle, but rarely the papillary muscle or right ventricle. Distribution rarely conforms to vascular territories.
547
Which of the following best describes the radiographic changes seen in acute rejection of a lung transplant? a. mosaic perfusion and air trapping b. pleural effusion and septal thickening with no left ventricular dysfunction c. bilateral consolidation at the bases d. increased lung volumes e. globular heart and bat-wing perihilar consolidation
b. Acute rejection usually develops 7 – 10 days post-surgery. Most patients experience at least one episode during their life. Clinically, there are reduced arterial oxygen levels without infection, associated with fatigue and reduced exercise tolerance. Chest imaging findings include ground- glass opacity, heterogeneous perihilar opacification and new enlarging pleural effusion with septal thickening, with no signs of left ventricular failure. Pleural oedema, peribronchial cuffing and airspace shadowing may also occur.
548
A 76-year-old female presents with haemoptysis and cough. A chest radiograph shows a mass in the right upper lobe that contains a crescent of air. Which feature on CT would make a cavitating malignancy more likely than aspergilloma? a. thin cavity wall b. high-density central mass c. enhancing central mass d. calcification e. adjacent bronchiectasis
c. Aspergilloma appears as a solid mass in a thin-walled cavity, which lies in a dependent position and is mobile. A crescent-shaped airspace is typical. It may calcify and be associated with pleural thickening. In contrast, cavitating malignancy often has irregular thick-walled margins with a central mass that often enhances, and is fixed and non-mobile. Calcification may occur in relation to malignancy but is usually adjacent to, rather than in, the lesion and is usually seen with a scar carcinoma. Neither causes high-density lesions. Adjacent bronchiectasis is more often seen when there has been previous infection such as tuberculosis, producing a cavity in which an aspergilloma develops.
549
A 42-year-old male presents with stridor and persistent cough. He previously has had several nosebleeds. Bloods show mild renal impairment. A chest radiograph shows multiple cavitary lesions with irregular lining, predominantly in the lower lobes. What is the most likely diagnosis? a. metastatic disease from nasopharyngeal carcinoma b. Wegener’s granulomatosis c. sarcoidosis d. pyogenic abscesses e. systemic lupus erythematosus
b. The combination of nosebleeds and cavitatory nodules is very suggestive of Wegener’s granulomatosis, especially with renal impairment. In the lungs there is often patchy alveolar infiltration, pleural effusions and cavitatory nodules. Systemic lupus erythematosus produces similar appearances by the same mechanism of vasculitis. Sarcoidosis produces many lung changes but cavitatory nodules are rare. Metastatic nasopharyngeal carcinomas are usually squamous cell tumours and can produce cavitatory metastases, but primary tumours usually present with nasal obstruction, although they may cause epistaxis. Renal impairment is not a feature. Multiple abscesses are not usually associated with epistaxis and present acutely, typically in intravenous drug users.
550
A 50-year-old male with known chronic asthma is seen in an outpatient clinic. He has no current symptoms. A ‘routine’ chest radiograph is performed. Which feature is most likely to be seen? a. hyperexpansion b. peribronchial cuffing c. bronchiectasis d. parenchymal scars e. normal chest radiograph
e. In chronic asthma, the majority of patients (73%) have normal appearances on chest radiographs between acute episodes. Features such as bronchiectasis or parenchymal scarring may be seen, especially with episodes of repeated infection. Hyperexpansion and peribronchial cuffing are features seen during acute exacerbations of asthma.
551
A 65-year-old man presents with painful wrists and ankles. There is no digital clubbing. A chest radiograph shows a well-defined pleural mass, forming an obtuse angle with the chest wall. CT confirms an ovoid, pleurally based, enhancing mass with no bone destruction, effusion or volume loss. Radiographs of the wrists and ankles show symmetrical periosteal reaction. What is the most likely diagnosis? a. hypertrophic osteoarthropathy with bronchogenic carcinoma b. hypertrophic osteoarthropathy with malignant mesothelioma c. hypertrophic osteoarthropathy with pleural fibroma d. rheumatoid arthritis e. tuberculosis
c. The periosteal reaction is typical of hypertrophic osteoarthropathy (HOA), which has many causes, both intra- and extrathoracic. It is associated with bronchogenic carcinoma and malignant mesothelioma, but features of the described lesion are not typical of either of these, and are more in keeping with pleural fibroma. These are associated with HOA in 20 – 35% of cases and rarely with clubbing. They may arise in pleura adjacent to the chest wall or in a fissure, and can vary in size from 2 cm to 30 cm in diameter. Tuberculosis is a rare cause of HOA. Rheumatoid arthritis is associated with bilateral periosteal reactions, but not HOA.
552
A right-sided aortic arch with mirror-image branching is most frequently associated with which congenital cardiac abnormality? a. pulmonary atresia and ventricular septal defect b. truncus arteriosus c. uncomplicated ventricular septal defect d. Fallot’s tetralogy e. corrected transposition of the great vessels
d. There is a 98% incidence of associated congenital heart disease with a right-sided aortic arch with mirror-image branching. Nearly all of these cases will be tetralogy of Fallot. All of the given options are associated with right-sided aortic arch, as well as dextrocardia with situs inversus and double-outlet right ventricle. Right-sided aortic arch with left subclavian artery is associated with only a 12% incidence of congenital heart disease, again with Fallot’s tetralogy being the most commonly associated abnormality.
553
A 68-year-old man presents with increasing dyspnoea. He has a history of asbestos exposure. CT of the chest demonstrates bilateral pleural thickening with calcification, and a 3 cm, rounded, lower lobe mass. This is related to an area of pleural thickening, with bronchovascular markings coursing from it towards the hilum in a curved path. What is the most likely diagnosis? a. rounded atelectasis b. bronchogenic carcinoma c. tuberculosis d. silicosis e. round pneumonia
a. Rounded atelectasis is an infolding of pleura associated with atelectasis, which occurs in the posterior lower lobes and abuts an area of pleural thickening. Often a ‘comet’s tail’ of bronchovascular markings is seen curving towards the hilum. Bronchogenic carcinoma may occur with asbestos exposure, but is usually of bronchioalveolar cell type and has a latent period of 25 –35 years. Round pneumonia produces spherical consolidation with air bronchograms. Silicosis may cause conglomerate masses, usually in the mid or upper zones, or progressive massive fibrosis, but reticulonodular changes predominate.
554
A 43-year-old female with Churg – Strauss syndrome has a high- resolution CT. What is the most likely finding? a. small centrilobular nodules b. bullous disease c. bilateral, symmetrical, hilar lymphadenopathy d. tumour-like mass e. pleural effusion
a. Churg – Strauss syndrome is rare, though pulmonary abnormalities are a common feature of the condition. Typically airspace and airway patterns are seen with the following features – centrilobular nodules, ground- glass opacities, bronchial wall thickening, bronchiectasis, consolidation and septal thickening. Lymphadenopathy may be seen in the mediastinum and at the hila but is not a common feature. Large nodules may occur but a tumour-like mass is not a feature. Pleural effusions and bullae are not seen.
555
A 25-year-old male presents with recurrent epistaxis, which is progressively worsening. On examination he is noted to have multiple, red, vascular skin blemishes. A chest radiograph shows several opacities in the lung measuring up to 3 cm with bands of opacification extending to the hila. No calcification is seen. What is the most likely diagnosis? a. hereditary haemorrhagic telangiectasia b. neurofibromatosis c. tuberous sclerosis d. Wegener’s granulomatosis e. sarcoidosis
a. Hereditary haemorrhagic telangiectasia (HHT), or Rendu – Osler – Weber syndrome, is a disorder that produces telangiectasia, arteriovenous malformations (AVMs) and aneurysms, affecting multiple organ systems. Recurrent epistaxis is seen in up to 85% of cases. Up to 15% of patients with HHT have multiple pulmonary AVMs, and 60% of patients with pulmonary AVMs have HHT. Wegener’s granulomatosis is also associated with epistaxis, due to granulomas of the nasal septum, but pulmonary findings are of multiple granulomas.
556
A patient with known polyarteritis nodosa presents with acute left loin pain. Which of the following is most likely to be seen on ultrasound scan? a. hydronephrosis b. a solid mass with a perinephric collection c. multiple renal artery aneurysms and perinephric collection d. crossed fused ectopia e. small kidneys with increased echogenicity
c. Polyarteritis nodosa is a systemic necrotizing inflammation of medium- sized and small muscular arteries. No glomerulonephritis (small, echogenic kidneys) is present. The condition most commonly affects the kidneys (85%) and is usually seen as multiple small intrarenal aneurysms, which may disappear due to thrombosis. Recognized complications are perinephric or subcapsular haemorrhage due to aneurysm rupture. Crossed fused ectopia and hydronephrosis are not recognized features of polyarteritis nodosa.
557
A 45-year-old female presents with malaise and cough. She has a history of multiple allergies. Her blood results show an eosino- philia, and the chest radiograph reveals two areas of peripheral consolidation. A further chest radiograph 2 days later shows these to be resolving. Which of the following is the most likely diagnosis? a. histiocytosis b. pseudomonas pneumonia c. Klebsiella pneumonia d. Loeffler’s syndrome e. lipoid pneumonia
d. Loeffler’s syndrome is of unknown aetiology and is characterized by areas of non-segmental consolidation, which are peripherally situated and transient. Histiocytosis produces ill-defined nodules which cavitate, first producing thick-walled and then thin-walled cysts. Klebsiella pneumonia usually affects the upper lobes, producing dense, lobar consolidation, sometimes with bulging fissure and empyema. Pseudomonas pneumonia is patchy but extensive and usually in the lower lobes. Lipoid pneumonia is segmental and homogeneously dense, and changes only slowly.
558
A 26-year-old intravenous drug user presents with reduced conscious level, associated pyrexia and malaise. Clinically, there is a systolic murmur, mild hypotension and an elevated white cell count. A chest radiograph shows multiple opacities in the mid and lower zones, some of which are cavitating. What is the most appropriate next investigation? a. CT of the thorax b. transthoracic echocardiogram c. white cell scan d. MRI of the heart e. transoesophageal echocardiogram
e. In this clinical scenario the patient is most likely to have multiple septic emboli secondary to intravenous drug usage. Given the multiple pulmonary abscesses and pneumonia, tricuspid endocarditis should be considered and an echocardiogram should be performed. Transoesophageal echocardiogram is more sensitive to valvular vegetations and should be the investigation of choice. MRI of the heart may have some value in endocarditis, but as yet its value as a routine investigation has not been proven.
559
A 64-year-old man presents with pain in the left arm when exercising, associated with a headache. The clinical team suspect subclavian steal syndrome. Ultrasound scan, however, shows normal flow in the carotid and vertebral arteries bilaterally. What is most likely to happen to the flow in the vessels during patient exercise to reproduce the pain? a. reversal of flow in the right carotid artery b. reversal of flow in the left carotid artery c. reversal of flow in the right vertebral artery d. reversal of flow in the left vertebral artery e. no change
d. In subclavian steal syndrome (SSS), there is a stenosis in the subclavian artery proximal to the vertebral artery origin. This causes reversal of flow in the ipsilateral vertebral artery to maintain blood supply to the upper limb. If the stenosis is not severe, there is normal flow at rest, but exercise aggravates this by increasing the blood supply to the limb. As the stenosis is unable to accommodate the increased flow, the flow in the ipsilateral vertebral artery is reversed. This is termed ‘occult SSS’.
560
A 42-year-old female with tuberous sclerosis presents with flank pain. Ultrasound scan and CT demonstrate a 7 cm renal angiomyo- lipoma, with multiple similar smaller lesions in both kidneys. No evidence of acute haemorrhage is seen. She undergoes embolization with polyvinyl alcohol particles. After 48 hours, she presents again with flank pain and is found to have a large perinephric haematoma on the side of the embolization. What is the most likely cause? a. vascular trauma during embolization b. spontaneous haemorrhage from non-embolized lesion c. post-embolization rupture d. revascularization of embolized lesion e. inadequate embolization material used
c. Following embolization of an angiomyolipoma, there is propensity to rupture, which appears to be more common if particulate embolization material is used alone without coiling of the feeding vessels. This latter procedure is therefore recommended by some practitioners. Vascular trauma from the original procedure may have a delayed presentation, but this is rarely seen. Spontaneous rupture of another lesion would remain a possibility, but, given the recent embolization, the cause of the haematoma would be more likely to be related to this.
561
In ventilation – perfusion scintigraphy, which of the following is suggestive of an intermediate probability of pulmonary embolism? a. matched non-segmental defects with a normal chest radiograph b. multiple unmatched small perfusion defects with normal ventilation c. large, segmental, matched defect with similar-sized opacity on chest radiograph d. reverse mismatch e. two large, unmatched, segmental, perfusion defects
c. With ventilation – perfusion scintigraphy, matched segmental defects are considered low probability. When there is a similar-sized area of opacification on the chest radiograph, which indicates ‘triple match’, this becomes intermediate probability. Matched non-segmental defects, reverse mismatch and multiple small perfusion defects are all indicators of low probability. Two large segmental perfusion defects that are not matched are considered high probability.
562
A 78-year-old man presents with superior vena caval syndrome. A CT scan shows an irregular mass in the superior mediastinum causing near-total occlusion of the superior vena cava. There is no sign of respiratory compromise or raised intracranial pressure. Which of the following would be the most appropriate next step in the patient’s management? a. obtain tissue diagnosis b. chemotherapy c. radiotherapy d. stenting of superior vena cava e. surgical bypass
a. Wherever possible, definitive diagnosis should be obtained, as it will enable the best possible treatment. Dependent upon the cause of the obstruction, the correct therapeutic option can then be employed. Radiotherapy often gives a good response, but, in small-cell lung cancer, chemotherapy often gives good results. Stenting should be used when radiotherapy or chemotherapy has failed. Surgical therapy is usually reserved for benign causes where conservative options have failed.
563
A 57-year-old man presents with chest pain and fever after an episode of vomiting. A chest radiograph shows a small left pleural effusion and pneumomediastinum. Which investigation will best establish the diagnosis? a. CT of the chest b. barium swallow c. water-soluble contrast swallow d. MRI e. transoesophageal echocardiogram
c. Oesophageal rupture is the most likely diagnosis. CT may be able to elicit suspicious signs, such as mediastinal gas, oesophageal thickening and pleural effusion, but cannot make a definitive diagnosis. A contrast swallow is best in confirming the diagnosis, but barium should not be used due to its potential to cause a severe inflammatory reaction and worsening mediastinitis.
564
A 65-year-old male presents with a 2-month history of cough and dyspnoea and has had swelling of the face, neck and arms for 1 week. He has had tuberculosis in the past. CT shows an irregular right paratracheal mass with calcification that is compressing the superior vena cava and right main bronchus, with patchy consolidation in the right lung. What is the most likely diagnosis? a. small-cell carcinoma b. lymphoma c. malignant teratoma d. fibrosing mediastinitis e. bronchogenic cyst
d. Fibrosing mediastinitis is a rare condition which has two forms: focal (usually secondary to tuberculosis or histoplasmosis) or diffuse (often idiopathic). When associated with tuberculosis, it is thought to be secondary to rupture of lymph nodes in the neck or mediastinum. Calcification is seen in 63% of cases, but the mass may be difficult to differentiate from malignant conditions when calcification is not present. Symptoms and signs are due to compression/obstruction of mediastinal structures. Small-cell carcinoma and lymphoma can produce middle mediastinal masses, but calcification is rare, unless in treated lymphoma when progressive symptoms would be unlikely. Malignant teratomas are typically anterior mediastinal masses which have well-defined lobulated margins, and usually do not calcify.
565
A 37-year-old male presents to accident and emergency following smoke inhalation in a fire. He feels well and a chest radiograph is normal. The following day he re-presents feeling short of breath and unwell. What are the most likely findings on the chest radiograph now? a. pulmonary oedema b. pleural effusions c. upper-zone consolidation d. diffuse reticular change e. pneumothorax
a. Inhalation of noxious gases, including smoke, produces focal or diffuse pulmonary oedema. With smoke this may be delayed by 1 – 2 days. Bronchiolitis obliterans may then ensue after 1 – 3 weeks, especially with chemical inhalation.
566
A 40-year-old female with known history of seizures and low IQ presents with severe shortness of breath, progressively worsening over a number of years. A chest radiograph shows extensive honey- combing throughout both lungs, which are of normal volume. A small pneumothorax is also seen on the right side. What is the most likely diagnosis? a. sarcoidosis b. cystic fibrosis c. tuberous sclerosis d. lymphangiomyomatosis e. idiopathic pulmonary fibrosis
c. The lung is involved in 5% of patients with tuberous sclerosis, with symptoms usually occurring in adult life, and it is predominantly women who are affected. There is interstitial fibrosis in the lower lungs with miliary nodular changes, which progress to honeycomb lung. Recurrent pneumothoraces occur in 50% of cases. Chylothorax may also be seen. Preservation of the lung volumes with honeycombing is seen in histiocytosis, neurofibromatosis and lymphangiomyomatosis, as well as tuberous sclerosis.
567
A 35 year old with asthma presents with malaise, flu-like illness and cough. Previous similar episodes have occurred. A chest radiograph shows patchy airspace opacification in the mid and upper zones. Which feature on high-resolution CT would make allergic broncho- pulmonary aspergillosis a more likely diagnosis than extrinsic aller- gic alveolitis? a. widespread centrilobular micronodules ,3 mm b. tubular finger-like opacities c. bronchiectasis d. upper-zone fibrosis e. pleural effusion
b. Allergic bronchopulmonary aspergillosis (ABPA) is hypersensitivity to aspergillus in people with asthma. Typical features are of a migratory pneumonitis, predominantly in the upper lobes. It may cause bronchiectasis and upper-zone fibrosis, which are features also seen in extrinsic allergic alveolitis (EAA). Tubular opacities, indicating mucus plugging, are seen in ABPA, but not in EAA. Centrilobular nodules are seen in EAA, along with mosaic perfusion and patchy ground-glass change. Pleural effusions are rarely seen in EAA and not in ABPA.
568
A 38-year-old pregnant female presents with haemoptysis and shortness of breath. A chest radiograph shows a large effusion, which is drained and found to be chylous. What is the most likely diagnosis? a. histiocytosis b. tuberous sclerosis c. neurofibromatosis d. lymphangiomyomatosis e. idiopathic pulmonary fibrosis
d. Lymphangiomyomatosis is exclusively seen in women of child-bearing age and is worsened in pregnancy. Classically, it causes coarse reticular/ reticulonodular interstitial changes, and patients develop recurrent chylous effusions and pneumothoraces. Lung volumes are preserved or may be increased (the only interstitial lung disease with increased lung volumes). It may progress to honeycombing. Histiocytosis usually affects men, and causes cystic change in the upper lobes, small nodules and septal thickening. Tuberous sclerosis has similar features to histiocytosis but is associated with skin changes, reduced IQ and epilepsy. Neurofibromatosis tends to cause progressive fibrosis, but effusions and pneumothoraces are uncommon. Idiopathic pulmonary fibrosis tends to affect the older age group with a reticulonodular pattern.
569
A 42-year-old female patient presents with a swollen calf, and deep venous thrombosis is suspected clinically. The D-dimer is elevated. Doppler ultrasound scan shows no thrombus in the thigh or calf veins. Spectral Doppler shows continuous signal with no respiratory variation. Which further investigation may be of value? a. no further investigation – normal findings b. pelvic ultrasound c. CT pulmonary angiogram d. chest radiograph e. echocardiogram
b. Even with no clot seen, the loss of respiratory variation with continuous flow suggests a more proximal occlusion. As the limb swelling is unilateral, the most likely site of occlusion would be in the pelvic veins.
570
A 27-year-old female with known sickle cell disease has an out- patient appointment. She is feeling unwell and bloods show an anaemia. A chest radiograph shows a right-sided, lobulated, para- mediastinal mass in the lower thorax with widening of the rib spaces. CT shows no calcification or bone erosion. What is the most likely diagnosis? a. neurogenic tumour b. bronchogenic cyst c. Bochdalek’s hernia d. tuberculous abscess e. extramedullary haematopoiesis
e. Extramedullary haematopoiesis occurs in conditions where there is prolonged anaemia. This can occur in the spleen, liver, lymph nodes, adrenals and many other sites. In the mediastinum it produces a rounded/lobulated, paraspinal soft-tissue mass, usually between T8 and T12. Unlike other causes of paraspinal masses, there is usually no pain or bone erosion. Bronchogenic cysts are of fluid density. Bochdalek’s hernia can present in adults as a paravertebral mass, which is usually asymptomatic and contains fat as well as abdominal organs (bowel, kidney or spleen). It is usually left sided. Tuberculous abscesses are usually associated with bone destruction. Neurogenic tumours are rounded/ovoid and may extend through the intervertebral foramen into the spinal canal and/or produce bone erosion.
571
A 52-year-old male presents with dyspnoea and cough. A chest radiograph shows an ill-defined opacity in the right mid-zone, obscuring the heart border. A lateral view shows a thin wedge- shaped opacity with base in contact with the pleura anteroin- feriorly, and pointing posterosuperiorly. What is the most likely diagnosis? a. right middle lobe collapse b. right middle lobe consolidation c. right lower lobe collapse d. right lower lobe consolidation e. encysted pleural fluid
a. In right middle lobe collapse, the horizontal fissure and lower half of the oblique fissure converge. This creates a wedge-shaped opacity on the lateral chest radiograph. On the frontal chest radiograph, there is an ill- defined mid-zone opacity. With right middle lobe consolidation, there is a mid-zone opacity with a well-defined superior margin, as the horizontal fissure remains in a normal position and is tangential to the radiograph beam. Both obscure the right heart border. Lower lobe collapse and consolidation cause basal opacity with loss of clarity of the right hemidiaphragm. The lateral view shows a triangular opacity at the right base posteriorly, larger in consolidation than collapse.
572
A 42-year-old female patient presents with dyspnoea and pleuritic chest pain. She has previously had pulmonary emboli diagnosed. A CT pulmonary angiogram is performed. Which feature would indicate chronic rather than acute thrombus on the CT? a. complete occlusion of segmental vessel b. filling defects centrally with peripheral contrast enhancement c. peripheral mural filling defect forming acute angle with wall d. peripheral mural filling defect forming obtuse angle with wall e. linear atelectasis
d. The differentiation of acute from chronic thromboembolic disease can be difficult. Secondary changes may be present, such as hypertrophy of the right atrium and ventricle with cardiomegaly as well as pulmonary hypertension. Chronic emboli usually form peripheral flattened defects, forming obtuse angles with the arterial wall. Complete vessel occlusion may be seen in both acute and chronic emboli. The presence of recanalization or collateral formation is also suggestive of chronicity. In addition, calcification of the clot may occur, which also indicates chronicity. Parenchymal abnormalities such as atelectasis and wedge- shaped opacities may be seen in both acute and chronic pulmonary emboli.
573
A 50-year-old male presents with gradual onset dyspnoea and cough. There is no preceding history. A chest radiograph shows bilateral ‘bat-wing’ consolidation, with normal heart size and no effusion. High-resolution CT shows diffuse ground-glass change with intralobular and interlobular septal thickening (‘crazy- paving’ appearance). What is the most likely diagnosis? a. pulmonary oedema b. atypical pneumonia c. pulmonary alveolar proteinosis d. acute respiratory distress syndrome e. hypersensitivity pneumonitis
c. Pulmonary alveolar proteinosis is due to accumulation of proteinaceous material in the alveoli, which causes hypoxia. There is a variable combination of airspace and interstitial changes, but diffuse ground-glass opacification and ‘crazy-paving’ are typical. A geographic distribution is also described. Lack of other features (such as cardiomegaly, lymphadenopathy and effusion) helps to distinguish it from infection and oedema. The lack of precipitating event and slow onset makes acute respiratory distress syndrome unlikely. Hypersensitivity pneumonitis often shows no abnormality on chest radiograph, but high-resolution CT shows surprisingly marked ground-glass change and centrilobular nodules with peripheral sparing.
574
A 56-year-old male presents with wheezing, cough and recurrent chest infections. A chest radiograph shows right middle lobe conso- lidation. CT of the chest shows a 3 cm mass arising within the right middle lobe bronchus with distal collapse and consolidation. Which feature of the mass would make hamartoma more likely than carcinoid? a. central location b. presence of calcification c. cavitation d. presence of fat e. prominent enhancement
d. Hamartomas are usually seen in the periphery of the lungs (two-thirds) with 10% being endobronchial. Calcification is seen in 15%, often popcorn type. Cavitation is rare but fat is seen in 50%. Carcinoids are usually located centrally and are endobronchial. Calcification is seen in one-third and they rarely cavitate. They do not contain fat and show prominent enhancement following contrast, as they are vascular.
575
A 52-year-old female presents with cough. She is on dialysis, but, apart from abnormal urea and creatinine, her bloods are normal. A chest radiograph is abnormal and high-resolution CT is per- formed. This demonstrates fluffy, nodular, 5 – 10 mm opacities of airspace-type appearance with foci of calcification, in an upper lobe distribution with subpleural sparing. Calcification of chest wall vessels is noted. What is the most likely cause of the appearances? a. varicella b. chronic renal failure c. tuberculosis d. histoplasmosis e. talcosis
b. The appearances are suggestive of ‘metastatic’ pulmonary nodular calcification secondary to chronic renal failure. The upper lobe distribution is due to the relative alkalinity of the upper lobes, caused by the higher ventilation-to-perfusion ratio. It is often associated with calcification of chest wall vessels. Varicella produces multiple calcified nodules of 1 – 3 mm after the acute episode has resolved. Patients with tuberculosis or histoplasmosis, where calcified nodules are seen, have 2 – 5 mm, well-defined nodules, and most have calcified lymphadenopathy. Talcosis characteristically produces 1 mm, very-high density nodules.
576
A 75-year-old man presents with worsening shortness of breath. He was a mine worker. A chest radiograph shows multiple nodules in the upper zones with a large upper-zone mass on the left. CT con- firms multiple small nodules up to 5 mm with a sausage-shaped mass paralleling the mediastinum. What is the most likely diagnosis? a. coal worker’s pneumoconiosis with bronchogenic carcinoma b. coal worker’s pneumoconiosis with progressive massive fibrosis c. tuberculosis d. primary lung carcinoma with metastases e. chronic extrinsic allergic alveolitis
b. The history of mining (dust exposure) with small nodules in the upper zones is typical of pneumoconiosis. The sausage-shaped mass is characteristic of progressive massive fibrosis (PMF), although malignancy cannot be excluded. PMF is often seen to have reduced nodularity surrounding it, as it incorporates the surrounding nodules and migrates towards the hilum. Tuberculosis usually produces a generalized distribution of 2 – 3 mm nodules in its miliary form, often with lymphadenopathy. Metastases from primary lung cancer are not uniformly small (though they can be in thyroid cancer and melanoma). Chronic extrinsic allergic alveolitis produces fibrotic changes in the mid and lower zones.
577
A 25-year-old male presents with dyspnoea on exertion, cough and haemoptysis. He has a history of recurrent chest infections as a child. A chest radiograph shows a hyperlucent left lung. A pulmonary embolus is suspected and a V/Q scan is arranged. This shows reduced perfusion and ventilation of the left lung, with delayed washout on ventilation. What is the most likely diagnosis? a. acute pulmonary embolus b. Macleod’s syndrome c. congenital lobar emphysema d. Poland’s syndrome e. hypogenetic lung syndrome
b. Macleod’s (Swyer – James) syndrome is a result of acute bronchiolitis in childhood causing obliterative bronchiolitis. There is usually a history of recurrent infections. Chest radiographs show hyperlucent lung with a small or normal-sized hemithorax. High-resolution CT shows reduced vascularity and attenuation of lung with air trapping. Congenital lobar emphysema presents in the first 6 months of life with respiratory distress and a hyperlucent, overexpanded lobe (usually left upper) on the chest radiograph. Poland’s syndrome is an absence of part of the pectoralis major muscle, which causes apparent hyperlucent lung, though the lung is normal. Hypogenetic lung syndrome is usually asymptomatic and causes reduced volume of lung with reduced lucency.
578
Which of the following is the most typical description of a myxoma? a. left atrial mass, no atrial enlargement, pulmonary oedema b. right atrial mass, enlarged right atrium, clear lungs c. left atrial mass, enlarged left atrium, calcified lung nodules, pulmonary oedema d. right atrial mass, enlarged right atrium, pulmonary oedema e. left atrial mass, dilated superior vena cava, inferior vena cava and azygos vein
c. Myxomas are more common in the left atrium (75 – 80%) and present with obstruction of the mitral valve. They cause pulmonary hypertension and oedema, left atrial enlargement and ossified lung nodules. They may also cause systemic emboli. Right-sided myxomas cause tricuspid obstruction, right atrial enlargement, dilatation of the SVC, IVC and azygos veins, and reduced pulmonary vascularity. They may also cause pulmonary emboli.
579
A 58-year-old male presents with malaise and left chest discomfort. A chest radiograph shows a well-defined mass in the left paraverte- bral region. CT shows that this is fatty but has soft-tissue stranding within it. Some enhancement of soft-tissue elements is seen along with foci of calcification. What is the most likely diagnosis? a. lipoma b. liposarcoma c. hamartoma d. neurofibroma e. thymolipoma
b. Liposarcoma is an uncommon tumour in the thorax. It contains variable amounts of fat, with soft-tissue components that may enhance following intravenous contrast. Calcification may occur within these lesions. Lipomas are more common and occur anywhere within the mediastinum. Hamartomas usually present as soft-tissue nodules within the peripheral lung and classically show popcorn calcification. They may contain fat. Neurofibromas present as paravertebral masses, often extending into the intervertebral canal, and may have fatty attenuation due to the presence of myelin. Thymolipomas occur in the anterior mediastinum and are found in young adults.
580
In patients with rheumatoid arthritis, what is the commonest pul- monary finding seen on the chest radiograph? a. pleural effusion b. fibrosis c. pulmonary nodules d. bronchiectasis e. heart failure
a. Between 2% and 54% of patients with rheumatoid arthritis have pulmonary abnormalities. Pleural abnormalities are most frequent, being either an effusion (unilateral in 92% of cases) or pleural thickening (usually bilateral). Fibrosis occurs in 30% of patients with pulmonary involvement. Nodules are unusual and seen in advanced disease. They are usually peripheral and may cavitate. Bronchial abnormalities are seen in 30% of patients with rheumatoid lung, and include bronchiectasis and bronchiolitis obliterans. Other findings include pulmonary arterial hypertension and heart failure secondary to carditis/pericarditis.
581
A 52-year-old male presents with mild dyspnoea. A chest radiograph shows a raised left hemidiaphragm which demonstrates paradoxical movement on fluoroscopy. Which of the following would be the most likely cause? a. Pancoast’s tumour b. left lower lobe tumour c. mediastinal small cell carcinoma d. eventration e. cerebrovascular accident
c. Diaphragmatic elevation with paradoxical motion is usually due to phrenic nerve paralysis. Mediastinal tumours are one of the commonest causes. Pancoast’s tumours invade the brachial plexus, though they can uncommonly invade the phrenic nerve. Eventration tends not to show paradoxical motion, though it can if large. Strokes may cause diaphragmatic elevation, but due to bilateral innervation of the diaphragm, this is not usually due to phrenic nerve involvement and has other causes, such as weakness of chest wall muscles.
582
A 74-year-old male presents with low back pain. MRI shows some degenerative changes but no disc protrusion or neural compromise. A 6 cm, abdominal aortic aneurysm is seen, which has an irregular wall, with patchy high signal within mural thrombus and in the wall on T1W images. No perianeurysmal fluid to suggest leak is seen. What advice should be given regarding the aneurysm? a. follow-up with ultrasound scan b. follow-up with CT c. follow-up with MRI d. routine referral to vascular surgeon e. emergency assessment by vascular surgeon
e. The patchy high T1 signal in the mural thrombus and wall is suggestive of haemorrhage and the aneurysm is therefore unstable. Impending rupture should be considered and urgent surgical assessment should be sought.
583
In MRI of the heart in the assessment of hypertrophic cardiomyo- pathy, muscle mass is best assessed by using a steady-state free- precession sequence in which plane? a. left ventricular vertical long axis b. left ventricular horizontal long axis c. left ventricular short axis d. left ventricular short axis oblique e. four-chamber plane
d. In the assessment of hypertrophic cardiomyopathy, MRI is better than echocardiography due to the latter being limited by poor visualization. MRI can assess the distribution of disease and wall thickness, especially the anterolateral wall. This has important implications in assessing areas of myocardial thickness of .30 mm, which is of prognostic value. Wall thickness is generally best assessed on short axis images, except at the apex when long axis planes are better, and should be done at end- diastole. The technique of assessing left ventricular volume, function and mass, using the short axis oblique plane, is well described in the literature. Inclusion of papillary muscles varies between institu
584
A 33-year-old female with renal failure has an indwelling right internal jugular venous catheter. She presents with swelling of the right arm. Ultrasound Doppler scan of the neck and arm veins is performed. Which feature would suggest occlusion of the right brachiocephalic vein? a. collapse of right internal jugular vein on sniffing b. variation of flow with respiration in right subclavian vein c. variation with cardiac cycle in right subclavian vein d. continuous monophasic flow in the right subclavian vein e. collapse of the left internal jugular vein on sniffing
d. Ultrasound evaluation of the central veins is difficult, as the brachiocephalic veins and superior vena cava cannot be directly visualized. Secondary features to confirm patency can be seen, such as collapse of the internal jugular veins on sniffing (Valsalva manoeuvre), and variability of flow with respiration and the cardiac cycle. A continuous monophasic flow with loss of variability suggests a proximal occlusion or stenosis.
585
A 20 year old presents with shortness of breath and cough. A chest radiograph shows a well-defined right hilar mass. CT shows a 4 cm, rounded, soft-tissue mass arising from the mediastinum adjacent to the right side of the carina. The attenuation value of the lesion is 10 HU. No other abnormal findings are seen. What is the most likely diagnosis? a. bronchogenic cyst b. pericardial cyst c. carcinoid tumour d. lymphoma e. bronchogenic carcinoma
a. Bronchogenic cyst is an abnormality of the ventral diverticulum of the primitive foregut and is the most common foregut abnormality in the thorax. Typical appearances are of a thin-walled cyst containing mucus or fluid. CT shows a well-defined mass of water density in 50% (0 – 10 HU) or slightly higher density in the rest (10 – 50 HU); 86% are mediastinal and 50% pericarinal. Pericardial cysts largely occur in the cardiophrenic angle and are rarely mediastinal. Like bronchogenic cysts, these may change shape with position and respiration. Carcinoids occur in the lungs within bronchi and are mainly central. These are solid and may calcify. Bronchogenic carcinomas are usually solid, and centrally located in 38% of cases, where they are usually small cell tumours. These occur in the older population. Lymphadenopathy secondary to lymphoma is usually solid (unless treated) and involves multiple nodes in 95% of cases. The anterior mediastinum and retrosternal lymph nodes are usually affected first.
586
A 38-year-old male presents with a cough, not responding to antibiotics. A chest radiograph shows reticular change in both lungs with normal volumes. High-resolution CT confirms multiple thin-walled cysts with centrilobular nodules of 3 – 10 mm through- out the lungs. The intervening lung is normal. No pleural effusion is present. What is the most likely diagnosis? a. neurofibromatosis b. tuberous sclerosis c. lymphangiomyomatosis d. histocytosis e. usual interstitial pneumonitis
d. Honeycombing is seen with all of these conditions, but the combination of normal volumes, cysts and peribronchial nodules is very suggestive of histiocytosis. Often the lung bases are preserved early in the disease but the whole lung is eventually involved. Lung volumes in usual interstitial pneumonitis are reduced. Neurofibromatosis causes interstitial fibrosis with little nodularity. Tuberous sclerosis causes lower-zone fibrotic change with miliary nodules that progresses to honeycombing. Lymphangiomyomatosis produces coarse reticular change due to cyst formation, but nodularity is not seen. Pneumothoraces and pleural effusions are seen with neurofibromatosis, tuberous sclerosis and lymphangiomyomatosis.
587
A 42-year-old male suffers a chest injury in a road traffic accident. The presenting chest radiograph shows fractures of the fifth and sixth ribs on the right side with patchy airspace changes. He is admitted and has supportive care. A repeat chest radiograph shows the consolidation to have largely resolved, but a rounded opacity is now present with an air – fluid level. He is otherwise well. What is the most likely diagnosis? a. abscess b. bronchopleural fistula c. bronchogenic cyst d. pulmonary infarct e. pulmonary laceration
e. Pulmonary lacerations occur following trauma disrupting the lung parenchyma. The typical appearance is of a rounded cavity containing blood and/or air. On plain films, consolidation due to contusion often obscures the laceration. The laceration appears as a rounded/ovoid opacity with a pseudocapsule of compressed lung (2 – 3 mm), and may be fully opacified, be filled with air or have an air – fluid level. Complications are uncommon and include abscess (causes fever), bronchopleural fistula (causes pneumothorax) and progression. Bronchogenic cysts could present as an incidental finding after trauma but are usually mediastinal in location. The less common intrapulmonary bronchogenic cysts may cavitate. Pulmonary infarcts may present post-trauma due to immobilization and other risk factors causing pulmonary embolus, but these do not usually cavitate.
588
A 52-year-old man presents 1 year post-heart transplantation and has a routine follow-up chest radiograph. This shows multiple nodules of varying sizes, with enlarged hilar lymph nodes. What is the most likely diagnosis? a. graft-versus-host disease b. aspergillosis c. cytomegalovirus infection d. post-transplantation lymphoproliferative disorder e. Epstein – Barr virus infection
d. Post-transplant lymphoproliferative disorder (PTLD) occurs after bone marrow or solid organ transplantation, usually within 2 years. The type of tumour varies. This can produce single or multiple lung nodules with or without hilar and mediastinal lymphadenopathy. The nodules may be diffuse, subpleural or peribronchial and may have a surrounding halo of ground-glass opacification. The findings in graft-versus-host disease are of bronchiolitis obliterans – hyperinflation, bronchial dilatation and wall thickening, reduced vascularity/mosaic perfusion, and air trapping. In the acute phase, it often presents as non-cardiogenic pulmonary oedema. Aspergillosis presents with nodular opacities or consolidation, which may have a halo of ground-glass opacification, but occurs in the first 30 days. Cytomegalovirus infection usually occurs within the first 6 months after bone marrow transplantation, with a variety of appearances. Epstein – Barr virus is a causative factor in PTLD but is not in itself a cause of the pulmonary changes
589
In normal anatomy, which vascular structure lies most anteriorly at the level of the thoracic inlet, posterior to the manubrium? a. left common carotid artery b. brachiocephalic artery c. superior vena cava d. left brachiocephalic vein e. right brachiocephalic vein
d. In the superior mediastinum the venous structures lie most anteriorly. The superior vena cava does not extend up to reach the thoracic inlet but is formed inferiorly by the convergence of the brachiocephalic veins. The right has a short vertical course to the right of the midline, while the left crosses from the root of the neck on the left to the right side of the superior mediastinum behind the manubrium, where it lies anterior to all of the other vascular structures.
590
Which of the following descriptions would be most suggestive of a pulmonary hamartoma on imaging? a. round, 2 cm, soft-tissue mass with no calcification or fat, in a central location b. irregular, 8 cm mass with cavitation and associated effusion c. multiple lesions of 1 – 3 cm with calcification, throughout lungs d. lobulated, 3 cm mass with calcification and fat, in a peripheral location e. peripheral, 5 cm lesion with no calcification, and band-like opacity connecting it to the hilum
d. Hamartomas are seen in 0.25% of the population and are the commonest benign lung tumour. Two-thirds are found peripherally. They are rarely multiple or cavitatory. Fifteen per cent calcify (classically popcorn) and 50% contain fat. Option (a) is more typical of carcinoid, while (c) is suggestive of multiple granulomas, probably secondary to chickenpox. Option (e) is characteristic of pulmonary arteriovenous malformation.
591
In persistent left-sided superior vena cava, drainage usually occurs into which structure? a. left atrium b. right atrium c. normal right superior vena cava d. hemiazygos vein e. coronary sinus
e. Persistent left-sided superior vena cava occurs in 0.3% of the general population and in 4.3 – 11% of patients with congenital heart disease. It is associated with atrial septal defects and azygos continuation of the inferior vena cava. It lies lateral to the aortic arch and anterior to the left hilum. It usually drains into the coronary sinus, but rarely drains into the left atrium, causing a left-to-right shunt. The normal right-sided superior vena cava is absent in 10 –18% of cases of left-sided superior vena cava.
592
An 18-year-old male presents with a chest radiograph performed for immigration purposes. He is noted to have dextrocardia. Further investigation reveals nasal polyposis and bronchiectasis. Which further investigation should be considered? a. renal function b. CT of brain c. fertility assessment d. renal angiogram e. thyroid function test
c. The presence of dextrocardia, nasal polyposis and bronchiectasis raises the possibility of Kartagener’s syndrome. This is immotile/dysmotile cilia syndrome and may also present with deafness and infertility. There is also an association with transposition of the great vessels, pyloric stenosis, post-cricoid web and epispadias. Situs inversus is seen in 50% of cases.
593
A CT scan performed on a patient shows a soft-tissue mass in the medial aspect of the left lung, invading the mediastinum between the aortic arch and pulmonary artery. Neither vessel is compro- mised. Which symptom may the patient have presented with? a. stridor b. dysphagia c. pain d. swelling of face and neck e. hoarse voice
e. The space between the pulmonary artery and aortic arch is the aortopulmonary window, which contains the ligamentum arteriosum and the left recurrent laryngeal nerve. Invasion here by tumours can lead to paralysis of the left vocal fold, which attains a fixed adducted position, by involvement of the recurrent laryngeal nerve. Stridor and dysphagia could result from deeper invasion into the mediastinum, as the trachea and oesophagus form the medial border of the aortopulmonary window. Swelling of the face, neck and upper limbs occurs with superior vena cava obstruction, which is a feature of right- sided mediastinal disease.
594
A 78-year-old patient who had an endovascular aortic aneurysm repair undergoes a routine 6-month follow-up scan. This shows that the aneurysm sac has increased in size compared with the 1-month follow-up scan. On the delayed-phase part of the scan, there is contrast seen in the periphery of the aneurysm sac, not in contact with the stent. What is the most likely diagnosis? a. type I endoleak b. type II endoleak c. type III endoleak d. type IV endoleak e. type V endoleak
b. Endovascular aortic aneurysm repair (EVAR) procedures require lifelong follow-up imaging – 6-monthly CT scans are recommended. Complications include expansion and rupture of the sac, or endoleak. The latter has five described types: type I – leak from stent – graft attachment, which can be subdivided into subtypes a and b, corresponding to leaks at proximal and distal attachments respectively; type II (commonest) – retrograde flow through aortic branches, usually inferior mesenteric artery or lumbar arteries; type III – structural failure of stent –graft; type IV – due to porosity of the graft; type V – endotension, which is expansion of the sac without obvious cause, although it may be due to an occult type I, II or III.
595
In thoracic lymphoma, which feature would favour non-Hodgkin’s lymphoma over Hodgkin’s disease? a. predominantly anterior mediastinal lymph nodal involvement b. predominantly middle mediastinal lymph nodal involvement c. predominantly posterior mediastinal lymph nodal involvement d. nodal calcification e. mass larger than 5 cm
c. Mediastinal nodal involvement is generally more suggestive of Hodgkin’s disease, but disease is usually seen in the middle and anterior mediastinum. Posterior mediastinal involvement, with little or no anterior or middle mediastinal involvement, suggests non-Hodgkin’s lymphoma as a more likely diagnosis. Calcification can occur in either condition, nearly always post-therapy. The size of the lymph node masses is not discriminatory.
596
In normal anatomy, which structure lies immediately anterior to the left main bronchus at the left hilum? a. left pulmonary artery b. left inferior pulmonary vein c. left superior pulmonary vein d. left phrenic nerve e. left vagus nerve
c. The left pulmonary artery crosses over the superior aspect of the left main bronchus giving off the upper lobe artery and the inferior pulmonary artery, and then lies posterior to the left main bronchus. The left inferior pulmonary vein drains into the left atrium and does not reach the level of the left main bronchus. The vagus nerve lies posterior to the hilum adjacent to the oesophagus. The phrenic nerve lies anterior to all of the left hilar structures on the pericardium.
597
A 62-year-old male presents with increasing shortness of breath. Clinically, he has oedematous ankles, raised central venous pressure, ascites and hepatomegaly. Blood tests show mildly raised inflammatory markers. Which feature on CT would make restrictive cardiomyopathy a more likely diagnosis than constrictive pericarditis? a. dilated inferior vena cava b. pleural effusions c. normal pericardial thickness d. pericardial calcification e. previous coronary artery surgery
c. Both restrictive cardiomyopathy (RCM) and constrictive pericarditis (CP) present in the same way, with signs and symptoms of reduced heart filling and venous congestion. Distinguishing between these causes is important, as CP can be cured. Pericardial calcification occurs in CP and is seen in 50% of cases on chest radiograph. Pericardial thickening of .4 mm is usually seen with CP. A normal pericardial thickness excludes CP and makes RCM a likely diagnosis instead. Previous cardiac surgery is a cause of CP.
598
In an adult patient, which structure, along with the right atrium and superior vena cava, forms the right mediastinal border? a. right brachiocephalic vein b. inferior vena cava c. right ventricle d. trachea e. brachiocephalic artery
a. In an adult, the right mediastinal border normally comprises the right brachiocephalic vein, the superior vena cava and the right atrium. In young patients the thymus may produce a characteristic sail-shaped opacity over the right mediastinal border. The right tracheal wall can be seen as the paratracheal stripe through the right brachiocephalic vein and superior vena cava. The right ventricle does not form any part of the cardiac silhouette on a frontal chest radiograph. The brachiocephalic artery lies medial to the right brachiocephalic vein and does not form any part of the mediastinal border.
599
In bronchopulmonary sequestration, which of the following features would be more suggestive of intralobar than extralobar type? a. enclosed in visceral pleura b. no connection to bronchial tree c. systemic venous drainage d. presentation in infancy e. systemic arterial supply
a. Bronchopulmonary sequestration is a malformation consisting of a non- functioning lung segment with no communication to the bronchial tree and a systemic arterial supply. The intralobar type accounts for 75% of cases. It is enclosed in visceral pleura and presents in adulthood with pain, repeated infection, cough and haemoptysis. The extralobar type is enclosed in its own pleura and presents in infancy with feeding difficulties, respiratory distress, cyanosis and congestive heart failure (due to shunting). Systemic venous drainage is seen in 80% of cases of extralobar type, but only in 5% of cases of intralobar type
600
A 60-year-old female presents with increasing shortness of breath. She is known to have rheumatoid arthritis. A chest radiograph shows reticulonodular changes, and high-resolution CT is per- formed. Which feature would suggest underlying sarcoidosis as a more likely diagnosis than rheumatoid lung? a. lower-zone predominance of reticulation b. mid-zone predominance of reticulation c. pleural effusion d. multiple nodules larger than 20 mm e. cardiomegaly
b. Sarcoidosis is seen more commonly in females and affects the thorax in 90% of cases. It usually produces adenopathy with or without parenchymal disease. There is a mid-zone predominance with irregular septal thickening, perilymphatic nodules, ground-glass opacification, traction bronchiectasis and honeycombing. Rheumatoid lung is more common in males and mostly presents with pleural abnormalities, usually an effusion. It may present with an interstitial fibrotic picture with a lower-zone predominance. Multiple large nodules of up to 7 cm in diameter are seen with rheumatoid lung. Cardiomegaly may be seen with both sarcoidosis and rheumatoid, as a result of congestive cardiac failure.
601
A 23-year-old female who is 23 weeks’ pregnant presents with pleuritic chest pain, and pulmonary embolus is suspected. She asks about the relative radiation doses for CT pulmonary angiogram and ventilation – perfusion (V/Q) scintigraphy. What is the dose for a CT pulmonary angiogram relative to a V/Q scan? a. CT pulmonary angiogram has a higher total body dose but a lower uterine dose b. CT pulmonary angiogram has the same total body dose but a lower uterine dose c. CT pulmonary angiogram has a higher total body dose and uterine dose d. CT pulmonary angiogram has a lower total body dose and uterine dose e. CT pulmonary angiogram has a higher total body dose but the same uterine dose
a. The total body dose for CT pulmonary angiogram is approximately 2 – 3times higher than for V/Q scanning. However the uterine, and therefore fetal, dose has been found to be higher with V/Q scanning. A low-dose V/Q technique can reduce this, but the uterine dose still remains higher. Regardless of technique used, there remains a risk, but the risk of death from pulmonary embolus far outweighs any radiation risk to patient or fetus.
602
A 56-year-old female has arthralgia. A chest radiograph shows erosion of the lateral ends of the clavicles, superior notching of the third to fifth ribs, and narrowing of the humeral – acromial space. What is the most likely diagnosis? a. hyperparathyroidism b. rheumatoid arthritis c. osteoarthritis d. cleidocranial dysostosis e. neurofibromatosis
b. Erosion of the lateral ends of the clavicle is seen in several conditions, including rheumatoid arthritis, hyperparathyroidism and cleidocranial dysostosis. The associated rib notching and loss of the space between the humeral head and acromion (due to wasting/rupture of supraspinatus) are typically seen with rheumatoid arthitis. Clavicle erosions are not features of neurofibromatosis or osteoarthritis.
603
A 35-year-old female who smokes presents with hypertension and renal impairment. Ultrasound scan shows normal appearance of the kidneys. Doppler of the renal arteries demonstrates a peak vel- ocity of 180 cm/s and no diastolic flow. Angiography shows mul- tiple stenoses of the renal arteries bilaterally with a normal aorta. Which therapeutic option should be offered? a. angiotensin-converting enzyme inhibitors b. other antihypertensives c. angioplasty d. surgical correction e. no definitive therapy is of value
c. The appearances are suggestive of fibromuscular hyperplasia, which is the commonest cause of renovascular hypertension in young patients and causes 35% of all renal artery stenosis. It affects the aortic branches, but not the aorta itself. Complications include giant aneurysm and arteriovenous fistula. Angioplasty has a 90% success rate with a low re- stenosis rate and is the treatment of choice. Surgical correction can be performed (end-to-end anastomosis, vein grafts) but is usually reserved for refractory cases or where there is involvement of the segmental vessels rather than the main renal arteries.
604
On high-resolution CT of the chest, how can mosaic perfusion be distinguished from ground-glass opacification? a. the opaque areas have abnormally small vessels b. the lucent areas have abnormally small vessels c. mosaic perfusion is restricted to lower zones d. mosaic perfusion is restricted to subpleural regions e. cannot be reliably distinguished
b. Mosaic perfusion refers to areas of altered attenuation on CT and reflects vascular obstruction or abnormal ventilation. Differentiation from ground-glass opacification can be difficult but can usually be done by assessment of the centrilobular vessels. In comparing ‘lucent’ and ‘opaque’ areas, the area in which the vessels appear larger is generally the normal area, though this may be a difficult differentiation to make in practice, as the vessels may be of the same size throughout the lung. In mosaic perfusion, the vessels appear abnormal in the ‘lucent’ area, whereas, in ground-glass opacification, the vessels appear abnormal in the ‘opaque’ area. Expiratory scans may then help to distinguish airway causes from vascular causes, by the presence of air trapping, which is seen with airway disease.
605
A 25-year-old male presents with haemoptysis. A chest radiograph shows symmetrical, bilateral, perihilar consolidation, extending to the bases but with sparing of the apices. There is mild hilar enlarge- ment. The patient is also found to be in renal failure. Appearances on the chest radiograph subsequently progress to an interstitial pattern. What is the most likely diagnosis? a. primary pulmonary haemosiderosis b. secondary pulmonary haemosiderosis c. Goodpasture’s syndrome d. hereditary haemorrhagic telangiectasia e. histoplasmosis
c. Goodpasture’s syndrome is an autoimmune disease characterized by glomerulonephritis and pulmonary haemorrhage. Respiratory features are usually preceded by respiratory infection. Mild haemoptysis occurs associated with cough and dyspnoea. Bilateral consolidative changes and sometimes enlarged hilar nodes may occur. Pulmonary haemosiderosis presents in a similar manner, but usually occurs in patients under 10 years of age in the primary form, with adults affected in the secondary form, which is rare. Patchy airspace change occurs in the first 2 days and then resolves. Progression to fibrosis may occur. Patients with hereditary haemorrhagic telangiectasia usually have multiple arteriovenous malformations.
606
A 34-year-old woman presents with pain and swelling of the right knee over the previous 2 months. Plain films demonstrate a well- circumscribed, expansile, lytic lesion eccentrically located in the subarticular region of the right distal femur. The lesion has a narrow, non-sclerotic zone of transition. What is the most likely diagnosis? a. giant cell tumour b. enchondroma c. fibrous cortical defect d. fibrous dysplasia e. aneurysmal bone cyst
a. The vast majority of giant cell tumours occur in patients with closed epiphyses, and although they may originate in the metaphysis, lesions typically involve the epiphysis and abut the subarticular surface. They are classically eccentrically located lesions with a narrow zone of transition, no sclerosis, and no internal matrix mineralization. Giant cell tumours tend to be locally aggressive, with a high recurrence rate after initial treatment. Enchondromas are the commonest benign cystic lesion of the phalanges, though they are also seen in the long bones. However, those in the long bones almost always contain calcified chondroid matrix. Aneurysmal bone cysts are often seen as an eccentric lytic expansile lesion, but patients are nearly all under the age of 30. Monostotic fibrous dysplasia is more commonly seen in the proximal femur than distally, and lesions tend to have a sclerotic margin. Fibrous cortical defects are asymptomatic lesions seen in children, which usually regress spontaneously, so they are only rarely seen after the age of 30. They typically appear as lytic lesions with a thin sclerotic border in the metaphysis of a long bone
607
A 30-year-old women presents to her general practitioner with fatigue and painful stiff knees. She is subsequently found to be anaemic. Plain films show an Erlenmeyer flask deformity of the distal femora with cortical thinning. There are no erosions. What is the most likely underlying condition? a. mucopolysaccharidosis b. rheumatoid arthritis c. Gaucher’s disease d. Langerhans’ cell histiocytosis e. thalassaemia major
c. Gaucher’s disease is the most common lysosomal storage disorder with an incidence of 1:50 000 (100 times more common in Ashkenazi Jews). It is caused by a defect of hydrolase acid b -glycosidase, which results in accumulation of the fatty substance glucosylceremide within macrophages in the reticuloendothelial system. It characteristically causes an Erlenmeyer flask deformity of the distal femur or proximal tibia due to marrow infiltration. Patients may be asymptomatic or present with anaemia, large joint stiffness or bone pain. Diagnosis is by bone marrow aspirate. The mucopolysaccharidoses are a spectrum of lysosomal storage diseases that typically present in infancy with a variety of overt symptoms and signs. Rheumatoid arthritis can present with anaemia and joint stiffness, but marrow infiltration is not a feature on plain film. Musculoskeletal manifestations of Langerhans’ cell histiocytosis most commonly affect the skull (50%). Although Erlenmeyer flask deformity is seen in thalassaemia major, presentation is within the first 2 years of life.
608
Plain knee radiographs performed in accident and emergency following a sports injury in a 20-year-old footballer show an effusion, a small avulsion fracture immediately proximal to the fibular head, deepening of the lateral femoral sulcus and anterior translocation of the tibia. What is the most likely underlying ligamentous injury? a. complete posterior cruciate ligament rupture b. complete anterior cruciate ligament rupture c. partial anterior cruciate ligament rupture d. tibial collateral ligament rupture e. fibular collateral ligament rupture
b. The avulsion fracture described is a Segond fracture, which is classically associated with anterior cruciate ligament (ACL) rupture, and represents avulsion of the meniscotibial portion of the middle third of the lateral capsular ligament. Anterior translocation of the tibia occurs in complete ACL rupture, and manifests clinically as the anterior draw sign. Also associated with ACL rupture is an impaction injury of the lateral femoral condyle, which can be seen on radiographs as a deepened lateral femoral condylar sulcus, although sometimes this cannot be identified on acute films.
609
CT of the cervical spine is performed on an intubated emergency patient who was a restrained driver in a high-speed motor vehicle collision. This reveals bilateral C2 pedicle fractures. What is the most likely underlying mechanism of injury? a. hyperflexion and rotation b. hyperextension followed by hyperflexion c. axial loading d. hyperextension and traction e. hyper-rotation
d. The fracture described is a hangman fracture. This involves either the pedicles or pars interarticularis of C2 bilaterally. The mechanism is usually extension and traction (as caused during hanging). Hyperflexion injuries produce anterior tear-drop or of a vertebral body wedge fractures. Axial loading can produce a burst fracture of C1 (Jefferson’s fracture) or a vertebral body elsewhere in the spine. Hyperflexion and extension are associated with longitudinal ligament injury. Hyper- rotation is associated with soft-tissue injury or facet joint dislocation.
610
A young patient suffers a fractured femur and acetabulum in a road traffic collision and undergoes intramedullary nailing and plate- and-screw internal fixation of the acetabulum. He is well until 8 days postoperatively, when he develops acute shortness of breath and right-sided chest pain. A chest radiograph shows only a small right-sided pleural effusion. What is the most likely diagnosis? a. fat embolism b. bronchial pneumonia c. pulmonary embolism d. pneumothorax e. hyperventilation due to pain
c. Pulmonary embolism is a common complication following immobility and major surgery, particularly orthopaedic surgery of the pelvis. It typically occurs 7 – 10 days post-surgery. Chest radiograph findings can be normal but include small effusion, collapse or consolidation, elevation of the hemidiaphragm, a prominent pulmonary artery and hypertransradiance of the affected side (Westermark sign). Fat embolism is preceded by long bone injury in 90% of cases but usually occurs within 36 hours of the injury, and is much less common than pulmonary embolus from deep vein thrombosis even in the context of major trauma. Pneumonia and pneumothorax do of course occur in postoperative patients, but it would be reasonable to expect associated findings on the chest film. Hyperventilation should be a diagnosis of exclusion once other potentially serious causes have been excluded.
611
Of the lateral fibrous structures contributing to the stability of the posterolateral corner of the knee, which is the most likely to be congenitally absent and not identified on MRI, being present in only approximately two-thirds of patients? a. lateral collateral ligament b. popliteus tendon c. popliteofibular ligament d. arcuate ligament e. fabellofibular ligament
d. The structures of the posterolateral corner of the knee have a very important role in maintaining the rotational stability of the knee joint. The lateral collateral ligament forms the superficial layer, with the remainder of the structures comprising the deep layer. Injury is relatively common and results most frequently from a varus force on an extended joint. The lateral collateral ligament and popliteus tendon are present in all joints, with the popliteofibular ligament being present in approximately 98%. Both the arcuate ligament and fabellofibular ligaments are variable, with the former absent more frequently. Absence of one of these structures is often compensated for by hypertrophy of the other.
612
A child passenger is admitted to accident and emergency following a road traffic collision. Radiographs of the spine show a horizontal fracture involving the vertebral body and pedicles of L2. Associated injury to which of the following abdominal organs is most likely? a. duodenum b. pancreas c. spleen d. liver e. rectum
a. The spinal injury described is a Chance fracture, a fracture through the vertebral body and pedicles caused by hyperflexion, therefore causing compression of the spine anteriorly and distraction posteriorly. This injury typically occurs in back-seat passengers wearing a lap seat belt during a road traffic collision. In children, there is a 50% incidence of associated intra-abdominal organ injury. Retroperitoneal organs are most vulnerable, being closest to the spine. Duodenal injuries are most common, and have a significant associated mortality. The pancreas is also commonly injured due to its retroperitoneal location.
613
A middle-aged man with no significant medical history undergoes a radiograph of the pelvis for localized tenderness following a fall. Multiple longitudinally orientated, 2 – 10 mm rounded densities similar to cortical bone are seen throughout the cancellous bone, in a diffuse symmetrical pattern concentrated around the acetabu- lum. There is no fracture. What is the most likely diagnosis? a. osteopathia striata b. osteopetrosis c. bone metastases d. melorrheostosis e. osteopoikilosis
e. Osteopoikilosis is a rare condition causing multiple enostoses (bone islands), which are asymptomatic and usually of no clinical significance. They represent deposits of normal cortical bone within the cancellous bone. Osteopathia striata (Voorhoeve’s disease) is similar to osteopoikilosis in appearance and is usually asymptomatic, but it consists of linear longitudinal or sunburst striations rather than rounded densities. Osteopetrosis causes generalized increase in bone density, whereas melorrheostosis is a cortical process giving a ‘flowing wax’ appearance, usually affecting only one side of the affected bone. While metastases are plausible, the patient would probably be symptomatic and have evidence or a history of a primary tumour.
614
A 28-year-old physically active young man undergoes a hip MR arthrogram for chronic pain that is worse during exercise. There is a history of several months of hip pain when the patient was a teenager that was not investigated. Images show a loss of the femoroacetabular sulcus superiorly with an associated acetabular labral tear. What is the underlying condition? a. pincer femoroacetabular impingement b. cam femoroacetabular impingement c. combined femoroacetabular impingement d. traumatic labral tear e. osteochondritis dissecans
b. Cam is the most common form of femoroacetabular impingement in men, typically presenting in the third or fourth decade. It is often related to a previous slipped upper femoral epiphysis in the teenage years. A change in the rotational axis (increase in the alpha angle) causes the proximal superior femoral neck to impinge upon the superior acetabular margin and labrum, in turn causing intermittent pain, particularly in physically active individuals. Even without a history of slipped femoral epiphysis, an osseous bump on the superior femoral neck obliterating the femoroacetabular sulcus can cause symptoms. Labral or articular cartilaginous tears can follow repetitive microtrauma, leading to persistent pain and locking. The pincer type is more common in women and is caused by an abnormally deep acetabulum.
615
In the spectrum of perilunate ligamentous injuries and instability, volar tilt of the lunate, seen as a triangular or ‘pie-shaped’ lunate on the AP projection of the wrist, is most commonly a feature of which of the following? a. scapholunate dissociation b. perilunate dislocation c. lunate dislocation d. volar intercalated segmental instability e. dorsal intercalated segmental instability
c. The lesser arc refers to the arc of ligamentous attachments around the lunate. These ligaments become disrupted in a stepwise four-stage fashion. Stage I injury is to the scapholunate ligament, leading to dissociation with rotary subluxation of the scaphoid. Stage II is radiographically characterized by perilunate dislocation, caused by additional injury to the capitolunate joint. The carpus migrates dorsally and the lunate maintains a normal relationship with the radius. Stage III involves the triquetrolunate ligaments, and stage IV is complete disruption of the perilunate ligaments, allowing dislocation and rotation of the lunate. It is this rotation that creates the triangular outline on AP radiographs. Segmental instabilities relate to the spectrum of dynamic scaphoid instability.
616
MRI of the temporomandibular joint is performed for painful clicking, with no history of trauma. Which of the following internal derangements is the most likely underlying cause? a. torn intra-articular disc b. anteriorly displaced intra-articular disc c. hypertrophic/misshapen intra-articular disc d. erosive osteoarthritis e. insufficiency fracture of the condylar neck
b. Abnormal disc position and morphology are the earliest and most sensitive signs of derangement, occurring three to five times more commonly in women and usually manifesting by the fourth decade. The cause is often not found but includes trauma, malocclusion, bruxism (teeth grinding) and primary osseous abnormalities. Disc deformity, if unchecked, leads to secondary osseous abnormality. Degenerative arthritis is typically erosive with flattening of the condylar head and anterior osteophytosis. The disc may become biconcave, thickened, folded or torn. On MRI of the joint, the position of the disc should be assessed first on closed-mouth sagittal T1W images. It may become displaced in any direction, with anterior being the most common. In the initial stages the disc may reduce on mouth closing, but with progression it becomes fixed in an anteriorly subluxed position. The temporomandibular joint can also be an involved site in rheumatoid arthritis.
617
As seen on radiographs of a paediatric skeleton, generalized appendicular findings of poor mineralization of the epiphyseal centres, widening of the growth plate and cupping/fraying of the metaphyses are all frequently associated with which condition? a. osteogenesis imperfecta b. scurvy c. fibrous dysplasia d. rickets e. mucopolysaccharidosis
d. Rickets (vitamin D deficiency) results in failure of normal bone mineralization (osteomalacia) during bone growth and may have a dietary, cultural or metabolic cause. A widened growth plate in a child is due to rickets until proven otherwise. Other features include bowing deformity, metaphyseal cupping and/or fraying, poor epiphyseal mineralization, delayed closure of the fontanelles, enlargement of the costochondral junction (rachitic rosary) and craniotabes. Scurvy is a deficiency of vitamin C and is a primarily a disorder of collagen production, resulting in osteopenia with dense metaphyseal lines and a sclerotic rim around the epiphysis.
618
Which of the following skeletal findings on plain radiographs is not typically associated with achondroplasia? a. short interpedicular distance b. small foramen magnum c. rhizomelia d. horizontal acetabular roof e. atlantoaxial instability
e. Achondroplasia is the most commonly seen autosomal dominant rhizomelic dwarfism. Rhizomelia refers to relative shortening of the proximal compared with the distal portion of the limbs. Achondroplasia has widespread skeletal manifestations affecting the skull, chest, spine, pelvis and extremities. Intelligence and motor function are normal. The most significant complication is brain-stem or spinal cord compression due to spinal stenosis, which is caused by alignment abnormalities and decreased spinal canal size due to short pedicles with a reduced interpedicular distance. Atlantoaxial instability is defined as a predental space of 3 mm or more in adults and 5 mm or more in children, or where there is considerable change between flexion and extension. It is seen in inflammatory arthritides, in Down’s and Morquio’s syndromes, and with retropharyngeal abscess in a child.
619
Plain radiographs of the hands in a young woman are performed for unilateral deformity. These show multiple lytic lesions in the medul- lary cavities of the tubular bones with cortical expansion and matrix mineralization, and associated Madelung deformity. The changes are unilateral. What is the most likely diagnosis? a. Maffucci’s syndrome b. Ollier’s disease c. Trevor’s disease d. Lichtenstein – Jaffe ´ disease e. Morquio’s syndrome
b. Ollier’s disease or multiple enchondromatosis is characterized by the presence of benign intraosseous cartilaginous tumours. The estimated prevalence of the disease is 1 in 100 000. The distribution and number of lesions are variable, but are often unilateral and monomelic. Complications include pain, skeletal deformities, limb length discrepancy (including Madelung’s deformity) and the potential risk of malignant change to chondrosarcoma in 20 – 50% of cases. The condition in which enchondromas are associated with haemangiomas is known as Maffucci’s syndrome. Neither is usually inherited. Trevor’s disease is an epiphyseal dysplasia, whereas Lichtenstein – Jaffe ´ disease is another name for fibrous dysplasia. Morquio’s syndrome is one of the lysosomal storage disorders known as the mucopolysaccharidoses.
620
Looser’s zones – transverse linear lucencies representing areas of poorly mineralized osteoid – are seen with which underlying patho- logical process of bone? a. fracture b. osteomyelitis c. osteoporosis d. osteopetrosis e. osteomalacia
e. Looser’s zones or Milkman’s pseudofractures are seen as linear lucencies in the bone due to incomplete fractures that have non- mineralized osteoid deposited within them. The underlying failure of bone to mineralize is termed ‘osteomalacia’ (which means bone softening), and is most often due to vitamin D deficiency. Rickets is osteomalacia in an immature skeleton. The most common conditions resulting in an osteomalacic process and inadequate bone mineralization include renal osteodystrophy and vitamin D deficiency due to malnutrition/malabsorption of vitamin D or phosphate.
621
On plain radiographs of the neck in a 60-year-old man, which feature is most likely to support a diagnosis of diffuse idiopathic skeletal hyperostosis rather than ankylosing spondylitis? a. enthesopathy b. confluent intervertebral bony bridging c. sparing of the posterior elements d. sparing of the sacroiliac joints e. changes limited to the thoracic spine
d. Diffuse idiopathic skeletal hyperostosis (DISH) is an ankylosing disorder of the spine. It is most commonly seen in the thoracic region but may involve cervical and lumbar regions. Diagnostic criteria are of flowing calcification along the anterolateral border of at least four vertebral bodies, relative preservation of intervertebral disc height, and absence of sacroiliac joint or apophyseal involvement. These three criteria aid differentiation of spondylosis deformans, intervertebral osteochondromatosis and ankylosing spondylitis respectively. Extra-spinal manifestations of DISH include Achilles tendinosis, tennis elbow, calcaneal and olecranon enthesopathy and dysphagia. Whiskering is seen radiographically at tendinous insertions, particularly of the pelvis.
622
On lateral radiographs of the thoracolumbar spine, a central anterior beak of the vertebral bodies is most likely to suggest which of the following conditions? a. Scheuermann’s disease b. Morquio’s syndrome c. Hurler’s syndrome d. Down’s syndrome e. achondroplasia
b. Morquio’s syndrome is type IV and the most common of the mucopolysaccharidoses, a family of inherited disorders of metabolism. A central vertebral beak is relatively specific for the condition. Other spinal manifestations include odontoid hypoplasia with atlantoaxial subluxation (which can be life threatening), platyspondyly, ovoid vertebral bodies, widened intervertebral disc space and exaggeration of the normal lumbar lordosis. Other skeletal findings include dwarfism, as well as skull, face and appendicular abnormalities. Hurler’s syndrome belongs to the same family of disorders but has an inferior vertebral beak, which is also seen in achondroplasia and Down’s syndrome. Scheuermann’s disease does not show vertebral beaking.
623
A young man complains of early morning back pain and stiffness, and undergoes plain radiographs followed by MRI of the whole spine. Which single feature is most likely to suggest a diagnosis of psoriatic arthritis over ankylosing spondylitis? a. syndesmophytes b. parasyndesmophytes c. asymmetrical sacroiliitis d. ankylosis e. patchy bone marrow oedema
b. Seronegative spondyloarthritis is an umbrella term for inflammatory joint or spinal conditions that are not associated with rheumatoid factor or rheumatoid nodules. There are five described subgroups: ankylosing spondylitis, psoriatic arthritis, arthritis associated with inflammatory bowel disease, reactive arthritis (e.g. Reiter’s syndrome) and an undifferentiated subgroup. The subgroups may overlap both clinically and radiologically, and the diagnosis is more easily made on the basis of clinical history and examination. Imaging plays a limited role in differentiation, particularly early in the disease when there can be considerable overlap of appearances. The main exception is the identification of parasyndesmophytes, which are seen almost exclusively in psoriatic arthropathy. In addition, bone marrow oedema can involve the entire vertebral body in psoriatic arthritis, which may be a further useful distinguishing feature. Undifferentiated spondyloarthritis is diagnosed when there is no clinical or radiological evidence of sacroiliitis. All types may eventually progress to ankylosis.
624
A 40-year-old man falls down the stairs and remains unconscious for several hours. On admission to hospital, he is found to have bilateral upper limb weakness, patchy sensory loss, full power in the lower limbs and a normal level of consciousness. Plain radio- graphs of the cervical spine and CT of the brain are normal. On MRI of the cervical spine, there is a small area of oedema identified within the cord. Clinical symptoms persist for 4 days following injury. What is the most likely diagnosis? a. central cord syndrome b. anterior cord syndrome c. SCIWORA (spinal cord injury without radiological abnormality) d. spinal shock e. Brown-Sequard syndrome
a. In trauma, an incomplete spinal cord injury is one in which there is any degree of sparing of motor or sensory function distal to the site of injury, whereas complete cord injury results in complete lack of neurological function distal to the injury. The diagnosis can be made only in the absence of spinal shock, a transient spinal cord concussion. Central cord syndrome is the most common incomplete injury, and is associated with hyperextension injury in middle-aged patients; injury to centrally located grey matter in the cord causes a greater motor neurological deficit in the upper than in the lower extremities. Sensory involvement can be variable, and bowel and bladder function may be affected. Anterior cord syndrome, caused by anterior spinal vascular insufficiency, causes complete motor paralysis with sparing of the posterior columns. SCIWORA is seen in children, when the elastic cervical spine deforms sufficiently to cause cord injury but without any radiological findings. Brown-Se ´quard syndrome results from hemitransection and causes ipsilateral muscle paralysis and contralateral hyperaesthesia to pain and temperature.
625
Vertebral sclerosis confined to the upper and lower endplates with preservation of the intervertebral disc space (‘rugger jersey spine’), is seen most commonly with which underlying condition? a. osteoporosis b. discitis c. mucopolysaccharidosis d. Paget’s disease e. renal osteodystrophy
e. The ‘rugger jersey spine’ appearance refers to sclerotic bands along the superior and inferior endplates of the thoracic and lumbar vertebral bodies. These bands represent accumulation of excess osteoid and result in a striped appearance of the vertebral bodies. Despite being poorly mineralized, the accumulated osteoid appears opaque on plain radiographs because of its increased volume compared with that of normal bone. The ‘rugger jersey spine’ is said to be almost pathognomonic for the osteosclerosis seen with the secondary hyperparathyroidism of chronic renal failure. Renal osteodystrophy is a term for the constellation of musculoskeletal abnormalities occurring with chronic renal failure. Osteoporosis and Paget’s disease are more likely to affect the whole of the vertebrae diffusely. Discitis usually causes a reduction in the intervertebral disc space on radiographs, with indistinct endplates. The mucopolysaccharidoses result in anterior vertebral body beaking rather than sclerosis.
626
A young man is assaulted and attends accident and emergency with a painful left mandible and inability to open and close his jaw without pain. Radiographs show a simple linear fracture through the left body in the parasymphyseal region. A second fracture is most likely to be seen at which of the following sites? a. ipsilateral condylar neck b. ipsilateral angle c. symphysis menti d. contralateral body e. contralateral condylar neck
e. The mandible is best considered as a closed ring, and as such approximately half of all mandibular fractures are bilateral and multiple. The majority occur at the angle, and a significant portion occur at the condylar neck, a common pattern of injury being an ipsilateral body fracture from a direct blow, with a contralateral angle or condylar neck fracture due to transmitted rotation force compressing that side. Fractures of the condylar neck have a limiting effect on the opening and closing of the jaw and can be missed radiographically. Fractures in the midline are also subtle and account for a significant minority. A flail mandible occurs when the anterior support for the tongue is lost due to a bilateral fracture. This injury carries the risk of the tongue prolapsing posteriorly and occluding the airway.
627
A 49-year-old woman presents to her general practitioner with a history of mild midfoot pain exacerbated by walking and wearing tight shoes. Ultrasound scan demonstrates a hypoechoic, 7 mm, rounded lesion lying in the third tarsal interspace. The lesion is poorly demonstrated on MRI, returning intermediate T1 and low T2 signal. Which of the following conditions best explains these findings? a. tendon sheath ganglion b. tendon sheath giant cell tumour c. synovial cyst d. Morton’s neuroma e. paraganglioma
d. Morton’s or interdigital neuroma is a benign lesion consisting of perineural fibrosis that entraps a plantar digital nerve. It is frequently asymptomatic and women represent 80% of cases. Clinical presentation is with foot pain exacerbated by walking, and symptomatic lesions are surgically excised. They are not usually demonstrated on plain radiography and are poorly seen on MRI, returning intermediate T1 and low T2 signal (similar to surrounding tissues). Typical ultrasound appearances are of a hypoechoic rounded lesion, larger in the axial than the sagittal plane. Ganglia and cysts would return high signal on T2W images, and pathology arising from the tendon sheath itself can also show high signal. Giant cell tumours are usually painless.
628
Vertebral sclerosis confined to the upper and lower endplates with preservation of the intervertebral disc space (‘rugger jersey spine’), is seen most commonly with which underlying condition? a. osteoporosis b. discitis c. mucopolysaccharidosis d. Paget’s disease e. renal osteodystrophy
e. The ‘rugger jersey spine’ appearance refers to sclerotic bands along the superior and inferior endplates of the thoracic and lumbar vertebral bodies. These bands represent accumulation of excess osteoid and result in a striped appearance of the vertebral bodies. Despite being poorly mineralized, the accumulated osteoid appears opaque on plain radiographs because of its increased volume compared with that of normal bone. The ‘rugger jersey spine’ is said to be almost pathognomonic for the osteosclerosis seen with the secondary hyperparathyroidism of chronic renal failure. Renal osteodystrophy is a term for the constellation of musculoskeletal abnormalities occurring with chronic renal failure. Osteoporosis and Paget’s disease are more likely to affect the whole of the vertebrae diffusely. Discitis usually causes a reduction in the intervertebral disc space on radiographs, with indistinct endplates. The mucopolysaccharidoses result in anterior vertebral body beaking rather than sclerosis.
629
A middle-aged male patient who has previously undergone partial discectomy for radiculopathy, has a lumbar spine MRI due to a recurrence of his symptoms. T1W images show a low-signal area of tissue contiguous with the previously operated intervertebral disc and impinging upon the adjacent exiting nerve root. Which single additional finding favours a diagnosis of postoperative fibrosis over recurrent disc protrusion? a. high signal on STIR sequence b. enhancement with intravenous gadolinium c. evolution at 6-month serial imaging d. oedema in the surrounding bone e. low signal on T2W images
b. In MRI of the spine in postoperative discectomy patients with recurrent or persistent radiculopathy, a T1W sequence with intravenous gadolinium enhancement is added to distinguish between postoperative epidural fibrosis (or scarring) and recurrent disc herniation. Both can have similar, low-signal appearances on unenhanced T1W and T2W images, but fibrosis will show enhancement with gadolinium whereas recurrent disc prolapse will not. Difficulties arise where both conditions exist concurrently, and fibrosis that is not causing nerve root irritation may also enhance. The importance of distinguishing between the two is that surgical treatment is indicated for recurrent disc herniation but is of no value in treating postoperative fibrosis, also known as failed back syndrome.
630
631
Following major trauma, which of the following fractures of the thoracic skeleton is most likely to indicate a significant injury to the underlying intrathoracic viscera? a. glenoid b. scapular blade c. clavicle d. first rib e. sternum
d. First rib fracture is considered a harbinger of major trauma, with approximately two-thirds of fractures being associated with major chest injury and carrying a significant mortality. The anatomy of the first rib is such that it is protected from the minor insults that often break other ribs, and fracture of the first rib usually indicates violent blunt trauma to the thorax. Associated local injuries include damage to the brachial plexus, major vascular structures and the underlying lung and heart. There is also an association with significant abdominal injury, but the major cause of death in patients with a first rib fracture is an associated head injury. It is rare for a first rib fracture to be an isolated finding.
632
A young man sustains an obvious head injury during an assault, with clinically apparent, left temporoparietal swelling and an underlying fracture on skull radiographs. The patient’s GCS is initially normal but begins to deteriorate progressively after 4 hours of observation, and CT of the brain is requested. Which finding other than a skull fracture would you expect to find on CT to explain the patient’s condition? a. diffuse axonal injury b. haemorrhagic contusions c. subarachnoid haemorrhage d. subdural haematoma e. extradural haematoma
e. Extradural (or epidural) haematoma is the accumulation of blood in the potential space between the inner table of the skull and the dura. Ninety per cent of cases are associated with a fracture of the temporal bone which traverses and ruptures the middle meningeal artery (in 60 – 90%) or vein. In children, vascular injury may occur here without a fracture. In half of cases, there is a lucent interval between injury and the onset of deteriorating consciousness level, as opposed to diffuse axonal injury where coma is immediate. CT appearances are of an extra-axial, biconvex, high-attenuation collection. A subdural haematoma is caused by shearing forces on small bridging veins and is not necessarily associated with a fracture of the calvarium, although this may coexist. It differs from an extradural haematoma not in radiographic location but in that it has a crescentic rather than a biconvex shape.
633
A young male patient sustains an external rotational injury to his left ankle and is unable to bear weight. A plain radiograph of the ankle performed in accident and emergency shows no fracture but does show soft-tissue swelling over the medial malleolus and widen- ing of the ankle joint space medially (lateral talar shift). Which of the following additional view(s) should be performed? a. mortise view b. calcaneus c. foot d. knee e. contralateral ankle
d. The Maisonneuve fracture is a spiral fracture of the upper third of the fibula associated with a tear of the distal tibiofibular syndesmosis and the interosseous membrane. The medial component of the injury may be an associated fracture of the medial malleolus or rupture of the deep deltoid ligament. The ankle joint is effectively a bony ring that extends up to the knee. Interruption of the ring in this way allows lateral displacement of the fibula and so disruption of the congruence of the ankle mortise, resulting in an unstable ankle injury that requires surgical fixation.
634
Of the following subtypes of osteosarcoma, which is associated with the most favourable 5-year survival? a. multicentric b. periosteal c. paraosteal d. telangiectatic e. soft-tissue
c. Osteosarcoma is the second most common primary malignancy of bone after multiple myeloma, accounting for 15% of all primary bone tumours. It usually affects those aged 10 – 30. Ninety-five per cent are of the primary osseous type and, of these, paraosteal osteosarcoma has the most favourable 5-year survival rate of 80%. Other osteosarcomas of the primary osseous type include periosteal (5-year survival rate 50%) and telangiectatic (less than 20%). Multicentric refers to synchronous osteoblastic osteosarcomas at multiple sites. It occurs exclusively in children aged 5 – 10, and carries an extremely poor prognosis. The soft-tissue type is rare, representing only 1.2% of all soft-tissue tumours. These lesions are primary soft-tissue tumours with no attachment to bone. Death occurs within 3 years in the majority of cases, tumour size being the major predictor of outcome.
635
A 30-year-old woman undergoes plain radiographic imaging of the hand for a palpable, painful hard lump on the dorsum. Plain radio- graphs show a well-defined bony mass applied closely to the diaphy- sis of the second metacarpal. CT shows a wide-based pedunculated lesion with a perpendicular orientation to the diaphysis, no cartilage cap and a matrix of mature trabeculated bone. What is the most likely diagnosis? a. osteochondroma b. multiple osteocartilaginous exostoses c. bizarre paraosteal osteochondromatous proliferation d. Codman’s tumour e. dysplasia epiphysealis hemimelica
c. Bizarre paraosteal osteochondromatous proliferation (also known as Nora’s lesion) is a rare condition usually seen in adults in the third and fourth decades of life. Osteochondroma-like lesions are seen most commonly at the proximal and middle phalanges, followed by the metacarpals and metatarsals. A relationship to trauma has been suggested but not proven. Other locations that may be affected include the long bones (especially those of the upper extremity), skull and jaw. It is thought to be a similar process to that which gives rise to lesions in myositis ossificans, reactive periostitis and subungual exostosis. On plain radiographs, a well-defined bony mass is seen attached to the surface of the parent bone. Features differentiating this from osteochondroma are the absence of angulation away from the nearby physis and a wide base.
636
An elderly female patient has plain radiographs performed in an outpatient clinic for bilateral painful, stiff hips, which demonstrate joint space narrowing. Which additional feature is more likely to support a diagnosis of rheumatoid arthritis rather than osteoarthritis? a. eccentric joint space loss b. soft-tissue swelling c. subchondral sclerosis d. subchondral cysts e. protrusio acetabuli
e. Even or eccentric joint space reduction representing cartilage loss is seen in both types of arthritis and not a distinguishing diagnostic feature. Although osteoarthritis is said to be classically eccentric, this is difficult to assess accurately on many hip radiographs, as they are not routinely taken in the upright, weight-bearing position. Subchondral sclerosis and cysts are typically associated with degenerative osteoarthritis. Although soft-tissue swelling is a feature of rheumatoid arthritis, the depth of the hip joint and the copious surrounding soft tissues mean that any synovial swelling is unlikely to be appreciated clinically or radiologically. Subtle osteophytes (in osteoarthritis) or erosive change/osteoporosis (in rheumatoid arthritis) can distinguish between the two entities. An inflammatory cause should be considered in young adults with hip pain and, if protrusio or other abnormalities are found, the sacroiliac joints should be examined.
637
A middle-aged woman, known to suffer from polyostotic fibrous dysplasia, presents with a palpable, 3 cm, soft-tissue mass in the upper left thigh. MRI shows a relatively homogeneous, smooth, well-defined lesion located in an atrophic quadriceps muscle, which returns low signal on T1W images and high signal on T2W images. Following administration of intravenous gadolinium, the lesion shows moderately intense heterogeneous enhancement. What is the most likely pathological nature of the soft-tissue lesion? a. soft-tissue myxoma b. malignant fibrous histiocytoma c. soft-tissue cavernous haemangioma d. multiple lipomatosis e. rhabdomyosarcoma
a. The association of fibrous dysplasia and soft-tissue myxoma is well established and is commonly termed Mazabraud’s syndrome. The key is identifying the relationship between the bone and soft-tissue pathology, with the osseous features of fibrous dysplasia usually preceding the formation of a soft-tissue mass. The condition is non-familial and more commonly affects women, the thigh being the most common location. Typical MRI appearances are of a well-defined lesion with signal intensity similar to water, and often a fat rind or adjacent muscle high signal on T2W images is seen. Although uncommon, there have been reported cases of malignant change into osteosarcoma.
638
Which of the following locations is most often associated with post-traumatic osteolysis? a. coronoid process of ulna b. surgical neck of humerus c. lateral clavicle d. femoral neck e. fibular head
c. The lateral third of the clavicle is the most common location for post- traumatic osteolysis. It is usually preceded by a fairly severe injury to the shoulder, typically dislocation or subluxation of the acromioclavicular joint. Changes may be evident radiographically after as little as 1 month. If no bone loss was apparent on radiographs at the time of injury, the diagnosis is unequivocal. However, if no such comparison can be made, then other causes of lateral clavicular osteolysis include rheumatoid arthritis, scleroderma and hyperparathyroidism. Other sites affected are the pubic and ischial rami, distal portions of the radius or ulna, the carpus and femoral neck. Widespread idiopathic osteolysis is termed Gorham’s or vanishing bone disease.
639
An elderly woman falls down the stairs and suffers a Malgaigne fracture of the pelvis and a 1% degloving injury to the left forearm. Due to significant medical co-morbidity, the decision is made not to treat with surgery. The patient dies overnight on the ward. What is the most likely mechanism of death? a. pulmonary embolism b. fat embolism c. septicaemia d. myocardial infarction e. intra-abdominal haemorrhage
e. A Malgaigne fracture of the pelvis is a fracture of the ischiopubic rami and an ipsilateral sacroiliac joint (or para-articular) fracture, and occurs due to high-energy blunt trauma. It represents complete disruption of the pelvic ring and therefore an unstable fracture. In such fractures, distortion and disruption of the pelvic soft tissues and vascular injury involving the rich blood supply in the pelvis will not be tamponaded by the bony ring, as the pelvis will expand to accommodate ever-increasing haematoma. Mortality rate from major pelvic trauma is 10%; other common causes of death include multiorgan failure and sepsis, the latter expected to take several days to evolve.
640
A 20-year-old man is a restrained driver in a high-speed road traffic collision. On admission to accident and emergency he undergoes CT of the brain for a reduced consciousness level. Images show diffuse brain injury. Which of the following findings would support a diagnosis of diffuse axonal injury rather than simple contusions? a. corticomedullary petechial haematoma b. anterior temporal petechial haematoma c. basofrontal petechial haematoma d. intraventricular haemorrhage e. brain oedema
a. Contusions are traumatic injuries to the cortical grey matter of the brain and make up approximately half of primary intra-axial traumatic lesions. They are often multiple and bilateral, with the most common locations being the inferior frontal lobes and temporal poles. Diffuse axonal injury results from rotational shearing forces on cerebral white matter and common locations are white matter-rich areas, such as the corticomedullary junction, centrum semiovale, corpus callosum and cerebellum. In comparison to diffuse axonal injury, contusions tend to be larger and more superficial, with a higher proportion being haemorrhagic due to the increased vascularity of grey compared with white matter. Local or widespread oedema can be seen in both conditions.
641
A 40-year-old woman presents to the emergency department with a painful, stiff shoulder, 12 hours after undergoing arthrography of the same joint. She describes onset of symptoms 8 hours previously with progressive worsening. She feels otherwise well with a temp- erature of 37.3oC. There is no overt joint swelling or overlying erythema. What is the most likely cause? a. septic arthritis b. chemical synovitis c. joint haemarthrosis d. joint effusion e. allergic contrast reaction
b. Post-arthrography pain due to sterile chemical synovitis is the most common complication of the procedure, typically beginning after 4 hours and peaking at 12 hours. Other, less common, immediate and short-term complications include allergic contrast reaction (rare in intra-articular injections), introduction of infection and vasovagal reaction.
642
Which of the following features is not a recognized primary musculoskeletal manifestation of the CREST (calcinosis, Raynaud’s phenomenon, oesophageal involvement, sclerodactyly and telangi- ectasia) syndrome? a. digital oedema b. calcinosis c. acro-osteolysis d. osteoporosis e. joint erosions
d. CREST syndrome represents the limited form of the autoimmune connective tissue disorder scleroderma. Five-year survival rate is 50 – 67%. The two other types of scleroderma are generalized (also called systemic sclerosis) and localized (morphoea). Common findings are digital soft-tissue oedema with sclerodactyly (tapered soft tissues), acro-osteolysis (autoamputation) and calcinosis. There may be associated arthritis that shows erosions (also seen in the ribs) or terminal phalanx resorption, with joint space narrowing a late sign. Osteoporosis is not usually a feature except in the context of disuse.
643
In reviewing a fracture of the spine at the thoracolumbar junction in a major trauma case, which single indicator on CT is most sensitive for inferring instability? a. widened facet joints b. two-column malalignment c. soft-tissue swelling d. rotational abnormality e. increased intervertebral disc space
b. The spine can be divided anatomically into three columns: the anterior column contains the anterior longitudinal ligament, anterior half of the vertebral body and anterior annulus fibrosus; the middle column contains the posterior half of the vertebral body, posterior longitudinal ligament and the posterior annulus fibrosus; and the posterior column contains the posterior elements of the spine, facet joint capsule and interspinous ligaments. Two intact columns are required for intrinsic spinal stability. Disruption of two columns can therefore be used to infer instability. Usual traumatic patterns are anterior and middle, or posterior and middle, disruption. Isolated middle column interruption can occur after trauma or surgery, or as a congenital abnormality, and is also considered potentially unstable.
644
Which of the following is not a recognized cause of myeloid hyperplasia (red marrow reactivation/reconversion) in a 50-year- old adult? a. sickle cell disease b. smoking c. chemotherapy d. long-distance running e. Gaucher’s disease
e. Marrow reconversion is the repopulation of yellow (fatty) marrow with haematopoietic cells, reconverting the fatty marrow to red marrow. This occurs when the haematopoietic capacity of the existing red marrow in an adult is insufficient. This can result from increased physiological requirement (long distance running), chronic anaemia (sickle cell disease) and chemotherapeutic treatment with granulocyte – macrophage colony-stimulating factor. The pattern of reconversion is predictable and it is the reverse of the age-related physiological conversion of red to yellow marrow. Reconversion begins in the axial skeleton and progresses distally through the appendicular skeleton, to end in the hands and feet. Knowledge and recognition of this pattern are important because neoplastic infiltration of adult yellow marrow in malignant disease may have similar MR appearances. However, malignant marrow replacement tends to have a more random distribution than reconversion; it may enhance with intravenous gadolinium, can cause cortical destruction and may extend into the soft tissues.
645
Plain radiographs of the knees are performed in a teenage girl with growth retardation and painful, deformed lower limbs. Which radiographic finding would suggest a diagnosis of scurvy rather than rickets? a. pathological fractures b. bowing deformity c. widened growth plate d. frayed metaphysis e. sclerotic epiphyseal rim
e. Rickets is a deficiency of vitamin D in a child that results in osteomalacia of the immature skeleton. Scurvy is a deficiency of vitamin C and is a disorder of collagen synthesis that can occur in children or adults. Pathological fractures may be seen in both conditions. Ground-glass osteoporosis is characteristic of scurvy, with other features including a sclerotic line in the metaphyseal zone of preparatory calcification (white line of Fra ¨nkel), a radiolucent zone immediately to the diaphyseal side of the white line (Tru¨mmerfeld’s zone), corner fractures (Parke’s corner sign) and a sclerotic ring around the epiphysis (Wimberger’s sign). In addition, bleeding diathesis is seen in scurvy; therefore, subperiosteal haematoma and haemarthrosis are also features.
646
Which of the following local factors is not associated with an increased risk of fracture non-union? a. infection b. fracture mobility c. avascular fragments d. impaction e. open fracture
d. Fracture sites that have a poor blood supply, either as a result of the original injury or due to subsequent surgical treatment, may go on to develop atrophic non-union where the bone ends become osteoporotic, thin and pointed (osteolysis) with no evidence of fracture healing. A fracture site that is very mobile may develop hypertrophic non-union where there is attempted healing denoted by excessive callus formation, but the fracture cleft persists. Open fractures are often high energy, with soft-tissue damage and comminution of fracture fragments, and are prone to infection, all of which predispose to non-union. Osteomyelitis in any fracture can result in delayed, non- or malunion. Non-union in most skeletal locations should not be diagnosed radiographically until 6 months have passed, particularly in the presence of complicating factors.
647
On MRI of the foot performed for non-specific pain, which single feature is most specific for a diagnosis of sinus tarsi syndrome? a. subtalar joint effusion b. subtalar sclerosis c. loss of fat signal in the sinus d. bone marrow oedema in the talus e. flexor tendon high signal on T2W images
c. Sinus tarsi syndrome is a common complication of ankle sprains, but may also result from an inflammatory arthropathy. It is associated with abnormalities of one or more structures in the tarsal sinus and tarsal canal that lead to pain and a feeling of instability of the hindfoot. Most patients with this syndrome present in the third or fourth decade of life with persistent lateral foot pain, though the pathogenesis of the condition is poorly understood. Conventional radiography generally is not valuable, but on MRI there is alteration of the fat signal, the most common changes being diffuse low-signal-intensity infiltration on both T1W and T2W images. Other common MR findings include synovial thickening and diffuse enhancement of the tarsal sinus following intravenous gadolinium.
648
A middle-aged man has a history of an undiagnosed wrist injury interfering with his playing golf. He presents with clinically appar- ent ulnar nerve compression at the wrist. Which of the following causes is most likely to be identified following investigation with CT and MRI? a. non-union of hook of hamate fracture b. non-union of scaphoid wrist fracture c. scapholunate dissociation d. pisiform osteoarthritis e. triangular fibrocartilage complex tear
a. Fractures of the hook of the hamate are the most frequent type of hamate fracture, and most often occur from the repetitive stress of swinging a bat, club or racket, or from the direct blow of a club on the ground. This may result in ulnar nerve compression at the wrist in Guyon’s canal, which is particularly exacerbated in the context of non-union due to secondary osteoarthritis or loose bodies in the pisotriquetral joint. Other causes of ulnar nerve compression at the wrist include adjacent masses, anomalous muscles and tendons, fibrous palmar arch, ulnar artery aneurysm, primary osteoarthritis of the pisotriquetral joint, os hamuli proprium and dislocation of the pisiform bone.
649
A 50-year-old mechanic with a long history of back pain presents to the spinal clinic complaining of sudden onset of numbness and pain over the right lateral calf and dorsum and sole of the right foot following heavy lifting. Which of the following spinal pathologies is most likely to explain the patient’s symptoms? a. lumbar spinal stenosis b. paracentral L4 – 5 disc protrusion c. paracentral L5 – S1 disc protrusion d. far lateral L4 – 5 disc protrusion e. central L5 – S1 disc protrusion
b. Degenerative disc disease of the spine is one of the leading causes of functional incapacity and chronic disability in the working population, affecting both men and women. Although there is no universally established nomenclature for describing disc herniation, ‘protrusion’ is commonly used if the herniation is broader than it is deep and ‘extrusion’ if it is deeper than it is broad. A disc ‘bulge’ is used to describe a herniation that is very broad based and may even be circumferential, with a generalised disc bulge being one that affects at least half of the periphery. As a result of the strong posterior longitudinal ligament, posterior disc herniation is often paracentral, i.e. to the side of the midline. This can result in compression of the transiting nerve root in the lateral recess, which is the one that will exit at the level below. A lateral disc herniation narrowing the neural foramen compresses the exiting nerve root. Therefore, for a given intervertebral disc, a paracentral herniation will affect the nerve that exits one level below, whereas a lateral protrusion affects the nerve root at that level.
650
A 72-year-old women presents to the rheumatologist with a long history of shoulder pain affecting her dominant arm that began at night with associated stiffness, but has suddenly worsened over the past few weeks. Radiographs show a superiorly subluxed humeral head forming a pseudarthrosis with the acromion, gleno- humeral joint space loss, humeral head collapse with cysts and scler- osis, and periarticular soft-tissue calcification. Ultrasound scan demonstrates an effusion with widespread degeneration of the rotator cuff and a complete tear of the supraspinatus tendon. Exam- ination of aspirated joint fluid shows calcium hydroxyapatite crys- tals. What is the most likely diagnosis? a. Milwaukee shoulder b. pseudogout c. myositis ossificans progressiva d. erosive osteoarthritis e. scleroderma
a. Milwaukee shoulder is a crystal deposition disease of basic calcium phosphate, predominantly affecting elderly women and resulting in a dysfunctional shoulder from destruction of the rotator cuff. It is often bilateral but always involves the dominant side. Radiographic findings include superior subluxation of the humerus due to loss of the superior rotator cuff, often forming a pseudarthrosis with the clavicle or acromion. Glenohumeral degeneration manifests as sclerosis and collapse of the humeral head, joint space narrowing and osteophyte formation. Erosion at the site of rotator cuff insertion and periarticular soft-tissue calcification is also a feature. Examination of effusion fluid is stereotypical, revealing spheroid-shaped aggregates of hydroxyapatite crystals. The condition is also seen in the knee, where, unlike osteoarthritis, it predominates in the lateral compartment.
651
652
Which of the following is not an appropriate indication for percutaneous polymethylmethacrylate cement vertebroplasty? a. progressive osteoporotic deformity b. painful osteoporotic collapse c. painful haemangioma d. painful osteoid osteoma e. painful metastases
d. Percutaneous cement vertebroplasty is a treatment for vertebral compression fractures that involves the injection of acrylic bone cement into the vertebral body in order to relieve pain, stabilize fractured vertebrae or, in some cases, restore vertebral height. Current guidelines from the National Institute for Health and Clinical Excellence (NICE), regarding the use of vertebroplasty in the UK, state that it may be used for pain relief in patients with severe painful osteoporosis with loss of height, compression fractures of the vertebral body, symptomatic vertebral haemangioma and painful vertebral body tumours (metastases or myeloma). Review of current evidence indicates some level of pain relief in 58 – 97% of patients.
653
By the middle of the third decade, the adult pattern of haemato- poietic and fatty marrow distribution is established. Red marrow remains in all but which of the following locations? a. sternum b. clavicles c. ribs d. proximal humeri e. distal femora
e. MRI is superior to other imaging modalities in evaluating red (haematopoietic) and yellow (fatty) bone marrow, as a result of its very high sensitivity for lipid, which is present in significantly higher concentrations in yellow marrow than in red. At birth, haematopoietic marrow is present throughout the entire skeleton. A predictable sequence of conversion of red to yellow marrow begins distally in the hands and feet, and migrates proximally (over a period of 20 years) until red marrow remains only in the axial skeleton and the proximal humeral and femoral metaphyses. Reversal of this process is called reconversion. Appreciation of the areas of remaining red marrow in an adult is important, because malignant marrow infiltration can have a similar appearance to haematopoietic marrow, with location being a major discriminator.
654
A 35-year-old woman of African origin is admitted to accident and emergency with acute abdominal and back pain with pyrexia. Radiographs of the chest, abdomen and lumbar spine show rib thinning with notching, sclerosis of the right humeral head and biconcave, ‘fish-shaped’, lumbar vertebral bodies. A subsequent radiograph of the skull reveals widening of the diploe ¨ with hair- on-end striations. What is the most likely underlying condition? a. neurofibromatosis b. thalassaemia major c. sickle cell disease d. syphilis e. Scheuermann’s disease
c. Sickle cell disease is a haemoglobinopathy that results from the presence of abnormal b -globin chains within haemoglobin, which may manifest as anaemia, infarction and superimposed infection. It is much more prevalent in those of African – Caribbean origin. Over time, the disease produces musculoskeletal abnormalities as a result of chronic anaemia, such as marrow proliferation (which produces the characteristic changes in the skull), marrow reconversion and extramedullary haematopoiesis. Other common skeletal complications include bone softening, infection or infarction.
655
Radiographic arthrography of the shoulder with injection of contrast into the glenohumeral joint is performed for a painful joint with a globally reduced range of movement. Which single finding is most likely to indicate a diagnosis of adhesive capsulitis? a. pain on injection of contrast b. small axillary recess c. contrast tracking along the subscapularis muscle d. contrast in the subacromial space e. obliteration of the subcoracoid fat
b. Adhesive capsulitis or frozen shoulder is clinically characterized by restriction of both active and passive elevation and external rotation. Patients are commonly 40 – 70 years old and predominantly female. It may be idiopathic, preceded by trauma, or associated with diabetes mellitus or other conditions. Patients have been shown to have a significantly thickened coracohumeral ligament and joint capsule, and an axillary recess significantly reduced in volume. Obliteration of the fat triangle between the coracohumeral ligament and the coracoid process is specific when seen on MR arthrography. Treatment options include physiotherapy, intra-articular corticosteroid injection, manipulation under anaesthetic and surgical capsulotomy.
656
656
On plain radiographs of the long bones or the spine, which of the following is not a recognized cause of a ‘bone within a bone’ appearance? a. infant physiology b. sickle cell anaemia c. nutritional disturbance d. renal osteodystrophy e. metastatic disease
d. ‘Bone within a bone’ is a term used to describe a radiographic appearance in which one bone appears to arise within another. It can be a physiological finding in a neonate or infant due to new bone formation. Pathological conditions that can cause the appearance include periosteal new bone formation, cortical splitting, subcortical osteopenia, altered bone growth, impairment of osteoclastic activity, altered bone metabolism, crystal deposition, and iatrogenic and technical radiological factors. It is not a feature of renal osteodystrophy but is seen in hypervitaminosis D and in healing rickets.
657
On radiographs and MRI of the spine performed for lower back pain with clinical signs of radiculopathy, which of the following features favours a diagnosis of discitis rather than degenerative disc disease? a. vacuum phenomenon in the discs b. reduced disc space c. intermediate signal posterior to the vertebral body on T1W images d. vertebral endplate low signal on T1W images e. Schmorl’s nodes
c. Intermediate signal in the extradural space on T1W images is the most common appearance of extradural abscess formation. The most common primary focus of infection is discitis, but abscess formation may also be spontaneous. Patients particularly at risk are those with a history of diabetes mellitus, intravenous drug use, trauma, haemodialysis or recent surgery (particularly dental). MRI features of extradural abscess include iso- or slight hyperintensity on T1W images when compared with the spinal cord. High signal on T2W and proton density sequences makes it difficult to differentiate abscess from CSF, but these sequences are useful, as osteomyelitis and paravertebral abscess are well visualized as high-signal lesions. Administration of intravenous gadolinium contrast characteristically demonstrates diffuse enhancement of the solid component of the abscess.
658
Vertebral bone marrow oedema, seen as low signal on T1W and high signal on T2W MR images, occurs typically in all but which of the following conditions? a. degenerative disc disease b. multiple myeloma c. osteoporotic collapse d. spondylolysis e. ankylosing spondylitis
b. Multiple myeloma is a malignant condition of plasma cells that commonly shows infiltration of the bone marrow, best seen on MRI. Patterns of infiltration can be classified as focal, diffuse or variegated. Although marrow infiltration returns similar signal to marrow oedema on T1W and T2W images, infiltration will show diffuse enhancement following administration of intravenous gadolinium. The pattern of infiltration also differs. Infiltration will be patchy and randomly distributed throughout the vertebral bone. In contrast, bone oedema occurs adjacent to its cause, being linear at the endplates in the case of degenerative disc disease, and linear with a fracture line in osteoporotic collapse, in the pedicles adjacent to spondylolysis or at the entheses in ankylosing spondylitis.
659
An elderly man undergoes 99mTc-labelled diphosphonate bone scin- tigraphy. There is no uptake of tracer in the soft tissues, urinary tract or appendicular skeleton, but the axial skeleton shows diffuse homogeneous tracer uptake. No focal lesions are seen. What is the most likely cause of these appearances? a. prostatic metastases b. renal osteodystrophy c. Paget’s disease d. mastocytosis e. myelofibrosis
a. The resulting pattern following diffuse uptake of 99mTc-labelled diphosphonates in the axial skeleton, with little or no uptake of tracer in the soft tissues or urinary tract, is frequently referred to as a superscan. When there is little uptake in the limbs, the cause is most likely to be diffuse metastases in the axial skeleton, most commonly prostatic or breast. Uptake in metabolic bone disease is more uniform in appearance, and extends into the distal appendicular skeleton. Intense calvarial uptake disproportionate to that in the remainder of the skeleton may also be seen. The most important factor is to recognize the scan as abnormal in the absence of focal lesions. The lack of renal uptake (absent kidney sign) is a useful discriminator.
660
A 65-year-old man undergoes radiographs of the lumbar spine and pelvis for lower back pain. A destructive lytic lesion is identified in the midline of the inferior sacrum with internal areas of calcifica- tion. Subsequent MRI reveals a heterogeneous lesion replacing much of the sacrum, which returns moderate low signal on T1W and high signal on T2W images, with a soft-tissue component extending into the presacral soft tissues. The lesion shows patchy moderate enhancement with intravenous gadolinium. What is the most likely diagnosis? a. metastasis b. giant cell tumour c. aneurysmal bone cyst d. chordoma e. plasmacytoma
d. Chordomas arise from notochordal rests and therefore almost always occur in the midline. They are the most common primary malignant sacral tumour and account for 2 –4% of all malignant tumours of bone. They are found at all ages but most commonly occur in the fourth to seventh decades of life. Approximately half develop in the sacrococcygeal region. There is usually a large soft-tissue component and the tumour may extend across the intervertebral disc space or sacroiliac joint. Overall, the most common sacral lesion is metastasis due to the high red marrow content, but other primary malignant lesions include myeloma, Ewing’s sarcoma and lymphoma. The most commonly found benign tumours are giant cell tumours and aneurysmal bone cysts. Despite being relatively common in the rest of the spine, haemangiomas and osteoid osteomas are rare.
661
A 70-year-old man attends a 6-week follow-up appointment after cemented total hip arthroplasty, complaining of a poor range of motion. Radiographs taken during the appointment show small areas of pericapsular bone, and formation of small bony spurs at the acetabular margin. CT demonstrates these areas to have well- defined mineralization peripherally and indistinct centres. Which of the following processes are responsible? a. femoral component loosening b. heterotopic ossification c. periprosthetic fracture d. postoperative infection e. stress shielding
b. Heterotopic ossification, also known as myositis ossificans, is a benign, self-limiting process of ossification occurring within skeletal muscle. Seventy-five per cent of cases are due to trauma (including iatrogenic trauma), with other causes including paralysis, burns, tetanus and intramuscular haematoma. The areas of new bone are surrounded by fibrotic connective tissue, which can be seen as a soft-tissue mass on MRI. Some heterotopic ossification is seen in half of all total hip replacements, with one-third considered clinically significant. It is classified radiographically according to the Brooker classification.
662
A 30-year-old woman in the third trimester of pregnancy complains of a 4-week history of gradual onset of pain in the left hip following minor trauma. Radiographs show a normal-appearing joint space, mild osteopenia of the femoral head and neck, and an indistinct subchondral femoral head. On subsequent MRI, the bone marrow in the affected regions returns patchy but diffuse low signal on T1W and high signal on T2W images. There is a similar small area of marrow abnormality in the acetabulum, and a small hip effusion is seen. What is the most likely diagnosis? a. septic arthritis b. infarction c. reflex sympathetic dystrophy d. rheumatoid arthritis e. transient osteoporosis
e. Transient osteoporosis of the hip is part of the bone marrow oedema syndromes that also encompass migratory regional osteoporosis and transient bone marrow oedema. The condition is spontaneous and self- limiting, clinical recovery occurring in several weeks to months with no specific treatment, although radiographic changes lag behind. It is seen in pregnant women in the third trimester and middle-aged men. The radiographic hallmark is the loss of subchondral cortex in the femoral head, and marrow oedema is seen on MRI with intense uptake of 99mTc-labelled diphosphonates on bone scintigraphy. The aetiology is uncertain, but speculation has been made that the bone marrow oedema syndromes are related to reflex sympathetic dystrophy. The appearance of transient osteoporosis of the hip may be mimicked by osteonecrosis.
663
Following wrist arthrography by a single-compartment radiocarpal injection technique, contrast seen on MR arthrographic images in the midcarpal compartment can be explained by disruption of which of the following structures? a. triangular fibrocartilage b. lunotriquetral ligament c. dorsal distal radioulnar ligament d. flexor retinaculum e. radioscapholunate ligament
b. The two most important intercarpal ligaments are the scapholunate and lunotriquetral ligaments. These are crescent shaped with strong anterior and posterior zones and a relatively thin middle membrane. Disruption of either of these will result in communication of the radiocarpal compartment proximally with the midcarpal compartment distally. Contrast material seen in the distal radioulnar joint indicates disruption to the triangular fibrocartilage complex or distal radioulnar ligaments. Some authors advocate selective midcarpal injection as superior in delineating injury to the scapholunate and lunotriquetral ligaments, and a sequential technique of three injections has also been described.
664
A young footballer sustains a twisting injury to the right knee in training. He is able to continue practising but complains of moder- ate medial knee pain. The following morning he wakes with a swollen stiff joint. Radiographs show an effusion only. Subsequent MRI confirms an effusion and reveals a truncated medial meniscus with a ‘bow-tie’ configuration seen on only a single sagittal image. Sagittal sequences reveal a ‘double’ appearance of the posterior cruciate ligament. He has not had any previous surgery. What is the most likely injury or combination of injuries? a. torn medial meniscus b. torn medial meniscus and anterior cruciate ligament c. torn medial meniscus and posterior cruciate ligament d. torn anterior cruciate ligament e. torn posterior cruciate ligament
a. Truncation of a meniscus may be due to previous injury or surgical resection, but in the absence of a relevant history it suggests meniscal tear with displacement of the body of the meniscus. On sagittal sequences, one would normally expect to see a full ‘bow-tie’-shaped meniscus on three or more contiguous images, as the meniscal body is approximately 11 mm in thickness (this of course will depend on slice thickness). Any fewer suggests a meniscal body tear with displacement of the fragment. The fragment often flips into the intercondylar groove of the femur to lie anterior and parallel to the posterior cruciate ligament, giving the impression of two similar structures. This injury is known as a bucket-handle tear.
665
Plain radiographs of the femur performed for pain reveal a centrally located lucent lesion in the medulla with a partially calcified matrix. Which of the following features favours a diagnosis of chondrosar- coma over enchondroma? a. arc-and-ring matrix calcification b. ground-glass matrix c. multiple lesions d. deep endosteal scalloping e. lesion size over 5 cm
d. Distinction of enchondroma and intramedullary chondrosarcoma in the appendicular skeleton proximal to the metacarpals/-tarsals is difficult radiologically. A series of 187 patients showed that chondrosarcoma was associated with endosteal scalloping, with scalloping involving more than two-thirds of the extent of the lesion being strongly suggestive of malignancy. Other powerful discriminating factors identified as favouring chondrosarcoma were cortical destruction, soft-tissue mass, periosteal reaction, radionuclide uptake at scintigraphy and pain associated with the lesion. Chondrosarcoma also tended to be larger with a mean size of 10 cm compared with 6.7 cm for enchondroma. A ground-glass matrix with arcuate calcification is characteristic of both types of cartilaginous lesion. Multiple lesions may be seen in both malignancy and enchondromatosis (Ollier’s disease).
666
A 70-year-old hospitalized male patient presents with watery diarrhoea and abdominal pain. CT of the abdomen demonstrates marked circumferential bowel wall thickening involving the entire colon, with minimal pericolonic stranding and a small amount of ascites. The small bowel appears normal. What is the most likely diagnosis? a. Crohn’s disease b. ischaemic colitis c. diverticulitis d. pseudomembranous colitis e. ulcerative colitis
d. Pseudomembranous colitis is an acute infectious colitis caused by Clostridium difficile and its toxins A and B; this pathogen has become increasingly common largely due to widespread use of broad-spectrum antibiotics. The commonest CT finding is of colonic wall thickening (due to mural oedema and the presence of pseudomembranes), which is typically greater than in other causes of colitis apart from Crohn’s disease. Pericolonic inflammatory changes are disproportionately mild relative to the marked wall thickening. Ascites is common, and this, together with the lack of small bowel involvement, can help to distinguish pseudomembranous colitis from Crohn’s colitis. Ischaemic colitis demonstrates a lesser degree of wall thickening, and is usually segmental, tending to affect the watershed areas of the colon.
667
A 64-year-old man undergoes a barium meal examination for upper abdominal pain. A 10 mm ulcer is demonstrated at the gastric antrum. Which radiological feature would favour a diagnosis of malignant rather than benign gastric ulcer? a. round ulcer shape b. ulcer crater confined within the gastric contour c. gastric folds identified up to the edge of the ulcer crater d. associated duodenal ulcer disease e. uniform mucosal collar around a centrally located ulcer
b. Many distinguishing features of gastric ulceration have been proposed in an attempt to classify gastric ulcers as benign or malignant, but there is significant overlap between the two categories. One reliable sign of a benign ulcer is the projection of the ulcer outside the gastric contour in profile, due to excavation into the mucosal wall. In contrast, a malignant ulcer occurring within a tumour mass does not usually extend beyond the confines of the gastric wall. Other features indicative of benignity include a round, centrally located ulcer with a uniform collar of oedematous mucosa, gastric folds extending to the edge of the ulcer crater and associated duodenal ulcer disease.
668
A 68-year-old woman presents with small bowel obstruction, and undergoes contrast-enhanced CT of the abdomen. This demon- strates dilated small bowel to the level of the mid-ileum, where a herniated loop of small bowel is seen emerging inferolateral to the left pubic tubercle. What is the most likely cause of small bowel obstruction in this patient? a. femoral hernia b. indirect inguinal hernia c. direct inguinal hernia d. spigelian hernia e. obturator hernia
a. External hernias are the second most common cause of small bowel obstruction after adhesions. A femoral hernia protrudes through the femoral ring, lying medial to the femoral vein, which may be compressed. On CT the hernia is seen inferolateral to the pubic tubercle, in contrast to inguinal hernias, which usually lie superomedial to the tubercle, though differentiation may be difficult in non-incarcerated cases. Femoral hernias are more prone to incarceration due to the inflexible margins of the femoral ring. Inguinal hernias may be classified as indirect (passing down the inguinal canal, seen lateral to the inferior epigastric vessels) or direct (protruding directly through the lower abdominal wall medial to the inferior epigastric vessels). A spigelian hernia protrudes through a defect in the inferolateral anterior abdominal wall. Obturator hernias protrude through the obturator foramen, between the pectineus and external obturator muscles.
669
A 34-year-old man presents with acute left lower quadrant pain following unaccustomed exercise. CT of the abdomen demonstrates a 2.5 cm oval lesion with attenuation value of – 60 HU abutting the sigmoid colon, with surrounding inflammatory changes. The sigmoid colon itself appears normal. What is the most likely diagnosis? a. omental infarction b. diverticulitis c. epiploic appendagitis d. liposarcoma e. appendicitis
c. Acute epiploic appendagitis is thought to result from torsion of one of the fatty epiploic appendages arising from the serosal surface of the colon. It usually occurs in young men, presenting as acute lower quadrant pain, and is associated with obesity and unaccustomed exercise. Typical CT findings are of an oval pericolonic fat density lesion of ,5 cm, with surrounding inflammatory changes, most commonly in the sigmoid, descending or right hemicolon. Right-sided epiploic appendagitis may be mistaken clinically for appendicitis. Omental infarction typically appears as a larger, heterogeneous lesion, usually affecting the caecum or ascending colon. Acute diverticulitis usually occurs in older patients, and is associated with colonic diverticula and wall thickening. Liposarcoma is rare, but is included in the differential of a fat-containing intra-abdominal mass.
670
A 65-year-old woman undergoes CT of the abdomen. An incidental finding of a well-defined 5 cm mass in the head of the pancreas is noted. It has a mean attenuation value of 5 HU, and contains multiple tiny cysts with a central nidus of calcification. There is no pancreatic duct dilatation. What is the most likely diagnosis? a. mucinous cystadenoma b. main duct intraductal papillary mucinous tumour c. serous cystadenoma d. pancreatic pseudocyst e. pancreatic insulinoma
c. Serous cystadenomas are benign neoplasia of the pancreas most commonly seen in older women and are frequently asymptomatic. Typical appearances are of a cystic lesion measuring up to 20 cm in size, containing innumerable small cysts, though these may be difficult to discriminate, giving the appearance of a solid mass. They may occur in any part of the pancreas but are slightly more common in the pancreatic head and neck. Characteristic features include a central stellate scar containing dystrophic calcification. Mucinous cystadenomas usually occur in the pancreatic tail (90%) and, when multilocular, contain larger cysts of .2 cm in diameter. Pancreatic pseudocysts are usually unilocular and occur following a history of pancreatitis. Intraductal papillary mucinous tumours of the main duct are typically associated with dilatation of the main pancreatic duct. Insulinomas are usually small (,2 cm) solid tumours that produce symptoms early due to recurrent hypoglycaemia.
671
Which venous structure divides the liver into the right and left lobes? a. right hepatic vein b. middle hepatic vein c. left hepatic vein d. left portal vein e. right portal vein
b. The functional segmental anatomy of the liver is based on the distribution of the three major hepatic veins. The middle hepatic vein divides the liver into left and right lobes. The left hepatic vein divides the left lobe into medial and lateral segments. The right hepatic vein divides the right lobe into anterior and posterior segments. In addition, the four sections are further subdivided in a transverse plane by an imaginary line drawn between the right and left portal veins. Segments run in a clockwise fashion, with segments III, IV(b), V and VI lying below the portal veins and segments VII, VIII, IV(a) and II lying above. The caudate lobe (segment I) lies posterior and to the right of the inferior vena cava.
672
A 65-year-old man presents with a several-week history of lower abdominal pain and diarrhoea. On examination he has tenderness and guarding in the left lower quadrant. On contrast-enhanced CT, the inferior mesenteric vein is dilated, with a thin rim of enhancement around a central area of low density. What is the most likely additional pathology demonstrated on the CT? a. sigmoid diverticulitis b. appendicitis c. Crohn’s disease d. pancreatitis e. caecal malignancy
a. The inferior mesenteric vein provides venous drainage for the rectum, sigmoid and descending colon, and is a potential route of spread of neoplastic and inflammatory conditions. Inferior mesenteric vein thrombosis may occur secondary to an inflammatory process, most commonly diverticulitis, or malignancy. Other potential causes include hypercoagulable states, surgery, trauma and bowel obstruction. Appearances are of an enlarged vein with rim enhancement surrounding central low-density thrombus. Superior mesenteric vein thrombosis is much more common (95% of mesenteric venous thrombosis) and may follow an inflammatory or neoplastic process affecting the small intestine, caecum, and ascending and transverse colon.
673
A previously well, 28-year-old man recently returned from the Far East becomes acutely unwell with fever and right upper quadrant pain. Ultrasound scan demonstrates a well-defined, rounded, 7 cm hypoechoic lesion in the right lobe of the liver contiguous with the liver capsule, with fine homogeneous, low-level internal echoes and acoustic enhancement. What is the most likely diagnosis? a. pyogenic abscess b. amoebic abscess c. fungal abscess d. hydatid disease e. incidental simple hepatic cyst
b. Pyogenic abscesses are the commonest type of liver abscess in developed countries, and are most frequently due to ascending cholangitis from benign or malignant obstructive biliary disease. They are often poorly defined with irregular walls on ultrasound scan, and may contain debris or demonstrate intense hyperechogenicity when containing gas. Amoebic abscesses tend to occur in younger, more acutely unwell patients from high prevalence areas or with a history of recent travel. They are treated medically whereas pyogenic abscesses usually require percutaneous or surgical drainage. Fungal abscesses are usually multiple and occur in immunosuppressed individuals. Hydatid disease tends to be asymptomatic or present with biliary colic. Characteristic ultrasound scan features include daughter cysts and detachment of the endocyst, giving rise to ‘floating membranes’ within the cyst cavity.
674
A 64-year-old woman presents to the dermatologist with erythema- tous maculopapular lesions on her legs, buttocks and face, and is diagnosed with necrolytic migratory erythema. Which initial imaging investigation is most appropriate? a. no imaging b. mammography c. CT of the brain d. chest radiograph e. CT of the abdomen
e. Necrolytic migratory erythema is a rare dermatological condition with a strong association with glucagonoma, an islet-cell tumour of the pancreas derived from alpha cells. Over 70% of patients with glucagonoma demonstrate the condition, and they may also complain of weight loss, diarrhoea and diabetes. The association is considered strong enough to warrant thorough investigation for pancreatic malignancy. Glucagonomas typically occur in the pancreatic body or tail, and are large (2.5 – 25 cm) hypervascular tumours with solid and necrotic components. They have a high rate of malignant transformation, and around 50% of patients have liver metastases at the time of diagnosis.
675
A 70-year-old man with a history of several months of dysphagia undergoes double-contrast barium swallow. This demonstrates a moderately dilated oesophagus with reduced peristalsis and smooth tapering of the distal oesophagus. What is the most likely diagnosis? a. primary achalasia b. gastric carcinoma c. scleroderma d. oesophageal carcinoma e. presbyoesophagus
b. Primary achalasia is an abnormality of the myenteric plexus resulting in reduced or absent peristalsis and failure of relaxation of the lower oesophageal sphincter. The oesophagus is typically markedly dilated with absent primary peristalsis and a smooth tapered narrowing at the contracted lower oesophageal sphincter. It usually presents in young adults with long-standing dysphagia. In contrast, secondary achalasia due to malignancy usually presents in older patients with a short duration of dysphagia. Decreased peristalsis and distal oesophageal tapering in these patients result from tumour infiltration of the myenteric plexus of the distal oesophagus by gastric carcinoma, lymphoma or metastatic disease. Distal oesophageal carcinoma tends to give rise to irregular, asymmetrical narrowing. Scleroderma typically appears as a dilated oesophagus with a patulous lower oesophageal sphincter. Presbyoesophagus is a disorder of oesophageal motility, characterized by oesophageal dilatation and repetitive, non-peristaltic, tertiary contractions in the distal oesophagus.
676
A 25-year-old woman presents with cramping abdominal pain and bleeding per rectum. On examination she has mucocutaneous pigmentation of her mucous membranes and face. A small-bowel follow-through examination demonstrates small-bowel intussus- ception. Which other finding is most likely to be demonstrated? a. separation and displacement of small bowel loops b. localized outpouching of the antimesenteric border of the distal ileum c. generalized irregular fold thickening d. multiple filling defects in the small bowel e. generalized dilatation of the small bowel
d. Peutz –Jegher syndrome is characterized by multiple benign hamartomatous intestinal polyps and mucocutaneous pigmentation. It is familial in 50% of cases, with an autosomal dominant inheritance, and sporadic in 50%. It is the most common polyposis syndrome to involve the small intestine, and frequently presents with intussusception. Typical findings are of multiple hamartomatous polyps in the small bowel, and less commonly the colon and stomach. Patients are at increased risk of gastrointestinal malignancy, but also of tumours of the pancreas, breast, ovary, endometrium and testis.
677
A 65-year-old woman, with a history of previous partial gastrect- omy 10 years earlier, presents with upper abdominal pain and early satiety. She undergoes a double-contrast barium meal, which demonstrates a 4 cm intraluminal, mottled filling defect in the gastric remnant with no fixed attachment to the gastric wall. What is the most likely diagnosis? a. suture granuloma b. trichobezoar c. phytobezoar d. gastric carcinoma e. villous adenoma
c. Bezoars are masses of accumulated ingested material forming in the stomach or intestines. Phytobezoars are the commonest type, composed of poorly digested fibre and vegetable matter. They are seen particularly in patients with previous gastric surgery, probably due to diminished gastric emptying. Patients may be asymptomatic or present with early satiety or symptoms of gastritis, as phytobezoars are irritant. Occasionally, they may obstruct the stomach with a ball – valve mechanism. They are seen as relatively mobile filling defects, the interstices of which are filled with barium. Trichobezoars are composed of hair, and are usually larger, and found in younger patients, particularly those with a psychiatric history. Gastric carcinoma, villous adenoma and suture granuloma are all causes of gastric filling defects but have a constant relationship to the gastric wall.
678
A 23-year-old man with dysphagia undergoes a double-contrast barium swallow, which demonstrates a smooth, well-defined, 12 cm submucosal lesion in the distal oesophagus causing deformity of the lumen. CT demonstrates coarse calcification within the mass. What is the most likely diagnosis? a. oesophageal lipoma b. oesophageal duplication cyst c. oesophageal carcinoma d. oesophageal varices e. oesophageal leiomyoma
e. Leiomyomas are benign tumours of smooth muscle, and represent the most common benign neoplasm of the oesophagus. They are often asymptomatic but may present with dysphagia and rarely haematemesis. They appear on barium swallow as large, well-defined, intramural masses causing luminal deformity. A characteristic finding is of coarse calcifications – leiomyoma is the only calcifying oesophageal tumour. Oesophageal lipomas and duplication cysts also appear as well-defined submucosal lesions (of fat and of water density respectively on CT), but are less common, and internal calcification is not a feature. Oesophageal carcinoma usually appears as an irregular ulcerated stricture. Oesophageal varices are seen as serpiginous filling defects.
679
A 54-year-old man with known metastatic malignant melanoma presents with epigastric pain and haematemesis. What is the most likely finding in the stomach on double-contrast barium meal? a. multiple submucosal nodules with central ulceration b. solitary ulcerated mass in the gastric antrum c. linitis plastica d. solitary, well-defined, pedunculated filling defect e. thickened tortuous gastric folds
a. GI tract metastases are seen in 4 – 8% of patients with malignant melanoma. The small intestine is most commonly affected, followed by the colon and stomach. Typical features are of multiple submucosal nodules, with a target appearance due to central ulceration. This appearance is particularly seen with malignant melanoma metastases but may also be seen with gastric metastases from breast, lung and renal cell carcinoma. Other common appearances of gastric metastases include linitis plastica in 20%, most typically from breast cancer, and a solitary mass in 50%.
680
A 20-year-old woman with anorexia nervosa presents with intermittent abdominal pain and vomiting relieved by lying prone. Barium meal examination reveals a vertical band-like narrowing of the third part of the duodenum, with proximal duodenal dilata- tion and vigorous to-and-fro peristalsis. What is the most likely diagnosis? a. duodenal duplication cyst b. annular pancreas c. Ladd’s bands d. superior mesenteric artery syndrome e. duodenal atresia
d. The third part of the duodenum is bounded posteriorly by the aorta, and anteriorly by the root of the mesentery carrying the superior mesenteric artery. In superior mesenteric artery syndrome, the third part of the duodenum is compressed by the superior mesenteric artery, and the angle between it and the aorta narrows to 10 – 22˚ (normal 45 –65˚). The condition is associated with severe weight loss, prolonged bedrest (particularly in a body cast), lumbar lordosis and pregnancy. Patients report intermittent abdominal pain and vomiting, relieved by lying prone or in the knee – elbow position. Duodenal atresia causes complete obstruction usually distal to the ampulla of Vater, and presents in neonates with a ‘double-bubble’ sign on plain abdominal radiograph. Annular pancreas is usually asymptomatic but may present with abdominal pain and vomiting, and barium meal demonstrates narrowing of the second part of the duodenum. Duodenal duplication cysts cause extrinsic compression of the first and second portions of the duodenum. Ladd’s bands are congenital peritoneal bands occurring in association with malrotation that may cause obstruction of the second part of the duodenum, but presentation is usually in infants and children.
681
A 46-year-old man presents with severe abdominal pain. An erect chest radiograph shows free intraperitoneal air below the diaphragm. What is the most likely cause? a. perforated anterior wall duodenal ulcer b. perforated posterior wall duodenal ulcer c. perforated gastric ulcer d. perforated appendix e. diverticulitis with perforation
a. The commonest cause of a free intraperitoneal perforation is an anterior wall duodenal ulcer. However, free peritoneal air is only apparent on an erect chest radiograph in 60% of perforated duodenal ulcers. Possible causes include sealing of the perforation, adhesions preventing the gas reaching the subphrenic space or insufficient time being allowed for gas to collect under the diaphragm. Anterior wall gastric ulcers also perforate into the peritoneal cavity but are a less common cause of pneumoperitoneum. Posterior wall duodenal and gastric ulcers perforate into the lesser sac or retroperitoneal region rather than the peritoneal cavity. Perforated appendix and diverticulitis may produce localized collections of extraluminal gas, but pneumoperitoneum is rare.
682
A 78-year-old man presents with abdominal pain. A plain abdomi- nal radiograph demonstrates a distended, inverted U-shaped loop of bowel devoid of haustra, extending from the left iliac fossa infer- iorly to just beneath the left hemidiaphragm superiorly. What is the most likely diagnosis? a. caecal volvulus b. sigmoid volvulus c. paralytic ileus d. large bowel obstruction due to distal malignancy e. small bowel malrotation and volvulus
b. Sigmoid volvulus usually occurs when the sigmoid loop twists around its mesenteric axis, creating a closed loop obstruction. Typical features are of an inverted U-shaped loop converging on the left side of the pelvis. The bowel loop is usually markedly distended, appears ahaustral, and may overlap the lower border of the liver (liver overlap sign) or the haustrated dilated descending colon (left flank overlap sign). The apex of the volvulus usually lies under the left hemidiaphram with its apex above the level of T10. Caecal volvulus occurs when the caecum is on a mesentery, and involves the caecum either twisting and inverting so the caecal pole lies in the left upper quadrant, or twisting in an axial plane so that the caecum remains right sided or central. Appearances are of a large, gas-distended viscus, usually with haustral markings, and occasionally the gas-filled appendix may be identified.
683
A 34-year-old woman presents with bloody diarrhoea and abdom- inal pain. Which feature on barium enema favours a diagnosis of ulcerative colitis rather than Crohn’s disease? a. thickened ileocaecal valve b. circumferential wall involvement c. fistula formation d. skip lesions e. normal rectum
b. Ulcerative colitis and Crohn’s disease are idiopathic inflammatory diseases of the bowel. Ulcerative colitis predominantly involves the mucosa and submucosa, and characteristically produces continuous, circumferential involvement of the colon. Crohn’s disease produces transmural inflammation, may affect the entire gastrointestinal tract, and is characterized by eccentric and discontinuous involvement. Typical features of ulcerative colitis include predominantly left-sided colonic involvement with rectosigmoid involvement in 95% of cases, a patulous ileocaecal valve and shallow ulceration. Typical features of Crohn’s disease include skip lesions (discontinuous disease), terminal ileal involvement with a thickened ileocaecal valve and fistula formation.
684
A 45-year-old woman with pleuritic chest pain and breathlessness undergoes CT pulmonary angiogram for suspected acute pulmonary embolism, which demonstrates multiple irregular areas of relatively poor enhancement in the visualized portion of the spleen. What is the most likely cause? a. normal arterial-phase enhancement b. splenic infarction c. splenic clefts d. splenosis e. spontaneous splenic rupture
a. A CT pulmonary angiogram is performed during pulmonary arterial phase enhancement. During arterial phase enhancement, variable rates of flow through the splenic parenchyma result in heterogeneous enhancement, which may appear as alternating bands of high and low attenuation, or give the impression of irregular, low-density mass lesions. Enhancement becomes homogeneous in the portal venous phase. Splenic infarction is the most common cause of (true) focal splenic defects, and typically appears as single or multiple, wedge- shaped, peripheral, low-attenuation defects. Clefts in the splenic contour are common normal variants, appearing as smoothly contoured, medially located defects, and should not be mistaken for lacerations. Splenosis is the implantation of splenic tissue in ectopic sites following traumatic rupture or splenectomy, and appears as multiple, small, homogeneous, enhancing masses that may mimic peritoneal deposits. Spontaneous splenic rupture is rare, though it may be delayed following trauma or be associated with splenomegaly. Appearances may include low-density, linear, parenchymal lacerations and areas of mottled parenchymal enhancement representing contusions.
685
A 26-year-old man known to have AIDS presents with a 2-week history of difficult and painful swallowing. He undergoes double-contrast barium examination of the oesophagus, which demonstrates multiple, small, superficial, round ulcers in the mid- oesophagus. The intervening mucosa is normal and no plaques are seen. What is the most likely diagnosis? a. HIV oesophagitis b. cytomegalovirus oesophagitis c. reflux oesophagitis d. candida oesophagitis e. herpes simplex oesophagitis
e. Candida oesophagitis is the commonest cause of infectious oesophagitis and is particularly seen in immunosuppressed individuals. It is frequently associated with oral thrush. It tends to affect the upper half of the oesophagus, and typical appearances are of linear, longitudinally oriented filling defects representing heaped-up areas of mucosal plaques consisting of necrotic debris and fungal colonies. In contrast, a normal intervening mucosa in oesophagitis is suggestive of a viral aetiology. Findings in cytomegalovirus and HIV oesophagitis are similar, with typical appearances of one or more large flat ulcers seen in the distal oesophagus. Distinction between the two is made by brushings or biopsy at endoscopy. In herpes simplex infection, the typical features of multiple, small, superficial ulcers are similar at all sites of potential involvement, including the oesophagus, oral cavity, rectum and anus.
686
A 45-year-old man presents with dysphagia and undergoes a double-contrast barium swallow. This demonstrates a smooth oblique indentation on the posterior wall of the oesophagus. What is the most likely cause of these appearances? a. enlarged left atrium b. aberrant right subclavian artery c. aberrant left pulmonary artery d. right-sided aortic arch e. coarctation of the aorta
b. A number of anomalies of the major vessels can cause extrinsic impressions upon the oesophagus. The commonest aortic anomaly is a right-sided aortic arch, which produces an indentation on the right lateral oesophageal wall in the absence of the normal left aortic arch impression. An aberrant right subclavian artery originates from the aortic arch just distal to the left subclavian artery, and passes upwards and to the right, behind the oesophagus, giving rise to an oblique posterior oesophageal indentation. In aortic coarctation, the pre- and post-stenotic dilatations of the aorta produce a characteristic reversed-3 impression upon the left wall of the oesophagus. An enlarged left atrium and an aberrant left pulmonary artery both cause anterior indentations upon the oesophagus.
687
A 30-year-old woman on the oral contraceptive pill undergoes unenhanced CT of the abdomen, which demonstrates a well- circumscribed, slightly hypoattenuating mass in the liver. Which additional radiological finding would favour a diagnosis of hepatic adenoma rather than focal nodular hyperplasia? a. measured lesion size of 3 cm b. accompanying acute subcapsular haematoma c. transient arterial-phase enhancement d. normal uptake on 99mTc-labelled sulphur colloid scan e. hypodense central stellate scar
b. Focal nodular hyperplasia (FNH) is a benign hamartomatous malformation commonest in young women. Lesions are usually smaller than 5cm, and contain a central stellate scar in up to one-third of cases. Hepatic adenomas are benign tumours averaging 8 – 10 cm in size, seen predominantly in young women and related to oral contraceptive use. Lesions have a propensity for spontaneous haemorrhage, presenting as subcapsular haematoma or haemoperitoneum. FNH, though highly vascular, rarely undergoes spontaneous haemorrhage. FNH usually contains sufficient functioning Kupffer cells to demonstrate normal or increased uptake on 99mTc-labelled sulphur colloid scan, whereas hepatic adenoma, composed of hepatocytes and non-functioning Kupffer cells, appears as a focal photopenic lesion. Both lesions demonstrate transient arterial enhancement following intravenous contrast.
688
A 25-year-old man presents with jaundice and malaise. Ultrasound scan demonstrates a general decrease in liver echogenicity and a well-distended gallbladder with a wall thickness of 4 mm. No gall- stones are seen and the intra- and extrahepatic bile ducts appear normal. What is the most likely diagnosis? a. acute cholecystitis b. cirrhosis c. fatty liver d. acute viral hepatitis e. primary sclerosing cholangitis
d. Acute hepatitis results in a diffuse decrease in liver echogenicity on ultrasound scan, with increased brightness of the portal triads resulting in a ‘starry sky’ appearance. Other imaging features include oedema of the gallbladder fossa and gallbladder wall thickening. Gallbladder wall thickening (anterior wall thickness .3 mm in a non-contracted gallbladder) may be seen in a wide range of intrinsic and extrinsic conditions. The commonest intrinsic cause is cholecystitis (acute and chronic), whereas common extrinsic causes include hepatitis, hypoalbuminaemia, heart failure and renal failure.
689
A 74-year-old man presents with severe abdominal pain and is admitted under the surgical team with suspected perforation. He is too unwell to undergo an erect chest radiograph. What is the most appropriate alternative plain film to detect the presence of free intraperitoneal gas? a. supine chest b. erect abdomen c. supine abdomen d. left lateral decubitus abdomen e. right lateral decubitus abdomen
d. An erect chest radiograph is best for demonstrating a small pneumoperitoneum, enabling the identification of as little as 1 ml of free intraperitoneal gas. It is superior to an erect abdomen, as the X-ray beam penetrates the diaphragmatic region almost tangentially, whereas, in an erect abdomen, the divergent beam penetrates this area obliquely. However, if the patient is too unwell to sit erect, the most appropriate projection is a left lateral decubitus abdominal radiograph, performed with the patient lying on the left side, using a horizontal X-ray beam. In this position, air will preferentially leave a perforated duodenal or antral ulcer, and any free gas in the lesser sac will enter the main abdominal cavity. The supine abdominal radiograph may demonstrate free gas in about 56% of patients with pneumoperitoneum. Characteristic features include gas outlining both inner and outer walls of a bowel loop (Rigler’s sign), triangular collections of gas between bowel loops, and outlining of the falciform ligament and other peritoneal reflections by free gas. The supine chest radiograph is not useful in the detection of free intraperitoneal gas.
690
The gastroduodenal artery normally has its origin from which vessel? a. superior mesenteric artery b. common hepatic artery c. left gastric artery d. coeliac axis e. aorta
b. In 75% of cases, the gastroduodenal artery arises from the common hepatic artery before its division into right and left branches. Less common origins include the left hepatic artery (4 – 11%), the right hepatic artery (7%) and the superior mesenteric artery via a replaced hepatic trunk (4 – 11%). The artery descends behind the first part of the duodenum, lying anterior to the pancreas and to the left of the common bile duct. At this site, erosion of the posterior duodenal wall by an ulcer may produce life-threatening haemorrhage if the gastroduodenal artery is involved. Its main branches are the posterior and anterior superior pancreaticoduodenal arteries, and the right gastroepiploic artery.
691
A 45-year-old patient with cirrhosis is found to have a focal liver lesion on ultrasound scan, clinically suspected to be hepatocellular carcinoma. What would be the expected appearances of the lesion on T2W MR images following infusion of superparamagnetic iron oxide particles? a. increased signal intensity compared with rest of liver b. decreased signal intensity compared with rest of liver c. lesion signal intensity unchanged; rest of liver increased signal intensity d. lesion signal intensity unchanged; rest of liver decreased signal intensity e. no effect on appearances on T2W images
d. Superparamagnetic iron oxide (SPIO) particles are iron-based particles of 30 – 150 nm, which, when administered as an infusion 1 – 4 hours prior to imaging, act as a negative MR contrast agent. They target the reticuloendothelial system, being taken up by macrophages throughout the body, but are preferentially accumulated by the Kupffer cells of the liver. Their superparamagnetic properties result in T2 and T2* shortening of the tissues that accumulate the particles, which show reduced signal intensity on T2W, T2 W and, to a lesser extent, T1W images. Most liver tumours do not exhibit uptake, as they are deficient in Kupffer cells. However, as the rest of the liver accumulates SPIO and darkens preferentially, the tumour appears of increased conspicuity. SPIO particles are particularly used, in combination with gadolinium, to improve detection of hepatocellular carcinoma in cirrhotic patients, in whom the parenchymal changes of fibrosis and regenerative nodules make detection with gadolinium alone difficult.
692
A 35-year-old woman on the oral contraceptive pill presents with right upper quadrant pain, shortness of breath and leg oedema. Ultrasound scan of the abdomen demonstrates hepatosplenomegaly and ascites. The hepatic veins are not visualized on Doppler ultrasound scan. What is the most likely diagnosis? a. acute Budd – Chiari syndrome b. primary biliary cirrhosis c. passive hepatic congestion d. hepatic veno-occlusive disease e. viral hepatitis
a. Budd – Chiari syndrome is caused by obstruction of hepatic venous outflow, which may, in turn, be caused by membranous obstruction of the suprahepatic IVC by a congenital web, or hepatic venous thrombosis due to hypercoagulable state, tumour or trauma. Patients develop hepatosplenomegaly and intractable ascites. Doppler ultrasound scan demonstrates non-visualization of, or thrombus within, one or more hepatic veins. CT findings reflect severely impaired blood flow to the liver, with a ‘flip-flop’ enhancement pattern after contrast administration. Early images show prominent central liver enhancement with poor peripheral enhancement, whereas delayed images show central washout with peripheral enhancement. The caudate lobe is typically spared because of its separate venous drainage directly into the IVC, and enhances normally. Passive hepatic congestion complicates heart failure, and results in distended hepatic veins and IVC. Hepatic veno-occlusive disease refers to occlusion of small centrilobular hepatic veins following radio- and chemotherapy in bone-marrow transplant recipients, or related to alkaloid consumption. The main hepatic veins and IVC are normal. Hepatic venous involvement is not a feature of viral hepatitis or primary biliary cirrhosis.
693
A 40-year-old woman undergoes abdominal ultrasound scan, which demonstrates three small, rounded, echogenic structures in relation to the anterior wall of the gallbladder. There is no posterior acoustic shadowing, and appearances remain constant with variation in patient position. The remainder of the gallbladder and biliary tree appear unremarkable. What is the most likely diagnosis? a. gallstones b. cholesterol polyps c. adenomyomatosis d. gallbladder carcinoma e. strawberry gallbladder
b. Cholesterolosis is a form of hyperplastic cholecystosis in which triglycerides, cholesterol precursors and cholesterol esters accumulate within the lamina propria of the gallbladder wall. Most cases are of the planar type, termed ‘strawberry gallbladder’ after the resemblance of the gallbladder mucosa to the surface of a strawberry, and produce no detectable ultrasound changes. In a minority of cases, cholesterol polyps are formed, which are the commonest type of gallbladder polyp. They are generally small, multiple echogenic lesions adjacent to the gallbladder wall. Non-mobility and a lack of posterior acoustic shadowing helps to distinguish polyps from gallstones. Small size and multiplicity distinguishes them from gallbladder malignancy, though, rarely, metastatic disease may produce multiple polypoid lesions, particularly malignant melanoma. Adenomyomatosis, the other form of hyperplastic cholecystosis, results in mucosal hyperplasia and thickening of the muscular layer of the gallbladder. It is characterized by bright reflections and comet-tail artefacts from the gallbladder wall on ultrasound scan.
694
A 51-year-old man with alcoholic cirrhosis presents with jaundice. CT of the abdomen reveals an encapsulated, 20 mm focal area of low density in the liver, which demonstrates arterial-phase enhance- ment and rapid washout on delayed imaging. What is the most likely diagnosis? a. regenerative nodule b. dysplastic nodule c. hepatocellular carcinoma d. hepatic haemangioma e. focal fatty sparing
c. Nodules are a common finding in cirrhosis, and differentiation of benign nodules from hepatocellular carcinoma (occurring in 7 – 12% of patients) is vital. Most nodules are regenerative nodules, representing reparative attempts by hepatocytes in response to liver injury. These are typically under 10 mm in size and appear isodense to liver parenchyma on CT, unless they contain iron deposits (siderotic nodules), in which case they may be slightly hyperdense. Dysplastic nodules are proliferative premalignant lesions found in 15 – 25% of cirrhotic livers. They resemble regenerative nodules on CT but are usually larger than 10 mm. Hepatocellular carcinomas usually appear as encapsulated hypodense masses that demonstrate rapid arterial enhancement and early washout of contrast on delayed images. Hepatic haemangioma usually appears as a low-density mass, but has different enhancement characteristics, demonstrating peripheral enhancement with complete fill-in on delayed images. Focal fatty sparing appears as an area of normal density in a generally hypodense liver and does not demonstrate contrast enhancement.
695
A 36-year-old man with ulcerative colitis develops progressive jaundice and pruritis. CT of the abdomen demonstrates multiple areas of dilatation and stenosis of tortuous intrahepatic bile ducts, with wall thickening and contrast enhancement of the extrahepatic bile ducts. What is the most likely diagnosis? a. primary sclerosing cholangitis b. choledocholithiasis c. primary biliary cirrhosis d. ascending cholangitis e. chronic pancreatitis
a. Primary sclerosing cholangitis is an idiopathic, progressive, fibrosing, inflammatory disorder of the biliary tree, causing multifocal strictures, cholestasis and biliary cirrhosis. There is an association with inflammatory bowel disease and autoimmune conditions. In most cases, both intra- and extrahepatic ducts are involved, and classic appearances on cholangiography are of a ‘string-of-beads’ appearance with alternating segments of dilatation and stenosis. Biliary cirrhosis develops in up to 49% of cases, and there is an increased risk of cholangiocarcinoma. In primary biliary cirrhosis, disease is limited to the intrahepatic bile ducts. In ascending cholangitis, there may be biliary dilatation and pneumobilia, but multifocal strictures are not a feature. CT features of choledocholithiasis include biliary dilatation and visualization of a stone in the bile duct, but again strictures are not a feature. Chronic pancreatitis may result in a smooth inflammatory stricture of the intrapancreatic portion of the common bile duct.
696
A 73-year-old man presents with lower abdominal pain and a change in bowel habit. A contrast enema demonstrates a stricture in the sigmoid colon. Which feature would favour a diagnosis of colorectal carcinoma rather than diverticulitis? a. long (.10 cm) segment of involvement b. mucosal ulceration c. presence of a colovesical fistula d. multiple diverticula in the sigmoid colon e. smoothly tapered stricture margins
b. The differentiation between complicated diverticular disease and a perforating colorectal carcinoma may be difficult. Both may appear as a focal area of eccentric luminal narrowing on contrast enema. A longer segment of involvement favours diverticulitis, as well as other inflammatory causes of colitis. In addition, stricture margins in diverticulitis tend to be smoothly tapered rather than the abrupt narrowing of carcinoma. Mucosal ulceration occurs in most cases of colorectal carcinoma, and is not a particular feature of diverticulitis. Fistula formation may occur in both conditions, most commonly between the colon and the bladder, though this is more commonly seen in diverticulitis. The presence of diverticula does not exclude the possibility of colorectal carcinoma, and the two conditions may coexist, as both are common in elderly people.
697
A 45-year-old man undergoes barium swallow for dysphagia, which demonstrates multiple flask-shaped outpouchings of barium arranged in longitudinal rows paralleling the long axis of the oesophagus. Which of the following is a commonly associated condition? a. scleroderma b. rheumatoid arthritis c. chronic obstructive airway disease d. AIDS e. diabetes
e. Oesophageal intramural pseudodiverticulosis is a condition causing dilatation of the ducts of the submucosal glands of the oesophagus. These appear on barium meal as multiple, tiny, flask-shaped collections of barium arranged in longitudinal rows. They may appear to ‘float’ outside the oesophagus, as the connection to the lumen may not be appreciated. Associated strictures in the distal oesophagus are common. The condition is commonly associated with diabetes and chronic alcoholism, but may also occur with severe oesophagitis of any cause. Candida may be cultured in around half the cases, but this may be a secondary infection due to stasis of secretions within the glands.
698
What is the most common cause of varices affecting the upper third of the oesophagus? a. portal hypertension due to cirrhosis b. splenic vein thrombosis c. inferior vena caval obstruction d. superior vena caval obstruction e. hepatic vein obstruction
d. Oesophageal varices are dilated submucosal veins, which may be classified by their direction of flow as uphill or downhill varices. Uphill varices occur in the lower oesophagus and represent collateral blood flow conveying portal venous blood to the azygos vein. They usually result from portal hypertension due to liver cirrhosis, but may also occur with splenic vein thrombosis, and obstruction of the hepatic veins or IVC. Downhill varices result from obstruction of the SVC. If it is obstructed superior to the entry of the azygos vein, varices will be confined to the upper third of the oesophagus. If the SVC is obstructed below the entry of the azygos vein, the varices convey all of the systemic venous blood from the upper half of the body into the portal vein and IVC, and they will run the entire length of the oesophagus. SVC obstruction is most commonly due to lung cancer or lymphoma.
699
A 66-year-old woman with a known large para-oesophageal hiatus hernia presents with sudden onset of severe epigastric pain and vigorous retching without production of vomitus. Passage of a nasogastric tube is unsuccessful. Plain abdominal radiograph demonstrates a markedly distended stomach in the left upper quadrant extending into the chest. What is the most likely diagnosis? a. pyloric stenosis b. ‘cup-and-spill’ stomach c. acute gastric volvulus d. acute gastric dilatation e. paraduodenal hernia
c. Acute gastric volvulus is abnormal rotation of one part of the stomach around another part, which may be classified as organoaxial, mesenteroaxial or combination type, depending on the axis of rotation. Predisposing factors include ligamentous laxity, hiatus hernia and diaphragmatic eventration. The classic presentation is with the Borchardt triad of sudden severe epigastric pain, intractable retching with no vomitus produced, and inability to pass a nasogastric tube into the stomach. Other plain film findings include unexpected location of the gastric bubble and air – fluid levels in the mediastinum or upper abdomen, but definitive diagnosis is by barium meal. The condition is a surgical emergency, as it may result in gastric ischaemia or perforation. Acute gastric dilatation and pyloric stenosis may result in gastric distension on plain film, but would not present with intractable retching or difficulty with nasogastric tube passage. A ‘cup-and-spill’ stomach is an anatomical variant on barium meal, which may simulate an organoaxial volvulus. A paraduodenal hernia usually presents acutely as small bowel obstruction.
700
A 60-year-old woman presents with weight loss and diarrhoea. CT of the abdomen demonstrates multiple, enlarged, low-attenuation mesenteric lymph nodes containing fat – fluid levels and splenic atrophy. What is the most likely diagnosis? a. tuberculosis b. coeliac disease c. Whipple’s disease d. lymphoma e. metastatic squamous cell carcinoma
b. Cavitating mesenteric lymph node syndrome is a rare complication of coeliac disease, in which multiple enlarged lymph nodes are seen in the jejunoileal mesentery. The nodes have central low attenuation and may contain fat or fluid, or fat – fluid levels. Splenic atrophy is usually seen, and jejunal or duodenal biopsy confirms villous atrophy of the small bowel mucosa. Low-attenuation lymphadenopathy may also be seen in tuberculosis, Whipple’s disease, lymphoma and necrotic metastases, but fat – fluid levels have been reported only in coeliac disease.
701
A 27-year-old woman presents with upper abdominal pain and is found to have a palpable right upper quadrant mass on examin- ation. CT demonstrates a low attenuation lesion in the right lobe of the liver. Which imaging feature would favour a diagnosis of fibrolamellar carcinoma of the liver rather than focal nodular hyperplasia? a. calcifications within a central scar b. lesion size of 3 cm c. multiple lesions d. increased uptake on sulphur colloid scan e. central scar hyperintense on T2W images
a. Fibrolamellar carcinoma of the liver is an uncommon variant of hepatocellular carcinoma, typically presenting as a large, 5 – 20 cm liver mass in a young patient with no risk factors. Typical features are of an encapsulated mass with a prominent central fibrous scar. The scar often contains areas of calcification, and appears hypointense on T1- and T2-weighted images. Focal nodular hyperplasia (FNH) is a hamartomatous malformation also most commonly seen in young woman. However, lesion size is usually ,5 cm and, although a central fibrous scar is also a common feature, this usually appears hyperintense on T2-weighted images due to vascular channels and oedema, and calcifications within it are extremely rare. Both pathologies may result in multiple lesions, with FNH being multiple in 20% and fibrolamellar carcinoma demonstrating satellite lesions in 10 – 15%. FNH is the only liver lesion with sufficient Kupffer cells to cause normal or increased uptake on sulphur colloid scan.
702
In patients with cystic fibrosis, which gastrointestinal pathology may occur as a result of high-dose lipase supplementation? a. rectal prolapse b. fibrosing colonopathy c. pneumatosis intestinalis d. gastro-oesophageal reflux e. meconium ileus equivalent syndrome
b. Fibrosing colonopathy is a condition causing progressive submucosal fibrosis predominantly affecting the proximal colon. It was first described in 1994 in children with cystic fibrosis taking high-dose lipase supplementation to relieve the symptoms of exocrine pancreatic insufficiency. It causes stricturing and longitudinal shortening of the right colon, and patients present with obstruction. Overall, the gastrointestinal tract is affected in 85 – 90% of patients with cystic fibrosis, and all of the above pathologies may occur, though only fibrosing colonopathy is associated with high-dose lipase supplementation.
703
A 45-year-old man undergoes ultrasound scan of the abdomen 2 days following orthotopic liver transplantation, which demon- strates periportal oedema and a small fluid collection at the hilum of the liver. What is the most likely diagnosis? a. graft rejection b. hepatic arterial thrombosis c. portal vein stenosis d. bile leak e. normal post-transplantation findings
e. Orthotopic liver transplantation is the treatment of choice for patients with end-stage liver disease for which no other therapy is available. Surgery involves one arterial anastomosis (hepatic artery), at least two venous anastomoses (portal vein and IVC) and a biliary anastomosis, and complications may occur at any of these sites. Vascular complications are the most frequent cause of graft loss, and most commonly involve the hepatic artery, with portal venous and IVC complications being relatively infrequent. Biliary complications occur in up to 34% of cases, and are the second most common cause of liver dysfunction after graft rejection. They include leak, stricture and obstruction. Other complications include fluid collections, infection and malignancy. Normal findings following liver transplantation include a small amount of free intraperitoneal fluid in the perihepatic region, especially at the hilum, or in the fissure for the ligamentum teres, which usually resolves within a few weeks. Other normal findings are a right fluid pleural effusion, and periportal oedema, attributed to lymphatic channel dilatation due to lack of normal lymphatic drainage.
704
A transjugular intrahepatic portosystemic shunt lies between the portal vein and which vessel? a. hepatic vein b. inferior vena cava c. aorta d. common hepatic artery e. left gastric vein
a. A transjugular intrahepatic portosystemic shunt (TIPSS) is an endovascular procedure performed to create a portosystemic shunt between the portal venous and hepatic venous systems, for decompression of portal hypertension, particularly in patients with variceal bleeding uncontrollable by endoscopic management. The shunt is normally formed between the right hepatic vein and right portal vein. The right internal jugular vein is accessed, and via this the right hepatic vein. Curved needle passes are made in an anterior direction to access the right portal vein. The portosystemic pressure gradient is measured and portal venography is performed to enable planning of stent placement. A stent is deployed following balloon dilatation of the stent tract. The goal is to decrease the portosystemic gradient to below 12 mmHg and to see no significant filling of varices. Portosystemic shunts require close follow-up, as there is a high incidence of shunt stenosis and occlusion.
705
In a 67-year-old female patient with jaundice and gallbladder wall thickening on ultrasound scan, which feature on CT favours a diagnosis of xanthogranulomatous cholecystitis rather than gallbladder carcinoma? a. pericholecystic fat infiltration b. intramural hypoattenuating nodules throughout the gallbladder c. biliary obstruction d. hepatic extension e. regional lymphadenopathy
b. Xanthogranulomatous cholecystitis (XGC) is an uncommon inflammatory disease of the gallbladder, which is characterized by multiple intramural nodules and proliferative fibrosis. It is thought to result from rupture and extravasation of bile and mucus following the occlusion of Rokitansky – Aschoff sinuses. There is considerable overlap between the clinical and radiological features of XGC and gallbladder carcinoma. Pericholecystic infiltration, biliary obstruction, regional lymphadenopathy and hepatic involvement may be seen in both conditions, and the difference in incidence between the two conditions is not statistically significant. Only the presence of multiple intramural hypoattenuating nodules (representing xanthogranulomas) occupying a large area of the thickened gallbladder wall allows a diagnosis of XGC to be made with any degree of certainty. Similar hypoattenuating intramural nodules (representing haemorrhage and necrosis) may be seen less commonly in gallbladder carcinoma, but these tend to occupy a much smaller proportion of the thickened gallbladder wall.
706
A 40-year-old man with hyperpigmentation, arthalgia and diabetes mellitus is clinically suspected to have primary haemochromatosis. What are the most likely findings on liver MRI in this condition? a. normal appearances of the liver b. decreased signal intensity on T1W and T2W images c. decreased signal intensity on T1W and increased signal intensity on T2W images d. increased signal intensity on T1W and decreased signal intensity on T2W images e. increased signal intensity on T1W and T2W images
b. In primary haemochromatosis, there is increased duodenal absorption and parenchymal retention of dietary iron, which is accumulated within the liver, pancreas, heart and pituitary gland. Intracellular iron deposits within hepatocytes result in a paramagnetic susceptibility effect, leading to marked shortening of T1 and T2 relaxation times of adjacent protons. This manifests as a marked reduction in liver signal intensity on T2W and T2 W images, and a moderate loss of signal intensity on T1W images.
707
A 58-year-old man with recurrent episodes of upper abdominal pain undergoes MRCP, which demonstrates pancreatic atrophy and marked dilatation of the main pancreatic duct, which contains high T2-signal material. A low T2-signal nodular filling defect is also identified within the dilated duct. ERCP demonstrates thick mucus protruding from a bulging papilla. What is the most likely diagnosis? a. intraductal papillary mucinous tumour b. chronic pancreatitis c. mucinous cystadenoma d. pancreatic pseudocyst e. acute pancreatitis
a. Intraductal papillary mucinous tumour (IPMT) of the pancreas is characterized by a mucin-producing tumour with dilatation of the main or branch ducts of the pancreas due to copious secretions. They may arise in the main duct or branch duct. Main duct tumours typically cause dilatation of all or part of the duct, which is filled with mucinous secretions, appearing hyperintense on T2W images. A T2-hypointense intraductal filling defect may be identified, which may represent the tumour or concretions of mucin. Chronic pancreatitis may result in parenchymal atrophy and duct dilatation, with intraductal filling defects due to calculi or debris, but a bulging papilla at ERCP makes IPMT more likely. The branch duct type usually consists of conglomerated communicating cysts or a unilocular cyst in the uncinate process; these appearances may be mimicked by mucinous cystadenoma and pancreatic pseudocyst.
708
On contrast-enhanced CT of the abdomen, what is the most common abnormality of the spleen seen in sarcoidosis? a. capsular calcification b. multiple low-attenuation nodules c. splenomegaly d. splenic rupture e. multiple cystic lesions
c. Although symptoms directly referable to the spleen are unusual, autopsy studies have demonstrated the spleen to be involved in 38 – 77% of patients with sarcoidosis. The commonest abnormality demonstrated on imaging is splenomegaly, occurring in up to 60% of patients. Multiple hypoattenuating nodules measuring up to 3 cm may be seen distributed diffusely throughout the spleen in around 15% of cases, and may occur in the absence of splenomegaly. The lesions appear hypointense on all sequences at MRI and are best seen on T1W or early phase, gadolinium-enhanced, T2W fat-suppressed sequences. Abdominal or systemic symptoms are more frequent in patients with nodular hepatosplenic sarcoidosis. Spontaneous splenic rupture in sarcoidosis has been described but is very rare. Capsular calcification and multiple cystic lesions are not features of the disease.
709
A 54-year-old woman undergoes CT of the abdomen and pelvis for weight loss and is found to have multiple, irregular, calcified, low-attenuation lesions in the liver, suggestive of metastases. What is the most likely primary lesion? a. invasive ductal carcinoma of the breast b. mucinous adenocarcinoma of the gastrointestinal tract c. osteosarcoma d. non-small-cell lung carcinoma e. carcinoid
b. Calcified liver metastases represent up to 3% of liver metastases, and are most commonly seen with mucinous carcinomas of the gastrointestinal tract. They are also seen in osteosarcoma, breast cancer, lung cancer and carcinoid, but these are less common.
710
A 68-year-old woman presents with abdominal pain, distension and vomiting. Plain abdominal radiograph demonstrates bowel obstruction, gas within the biliary tree, and an ectopic, calcified, 3 cm gallstone. What is the most likely site of bowel obstruction? a. pylorus b. duodenum c. proximal ileum d. terminal ileum e. sigmoid
d. Gallstone ileus accounts for up to 5% of intestinal obstruction, increasing in prevalence with age. It involves erosion of a large gallstone from the gallbladder or common bile duct into the bowel, which goes on to cause obstruction. The classic appearance on plain film (Rigler’s triad) is only seen in 10% of cases, and consists of partial or complete intestinal obstruction (usually small bowel), gas in the biliary tree and an ectopic calcified gallstone. The most common site of fistulous communication is between the gallbladder and the duodenum, seen in 60%, and this may be demonstrated on barium meal as a contrast collection lateral to the first part of the duodenum representing barium within the gallbladder. Fistulas occur less commonly between the common bile duct and duodenum, gallbladder and colon. The ectopic gallstone most often causes obstruction at the terminal ileum (60 – 70%), followed by the proximal ileum, distal ileum, pylorus, sigmoid and duodenum
711
A 45-year-old woman undergoes abdominal ultrasound scan. The portal vein measures 16 mm in diameter and demonstrates continu- ous monophasic flow without respiratory variation. Portal vein flow velocity is hepatopetal and is measured to be 7 cm/s. What is the most likely diagnosis? a. normal findings b. Budd – Chiari syndrome c. portal hypertension d. cavernous transformation of the portal vein e. portal vein thrombosis
c. The normal portal vein measures up to 13 mm in diameter when measured in the AP direction where the portal vein crosses the inferior vena cava during quiet respiration in a supine patient. Portal venous flow is normally 12 – 30 cm/s and demonstrates respiratory variation but little or no pulsatility, though this may be seen in thin patients. Normal flow is hepatopetal (anterograde flow into the liver). Portal hypertension is defined as an increase in portal venous pressure above 10 mmHg, and is most commonly caused by cirrhosis in the western world. As portal pressure increases, portal vein diameter increases, and portal flow loses its respiratory fluctuation and becomes slow and turbulent. Reversed (hepatofugal) flow may occur in 8% of patients and is generally associated with a reduced portal vein diameter. Other findings include portosystemic collaterals, splenomegaly and ascites. In portal vein thrombosis, portal vein diameter increases, but no flow is seen on Doppler ultrasound scan. Echogenic thrombus may be seen within the lumen. Cavernous transformation of the portal vein may occur with chronic portal vein thrombosis, representing a conglomerate of collateral veins. Budd – Chiari syndrome affects the hepatic veins.
712
A 73-year-old woman presents with intermittent lower gastro- intestinal bleeding and iron deficiency anaemia. She is clinically suspected to have angiodysplasia. What are the most likely findings on barium enema? a. normal appearances b. multiple small polyps in the colon c. multiple shallow ulcers in the colon d. multiple, serpiginous, filling defects in the colon e. a focal, irregular, circumferential narrowing in the colon
a. In angiodysplasia, there is degenerative dilatation of the normal vessels in the submucosa of the bowel wall. It is associated with increasing age, and in about 20% of cases with aortic stenosis. It occurs most commonly in the right colon and presents with intermittent, low-grade bleeding. Barium enema shows no abnormality, as the lesion is submucosal, but increased tracer accumulation may be seen at the site of haemorrhage on 99mTc-labelled red cell scanning. Angiography, if performed, may demonstrate a cluster of vessels along the antimesenteric border during the arterial phase and early opacification of the draining ileocolic vein.
713
A 48-year-old man presents with epigastric pain, weight loss and peripheral oedema. Blood tests demonstrate hypoalbuminaemia. At barium meal the stomach is well distended, but there is poor mucosal coating. Markedly enlarged and tortuous gastric rugae are seen in the fundus and body of the stomach, with sparing of the antrum. What is the most likely diagnosis? a. lymphoma b. Menetrier’s disease c. gastric carcinoma d. Zollinger – Ellison syndrome e. eosinophilic gastroenteritis
b. Menetrier’s disease is characterized by mucosal hypertrophy of the fundus and body of the stomach, with excessive mucus secretion and a protein-losing enteropathy. There may be associated gastric ulceration. Barium meal shows impaired mucosal coating due to hypersecretion and marked gastric fold thickening, though the stomach distends normally. The stomach is the commonest site for gastrointestinal lymphoma, which may be polypoid, ulcerating or infiltrative. The infiltrative form may cause pronounced thickening of gastric folds, with preserved stomach distensibility, but hypersecretion is not a feature. Infiltrating gastric carcinoma may also cause thickened gastric folds, but associated desmoplastic reaction results in a rigid, poorly distensible stomach. Zollinger –Ellison syndrome results in hypersecretion of gastric acid, which impairs mucosal coating of barium, and is associated with ulceration and enlargement of rugal folds, but hypoproteinaemia is not a feature. Eosinophilic gastroenteritis may cause enlarged gastric folds and be associated with protein-losing enteropathy if the small bowel is involved. However, the antrum is most commonly involved.
714
On a barium meal examination, the incisura angularis marks the border between which structures? a. lesser and greater curvatures of the stomach b. antrum and pylorus of the stomach c. fundus and body of the stomach d. body and antrum of the stomach e. oesophagus and the stomach
d. The stomach is divided into the fundus, body, antrum and pylorus. The fundus is that part of the stomach extending superiorly and to the left of the cardiac orifice. The body extends from the cardiac orifice to the incisura angularis, which is a constant notch at the lower end of the lesser curvature marking the border between the body and the antrum of the stomach. The antrum extends from the incisura angularis to the proximal pylorus.
715
At abdominal ultrasound scan, when scanning the abdomen in a transverse plane at the level of the pancreas, which of the following structures may normally be seen lying between the superior mesenteric artery and the aorta? a. splenic vein b. left renal vein c. neck of the pancreas d. inferior vena cava e. common bile duct
b. Ultrasound scan of the pancreas may be difficult and vascular landmarks are important in its identification. In the transverse plane, the splenic vein can be seen coursing from the splenic hilum towards the liver, and the body and tail of the pancreas lie immediately anterior to this. The neck of the pancreas lies immediately anterior to the confluence of the splenic and superior mesenteric veins, and the head and uncinate process of the pancreas lie around this confluence, anterior to the inferior vena cava. At this level, the left renal vein can be seen entering the inferior vena cava, passing between the superior mesenteric artery and aorta.
716
A 45-year-old man is admitted after a road traffic accident in which he sustained abdominal injuries. After fluid resuscitation he under- goes CT of the abdomen and pelvis with intravenous contrast. This demonstrates a serpiginous area of attenuation value 130 HU at the splenic hilum with surrounding lower-attenuation material. What is this most likely to represent? a. active arterial extravasation b. acute clotted blood c. acute unclotted blood d. splenic arterial calcification e. ascites
a. In the evaluation of haemoperitoneum by CT, attenuation values can help differentiate ascites, unclotted blood, active bleeding and haematoma. Blood usually has a higher measured attenuation than other body fluids, but its appearance depends on the age, extent and location of haemorrhage. Unclotted blood has an attenuation value of 30 –45 HU, but this may be lower in patients with a lower serum haematocrit and if the haemorrhage is more than 48 hours old. Clotted blood has an attenuation value of 45 – 70 HU, and identification of the area of highest- attenuation haematoma (sentinel clot) on CT indicates the site of bleeding. Active arterial extravasation is seen as an area of higher attenuation resembling that in the aorta, ranging from 85 HU to 370 HU. It may be surrounded by lower-attenuation haematoma. This finding indicates the need for urgent embolization or surgical treatment.
717
In the staging of rectal cancer by MRI, which sequence provides optimum visualization of the tumour? a. T1W b. contrast-enhanced T1W c. T2W d. FLAIR e. proton density
c. MR is a highly accurate method of local staging of rectal cancer, with better assessment of locoregional nodal involvement than CT and clear depiction of the mesorectal fascia, allowing accurate prediction of whether the circumferential resection margin will be tumour free. T2W images provide optimal visualization of the tumour, which appears as an intermediate signal-intensity mass. Contrast-enhanced T1W images result in enhancement of the normal bowel wall as well as the tumour, which may lead to upstaging. FLAIR sequences are not routinely used for rectal cancer staging.
718
A 67-year-old man presents with a sensation of incomplete evacua- tion and passage of thick mucus per rectum. Serum electrolytes demonstrate hypokalaemia and hyponatraemia. He undergoes barium enema, which demonstrates a broad-based, papillary, 2 cm lesion in the rectum with poor mucosal coating of barium. What is the most likely diagnosis? a. lipoma b. tubular adenoma c. villous adenoma d. colorectal carcinoma e. solitary rectal ulcer syndrome
c. Villous adenomas are a histological subtype of adenomatous polyps with predominantly villous elements, representing 10% of adenomatous polyps. Typical appearances are of a broad-based lesion often over 2 cm in diameter, with frond-like surface projections. They have a higher malignant potential than the other subtypes of adenomatous polyp (tubular and tubulovillous adenomas), which increases further with the size of the adenoma. Lesions under 5 cm have a 9% risk of malignant transformation to adenocarcinoma, whereas lesions over 10 cm have a 100% risk. Villous adenomas are associated with excretion of large amounts of thick mucus, which may result in diarrhoea and electrolyte depletion, as well as poor mucosal coating at barium enema. Lipomas are typically seen as smooth, rounded, submucosal masses. Tubular adenomas are usually ,10 mm and, like lipomas, are not associated with electrolyte depletion. Solitary rectal ulcer syndrome may appear as polypoid lesions in the rectum and be associated with mucus secretion; however, it is normally associated with ulceration and is usually seen in young women.
719
A 26-year-old man with AIDS presents with weight loss. He is noted to have multiple raised purple skin lesions on examination. Contrast-enhanced CT of the abdomen and pelvis demonstrates multiple, subcentimetre, low-attenuation nodules within the liver, as well as high-attenuation lymphadenopathy at the porta hepatis, retrocaval and aortocaval regions. What is the most likely diagnosis? a. fungal infection b. multiple haemangiomas c. lymphoma d. Kaposi’s sarcoma e. mycobacterial disease
d. Kaposi’s sarcoma is a low-grade tumour of the blood and lymphatic vessels that primarily affects the skin but may cause disseminated disease in other organs. It is an AIDS-defining illness and is the most common AIDS-related neoplasm. The commonest manifestation is of multiple raised purplish skin lesions, but lymphadenopathy is the second commonest feature in AIDS-related Kaposi’s sarcoma. Typical appearances are of abdominopelvic lymph nodes that enhance after intravenous contrast due to high vascularity, appearing to be of higher attenuation than skeletal muscle. Liver involvement occurs in 34% of cases at autopsy, and typically causes multiple 5 – 12 mm nodules that are hyperechoic on ultrasound scan and of low attenuation on CT. Skin lesions are present in most cases, and help to distinguish Kaposi’s sarcoma from other conditions such as fungal microabscesses and multiple haemangiomas, which may have similar appearances on CT. Mycobacterial disease is characteristically associated with low- attenuation lymphadenopathy. Non-Hodgkin’s lymphoma is the second commonest AIDS-related neoplasm, and may cause multiple low-attenuation liver lesions, but it is not associated with skin lesions or high-attenuation lymphadenopathy.
720
In the assessment of tumour response to treatment, what method of tumour measurement is used in the RECIST (Response Criteria in Solid Tumours) criteria? a. unidimensional (long axis dimension) b. unidimensional (short axis dimension) c. bidimensional (product of longest diameter and greatest perpendicular diameter) d. bidimensional (product of longest diameter and shortest diameter) e. volumetric
a. The WHO response criteria were devised in 1981 to standardize the criteria used for measuring therapeutic response in cancer patients. These criteria set out definitions of complete response, partial response, no change and progressive disease, based upon bidimensional measurements of tumour lesions in the axial plane. The product of the longest diameter multiplied by the greatest perpendicular diameter is calculated for each measurable lesion, and the sum of these products is used to determine treatment response. The RECIST criteria were introduced in 2000, and were designed to be used in clinical trials. The criteria involve classifying the disease burden into measurable and non-measurable disease, followed by selection of up to 10 representative target lesions. The sum of the long axis dimension of the target lesions in the axial plane is used to determine the final response category. Potential concerns about the use of the RECIST criteria include possible confusion arising when lymph node measurements are performed using the short axis dimension and the increased workload involved for the radiologist.
721
In normal anatomy, the portal vein is usually formed by the confluence of which vessels? a. left and right portal veins b. inferior and superior mesenteric veins c. superior mesenteric and splenic veins d. inferior mesenteric and splenic veins e. left, middle and right hepatic veins
c. The splenic and superior mesenteric veins join to form the main portal vein slightly to the right of the midline behind the neck of the pancreas at L1 – 2 level. The extrahepatic portal vein is about 8 cm long, and divides into the right and left portal veins at the porta hepatis. The inferior mesenteric vein most commonly drains into the splenic vein, but may drain into the splenic/superior mesenteric vein confluence in 30% of cases or the superior mesenteric vein in 30%.
722
A 47-year-old woman with dysphagia undergoes barium swallow, which demonstrates a persistent smooth posterior bulge at the pharyngo-oesophageal junction at the level of C5 – 6 with mild proximal pharyngeal dilatation. What is the most likely diagnosis? a. normal findings b. impaired cricopharyngeus relaxation c. pharyngeal web d. anterior cervical osteophytes e. thyroid enlargement
b. Impaired cricopharyngeus relaxation (or cricopharyngeal achalasia) is hypertrophy of the cricopharyngeus muscle with failure of relaxation. It is seen in up to 10% of asymptomatic adults as a normal variant, as a compensatory mechanism in gastro-oesophageal reflux, and in association with a range of neuromuscular disorders. It appears on barium swallow as a smooth, shelf-like posterior projection at the level of C5 – 6 that persists during a swallow. In severe cases, it may result in functional obstruction or overflow aspiration. Symptomatic patients may be treated by cricopharyngeal myotomy. Pharyngeal webs are thin, anterior, shelf-like protrusions into the cervical oesophagus. They are frequent incidental findings but occasionally cause dysphagia. There is an association with Plummer – Vinson syndrome. Anterior osteophytes may cause an indentation of the posterior oesophagus, but these are usually asymptomatic. Thyroid enlargement may cause a smooth impression on the lateral wall of the oesophagus.
723
A 22-year-old woman with known medullary sponge kidney presents with recurrent upper abdominal pain and jaundice. Cholangiography demonstrates segmental saccular dilatation of the intrahepatic bile ducts and ectasia of the extrahepatic ducts. What is the most likely diagnosis? a. choledochocele b. choledochal cyst c. primary sclerosing cholangitis d. Caroli’s disease e. polycystic liver disease
d. Caroli’s disease is a rare congenital disorder characterized by multifocal segmental saccular dilatation of the intrahepatic bile ducts. It presents in childhood and early adulthood with upper abdominal pain, fever and transient jaundice. Up to 80% of patients have associated medullary sponge kidney. Typical CT findings are of multiple cystic structures with a central enhancing ‘dot’, representing the portal vein radicles surrounded by dilated ducts. Cholangiography is diagnostic and demonstrates saccular dilatation of the intrahepatic ducts of up to 5 cm in diameter, with frequent associated ectasia of the extrahepatic ducts. Choledochal cysts primarily cause cystic dilatation of the common bile duct, but there may be associated intrahepatic biliary dilatation. A choledochocele is a cystic dilatation of the intraduodenal portion of the common bile duct. Primary sclerosing cholangitis classically causes multifocal strictures of the intrahepatic and extrahepatic ducts, alternating with segments of dilatation. In polycystic liver disease, there is no communication of the cysts with the biliary tree.
724
A 65-year-old man presents with early satiety and bloating, and undergoes barium meal. This demonstrates a smoothly marginated, 15 cm mass in the body of the stomach, making an obtuse angle with the gastric wall. CT demonstrates peripheral enhance- ment of the mass with central areas of low attenuation and extragastric extension into the lesser sac. There is no associated lymphadenopathy. What is the most likely diagnosis? a. gastrointestinal stromal tumour b. gastric carcinoma c. gastric lymphoma d. adenomatous polyp e. gastric carcinoid
a. Gastrointestinal stromal tumours are the commonest mesenchymal tumours of the gastrointestinal tract. They are characterized by expression of KIT, a tyrosine kinase growth factor receptor, which distinguishes them from leiomyomas and leiomyosarcomas. They occur most commonly in the stomach, and have the classic appearance of a submucosal mass on barium meal, forming an obtuse angle with the gastric wall in profile. Focal areas of ulceration are seen in 60%. On CT, the tumours measure up to 30 cm and are often predominantly extragastric. Typical features are of peripheral enhancement, with central low attenuation representing necrosis, haemorrhage and cyst formation. Lymphadenopathy is not a feature. Gastric carcinoma and lymphoma rarely demonstrate exophytic growth and commonly have associated lymphadenopathy. Adenomatous polyps are mucosal lesions. Gastric carcinoid is usually seen in the antrum and characteristically shows associated ulceration.
725
From which vessel does the majority of the arterial supply to the pancreas derive? a. splenic artery b. left gastric artery c. superior pancreaticoduodenal artery d. right hepatic artery e. superior mesenteric artery
a. The main arterial supply to the pancreas is from the splenic artery, which provides numerous small branches into the pancreatic substance as it runs along the superior pancreatic border, as well as several larger arteries including the dorsal pancreatic artery from its proximal end (which may alternatively arise from the coeliac artery) and the arteria pancreatica magna halfway along its length. The pancreatic head has a dual blood supply, from the superior pancreaticoduodenal artery (derived from the gastroduodenal artery) and the inferior pancreaticoduodenal artery (derived from the superior mesenteric artery). The transverse pancreatic artery also runs along the length of the pancreas beside the main duct, and there are multiple anastomoses between the various vessels, allowing multidirectional flow.
726
A patient who has no function in their native kidneys is found to have declining renal function 1 day after transplantation. A MAG3 renogram shows normal perfusion but diminished excretion. Which of the following processes is affecting the transplanted kidney? a. acute rejection b. chronic rejection c. acute tubular necrosis d. renal vein thrombosis e. ciclosporin toxicity
c. Acute tubular necrosis is the commonest acute reversible cause of renal failure in the transplanted kidney and usually occurs within 24 hours. Of the complications of a transplanted kidney causing renal impairment, normal perfusion is seen in acute tubular necrosis, whereas renal vein thrombosis and transplant rejection have reduced perfusion accompanying the diminished excretion. Ciclosporin can cause a similar pattern of renal impairment but would be expected to occur 1 month after transplantation. Functional assessment of a transplanted kidney involves perfusion and excretion assessment with a MAG3 or DTPA renogram, MAG3 being the better test in transplant recipients with renal impairment. Doppler ultrasound resistive index measurement is also used, with a value of ,0.7 regarded as normal.
727
A portal venous-phase CT of the abdomen and pelvis is performed in a 60-year-old man to investigate upper abdominal and back pain, which is attributed to features of pancreatitis on the scan. An inci- dental finding is of a rounded, renal lesion of diameter 3 cm, with average attenuation value of 80 HU and containing no significant component with a negative attenuation value on pixel densitometry. There are no previous images for comparison. What is the most likely diagnosis of the renal lesion? a. angiomyolipoma b. renal cell carcinoma c. simple cyst d. high-density cyst e. infected cyst
b. A single portal venous phase CT is not the optimum image set to characterize renal parenchymal lesions. However, renal cell carcinoma is more commonly encountered than high-density cysts. Furthermore, carcinoma is most frequently found in men (2:1) aged over 50 years. Kidney neoplasms tend to have densities above 30 HU on an unenhanced CT and rise by more than 10 – 20 HU post-contrast, usually being above 70 HU in the portal phase.
728
A 30-year-old woman has a well-circumscribed, cystic, adnexal mass with areas of dense focal calcification, small enhancing soft-tissue elements, fluid – fluid levels and bright regions on T1W MRI that become dark on fat-saturated sequences. Which of the following pathologies is most likely? a. ovarian cyst with proteinaceous contents b. endometrioma c. mature cystic teratoma of the ovary d. ovarian cyst adenofibroma e. ovarian adenocarcinoma
c. The main differentials for an ovarian mature cystic teratoma (dermoid cyst) are endometriomas and proteinaceous ovarian cysts, which can also have fluid – fluid levels. Fat is frequently demonstrated in a dermoid cyst, but not in these differentials. Fat can be proven by a significant negative attenuation value on CT, or on MRI with chemical shift artefact in the frequency-encoding direction, a gradient echo sequence in which fat and water are in opposite phase or frequency-selective fat saturation sequences. Mature cystic teratoma contains mature tissues of germ cell (pleuripotent) origin. At least two of the three germlines should be represented. Mean patient age is 30 years, younger than for epithelial ovarian neoplasms, and it is the commonest ovarian mass in children. Usually asymptomatic, they can cause abdominal pain or other non- specific symptoms. They are bilateral in 10% of cases.
729
A 55-year-old male has an ultrasound scan of the renal tract prompted by a single urinary tract infection. A kidney cyst of diameter 2 cm with a thin septum is seen. The septum has perce- ptible enhancement on CT. What is the most appropriate manage- ment from the choices below? a. discharge with no follow-up b. imaging follow-up c. partial nephrectomy d. nephrectomy e. nephroureterectomy
b. An incidental, mildly complicated renal cyst has been uncovered. The Bosniak classification is a useful tool for evaluating cystic renal lesions, and guiding management. Simple cysts (Bosniak grade I) are thin walled, are of water density and have no enhancement. Minimally complicated cysts (grade II) may be clustered or septated, and have small curvilinear calcifications, a minimally irregular wall or high-density contents. Follow-up lesions (grade IIF) have perceptible enhancement of otherwise thin septations or are above 3 cm in diameter with high-density contents. Surgical lesions (grade III) have thicker septa or walls, measurable enhancement, coarse irregular calcification and irregular margin, are multiloculated or can be a non-enhancing nodular mass. Clearly malignant lesions (grade IV) can have necrotic components, irregular wall thickening and enhancing solid elements.
730
A 78-year-old man presents with a palpable, non-tender, left breast lump. Mammography demonstrates a well-defined, high-density, lobulated mass in the retroareolar region. Ultrasound appearances are of a hypoechoic mass with an eccentric position relative to the nipple. The ipsilateral axilla appears unremarkable. What is the most likely diagnosis? a. invasive ductal carcinoma b. lipoma c. breast abscess d. gynaecomastia e. lymphoma
a. Most symptomatic male breast lesions are benign, with gynaecomastia representing the commonest benign entity. Characteristic mammographic features are of a central, retroareolar, flame-shaped density. Male breast cancers are usually invasive ductal carcinomas, which typically appear as a discrete, high-density, well-defined mass with lobulated or spiculated margins at mammography. Microcalcification is seen less commonly than in females, but secondary signs, such as nipple retraction and skin thickening, occur earlier than in females due to smaller breast size. Ultrasound scan is particularly helpful in assessing the relationship of the mass to the nipple. An eccentric position is highly suspicious for breast cancer. Axillary lymphadenopathy is seen in approximately 50% of patients.
731
At the 20-week fetal anomaly ultrasound scan, the cervix of a 25-year-old primagravida is measured to be 22 mm long. She is most likely to have been treated with which of the following? a. oestrogens b. progestogens c. heparin d. salbutamol e. corticosteroids
a. Uterine cervix length can be measured transabdominally or transvaginally. With the former approach, the measurement can be 10% greater than the corresponding transvaginal measurement, because, while the full urinary bladder is a desirable acoustic window, it increases the cervical length. Transvaginally, the normal cervical length is 40+8 mm in the first 14 weeks of pregnancy, 42+10 mm in the second trimester and 32+12 mm from 28 weeks on. An incompetent cervix usually develops during the second or early third trimester. Incidence is increased after cervical trauma, in utero diethylstilbestrol exposure (cervical hypoplasia), and oestrogen treatment. On imaging, dilatation of the cervical canal is seen to begin at the internal os and extend out. It produces a beaking or funnelling appearance and shortens the cervical canal to less than 25 mm. Clinically, the membranes or even fetal parts may be seen through the external os.
732
A 40-year-old man has a testicular ultrasound scan, which demonstrates a multilobular mass that is homogeneous and hypo- echoic with Doppler flow seen in internal hypoechoic bands. Which of the following is the most likely diagnosis? a. teratoma b. lymphoma c. metastasis d. seminoma e. focal infarction
d. Ninety-five per cent of testicular tumours are germ cell tumours. Others include sex-cord and stromal tumours such as Leydig cell and Sertoli cell tumours. Primary lymphoma and metastases can also occur in the testicle. Non-seminomatous germ cell tumours include teratoma, embryonal carcinoma and choriocarcinoma, but these affect a younger population of 20 – 30 years. Ultrasound scan is the investigation of choice for detection of a testicular tumour and for assessing normality of the contralateral testicle. Seminomas present with mass or pain, and are generally lobulated masses on ultrasound scan with hypoechoic fibrovascular septations in which colour flow can be visible. T2W MRI demonstrates uniform intermediate signal with band-like low-signal septa. There is contrast enhancement, especially of the septations. They rarely calcify, but, if they do, the calcification is speckled or stippled.
733
A general practitioner performs a vaginal examination prior to intended removal of an intrauterine contraceptive device. The locator device cannot be seen or palpated. What is the most appropriate initial investigation for this patient? a. abdominal radiograph b. pelvic ultrasound scan c. pelvic CT d. pelvic MRI e. hysteroscopy
b. The device should be seen within the endometrial cavity on ultrasound scan as an echo-bright structure casting an acoustic shadow. If it is not identified in the uterus on ultrasound scan, then a plain abdominal film is indicated to exclude perforation and migration.
734
A 40-year-old woman with a history of prior pelvic radiotherapy for cervical cancer has an ultrasound scan for cyclical pelvic pain. The endometrium is distended by predominantly echo-poor material, and both ovaries have moderately large cysts containing low-level echoes. On MRI, the cervix returns low T2 signal and the ovarian cysts return high signal on fat-suppressed T1W sequences. Which of the following is the most likely diagnosis? a. recurrent cervical tumour with bilateral ovarian metastases b. recurrent cervical tumour and synchronous bilateral ovarian teratomas c. cervical stenosis and bilateral endometriomas d. cervical stenosis and bilateral ovarian cystadenocarcinomas e. new primary endometrial carcinoma with bilateral ovarian secondaries
c. Cervical stenosis can be congenital or acquired. When it is acquired, causes include cervical (after the menopause) or endometrial (before the menopause) carcinoma. Radiation and curettage can also produce cervical stenosis. On imaging, the endometrial cavity is distended by secretions and blood products. Reflux endometriosis can complicate cervical stenosis.
735
A postmenopausal patient has a hysterectomy and bilateral salpingo-oophorectomy for bilateral ovarian masses. Histological examination confirms bilateral ovarian tumours and reveals concomitant endometrial adenocarcinoma. What is the most likely histological diagnosis of the ovarian lesions? a. benign serous cystadenoma b. benign mucinous cystadenoma c. malignant serous cystadenocarcinoma d. malignant mucinous cystadenocarcinoma e. endometrioid tumour
e. Benign serous cystadenoma is bilateral in 20% of cases, benign mucinous cystadenoma in 5%, malignant serous cystadenocarcinoma in 50% and malignant mucinous cystadenocarcinoma in 25%. However, not only are endometrioid ovarian tumours frequently bilateral (30 – 50%) but they are also often (30%) found with concomitant endometrial adenocarcinoma.
736
A 60-year-old male, treated long term for hypertension with hydralazine, develops bilateral hydronephrosis. On CT KUB, the ureters are deviated medially and obstructed by a large, plaque-like, para-aortic, soft-tissue density. The aorta appears ‘taped-down’ to the vertebral column rather than elevated by the para-aortic tissue. Which of the following is the most likely diagnosis? a. enlarged retroperitoneal lymph nodes due to Hodgkin’s disease b. enlarged retroperitoneal lymph nodes due to non-Hodgkin’s lymphoma c. retroperitoneal fibrosis d. bilateral ureteral transitional cell carcinoma e. metastatic lymph node enlargement from testicular embryonal cell carcinoma
c. Retroperitoneal fibrosis can cause extrinsic compression of both ureters and retroperitoneal vascular structures such as the aorta, inferior vena cava and iliac vessels. It can be idiopathic or secondary to inflammatory aortic aneurysm, retroperitoneal metastases, haemorrhage, abscess, urinoma, diverticulitis, appendicitis, Crohn’s disease, and drugs such as ergot alkaloids and hydralazine. Malignant retroperitoneal lymphadenopathy causing ureteric obstruction tends to encircle the aorta, elevating it off the vertebral column. In contrast, retroperitoneal fibrosis rarely extends between the aorta and the vertebrae, and therefore appears to tape the aorta down to the spine.
737
A 30-year-old male is investigated by renal tract ultrasound scan for renal impairment. Both kidneys are smooth in outline but enlarged. Which of the following diagnoses typically produces this pattern of renal enlargement? a. autosomal dominant polycystic kidney disease b. von Hippel –Lindau disease c. sickle cell disease d. metastases e. nephroblastomatosis
c. Causes of bilateral smooth renal enlargement include diabetic nephropathy, acute glomerulonephritis, collagen vascular disease, vasculitis, AIDS nephropathy, leukaemia, lymphoma, autosomal recessive polycystic renal disease, acute interstitial nephritis, sickle cell disease, thalassaemia, acromegaly, amyloidosis, myeloma and acute urate nephropathy. When the bilateral renal enlargement is caused by masses, the differential diagnosis includes autosomal dominant polycystic disease, acquired renal cystic disease, lymphoma, metastases, Wilms’
738
A 20-year-old man has radical orchidectomy for a non-seminoma- tous germ cell tumour. A CT of the thorax, abdomen and pelvis shortly after surgery shows no lymphadenopathy or metastasis. Which of the following is the most appropriate follow-up regimen? a. repeat CT of the chest, abdomen and pelvis only in response to symptoms suggesting recurrence b. repeat CTof the chest, abdomen and pelvis only when serum tumour markers rise c. serial serum tumour marker measurement with yearly CT of the chest, abdomen and pelvis d. serial serum tumour marker measurement with 3-monthly CT of the chest, abdomen and pelvis for 1 year followed by 6-monthly CTof the chest, abdomen and pelvis for 1 year e. 3-monthly whole-body PET/CT
d. Stage 1, non-seminomatous, germ cell tumour patients should enter a surveillance programme of this type following orchidectomy. Such programmes are rarely used for seminoma, particularly when retroperitoneal radiation treatment is used. Rising tumour markers between surveillance scans or thereafter should provoke CT of the chest, abdomen and pelvis plus ultrasound scan of the remaining testicle. If no new disease is identified, MRI of the brain is indicated.
739
On ultrasound scan, a 30-year-old man is found to have bilateral testicular microlithiasis and unilateral testicular atrophy. There is a history of orchidopexy of the atrophic testicle. Which of the following is the most appropriate management? a. discharge b. self-examination only c. follow-up clinical examination and surveillance sonography d. further investigation with MRI e. testicular biopsy
c. Atrophic maldescended testes are at higher risk of developing malignancy, particularly seminoma, even after orchidopexy. The increased risk applies to the contralateral testicle also. Microlithiasis is also associated with testicular cancer. In combination, these features require clinical and sonographic follow-up.
740
A 50-year-old man complains to his general practitioner of painful sexual intercourse. An ultrasound examination of the penis is performed, which identifies dense, shadow-casting abnormalities of the periphery of both corpora cavernosa. What is the likely diagnosis? a. priapism b. Zoon’s balanitis c. penile squamous cell carcinoma d. Peyronie’s disease e. balanitis xerotica obliterans
d. The cause of Peyronie’s disease is unknown, but the result is fibrous thickening of Buck’s fascia and the septum between the corpora cavernosa. Calcified plaques are also seen. The fibrous areas do not engorge with blood, causing the penis at erection to be bent; this can make intercourse painful or impossible. Priapism is persistent painful erection of the penis but is not associated with penile calcification. Zoon’s balanitis is an idiopathic lymphocytic inflammatory condition of the penis, which may respond to topical steroid but is also treated by circumcision. Balanitis xerotica obliterans is the severe form of penile lichen sclerosus, which is an uncommon inflammatory dermatosis. It can cause phimosis and urinary retention.
741
On an axial MRI of the penis, three tubular masses of tissue, occu- pying much of the cross-sectional area of the penis, are entirely surrounded by a T1- and T2-hypointense layer. Breach of this layer upstages a penile cancer from T1 to T2. From the following choices, name this structure or structures. a. corpora cavernosa b. corpora spongiosa c. cavernosal arteries d. urethra e. tunica albuginea
e. Invasion of the corpora cavernosa or corpora spongiosa by a penile cancer is via the tunica albuginea, making the local stage T2. If the urethra, found centrally within the corpora spongiosa, is involved, it becomes T3. Each corpus cavernosum contains a central artery. Having rapidly flowing blood, this will also be hypointense on both T1W and T2W images because of flow void.
742
A 65-year-old female with biopsy-proven ovarian cancer has a staging CT scan. It reveals a left basal pleural effusion that after aspiration contains no malignant cytology. There is a large, complex, abdominopelvic mass, with ascites and peritoneal depos- its outside the pelvis measuring over 2 cm in diameter. Pelvic and para-aortic lymph nodes are enlarged. There are liver surface and parenchymal deposits. Which of the described features results in a classification of stage IV disease? a. ascites b. pleural effusion c. liver surface deposits d. liver parenchymal deposits e. 2 cm deposits outside the pelvis
d. Liver capsule deposits are stage T3/III. The pleural effusion cannot be regarded as M1/IV, because it requires positive cytology for this. Any involved regional nodes give stage IIIc and include obturator, common, internal and external iliac, laterosacral, inguinal and para-aortic.
743
A 35-year-old female has investigations for episodic right loin pain. Ultrasound scan of the renal tract is unremarkable. A DMSA scan is performed with the patient sitting, and shows only 30% contri- bution to the total tracer activity from the right kidney. When the counts are repeated supine, the contribution from the right kidney is 50%. What is the most likely abnormality of the right kidney? a. nutcracker kidney b. nephroptosis c. pelviureteric junction obstruction d. ureteric calculus e. vesicoureteric reflux
26) b. Ptosis of the mobile kidney when erect can cause symptoms and underestimation of parenchymal DMSA uptake. Since the differential function may be a factor in considering removal of a kidney, the technique should account for the possibility of nephroptosis influencing the counts. A nutcracker kidney is a rare cause of left-sided loin pain and haematuria; it is caused by compression of the left renal vein between the aorta and superior mesenteric artery.
744
An imaging request is received with the clinical information, ‘biopsy-proven adenocarcinoma of the cervix, for local staging’. Which of the following is the most appropriate technique? a. transvaginal ultrasound scan b. endoanal ultrasound scan c. CT abdomen and pelvis with intravenous and oral contrast d. MRI with pelvic phased-array coil e. 18FDG PET
d. MRI is the technique of choice for local staging of cancer of the uterine cervix. CT is less useful for staging of the primary tumour but has value in detecting involved lymph nodes and distant metastases. 18FDG PET may be useful in some cases for detection of distant metastases or the identification of recurrent disease. Its value will vary with the histological diagnosis on account of varying radiotracer avidity, with squamous cell carcinomas typically being avid.
745
A 65-year-old male being investigated for microscopic haematuria has an ultrasound scan, which suggests a 20 mm tumour in the cortex of the interpolar region of the left kidney. CT scan confirms an enhancing mass in the same location. On DMSA SPECT, this abnormality has good uptake. Which of the following is the most appropriate management? a. no further action b. biopsy c. nephrectomy d. image-guided drainage e. chemotherapy
a. The abnormality described is prominent or hypertrophic cortex since it takes up DMSA, which in the kidneys is a parenchymal tracer. Renal cell carcinoma, cysts, abscess, haematoma, scar and infarct would be seen as photopenic areas on DMSA SPECT, if large enough.
746
A 50-year-old man has surgery to remove a tumour confined to the adrenal gland. Histology reveals a phaeochromocytoma. Sub- sequently, he develops hypertension and urinary vanillylmandelic acid is found to be elevated. An MIBG scan is performed. Activity in which of the following organs is most likely to be a metastasis? a. lung b. bladder c. thyroid d. colon e. spleen
a. Normal MIBG uptake is seen in myocardium, liver, spleen, bladder, adrenal glands, salivary glands, nasopharynx, thyroid and colon. Abnormal MIBG activity is seen in phaeochromocytoma (paraganglioma when extra-adrenal), neuroblastoma, carcinoid tumour, medullary thyroid carcinoma and ganglioneuroma. Ten per cent of phaeochromocytomas are familial, 10% bilateral or multiple, 10% extra- adrenal and 10% malignant. Metastatic spread is to bone, lymph nodes, liver and lung.
747
A 23-year-old nulliparous woman is examined for dyspareunia. Biopsy confirms a clinically small but malignant-looking cervical lesion to be adenosquamous carcinoma. In such cases, local imaging staging must indicate which of the following? a. b. c. tumour size and distance from the internal os plus the cervix length tumour size and distance from the external os plus the uterine length tumour size and distance from the vaginal introitus plus length of the vagina d. tumour size and vascularity e. ovarian position
a. Trachelectomy may be considered to conserve the uterus and preserve fertility in young women with small tumours. Tumour size, distance from the internal os, cervix length and size of the uterus are required from the imaging. Surgery, radiation and chemotherapy are treatment options for cervical cancer dependent on stage. From 85% to 90% of cervical carcinomas have squamous cell histology, the remainder being mostly adenocarcinoma or adenosquamous.
748
A 35-year-old male with autosomal dominant polycystic kidney disease has been shown on CT to have, among the innumerable renal cysts, several high-density cysts. Which of the following MRI sequences would be most useful in detecting a renal cell carcinoma among haemorrhagic or proteinaceous cysts? a. T1W b. T2W c. T1W post-gadolinium d. T1W fat-suppressed post-gadolinium e. T1W post-gadolinium with pre-contrast T1W signal subtracted
e. The cornerstone of diagnosis here is the post-contrast enhancement of renal cell carcinoma. To identify this among the high T1 signal of haemorrhage or protein within cysts, it is ideal to subtract the pre- contrast T1 signal. Fat suppression will not remove the distracting high signal from mildly complicated cysts. Risk of renal cell carcinoma is increased in adult polycystic renal disease when in renal failure. Hence, caution may be required since at certain levels of renal impairment the use of MRI contrast is not advised.
749
A man is found to have a single adrenal mass of diameter 35 mm. On an unenhanced CT scan, the average attenuation value is 30 HU. On a CT timed at 60 seconds after iodinated contrast medium injection, the attenuation value of the mass is 90 HU. By 15 minutes after contrast, the attenuation value is 50 HU. Which of the following is the most likely diagnosis? a. lipid-rich adenoma b. lipid-poor adenoma c. metastasis d. adrenal cortical cancer e. adrenal haemorrhage
b. An unenhanced CT attenuation value of less than 10 HU is in keeping with a lipid-rich adenoma. With a threshold of 60% or higher for absolute contrast-enhancement washout, a sensitivity of 98% and specificity of 92% can be achieved in differentiating adenomas from non-adenomas. Percentage of enhancement washout¼ ([attenuation at 60 s – attenuation at 15 min] / [attenuation at 60 s – attenuation on plain CT]) 100. Applying this to the figures quoted in the question gives an absolute washout of around 66.6%.
750
An adult male patient who has been taking over-the-counter anal- gesics regularly for years has an IVU for ureteric colic. No radio- opaque calculi are seen on the control film. With contrast in the renal calyces, they are noted to be club shaped on the side of the pain. On the same image, there is a triangular filling defect in the renal pelvis. The colic is most likely to be caused by which of the following? a. radio-opaque stone b. radiolucent stone c. sloughed papilla d. blood clot e. transitional cell carcinoma
c. Predisposing factors to papillary necrosis include diabetes mellitus, analgesic nephropathy/abuse, sickle cell disease, pyelonephritis, obstructive uropathy, tuberculosis, trauma, cirrhosis, coagulopathy and renal vein thrombosis. None of the other options easily explains the club-shaped calyx. Blood clots from renal or tumour haemorrhage do cause ureteric colic but tend to elongate along the pelvis and ureter.
751
A 55-year-old man has biopsy-proven penis cancer. An MRI is performed. Which of the following is the best reason for performing this examination? a. to confirm the diagnosis b. to refute the diagnosis c. to perform local staging d. to assess metastatic spread e. to assess regional lymph node involvement
c. MRI offers good soft-tissue contrast that is of value in local staging of the primary tumour. Local extent is used to guide the type of treatment, which includes partial penile amputation, total penectomy and radiation therapy. MRI will also show enlarged regional lymph nodes, but the principal purpose of MRI is local staging. CT can also be used for nodal spread and metastatic disease.
752
A 38-year-old man presents with a classic history of ureteric colic. Plain abdominal film is unremarkable, and CT KUB shows ureteric dilatation and periureteric stranding down to the vesicoureteric junction on the side of the pain. No radio-opaque calculi are seen on the CT scan. Ultrasound examination shows a tiny, densely echogenic focus within the bladder wall, at the same vesicoureteric junction. For which of the following conditions is the patient most likely to be receiving treatment? a. diabetes mellitus b. asthma c. HIV d. gastro-oesophageal reflux e. headaches
c. While around 10% of renal and ureteric stones are radiolucent on plain film, almost all are opaque on CT. One exception is the tiny radiolucent calculi formed in patients on protease inhibitors such as indinavir used in the treatment of HIV/AIDS.
753
A patient with a lower ureteric transitional cell carcinoma has an MRI for locoregional staging purposes and a CT of the abdomen and pelvis for lymph node involvement and metastases. An 8 mm short axis node is recorded. In which of the following abdominopel- vic groups would this be significant by size criteria? a. inguinal b. common iliac c. external iliac d. internal iliac e. retroperitoneal
d. A short axis measurement of 7 mm or greater represents significant enlargement of internal iliac nodes. Regarding other nodal regions, significant enlargement for inguinal nodes is 10 mm, for common iliac 9 mm, for external iliac 10 mm, for obturator 8 mm and for retroperitoneal nodes between renal arteries and the aortic bifurcation 12 mm. In addition to size, there may be morphological clues to nodal involvement by cancer. Clustering of nodes, round nodes, nodes with irregular capsules, and nodes sharing CT or MRI characteristics of the primary tumour (attenuation, signal, cystic or necrotic changes, and contrast-enhancement pattern) are features suggesting lymph node involvement.
754
A 23-year-old woman is found on a 10-week dating ultrasound scan to have a twin pregnancy. A repeat examination prompted by blood spotting per vaginum, later attributed to a cervical erosion, shows a singleton pregnancy and no evidence of the twin. What should this be termed? a. foetus papyraceus b. vanishing twin c. fetal death in utero d. immune fetal hydrops e. non-immune fetal hydrops
b. ‘Vanishing twin’ occurs at less than 13 weeks when one twin is completely resorbed with no residuum evident on ultrasound scan. In foetus papyraceus, one twin is compressed and seen plastered to the adjacent membranes. Fetal death in utero or intrauterine death is signalled by absent heart and somatic movement in the second and third trimesters. Hydrops is excess total body water manifested as extracellular liquid accumulation in tissues and serous cavities. In hydrops of immune orgin, antibodies to red blood cells are present.
755
A 45-year-old female has imaging to stage a cervical carcinoma. The primary tumour is 5 cm in longest dimension, is seen to involve the uterine corpus, and has small-volume parametrial spread that does not reach the pelvic side wall. Parametrial lymph nodes are signifi- cantly enlarged. There is no hydronephrosis. Vaginal involvement is also seen, with the caudal extent of the tumour being below the level of the urethral orifice into the bladder base. Which of the described features causes the local stage to be T3a? a. size over 4 cm b. uterine corpus invasion c. parametrial spread d. vaginal invasion e. parametrial nodal involvement
d. The urethra is used as a landmark for the lower third of the vagina. Cervical cancer involvement of the upper two-thirds of the vagina is T2a. When the lower third is involved, it becomes T3a. T3b disease denotes disease that reaches the pelvic side wall or has caused hydronephrosis. Extension of disease into bladder or rectal mucosa is T4, as is disease extending out of the true pelvis. Extension into the corpus only is disregarded. T1b1 disease and T1b2 disease differ in being less or greater than 4 cm respectively. Parametrial lymph nodes are regional nodes and represent N1 disease; they do not influence the T stage.
756
A 14-year-old boy slides off his saddle while cycling, injuring his testicles. On ultrasound scan, one testicle has a paratesticular complex cystic mass, loss of testicular outline and several avascular planes within the testicle. Which of the following is the most appropriate management? a. discharge with analgesia b. admission for analgesia c. admission for elective surgery d. admission for surgery the following day e. immediate surgical intervention
e. Testicular rupture is described; it is an indication for immediate surgical intervention to salvage the testicle and prevent anti-sperm antibody development (testicles are immune-privileged sites). Other consequences of testicular trauma are fracture, haematoma and haematocele. Associated torsion may occur, due to trauma-stimulated, forceful, cremasteric muscle contraction.
757
A 50-year-old female with a breast carcinoma that clinically involves the skin is to be staged by CT. Other than those related to the primary tumour, there are no specific symptoms. Which of these CT protocols should be used? a. post-contrast brain b. non-contrast brain, neck, chest, abdomen and pelvis c. non-contrast brain, neck and chest, post-contrast abdomen and pelvis d. post-contrast brain, neck, chest, abdomen and pelvis e. post-contrast neck, chest, abdomen and pelvis
e. A T4 stage primary is described. Lower-stage breast cancers (T1/T2, less than 5 cm) are not usually staged by CT, as there is a less than 2% incidence of metastases at the time of diagnosis. Incidence of metastases at diagnosis for higher-stage cancers (T3/T4) is 15 – 20%. When staged with CT, a suitable protocol would be 100 – 150 ml of iodinated intravenous contrast agent used, with the neck and chest scanned 20 – 25 seconds after injection, and the abdomen and pelvis scanned 70 – 80 seconds after injection.
758
A 35-year-old male is prompted to see his general practitioner by his wife, who has noticed blood in the man’s semen. An ultrasound scan of the scrotum is performed. What is the most common appearance that would accompany this symptom? a. normal appearances b. enlarged spermatic cord, epididymis and testicle with decreased echogenicity c. testicular enlargement showing a hypoechoic lesion with a fluid – fluid level d. paratesticular ‘bag of worms’ appearance e. scrotal skin thickening
a. Investigation in most cases of haemospermia is not fruitful. In patients under 40 years, the causes are usually idiopathic or inflammatory (prostatitis, epididymo-orchitis, urethritis and urethral warts). The same causes apply in those over 40 years, but further possible causes include prostate cancer, benign prostatic hypertrophy, prostatic or seminal vesicle calculi, hypertension and carcinoma of the seminal vesicles.
759
A postmenopausal patient is investigated for ascites. Cytology from the ascites reveals cells in keeping with an epithelial ovarian malignancy. Which of the following is the most appropriate staging investigation? a. CT of the abdomen and pelvis with oral and intravenous contrast b. CT of the chest, abdomen and pelvis with oral and intravenous contrast c. MRI of the pelvis d. 18FDG PET e. PET/CT
a. Plain chest radiograph may be added to this as a routine, but chest CT would be requested only with an additional reason to do so. MRI of the ovaries can be helpful in characterizing ovarian masses where ultrasound scan and CA-125 are equivocal. There may be a role for PET/CT in defining disease extent, but cystic tumour deposits, particularly when they may be on or close to bowel or associated with ascites, present a challenge for this technique.
760
A CT KUB is performed on a 55-year-old South African man with unilateral loin pain. This demonstrates moderate ipsilateral hydroureteronephrosis with a stricture in the distal ureter. There is also widespread bladder calcification and bilateral distal ureter calcification. The responsible organism is most likely to be which of the following? a. Escherichia coli b. Schistosoma mansoni c. Schistosoma haematobium d. Schistosoma japonicum e. Mycobacterium tuberculosis
c. Schistosomiasis (bilharzia) is a parasitic infection that, worldwide, is the commonest cause of bladder wall calcification. Schistosoma japonicum and S. mansoni cause gastrointestinal tract infection, while S. haematobium affects the genitourinary tract. Schistosomiasis is endemic in South Africa, Egypt, Nigeria, Tanzania, Zimbabwe and Puerto Rico. The calcification spreads proximally up the ureters. In contrast, tuberculosis begins in the kidneys and spreads distally. Transitional cell carcinoma and cyclophosphamide-induced cystitis also cause bladder wall calcification. Causes of calcification within the urinary bladder lumen include stones and encrusted foreign bodies such as catheter balloons.
761
Into which of the following lymph node groups does lymph from the scrotum initially drain? a. para-aortic at the L1 – 2 level b. superficial inguinal c. obturator d. internal iliac e. presacral
b. Testes drain to the para-aortic nodes at the L1 – 2 level. The penile body drains to superficial inguinal nodes, while the proximal penis lymph drainage is to deep inguinal nodes. Appreciation of these patterns of lymph drainage is of vital importance when staging testicular, penile and other scrotal malignancies.
762
A 60-year-old male has an ultrasound scan of the renal tract for renal colic. There is an echo-free, thin-walled structure in the renal sinus with posterior acoustic enhancement and dilatation of the major calyces. A CT KUB (unenhanced) does not add to this appearance, but a 10-minute delayed, contrast-enhanced CT shows that the calyces are obstructed while the renal pelvis is not dilated but stretched over the non-enhancing sinus abnormality. What is the most likely diagnosis? a. sinus lipomatosis b. parapelvic cyst c. pelviureteric junction obstruction d. transitional call carcinoma e. renal cell carcinoma
b. The main differential diagnosis for a parapelvic or renal sinus cyst is hydronephrosis. Such cysts may present with pain due to obstructive caliectasis, but rarely cause hydronephrosis. They are found in 1.5% of autopsies and represent 4 – 6% of all renal cysts. A distinction is made in this question between renal and ureteric colic, the former symptom located to the loin and the latter more typically being loin to groin.
763
A 40-year-old man has a right-sided intratesticular mass of indeterminate ultrasound appearance. A CT scan of the chest, abdomen and pelvis reveals an enlarged, round, cystic, right paraca- val lymph node just caudal to the right renal vein. Other enlarged cystic lymph nodes are demonstrated in the mediastinum. No enlarged suprarenal or retrocrural nodes are seen. Considering these findings, which tumour type is most likely to be found in the orchidectomy specimen? a. epidermoid tumour b. malignant teratoma c. seminoma d. metastatic lung squamous cell carcinoma e. adenomatoid tumour
b. Seminoma tends to spread to contiguous nodes with the primary sites being, for a right-sided testicular tumour, right paracaval and interaortocaval nodes just below the junction of the right renal vein and inferior vena cava. Left-sided testicular tumours usually spread first to nodes just caudal to the left renal vein. Teratoma and other malignant, non-seminomatous, germ cell tumours of the testes can occupy mediastinal nodes without such direct cranial extension along the nodal groups. Nodes involved by seminoma tend to be soft-tissue density while nodes inhabited by non-seminomatous, germ cell tumours are frequently cystic. Metastases to the testicle are rare on account of the testicle being an immunologically privileged site with a blood – testicular barrier. Adenomatoid tumour is a benign epididymal lesion. Epidermoid tumour is also benign, appearing as an intratesticular, hypoechoic lesion, characteristically with an echogenic capsule. Classically, it assumes an ‘onion-skin’ appearance of concentric echogenic layers. Internal shadowing can be produced by calcification.
764
An 80-year-old man undergoes cystoscopy for macroscopic haematuria. He is found to have a 6 cm bladder tumour, biopsy of which confirms small-cell bladder cancer. He is considered suit- able for radical treatment. Which of the following is the most appropriate staging strategy? a. whole-body PET/CT b. MRI of the bladder c. MRI of the bladder plus CT of the abdomen and pelvis with intrave- nous contrast d. MRI of the bladder plus CT of the chest, abdomen and pelvis with intravenous contrast e. MRI of the bladder plus CT of the brain, chest, abdomen and pelvis with intravenous contrast
d. MRI is indicated for local (in fact locoregional) staging. CT of the chest is required in addition to the abdomen and pelvis because of the histological tumour type. 18FDG PET is not useful for staging urothelial tumours because of the urinary excretion of this radiotracer.
765
Lymphatic drainage from the lower third of the vagina is normally first to which of the following lymph node groups? a. obturator b. internal iliac c. external iliac d. inguinal e. retroperitoneal
d. The upper two-thirds of the vagina drain to the pelvic nodes, which is of relevance when imaging vaginal cancer. This cancer is uncommon, representing 1 – 2% of gynaecological malignancy. Eighty-five per cent of cases of vaginal cancer are squamous and 15% are adenocarcinoma. Clear-cell carcinoma is a rare form of adenocarcinoma found in young patients with in utero diethylstilbestrol exposure. Even less common are melanoma, sarcoma and adenosquamous carcinoma occurring as vaginal primaries. The two commonest cell types have different natural histories. Adenocarcinoma tends to involve pelvic and is more likely to involve supraclavicular lymph nodes, while squamous carcinomas are more likely to give rise to liver metastases. They are equally likely to metastasize to the lungs.
766
Cystoscopy is attempted on a 65-year-old female for persistent microscopic haematuria, but the scope cannot be advanced along the urethra. A biopsy is taken and MRI is performed. Axial T2W images show a mass of high signal intensity disrupting the normal, target-like, zonal anatomy of the urethra. Which of the fol- lowing cell types is the most likely histology from the biopsy? a. squamous cell b. transitional cell c. adenocarcinoma d. clear cell e. mastocyte
a. Urethral tumour is rare and occurs more in women than in men. Squamous cell carcinoma is the most common histological type followed by transitional cell carcinoma and then adenocarcinoma. MRI is the technique of choice for local staging.
767
MRI is performed for locoregional staging of vaginal cancer. Which of the following descriptions is the most likely appearance on a T2W sequence, given a small primary tumour confined to the vagina? a. central high signal within the vagina; focal homogeneous, low-signal mass not breaching the surrounding ring of intermediate-signal vaginal wall b. central high signal within the vagina; focal homogeneous, high-signal mass not breaching the surrounding low-signal vaginal wall c. central high signal within the vagina; focal homogeneous, intermedi- ate-signal mass breaching the surrounding low-signal vaginal wall d. central high signal within the vagina; focal homogeneous, intermedi- ate-signal mass not breaching the surrounding low-signal vaginal wall e. central intermediate signal; focal homogeneous, high-signal mass contained by low-signal vaginal wall
d. The vaginal epithelial layer and mucus are bright on T2W images. This is normally surrounded by low-signal (fibromuscular) vaginal wall. Tumours are typically intermediate signal on T2W images. If gadolinium is used, cancers often have early phase enhancement. Large tumours may have central necrosis. T1 tumours do not breach the low-T2-signal vaginal wall, whereas T2 tumours do and extend into the paracolpal fat. T3 tumours reach the pelvic side wall while T4 tumours extend beyond the true pelvis or involve bladder or rectal mucosa.
768
A 30-year-old, nulliparous woman with Stein – Leventhal syndrome is being treated for subfertility with clomiphene. She develops abdominal pain, distension, nausea and vomiting. Ultrasound examination of the abdomen reveals both ovaries to be larger than 7 cm in length and packed with large follicles, and also reveals an ovarian cyst 12 cm in diameter. Ascites and a pleural effu- sion are also seen. What is the most likely diagnosis? a. endometriosis b. ovarian cyst torsion c. ovarian hyperstimulation syndrome d. ovarian serous cystadenoma e. corpus luteum of menstruation
c. Ovarian hyperstimulation syndrome is more commonly seen with human menopausal gonadotrophin therapy but can also be seen with clomiphene. Severe complications relate to volume depletion, such as hypovolaemia, oliguria, electrolyte imbalance and thromboembolic events. Intra-abdominal haemorrhage is also reported.
769
A patient with a raised PSA has MRI of the prostate. There is diffuse, low-signal change throughout the peripheral zone on T2W images. At the right base, there is focal high signal on T1W images. At the left base there is restricted diffusion. In the right mid-gland, spectroscopic analysis reveals a choline plus creatine/ citrate ratio of considerably less than 0.8. In the left mid-gland, there is a diminished contrast wash-in rate. At the apex, the relative peak enhancement is less than in other regions of the prostate. Which is the most likely site of focal prostate carcinoma? a. left base b. right base c. left mid-gland d. right mid-gland e. apex
a. In staging prostate cancer with MRI, the large-FOV T1W sequence is useful for demonstrating haemorrhage (high signal), enlarged lymph nodes and bone metastases. Small-FOV T2W images of the prostate in axial, coronal and sagittal planes show the zonal anatomy well, with normal peripheral zone returning high signal. Cancer within the peripheral zone typically returns low signal on T2W images. Other typical findings in carcinoma of the prostate are increased relative peak enhancement, increased contrast wash-in rate, reversal of the choline plus creatine/citrate ratio on spectroscopy, restricted diffusion and increase in permeability on pharmacokinetic modelling.
770
In antenatal ultrasound scanning, which of the following is a major marker associated with trisomy 21? a. echogenic bowel before 20 weeks b. echogenic intracardiac focus c. brachycephaly d. small cerebellum e. hydrothorax
e. Major markers for Down’s syndrome include ventriculoseptal defect, cystic hygroma, omphalocele, duodenal atresia, hydrothorax, mild cerebral ventricular dilatation, corpus callosum agenesis and imperforate anus. The other options given are minor markers.
771
Which of the following unusual benign breast tumours is most likely to be locally infiltrating, aggressive and proliferative, and consist of only well-differentiated fibroblasts? a. neurofibroma b. granular cell tumour c. fibromatosis d. lipoma e. areolar leiomyoma
c. In 80% of cases of fibromatosis of the breast, there is b -catenin or adenomatous polyposis coli gene mutation. Granular cell tumour is most commonly found in the upper inner quadrant corresponding to the supraclavicular nerve territory and is thought to be of Schwann cell origin. Neurofibromas of von Recklinghausen’s disease are associated in an autosomal dominant fashion with a gene on chromosome 17.
772
A 68-year-old female patient has a pelvic ultrasound scan for a palpable mass. Arising within the left ovary is a 15 cm cyst with an irregular thick wall, frond-like solid elements, multiple septations over 2 mm thick and a pulsatility index of 0.5. These sonographic appearances are most in keeping with which of the following ovarian cystic structures? a. corpus luteum cyst b. follicular cyst c. polycystic ovaries d. benign ovarian neoplasm e. malignant ovarian neoplasm
e. Features of an ovarian cyst that suggest malignancy are thick irregular walls and thick septations (.2 mm), large overall size, solid elements and, on Doppler scan, a high peak systolic velocity and low-impedance diastolic flow. Together, these give a resistive index (RI) of ,0.4 and a pulsatility index (PI) of ,1.
773
A 17-year-old female with primary amenorrhoea is found on clini- cal examination to have a hypoplastic upper/middle vagina. MRI shows an absent uterus but normal tubes and ovaries. Which of the following is the most likely diagnosis? a. uterus didelphys b. unicornuate uterus c. Mayer – Rokitansky – Ku¨ster – Hauser syndrome d. uterine agenesis e. septate uterus
c. The uterus, fallopian tubes and upper vagina arise from the paired paramesonephric (mu¨llerian) ducts. The caudal parts fuse and ultimately form the uterus and upper vagina with resorption of the midline septum. The cranial parts remain unfused and form the fallopian tubes. Congenital uterine abnormalities arise with failure of development or fusion of this duct, or failure of midline resorption following fusion. Mayer –Rokitansky – Ku¨ster –Hauser syndrome describes uterine agenesis accompanied by hypoplastic proximal/middle third of the vagina but normal tubes and ovaries. Forty per cent of patients with the syndrome have pelvic kidneys and other urinary tract anomalies are also associated. They have a normal genotype.
774
A 60-year-old female who has declined intervention for a renal angiomyolipoma of diameter 6 cm presents with flank pain, hypo- tension and tachycardia. In this scenario, which of the following is most likely to account for the presentation? a. torsion of the angiomyolipoma b. haemorrhage from the angiomyolipoma c. rupture of the angiomyolipoma d. leak from the non-aneurysmal abdominal aorta e. nephroptosis
b. Angiomyolipomas are benign hamartomatous tumours that can occur in the kidneys. They contain fat, smooth muscle and abnormal blood vessels. Eighty per cent are sporadic and occur most often in females aged 50 – 80 years. Twenty per cent of patients with angiomyolipomas have tuberous sclerosis. Retroperitoneal bleed is the most significant complication and can be catastrophic. The risk increases with size of the lesion due to increased abnormal vasculature. Haemorrhage occurs because of large tortuous vessels and aneurysms. Embolization is performed for symptomatic (flank pain) angiomyolipomas, those that have bled at any size, and prophylactically when over 4 cm.
775
A postmenopausal woman is found on MRI to have a multicystic adnexal mass that contains fluid – fluid levels and does not show any fat suppression. In addition, her uterus shows a widened junctional zone containing small bright foci on T2W images. For which of the following diseases is she most likely to be receiving oral treatment that can account for these findings? a. urinary tract infection b. deep venous thrombosis c. endometrial cancer d. breast cancer e. bipolar disorder
d. The patient is receiving tamoxifen. Side effects include subendometrial cystic atrophy, endometrial hyperplasia and endometrial polyps. Less frequent side effects are endometriosis, polypoid endometriosis, adenomyosis and cervical polyps. There is an increased risk of endometrial carcinoma. On MRI, an endometrioma can appear as a multicystic adnexal mass of high T1 and both hypo- and hyperintense T2 signal, but without the fat suppression that would be expected with a mature cystic teratoma. Adenomyosis on MRI may manifest as a uterus with a thickened, low-signal, junctional zone on T2W images, containing small foci of high T2 signal.
776
An 80-year-old female is found incidentally to have a unilateral, unilocular, echo-free, thin-walled ovarian cyst of diameter 4 cm. There are no papillary projections or solid parts, and the CA-125 is less than 30 U/ml. Which of the following is the most appropriate management? a. pelvic exenteration b. total abdominal hysterectomy and bilateral salpingo-oophorectomy c. bilateral oophorectomy d. laparoscopic staging e. repeat transvaginal ultrasound scan in 4 months
e. The Risk of Malignancy Index (RMI) is used to stratify the likelihood of an incidentally identified ovarian cyst being malignant. The RMI is the product of the ultrasound score, the CA-125 level and the score assigned according to menopausal status. Low-risk cysts can be managed conservatively. A cyst below 5 cm in diameter that is unilocular, unilateral and echo-free, and has no solid parts or papillary formations has a risk of malignancy of less than 1%, and a 50% chance of resolving spontaneously in 3 months.
777
A 70-year-old woman is known to have uterine fibroids. There has been a clinically apparent increase in the uterine size. Transvaginal ultrasound appearances are in keeping with a large myometrial fibroid. Which of the following diagnoses must be considered in this patient? a. lipoleiomyoma b. endometrial hyperplasia c. adenomyoma d. leiomyosarcoma e. Bartholin’s gland tumour
d. Uterine fibroids are oestrogen dependent and should involute following the menopause. Increase in size of a fibroid after the menopause should raise the possibility of sarcomatous degeneration. On ultrasound scan, the appearance of leiomyosarcoma may be indistinguishable from that of a benign fibroid.
778
On an antenatal ultrasound scan, unilateral fetal ureteric dilatation is identified. Follow-up imaging after birth reveals persistent dilata- tion of a non-tortuous ureter that is aperistaltic in its distal segment. There are no other associated urogenital anomalies. Which of the following is the most appropriate diagnosis? a. prune-belly syndrome b. primary vesicoureteric reflux c. primary megaureter d. neuropathic bladder e. ureterocele
c. The primary megaureter occurs because of developmental aperistalsis of the distal ureter. The ureter tends to be dilated but straight in a congenital primary megaureter. Primary vesicoureteric reflux and obstructive secondary megaureter tend to cause tortuous ureteric dilatation.
779
A 28-year-old female has a hysterosalpingogram for infertility. Both fallopian tubes distend progressively with contrast injection but without peritoneal spill of contrast. A delayed plain abdominal radiograph shows continued distension of both tubes by dense col- lections of contrast and no peritoneal spill. Given these findings, which of the following is the most likely predisposition to infertility for this patient? a. tuberculosis b. endometriosis c. pelvic inflammatory disease d. submucosal uterine fibroids e. Asherman’s syndrome
c. The patient’s fallopian tubes are occluded, giving bilateral hydrosalpinx. The commonest cause of proximal or distal tubal occlusion is pelvic inflammatory disease. Endometriosis, infection following birth or abortion, and tuberculosis are other causes. Indications for hysterosalpingogram include infertility, recurrent miscarriage, assessment of the tubes after surgery and assessment of the integrity of a post-caesarean uterine scar. Contraindications are pregnancy, purulent vulval or cervical discharge, pelvic inflammatory disease in the preceding 6 months and contrast sensitivity. Historical contraindications include the immediate post-menstruation phase and recent dilatation and curettage, because of the risks associated with intravasation of oily contrast media.
780
An IVU is performed on a teenager. On a 15-minute, full-length radiograph, the right ureter and collecting system appear normal, while the left collecting system is displaced laterally and inferiorly, giving a ‘drooping flower’ appearance. What is the most likely con- genital anomaly? a. bilateral ureteral duplication b. left ureteral duplication c. left ureteral diverticulum d. recently passed left ureteric stone e. crossed fused ectopia
b. The ‘drooping flower’ appearance on IVU occurs with a duplicated system when the obstructed dilated upper pole collecting system displaces the contrast-opacified, lower collecting system laterally and inferiorly. The Weigert – Meyer rule describes the commonest relationship of insertion of the duplicated ureters into the bladder. The upper collecting system ureter inserts ectopically into the bladder inferior and medial to the orthotopic ureter, which enters the bladder near the trigone. The ectopic ureter may be stenotic or obstructed with or without ureterocele.
781
A 25-year-old woman with pelvic inflammatory disease has a raised serum b -hCG level. Ultrasound scan reveals an empty uterine cavity and an extrauterine amniotic sac. MRI some weeks later shows cir- cumferential bowel involvement by the placenta, which appears to be continuous with the bowel wall muscle. Which of the following is the most compelling reason for a further follow-up MRI? a. to check that the placenta has been fully removed by surgery b. to check for response to chemotherapy c. to ensure that the placenta involutes following delivery and no abscess has developed d. to stage the gestational trophoblastic neoplasia e. to date the pregnancy
c. The history and imaging features are of extrauterine abdominal pregnancy, which occurs when the fertilized ovum implants directly on the peritoneal surface of the abdomen. This is more likely when the prevalence of pelvic inflammatory disease and ectopic pregnancy is higher. The diagnosis is often established with ultrasound scan. MRI can be used to identify the location and assess adherence to abdominal viscera by the placenta. MR angiography can suggest feeding arteries. MRI at this stage also has a role in detecting fetal anomalies. If the placenta is adherent to abdominal viscera, it is not removed, because this could precipitate catastrophic arterial haemorrhage. Therefore, MRI is performed later to ensure involution of the placenta and exclude abscess formation. Placental adherence is suggested on MRI when it is contiguous with liver or spleen parenchyma, shows circumferential involvement of bowel or shows continuity with muscle of bowel wall.
782
A 70-year-old man with previously diagnosed, bilateral renal calculi is seen with fever, rigors and left loin pain. Ultrasound scan shows a dilated left renal collecting system. Which of the following is the most appropriate management? a. oral antibiotics with outpatient follow-up b. urinary catheter, intravenous antibiotics and admission to hospital c. left nephrostomy and antibiotics d. bilateral nephrostomy e. intramuscular anti-spasmodics
c. Pyonephrosis secondary to an obstructing calculus is the likely diagnosis. Percutaneous nephrostomy is indicated for temporary or permanent relief of an obstructed urinary system (malignant or benign obstructive uropathy), pyonephrosis, renal stones, iatrogenic ureteric injury, transplant kidney ureteric obstruction, and vesicovaginal fistula. A ‘one- stab’ puncture technique or Seldinger technique can be used. For any procedures where urinary infection is suspected or in stone disease, prophylactic antibiotics are mandatory – for example, 80 mg gentamicin or 750 mg cefuroxime. Clotting must also be checked and if necessary corrected. The major complication rate is around 3% while minor complications occur in around 15%. Major complications are septicaemia, blood loss requiring transfusion, pleural or abdominal viscera puncture, or transcolonic approach. Minor complications include retroperitoneal urine extravasation, clot colic from macroscopic haematuria and tube complications. Tube complications include catheter dislodgement, blockage, leaking, kinking and fracture.
783
A 20-year-old man presents with a 4-week history of a scrotal mass. There has been no trauma and no pain. Ultrasound scan confirms an intratesticular, partly cystic, heterogeneous mass. Which of the following tumour markers is most likely to be elevated? a. CA-125 and AFP b. CA-15-3 and CEA c. PSA d. CA-19-9 e. AFP and b -hCG
e. The patient’s age and ultrasound findings favour a non-seminomatous germ cell tumour. These are associated with elevated serum AFP and b -hCG. Together, CA-125 and AFP are associated with hepatocellular carcinoma, while CA-125 is also associated with ovarian cancer. CA-15-3 and CEA together are serum markers associated with breast cancer. PSA is associated with prostate cancer as well as several benign prostate conditions. CA-19-9 is mainly associated with malignancy of the pancreas and biliary tree.
784
A 28-year-old female is investigated for infertility. She has raised androgen levels and a higher than normal luteinizing hormone: follicle-stimulating hormone ratio. Pelvic ultrasound scan demon- strates bilaterally large ovaries with multiple small follicles. Which of the following is the most likely reason for the patient’s infertility? a. cervical fibroids b. hostile cervical mucus c. ovarian torsion d. polycystic ovarian disease e. bilateral ovarian endometriosis implants
d. Polycystic ovarian disease is diagnosed by clinical, biochemical and ultrasound findings. Clinically, oligomenorrhoea, hirsutism and obesity are features. Luteinizing hormone is increased as is the luteinizing hormone: follicle-stimulating hormone ratio. Androgen levels are increased. Sonographic findings vary from normal-looking ovaries through hypoechoic ovaries without individual cysts to multiple, 5 mm or more, peripherally located cysts in bilaterally large ovaries.
785
A 60-year-old man has a 4 cm rounded mass arising within the right kidney. It has heterogeneous, strong post-contrast enhancement. Calcification is also evident within the tumour. Which of the follow- ing features of this renal mass would favour the diagnosis of renal cell carcinoma over angiomyolipoma? a. marked vascularity b. calcification c. fat within the tumour d. round morphology e. hyperechogenic on ultrasound scan
b. Angiomyolipoma should not have calcification whereas it is seen in 10% of renal cell carcinomas. Both these tumours can be hypervascular. The cornerstone of diagnosis of an angiomyolipoma is identifying fat on CT or MRI; however, fat has been reported in renal cell carcinoma, and peripheral or renal sinus fat can become trapped in any large renal tumour.
786
A 75-year-old female is investigated for a slowly enlarging breast mass. There are no involved lymph nodes clinically. Following biopsy, clumps of tumour cells floating in pools of extracellular mucin and without a capsule are seen on histology. The 10-year sur- vival rate for this tumour is in the region of 70 – 90%. Which of the following is the most likely type of breast tumour? a. mixed mucinous carcinoma b. pure mucinous carcinoma c. phyllodes tumour d. inflammatory carcinoma e. melanoma metastasis
b. Pure mucinous breast carcinoma tends to be slow-growing, rarely metastasizes and has a good prognosis. It is important to differentiate this from mixed mucinous carcinomas, which are invasive carcinomas of no specific type with a mucinous component. The prognosis of mixed mucinous carcinomas is worse than that of pure mucinous carcinoma, tumour behaviour depending on the non-mucinous part. Mixed mucinous tumours comprise around 2% of breast cancers, and 33 – 46% have lymph node metastases at presentation. Pure mucinous carcinoma accounts for 1 – 2% of breast malignancies with an average age of 65, older than the average for breast cancer in general, which is 60 years. Many features of pure mucinous tumours mean that there is significant potential for them to be misdiagnosed as benign masses.
787
On MRI of the penis, a squamous cell carcinoma is typically seen as hypointense to the corpora on T1W and T2W images. What is the most likely appearance of this cancer on T1W images following intravenous contrast administration? a. no change in signal compared with that on the pre-contrast T1W sequence b. post-contrast enhancement greater than that of the corpus spongiosum but less than that of the corpora cavernosa c. post-contrast enhancement less than that of all corpora d. post-contrast enhancement greater than that of all corpora e. post-contrast enhancement greater than that of the corpora cavernosa but less than that of the corpus spongiosum
c. Most cancers of the penis (95%) are squamous cell carcinoma, but basal cell carcinoma, sarcoma, melanoma, lymphoma and urethral transitional cell carcinoma are also possible. Typical appearances of a primary penile cancer are of an ill-defined infiltrating lesion hypointense to the corpora on both T1W and T2W images. Tumours enhance following contrast, but to a lesser degree than the normal corporal bodies. Signal characteristics of melanotic melanoma will be notably different from the other tumour types, returning a bright signal on T1W images.
788
A 45 year old who is assumed to be pregnant presents with a uterus large for dates and hyperemesis gravidarum. The b -hCG levels are raised. Transvaginal ultrasound scan shows hyperechoic soft tissue with cysts filling the uterine cavity and a septated large left ovarian cyst. Which of the following additional features favours the diagno- sis of complete hydatidiform mole as opposed to any other gesta- tional trophoblastic disease? a. no fetal parts b. dysmorphic fetus c. associated prominent vessels d. pelvic lymph node involvement e. lung metastases
a. Gestational trophoblastic disease (GTD) is abnormal proliferation of the trophoblast, which can give rise to a complete or partial hydatidiform mole, invasive mole or choriocarcinoma. Increasing age and previous GTD are risk factors. Elevation of b -hCG aids diagnosis and is of value in assessing risk of metastatic disease (hence prognosis), and can be used to assess treatment response or detect recurrence. Complete moles have a higher malignant potential than partial moles. A complete mole has no fetal parts and has a 46,XX or, less often, a 46,XY karyotype. A partial mole has fetal parts and a triploid karyotype with 69 chromosomes. Eighty per cent of hydatidiform moles resolve with evacuation, 15% are locally invasive and 5% give rise to metastatic choriocarcinoma. When GTD is staged, there are no ‘regional’ nodes, and any nodal spread is considered metastatic with a significant worsening of prognosis. On ultrasound, the mole is echogenic but with a vesicular appearance. Fifty per cent of cases are associated with a large, septated theca lutein cyst. On Doppler ultrasound scan, they have prominent associated vessels with low resistance and high peak systolic velocity.
789
A portal venous phase abdominal CT scan of a 65-year-old man demonstrates an ill-defined, rounded area 4 cm in diameter within a kidney. It is heterogeneous but predominantly of attenuation value above 70 HU. It contains small dense calcific foci. Which additional feature suggests that the lesion is more likely to be a renal cell carcinoma than a transitional cell carcinoma of the collecting system? a. thickened indurated pelvicalyceal wall b. central location of the tumour with centrifugal expansion that compresses renal sinus fat c. renal parenchymal invasion with renal contour preservation d. renal vein thrombus e. further mass arising from the urinary bladder wall
d. Differentiation of renal cell from transitional cell carcinoma is helpful for planning surgical treatment since transitional cell carcinoma of the renal collecting system requires the more extensive surgical procedure of nephroureterectomy. Renal vein thrombus is seen with renal cell carcinoma while all other options given are features of transitional cell carcinoma of the kidney. Delayed contrast CT offers a pyelographic phase on which collecting system, ureter and bladder filling defects are clearly demonstrated. A urothelial field effect can occur, resulting in multiple transitional cell carcinomas throughout the renal tract. Renal cell carcinoma, as it expands, tends to distort the renal outline and is more likely to be peripheral and exophytic.
790
An adult male is initially investigated for abnormal liver function tests. Eventually, the diagnosis of Stauffer’s syndrome is pronounced. What are the likely CT findings? a. liver mass in keeping with hepatocellular carcinoma with renal metastases b. renal mass in keeping with renal cell carcinoma with liver metastases c. renal mass in keeping with renal cell carcinoma and hepatospleno- megaly without focal hepatic or splenic lesions d. hepatosplenomegaly and bilateral renal enlargement without focal lesions in any of these organs e. renal mass in keeping with renal cell carcinoma with a pancreatic head metastasis
c. Stauffer described a syndrome of nephrogenic hepatopathy in which a renal cancer without liver metastases causes hepatosplenomegaly and abnormal liver function. Renal cell carcinoma paraneoplastic phenomena include erythrocytosis and hypercalcaemia.
791
A plain abdominal radiograph is acquired for left-sided abdominal pain. The lumbar spine is osteoporotic with intervertebral disc space narrowing, vacuum phenomenon and calcification. Marginal osteophytes and endplate sclerosis are also present. In addition to nephrocalcinosis, a radio-opaque calculus is noted along the path of the left ureter. Which of the following is the most likely pattern of inheritance? a. autosomal recessive b. autosomal dominant c. autosomal dominant with partial penetrance d. mitochondrial e. X-linked
a. Alkaptonuria is the unifying diagnosis and is usually autosomal recessive. In this condition the absence of homogentisic acid oxidase causes accumulation of homogentisic acid, which is excreted in urine and sweat. Ochronotic deposition in the cardiovascular system causes atherosclerosis, aortic and mitral valve calcification, and myocardial infarction.
792
Which of the following is the strongest indication for a PET/CT scan? a. cervical cancer staging b. endometrial cancer staging c. ovarian cancer staging d. prostate cancer staging e. bladder transitional cell carcinoma staging
a. In the pelvis, 18FDG PET/CT is recommended for use in staging colorectal cancer, cervical cancer and non-Hodgkin’s lymphoma. It is also indicated for detecting recurrence of colorectal, cervical, endometrial and ovarian cancers. It is not recommended for primary urothelial malignancy or prostate cancer. Usefulness is limited in the renal tract by urinary excretion of FDG. It is less than satisfactory in prostate cancer because of the poor sensitivity for osseous metastases.
793
On the third day postpartum, a 25-year-old female develops right- sided lower abdominal pain and breathlessness. CT pulmonary angiogram confirms a pulmonary embolus. Bilateral leg Doppler scan is normal. Which of the following diagnoses requires the most serious consideration? a. appendicitis b. right ovarian vein thrombosis c. torsion of ovarian cyst d. broad ligament haematoma e. pelvic abscess
b. The puerperium is a hypercoagulable state, and puerperal endometritis can seed bacteria along the ovarian vein. Eighty per cent of thromboses are on the right and 14% are bilateral. Incidence is between 1 in 600 and 1 in 2000 deliveries. On contrast-enhanced CT, a tubular structure with low-density centre and peripheral enhancement is seen. Complications include inferior vena caval thrombosis, pulmonary embolus (25%), septicaemia, metastatic abscess formation and death (5%).
794
A 14-year-old boy who is a keen gymnast and fast bowler gives a history of several months of central low back pain that suddenly worsened during a game of cricket when he also developed bilateral shooting pains in his legs. There is no overt deformity on clinical examination, but lower back tenderness with generally restricted movement is noted. He undergoes radiographic, CT and MR imaging. What is the most likely radiological finding to explain the patient’s current symptoms? a. herniated intervertebral disc b. discitis c. Scheuermann’s disease d. spondylolysis e. spondylolisthesis
e. Back pain in adults is common and most frequently non-specific. In contrast, back pain in children is less common and often caused by a serious underlying condition. Spondylolysis is a defect in the pars interarticularis, the weakest part of the vertebra, and is an acquired condition even in childhood, where it is usually due to repetitive microtrauma in athletically active children. In isolation, it does not cause neurological symptoms, but bilateral defects can allow slippage of one vertebra over another, creating an abnormality of alignment, a spondylolisthesis. Disc herniation in children is rare and occurs as a result of a traumatic event rather than degeneration. It is usually lateral. Scheuermann’s disease is associated with a kyphotic deformity.
795
A 3-year-old child with aniridia is found to have a palpable abdom- inal mass. The mass is shown to arise from a kidney and to contain cystic elements on ultrasound scan. No calcification is seen in the tumour on CT. Which of the following diagnoses is the most likely? a. renal cell carcinoma b. neuroblastoma c. angiomyolipoma d. Wilms’ tumour e. ossifying renal tumour of infancy
d. Wilms’ tumour is the commonest renal tumour of childhood. Seventy- five per cent occur in children under 5 years, 5 – 10% are bilateral and 10% are multifocal. Calcification is seen in less than 15%. Nephroblastomatosis is a precursor, and the disease is associated with the WT1 and WT2 genes of chromosome 11. The WT1 abnormal gene is found in the WAGR syndrome of Wilms’ tumour, aniridia, genitourinary abnormalities and learning disability. It is also found in the DRASH syndrome of male pseudohermaphroditism and progressive glomerulonephritis. The abnormal WT2 gene is found with the Beckwith – Wiedemann syndrome and hemihypertrophy.
796
A 6-month-old infant presents with irritability, joint swelling and flattened occiput. Radiographs of the limbs show bowing of the long bones, widened growth plates with cupped metaphyses and generalized osteopenia with coarse trabeculation seen throughout. A diagnosis of rickets is made. Which additional finding is likely to suggest hypophosphatasia as the underlying cause rather than primary vitamin D deficiency? a. irregular poorly mineralized epiphyses b. sclerotic rim to the epiphyses c. lucent extensions into the metaphyses d. fraying of the metaphyses e. relatively normal mineralization
c. Rickets is osteomalacia occurring in an immature skeleton, resulting in poorly mineralized osteoid and bone softening. The fundamental deficiency is of the active form of vitamin D (1,25- dihydroxycholecalciferol), which is required for normal osteoid mineralization. Dietary vitamin D (7-dehydrocholesterol) is converted in the skin to cholecalciferol (vitamin D3) by sunlight. In the liver, cholecalciferol is hydroxylated to 25-hydroxycholecalciferol by the enzyme 25-hydroxylase; this is then further converted into the active form of vitamin D in the kidney. Abnormality anywhere in this pathway can lead to rickets, and its causes include dietary deficiency (most common), lack of sunlight, and renal or liver disease. Phosphate has a significant influence in the regulation of vitamin D metabolism, and hypophosphataemia is noted in many cases of vitamin D-refractory rickets. Hypophosphatasia is a rare inherited metabolic disorder of decreased tissue non-specific alkaline phosphatase causing defective bone mineralization, which is refractory to vitamin D treatment. It resembles rickets radiologically, with irregular, poorly mineralized epiphyses with a sclerotic rim and metaphyseal fraying. However, in hypophosphatasia, associated irregular lucent extensions into the metaphyses representing uncalcified bone matrix are characteristic, allowing differentiation from rickets.
797
A 5-year-old, previously well girl is found to have a palpable abdominal mass. Ultrasound scan demonstrates a 5 cm cystic lesion in the right upper quadrant. The wall of the lesion has an outer hypoechoic layer and an inner echogenic layer. The remainder of the abdomen appears normal. What is the most likely diagnosis? a. Wilms’ tumour b. ovarian cyst c. pancreatic pseudocyst d. duplication cyst e. choledochal cyst
d. Gastrointestinal duplication cysts account for 15% of paediatric abdominal masses. They most commonly arise from the small bowel and colon and, although usually asymptomatic, those that contain ectopic gastric or pancreatic tissue may present with ulceration or haemorrhage. Contrast studies are not useful for diagnosis, as most cysts do not communicate with the bowel lumen. Ultrasound scan is the most appropriate technique and demonstrates a simple anechoic cyst with a characteristic two-layered wall, representing the inner echogenic mucosa and the outer hypoechoic muscle. These appearances are characteristic of bowel wall, and help to distinguish a duplication cyst from other cystic lesions such as ovarian cyst, which are typically thin walled. Pancreatic pseudocyst is unlikely in the absence of previous episodes of pancreatitis.
798
A 12-year-old boy with known Langerhans’ cell histiocytosis pre- sents with tachypnoea, cough and dyspnoea. A chest radiograph and then high-resolution CT are performed. What are the most likely findings? a. bilateral, symmetrical consolidation b. bronchiectasis c. reticulonodular changes in mid-zones d. bronchial wall thickening e. bilateral hilar lymphadenopathy
c. Langerhans’ cell histiocytosis presents with features similar to adults in children over 10 years. Typically, there are reticular/reticulonodular changes that progress to cysts and honeycombing. These predominate in the upper zones and mid-zones with sparing of the costophrenic angles. The diagnosis is made by the characteristic radiographic and clinical findings. Bronchoalveolar lavage or biopsy may be necessary if the diagnosis is in doubt.
799
A 2-year-old girl presents with respiratory distress following recent treatment for a chest infection that has been slow to resolve. A chest radiograph shows a large, well-defined mass in the lower right thorax. This comprises multiple cysts, with apparent hypoplasia of the ipsilateral lung, and is causing contralateral mediastinal shift. What is the most likely diagnosis? a. bronchopulmonary sequestration b. hypogenetic lung syndrome c. congenital lobar emphysema d. bronchogenic cyst e. type I cystic adenomatoid malformation
e. Cystic adenomatoid malformation (CAM) is a congenital cystic abnormality of the lung. There are three types: I – single/multiple large cysts of >20 mm; II – multiple cysts of 5 – 12 mm; III – solitary mass with microcysts. If undetected in utero, they usually present in the first year with respiratory distress and cyanosis, but can present later with recurrent infections. In type I CAM, the chest radiograph shows an expansile mass with multiple air- or fluid-filled cysts, with compression or hypoplasia of ipsilateral lung and contralateral mediastinal shift. Extra- lobar bronchopulmonary sequestration produces a wedge-shaped mass, usually posteromedially in either lung. Bronchogenic cysts are usually well-defined soft-tissue masses arising from the mediastinum and often present later in life. Hypogenetic lung syndrome produces a small hemithorax with reduced vascularity rather than a mass. Congenital lobar emphysema produces overinflation of one, or occasionally two, lobes, with hyperlucency. This may be mass like at birth due to delayed clearance of fluid.
800
A newborn baby presents with coughing and choking during feeding. On examination, the baby is noted to drool excessively. Attempted passage of a feeding tube into the stomach is unsuccess- ful, and a chest radiograph shows the coiled feeding tube in the proximal oesophagus. An abdominal radiograph demonstrates gas within the stomach. What is the most likely diagnosis? a. oesophageal atresia alone b. oesophageal atresia and proximal tracheo-oesophageal fistula c. oesophageal atresia and distal tracheo-oesophageal fistula d. oesophageal atresia with proximal and distal tracheo-oesophageal fistulae e. tracheo-oesophageal fistula without oesophageal atresia
c. Oesophageal atresia is suspected when a newborn baby presents with drooling, and is confirmed on a chest radiograph by the presence of a gas-distended, proximal oesophageal pouch, or a coiled feeding tube within the pouch. Absence of gas in the abdomen on an abdominal radiograph implies oesophageal atresia either alone or with a proximal fistula. However, gas within the abdomen implies the presence of a distal tracheo-oesophageal fistula. Occasionally, there may be both proximal and distal tracheo-oesophageal fistulae, but this situation is rare, accounting for only 1% of patients, as opposed to around 80% with distal fistula alone. Oesophageal atresia and tracheo-oesophageal fistula may be part of the VACTERL association, which includes vertebral, anorectal, cardiovascular, tracheo-oesophageal fistula, renal and limb anomalies.
801
A 3-day-old boy presents with respiratory distress without cyanosis. Clinically, there is reduced air entry in the right hemithorax with dull percussion note. A chest radiograph shows an opaque right hemithorax with mediastinal shift to the left. Ultrasound scan shows a large effusion, which aspiration demonstrates to be milky. What is the most likely cause? a. idiopathic b. birth trauma c. lymphangioleiomyomatosis d. thoracic duct atresia e. lymphangiectasia
a. Chylothoraces in neonates are usually right sided, and in most cases no obvious cause is found. Treatment is conservative with special formula and intermittent aspiration. All of the listed conditions are causes of chylothorax, but lymphangioleiomyomatosis presents in adult females and not in the neonatal period.
802
Of the following findings on a cervical spine radiograph in a 10- year-old child, which is abnormal in the context of a traumatic injury? a. anterior wedging of the C3 vertebral body b. anterolisthesis in flexion at C2 – 3 c. prevertebral soft-tissue thickness of 15 mm at C6 d. predental space of 6 mm in flexion e. predental space of 3 mm in neutral
d. The maximum predental space is 2.5 – 3 mm in an adult and 5 mm in a child. Any widening suggests injury to the alar ligamentous complex in the context of trauma. Other causes of widening are Down’s syndrome, rheumatoid arthritis, neurofibromatosis and osteogenesis imperfecta. Anterior wedging of C3 and pseudosubluxation at C2 – 3 and C3 – 4 are within normal limits in children. Additionally, prevertebral soft tissues can be greater than in the adult, certainly up to 100% of the anteroposterior dimension of the vertebral body at the C6 level.
803
A newborn is found to have reduced air entry and breath sounds in the right hemithorax, but is otherwise well. A chest radiograph shows an opaque right hemithorax with ipsilateral mediastinal shift. Which feature on ventilation – perfusion scintigraphy would make the diagnosis of pulmonary hypoplasia more likely than complete collapse due to bronchial obstruction? a. matched marked reduction in ventilation and perfusion b. more marked reduction in perfusion than ventilation c. more marked reduction in ventilation than perfusion d. normal ventilation with reduced perfusion e. normal perfusion with reduced ventilation
a. Pulmonary hypoplasia is the presence of a completely formed but congenitally small bronchus with rudimentary parenchyma and vessels. This produces a matched marked reduction in ventilation and perfusion or, in severe cases, complete absence of both ventilation and perfusion. In total lung collapse, the ventilation would be reduced or absent with often reduced, but better, perfusion.
804
A 6-year-old girl is investigated for abdominal pain, jaundice and a palpable right upper quadrant mass. Ultrasound scan of the abdomen demonstrates a 5 cm cystic structure at the porta hepatis, which is separate from the normal gallbladder, and com- municates with normal intrahepatic ducts. What is the most likely diagnosis? a. biliary atresia b. choledochal cyst c. pancreatic pseudocyst d. duodenal duplication cyst e. pericholecystic abscess
b. Choledochal cyst is a congenital condition characterized by aneurysmal dilatation of the common bile duct, which in the majority of cases is associated with an anomalous junction of the common bile duct and pancreatic duct, allowing reflux of pancreatic enzymes into the common bile duct and resulting in weakening of the wall. The most common type (type I) is a localized dilatation of the common bile duct below the cystic duct. Communication with the common hepatic or intrahepatic ducts is demonstrated, and intrahepatic ducts are usually not dilated. Biliary atresia presents in the neonatal period with jaundice. Typical findings include an echogenic triangular structure at the porta, representing the atretic biliary plate, and a small or non-visualized gallbladder. Pancreatic pseudocyst and duodenal duplication cyst do not communicate with the biliary tree. A pericholecystic abscess would be unlikely in the presence of a normal gallbladder.
805
A 3-year-old boy presents with fever, skin rash and abdominal pain. On examination, he has a maculopapular rash on the extensor sur- faces, erythema of the oral mucosa, palms and soles, and multiple enlarged cervical lymph nodes. He is tender in the right upper quad- rant. Abdominal ultrasound scan demonstrates a markedly enlarged gallbladder with a thin wall and a positive sonographic Murphy’s sign is elicited. No gallstones are seen. What is the most likely diagnosis? a. acute acalculous cholecystitis b. acute gallbladder hydrops c. choledochal cyst d. acute calculous cholecystitis e. emphysematous cholecystitis
b. Kawasaki’s syndrome is an acute multisystem vasculitis with a predilection for the coronary arteries, generally affecting children under 5 years of age. As well as skin, joint and cardiovascular manifestations, patients may develop acute hydrops of the gallbladder, probably caused by transient obstruction of the cystic duct. Ultrasound scan demonstrates a markedly enlarged and tender gallbladder with a thin wall. Acute acalculous cholecystitis may be seen in children in the high dependency or intensive care setting, especially with septicaemia and trauma, but it usually results in gallbladder wall thickening and less marked gallbladder dilatation. Choledochal cyst (aneurysmal dilatation of the common bile duct) is seen as a fusiform cyst beneath the porta hepatis separate from the gallbladder. Acute calculous cholecystitis is associated with gallstones within a thickened gallbladder wall. Emphysematous cholecystitis usually occurs in adults over 50, and gas is seen as arc-like, high-level echoes outlining the gallbladder wall.
806
A 7-year-old boy who has recently arrived in the country from south-east Asia presents with colicky abdominal pain. Blood tests show eosinophilia. He undergoes small bowel follow-through examination, which reveals multiple tubular filling defects in the small bowel averaging 20 cm in length, some of which contain a central, barium-filled canal. What is the most likely diagnosis? a. ancylostomiasis (hookworm) b. ascariasis (roundworm) c. strongyloides infection d. anisakiasis (herring worm disease) e. taeniasis (tapeworm)
b. Ascariasis is the most common parasitic infection worldwide, predominantly affecting children aged 1 – 10 years. Worms mature in the small bowel and may be identified on plain films as tubular soft-tissue densities or on barium studies as linear or coiled filling defects of 15 –35 cm in length. Barium ingested by the worms causes opacification of their central linear enteric canals. Patients may present with abdominal pain, appendicitis and haematemesis; occasionally, a bolus of worms may cause small bowel obstruction. Tapeworms may also appear as linear filling defects, but are usually much longer, reaching many feet in length. In addition, tapeworms have no alimentary canal and do not ingest barium. Hookworms measure 8 – 13 mm and cannot be visualized on barium studies. Anisakiasis of the small intestine usually appears radiologically as bowel wall thickening and luminal narrowing. Strongyloides infection may manifest as fold thickening and effacement, with a pipestem appearance of the jejunum in advanced cases.
807
A 7-year-old boy presents with a 1-week history of a cough productive of green sputum. He is pyrexial with a mildly raised white cell count. Which feature on the chest radiograph would be more suggestive of bacterial pneumonia than viral pneumonia? a. pleural effusion b. peribronchial cuffing c. atelectasis d. airspace opacification e. cavitation
e. Bacterial pneumonia is a combination of airway and alveolar disease, whereas viral pneumonia tends to affect airways and peribronchial tissues. Bacterial pneumonia produces two patterns of disease: lobar (consolidation, no volume loss, usually in one lobe, may cavitate) or bronchopneumonic (patchy airspace change which enlarges and coalesces, and volume loss due to mucus plugging). Viral pneumonia tends to produce peribronchial linear densities and an interstitial pattern, though airspace change may be seen in up to 50% of cases. Hilar adenopathy is not often seen, but effusions are present in 20% of cases. Pneumatoceles, pneumothorax and cavitation do not occur. Viral pneumonia may sometimes be complicated by bacterial pneumonia.
808
A 10 year old presents with increasing dyspnoea on exertion. A chest radiograph shows dilatation of the left pulmonary artery with mural calcification. The right pulmonary artery and lungs are normal. An ECG shows right ventricular hypertrophy. What is the most likely diagnosis? a. aneurysm of pulmonary artery b. patent ductus arteriosus c. right pulmonary atresia d. pulmonary stenosis e. Swyer – James syndrome
d. Pulmonary stenosis is often asymptomatic, but it presents when there is a high gradient across the pulmonary valve. There is dilatation of the pulmonary trunk and left pulmonary artery (post-stenotic dilatation), but the right pulmonary artery is unaffected, as the jet preferentially enters the left pulmonary artery. Calcification of the pulmonary arterial wall is highly suggestive. Pulmonary atresia is associated with a small hemithorax and mediastinal shift towards the affected side. Swyer– James syndrome presents with a hyperlucent lung with reduced vascularity and a small hilum. Neither of these conditions is associated with dilatation of the pulmonary arteries.
809
A 4-year-old boy is investigated for abnormal gait, with swelling and deformity of the right lower leg. Radiographs reveal epiphyseal irregularity and multiple abnormal ossifications around the medial portions of the distal femoral, and proximal and distal tibial epi- physes of the affected leg, with normal appearances of the lateral epiphyses and the whole of the contralateral lower limb. MRI demonstrates that the ossifications lie within the epiphyseal carti- lage. What is the described condition? a. hereditary multiple exostoses b. Trevor’s disease c. Ollier’s disease d. Morquio’s syndrome e. warfarin embryopathy
b. Trevor’s disease (also called dysplasia epiphysealis hemimelica) is a rare developmental bone dysplasia. It primarily occurs in children aged 2 – 4 years and affects boys more commonly than girls. It shows a preponderance for the lower limbs, most commonly affecting the knee and ankle, and demonstrates single or multiple osteocartilaginous tumours arising from epiphyses. The lesion is characteristically hemimelic, involving either the medial (two-thirds of cases) or lateral aspect of the ossification centres. Cases can be classified as localized, classic or generalized.
810
MRI of the brain in a premature baby reveals ischaemic lesions adjacent to the trigone of the lateral ventricle. What is the most likely insult to have caused these appearances? a. prolonged partial asphyxia b. acute profound asphyxia c. germinal matrix haemorrhage d. rupture of a choroid plexus cyst e. venous sinus thrombosis
a. There are three patterns of hypoxic ischaemic encephalopathy. Periventricular leukomalacia occurs in watershed areas of arterial distribution. It is caused by prolonged partial asphyxia in preterm or term babies. Acute profound asphyxia causes lesions in the deep grey matter, hippocampus and dorsal brain stem. Lastly, there is multicystic encephalomalacia that follows devastating encephalopathy and generalized brain oedema.
811
Abdominal ultrasound is performed on a neonate on the high- dependency unit to investigate a palpable mass. A heterogeneous avascular suprarenal mass is identified. Cystic change and a periph- eral hyperechoic rim develop over a series of scans. Which of the following is the most likely cause of the abdominal mass? a. nephroblastoma b. neuroblastoma c. adrenal haematoma d. phaeochromocytoma e. myolipoma
c. Adrenal haemorrhage is not only the commonest cause of neonatal adrenal mass, but is also more likely to be seen in neonates in a high-dependency unit because it is associated with perinatal stress, hypoxia, septicaemia and hypotension. It can be unilateral or bilateral, but, even when bilateral, it does not usually cause adrenal insufficiency. Initially, the haematoma appears as an avascular heterogeneous mass on ultrasound scan that becomes cystic and smaller over a period of weeks. A hyperechoic rim can form, representing peripheral calcification. Haematomas can become infected, resulting in an abscess. Neuroblastoma is the main differential diagnosis; on ultrasound scan, it appears as a hyperechoic mass that can have internal flecks of calcification. Repeat ultrasound scan at 1 week will not show the changes that haematoma undergoes.
812
A 6-year-old boy attends accident and emergency following an injury to his elbow. Radiographs show apparent ossification in the regions of the capitellum, radial head and trochlea, and there is marked soft-tissue swelling and a joint effusion. Which of the following is the most likely underlying injury? a. avulsed medial epicondyle b. avulsed lateral epicondyle c. avulsed olecranon d. supracondylar fracture e. fractured radial head
a. There are six ossification centres around the elbow. The absolute age at which they appear varies slightly, but the order of appearance does not. The order in which they appear is capitellum, radial head, internal (medial) epicondyle, trochlea, olecranon and external (lateral) epicondyle. This can be remembered as ‘CRITOE’, with typical ages in boys of 1, 5, 7, 10, 10 and 11 years, being up to 2 years earlier in girls. The importance of the order is in recognizing medial epicondyle avulsion injuries, which are relatively common due to the powerful forearm flexors. The medial epicondyle epiphyseal fragment is pulled medially into the joint to lie in the region of the trochlea. This gives the appearance of the ‘C’, ‘R’ and ‘T’ but no ‘I’, and therefore can be identified as pathological.
813
A 7-year-old boy presents with a painless, 2 cm midline mass in the neck just below the hyoid bone. This moves superiorly on protru- sion of the tongue. Ultrasound scan shows a cystic lesion. What is the most likely diagnosis? a. branchial cleft cyst b. ectopic thyroid c. thyroglossal duct cyst d. obstructed laryngocele e. necrotic lymphadenopathy
c. Thyroglossal duct cyst is the commonest congenital neck mass. It presents as a painless midline neck lump, which moves superiorly on protruding the tongue. Imaging shows a smooth cystic lesion, which may take up pertechnetate on nuclear medicine studies due to the presence of functioning thyroid tissue. Ectopic thyroid is an important differential diagnosis, as this may be the only functioning thyroid tissue present and therefore should not be excised. Laryngoceles and branchial cleft cysts present with masses to the side of the neck rather than in the midline. Lymphadenopathy will usually present as solitary or multiple solid lumps in either side of the neck, but may be ‘cystic’ when necrotic.
814
What is the normal position of the duodenojejunal junction on frontal views during an upper gastrointestinal contrast examination? a. to the left of the left pedicles of the vertebral body at the level of the duodenal bulb b. to the left of the left pedicles below the level of the duodenal bulb c. overlying the vertebral body at the level of the duodenal bulb d. e. to the right of the right pedicles below the level of the duodenal bulb to the right of the right pedicles above the level of the duodenal bulb
a. Intestinal malrotation is defined as a congenital abnormal position of the bowel within the peritoneal cavity, occurring as a result of disruption of the normal embryological process of gut rotation and fixation. It is associated with abnormal bowel fixation by mesenteric bands or lack of fixation of parts of the bowel, which may result in obstruction, volvulus and bowel necrosis. The position of the duodenojejunal junction (and by implication the ligament of Treitz) can help to identify the presence of malrotation and its type, and should be determined on every paediatric upper gastrointestinal contrast examination. The duodenojejunal junction normally lies to the left of the left-sided pedicles of the vertebral body, at the level of the duodenal bulb on frontal views. Malrotation is associated with medial and inferior displacement of the duodenojejunal junction. Occasionally, the normal duodenojejunal junction may be displaced inferiorly by a distended stomach or dilated adjacent bowel segment, due to laxity of the peritoneal ligaments, which may mimic malrotation.
815
In hyaline membrane disease, which is the first feature usually seen on a chest radiograph in the initial stages? a. reduced lung volumes b. bilateral consolidation c. granularity in both lungs d. pleural effusions e. white-out of lungs
c. Hyaline membrane disease is due to deficiency of pulmonary surfactant, which causes alveolar collapse. Prematurity, caesarean section and perinatal asphyxia are predisposing factors. In the mild form, granularity is seen throughout the lungs as the first sign. As the condition progresses, air bronchograms appear with eventual complete opacification of the lungs. Changes are usually symmetrical if the condition is uncomplicated.
816
An 8-year-old boy has sudden-onset, severe, unilateral, testicular pain. What is the most likely cause? a. torsion of appendix testis b. torsion of appendix epididymis c. torsion of testis d. epididymitis e. orchitis
c. Torsion of the testicle is the commonest acute problem in the prepubertal age group. Including all ages below 20 years, epididymitis occurs in a ratio of 3:2 with torsion. This ratio is 9:1 above 20 years. Torsion of the testicular appendages accounts for around 5% of scrotal pathology overall, with the appendix testis being affected far more commonly than the appendix epididymis.
817
Of the choices below, which best describes the pattern of normal myelination of the brain in the first 9 months of life? a. cranial to caudal, posterior to anterior, deep to superficial b. cranial to caudal, anterior to posterior, deep to superficial c. cranial to caudal, posterior to anterior, superficial to deep d. caudal to cranial, posterior to anterior, deep to superficial e. caudal to cranial, anterior to posterior, superficial to deep
d. Myelination is an important feature of the maturation of the normal central nervous system. It is a dynamic process that begins in utero and continues after birth in a predetermined manner. It is well demonstrated on MRI, where initially white and grey matter show the reverse signal characteristics to those seen in the adult brain, with the white matter appearing of lower signal on T1 and higher signal on T2 than grey matter. As myelination occurs, the white matter gains fat content and so becomes of higher signal on T1 and lower signal on T2 than grey matter, with completion at around 9 months. The process progresses caudal to cranial, posterior to anterior, and deep to superficial, beginning with the brain stem and cerebellum, then the basal ganglia, with the final areas to mature being the peripheral cortical white matter.
818
Ultrasound scan of the abdomen of a newborn girl reveals an abdominopelvic cyst. It is thin walled and anechoic, and has a ‘daughter cyst’. Of the following which is the most likely diagnosis? a. Wilms’ tumour b. ovarian cyst c. cystic lymphatic malformation d. choledochal cyst e. cystic teratoma
b. Ovarian cysts in the newborn are more common than enteric duplication cysts, giant meconium pseudocysts, cystic lymphatic malformations or choledochal cysts. Other rarer causes of intra- abdominal cystic structures in the newborn include cystic teratomas, gastric teratomas, cystic granulosa cell tumour of the ovary, ovarian teratomas and cystadenomas. Ovarian cysts may become echogenic due to the haemorrhage that can occur if they tort (twist). These cysts also have associated normal ovarian tissue, and the daughter cyst represents a follicle along the wall. Wilms’ tumours are solid and occur later in life. A giant meconium pseudocyst has a thick echogenic wall, and viscous echogenic contents. It is formed by meconium leak following fetal bowel perforation due to intestinal obstruction in meconium ileus, ileal atresia or volvulus. Twenty-five per cent show peritoneal or cyst calcification, which is pathognomonic for meconium pseudocyst. Bowel obstruction may be present also. Cystic lymphatic malformations appear as large, well-circumscribed, cystic, thin-walled structures with multiple thin septa. Internally, the fluid can be echo free or echoic because of haemorrhage, debris, chyle or infection. Mesenteric, omental or retroperitoneal cysts are seen. Choledochal cysts are subhepatic or in the porta hepatis. They are seen separate from the gallbladder and are round, tubular or teardrop shaped, and connected to the biliary tree.
819
In a review of a paediatric skull radiograph that shows a generalized increase in bone density, which additional feature would suggest a diagnosis of pyknodysostosis rather than the more common osteopetrosis? a. premature closure of the anterior fontanelle b. multiple wormian bones c. insufficiency fractures d. sparing of the calvarium e. narrowing of the medulla
b. Causes of a generalized increase in bone density in childhood include osteopetrosis, pyknodysostosis and craniodiaphyseal dysplasia (in order of increasing rarity). Features of pyknodysostosis that allow differentiation from the other conditions are thick calvaria (spared in osteopetrosis), multiple wormian bones, and widened lambdoid sutures and fontanelles. Other manifestations include short limbs, mandibular hypoplasia, poor pneumatization of the paranasal sinuses, non- segmentation of C1 – 2 and L5 – S1, and clavicular dysplasia. This final feature has led to speculation that it is a variant of cleidocranial dysostosis. Insufficiency fractures may be seen in both conditions. Cortical expansion resulting in narrowing of the medulla is a feature of fluorosis.
820
A 1-month-old girl has liquid discharge from the umbilicus. Which of the following provides a suitable explanation? a. vesicourachal diverticulum b. urachal cyst c. patent urachus d. bladder exstrophy – epispadias complex e. cloacal exstrophy
c. Embryologically, the cloaca is divided by the urorectal septum into a dorsal part that develops into the rectum and a ventral part that gives rise to the allantois, bladder and urogenital sinus. The wolffian and mu¨llerian ducts drain into the ventral cloaca. The allantois becomes the urachus, which is the umbilical attachment of the bladder. Ordinarily, this atrophies to become the umbilical ligament. If it remains patent throughout its entire length, urine can drain via the umbilicus. A urachal sinus and a vesicourachal diverticulum describe patent portions of the urachus at the umbilical and bladder ends respectively.
821
A female neonate develops respiratory distress. A radiograph of the chest shows a narrow chest with very short ribs that do not extend beyond the anterior axillary line, platyspondyly with curved ver- tebral bodies and short curved humeri. What is the described dysplasia? a. thanatophoric dysplasia b. asphyxiating thoracic dysplasia c. camptomelic dysplasia d. achondroplasia e. Ellis – van Creveld syndrome
a. Thanatophoric dysplasia is transmitted by a dominant gene mutation, and is the commonest lethal neonatal skeletal dysplasia after osteogenesis imperfecta type II. Infants with this condition are frequently stillborn or die shortly after birth from respiratory failure. The appearances on the chest radiograph are pathognomonic, with short ribs that characteristically do not extend beyond the anterior axillary line. The vertebral bodies are curved into an H or U shape (best seen on a lateral radiograph), and the humeri are curved and short. Other skeletal abnormalities such as polydactyly may be present. Dysplasias associated with short ribs often present early in life with respiratory distress due to a critically small chest diameter.
822
Immature skeletons often break with a pattern that differs from the mature skeleton, with incomplete fractures and plastic bowing. What is the most important factor in the paediatric skeleton that accounts for this? a. malleable bones b. thick periosteum c. loosely tethered periosteum d. decreased bone mineral density e. partially mineralized osteoid
b. Paediatric fractures are commonly classified into five types: plastic deformation, buckle fracture, greenstick fracture, complete fracture and physeal injuries. The most important anatomical characteristics in the paediatric skeleton that result in these fracture patterns are the presence of growth plates and the thick periosteum, which in children contributes immensely to rapid fracture healing and helps in the reduction and the maintenance of reduction. It also decreases the likelihood that fractures will involve the entire circumference of the bone, creating torus and greenstick patterns, with the intact periosteum on one cortex preventing displacement of that cortex and acting as a hinge (particularly in greenstick patterns). This allows for accurate reduction with manipulation and retention with a cast, avoiding open reduction and internal fixation.
823
A neonate with Down’s syndrome presents with persistent bilious vomiting within a few hours of birth. An abdominal radiograph demonstrates a dilated, gas-filled stomach and duodenal bulb, with a total absence of gas in the small and large bowel. What is the most likely diagnosis? a. Hirschsprung’s disease b. hypertrophic pyloric stenosis c. duodenal atresia d. duodenal duplication cyst e. annular pancreas
c. Duodenal atresia is the commonest cause of congenital duodenal obstruction, and is often associated with Down’s syndrome and other congenital anomalies. Patients present soon after birth with persistent vomiting. The site of obstruction is usually distal to the ampulla of Vater, and a ‘double-bubble’ sign is seen on plain abdominal radiograph, representing gas within the duodenal bulb and stomach, but no gas is seen distally. Annular pancreas is also associated with Down’s syndrome, and may present neonatally with vomiting and the double- bubble sign, but obstruction is incomplete, with gas seen in the bowel distal to the stenosis. Hypertrophic pyloric stenosis usually presents with vomiting around 4 –6 weeks of life, and a dilated duodenal bulb is not a feature. Duodenal duplication cyst may cause compression or displacement of the first and second parts of the duodenum, but rarely causes high-grade stenosis. The double-bubble sign is not a feature of Hirschsprung’s disease.
824
At 6 months old, an infant with a cardiac murmur, learning disabil- ity, physical handicap, and abnormal facies is investigated after a urinary tract infection. Ultrasound scan reveals unilateral ureteric obstruction by an echo-bright focus in the bladder wall at the vesico- ureteric junction. On the same examination, a bladder diverticulum is also noticed. Choose the most likely unifying diagnosis. a. Marfan’s syndrome b. Maffucci’s syndrome c. Morquio’s syndrome d. Williams’ syndrome e. Mikity – Wilson syndrome
d. The features of idiopathic hypercalcaemia of infancy (Williams’ syndrome) include elfin-like facies, neonatal hypercalcaemia that can form renal stones, learning disability, physical retardation, colonic and bladder diverticula, aortic and pulmonary valve stenosis, cardiac septal defects, osteosclerosis and metastatic calcification. Marfan’s syndrome is an autosomal dominant, connective tissue disorder that, in addition to skeletal manifestations, may involve the cardiovascular system, particularly the mitral valve and ascending aorta. Morquio’s syndrome is the commonest of the mucopolysaccharidoses, with multiple skeletal manifestations occurring within the first 18 months of life. Wilson – Mikity syndrome is similar to bronchopulmonary dysplasia, occurring in normal preterm infants breathing room air, but is seldom seen now due to the use of mechanical ventilation. Maffucci’s syndrome is a dysplasia characterized by enchondromatosis and multiple soft-tissue haemangiomas. Learning disability and urinary tract abnormalities are not features of any of these conditions.
825
At a 20-week, antenatal ultrasound scan, a fetus has a renal pelvis diameter measured at 7 mm unilaterally. When the scan is repeated at 35 weeks, the renal pelvis measures 12 mm. Which of the following is the most appropriate follow-up for the neonate? a. discharge b. ultrasound scan at 6 weeks c. ultrasound scan within 24 hours of birth d. micturating cystourethrogram soon after birth e. 99mTc-labelled DMSA scan soon after birth
c. Pyelectasis can be regarded as a renal pelvis diameter greater than 4 – 5 mm at 20 weeks’ gestation or 7 mm at 33 weeks, or above 10 mm at birth. Local protocol varies as to which fetuses to follow up and how. Some centres perform an ultrasound scan on all neonates who have had a renal pelvis diameter of above 5 mm at any point; others may only investigate if there is a renal pelvis above 10 mm persisting to birth. Ultrasound scan soon after birth, however, is the best way of detecting severe obstructive pathology, such as posterior urethral valves, that may warrant rapid surgical intervention. On this scan, if there is persisting dilatation above 10 mm, antibiotic prophylaxis and micturating cystourethrogram (MCUG) are appropriate. If reflux is seen on the MCUG, DMSA and repeat ultrasound scan at 6 weeks are appropriate. DMSA is used to assess parenchyma for scarring. If no reflux is seen, MAG3 scan within 6 weeks is suggested to look for obstruction. If the renal pelvis diameter is less than 10 mm within 24 hours of birth, follow- up ultrasound scan 6 weeks later is suggested.
826
Which of the following findings on ultrasound examination of the hips in an 8-week-old child suggests the diagnosis of a concentri- cally located but unstable joint? a. multiple epiphyseal ossification centres b. alpha angle of 48˚ c. beta angle of 55˚ d. acetabular coverage of 50% e. inverted acetabular labrum
b. In performing hip ultrasound scan, the views should be taken in the coronal plane with the leg in internal rotation. The alpha angle refers to the angle between the iliac bone and the acetabular margin, and 60˚ or greater represents normal acetabular development. The femoral head should sit so that half of it lies within the acetabulum (equator sign). In poorly developed sockets seen in developmental dysplasia, the shallow acetabulum shows a reduced alpha angle of less than 49˚, which is suggestive of potential instability regardless of concentric or eccentric location of the head. The beta angle allows for sub-categorization of the hip sonographically, less than 55o being normal and over 70o suggestive of instability. An inverted labrum is seen in severe cases only.
827
A 10-year-old girl being treated for acute lymphoblastic leukaemia presents with a sudden onset of weakness. The leukocyte count is well below 100 000/mm3, and the platelet count is well below 10 000/mm3. Unenhanced CT brain is normal. Which of the follow- ing imaging techniques is most likely to demonstrate the cause of the signs? a. post-contrast CT brain b. CT venogram c. MR venogram d. DWI e. gradient echo MRI
e. In children with cancer, acute neurological symptoms may be caused directly by the cancer or by its treatment. Paediatric tumours commonly metastasizing to the brain include neuroblastoma, osteosarcoma and Ewing’s sarcoma. Rarer causes are melanoma, Wilms’ tumour and rhabdomyosarcoma. Of these, the sarcomas are most commonly associated with intratumoral bleeding. Spontaneous bleeding is seen in thrombocytopenia (platelets ,10 000/mm3) and disseminated intravascular coagulation. Large areas of haemorrhage will be evident on CT, but petechial haemorrhage is best seen on gradient echo MR. Ischaemia or infarction can be caused by non-bacterial thrombotic endocarditis, disseminated intravascular coagulation, septic infarction, tumour embolism, venous occlusion and radiation-induced vasculopathy. Radiation-induced vasculopathy can be acute (1 – 6 weeks) or early delayed (3 weeks to several months), and both are caused by oedema and transient myelin injury. All radiation injury is confined to the field of treatment. Late delayed reaction (several months to years) is accelerated atherosclerosis and white matter infarction consequent on this. Extensive demyelination, white matter necrosis and subsequent astrogliosis can also occur. With ischaemic damage, hyperintense signal may be seen on DWI as early as 1 hour. Hyperleukocytosis (leukocyte count >100 000/mm3) causes vascular thrombosis, into which there can be secondary haemorrhage. CT or MR venography can be used to look for venous occlusion, particularly venous sinus thrombosis.
828
An 18-month-old girl has a cough. A chest radiograph shows con- solidation in the right upper lobe. This is treated and a follow-up chest radiograph is performed, which shows a triangular-shaped mass arising from the mediastinum projecting over the right upper lobe. This has a rippled border. What is the most appropriate next investigation? a. CT of chest b. blood film c. MR of chest d. denatured red blood cell scintigraphy e. no further investigation
e. The appearance is of a normal thymus, which is visible in 50% of children up to 2 years of age. It is usually seen as a triangular density arising from the superior border of the mediastinum (‘sail sign’), which has a rippled border due to indentation by the ribs (‘wave sign’). The shape may change with position and respiration.
829
What is the commonest chest radiograph finding seen in tetralogy of Fallot? a. pulmonary plethora b. bulging right heart border c. concavity at the level of the pulmonary trunk d. elevated cardiac apex e. bulging of the superior left heart border
d. Tetralogy of Fallot accounts for 8% of congenital heart disease and consists of ventricular septal defect, overriding aorta, right ventricular hypertrophy and right ventricular outflow obstruction. On the chest radiograph, the most common finding is elevation of the cardiac apex due to right ventricular enlargement. This, with concavity at the level of the pulmonary trunk due to reduced pulmonary blood flow, produces the classic boot-shaped heart. A slight bulge in the upper left heart border may be seen due to the right ventricular infundibulum. Other associated features sometimes seen are aortic enlargement and right-sided aortic arch.
830
A 2-day-old, full-term, male neonate presents with failure to pass meconium. A water-soluble contrast enema demonstrates a dilated sigmoid colon with a narrowed, poorly distensible rectum and a persistent corrugated appearance to the rectal mucosa. What is the most likely diagnosis? a. colonic atresia b. meconium ileus c. imperforate anus d. Hirschsprung’s disease e. meconium plug syndrome
d. Hirschsprung’s disease is caused by an absence of parasympathetic ganglia in the muscle and submucosal layers of the distal colon, resulting in abnormal peristalsis and impaired evacuation of the colon. The rectum is virtually always involved, but the proximal extent of disease varies. Presentation is usually with failure to pass meconium by 48 hours. Characteristic features on contrast enema in the neonate include inversion of the rectosigmoid index (the normal neonatal rectum is of greater calibre than the sigmoid colon), tortuosity and corrugation of the narrowed aganglionic segment, and difficulty in obtaining good rectal distension. A discrete zone of transition is more commonly seen in older infants and children. Colonic atresia causes massive distension of the colon proximal to the area of stenosis, but is relatively rare. Meconium plug syndrome typically results in a dilated right and transverse colon with a transition point at the splenic flexure. Meconium ileus causes small bowel obstruction.
831
A 17-year-old girl presents with abdominal pain and rectal bleed- ing. She undergoes colonoscopy, which demonstrates multiple poly- poid lesions in the colon. Which feature would favour a diagnosis of juvenile polyposis rather than familial adenomatous polyposis? a. a total of 10 polyps in the colon b. a histological diagnosis of tubulovillous polyps c. involvement of the rectum d. mucocutaneous pigmentation e. a first-degree relative with multiple colonic polyps
a. In familial adenomatous polyposis (FAP), multiple (usually around 1000) tubular or tubulovillous adenomatous polyps are seen in the GI tract, predominantly in the colon. Patients usually become symptomatic in the third to fourth decades and present with abdominal pain, weight loss and diarrhoea. Juvenile polyposis (JP) is the commonest cause of colonic polyps in children, and usually presents with rectal bleeding. The polyps are hamartomatous and may occur throughout the GI tract. They are less numerous than in FAP, and the condition may be diagnosed with five or more polyps. Both conditions are autosomal dominant, with 80% penetrance in FAP and variable penetrance in JP. The rectosigmoid is involved in 80% of cases of JP, whereas the rectum is always involved in FAP. In both conditions, patients are at increased risk of associated adenocarcinoma, seen in 15% of patients by 35 years of age in JP, but in 100% of patients by 20 years after diagnosis in FAP. Mucocutaneous pigmentation is a feature of Peutz – Jeghers syndrome.
832
A 7 year old presents with back pain, mild fever and leukocytosis. A plain film shows collapse of a lower thoracic vertebra with increased density. There is no kyphosis and disc spaces are pre- served. The remaining bones are normal. What is the most likely diagnosis? a. osteomyelitis b. Ewing’s sarcoma c. eosinophilic granuloma d. steroid usage e. osteogenesis imperfecta
c. Eosinophilic granuloma is a benign variety of Langerhans’ cell histiocytosis. Fifty per cent of cases involve the skull and 25% affect the axial skeleton. It is the commonest cause of vertebra plana (collapsed dense vertebra) in children. The posterior elements are rarely involved. Ewing’s sarcoma may cause destruction of a vertebra, but vertebra plana is not a feature. The other conditions are all causes of vertebra plana but are less common in this age group.
833
A 14-year-old girl is admitted following trauma and undergoes CT of the abdomen and pelvis. The large bowel is incidentally noted to lie on the left side of the abdomen, with the small intestine on the right. The superior mesenteric artery lies to the right of the superior mesenteric vein. What is the most likely diagnosis? a. normal appearances b. non-rotation c. malrotation d. reversed rotation e. midgut volvulus
b. Non-rotation is an abnormality of bowel rotation in which the midgut loop returns to the peritoneal cavity without undergoing rotation. It is usually asymptomatic and is often found incidentally in older children or adults. The small bowel is located on the right side of the abdomen and the colon on the left side, and the small and large bowel have a common mesentery. The superior mesenteric vein lies to the left of the superior mesenteric artery, the reverse of the normal situation. In malrotation, the superior mesenteric vein tends to lie anterior to the superior mesenteric artery, and there is abnormal positioning of the duodenojejunal junction to the right and inferiorly, usually also with cephalad positioning of the caecum. Reversed rotation is rare, and results in the colon lying posterior to the superior mesenteric artery, with duodenum and jejunum in front of it. Midgut volvulus may complicate abnormalities of bowel rotation, resulting in torsion of the gut around the superior mesenteric artery due to a short mesenteric small bowel attachment. A clockwise ‘whirlpool’ sign may be seen on colour Doppler ultrasound scan, representing the superior mesenteric vein wrapping clockwise around the superior mesenteric artery.
834
A 12-year-old girl presents with altered sensation in the upper and lower limbs. Clinical assessment demonstrates weakness of the lower limbs with reduced pain and temperature sensation. MRI shows syringohydromyelia and tonsillar ectopia of 7 mm with the fourth ventricle in normal position. No myelomeningocele is seen. What is the most likely diagnosis? a. Chiari I malformation b. Chiari II (Arnold – Chiari) malformation c. Chiari III malformation d. Dandy – Walker syndrome e. diastematomyelia
a. Chiari I malformation is characterized by tonsillar ectopia. The fourth ventricle is elongated but normal in position. It is associated with syringohydromyelia, hydrocephalus and malformations of the skull base. Chiari II (Arnold – Chiari) malformation is characterized by hindbrain abnormalities, with caudal displacement of the fourth ventricle. A lumbar myelomeningocele is seen in over 95% of cases. Chiari III malformation is rare and has a high cervical/low occipital meningomyelo-encephalocele. Survival beyond infancy is unusual. In Dandy – Walker syndrome, there is enlargement of the posterior fossa with cystic dilatation of the fourth ventricle and abnormalities of the cerebellar vermis. Diastematomyelia results in sagittal splitting of the spinal cord into two hemi-cords, and is sometimes associated with a myelomeningocele.
835
Antenatal ultrasound scan performed at 31 weeks’ gestational age shows hydrops, for which there is no identifiable immunological cause. The scan also demonstrates cardiac enlargement and hydrocephalus. Which of the following is the most likely associated ultrasound finding? a. unilateral megalencephaly b. renal cysts c. aberrant right subclavian artery d. cerebral median tubular cystic space with high-velocity, colour Doppler flow e. retinal tumour
d. The unifying diagnosis is vein of Galen aneurysm. Three anatomical types are recognized, all of which are vascular malformations that dilate the vein of Galen, straight and transverse sinuses, and torcular herophili secondarily. Type 1 is an arteriovenous fistula, type 2 is an angiomatous malformation of the basal ganglia, thalami and midbrain, and type 3 has both features. It can be detected in utero or may present with a neonatal pattern of features (less than 1 month), an infantile pattern or an adult pattern (above 1 year). The in utero and neonatal manifestations are due to high-output cardiac failure and mass effect of the vein of Galen aneurysm, particularly on the aqueduct. It can undergo haemorrhage and cause infarction by a steal mechanism.
836
A 12-year-old boy is investigated for abdominal pain. On ultra- sound scan, there are large, echo-free, septated, cystic collections around both kidneys causing scalloping of the renal outline. On CT, these collections are close to water density. Which of the fol- lowing is the most likely diagnosis? a. renal lymphangiectasia b. bilateral Wilms’ tumours c. bilateral adrenal neuroblastoma d. bilateral hydronephrosis e. medullary sponge kidney
a. Renal lymphangiectasia is a very rare developmental malformation probably caused by a failure of the developing kidney lymphatics to establish communication with the extrarenal lymphatic system. Abnormal lymphatic channels dilate, resulting in cystic lesions in the parapelvic, perinephric and, less commonly, retroperitoneal regions. The lesions are of water attenuation on CT and may cause contour deformities of the renal outlines.
837
A 15-year-old female, whose father had progressive renal failure, presents with anaemia, polyuria and haematuria. On ultrasound scan, her kidneys are small and smooth. Which associated finding is most likely? a. pancreatic cysts b. posterior fossa haemangioblastoma c. cystocele d. nerve deafness e. hypertension
d. Alport’s syndrome or chronic hereditary nephritis is the unifying diagnosis. It is inherited, probably in an autosomal dominant fashion. Ocular abnormalities can also occur, including congenital cataracts, nystagmus, myopia and spherophakia. Hypertension is not a feature. The renal impairment is progressive in affected males but non-progressive in females. Cerebellar and retinal haemangioblastomas occur in von Hippel – Lindau syndrome, along with renal, pancreatic and adrenal cysts.
838
A neonate presents with respiratory distress shortly after birth. A chest radiograph shows hazy opacification in the left upper zone, which improves over the next 7 days, and the chest radiograph becomes normal. Two months later, the baby re-presents with res- piratory distress, and a chest radiograph shows hyperlucency in the left upper zone with contralateral mediastinal shift and com- pression of the adjacent left lower lobe. What is the likely diagnosis? a. congenital lobar emphysema b. Macleod’s syndrome c. bronchiolitis obliterans d. foreign body e. carcinoid
a. Congenital lobar emphysema results in progressive overinflation of one or more pulmonary lobes; it presents in the first 6 months of life with respiratory distress and cyanosis. The left upper lobe is most commonly affected. Imaging features immediately after birth are of a hazy, mass-like opacity, which represents delayed clearing of lung fluid in the emphysematous lobe. After clearing of the fluid, the affected lobe becomes expanded and hyperlucent on imaging, and causes mass effect and mediastinal shift. Macleod’s syndrome is a complication of bronchiolitis, usually presenting with hyperlucency of one or both lungs. Bronchiolitis obliterans usually occurs in adults, but may present in children following infection.
839
A full-term baby boy born to a diabetic mother presents during the first day of life with abdominal distension and failure to pass meconium. Plain abdominal radiograph demonstrates a dilated, meconium-filled, proximal colon to the level of the splenic flexure, with an empty descending colon, sigmoid and rectum. Administration of a water-soluble enema results in passage of meconium, and the baby is discharged well 2 days later. What is the most likely diagnosis? a. intussusception b. Hirschsprung’s disease c. meconium plug syndrome d. meconium ileus e. ileal atresia
c. Meconium plug syndrome is also known as small left colon syndrome and functional immaturity of the colon. It occurs most commonly in large infants and infants of diabetic mothers and is believed to be due to relative immaturity of bowel innervation. Characteristic features are of a dilated proximal colon with a calibre change at the splenic flexure and an empty descending colon. The functional obstruction is transient and usually self-limiting, and contrast enema may act as a stimulus for subsequent passage of (normal) meconium. Hirschsprung’s disease with a transition zone at the splenic flexure may mimic these findings, but the obstruction usually persists after contrast enema. Intussusception in this age group and at this site would be unusual. Meconium ileus and ileal atresia are causes of distal small bowel obstruction.
840
A 3-year-old boy presents with a cough of 1 week’s duration. There is no relevant preceding history. A chest radiograph shows lucency in the right lower zone, and an expiratory film demonstrates air trapping in the same region as well as mediastinal shift to the left. No masses are seen. What is the likely diagnosis? a. foreign body aspiration b. congenital lobar emphysema c. congenital cystic adenomatoid malformation type I d. bronchiolitis obliterans e. bronchogenic cyst
a. Foreign body aspiration is most common in children under 3 years of age. It presents with varying degrees of cough, and the chest radiograph usually shows overinflation, atelectasis, infiltrates and/or air trapping, which almost exclusively involve the lower lobes, with the right most commonly affected. A foreign body is seen in only about 9% of cases. Congenital lobar emphysema presents with increased lucency, usually in the upper zones, with air trapping and mediastinal shift. Cystic adenomatoid malformation type I usually presents as an expansile mass with multiple cysts. Bronchiolitis obliterans tends to present in adults, though it may present in children, usually with generalized hyperinflation and patchy air trapping.
841
A 7-year-old boy is admitted with colicky abdominal pain and rectal bleeding. He is noted to have a purpuric skin rash on the extensor surfaces of his arms and legs. What are the most likely findings on CT of the abdomen? a. multifocal bowel wall thickening b. multiple small polyps in the colon c. focal outpouching of the distal ileum d. inflammatory changes at the terminal ileum e. dilatation of the small bowel
a. Henoch – Scho ¨ nlein purpura is an acute systemic vasculitis that affects the skin, gastrointestinal tract, joints and kidneys. Children aged 3 – 10 years are predominantly affected, and may present with a purpuric rash on the extensor surfaces of the limbs, arthralgia and crampy abdominal pain with intestinal bleeding. Imaging demonstrates multifocal areas of bowel wall thickening due to intramural haemorrhage and oedema. Gastrointestinal complications include bowel infarction and perforation, as well as intussusception.
842
A 10-year-old boy is brought to accident and emergency after falling on his bike and sustaining a blunt abdominal injury by impacting the handlebars. CT shows free intraperitoneal fluid with a more focal collection of fluid at the mesenteric root. Additional findings of mesenteric stranding and focal bowel wall thickening are seen. There is diffuse homogeneous hyperattenuation of the pancreas, kidneys and bowel wall. What is the most likely underlying injury? a. mesenteric rupture b. bowel perforation c. splenic rupture d. renal contusion e. pancreatic contusion
a. The most common mechanism for traumatic mesenteric rupture is a road traffic collision, although such an injury can occur in children following any blunt abdominal trauma, with handlebar injuries and child abuse being well-recognized causes. Use of a lap belt makes children more prone to the injury. Bowel perforation, infarction and active haemorrhage are indications for immediate surgery and can be identified on CT. Most of the remaining cases are treated conservatively. Free intraperitoneal air, extraluminal contrast or visible bowel wall defect is specific for perforation. Free fluid, haemorrhage and stranding are seen in perforation or mesenteric injury, with fluid at the root more likely to indicate the latter. Hyperattenuation of the pancreas, kidneys and bowel wall is a feature of hypoperfusion, along with small-calibre aorta and vena cava, and should not be mistaken for visceral injury.
843
A 10 month old presents with recurrent chest infections, the most recent caused by Pseudomonas aeruginosa. The chest radiograph shows patchy infiltrates in the right base. Which feature on high- resolution CT would make the diagnosis of cystic fibrosis more likely than Williams – Campbell syndrome? a. cystic bronchiectasis b. emphysematous bullae c. mucus plugging d. lower lobe involvement e. pleural effusions
c. Cystic fibrosis usually presents in the first year of life with cough and recurrent infections or progressive respiratory insufficiency. The upper lobes are predominantly affected. High-resolution CT shows mucus plugging (tubular opacities), bronchiectasis, peribronchial thickening, bullous formation and collapse/consolidation. Williams – Campbell syndrome is a congenital deficiency of cartilage in the fourth to sixth generation bronchi. This produces cystic bronchiectatic changes beyond the third bronchial generation and bullous changes. Mucus plugging is not a feature.
844
A frontal radiograph of the pelvis in a child demonstrates flaring of the iliac wings with flattening of the acetabular roof and decreased acetabular and iliac angles giving an abnormal iliac index. There is associated elongation and tapering of the ischia. These are pelvic features of which condition? a. achondroplasia b. osteodysplasty c. Down’s syndrome d. mucopolysaccharidosis e. fibrous dysplasia
c. Down’s syndrome is a common genetic abnormality that results from trisomy of chromosome 21. The pelvic features in infantile Down’s syndrome were first described by Caffey. The iliac index (sum of the acetabular and iliac angles) is decreased in Down’s syndrome, and is particularly useful in making an early diagnosis. Flaring of the iliac wings due to their rotation into the coronal plane is a typical finding that gives them an increased prominence on frontal radiographs, with the appearance likened to Mickey Mouse or elephant ears. Down’s syndrome is associated with a wide array of skeletal abnormalities, with the skull, spine and appendicular skeleton all potentially involved.
845
During a routine new-baby check, a unilateral, firm, tense, non- pitting, parietal mass is noticed. Ultrasound scan demonstrates a crescent-shaped lesion adjacent to the outer table of the skull. The mass is most likely to be which of the following? a. caput succedaneum b. cephalocele c. cephalhaematoma d. leptomeningeal cyst e. fibrous dysplasia
c. Cephalhaematoma is seen with birth trauma, particularly following poor instrumentation and skull fracture during delivery. It is seen in 1 – 2% of deliveries. It can grow after birth and takes weeks or months to resolve. It does not cross sutural lines because the haematoma is beneath the outer layer of periosteum. The haematoma can calcify. Caput succedaneum is localized scalp oedema that does cross sutural lines. A cephalocele is a skull defect through which meninges, brain and cerebrospinal fluid may protrude.
846
Hyperechoic lesions are seen within the brain of a preterm neonate during a cranial ultrasound scan. Of the following, which most strongly suggests that the appearances are due to germinal matrix haemorrhage rather than periventricular leukomalacia? a. the baby is premature b. the hyperechoic changes are anterior to the caudothalamic groove c. the hyperechoic changes are adjacent to the trigone of the lateral ventricle d. cystic changes follow the acute phase in the affected brain e. there was no birth trauma
b. Germinal matrix is highly vascular tissue seen at 24 – 32 weeks’ gestational age, located anterior to the caudothalamic groove and inferior to the lateral ventricles. It is at risk of hypoxaemia and ischaemia. Haemorrhage here can be promoted by trauma at birth or coagulopathy including rhesus incompatibility. Germinal matrix haemorrhage less than 7 days old is hyperechoic but without shadowing. Within 2 – 3 weeks, the abnormal area decreases in size and the echogenicity reduces. Periventricular leukomalacia is white matter necrosis following hypoxaemia. It is seen in 5– 10% of preterm babies. It occurs in arterial watershed areas and may appear acutely as a broad region of periventricular increased echogenicity. Cystic degeneration may follow after 2 weeks or more.
847
On plain radiographs of paediatric long bones, osteomalacia may manifest as all but which of the following? a. Looser’s zones b. bowing c. insufficiency fracture d. generalized osteopenia e. increased trabeculae
e. Osteomalacia is bone softening due to accumulation of excessive amounts of uncalcified osteoid and poor bone mineralization of that osteoid, which in children is called rickets. As there is reduced mineralization, the bones may appear of diffusely reduced density on radiographs, which should be termed ‘osteopenia’ rather than ‘osteoporosis’, as the latter term refers to a reduced quantity of normally mineralized bone. There is also a decrease in the number of trabeculae, which appear fuzzy and coarse. The softening of the bone leads to insufficiency fractures and bowing deformities. Fractures may heal with unmineralized osteoid that gives the appearance on radiographs of persistent lucent lines called Looser’s zones. The underlying abnormality is a deficiency of vitamin D, which is required for normal bone mineralization. The two commonest causes of this are dietary and secondary to renal pathology (renal osteodystrophy).
848
Which of the following conditions is the most common underlying aetiology for intussusception in childhood? a. viral gastroenteritis b. Meckel’s diverticulum c. Henoch – Scho ¨ nlein purpura d. duplication cyst e. small bowel polyp
a. Most intussusceptions occur in childhood, usually between 6 months and 2 years of age. The vast majority are idiopathic or related to mucosal oedema and lymphoid hyperplasia following viral gastroenteritis. In the 5% of patients with a pathological lead point, causes include those listed above. Patients classically present with cramping abdominal pain, screaming episodes, ‘redcurrant jelly stools’ and a palpable abdominal mass. Most childhood intussusceptions are ileocolic. In contrast, a specific cause is identified in 80% of adult cases, and ileoileal intussceptions are more common than ileocolic.
849
A 15-year-old girl with a history of multiple febrile convulsions as an infant presents to the neurologist with complex partial seizures. The episodes start with special and somatosensory aura followed by a wide-eyed stare with behavioural arrest, before finally proceeding to a generalized tonic-clonic seizure. MR imaging is requested. Which sequence is best to evaluate the temporal lobes for signs of mesial temporal sclerosis? a. axial T1W b. axial T2W c. axial FLAIR d. coronal T1W e. coronal T2W
e. Mesial temporal sclerosis is a pattern of hippocampal neuronal loss that can occur with long-standing temporal lobe epilepsy as a result of excessive neuronal depolarization leading to cytotoxic oedema. Three patterns of loss are described, with relative sparing of the CA2 subfield often a feature. The typical findings are of asymmetrical atrophy of the hippocampus with abnormally high signal returned on T2W images. Ipsilateral findings in the limbic system include atrophy of the fornix and maxillary body. Cortical abnormalities may exist in the temporal cortex, and this is best evaluated on high-resolution T1 sequences.
850
A 14-year-old, overweight boy is seen in orthopaedic clinic with a 3-week history of left hip pain and reduced range of movement. Radiographs show mild proximal femoral osteopaenia, an ill- defined area of increased opacity in the proximal metaphysis, widening of the growth plate, and a small epiphysis. What is the most likely diagnosis? a. irritable hip b. Perthes’ disease c. slipped upper femoral epiphysis d. septic arthritis e. insufficiency fracture
c. Slipped upper femoral epiphysis is the most common hip abnormality in adolescents and is a frequent cause of premature osteoarthritis, pain and disability. The most commonly affected demographic are overweight teenage boys. Radiographs are still the initial investigation of choice, and recognition of early subtle signs allows earlier diagnosis and prompt treatment, improving long-term prognosis. The femoral head often slips posteriorly prior to slipping medially, and early slippage is best seen on lateral or frog lateral views. Early signs on the AP projection include widening of the physis with local demineralization. Posterior slippage of the femoral head may then give the appearance of a small epiphysis due to rotation and narrowing of the physis, and lateral views may show the displacement. The line of Klein is a line drawn along the lateral femoral neck, which should intersect with a small portion of the lateral femoral head. If it does not, it suggests medial slippage. An ill- defined area of increased opacification (metaphyseal blanch sign) may be seen in the proximal metaphysis, which is thought to represent a healing response.
851
An incidental finding made in a 13-year-old girl is of unilateral ovarian atrophy. The atrophic ovary has stippled calcification. Given these features, which is the most likely explanation? a. ovarian teratoma b. amputated ovary c. follicular ovarian cyst d. ovarian leiomyoma e. ovarian vein thrombosis
b. An amputated ovary occurs as the result of ovarian torsion and infarction. Both an ovarian teratoma and leiomyoma will enlarge the affected ovary rather than appear atrophic. A follicular cyst will usually be a simple cyst, although it may have internal echoes produced by haemorrhage.
852
A neonate is delivered following an uncomplicated pregnancy and presents with respiratory distress but no cyanosis. No resuscitation or ventilation is required. A chest radiograph shows a pneu- mothorax, which is treated by aspiration. What investigation should be considered? a. cranial ultrasound scan b. renal ultrasound scan c. abdominal radiograph d. barium swallow e. ascending urethrogram
b. Spontaneous pneumothorax may occur in babies where there are renal anomalies, and routine ultrasound scan is recommended. This is often associated with maternal oligohydramnios, but this may not necessarily be present.
853
Which of the following features on ultrasound scan is most suggestive of a horseshoe kidney? a. unilateral upper pole calyx dilatation b. unilateral lower pole calyx dilatation c. bilateral upper pole calyx dilatation d. laterally oriented renal long axis e. medially oriented renal long axis
e. Ninety per cent of horseshoe kidneys are joined at the lower poles by a parenchymal or fibrous isthmus. The isthmus lies at the L4 – 5 level, where renal ascent is arrested by the inferior mesenteric artery. The pelves and ureters are anterior, and pelviureteric junction obstruction is more common. Incidence is 1 – 4/1000 births, making it the commonest renal fusion abnormality. Cardiovascular, skeletal, central nervous system, anorectal and genitourinary malformations are associated. There are associations with trisomy 18 and Turner’s syndrome. Associated genitourinary anomalies include hypospadias, undescended testes, bicornuate uterus and ureteral duplication. The incidence of infection and stones increases in a horseshoe kidney.
854
What type of fracture is the most commonly seen in the presentation of non-accidental injury in a child? a. diaphyseal transverse b. metaphyseal corner c. posterior ribs d. skull vault e. vertebral body wedge
a. Non-accidental injury (NAI) is the third leading cause of death in children after sudden infant death and true accidents, and represents about 1% of all childhood trauma. The hallmark of NAI is multiple fractures at different sites and of different ages, suggesting repeated episodes of abuse. Often, as these fractures are not treated, they show exuberant callus formation. Several locations and patterns of injury have been described as being relatively specific for NAI. However, a study of over 400 fractures showed that the most common presentation was a single, simple, transverse fracture of the middle third of a long bone, which is indistinguishable from a fracture sustained in a true accident. Whole-body imaging with isotope bone scan is sensitive for identifying all sites of injury in equivocal cases.
855
A 30-week premature baby is delivered normally and shortly after birth develops tachypnoea and expiratory grunting. What feature on the chest radiograph would make respiratory distress syndrome of the newborn more likely than meconium aspiration syndrome? a. bilateral consolidation b. pneumothorax c. pleural effusion d. air bronchograms e. hyperinflation with air trapping
d. Respiratory distress syndrome of the newborn is the commonest cause of respiratory distress in premature neonates, and is due to relative immaturity of type II pneumocytes. Features seen on chest radiograph are reduced lung expansion, bilateral and symmetrical consolidation, and prominent air bronchograms. These resolve over several days. Meconium aspiration syndrome is the commonest cause of respiratory distress in full-term neonates. Hyperinflation, pneumothorax and pleural effusions are seen, as are diffuse patchy opacities due to atelectasis and consolidation. No air bronchograms are seen. Changes usually resolve in 48 hours.
856
Germinal matrix haemorrhage is identified on a cranial ultrasound scan of a neonate. Which of the following imaging features confers the worst prognosis? a. subependymal haemorrhage b. intraventricular haemorrhage c. intraventricular haemorrhage with ventricular dilatation d. intraparenchymal haemorrhage e. choroid plexus pulsation
d. Subependymal haemorrhage (grade 1) usually has no long-term consequence. Mortality rates for intraventricular haemorrhage, intraventricular haemorrhage with ventricular dilatation and intraparenchymal haemorrhage are 10%, 20% and more than 50%, respectively. These represent grades 2, 3 and 4 haemorrhage. Normal choroid plexus pulsates while haematoma at the same location is not pulsatile.
857
On antenatal ultrasound scan, oligohydramnios, non-visualization of the urinary bladder and bilateral enlarged hyperechoic kidneys are recorded. Renal failure ensues within the first month of life. An older sibling aged 5 years has established portal hypertension. The parents are phenotypically normal. What are the odds of further offspring of the same parents having the same disease? a. zero b. 1 in 5 c. 1 in 4 d. 1 in 2 e. unity
c. Infantile polycystic kidney disease is autosomal recessive. Both parents and half the children will be carriers. The sonographic hallmark is enlarged echogenic kidneys. The individual cysts are small, measuring 1 – 2 mm, and can be defined only with a high-resolution probe. Autosomal kidney disease affects the liver also. The less severe the renal disease, the more severe the hepatic periportal fibrosis.
858
A 3-week-old baby boy born at full term presents with persistent jaundice. Clinical examination demonstrates hepatomegaly, and blood tests confirm hyperbilirubinaemia. He undergoes pheno- barbital-augmented 99mTc-labelled IDA (iminodiacetic acid) cho- lescintigraphy, which demonstrates good hepatic activity within 5 minutes but no biliary clearance into the bowel on delayed images at 24 hours. What is the most likely diagnosis? a. biliary atresia b. neonatal hepatitis c. cystic fibrosis d. choledochal cyst e. physiological jaundice
a. Congenital biliary atresia is a destructive inflammatory process resulting in atresia of the bile ducts, which presents in neonates with persistent jaundice. Diagnosis is usually made with a combination of liver biopsy and hepatobiliary scintigraphy with a 99mTc-labelled iminodiacetic acid (IDA) analogue. Phenobarbital is given for 5 days prior to imaging to stimulate biliary secretion. Normal findings are of activity within the small bowel due to biliary excretion by 60 minutes, but in biliary atresia no biliary clearance is seen by 24 hours. Patients with non-obstructive causes of neonatal jaundice usually demonstrate biliary clearance into the bowel during the first 24 hours. Choledochal cyst usually presents in children, and cholescintigraphy may be used to confirm that the cystic structure is connected to the biliary system. The 99mTc-labelled IDA analogue will fill the cyst, and may show prolonged retention depending on the degree of obstruction. Cystic fibrosis may be associated with steatosis and biliary cirrhosis. Physiological jaundice resolves spontaneously within 2 weeks.
859
Which of the following sites is not typical of an avulsion fracture in a child? a. anterior inferior iliac spine b. ischial tuberosity c. tibial spine d. medial epicondyle e. radial head
e. Avulsion fractures occur in both adults and children due to a tensile force being applied through a tendon or ligament where the skeleton is the weakest link. The mechanism can be either a powerful normal contraction (as seen in sportspersons) or an abnormal force, as in anterior teardrop fractures of the cervical spine seen in road traffic collisions. They may be acute or chronic from repetitive minor trauma. In the paediatric skeleton, the spectrum of avulsion injuries is larger, as many of the growing apophyses are weaker than the soft-tissue structures that attach to them. For example, avulsion of the tibial spine in the knee is the paediatric equivalent of a ruptured anterior cruciate ligament, and it follows therefore that the ligament rupture is rarely seen in children. Fractures of the radial head in both adults and children are usually due to compressive forces, although one can see avulsion of the radial tuberosity by the biceps brachii tendon.
860
A 13-year-old male presents with recurrent epistaxis. CT shows a highly vascular mass in the nasopharynx, with widening of the pterygopalatine fossa and invasion of the sphenoid sinus. Which arterial branch is the feeding vessel likely to be arising from? a. ascending pharyngeal b. facial artery c. superficial temporal artery d. internal maxillary artery e. internal carotid artery
d. Juvenile angiofibromas are the commonest benign tumour of the nasopharynx and can grow to enormous sizes. They tend to present in teenagers with recurrent and severe epistaxis, as well as nasal obstruction. They are highly vascular and biopsy is contraindicated. In most cases, they are supplied primarily by the internal maxillary artery.
861
Radiographs of the lower limbs in a child show bilateral bowing of the femora and tibiae with genu varum. Additional findings of generalized osteopenia, deficient trabeculation, multiple fractures and cortical thinning suggest which underlying cause? a. rickets b. Blount’s disease c. osteogenesis imperfecta d. neurofibromatosis e. yaws
c. Osteogenesis imperfecta is one of the more common heritable connective tissue disorders and is characterized by micromelic dwarfism. In all four major types, findings include bowing of the long bones as a result of softening caused by osteomalacia and multiple fractures once the child is walking. Other findings include cortical thinning, exuberant callus formation and pseudarthroses. Rickets and Blount’s disease both show abnormal metaphyses, with rickets also demonstrating a coarse trabecular pattern. Neurofibromatosis causes bowing most commonly localized to the junction of the middle and distal thirds of the tibia, with associated sclerosis and cystic changes
862
A term neonate whose birth per vaginum was notably protracted has an MRI of the brain at age 4 days. There are changes in keeping with hypoxic ischaemic encephalopathy. Affected areas are marked by T1 shortening without T2 signal change. Which of the following is the most likely explanation for this pattern of abnormal signal? a. high T2 signal is masked by a high degree of myelination b. high T2 signal is masked by a low degree of myelination c. high T1 signal is accentuated by a high degree of myelination d. high T1 signal is accentuated by a low degree of myelination e. low T1 signal is masked by a high degree of myelination
b. Neonatal hypoxic ischaemic encephalopathy occurs in 1 – 2/1000 live births. Clinically, it manifests as disturbed neurological function such as difficulty with respiration, abnormal tone, depressed reflexes, altered level of consciousness, feeding difficulties or seizures. Initial MRI findings, particularly in the first week of life, can be T1 shortening (bright) rather than a high T2 signal. This is because the unmyelinated white matter is brighter on T2W images and can disguise pathological lesions. T1 shortening is thought to be due to lipid breakdown products of damaged myelin or to mineralization.
863
A 2-year-old boy presents with jaundice, abdominal pain and precocious puberty. Ultrasound scan of the abdomen reveals a heterogeneous, hyperechoic, 10 cm mass with areas of coarse calcification in the right lobe of the liver. What is the most likely diagnosis? a. hepatocellular carcinoma b. hepatoblastoma c. haemangioma d. pyogenic abscess e. choledochal cyst
b. Hepatoblastoma is the commonest malignant liver tumour of early childhood, typically presenting before 3 years with pain, palpable mass, jaundice and weight loss, as well as precocious puberty due to hormone production. Tumours are typically large (average 10 – 12 cm) and appear to be heterogeneous with areas of calcification and necrosis. Metastases to the lung are common. Hepatocellular carcinoma is the second most common malignant liver tumour of children, but usually affects children over 5 years of age. Although it also commonly appears of heterogeneous echogenicity, calcifications are rare. Haemangiomas commonly appear hyperechoic on ultrasound scan and may contain calcifications, but are usually ,4 cm and are rarely seen in young children. Pyogenic abscess and choledochal cyst are typically hypoechoic lesions.
864
Which of the following would be regarded as abnormal during ultrasound scan of the kidneys in a neonate? a. echogenicity of the renal cortex similar to that of liver b. large hyperechoic medullary pyramids c. paucity of renal sinus fat d. lobulated renal contour e. echogenic septum at anterosuperior margin of the kidney
b. The average length of the normal neonatal kidney is 4.5 cm. Ultrasound appearances of the kidney in the neonate and infant show a number of differences from those of the older child and adult. Cortical echogenicity is increased, as the glomeruli form 20% of the cortex in the neonate but only 9% in the adult, and may be comparable to that of the liver or the spleen. The medullary pyramids are larger and more hypoechoic than in older children and adults, and there may be little or no fat in the renal sinus. Persistent fetal lobulation may give the kidneys a lobulated contour. In addition, echogenic septa may be normally seen at the anterosuperior or posteroinferior margins of the kidney, representing the sites of fusion of metanephric elements. Most kidneys attain an adult pattern by 6 months of age.
865
From the following renal and adrenal findings, choose that which favours a diagnosis of neurofibromatosis type 1 above von Hippel – Lindau syndrome or tuberous sclerosis. a. renal cysts b. renal cell carcinoma c. phaeochromocytoma d. renal artery aneurysm e. angiomyolipoma
d. Renal artery stenosis and aneurysms plus abdominal coarctation are associated with neurofibromatosis type 1 (NF1). Renal cysts are found in von Hippel – Lindau syndrome (VHL) and tuberous sclerosis, while renal cell carcinoma is found in VHL. NF1 and VHL are associated with phaeochromocytoma. Angiomyolipomas, usually multiple and bilateral, are found in 50% of cases of tuberous sclerosis.
866
An 8-year-old boy with a 3-month history of increasing chest wall pain presents with a tender lump on the chest wall. Imaging of the chest shows a large inhomogeneous mass arising from a rib with a large intrathoracic component and preservation of tissue planes. There is associated rib destruction and a lamellated periosteal reaction is seen. What is the most likely diagnosis? a. Ewing’s sarcoma b. neuroblastoma c. Hodgkin’s disease d. osteomyelitis e. hamartoma of chest wall
a. Ewing’s sarcoma is the commonest malignant bone tumour in children, and the ribs are involved in 30% of cases in children under 10. It is the commonest malignant chest wall tumour. Neuroblastoma presents in children under 5 as a well-defined, soft-tissue mass that may calcify and erodes/splays the ribs. Hodgkin’s disease presents in young adults, and usually involves bones by secondary involvement with direct invasion of sternum or ribs. Osteomyelitis presents at any age with a shorter history (usually less than 2 weeks), and imaging shows rib destruction with a relatively small mass and loss of tissue planes. Hamartoma of the chest wall presents in the first year of life as an extrapleural mass, causing partial/complete destruction of adjacent ribs. Significant calcification and compression of the adjacent lung occur.
867
Which of the following injuries is one that is not suggestive of non-accidental injury in a 2-year-old boy? a. metaphyseal corner fracture b. lateral rib fractures c. posterior rib fracture d. spiral tibial fracture e. depressed occipital fracture
d. In ambulant children, spiral fractures of the tibia (toddler’s fractures) are seen reasonably commonly, often with no history of significant trauma, and are not by themselves suggestive of abuse. However, a spiral fracture in a non-ambulant child is quite suggestive of non-accidental injury (NAI). The metaphyseal corner fracture is the most specific of all the findings observed in NAI and is considered virtually pathognomonic of abuse. The injury consists of a series of microfractures orientated parallel to the physis, due to a shearing injury across the bone end. This mechanism of injury is not seen in blunt trauma or falls, although similar lesions may be seen in some metabolic diseases (particularly scurvy). Older children and adults may suffer rib fractures after trauma such as falls and road traffic accidents. However, they are unusual in infants and are strongly correlated with inflicted injury caused by chest wall compression.
868
An 8-year-old girl presents with back pain. Clinically, there is a double-curve scoliosis convex to the left in the thoracic region and to the right in the lumbar region. There is no focal neurology. MRI shows the conus behind the L3 vertebral body, and the filum terminale is 3 mm in thickness at the L5 – S1 level. A high-signal lesion is seen on T1W and T2W images in the canal behind the L5 and S1 vertebral bodies. What is the most likely diagnosis? a. diastematomyelia b. meningocele c. syringomyelia d. tethered cord e. developmental scoliosis
d. Tethering of the cord results in the conus lying lower than normal and is associated with scoliosis, thickening of the filum terminale (>2 mm at the L5 – S1 level on axial T1 image) and spinal lipoma. Less frequent associations are Chiari malformations, syrinx, myelomeningocele, diastematomyelia and dermal sinus. Diastematomyelia is a midline sagittal cleft in the cord, often with a bony/fibrous septum. Syringomyelia is dissection of cerebrospinal fluid through the cord, producing high T2 signal within the cord. This is associated with several neurological abnormalities. Developmental scoliosis occurs in adolescent girls, is convex to the right and has no associated neurological symptoms.
869
A 3-week-old girl presents with congestive cardiac failure and is found to have hepatomegaly on examination. Ultrasound scan of the abdomen shows an ill-defined, complex, heterogeneous, 5 cm mass in the right lobe of the liver containing multiple vascular channels on Doppler ultrasound scan. What is the most likely diagnosis? a. infantile haemangioendothelioma b. hepatoblastoma c. cavernous haemangioma d. mesenchymal hamartoma e. hepatocellular carcinoma
a. Infantile haemangioendothelioma is the commonest benign hepatic tumour occurring in the first 6 months of life, consisting of multiple sinusoidal vascular channels with surrounding connective tissue stroma. Patients typically present with an abdominal mass, or high-output cardiac failure secondary to arteriovenous shunting. Kasabach – Merritt syndrome (consumptive coagulopathy) is seen in 11% of patients. Typical CT features are of early peripheral enhancement and variable delayed central enhancement. Hepatoblastomas tend to occur in slightly older infants (peak 18 – 24 months) and congestive cardiac failure is not usually a feature. Mesenchymal hamartoma is a rare developmental liver tumour, which appears typically as a multiloculated cystic mass, but may appear solid and echogenic in infants due to innumerable microcysts. However, these are generally hypovascular lesions. Hepatocellular carcinoma is more commonly seen in older children and adolescents. Cavernous haemangiomas are rarely seen in young children.
870
A 15-year-old boy presents to accident and emergency with signs of meningitis but no pyrexia. CT and MRI show a retroclival midline cystic tumour with localized mass effect. Which feature would support the diagnosis of a dermoid rather than an epidermoid cyst? a. restricted diffusion on DWI b. no enhancement with intravenous gadolinium on MRI c. multiple septations on either modality d. focal low attenuation on CT e. well-defined calcifications on CT
d. Dermoid and epidermoid cysts are ectoderm-lined congenital inclusion cysts. They may not present until early adulthood due to slow growth (particularly epidermoids). Epidermoids contain only squamous epithelium whereas dermoids contain hair, sebaceous and sweat glands, and squamous epithelium. Unlike teratomas, neither is a true neoplasm. Clinical presentation is often with chemical meningitis from rupture of fatty contents. Epidermoids usually have imaging characteristics similar to water and can be differentiated from arachnoid cysts by restricted diffusion on DWI. Attenuation varies according to the keratin:cholesterol ratio and therefore can be similar to fat but is usually homogeneous. The cyst wall is very thin and is often not visible in epidermoids, and areas of calcification can infrequently be seen. Dermoids may appear more complex but are still unilocular. The wall may be thicker and calcification is more frequent. The sebaceous lipid material in a dermoid has attenuation and signal intensity characteristics of fat on CT (low attenuation) and MR (high signal on T1). ‘White epidermoids’ with high signal on T1 may be seen rarely; they are due to haemorrhage or a high fat content. However, the latter usually produces a homogeneous fat signal as opposed to dermoids, where the appearances are more heterogeneous due to the increased complexity of contents.
871
An 11-year-old boy presents with fever, vomiting and right lower quadrant pain. Ultrasound scan of the right lower quadrant demonstrates a blind-ending, compressible, fluid-filled, tubular structure measuring 4 mm in diameter, as well as enlarged mesen- teric lymph nodes and several thickened loops of small bowel. What is the most likely diagnosis? a. acute appendicitis b. mesenteric lymphadenitis c. Crohn’s disease d. Meckel’s diverticulum e. enteric duplication cyst
b. Mesenteric lymphadenitis is an inflammatory process affecting the mesenteric lymph nodes; it is most frequently caused by viral infection but also by Yersinia enterocolitica and other pathogens. It affects children and young adults, and clinically mimics acute appendicitis. Imaging findings are of enlarged mesenteric lymph nodes, and ileal and colonic wall thickening. The normal appendix must be visualized to differentiate it from acute appendicitis. Features of acute appendicitis on ultrasound scan are of a blind-ending, non-compressible, tubular, aperistaltic, fluid- filled structure measuring over 6 mm in diameter. Mesenteric lymph nodes may also be seen anterior to the right psoas muscle, but tend to be smaller and fewer than in mesenteric lymphadenitis. Thickened bowel wall may be seen on ultrasound scan in Crohn’s disease, but this would be less likely in this age group. Symptomatic Meckel’s diverticulum usually presents in younger children with gastrointestinal bleeding. Small bowel duplication cyst usually presents under 2 years of age, and lymphadenopathy is not a feature.
872
A 16-year-old female with a history of imperforate anus corrected soon after birth is investigated for primary amenorrhoea. Ultra- sound scan shows the uterus and upper vagina distended by moder- ately echogenic material. Which of the following is the most likely cause of the amenorrhoea? a. longitudinal vaginal septum b. transverse vaginal septum c. cervical dysgenesis d. obstructed uterine horn e. endometritis
b. Haematometrocolpos is described, which at this age can be due to a transverse vaginal septum or imperforate hymen. There is an association with imperforate anus, hydronephrosis, renal agenesis and dysplasia, polycystic kidneys, duplication of the vagina and uterus, sacral hypoplasia and oesophageal atresia. Cervical dysgenesis and obstructed uterine horn would produce haematometra.
873
An 11-year-old boy falls at school and lands on his outstretched arm. He complains of elbow pain and is taken to accident and emer- gency, where radiographs show no fracture but elevation of the anterior humeral fat pad and a visible posterior fat pad. What is the most likely occult injury? a. supracondylar fracture b. radial head fracture c. medial epicondyle avulsion d. lateral epicondyle avulsion e. articular chondral fracture
a. Elevation of the anterior humeral fat pad into a triangular shape (the sail sign) occurs secondary to elbow effusion. In the context of trauma, the probable cause is haemarthrosis. The posterior fat pad lies in the olecranon fossa and should not be visible on normal lateral radiographs in 90˚ of flexion, although it may normally be seen in extension, when it is displaced by the olecranon process. In trauma the posterior fat pad may be elevated out of the fossa by effusion, and is particularly valuable in predicting an intra-articular disease process when no bony abnormality is apparent radiographically. Rarely the fat pad sign may not be apparent, due to capsular rupture or surrounding soft-tissue swelling. In children, supracondylar fractures represent around 60% of elbow fractures; other causes include medial and lateral epicondyle injuries. The commonest adult elbow fracture is a fracture of the radial head or neck.
874
A 4-month-old boy presents with cyanosis. Examination reveals right ventricular heave and systolic murmur. A chest radiograph shows a bulging right heart border and widening of the superior mediastinum, creating a ‘snowman’ appearance. What is the most likely diagnosis? a. Fallot’s tetralogy b. total anomalous pulmonary venous return c. partial anomalous pulmonary venous return d. transposition of the great vessels e. coarctation of the aorta
b. Total anomalous pulmonary venous return presents in the first year of life with cyanosis. It is due to failure of the pulmonary veins to drain into the left atrium, with drainage instead into another vascular structure. There are four types – the commonest, type I (supracardiac), has the four pulmonary veins draining into one common vein, the vertical vein, which drains into the left brachiocephalic vein. This dilated vein, along with the dilated left brachiocephalic vein and superior vena cava, causes widening of the superior mediastinum, which is the ‘head’ of the ‘snowman’. The body is formed by enlargement of the right atrium, producing a rounded appearance of the lower mediastinum. There may be increased pulmonary vascularity. Partial anomalous pulmonary venous return presents later in life with symptoms similar to atrial septal defect. Transposition of the great vessels presents in the first 2 weeks of life with cyanosis. Chest radiograph shows an ‘egg-on-string’ appearance of the mediastinum with increased pulmonary vascularity. Infantile-type coarctation presents with symptoms and signs of congestive heart failure in infancy. The classic figure-3 sign seen in coarctation is often hidden by the thymus in infants.
875
A 9-year-old boy with leukaemia and severe neutropenia presents with right lower quadrant abdominal pain and bloody diarrhoea. CT of the abdomen demonstrates circumferential thickening of the caecum with decreased bowel wall attenuation and pericolonic inflammatory changes. What is the most likely diagnosis? a. appendicitis b. typhlitis c. leukaemic infiltration of the bowel d. diverticulitis e. Crohn’s disease
b. Typhlitis is acute inflammation of the caecum, appendix and occasionally terminal ileum, initially described in neutropenic children with leukaemia, but also seen with lymphoma, following immunosuppressive therapy and with clinical AIDS. Patients present with abdominal pain and diarrhoea, and may have a palpable, right lower quadrant mass. Characteristic findings are of circumferential caecal wall thickening, with oedematous bowel wall and inflammatory changes. Pericolonic fluid and intramural pneumatosis may be seen. Leukaemic deposits would be expected to cause more eccentric bowel wall thickening. Appendicitis may result in apical caecal wall thickening but would be accompanied by abnormal appendix. Crohn’s disease may produce a similar picture, but, in this clinical setting, typhlitis is the most likely diagnosis. Diverticulitis would be very unlikely in this age group.
876
A preterm male infant develops abdominal distension and blood-stained stools 2 days after birth. Plain abdominal radiograph shows distended and thickened bowel loops with curvilinear gas collections within the bowel wall. What is the most likely diagnosis? a. necrotizing enterocolitis b. Hirschsprung’s disease c. meconium ileus d. imperforate anus e. viral gastroenteritis
a. Necrotizing enterocolitis is an acute inflammatory bowel condition seen predominantly in preterm infants. It usually presents 2 – 3 days after birth, with abdominal distension, vomiting and blood-stained stools. Plain film signs include distended small and large bowel, and bowel wall thickening, but the hallmark of the condition is pneumatosis intestinalis, seen in 80% of cases. This may be curvilinear (subserosal) or bubbly (submucosal) and may also be associated with portal venous gas. Hirschsprung’s disease is characterized by an aganglionic segment of distal colon, resulting in abnormal peristalsis and impaired evacuation of the colon. In this age group, it presents with failure to pass meconium, but is extremely rare in preterm infants. Meconium ileus and imperforate anus also present with failure to pass meconium. Pneumatosis intestinalis may be seen in association with mechanical obstruction resulting from these three conditions, but it is rare. It is not a feature of viral gastroenteritis.
877
A 70-year-old woman presents with a sudden onset of right-sided hemiplegia and expressive dysphasia. She is otherwise well and has normal blood pressure on examination. Emergency CT shows a small subcortical acute haemorrhage in the inferior posterior left frontal lobe. Elsewhere, throughout the brain, there are smaller acute and subacute haemorrhages in the subcortex and a background of lacunar infarction. There is also marked brain atrophy in excess of that expected for the patient’s age. Changes consistent with diffuse leukoencephalopathy are also seen. What is the most likely underlying pathological condition? a. malignant hypertension b. acute disseminated encephalomyelitis c. neurosarcoidosis d. neuroamyloidosis e. haemorrhagic metastases
d. Cerebral amyloid angiopathy is characterized by deposition of b -amyloid protein in cortical, subcortical and leptomeningeal vessels. It usually occurs in sporadic form and increases in frequency and severity with increasing age. The condition produces a wide variety of clinical symptoms and varied imaging appearances. Many cases are asymptomatic but progressive neurological symptomatology and cognitive decline may be a feature. Chronic haemorrhage in a distinctive distribution, or catastrophic acute intracerebral/subarachnoid haemorrhage, may also occur. Amyloid angiopathy should be strongly considered in elderly normotensive patients with spontaneous intracranial haemorrhage, particularly when associated with leukoencephalopathy and atrophy related to small-vessel cerebrovascular disease. Definitive diagnosis is usually only made postmortem, with a presumptive diagnosis made on clinical presentation and imaging findings.
878
A child who undergoes MR of the brain for clinically apparent facial abnormalities is shown to have a defect of midline cleavage of the brain. What structure is abnormal or absent in all forms of holoprosencephaly, and therefore is the most sensitive indicator of a midline cleavage abnormality? a. falx cerebri b. third ventricle c. fourth ventricle d. corpus callosum e. septum pellucidum
e. Holoprosencephaly is failure of the primitive brain to cleave into two hemispheres, and is commonly associated with midline facial abnormalities (ranging from cyclopia to hypertelorism) and absence of many intracranial midline structures. There are three types, the most severe being the alobar form, which shows no cleavage at all, with absence of the falx cerebri and third ventricle, fusion of the cerebral hemispheres and thalami, and a single large lateral ventricle. The semilobar form has variable cleavage with a partially formed falx, rudimentary third ventricle, and variable cleavage of the thalami, lateral ventricles and cerebral hemispheres. In the lobar type of holoprosencephaly, brain formation may be nearly normal, but the septum pellucidum is always absent, as in all forms. The falx, corpus callosum and ventricular system may be normal in the lobar type.
879
A 37-year-old man with AIDS presents with confusion, lethargy and memory loss. CT of the brain demonstrates multiple supratentorial enhancing masses. Which imaging feature favours a diagnosis of toxoplasmosis rather than primary CNS lymphoma? a. subependymal distribution b. lesions hyperdense on unenhanced CT c. lesion size .3 cm d. hypovascularity on MR perfusion study e. increased uptake of thallium-201 on SPECT
d. Toxoplasmosis is the most common cause of a cerebral mass lesion in patients with AIDS. Typical appearances are of multiple, hypoattenuating, ,2 cm lesions with a predilection for the basal ganglia. Lymphoma is the second commonest mass lesion, with characteristic features including hyperdense lesions (though less frequently than in non-AIDS lymphoma) in a periventricular location with subependymal spread. Lesions in both conditions may show solid or ring enhancement. Haemorrhage is unusual in lymphoma but may be seen in toxoplasmosis, particularly following treatment. Thallium scanning may be useful to distinguish the two if there is diagnostic uncertainty. CNS lymphoma is thallium avid whereas toxoplasmosis does not show uptake. MR perfusion studies may also help to differentiate the two conditions. Lymphomas demonstrate increased perfusion relative to surrounding tissue, while toxoplasmosis is hypovascular. Differentiation is important, as early radiation therapy confers a significant survival advantage in CNS lymphoma.
880
An 80-year-old man presents acutely with a dense hemiplegia. CT perfusion is performed soon after admission, which suggests that the entire involved arterial territory is beyond recovery. Which of the following options represents the most likely combination of cerebral blood flow, mean transit time and cerebral blood volume, respectively, seen within the affected brain parenchyma, compared with unaffected parenchyma? a. increased, increased, increased b. increased, increased, decreased c. increased, decreased, decreased d. decreased, decreased, decreased e. decreased, increased, decreased
e. Cerebral perfusion CT can distinguish viable but ischaemic tissue (the penumbra) from tissue that is beyond recovery. Other uses include evaluation of vasospasm after subarachnoid haemorrhage, assessment of cerebrovascular reserve with acetazolamide (cerebral arteriole vasodilator) in cases of vascular stenosis, evaluation of collateral flow and cerebrovascular reserve in patients having temporary balloon occlusion and assessment of microvascular permeability of intracranial neoplasms. Cerebral perfusion CT utilizes the central volume principle. This states that CBF¼CBV/MTT, where CBF is cerebral blood flow, CBV is cerebral blood volume and MTT is mean transit time. In practice, two CT perfusion techniques can be used. One is perfused-blood- volume mapping, in which a quantity is assigned to cerebral blood volume by subtracting unenhanced CT data from CT angiographic data. It has the advantage of imaging the whole brain. The second technique is a dynamic, contrast-enhanced technique that acquires data from a limited number of axial slices, and monitors the first pass of an iodinated contrast agent bolus through the cerebral circulation. This requires an unenhanced CT brain, followed by a dynamic CT performed during injection of 50 ml of iodinated contrast (300 mg I/ml) at 4 ml/s. The first pass of contrast is observed in the brain. Cerebral perfusion is related to the concentration of iodinated contrast, which is directly related to the attenuation measured. Several maps are produced, including the CBV, CBF and MTT. MTT is derived from arterial and venous enhancement curves, measured by using regions of interest placed on an artery (one that is not occluded as part of an acute event) and a venous sinus. CBV is the area under the enhancement curves, and CBF is obtained from the central volume equation. Differentiation of infarcted brain from penumbra is important because, while penumbra can be saved by timely thrombolysis, infarcted tissue has an increased risk of bleeding from thrombolysis with no chance of recovery. CBF is decreased in both ischaemia and infarction, MTT is longer (.6 s) in both, while CBV is decreased in infarct but increased (or normal) in the penumbra due to cerebral autoregulatory mechanisms. MTT is the most sensitive for stroke. So this or CBF can be used to detect stroke while CBV is used to determine whether there is infarct or reversible ischaemia.
881
A 14-year-old boy with a recent history of viral upper respiratory tract infection presents with a 2-day history of rapid onset of fever and lethargy progressing to reduced consciousness and seiz- ures. MRI shows several large lesions in the cerebral white matter, and a diagnosis of acute disseminated encephalomyelitis is made. Which additional finding is associated with the most fulmi- nant course of this disease, with a median survival of only 6 days? a. grey matter involvement b. high signal in the CSF on FLAIR images c. generalized brain oedema d. areas of petechial haemorrhage e. enhancement with intravenous gadolinium
d. Acute disseminated encephalomyelitis is an acute autoimmune demyelinating disease of the central nervous system that affects children more commonly than adults. It is usually triggered by an inflammatory response to viral infections or vaccinations. The main symptoms are decreased level of consciousness, varying from lethargy to coma, convulsions, and multifocal neurological symptoms. MR imaging findings are of multifocal, large, confluent or punctate, high-signal, white matter lesions on T2W/FLAIR images. Lesions are responsive to treatment with steroids and in 80% of cases completely resolve. A haemorrhagic, hyperacute variant (acute haemorrhagic leukoencephalitis) has also been described and is known as Hurst’s disease. In addition to the usual findings, there are multiple, white matter haemorrhages and rapid development of profound generalized mass effect. Hurst’s disease is usually fatal within a few days.
882
Which of the following conditions will typically demonstrate unrestricted diffusion on MR DWI and ADC map? a. epidermoid cyst b. acute infarction c. cerebral abscess d. glioblastoma multiforme e. viral encephalitis
d. Diffusion-weighted MRI provides image contrast which is different from that provided by conventional MR techniques. The sequence enables the measurement of net macroscopic water movement, which is anisotropic (varies in different directions) particularly in white matter. Restricted diffusion is seen as high signal on DWI (which is a T2W image with signal degraded by diffusion) and low signal on the ADC map. Restricted diffusion occurs in tissue that does not allow free movement of water molecules, such as areas of infection due to the high viscosity and cellularity of pus. Similarly, epidermoid cysts are very cellular and so also show restricted diffusion, a feature that helps distinguish them from arachnoid cysts, which are fluid structures. In stroke, restriction in water diffusion occurs within minutes after the onset of ischaemia. The basis of this change is not completely clear but is thought to be related to the cytotoxic oedema seen in ischaemic cells due to the impairment of the Na+/K+ ATPase pumps (which are very energy dependent), leading to loss of ionic gradients and a net translocation of water from the extracellular to the intracellular compartment, where water mobility is relatively more restricted.
883
A 62-year-old man with a history of falls and confusion undergoes MR of the brain. This demonstrates a subdural haematoma of high signal intensity on T1W images and of high signal intensity with a hypointense rim on T2W images. What is the most likely age of the haematoma? a. ,6 hours b. 8 – 72 hours c. 3 days to 1 week d. 1 week to several months e. several months to several year
d. In the first 3 – 6 hours (hyperacute stage) following haemorrhage, the intact red cells contain mostly oxyhaemoglobin, which appears hyperintense on T2W images. Desaturation occurs peripherally, forming deoxyhaemoglobin, which is seen as hypointensity on T2W images. In the acute stage (8 – 72 hours), there is rapid deoxygenation of the oxyhaemoglobin to deoxyhaemoglobin, which, together with the high protein content of the clot and susceptibility effects, results in iso- to hypointensity on T1W images and hypointensity on T2W images. In the early subacute stage (3 – 7 days), oxidation of deoxyhaemoglobin to methaemoglobin occurs inside the red cell, resulting in characteristic hyperintensity on T1W images due to paramagnetic effects, and marked hypointensity on T2W images. In the late subacute stage (1 week to months), extracellular methaemoglobin results in persistent hyperintensity on T1W images, but increasing signal intensity on T2W images, with peripheral susceptibility effects causing a low intensity rim. In the chronic stage (months to years), iron atoms are deposited as haemosiderin and ferritin, which cause susceptibility effects resulting in low signal intensity on both T1W and T2W images.
884
Degenerative spinal vertebral body endplate changes, as seen on MRI, may have which of the following appearances? a. type I – high T1W and low T2W signal b. type I – low T1W and low T2W signal c. type II – high T1W and high T2W signal d. type II – low T1W and high T2W signal e. type II – high T1W and low T2W signal
c. The endplates in degenerative disc disease have three described appearances during their evolution, which are also known as Modic changes. Type I (marrow oedema) changes show low signal on T1W and high signal on T2W sequences. Type II (fatty marrow) changes show high signal on both T1W and T2W sequences. Type III (sclerosis) changes are low signal on both T1W and T2W sequences.
885
A 14-year-old boy presents with a grossly abnormal gait, kyphoscoliosis and upper limb tremors. MRI shows mild atrophy of the cerebellum and much more marked cervicomedullary junc- tion thinning with decreased anteroposterior diameter of the upper cervical spinal cord, which returns normal signal. What is the most likely diagnosis? a. Friedreich’s ataxia b. Creutzfeldt – Jakob disease c. olivopontocerebellar atrophy d. Huntington’s disease e. tuberous sclerosis
a. Loss of myelinated fibres and gliosis in the posterior and lateral columns of the spinal cord are the histopathological hallmarks of Friedreich’s ataxia. On MRI, the predominant radiological finding is thinning of the cervical spinal cord and medulla, and there may be associated mild cerebellar atrophy. Conversely, in the majority of other forms of ataxia such as early onset cerebellar ataxia and olivopontocerebellar atrophy, atrophy of the cerebellum and pons predominates with relative sparing of the cord. Although cerebellar atrophy may be seen in Friedreich’s ataxia, it is less pronounced than in these other diseases.
886
A 20-year-old man presents with gradual onset of neck pain and a painful lump in the upper neck posteriorly. Plain films show an apparent destructive lesion of the C2 vertebra. MRI shows a large lesion arising from the posterior elements of C2 and comprising multiple cysts with fluid –fluid levels, with preservation of the vertebral body. What is the most likely diagnosis? a. aneurysmal bone cyst b. giant cell tumour c. chordoma d. fibrous dysplasia e. telangiectatic osteosarcoma
a. Aneurysmal bone cysts are seen mainly in patients under 20 years of age (75%) and affect the posterior elements when involving the spine. They may arise de novo, or secondary to another lesion such as a giant cell tumour (GCT) or fibrous dysplasia. Both GCTs and telangiectatic osteosarcomas may cause cysts with fluid – fluid levels on MRI, but GCTs arise from vertebral bodies and usually occur in the sacrum. Telangiectatic osteosarcomas usually affect long bones. Chordomas are malignant tumours that usually affect the vertebral body, with destruction and invasion of the discs and adjacent structures.
887
A 46-year-old female presents with back pain and increasing weakness of the lower limbs. An MR scan shows a lesion in the cord at the level of T11. Which of the following features would suggest an ependymoma rather than demyelination? a. multiple lesions b. expansion of the cord c. high signal on T2W images d. enhancement with gadolinium e. peripheral low signal on all sequences
b. Ependymomas are the commonest tumour of the spinal cord in adults, accounting for 40 – 60% of cord tumours. They present with a long history of pain, and sensory or motor disturbance. Less commonly, bladder and bowel dysfunction may occur. Expansion of the cord is more often seen with ependymomas than with demyelination. Both lesions may enhance and have high signal on T2W images, but multiplicity is more often seen with demyelination. Peripheral low signal, usually indicating haemosiderin, is not a feature of either of these lesions.
888
A 35-year-old woman presents with progressive deafness and tinni- tus in the left ear. She undergoes MRI, which demonstrates a 2 cm mass at the left cerebellopontine angle. Which of the following fea- tures would favour a diagnosis of meningioma rather than vestibu- lar schwannoma? a. acute angle with the petrous bone b. hyperintensity on T2W images c. expansion of the internal auditory canal d. presence of a dural tail e. internal cystic degeneration and haemorrhage
d. The most common causes of a cerebellopontine angle mass are vestibular schwannoma (also called acoustic neuroma) (75%), meningioma (10%) and epidermoid cyst (5%). Features suggestive of a meningioma include a dural tail (thickening of enhancing adjacent dura resembling a tail extending from the mass), adjacent hyperostosis and an obtuse angle with the petrous bone (vestibular schwannomas make an acute angle). Distinguishing features of schwannomas include extension into the internal auditory canal, causing expansion of the canal and flaring of the porus acousticus (bony opening of the internal auditory canal). Meningiomas may show a small tongue of extension into the canal but usually no expansion. Schwannomas undergo cystic degeneration and haemorrhage more commonly than meningiomas (particularly larger lesions), and may show very high signal on T2W images, which is unusual for a meningioma.
889
A 64-year-old woman presents with progressive headache and con- fusion. On CT, she is found to have multiple, well-defined, rounded, low-attenuation masses of varying sizes in both hemispheres at the grey –white matter junction. The masses demonstrate intense enhancement following intravenous contrast, and there is consider- able surrounding oedema. Which of the following is the most appropriate next imaging investigation? a. mammography b. thyroid ultrasound scan c. barium enema d. renal ultrasound scan e. chest radiograph
e. Brain metastases are the most common intracranial tumours. Six primary tumours account for 95% of all brain metastases. Primary bronchial carcinoma is the most common (47% of cases), though squamous cell carcinoma rarely metastasizes to the brain. Other common primary tumours are breast carcinoma (17%), gastrointestinal malignancy (15%), renal cell carcinoma, melanoma and choriocarcinoma. Metastases characteristically occur at the grey – white matter junction, are multiple in 66% of cases, and typically appear as hypodense masses that demonstrate solid or ring enhancement.
890
A 78-year-old male patient presents with signs of acute lumbar myelopathy. Lumbar spine radiographs show collapse of the L3 vertebral body, which is encroaching upon the spinal canal. Additionally, throughout the remaining visualized vertebral bodies, there is diminished density of the central trabecular bone with relative preservation of the cortex and thickening of the mar- ginal trabeculations, which appear coarse. What is the most likely underlying condition of bone? a. Paget’s disease b. pyknodysostosis c. osteopetrosis d. Gaucher’s disease e. hyperparathyroidism
a. The described appearance is of ‘picture-frame’ vertebral bodies and is seen in Paget’s disease. The central osteoporosis can lead to insufficiency fractures and complications thereof. ‘Rugger jersey spine’ has a similar appearance due to sclerosis of the endplates (but the anterior and posterior cortices are spared) and is seen with both hyperparathyroidism (usually secondary and associated with renal failure) and osteopetrosis. Pyknodysostosis is a congenital disorder with sclerosis of long bones, causing obliteration of the medullary canal. Gaucher’s disease causes osteosclerosis secondary to bone infarcts, often causing H-shaped vertebra due to central endplate collapse.
891
A 52-year-old female presents with increasing short-term memory problems. CT and MR scans are normal. A perfusion scintigraphy study of the brain shows reduced perfusion in the posterior parietal regions bilaterally. What is the most likely diagnosis? a. Alzheimer’s disease b. vascular dementia c. Lewy body dementia d. frontotemporal dementia e. Parkinson’s disease
a. Alzheimer’s disease typically presents with atrophy and reduced perfusion to the mesiotemporal regions bilaterally, but, in younger patients, it often presents with posterior parietal perfusion abnormalities. Vascular dementia shows patchy cortical and subcortical perfusion defects, with involvement of the cerebellum. Lewy body dementia is similar to Alzheimer’s disease but with less occipital sparing; Parkinson’s dementia is also similar but with less mesiotemporal change and more involvement of the visual cortex.
892
A 22-year-old man presents with a 6-week history of progressive worsening confusion. Initial CT examination is normal. Following neurological review, the patient undergoes MRI, at which the only abnormality seen is high signal in the posterior portions of the thalami bilaterally on T2W and FLAIR images. What is the most likely condition? a. acute disseminated encephalomyelitis b. variant Creutzfeldt – Jakob disease c. multiple sclerosis d. metachromatic leukodystrophy e. Rasmussen’s encephalitis
b. Variant Creutzfeldt –Jakob disease is a rare but important, transmissible, rapidly progressive spongiform encephalopathy and a cause of dementia and death in young patients. The transmissible agent is thought to be a prion protein (proteins devoid of nucleic acid), and the condition is causally linked to bovine spongiform encephalopathy. CT is typically normal, but symmetrical hyperintensity in the pulvinar nuclei of the posterior thalami (pulvinar sign) on T2W or FLAIR MR images has been described as a specific sign for the variant form. The classic form affects an older age group and is often genetically linked, whereas the variant form is sporadic. The pulvinar sign is seen only in the variant form, with the classic type typically showing similar hyperintense abnormalities located in the caudate nucleus and putamen.
893
A 60-year-old female admitted with severe, sudden-onset headache is found to have widely distributed subarachnoid haemorrhage. A saccular aneurysm is identified on CT angiography. From which of the following locations in the circle of Willis is this aneurysm most likely to arise? a. basilar tip b. middle cerebral artery bifurcation c. junction of anterior cerebral and anterior communicating arteries d. pericallosal artery e. vertebral artery
c. It is at this location that 35% of berry aneurysms are found. This is the same proportion that occurs at the junction of the internal carotid and posterior communicating arteries. Five per cent occur at the basilar tip and 20% at the middle cerebral artery bifurcation. Two per cent of the population have cerebral aneurysms. They are multiple in 20% of cases and giant in 25% (over 25 mm in diameter). They are caused by degenerative vascular changes, trauma, infection, tumour and vasculopathy. The incidence is increased in adult polycystic renal disease, aortic coarctation, fibromuscular dysplasia, and Marfan’s and Ehlers –Danlos syndromes.
894
A 43-year-old man presents with cough and numerous masses in the neck bilaterally. CT confirms multiple lymph nodes that enhance peripherally and contain areas of calcification. A cavitating lesion is noted in the right lung apex. What is the most likely diagnosis? a. tuberculosis b. metastatic laryngeal carcinoma c. metastatic nasopharyngeal carcinoma d. metastatic papillary thyroid carcinoma e. metastatic squamous cell carcinoma of the lung
a. Tuberculous lymphadenitis is the most common form of head and neck tuberculosis, representing 15% of all extrapulmonary tuberculous infections. It is frequently bilateral and, the more inferior the involved nodes, the higher the prevalence of associated pulmonary disease. Peripherally enhancing lymph nodes are seen with tuberculosis, metastatic disease (usually squamous cell tumours), lymphoma or infection. The presence of calcification suggests tuberculosis, but it may also be seen with papillary or medullary thyroid carcinoma. The other diagnoses listed may cause cervical lymphadenopathy, but the lung lesion would be unusual in all except a cavitating squamous cell carcinoma of the lung.
895
With ageing, iron accumulates in tissues of the brain and is seen as low signal on both T1W and T2W sequences at MRI. Which of the following locations for iron deposition is pathological in a 60-year- old man and may indicate onset of dementia? a. putamen b. globus pallidus c. red nucleus d. dentate nucleus e. pars reticulata
a. Non-haem brain iron is normally found within oligodendroglia and astrocytes, with smaller amounts in neurons and myelinated axons. Half is found within the mitochondria and 10% in the nuclei, and 40% represents a soluble fraction. The areas of maximum iron concentration in normal adults are found in the globus pallidus, red nucleus, pars reticulata of the substantia nigra and dentate nucleus of the cerebellum. The brain concentration is independent of general body stores, and the mechanism by which it crosses the blood – brain barrier is not fully understood. By the eighth decade, the concentration of iron may increase in the caudate nucleus and putamen to levels similar to those seen in the globus pallidus. Excessive or premature accumulation in these areas has been seen in various forms of senile dementia including Alzheimer’s disease. Iron deposition is identified on T1W and T2W sequences as signal hypointensity due to magnetic susceptibility.
896
An 85-year-old female presents with low back pain and is tender over the sacrum. Radiographs show a generalized reduction in bone density. MRI demonstrates symmetrical, bilateral, linear areas of signal abnormality in the sacral alae, which are low signal on T1W and high signal on STIR images. There is no history of trauma, but the patient does have a history of radiother- apy for cervical carcinoma many years previously. What is the most likely diagnosis? a. insufficiency fracture b. metastases c. ankylosing spondylitis d. Reiter’s syndrome e. post-radiotherapy change
a. Insufficiency fractures tend to occur in elderly female patients and manifest with pain. They are notoriously difficult to diagnose on plain film due to overlying soft tissues and bowel gas. MR and bone scans demonstrate them well, often in an ‘H’ configuration (Honda sign). Metastases tend to cause multiple focal areas of change. Ankylosing spondylitis and Reiter’s syndrome are known causes of sacroiliitis, which would cause changes on MRI more localized to the iliac sides of the sacroiliac joints. Post-radiotherapy changes tend to be those of fatty marrow replacement, with high signal on both T1W and T2W, and low signal on STIR images.
897
A 5-year-old boy undergoes CT of the brain for investigation of headaches, vomiting and ataxia. This demonstrates a well- defined, multilobulated, isodense mass within the fourth ventricle containing areas of punctate calcification. The mass is seen to extend out of the foramina of Luschka into the cerebellopontine angles. There is associated hydrocephalus. What is the most likely diagnosis? a. metastasis b. haemangioblastoma c. juvenile pilocytic astrocytoma d. medulloblastoma e. ependymoma
e. Ependymomas most commonly arise in the floor of the fourth ventricle and are usually isodense. They have a greater incidence of calcification than other posterior fossa paediatric tumours; it is typically punctate and seen in 40 – 50% of cases. A characteristic feature of ependymomas is their propensity to extend through and widen the foramina of Luschka and Magendie. Juvenile pilocytic astrocytomas are the commonest paediatric infratentorial neoplasms and typically occur in the cerebellar hemispheres. They appear cystic with an enhancing mural nodule. Medulloblastomas tend to be homogeneous hyperdense lesions located in the vermis, and the presence of calcification is uncommon. Metastases are the commonest infratentorial tumour to occur in adults, but are uncommon in children. Haemangioblastomas usually occur in young adults and are classically cystic masses with a solid mural nodule.
898
An 18-month-old toddler presents with progressive gait disturb- ance, motor developmental delay and muscle weakness. Biochemi- cal tests show abnormally low levels of arylsulphatase A enzyme in peripheral blood leukocytes and urine. MRI demonstrates sym- metrical, confluent areas of high signal on T2W images affecting the periventricular and cerebellar white matter, corpus callosum and corticospinal tracts with sparing of the subcortical U fibres. The periventricular abnormality shows a striped ‘tigroid’ pattern. There is no enhancement with administration of intravenous gadolinium. What is the most likely condition? a. Alexander’s disease b. Canavan’s disease c. acute disseminated encephalomyelitis d. mucopolysaccharidosis e. metachromatic leukodystrophy
e. Dysmyelinating diseases (leukodystrophies) are a wide spectrum of inherited neurodegenerative disorders affecting myelin in the brain and peripheral nerves. Most fall into one of three categories: lysosomal storage diseases, peroxisomal disorders and diseases caused by mitochondrial dysfunction. The most common, metachromatic leukodystrophy, is an autosomal recessive disorder caused by a deficiency of the lysosomal enzyme arylsulphatase A, which leads to accumulation of sulphatides in tissues. It usually manifests as a late infantile subtype in children at 12 – 18 months of age, and is characterized by motor signs of peripheral neuropathy followed by deterioration in intellect, speech and coordination, leading to death within a few years. On T2W MRI, symmetrical confluent areas of high signal intensity are seen in the periventricular white matter with sparing of the subcortical U fibres. Sparing of the perivascular white matter gives a striped or tigroid appearance to the periventricular area of abnormality, particularly well seen in the centrum semiovale. Other sites often affected are the corpus callosum, internal capsule and corticospinal tracts.
899
An elderly male patient presents with signs suggesting acute middle cerebral artery infarction. Around 21 2 hours after symptom onset, an unenhanced CT of the brain is performed. Among other subtle signs, the basal ganglia are obscured. Reduced perfusion through which of the following vessels best explains this sign? a. lenticulostriate arteries b. anterior choroidal artery c. callosomarginal artery d. recurrent artery of Heubner e. angular artery
a. The lenticulostriate arteries are vessels arising from the M1 segment of the middle cerebral artery; there are medial and lateral groups. Collectively, they supply the thalamus, caudate and lentiform nuclei. The callosomarginal artery and the recurrent artery of Heubner are anterior cerebral artery branches. The latter provides some supply to the anterior limb of the internal capsule, and parts of the caudate nucleus and globus pallidus. The angular artery is a cortical branch of the middle cerebral artery. The anterior choroidal artery also supplies parts of the internal capsule and basal ganglia but is a branch of the internal carotid artery. The nuclei of the basal ganglia are the amygdala, claustrum, lentiform and caudate nuclei, with the internal, external and extreme capsules being associated white matter tracts.
900
A 39-year-old man known to have HIV and with clinically deteriorating dementia undergoes MRI of the brain with a provi- sional diagnosis of HIV encephalopathy. Images show abnormal high signal on T2W and FLAIR images affecting the white matter, caudate nucleus and basal ganglia on a background of brain atrophy. Which additional MR characteristic of these lesions is likely to suggest progressive multifocal leukoencephalo- pathy over HIV encephalopathy? a. frontal white matter preponderance b. clustering around the basal ganglia c. involvement of subcortical U fibres d. areas of haemorrhagic necrosis e. enhancement with intravenous gadolinium
c. HIV encephalopathy is a progressive subcortical dementia caused by direct infection of the central nervous system by HIV. It is therefore not an opportunistic infection and is seen in up to 60% of AIDS cases. The most striking feature is cerebral atrophy and diffuse myelin pallor, manifested as ill-defined, confluent areas of high signal in the white matter on T2W sequences or low attenuation on CT. Changes are usually bilateral but asymmetrical, with characteristic sparing of the grey matter. Progressive multifocal leukoencephalopathy occurs in only 2 – 4% of AIDS cases and is a reactivation of the JC (John Cunningham) virus, resulting in destruction of oligodendrocytes. The most common location is posterior whereas HIV encephalopathy is more commonly frontal; however, this is an unreliable discriminator. Progressive multifocal leukoencephalopathy characteristically involves the subcortical U fibres and also affects grey matter. It has a very poor prognosis, death occurring within 2 –5 months.
901
Which of the following MR sequences is most sensitive for detecting subcortical and periventricular lesions in multiple sclerosis? a. axial T1W b. axial T2W c. axial FLAIR d. axial proton density e. axial T1W post-gadolinium contrast
c. The FLAIR MR sequence is a heavily T2-weighted inversion recovery sequence designed to nullify the signal from CSF. This highlights any T2 high-signal lesions that may be masked on standard T2W images by adjacent high signal of CSF. This is of particular benefit in the periventricular region and corpus callosum, and studies have shown that FLAIR has higher accuracy than intermediate and T2W sequences in the detection of supratentorial cortical, subcortical and periventricular multiple sclerosis lesions. Depending on location, some lesions can be best seen on standard T2W images; these areas include the posterior fossa, brain stem and spinal cord.
902
A 65-year-old, previously well man with a short history of head- aches and behavioural change undergoes CT of the brain. This demonstrates an irregular, ill-defined mass in the left frontal lobe extending across the corpus callosum to involve the right frontal lobe. The mass is of low attenuation and contains cystic areas, demonstrates ring enhancement following intravenous contrast, and has considerable surrounding oedema. What is the most likely diagnosis? a. progressive multifocal leukoencephalopathy b. glioblastoma multiforme c. lymphoma d. abscess e. metastasis
b. Glioblastoma multiforme is the most malignant form of astrocytoma. It occurs in older patients, and most commonly affects the deep white matter of the frontal lobes. Classic appearances are of an irregular, ill- defined hypodense mass with necrosis, haemorrhage and extensive surrounding white matter oedema. Ninety per cent of cases show enhancement, which may be diffuse, heterogeneous or ring like. Tumour spread is directly along white matter tracts, and commonly occurs across the corpus callosum to involve both frontal lobes (butterfly glioma). Lymphomas also have a propensity to involve the corpus callosum but usually are slightly hyperdense due to a high nuclear- to-cytoplasmic ratio. Metastases may also involve the corpus callosum but tend to be better defined and would be less likely in the absence of a known primary tumour. Progressive multifocal leukoencephalopathy may involve the corpus callosum but occurs in immunocompromised patients. Involvement of the corpus callosum is not usually a feature of abscesses
903
A 35-year-old man is involved in a low-velocity road traffic acci- dent. Within minutes, he experiences an occipital headache and neck pain. On arrival in hospital, he complains of nausea, vertigo and diplopia. An unenhanced CT scan of the brain is performed. Which of the following abnormal findings is most likely? a. high density seen in CSF of the sylvian fissure b. low density and loss of grey – white matter differentiation in the insular region c. expansion of a vertebral artery with a peripheral, high-density crescent d. lenticular high attenuation between the temporal lobe and temporal bone e. crescent-shaped high attenuation between the temporal lobe and temporal bone
c. Minor trauma stretching the vertebral artery over the lateral mass of C2 can cause vertebral artery dissection. Symptoms include headache and neck pain, and as many as 95% of patients develop a stroke after hours to weeks. Imaging will show an axially enlarged vessel with a narrow lumen and a periarterial rim sign. Angiography may demonstrate tapering or occlusion of the artery or the dissection flap. Predisposing factors to spontaneous arterial dissection include fibromuscular dysplasia, Marfan’s syndrome, collagen vascular disease and homocysteinuria.
904
A 70-year-old man has severe rhinorrhoea and then develops a cough with haemoptysis. A chest radiograph shows a large nodule in the right lung, and a subsequent CT thorax demonstrates a cavi- tary, left-lung nodule. A cerebral catheter angiogram, undertaken to investigate a focal motor deficit, is most likely to reveal which of the following? a. a giant berry aneurysm b. multiple aneurysms with stenoses and occlusions c. dural arteriovenous malformation d. contrast extravasation e. numerous collaterals supplying the anterior circle of Willis
b. Nasal and paranasal involvement and migratory lung nodules, which can be cavitary, are typical features of Wegener’s granulomatosis. Cerebral vasculitis can also be a feature. MRI findings are non-specific and include hyperintensities on T2W images, infarcts and haemorrhage. Angiography may demonstrate occlusion, stenoses and aneurysms.
905
An elderly man undergoes CT for gradual onset of confusion and memory loss. A pattern of generalized global brain atrophy showing marked temporal lobe predominance with an increase in size of the hippocampal – choroidal fissure is seen. Coronal T2W images show atrophy of the hippocampus bilaterally. Which con- dition is most likely to result in such a pattern of volume loss? a. Pick’s disease b. Alzheimer’s disease c. Parkinson’s disease d. Lewy body dementia e. multi-infarct dementia
b. Alzheimer’s disease is a progressive neurodegenerative disorder and the most common cause of dementia in elderly people. Current thinking is that imaging can help in the early diagnosis by documenting or quantifying atrophy in certain regions of the brain such as the hippocampus and entorhinal cortex. However, conventional structural imaging has a relatively low sensitivity in Alzheimer’s disease and may be normal. Therefore, use is limited to identification of patterns of atrophy and the exclusion of other potential causes of dementia such as normal pressure hydrocephalus, vascular dementia or space-occupying lesion. Patterns of atrophy, when seen, may be relatively specific for different causes of dementia, with hippocampal atrophy a cardinal radiological sign of Alzheimer’s disease. Pick’s disease has a frontal and temporal preponderance and is part of the rarer group of diseases under the umbrella term ‘frontotemporal lobar degeneration’. Lewy body dementia typically affects the parietal and occipital lobes and the cerebellum, which are usually spared in Alzheimer’s disease. Multi- infarct dementia is vascular in origin and will show a patchy distribution of change. The different patterns of atrophy have also been recently demonstrated with 18FDG PET.
906
A 21 year old presents with back pain, increasing over time. There are no neurological symptoms. A radiograph of the lumbar spine shows a grade II spondylolisthesis at L5 – S1. Which of the following features would suggest the presence of bilateral spondylolysis as the cause? a. narrowing of AP diameter of spinal canal at L5 – S1 b. widening of AP diameter of spinal canal at L5 – S1 c. lucencies through the laminae of L5 d. sclerosis of the pedicles of L5 e. reduced height of the L5 – S1 disc
b. Spondylolysis (pars defect) is seen in 3 – 7% of the population, with 50% being symptomatic. L5 is the most commonly affected level. There are fractures through the pars interarticularis, which may be unilateral or bilateral. Spondylolisthesis can occur only when pars defects are bilateral. The AP diameter of the canal is widened, as the vertebral body and pedicles are detached from the posterior elements and migrate anteriorly. Narrowing of the canal is seen with other causes of spondylolisthesis, particularly degenerative causes. Sclerosis of one pedicle is seen with unilateral pars defects, as the contralateral pedicle undergoes reactive sclerosis due to excessive stress.
907
A 17-year-old boy presents with headache and is found to have paralysis of upward gaze (Parinaud’s syndrome) on examination. MR scan of the brain identifies an abnormality. What is the most likely site of the lesion? a. thalamus b. occipital lobe c. optic chiasm d. pineal gland e. cerebellar vermis
d. Parinaud’s syndrome (also known as dorsal midbrain syndrome) is characterized by supranuclear paralysis of upward gaze. It results from injury or compression of the dorsal midbrain, in particular the superior colliculi, and is most commonly seen in young patients with tumours of the pineal gland or midbrain, with pineal germinoma being the most common lesion producing the syndrome. Young women with multiple sclerosis and elderly patients with brain-stem stroke may also present with Parinaud’s syndrome.
908
A 28-year-old woman presents with a history of headaches and refractory temporal lobe epilepsy. CT of the brain demonstrates a mixed solid – cystic, intraparenchymal mass located peripherally in the right temporal lobe, which contains calcification and demon- strates faint enhancement following intravenous contrast. There is minimal surrounding oedema. What is the most likely diagnosis? a. arachnoid cyst b. ganglioglioma c. epidermoid d. meningioma e. dysembryoplastic neuroepithelial tumour
b. Gangliogliomas are low-grade tumours with a good prognosis, generally occurring in patients under the age of 30. Typical presentation is with focal seizures, and ganglioglioma is the most common tumour seen in patients with chronic temporal lobe epilepsy. They are usually well- circumscribed, hypo- or isodense lesions in the temporal lobes. Calcification (30%) and cyst formation (.50%) are common features. There is usually minimal mass effect and surrounding oedema. Meningiomas commonly calcify and have minimal surrounding oedema, but are extra-axial, and usually demonstrate intense uniform enhancement following intravenous contrast. Dysembryoplastic neuroepithelial tumours are commonly associated with partial complex seizures, but usually occur before the age of 20, and characteristically appear as a soap-bubble, multicystic lesion, which may remodel the calvarium. Epidermoids and arachnoid cysts are of CSF density, do not enhance with contrast and are extra-axial lesions.
909
A 24-year-old male patient presents following a head injury with GCS of 13. There is bruising over the right temporal region. A CT scan shows no intracranial haemorrhage but does identify a longitudinal fracture through the petrous temporal bone. What complication should be considered? a. sensorineural hearing loss b. conductive hearing loss c. vertigo d. carotid artery injury e. sigmoid sinus injury
b. Longitudinal fractures of the temporal bone represent 75% of temporal bone fractures and run parallel to the axis of the petrous pyramid. They may cause dislocation of the auditory ossicles, usually the incus, causing a conductive deafness. Sensorineural hearing loss is associated with transverse fractures of the temporal bone, as is vertigo. Facial nerve palsy is seen in both fracture types, but is less common in longitudinal fractures, where it frequently recovers spontaneously. Carotid artery and major sinus injuries are not directly associated with petrous temporal fractures.
910
A 44-year-old man presents with a long history of headaches and more recent onset of seizures. CT of the brain demonstrates an oval, well-defined, heterogeneous, hypodense mass containing large nodular clumps of calcification located peripherally in the right frontal lobe. The mass extends to the cortical margin, and there is erosion of the inner table of the skull. There is minimal surrounding vasogenic oedema. What is the most likely diagnosis? a. meningioma b. oligodendroglioma c. astrocytoma d. glioblastoma multiforme e. ganglioglioma
b. Oligodendrogliomas are slow-growing tumours, usually presenting in adults aged 30 – 50 years. They occur most commonly in the frontal lobe, and often extend to the cortex, where they may erode the inner table of the skull. Calcification is seen in 70% of cases, typically appearing as large nodular clumps. There is usually a relative absence of surrounding oedema. Astrocytomas also usually appear as hypodense calcified lesions with little surrounding oedema, but calvarial erosion is not usually a feature. Glioblastoma multiforme usually has considerable surrounding oedema and rarely calcifies. Gangliogliomas show calcification in a third of cases but tend to occur in children and young adults, and have a predilection for the temporal lobes.
911
A patient presents with colicky, right-sided, abdominal pain and is found to have multiple renal calculi. Blood tests reveal hypercalcae- mia and hyperparathyroidism. Ultrasound scan shows a hypoechoic nodule posterior to the left lobe of the thyroid suggestive of para- thyroid adenoma. Which features on parathyroid scintigraphy, using pertechnetate and sestamibi (with delayed washout images), would suggest a functioning parathyroid adenoma at this site? a. increased uptake on pertechnetate and sestamibi studies with delayed washout b. no uptake on pertechnetate; increased uptake on sestamibi with normal washout c. no uptake on pertechnetate; increased uptake on sestamibi with delayed washout d. no uptake on pertechnetate or sestamibi studies including delayed image e. increased uptake on pertechnetate and no uptake on sestamibi study including delayed image
c. Parathyroid adenomas do not take up pertechnetate, which is accumulated by thyroid tissue, whereas sestamibi is taken up by both thyroid and parathyroid tissue. Subtraction of these two images can then be used to show any difference that can be attributed to a parathyroid adenoma. The delayed image typically shows retention of sestamibi in parathyroid adenomas (delayed washout) compared with normal parathyroid and thyroid. Small parathyroid adenomas are often missed by scintigraphy.
912
A 20-year-old male is involved in an accident in which one femur is fractured. No other significant injury is revealed by CT of the brain, cervical spine, chest, abdomen and pelvis. Several hours after the injury, petechial skin haemorrhages appear, associated with respir- atory distress and hypoxia. The patient complains of headache shortly before a seizure is witnessed. Repeat CT of the brain is unchanged and looks normal. Which of the following brain MRI findings best accounts for the patient’s condition? a. diffuse low T2 signal in subarachnoid space b. crescent-shaped, space-occupying, extra-axial, low-T2-signal abnormality c. lenticular, space-occupying, extra-axial, low-T2-signal abnormality d. multiple, small, non-confluent, hyperintense lesions on T2W images and DWI within the cerebral deep white and deep grey matter e. multiple, small, non-confluent, hypointense lesions on T2W and DWI within the cerebral deep white and deep grey matter
d. The syndrome described is the fat embolism syndrome. This consists of the triad of acute respiratory distress with hypoxia, petechial skin haemorrhage and varying degrees of neurological dysfunction. The last includes headache, diminished GCS, seizures and irritability. Other features are tachycardia, fever, thrombocytopenia and anaemia. It occurs in 0.5 – 3.5% of long-bone fractures. The cause is likely to be the release of bone marrow elements into the circulation as a result of trauma. These act as emboli and they initiate a systemic inflammatory response when free fatty acids are released by the action of pulmonary lipases. The brain lesions seen on MRI are believed to be a combination of these two insults. Microinfarcts result from cerebral fat embolism, and oedema results from blood – brain barrier disruption which occurs because of the toxic effect of free fatty acids on brain tissue. Acute extra-axial haemorrhage might explain the neurological features of this case but would not account for the rash or hypoxia.
913
A 30-year-old female presents with left-sided hearing loss and facial nerve palsy. CT shows a solid mass in the left middle ear behind an intact tympanic membrane, occupying an enlarged attic and eroding the scutum and middle-ear ossicles. What is the most likely diagnosis? a. Bell’s palsy b. cholesteatoma c. malignant otitis externa d. acoustic neuroma e. squamous cell carcinoma
b. Cholesteatoma is an abnormal collection of keratinized debris arising from an ingrowth of stratified squamous epithelium and occurs in primary (2%) or acquired (98%) types. The acquired type can be further subdivided, with the most common being a primary, acquired, epidermoid-type lesion of the pars flaccida, located in the attic of the middle ear. Cholesteatomas are benign lesions but cause bone erosion, including the auditory ossicles, resulting in conductive hearing loss. Local extension may compress the geniculate ganglion in the facial canal.
914
A 46-year-old man presents with headaches and visual disturbance and is found to have bitemporal hemianopia on visual field testing. CT of the brain demonstrates a multilobulated, heterogeneous, suprasellar mass containing cystic areas and rim-like calcification. Enhancement of the solid component is observed following intra- venous contrast. What is the most likely diagnosis? a. epidermoid b. craniopharyngioma c. pituitary macroadenoma d. meningioma e. Rathke’s cleft cyst
b. Craniopharyngiomas are the most common suprasellar mass, predominantly occurring in the first and second decades, but with a second peak in the fifth decade. Presenting symptoms include headache secondary to hydrocephalus, bitemporal hemianopia (compression of the optic chiasm) and diabetes insipidus (compression of the pituitary gland). Typical imaging features are calcification, cyst formation and enhancement that may be solid or nodular. Meningiomas may arise in the suprasellar region and commonly demonstrate calcification but are generally not cystic. Epidermoids may occasionally demonstrate rim calcification but rarely enhance following intravenous contrast. Pituitary macroadenomas may undergo haemorrhage, resulting in heterogeneity that can cause confusion with craniopharyngioma, but this typically occurs in adolescence. Calcification in macroadenomas is infrequent. Rathke’s cleft cysts are thin-walled, benign cysts arising in the anterior sellar or suprasellar region. They show no contrast enhancement and rarely calcify.
915
A 47-year-old female presents with gradual onset back pain over 4 weeks, with associated pyrexia and tenderness at the thoracolumbar junction. Radiographs show destruction of the endplates at the T12 – L1 disc level. Which of the following features on further imaging would suggest tuberculous over pyogenic discitis as a cause? a. single-level involvement b. paravertebral, soft-tissue mass c. epidural abscess d. disc-space loss e. calcification
e. Tuberculous discitis tends to occur in children and adults around the age of 50 years. It most commonly affects T12 – L1, compared with pyogenic discitis, which tends to occur more distally. Tuberculous discitis often affects more than one level contiguously. Paravertebral masses and epidural abscesses are seen as complications in all types of discitis, but calcification within an abscess is virtually diagnostic for tuberculosis. Disc-space loss is also seen in all types of discitis, although it tends to be better preserved with tuberculous infection.
916
Ultrasound examination of the face and neck is performed to inves- tigate a buccal, soft-tissue mass that became noticeable during preg- nancy. The lesion is heterogeneous and hypoechoic, and has sinusoidal spaces demonstrating slow flow and circular calcifications. Which of the following is the most likely diagnosis? a. benign lymph node b. malignant lymph node c. pleomorphic parotid adenoma d. arteriovenous malformation e. venous vascular malformation
e. Phleboliths if present are unique to vascular malformations. Arterial malformations are high flow, while venous, capillary or combined malformations are low flow. MRI is required to assess the full extent, particularly intraosseous and intracranial, of head and neck vascular malformations. Benign lymph nodes are smooth, elliptical and hypoechoic with hilar architecture and vascularity. Malignant lymph nodes are typically round, are hypoechoic, have no hilum and show peripheral vascularity. Malignant lymph nodes with necrosis are seen with squamous cell and papillary cell carcinoma of the thyroid. Internal punctate calcification is seen in metastases from papillary or medullary carcinoma of the thyroid.
917
A 20-year-old male patient has an MR scan of his spine for investigation of back pain. He has low IQ and multiple skin patches. The MR scan shows bilateral branching tubular paraspinal masses, with widening of the intervertebral foramina and posterior scalloping of the vertebral bodies. A sharply angulated kyphosis is present at the thoracolumbar junction. What is the most likely diagnosis? a. neurofibromatosis type 1 b. neurofibromatosis type 2 c. tuberous sclerosis d. Marfan’s syndrome e. Ehlers – Danlos syndrome
a. Neurofibromatosis type 1 (peripheral neurofibromatosis or von Recklinghausen’s disease) is a multisystem disorder affecting the majority of organ systems. The presence of plexiform neurofibroma is pathognomonic. In the spine, there is abnormal development of the vertebral bodies with hypoplasia of pedicles and posterior elements. Dural ectasia is seen secondary to weakness of the meninges. Neurofibromatosis type 2 (central neurofibromatosis) is characterized by bilateral acoustic neuromas, meningiomas and ependymomas. Tuberous sclerosis produces multiple hamartomas and malformations of several organ systems. Marfan’s and Ehlers – Danlos syndromes may also cause posterior scalloping of vertebral bodies.
918
A severely hypoplastic cerebellar vermis in an enlarged bony pos- terior fossa, with associated hydrocephalus and communication of the fourth ventricle with a posterior midline CSF cyst, are features of which of the following posterior fossa malformations? a. mega cisterna magna b. Dandy – Walker malformation c. Dandy – Walker variant d. Arnold – Chiari malformation e. Joubert’s syndrome
b. Classically, the Dandy – Walker malformation consists of partial or total absence of the cerebellar vermis, dilatation of the fourth ventricle into a large cystic mass, an enlarged posterior fossa, hydrocephalus (in 75% of cases) and torcular – lambdoid inversion (elevation of the torcular herophili above the lambdoid suture). The proposed aetiology is obstruction of CSF outflow at the foramina of Magendie and Luschka. The vermis abnormality is the key component in all forms of the Dandy – Walker complex. The variant is less severe with a better prognosis. Chiari malformations have the fundamental abnormality of an underdeveloped, small posterior fossa, in contrast to the Dandy – Walker complex where it is normal or enlarged.
919
Following a large postpartum haemorrhage, a 25-year-old woman develops a severe headache and sudden visual field defect. What is the most likely diagnosis? a. intracerebral haemorrhage b. reversible posterior leukoencephalopathy c. subarachnoid haemorrhage d. Sheehan’s syndrome e. vertebral artery dissection
d. Many of the acute neurological conditions of pregnancy occur with rising blood pressure. Sheehan’s syndrome results from haemorrhage-induced hypotension causing pituitary infarction. Early on, this appears as an enlarged homogeneous pituitary with low T1 signal, high T2 signal and post-contrast ring enhancement. Later, there is an empty sella. Clinical manifestations include visual field loss, headache, ophthalmoplegia and pituitary dysfunction (diabetes insipidus). Reversible posterior leukoencephalopathy produces cortical blindness, headaches, confusion and seizures. Those affected are often taking immunosuppressant treatment. Imaging features can be identical to eclampsia, peripartum cerebral angiopathy and hypertensive encephalopathy, but with a posterior predominance. On CT, there is low attenuation change. On MRI, there is high signal on T2W/FLAIR images. ADC maps can differentiate between likely reversible vasogenic oedema (high signal on ADC map showing unrestricted diffusion) and cytotoxic oedema (low signal due to restricted diffusion), which is more likely to progress to infarct. Microangiopathic haemolytic anaemias, such as thrombotic thrombocytopenic purpura and haemolytic uraemic syndrome, give widespread ischaemia/infarction and haemorrhagic transformation. There is no increased risk in pregnancy of vasculitis such as systemic lupus erythematosus, Takayasu’s syndrome or Moyamoya syndrome. Arteriovenous malformation is no more likely to bleed in pregnancy, but there is an increased risk with arterial aneurysms. Haemorrhage, sepsis and pulmonary embolism cause hypotension that can cause watershed infarction as well as Sheehan’s syndrome.
920
In the presence of raised intracranial pressure, the anterior cerebral artery is at risk of compression during which of the following types of brain herniation? a. transforaminal herniation b. sphenoid herniation c. ascending transtentorial herniation d. descending transtentorial herniation e. subfalcine herniation
e. Transtentorial herniation may be descending (towards the posterior fossa) or ascending (upward displacement of the cerebellum through the tentorial incisura). Descending transtentorial herniation causes shift of the temporal lobe over the tentorium, which may compress the third cranial nerve, the posterior cerebral and anterior choroidal arteries, and the midbrain. Contralateral hemiparesis may occur due to compression of the ipsilateral cerebral peduncle. Ipsilateral hemiparesis may also occur due to compression of the contralateral cerebral peduncle against the tentorial edge (Kernohan’s notch phenomenon, a false localizing sign). Subfalcine herniation occurs when the cingulate gyrus shifts beneath the falx, due to medially directed supratentorial mass effect. This may cause compression of the anterior cerebral artery (resulting in ipsilateral distal anterior cerebral infarction) and internal cerebral veins. Sphenoid herniation involves herniation of the frontal lobe posteriorly across the edge of the sphenoid ridge, and rarely produces significant clinical symptoms. Transforaminal herniation results in herniation of the inferior cerebellum downward through the foramen magnum, which can result in obtundation and death.
921
A 30-year-old man with a history of metastatic malignant melanoma presents with sudden onset of visual loss of the upper right quadrant in both eyes (right superior homonymous quadran- tanopia). Emergency CT of the brain demonstrates a haemorrhagic cerebral metastasis with surrounding oedema. Which of the follow- ing is the most likely location of the lesion? a. optic chiasm b. right temporal lobe c. left temporal lobe d. left thalamus e. right thalamus
c. The optic radiation runs from the optic chiasm posteriorly to the occipital visual cortex. Each radiation carries with it optical fibres carrying information from the contralateral half of the visual field of each eye. This means a lesion in the left optic radiation will result in loss of vision of the right half of the visual field in both eyes (right homonymous hemianopia). However, as the radiation passes posteriorly from the lateral geniculate nucleus of the thalamus, it divides into two, with one division taking a relatively direct course posteriorly and the other a longer course through the temporal lobe. This is known as Meyer’s loop, and the lengthier course means that these fibres are more prone to disruption. The fibres in Meyer’s loop carry information from the upper visual field only, so a left temporal lobe lesion that affected Meyer’s loop would result in loss of vision only of the upper right quadrants of each eye (right superior homonymous quadrantanopia). A lesion at the optic chasm, such as a pituitary macroadenoma, will affect only the fibres that decussate at the chiasm, causing bitemporal loss of vision (bitemporal hemianopia).
922
Sequelae and complications of meningitis that may be identified on contrast-enhanced CT of the brain in the acute phase of the disease include all but which of the following? a. venous sinus thrombosis b. leptomeningeal enhancement c. encephalomalacia d. ring-enhancing mass lesion e. infarction
c. Diagnosis of meningitis is made by clinical examination and examination of CSF. Imaging is reserved for complications and to identify contraindications to lumbar puncture. The mechanism of spread is usually haematogenous, common organisms being Neisseria meningitidis (meningococcus) in young adults, Escherichia coli and Haemophilus influenzae. Complications that can be identified on CT include subdural empyema (sterile effusion can be seen with H. influenzae), venous sinus thrombosis, infarction, cerebritis and abscess formation. Leptomeningeal thickening and enhancement may be seen, but its absence does not exclude a diagnosis of uncomplicated meningitis. CT may also identify a potential source of infection such as otitis media, mastoiditis, sinusitis or orbital cellulitis.
923
A 23-year-old man presents with acute headache. Unenhanced CT of the brain demonstrates a heterogeneous mass at the inferior cer- ebellar vermis. It is predominantly of fat attenuation with areas of calcification and does not enhance following administration of intravenous contrast. Multiple droplets of fat attenuation are noted throughout the subarachnoid space. What is the most likely diagnosis? a. lipoma b. arachnoid cyst c. dermoid d. epidermoid e. teratoma
c. Epidermoids and dermoids are congenital lesions resulting from inclusion of ectodermal elements during closure of the neural tube. Both have a squamous epithelial lining and produce keratin, but dermoids contain both ectodermal and mesodermal elements (hair follicles, sweat and sebaceous glands), while epidermoids contain only ectodermal elements. Epidermoids are lobulated masses, usually located off the midline, which compress adjacent structures such as cranial nerves, and tend to have appearances on CT and MR scan following that of CSF. Dermoids are usually midline in location and cause symptoms by obstruction of CSF pathways or by rupture and leakage of fat contents, causing chemical meningitis. They have appearances on CT and MR scan following that of fat, and are often heterogeneous with areas of calcification and other soft-tissue components. This heterogeneity helps to distinguish a dermoid from a lipoma. Arachnoid cysts are of CSF density, but may be distinguished from epidermoids on DWI, where they do not show evidence of water restriction. Teratomas are composed of ectodermal, mesodermal and endodermal elements, and also appear heterogeneous with areas of fat, calcification and cystic components, but these lesions occur most commonly in the pineal and suprasellar regions.
924
A 65-year-old man presents with back stiffness and painful hips. Radiographs of the thoracolumbar spine and pelvis show ossifica- tion of the iliolumbar and sacroiliac ligaments with whiskering of the ischial tuberosities. Which of the following additional features is most likely to be seen on the spine radiographs? a. flowing osteophytes over several vertebral levels b. squaring of vertebral bodies c. reduced disc spaces d. sclerosis of vertebral bodies e. posterior longitudinal ligament calcification
a. The pelvic features are suggestive of diffuse idiopathic skeletal hyperostosis (DISH). Flowing anterior vertebral osteophytes, especially in the lower thoracic region, are very suggestive of this condition, and disc spaces are usually well preserved. Vertebral body squaring is seen in a number of conditions, including ankylosing spondylitis, but not DISH.
925
Bilateral globus pallidus injury manifest radiologically as high signal on T2W MR sequences is indicative of poisoning by which of the following substances? a. lead b. methanol c. carbon monoxide d. carbon dioxide e. mercury
c. Carbon monoxide poisoning results in irreversible formation of carboxyhaemoglobin in the blood, causing anoxic ischaemic encephalopathy. These changes are usually bilateral and affect the basal ganglia, most commonly the globus pallidus. Injury is demonstrated as high signal on T2W and FLAIR images, and shows restricted diffusion on DWI. Areas less commonly affected acutely are the putamen (which is characteristically involved in methanol poisoning) and caudate nucleus. Involvement elsewhere can occur but is less common than basal ganglia changes. Delayed post-anoxic encephalopathy may develop several weeks after carbon monoxide poisoning, and MRI then shows further high T2 signal changes in the corpus callosum, subcortical U fibres, and internal and external capsules, with low T2 signal changes in the thalamus and putamen.
926
A 31-year-old woman presents with a painless mass involving the right side of the mandible. Radiographs show a well-defined, lucent lesion. Which of the following additional findings favour a diagnosis of ameloblastoma over a radicular cyst? a. absence of matrix mineralization b. location at the symphysis menti c. location at the root of a tooth d. soap-bubble appearance e. rim of cortical bone
d. Both ameloblastoma and radicular cysts most commonly present in the third to fifth decades of life as a painless lump, often as an incidental finding. Neither typically shows matrix mineralization. Ameloblastoma represents 10% of all odontogenic (developing during or after the formation of teeth) tumours, with the majority located in the posterior body or ramus. They are frequently associated with the crown of an impacted or unerupted tooth. They can vary in radiographic appearance, an expansile, multiloculated lesion being typical. They are often resected due to locally aggressive infiltration. Radicular cysts are unilocular with a rim of cortical bone and usually develop at the root apex of a carious tooth as the end stage of the inflammatory process.
927
A hypertensive, 75-year-old female admitted with an acute stroke is found, on unenhanced CT of the brain, to have an acute basal ganglia haemorrhage. If an MRI were performed 2 weeks later, what signal characteristics would the region of haemorrhage return? a. isointense on T1W images and hyperintense on T2W images b. hyperintense on T1W images and hyperintense on T2W images c. isointense on T1W images and hypointense on T2W images d. hyperintense on T1W images and hypointense on T2W images e. hypointense on T1W images and a rim of hypointensity on T2W images
b. The MRI signal of blood depends first on whether it is moving or static, since on most sequences movement produces a signal void. When it is static, the signal returned by blood reflects the magnetic properties of the blood products and their location. Hyperacute haemorrhage is intracellular oxyhaemoglobin that is diamagnetic, returning an isointense T1 and bright T2 signal. At 1 – 2 days, deoxygenation has occurred, making the iron paramagnetic. It remains intracellular and returns an isointense T1 signal and is dark on T2W images. At 2 – 7 days, haemorrhage contains paramagnetic intracellular methaemoglobin. This is bright on T1W images and dark on T2W images. The methaemoglobin becomes extracellular from 1 week to 4 weeks, and the MRI signal is bright on both T1W and T2W sequences. Chronic haemorrhage contains haemosiderin/ferritin, which is ferromagnetic, is dark on T1W images and has a dark rim on T2W sequences.
928
A 24-year-old man is an unrestrained passenger in a car involved in a high-speed collision. He is found unconscious at the scene. CT of the brain is normal. MR scan of the brain demonstrates multiple small foci of high signal on T2W images in the white matter of the parasagittal regions of the frontal lobes and the periventricular regions of the temporal lobes. What is the most likely diagnosis? a. acute subdural haematoma b. diffuse axonal injury c. cortical contusions d. intracerebral haematoma e. subcortical grey matter injury
b. Diffuse axonal injury is characterized by widespread axonal disruption occurring in response to acceleration or deceleration forces – direct impact is not necessary. Typically, patients are immediately unconscious after the injury. CT is commonly negative, though 20% of lesions contain sufficient haemorrhage to be visible. On MR scan, typical findings are of multiple small foci of decreased signal intensity on T1W images and increased signal intensity on T2W images. Characteristic locations are the frontal and temporal white matter near the grey– white matter junction. More severe injuries may involve the lobar white matter and corpus callosum, with the brain stem involved in the most severe cases. Cortical contusions usually involve the superficial grey matter, and patients are less likely to present with immediate loss of consciousness. They characteristically occur near bony protuberances and are more commonly haemorrhagic. Subcortical grey matter injury is an uncommon type of injury seen after severe head trauma, with petechial haemorrhages in the basal ganglia and thalamus.
929
A 31-year-old woman presents to the neurologist with a transient episode of facial numbness. On close questioning, she reveals a tran- sient episode of blurred vision which occurred several months previously. MRI, including MR angiography, shows several small areas of low signal on T1W and high signal on T2W images in the optic nerves and cerebellar peduncles. The FLAIR sequence demonstrates further multiple, ovoid, high-signal areas in the corpus callosum and periventricular white matter. The angiogram demonstrates normal head and neck vessels. These imaging findings support which of the following diagnoses? a. internal carotid artery dissection b. multiple sclerosis c. progressive multifocal leukoencephalopathy d. acute disseminated encephalomyelitis e. wallerian degeneration
b. MRI is an important paraclinical tool to support the clinical diagnosis of multiple sclerosis. Multiple sclerosis is an inflammatory autoimmune disease of the brain and spine characterized by demyelination and damage to axons. It typically presents in a relapsing and remitting way with symptoms dependent on the location of lesions. MR enables identification of areas of tissue injury and disease progression, and location of active lesions. The lesions are high signal on T2W sequences (including FLAIR) and can be found throughout the brain, but have a predilection for periventricular white matter, appearing perpendicular to the ventricles with an ovoid conformation (Dawson’s fingers). The corpus callosum and cranial nerves are also common sites, and the FLAIR sequence helps to reveal lesions that would otherwise be masked by the high T2 signal of CSF. The corpus callosum is best examined on sagittal sequences. On T1W sequences, lesions are often isointense, but low signal can indicate areas of severe inflammation (so-called black holes) resulting in disease progression and disability. Enhancement with intravenous gadolinium is sometimes seen in lesions during the acute inflammatory phase and is thought to be the earliest detectable sign on MRI.
930
In normal anatomy of the nasal cavity, which structure opens into the inferior meatus below the inferior nasal turbinate? a. anterior ethmoidal ostium b. posterior ethmoidal ostium c. frontal sinus ostium d. maxillary sinus ostium e. nasolacrimal duct
e. The lateral nasal wall is separated into superior, middle and inferior meatuses by three curled bony shelves called turbinates (or conchae). The nasolacrimal duct opens into the anterior aspect of the inferior meatus and is usually the only opening seen there. The other ostia all open into the middle meatus, with the exceptions of the posterior ethmoidal ostia (superior meatus) and sphenoidal ostia (posterior to the superior turbinate in the sphenoethmoidal recess). The sphenopalatine foramen lies inferior to the sphenoethmoidal recess posterior to the middle turbinate.
931
A 30-year-old woman presents acutely with seizures, fever and headache, followed by rapid deterioration to coma. Emergency MRI shows asymmetrical swelling of the anterior temporal lobes on T1W images. T2W images reveal concordant asymmetrical but bilateral areas of high signal in the anterior temporal lobes, insular cortices and hippocampi. There is no enhancement follow- ing administration of intravenous gadolinium. What is the most likely condition? a. lymphoma b. HIV encephalitis c. cytomegalovirus encephalitis d. herpes simplex encephalitis e. toxoplasmosis
d. Herpes simplex virus is the most common cause of fatal endemic encephalitis, often leaving survivors with severe memory and personality problems. Both oral (type 1) and genital (type 2) strains may produce encephalitis with a multimodal distribution, affecting neonates (due to cross-infection with type 2 from the mother during birth), children and adults. Childhood and adult infection is caused by the type 1 virus and results in fulminant necrotizing encephalitis presenting with acute confusion and deteriorating rapidly to coma. Focal neurological deficits are seen in only 30% of cases. The virus asymmetrically affects the temporal lobes, insula, orbitofrontal region and cingulate gyrus, causing oedema. This is seen as high signal on T2W/FLAIR images, with DWI appearances variable depending on the presence of infarction. The putamen is characteristically spared, and the areas of encephalitis typically do not show enhancement on CT or MRI.
932
A young patient presents with double vision and is found on exam- ination to have a ptosis and dilated left pupil. The gaze in the same eye is fixed inferiorly and laterally and, when the ipsilateral light reflex is tested, there is constriction of the contralateral pupil only. CT shows a small spontaneous brain-stem haemorrhage thought to be due to an arteriovenous malformation. Which of the following locations of the haemorrhage best explains the presenting symptoms? a. superior pons b. inferior pons c. superior midbrain d. inferior midbrain e. superior medulla
c. The signs describe oculomotor nerve palsy. This will result in a characteristic down-and-out position of the affected eye due to the unantagonized action of the superior oblique and lateral rectus muscles, which are supplied by the trochlear and abducent nerves respectively. The palsy will also cause ptosis and pupillary dilatation due to loss of the motor component of the light reflex. The nuclei of the oculomotor nerves are found in the superior midbrain within the tegmentum, at the level of the superior colliculi. Those of the trochlear nerve are situated at the level of the inferior colliculi. The oculomotor nerve arises from the anterior surface of the midbrain on the medial side of the cerebral peduncle, passing between the posterior cerebral and superior cerebellar arteries to enter the cavernous sinus and pass into the orbit via the superior orbital fissure.
933
A 27-year-old female presents with emotional lability and head- aches. MRI of the brain demonstrates a well-defined mass in the pericallosal region. The mass is hyperintense on T1W images and demonstrates no enhancement with intravenous gadolinium. There is associated agenesis of the corpus callosum. What is the most likely diagnosis? a. dermoid b. lipoma c. interhemispheric arachnoid cyst d. epidermoid e. lymphoma
b. Intracranial lipomas appear as well-circumscribed masses of fat density on CT with occasional rim calcification. On MR scan, characteristic appearances are of hyperintensity on T1W images, with chemical shift artefact or signal suppression on fat-saturated sequences. Approximately 30% of intracranial lipomas occur in the pericallosal region, and there is a high incidence of associated congenital anomalies, most commonly agenesis of the corpus callosum, but also encephalocele and cutaneous frontal lipomas. Interhemispheric arachnoid cysts may occur in association with agenesis of the corpus callosum, but they are of CSF density and, like epidermoids, appear hypointense on T1W images. Dermoids have signal characteristics following those of fat, but are usually more heterogeneous and not associated with callosal anomalies. Lymphoma may involve the corpus callosum but appears iso- or hypointense on T1W images.
934
A 38-year-old male presents with a history of recent head injury and unilateral, painful, pulsatile exophthalmos with reduced visual acuity. CT shows swelling of the extraocular muscles with dilata- tion of the superior ophthalmic vein and cavernous sinus on the ipsi- lateral side, both of which enhance avidly. What is the most likely diagnosis? a. ophthalmic varix b. arteriovenous malformation c. lymphangioma d. carotid – cavernous fistula e. pseudotumour of orbit
d. Carotid – cavernous sinus fistula may occur spontaneously (secondary to aneurysm rupture, dural sinus thrombosis or atherosclerosis) or after trauma/surgery. It presents with a classic triad of pulsatile exophthalmos, conjunctival chemosis and auscultatory bruit. However, reduction in visual acuity may be the only sign. Secondary findings such as proptosis, congestive extraocular muscle enlargement, and distension with enhancement of the superior ophthalmic vein and cavernous sinus are usually identified on routine CT or MRI. However, catheter angiography is usually required for lesion classification and treatment planning prior to embolization. Drainage may also occur into the contralateral cavernous sinus, resulting in dilatation and enhancement. Ophthalmic varices present with intermittent exophthalmos related to straining, with dilatation of superior and/or inferior ophthalmic veins, which may thrombose. Orbital pseudotumour is an idiopathic inflammatory condition affecting all orbital contents, but vascular dilatation is not a feature.
935
A 30-year-old female with a past medical history of spontaneous pulmonary embolus presents 2 weeks after giving birth with severe headache, vomiting and drowsiness. Unenhanced CT of the brain shows areas of low attenuation with sulcal effacement and small areas of parenchymal haemorrhage. These changes do not conform to an arterial distribution. What is the most likely finding on the post-contrast CT brain? a. ring enhancement of the low-attenuation regions b. demonstration of a basilar tip aneurysm c. ‘empty delta’ sign d. anterior pituitary enlargement e. vertebral artery dissection flap
c. The patient has a venous sinus thrombosis causing congestion and venous infarction. Veno-occlusive disease is commoner in the first 3 weeks postpartum, especially if there is underlying hypercoagulability, including factor V Leiden abnormality, antiphospholipid antibody syndrome, and protein C, protein S or antithrombin III deficiency. On unenhanced CT, there may be hyperdense veins, grey – white matter junction haemorrhage and brain oedema. On CT venography, an ‘empty delta’ can be seen because thrombus rather than iodinated contrast occupies the affected dural venous sinus.
936
Which of the following structures lies in the parapharyngeal space? a. internal carotid artery b. hypoglossal nerve c. maxillary artery d. lingual tonsil e. vagus nerve
c. The parapharyngeal space is triangular shaped and extends from the skull base to the hyoid. It contains fat, branches of the mandibular division of the trigeminal nerve, maxillary artery, ascending pharyngeal artery and pharyngeal venous plexus. The internal carotid artery, vagus nerve and hypoglossal nerve lie in the carotid space. The lingual tonsils lie in the pharyngeal mucosal space.
937
A 31-year-old woman attends neurology clinic with a history of orthostatic headache, worst on standing, which sometimes induces vomiting, and relieved by lying down. MRI of the brain and cervical spine shows crowding of the foramen magnum due to low-lying cerebellar tonsils, elongation of the fourth ventricle, effacement of the prepontine cistern and a prominent pituitary gland. In the spine, an extradural fluid-signal collection is identified ventral to the cord. Administration of intravenous gadolinium reveals smooth areas of intracranial pachymeningeal enhancement. Which of the following diagnoses is best supported by these findings? a. intracranial hypotension b. intracranial hypertension c. migraine d. Chiari I malformation e. Dandy – Walker malformation
a. Spontaneous intracranial hypotension is a syndrome of low CSF pressure characterized by postural headaches in patients without any history of dural puncture or penetrating trauma. It is thought to arise from an occult CSF leak due to dural defects reducing CSF volume and subsequently pressure. Intracranial findings include downward displacement of the brain, subdural effusions, engorgement of other venous structures (including the hyperaemic pituitary) and low-lying cerebellar tonsils. There may be flattening of the pons as the brain sags against the skull base. Diffuse pachymeningeal enhancement is due to increased venous supply in an attempt to maintain intracranial volume and therefore pressure, according to the Monro –Kellie doctrine. In the spine, extradural fluid collection is indicative of an occult leak. Treatment is with an epidural blood patch where autologous blood is introduced into the extradural (epidural) space in an attempt to seal the microscopic dural defects.
938
A 40-year-old male presents with pain in the jaw associated with dental caries. An orthopantomogram demonstrates a lytic lesion of 1 cm in diameter, associated with the apex of a carious tooth. The lesion has a thin rim of cortical bone and there is some resorp- tion of the root. What is the likely diagnosis? a. ameloblastoma b. odontogenic keratocyst c. dentigerous cyst d. radicular cyst e. odontoma
d. A radicular cyst is the commonest type of cyst in the jaw and is associated with the apex of a tooth. It may displace teeth and cause mild root resorption. Ameloblastoma (adamantinoma of the jaw), odontogenic keratocyst and dentigerous cyst are all associated with the crown of a tooth, usually unerupted, and can be difficult to distinguish from each other on imaging alone. They are all odontogenic with no mineralization, and may be large and expand the mandible. Odontomas are the commonest odontogenic mass (67%) and show mineralization. They are seen in 10 – 20 year olds presenting with single or multiple, tooth-like masses.
939
A 14-year-old boy is hit by a vehicle while riding his bike and sus- tains a left-sided head injury. Paramedics report a transient loss of consciousness at the scene. On examination, he is confused with a GCS of 13/15. Unenhanced CT of the brain is performed, which shows a biconvex extra-axial collection in the left temporoparietal region with attenuation value of 65 HU. On bone windows, there is an undisplaced, linear, left temporoparietal skull fracture. What is the most likely diagnosis? a. acute subdural haematoma b. acute extradural haematoma c. acute subarachnoid haemorrhage d. acute intracerebral haematoma e. cerebral cortical contusion
b. Acute extradural haematomas are traumatic in origin and tend to be commoner in younger patients. Two-thirds of cases involve the temporoparietal region, and 75 – 95% of patients have an associated skull fracture. Bleeding is usually from the underlying middle meningeal artery, though it may also be due to disruption of the middle meningeal vein, dural venous sinuses or diploic veins. Typical appearances are of a high-density lentiform or biconvex collection, which forms between the inner table of the skull and the dura mater. The dura is firmly bound to the skull at sutural margins, so extradural haematomas tend not to cross suture lines. Acute subdural haematomas usually follow severe trauma, and have a poorer prognosis than acute extradural haematomas due to a high incidence of associated contusions and other brain injuries. They tend to be crescentic in shape and may freely extend across suture lines, being limited only by the interhemispheric fissure and tentorium. They are commonly bilateral, particularly in infants, and have no particular association with skull fractures.
940
A 64-year-old man with squamous cell carcinoma of the lung pre- sents with difficulty in speaking. On examination, he is noted to have dysarthric speech and deviation of the tongue to the left. CT of the brain is unremarkable, but review on bone windows reveals a destructive lesion of the left side of the skull base consistent with a bony metastasis. Which of the following skull base structures is most likely to be involved? a. foramen ovale b. foramen rotundum c. foramen lacerum d. jugular foramen e. hypoglossal canal
e. Hypoglossal nerve (cranial nerve XII) palsy is uncommon, characteristically producing unilateral atrophy of the tongue musculature, and resulting in deviation of the tongue towards the weak side and dysarthric speech. Supranuclear lesions cause contralateral paralysis (tongue deviation away from the side of the lesion) whereas nuclear and infranuclear lesions cause ipsilateral paralysis (tongue deviation towards the side of the lesion). The hypoglossal nerve exits the skull base via the hypoglossal canal, and this segment of the nerve may be affected by benign or malignant tumours and trauma of the skull base. Metastatic tumours most commonly arise from the lung, breast or prostate primaries. Direct extension from nasopharyngeal squamous cell carcinoma may also produce skull base erosion involving the hypoglossal canal. Other pathological conditions that can affect the nerve at this site include skull base infections, Paget’s disease and fibrous dysplasia.
941
A 65-year-old man has an unenhanced CT of the brain for recent- onset, unilateral hand weakness. The CT shows multiple, bilateral, small supratentorial regions of low attenuation. Which of the fol- lowing is the most likely associated finding? a. unilateral carotid atherosclerosis b. cardiac valve disease c. coarctation of proximal descending thoracic aorta d. subclavian stenosis e. sinus tachycardia
b. Thromboembolic cerebral infarction occurs in atrial fibrillation, cardiac valve disease, fibromuscular dysplasia, intracranial aneurysms, sickle cell disease, atherosclerosis and thrombotic thrombocytopenic purpura. Multiple infarcts are more likely with extracranial disease and can take the form of a shower of emboli. In these cases, the distribution is typically bilateral and more commonly supratentorial.
942
In brain imaging performed to characterize a chronic subdural collection, which feature is more likely to favour a diagnosis of subdural empyema over a sterile effusion? a. isointense MR signal on T1W images b. hyperintense MR signal on T2W images c. subfalcine herniation d. restricted diffusion on MR DWI e. low attenuation on CT
d. Abscess cavities and empyemas are homogeneously hyperintense on MR DWI and low on ADC map due to the increased viscosity of the purulent material that they contain, resulting in restricted diffusion of water. Sterile effusions are hypointense on DWI and have an ADC appearance similar to that of CSF due to their lower viscosity and free macro-diffusion of water. Diffusion images can therefore be important when deciding whether to intervene surgically or conservatively manage subdural collections, as empyema requires timely surgical drainage. Contrast enhancement of the wall of the collection may be seen on CT and MRI, but its absence does not exclude the diagnosis of a purulent collection. Likewise, one may expect to see additional mass effect with an infected collection, but this too is not sensitive enough to exclude infection by its absence.
943
A 50-year-old female presents with increasing weakness of the lower limbs and sensory disturbance. MRI shows an extramedul- lary, intradural ovoid mass in the mid-thoracic region. The lesion has signal isointense to the cord on T1W and T2W images, and enhances avidly with intravenous gadolinium. What is the most likely diagnosis? a. meningioma b. nerve sheath tumour c. ependymoma d. dermoid e. arachnoid cyst
a. Meningiomas in the spine are seen mainly in females (80%) and in those over 40 years of age. They have similar signal characteristics to the cord on MRI and enhance avidly. They can cause symptoms related to cord compression. Bone erosion is seen in ,10%. Nerve sheath tumours produce fusiform masses arising from the nerve roots, often extending through the intervertebral foramen, causing them to be dumb-bell shaped. They are isointense to muscle on T1W and hyperintense to fat on T2W images. Ependymomas are usually located in the filum terminale and show low/intermediate signal on T1W images, with foci of high signal on T2W images, and they often enhance. Dermoids usually occur in the conus or cauda equina and are associated with spinal dysraphism in one-third of cases. They are variable in signal on T1W but are high signal on T2W images.
944
A 21-year-old man presents with acute headache. He undergoes CT of the brain, which demonstrates a well-circumscribed, lobulated, partially cystic, calcified mass in the frontal horn of the left lateral ventricle, attached to the septum pellucidum. There is acute blood seen layering in the left lateral ventricle and mild hydrocephalus. What is the most likely diagnosis? a. choroid plexus papilloma b. colloid cyst c. intraventricular oligodendroglioma d. central neurocytoma e. subependymoma
d. Central neurocytomas (also known as intraventricular neurocytomas) are benign tumours of the lateral and third ventricles usually presenting in adults aged 20 – 40 years. They frequently calcify (69%) and contain cystic spaces. Attachment to the septum pellucidum is a characteristic feature. Lesions appear isointense to grey matter on all MR sequences and show mild-to-moderate contrast enhancement. Central neurocytomas were previously frequently mistaken for intraventricular oligodendrogliomas, which have very similar imaging features but are actually quite rare. In addition, neurocytomas undergo haemorrhage into the tumour or ventricle more frequently, helping to distinguish the two. Colloid cysts arise within the third ventricle, and rarely calcify. Subependymomas may arise in the lateral ventricles with an attachment to the septum pellucidum, and cyst formation and calcification may be seen in large tumours. However, most occur in patients over 40 years of age. Choroid plexus papillomas generally occur in children under 5 years of age
945
A 2-year-old boy presents with involuntary saccadic eye movements (opsoclonus) and myoclonus of the trunk and limbs. What are the most likely findings on MRI of the brain? a. normal appearances b. cystic lesion in the posterior fossa with enhancing mural nodule c. cerebellar atrophy d. central symmetrical lesions in the pons of high signal intensity on T2W images e. caudally displaced brain stem and fourth ventricle with tonsillar herniation
a. Opsoclonus – myoclonus syndrome is characterized by opsoclonus in combination with myoclonus of the trunk, limbs or head. It may be idiopathic or occur as a paraneoplastic syndrome, when it may follow a relapsing – remitting course. In adults, it is most commonly associated with breast and squamous cell lung carcinoma, while in children it is associated with neuroblastoma. The syndrome usually precedes diagnosis of the underlying malignancy, and in children should prompt investigation to identify an underlying neuroblastoma. MIBG whole-body scintigraphy may be helpful in identifying occult disease if conventional imaging is negative. MR scan of the brain is usually normal.
946
A 25-year-old male presents with a 3-week history of dry cough and chest pain. A plain chest radiograph reveals a well-defined opacity in the right paratracheal region. CT chest reveals a well-defined soft tissue mass with a peripheral rim calcification that measures 3 cm and has an average Hounsfield unit of 45 and there is no enhancement following contrast. At MRI, the mass returns high signal on both T1 and T2 weighted images. What is the most likely diagnosis? A. Bronchogenic cyst B. Pericardial cyst C. Haemangioma D. Ascending thoracic aortic aneurysm E. Neurogenic tumour
A. Bronchogenic cyst All of the options are causes of a mediastinal mass. While the majority of bronchogenic cysts occur in subcarinal region of the middle mediastinum, they can sometimes occur in the right paratracheal region or in the posterior mediastinum. The diagnosis is often confirmed by CT, which shows a thin walled cyst containing fluid of either low attenuation or soft tissue attenuation if it contains mucinous material. High T1 and T2 signal intensities reflect the presence of mucinous contents. They are usually detected incidentally in patients without symptoms. In some, they may produce chest pain, dyspnoea or cough. Rapid increase in size of the mass indicates secondary haemorrhage or infection. Pericardial cysts typically occur in the right anterior cardiophrenic angle, can produce similar symptoms, but should not have mucinous contents. Haemangiomata are round or oval soft tissue masses that may contain coarse calcification and avidly enhance after intravenous contrast. An ascending aortic aneurysm may exhibit peripheral calcification but signal void due to flowing blood is typical at MRI. Neurogenic tumours are usually seen as incidental soft tissue masses in the posterior mediastinum but return low to intermediate in T1 weighted imaging. McAdams HP, Kirejczyk WM, Rosado-de-Christenson ML, et al. Bronchogenic cyst: imaging features with clinical and histopathologic correlation. Radiology 2000; 217: 441–6.
947
A 40-year-old obese male had a chest radiograph following a low-impact road traffic accident. This revealed a widened superior mediastinum with a central trachea and a large epicardial fat pad. Lungs were clear and heart size was normal. He describes right-sided pleuritic chest pain, but is haemodynamically stable and clinical examination is normal. A CT chest is performed: which of the following is likely in this patient? A. Diffuse increased attenuation within the mediastinum with a higher CT number than soft tissue B. Streaky soft tissue attenuation and calcification within the mediastinal fat with compression and distortion of the superior vena cava C. Diffuse low attenuation within the mediastinum with CT number of –110 D. Multiple nodular soft tissue deposition within the mediastinal fat E. Normal mediastinum and central great vessels
C. Diffuse low attenuation within the mediastinum with CT number of –110 The most likely diagnosis in this case is mediastinal lipomatosis, which is commonly seen in an asymptomatic obese subject or in patients on long-term steroid treatment. The redistribution of body fat in the anterior mediastinum, cardiophrenic angles and paravertebral regions produces smooth widening of superior mediastinum, a large epicardial fat pad and lateral displacement of the paraspinal lines on the chest radiograph. The widening of the mediastinum can be difficult to differentiate from mediastinal haemorrhage or generalized lymphadenopathy but can be easily confirmed in CT. Answer A describes mediastinal haemorrhage, which can be seen in trauma and haemorrhagic disorders but the trachea is usually displaced. Answer B describes mediastinal fibrosis, which is seen in a wide variety of conditions, for example, previous radiotherapy, tuberculosis and fungal infections. Patients may be asymptomatic or present with signs of superior vena cava obstruction. Answer D is seen with mediastinal lymphadenopathy of any aetiology, which causes a lobulated mediastinal contour. Homer MJ, Wechsler RJ, Carter BL. Mediastinal lipomatosis. Radiology 1978; 128: 657–61. doi: 10.1148/128.3.657.
948
A 30-year-old male was treated with combined chemoradiotherapy for mediastinal Hodgkin’s disease and underwent complete remission. A follow-up CT scan depicted a homogeneous soft tissue mass with biconvex margins in the anterior mediastinum. Which of the following is the best imaging modality to differentiate between rebound thymic hyperplasia and recurrent lymphoma? A. Contrast-enhanced CT B. In- and opposed-phase MRI C. Fat suppressed T1 weighted MRI D. Fludeoxyglucose positron emission tomography/CT E. 201Tl scintigraphy
B. In- and opposed-phase MRI Rebound thymic hyperplasia is common in patients treated for lymphoma or other malignancies. A reliable non-invasive technique to diagnose it from residual, recurrent or metastatic disease is extremely useful in clinical practice. Chemical shift MRI is a sensitive imaging modality to detect the presence of microscopic fat, typical of thymic hyperplasia. Chemical shift MRI demonstrates the intravoxal mixture of fat and water by showing homogenously decreased signal in the opposed phase image when compared to the in phase image. Lymphoma recurrence would display no such change in signal. Contrast-enhanced CT would not be sensitive to delineate the tiny amounts of fat that are present in thymic hyperplasia. Fat suppressed MRI sequences are extremely useful to detect the presence of macroscopic fat in a lesion but it cannot detect the presence of microscopic fat. 201Tl scintigraphy is useful for differentiating normal thymus from hyperplastic thymus, and thymic lymphoma but it has several disadvantages including its low spatial resolution and overlap in diagnostic criteria. Increased thymic 18F-fludeoxyglucose activity may be striking in rebound hyperplasia causing diagnostic uncertainty. Nasseri F, Eftekhari F. Clinical and radiologic review of the normal and abnormal thymus: peals and pitfalls. Radiographics 2010; 10: 413–28. doi: 10.1148/rg.302095131.
949
A 40-year-old female presents to the rheumatology clinic with pain in the small joints of her hands and feet. Blood tests show a normochromic, normocytic anaemia with a high erythrocyte sedimentation rate and positive rheumatoid factor. She is diagnosed with rheumatoid arthritis and commenced on oral methotrexate and analgesia. After 6 months, she represents with a dry cough and breathlessness. A chest radiograph is obtained. Which of the following is the most common radiographic finding in this patient? A. Pleural effusion and thickening B. Upper zone volume loss C. Pulmonary nodules D. Reticulo-nodular opacity involving the lower zones E. Pericardial effusion
A. Pleural effusion and thickening Thoracic manifestations of rheumatoid arthritis develop as disease progresses and pleural involvement is the most common manifestation of thoracic disease. Pleural thickening is the most common thoracic manifestation. Pleural effusion is seen less often and it is usually unilateral and occurs late in the disease process. It is not possible to accurately differentiate between the two on a plain film alone. Pericardial effusions and subcutaneous nodules are often seen. Pulmonary nodules are uncommon, usually seen in patients with subcutaneous nodules although they may cavitate and tend to occur in advanced disease. Lower lobe pulmonary fibrosis manifests as reticular and reticulo-nodular opacities and is seen in a small percentage of patients with advanced disease. High-resolution CT usually shows thick irregular interlobular septa in peripheral zones of both lower lobes. As the disease progresses, honeycombing and volume loss becomes evident. Other radiological manifestations of thoracic disease in rheumatoid arthritis are pericardial effusion and bronchiolitis obliterans. Shoulder joint arthritis and distal clavicular erosions may be seen. Mayberry JP, Primack SL, Müller NL. Thoracic manifestations of systemic autoimmune diseases: radiographic and high-resolution CT findings. Radiographics 2000; 20: 1623–35.
950
A young Asian male presents to the accident and emergency department with worsening shortness of breath and chest pain. On further questioning, he admits to night sweats and fever. The chest radiograph reveals a globular-shaped heart with bilateral perihilar lymph node enlargement and some left upper zone pulmonary infiltrate. What is the most likely diagnosis? A. Pericardial effusion and tuberculosis B. Hodgkin’s lymphoma C. Dilated cardiomyopathy D. Pulmonary stenosis E. Sarcoidosis
A. Pericardial effusion and tuberculosis While tuberculosis may affect any body system, cardiac involvement is rare. When it does occur, the commonest feature is pericardial effusion that results in a globular cardiac silhouette when chronic. While there are numerous causes of pericardial effusion (for example, heart failure, empyema, trauma, malignancy), the other features in the history point to tuberculosis: the patient is Asian and has clinical features of night sweats and fever. Lymphoma would also account for the night sweats and fever and is one of the commoner causes (although still rare) of malignant pericardial effusions. However, the upper zone shadowing is better explained by tuberculosis. Apical fibrosis and Hodgkin’s disease is rare amongst Asian populations. Sarcoidosis is also less likely than tuberculosis; while it may present with bilateral hilar lymphadenopathy, other features such as lupus pernio or erythema nodosum might be expected. Dilated cardiomyopathy primarily affects the left ventricle, but may also cause atrial enlargement. Pulmonary oedema and pleural effusions may be seen, due to left ventricular dysfunction. Pulmonary stenosis may cause a dilated right ventricle, with dilatation of the pulmonary trunk, but the other features do not fit. Harisinghani MG, Mcloud TC, Shepard JA, et al. Tuberculosis from head to toe. Radiographics 2000; 20: 449–70.
951
A 65-year-old male presents to the accident and emergency department with an episode of central chest pain. There is no history of ischaemic heart disease. An electrocardiogram demonstrates ST elevation in leads IV, V and aVF and he is admitted to coronary care unit. His hospital stay is uncomplicated and he undergoes a cardiac MRI as an outpatient. What is it most likely to show? A. Delayed enhancement with sparing the subendocardium B. Delayed subendocardial enhancement of the left ventricular wall C. Global hyperenhancement D. Delayed subendocardial enhancement of the right ventricular wall E. Patchy epicardial enhancement
B. Delayed subendocardial enhancement of the left ventricular wall The patient has had an ST elevation myocardial infarction. The lateral leads (IV, V, VI) correspond to the lateral portion of the left ventricle and are supplied by the circumflex artery. It is a branch of the left coronary artery that descends in the posterior A-V groove and usually supplies the postero-lateral left ventricular wall. The other branch of the left coronary artery is the left anterior descending artery and it supplies the anterior wall (leads I, II, III). The right coronary artery supplies the inferior myocardium (leads II, III and aVF). The classical MRI features following myocardial infarction is delayed subendocardial enhancement of the ventricular wall. The myocardium (subendocardial) will show delayed enhancement corresponding with a vascular territory, in this case, the left ventricle. Delayed hyperenhancement with sparing the subendocardium is seen in myocarditis and may have a nodular pattern – it will not conform to a vascular territory. Global hyperenhancement is seen in amyloidosis. Patchy epicardial enhancement is seen in cardiac sarcoidosis. Hunold P. Myocardial late enhancement in contrast enhanced cardiac MRI: distinction between infarction scar and non-infarction related disease. AJR Am J Roentgenol 2005; 184: 1420–26.
952
A 21-year-old African female presents to her general practitioner with ongoing symptoms of tiredness, lethargy and shortness of breath on exertion. She describes having a “blood disorder” as a child, but has not had any follow-up since moving to the UK. On examination, she is found to have pale sclerae, and spoon shaped nails. A chest radiograph is performed and reveals a lobulated mass seen separately from the heart, which somewhat obscures the descending aorta. What is the diagnosis? A. Recurrent Burkitt’s lymphoma B. Lymph node mass C. Extramedullary haematopoiesis D. Neuroblastoma E. Oesophageal carcinoma
C. Extramedullary haematopoiesis Extramedullary haematopoiesis occurs in haemaglobinopathies, myeloproliferative disorders and bone marrow infiltration. It can be visualized as a well-defined, often lobulated, soft tissue mass adjacent to the spine, with the preservation of the ribs and vertebral bodies. The “blood disorder” she refers to is Sickle cell disease, an autosomal dominant blood disorder found predominantly in north and west African countries. Thoracic extramedullary haematopoiesis has also been described in thalassemias and hereditary spherocytosis. Burkitt’s lymphoma does not typically present in this location, and is often associated with immunocompromise. A lymph node mass would have similar characteristics on chest X-ray, but does not tie in with the clinical history. Neuroblastoma is a childhood malignancy that tends to occur in the adrenal gland, but may arise anywhere along the sympathetic chain (when is can be seen on chest X-ray as a posterior mediastinal mass). Oesophageal carcinoma is unlikely at this age and there is unlikely to be any evidence of the primary tumour on plain films. Gumbs RV, Higginbotham-Ford EA, Teal JS, et al. Thoracic extramedullary hematopoiesis in sickle-cell disease. AJR Am J Roentgenol 1987; 149: 889–93.
953
A 21-year-old African female presents to her general practitioner with ongoing symptoms of tiredness, lethargy and shortness of breath on exertion. She describes having a “blood disorder” as a child, but has not had any follow-up since moving to the UK. On examination, she is found to have pale sclerae, and spoon shaped nails. A chest radiograph is performed and reveals a lobulated mass seen separately from the heart, which somewhat obscures the descending aorta. What is the diagnosis? A. Recurrent Burkitt’s lymphoma B. Lymph node mass C. Extramedullary haematopoiesis D. Neuroblastoma E. Oesophageal carcinoma
C. Extramedullary haematopoiesis Extramedullary haematopoiesis occurs in haemaglobinopathies, myeloproliferative disorders and bone marrow infiltration. It can be visualized as a well-defined, often lobulated, soft tissue mass adjacent to the spine, with the preservation of the ribs and vertebral bodies. The “blood disorder” she refers to is Sickle cell disease, an autosomal dominant blood disorder found predominantly in north and west African countries. Thoracic extramedullary haematopoiesis has also been described in thalassemias and hereditary spherocytosis. Burkitt’s lymphoma does not typically present in this location, and is often associated with immunocompromise. A lymph node mass would have similar characteristics on chest X-ray, but does not tie in with the clinical history. Neuroblastoma is a childhood malignancy that tends to occur in the adrenal gland, but may arise anywhere along the sympathetic chain (when is can be seen on chest X-ray as a posterior mediastinal mass). Oesophageal carcinoma is unlikely at this age and there is unlikely to be any evidence of the primary tumour on plain films. Gumbs RV, Higginbotham-Ford EA, Teal JS, et al. Thoracic extramedullary hematopoiesis in sickle-cell disease. AJR Am J Roentgenol 1987; 149: 889–93.
954
A 65-year-old obese male with a history of cardiovascular disease presents with ischaemic limb pain. An MR angiogram is performed and he is subsequently referred for angioplasy. When performing the angioplasty, which is the least appropriate puncture site for the stated target? A. Right retrograde popliteal puncture: proximal right superficial femoral lesion B. Left retrograde common femoral puncture: proximal right superficial femoral lesion C. Left antegrade common femoral puncture: distal left superficial femoral lesion D. Right retrograde common femoral puncture: proximal right common iliac lesion E. Right antegrade common femoral puncture: proximal right superficial femoral lesion
E. Right antegrade common femoral puncture: proximal right superficial femoral lesion The most common arterial puncture site is the common femoral artery. It is relatively superficial and lies over the bony femoral head allowing it to be easily palpated and controlled postpuncture. An antegrade puncture allows in-line access to ipsilateral distal lesions. Retrograde puncture allows access to ipsilateral proximal in-line lesions, the aorta and cross-over to lesions in the contralateral lower limb. An antegrade common femoral artery puncture for a proximal superior femoral artery (SFA) lesion will not allow enough room between the puncture site and lesion to place a sheath to gain stable access. Retrograde popliteal artery puncture for accessing proximal SFA lesion can be performed using an ultrasound-guided puncture with the patient lying prone. Using the cross-over technique and a long sheath, retrograde common femoral artery puncture can be used for a proximal contralateral SFA lesion. In-line access to the ipsilateral lesion is described by a left antegrade common femoral artery puncture for distal left superficial femoral artery lesion and right retrograde common femoral puncture for proximal right common iliac artery lesion. Watkinson A, Adam A. Interventional Radiology: A Practical Guide. Milton Keynes, UK: Radcliffe Publishing Ltd; 1996. ISBN 1857750314.
955
A 55-year-old male presents with symptoms of worsening shortness of breath. He has a history of lymphoma in his 20s, from which he has remained in remission. He has recently given up smoking. His symptoms have caused him to take sick leave from his job as a supervisor at an industrial plant. CT chest reveals well-defined fibrotic change anteriorly. In addition, there is evidence of pulmonary venous hypertension, pleural effusions and calcification of the pericardium. What is the most likely diagnosis? A. Silicosis B. Constrictive pericarditis C. Pneumoconiosis D. Chronic obstructive pulmonary disease E. Alpha-1 antitrypsin deficiency
B. Constrictive pericarditis Constrictive pericarditis may be a long-term sequelae of radiotherapy treatment for lymphoma (with modern techniques it is becoming less common). Pericardial calcification is common; 90% of patients with constrictive pericarditis have calcifications. Resultant diastolic dysfunction causes pulmonary venous congestion. Silicosis occurs due to the inhalation of silicone dioxide, which is phagocytosed and forms non-caseating granulomata. It predominantly affects the upper lobes, with nodules, fibrosis, and eggshell calcification of hilar lymph nodes (5%). Pneumoconiosis due to inhalation of coal dust is similar to silicosis (causing nodules and fibrosis within the mid to upper zones). Progressive massive fibrosis can complicate pneumoconiosis. COPD is a clinical diagnosis, but radiological features may include increased lung volumes. Calcification is not a feature, but pulmonary arterial hypertension may feature in severe disease. Alpha-1 antitrypsin deficiency is an autosomal recessive condition that leads to emphysematous changes within the lungs at an early age. It classically produces pan-acinar emphysema at the lung bases. Bertog SC, Thambidorai SK, Parakh K, et al. Constrictive pericarditis: etiology and cause-specific survival after pericardiectomy. J Am Coll Cardiol 2004; 43: 1445–52.
956
A 78-year-old smoker presented to chest clinic with a history of dry cough and shortness of breath. He has unintentional weight loss of 5 kg over the last year. Clinical examination reveals bilateral wheeze and pulmonary function tests show an obstructive pattern. A chest radiograph is performed and shows emphysematous change. A 5 mm pulmonary nodule in the right upper lobe with no calcification or focal fat content is discovered at subsequent CT. What is the most appropriate next management step for this patient? A. PET/CT B. CT in 3 months C. Contrast-enhanced CT chest D. Repeat CT chest in a year E. CT-guided biopsy
D. Repeat CT chest in a year Lung nodules are detected very commonly on CT scans of the chest, and the ability to detect very small nodules improves with each new generation of CT scanner. The Fleischner Society has produced guidelines for the follow-up of small pulmonary nodules detected on CT scans. In high-risk patient like smokers, non-calcified pulmonary nodules less than or equal to 4 mm should be followed up by a repeat CT scan in 12 months to monitor any interval growth. Several studies have confirmed that there is no advantage in repeat scanning at shorter intervals. A single follow-up scan is sufficient unless there is an interval growth of the lesion. Positron emission tomography CT is usually non-diagnostic for pulmonary nodule less than 8 mm and hence is not an option for this patient. If the nodules are larger than 8 mm, additional options such as contrast-enhanced CT, positron emission tomography CT, percutaneous needle biopsy, and thoracoscopic resection can be considered. McMahon PM, Kong CY, Johnson BE, et al. Estimating long-term effectiveness of lung cancer screening in the Mayo CT screening study. Radiology 2008; 248: 278–87. doi: 10.1148/radiol.2481071446.
957
A 60-year-old male presents to the accident and emergency department with acute onset shortness of breath and chest pain. There is a past medical history of recurrent admissions with repeated atelectasis and pneumonia. A chest radiograph again reveals right middle and lower lobe collapse. The lungs and pleural spaces are otherwise clear. A central obstructive lesion was suspected and a staging CT chest is performed. There are multiple tracheal nodules and multifocal irregular narrowing of the trachea. In addition, there are multiple pulmonary nodules. What is the most likely diagnosis? A. Sabre sheath trachea B. Tracheobronchial amyloidosis C. Tracheobronchopathia osteochondroplastica D. Relapsing polychondritis E. Tracheobronchomalacia
B. Tracheobronchial amyloidosis Tracheobronchial amyloidosis is characterized by fibrillar protein polysaccharide deposition along the wall of trachea causing circumferential narrowing resulting in recurrent atelectasis and infection. Calcification of the deposits is rare. The diagnosis is made on Congo-red staining of a tracheal or bronchial biopsy. Sabre sheath trachea is a fixed deformity of the intrathoracic trachea seen in COPD patients where the coronal diameter of the trachea is less than two thirds of sagittal diameter. Tracheobronchopathica osteochondroplastica produce nodular calcified submucosal deposits in the anterior and lateral walls of the trachea. Relapsing polychondritis is a systemic autoimmune condition affecting the cartilage at multiple sites. The destruction of the cartilaginous tracheal rings initially causes dilatation but subsequent fibrosis leads to fixed narrowing of the tracheal lumen. The pinnae and bridge of the nose are also usually affected. Diagnosis requires recurrent inflammation at two or more cartilaginous sites. Tracheobronchomalacia causes tracheal lumen dilatation from a congenital or acquired defect of cartilage. Prince J, Duhamel D, Levin DL, et al. Nonneoplastic lesions of the tracheobronchial wall: radiologic findings with bronchoscopic correlation. Radiographics 2002; S215–30. doi: 10.1148/radiographics.22.suppl_1.g02oc02s215.
958
A 30-year-old male who has a history of many previous cranial surgical procedures presents to casualty with sudden onset breathlessness and pleuritic chest pain. His drug history includes carbamazepine, omeprazole and ferrous sulphate. Clinical examination reveals a maculopapular erythematous rash around his nose and bilateral spontaneous pneumothoraces are confirmed on his chest radiograph. What will be the findings at high-resolution CT? A. Multiple thin-walled intraparenchymal cysts and normal lung volume B. Bilateral symmetric reticulonodular opacities with reduced lung volume C. Coarse reticular opacities intermixed with thin-walled irregular parenchymal cysts D. Geographic ground glass opacities with thickened intralobular and interlobular septa E. Multiple small interstitial nodules clustered around the pulmonary fissures
A. Multiple thin-walled intraparenchymal cysts and normal lung volume This patient has tuberous sclerosis, which is characterized by a triad of mental retardation, seizures and adenoma sebaceum. Patients have multiple, recurrent posterior fossa cysts and tumours resulting in cranial surgery and in some, epilepsy (hence the carbamazepine). The nasal rash is adenoma sebaceum. Pulmonary involvement in tuberous sclerosis is rare but includes symmetric bilateral reticulonodular opacities with normal or increased lung volume. In later stages of the disease, small, uniform intraparenchymal cysts develop. Answer C is typical of Langerhanns cell histiocytosis, which typically affects young adults who smoke. Lung volumes are either normal or increased and there is preferential involvement of upper and mid zones. Answer D is that of alveolar proteinosis where the pulmonary appearance has been described as “crazy paving”. Sarcoidosis typically presents as interstitial nodules clustered around the peri-fissural distribution. Evans JC, Curtis J. The radiological appearances of tuberous sclerosis. Br J Radiol 2000; 73: 91–8.
959
A 30-year-old male developed rapid onset painful swelling of the jaw following extraction of an impacted molar tooth. Over the following days, he develops a productive cough and breathlessness. A chest radiograph reveals multifocal consolidation with a right-sided pleural effusion. Inflammatory markers were raised, but he failed to respond to amoxicillin treatment. A CT chest reveals non-segmental multifocal airspace opacification in both lower lobes, an empyema, osteomyelitis of right eighth and ninth ribs and a soft tissue mass in the chest wall. Which of the following is the most likely diagnosis? A. Nocardiosis infection B. Actinomycosis infection C. Atypical pulmonary tuberculosis D. Histoplasmosis E. Bacillus antracis
B. Actinomycosis infection Actinomycosis of the chest typically follows dental extraction, manifesting as mandibular osteomyelitis. It is a commensal of human oropharynx, which grows in devitalized and infected tissues. Pulmonary involvement usually occurs after aspiration of infectious oral debris, hence the lower lobe predominance. It spreads with disregard for normal anatomic barriers and extension into the pleural space, chest wall and ribs is common. Thoracic nocardiosis occurs in the immunocompromised and also disregards anatomic barriers and produces non-segmental airspace opacification, pleural and chest wall disease. Atypical pulmonary tuberculosis manifests with chronic fibrocavitary disease involving upper lobes. Pleural effusion and contiguous spread would be unusual. Histoplasmosis is endemic in certain areas of North America and presents as multiple well-defined subcentimetre calcified nodules. Thoracic anthrax causes haemorrhagic lymphadenitis, mediastinitis, and haemorrhagic pleural effusion, but areas of consolidation are usually absent. Kim TS, Han J, Koh WJ, et al. Thoracic actinomycosis: CT features with histopathologic correlation. AJR Am J Roentgenol 2006; 186: 225–31. doi: 10.2214/AJR.04.1749.
960
A 34-year-old male with a history of alcohol abuse presents with haematemesis. Endoscopy is performed and the varices are banded. However, he continues to bleed despite subsequent endoscopy and banding, and he is referred for transjugular intrahepatic portosystemic shunt. During the procedure, which fluoroscopic appearance would be consistent with successful cannulation of the portal vein? When contrast is injected into the transjugular intrahepatic portosystemic shunt needle: A. Contrast flows via large vessels to the periphery of the liver B. Contrast flows through small vessels towards the periphery of the liver C. Contrast pools at the tip of the catheter D. Contrast flows to the right atrium E. Contrast flows to the portal bifurcation
A. Contrast flows via large vessels to the periphery of the liver A transjugular intrahepatic portosystemic shunt reduces portal venous pressure. This may be of use in the management of variceal haemorrhage, refractory ascites and Budd–Chiari syndrome. The principal aim is to create a shunt between the hepatic vein and the portal vein. Usually a right internal jugular vein puncture is used to gain access to the hepatic vein. The difficulty lies in cannulating the portal vein, which may take a number of attempts, and angiography is used to determine position. Successful cannulation of the portal vein is shown by contrast flowing towards the periphery of the liver in large (portal vein) vessels. If contrast flows through small vessels towards the periphery of the liver, it would suggest that the hepatic artery has been cannulated. Where contrast pools at the tip of the catheter, the tip is no longer in a vessel and is within the liver parenchyma. If contrast flows to the right atrium, the tip is still within the hepatic vein and where contrast flows to the portal bifurcation, bile ducts have been cannulated. Kessel D, Robertson I. Interventional Radiology: A Survival Guide. 3rd edn. London, UK: Churchill Livingstone; 2010. p. 225.
961
A 50-year-old female with a history of rheumatoid arthritis presents with a 1-month history of slowly progressive breathlessness, low-grade fever and malaise. Her pulmonary function tests show a restrictive pattern. Sputum culture is negative for AAFB or any other organism. A chest radiograph is normal. High-resolution CT shows bilateral lower zone changes with mosaic attenuation and cylindrical bronchiectasis. What is the most likely diagnosis? A. Extrinsic allergic alveolitis B. Acute interstitial pneumonia C. Bronchiolitis obliterans D. Usual interstitial pneumonia E. Eosinophilic pneumonia
C. Bronchiolitis obliterans Bronchiolitis obliterans represents chronic inflammation of the bronchioles that causes obstruction of the bronchiolar lumen. Risk factors include connective tissue disorders and organ transplant recipients. The pulmonary changes are typically bilateral, subpleural and lower zone in distribution. Patchy ground glass opacities and centrilobular pulmonary nodules are common. These patients usually respond well to steroids. Extrinsic allergic alveolitis usually presents with ground glass changes and pulmonary nodules in a centrilobular distribution. However, there will be a history of exposure to allergen with predominantly upper lobe changes. In early stages, clinical and radiological findings of acute interstitial pneumonia and bronchiolitis obliterans can be similar. The high-resolution CT findings of usual interstitial pneumonia are septal thickening and traction bronchiectasis with areas of honeycombing in a subpleural distribution. Centrilobular nodules are not a feature. Eosinophilic pneumonia usually presents with acute febrile illness with markedly elevated levels of eosinophils in bronchoalvelolar lavage fluid. High-resolution CT typically shows peripheral ground glass changes with interlobular septal thickening. Lynch D, Travis WD, Müller NL, et al. Idiopathic interstitial pneumonias: CT features. Radiology 2005; 236: 10–21.
962
When performing angiography, which of the following parameters for contrast volume and injection rate is least appropriate to the vessel/region under investigation? A. Proximal superficial femoral artery: 15 ml at 8 ml/second B. Lateral foot: 20 ml at 5 ml/second C. Aortic arch: 40 ml at 20 ml/second D. Coeliac axis: 5 ml at 5 ml/second E. Gastroduodenal artery: 15 ml hand injected
D. Coeliac axis: 5 ml at 5 ml/second In order to achieve optimum images, contrast volume and flow rate must be adjusted according to the vessel under investigation. Larger calibre vessels with greater velocity flow will require larger contrast volumes and injection rates. The coeliac axis is the first midline branch from the abdominal aorta and 5 ml of contrast at 5 ml/s would not be satisfactory to adequately opacity it. 30 ml at 5 ml/s would be more suitable. Depending on patient size, contrast volume, and injection rates in the region of 10–15 ml at 5–10 ml/s is typical for angiography of the proximal superficial femoral artery. In the lateral foot, a greater volume is required to opacify the distal vessels (20 ml) although too great an injection rate will cause the patient pain (5 ml/s is satisfactory). In the aortic arch, the high velocity of blood within the vessel requires rapid injection of large contrast volumes: 40 ml at 20 ml/s. When angiography is performed to visualize the gastroduodenal artery, a combination of lower flow velocities and the typical clinical question to be answered (gastrointestinal bleeding) makes hand injection most appropriate. Kessel D, Robertson I. Interventional Radiology: A Survival Guide. 3rd edn. London, UK: Churchill Livingstone; 2010. pp. 100, 119, 131.
963
A 32-year-old male smoker is diagnosed with Langerhans cell histiocytosis after high-resolution CT. What are the likely imaging findings? A. Centrilobular, poorly defined, peribronchiolar micronodules sparing the bases with normal intervening lung and normal lung volume B. Multiple cysts of varying sizes and wall thickness and loss of volume C. Cavitating, upper lobe predominant nodules of varying sizes and mediastinal lymphadenopathy D. Solid predominantly basal soft tissue nodules of varying sizes with mediastinal and axillary lymphadenopathy E. Upper lobe predominant centrilobular nodules with coarse reticulations and mosaic perfusion on inspiratory or expiratory series
A. Centrilobular, poorly defined, peribronchiolar micronodules sparing the bases with normal intervening lung and normal lung volume These appearances are typical in the early stages of Langerhans’ cell histiocytosis. Centrilobular, granulomatous nodules progress to become cavitated and on to become thick walled cysts, which enlarge and cause honeycombing (fibrosis) and volume loss. It is much more common in young male smokers, although it does occur in women. Prognosis is good and with smoking cessation, treatment may not be required. Answer B describes the later stages of Langerhans’ cell histiocytosis and answer C, suggests pulmonary TB although the patient would usually be septic. Answer D describes the typical appearance of lung metastases and with this description, testicular and thyroid tumours should be excluded primarily. Answer E describes late phase hypersensitivity pneumonitis. In its acute and reversible phase, basally predominant airspaces shadowing prevails due to alveolar infiltration of neutrophils. Basally predominant nodules form in the subacute phase as interstitial infiltrate organizes with granuloma formation. As it is fibrotic, nodular and upper lobe predominant, chronic hypersensitivity pneumonitis should be excluded in these patients. Brauner MW, Grenier P, Tijani K, et al. Pulmonary Langerhans cell histiocytosis: evolution of lesions on CT scans. Radiology 1997; 204: 497–502.
964
A 9-year-old hypertensive boy presents with failure to thrive. MR angiogram shows smooth tapering of the supra-renal abdominal aorta with occlusion of the coeliac axis and superior mesenteric artery. The infra-renal aorta and iliac vessels are of normal calibre and disease free. What is the most likely diagnosis? A. Mid-aortic syndrome B. Aortic coarctation C. Takayasu’s arteritis D. Syphilitic aortitis E. Neurofibromatosis
A. Mid-aortic syndrome Mid-aortic syndrome is a rare non-inflammatory vascular abnormality of unknown aetiology. It tends to present in children and young adults. It can affect the renal arteries (causing hypertension) as well as the coeliac axis and superior mesenteric artery resulting in abdominal pain and failure to thrive. It usually begins in the infra-renal aorta, but can ascend – the inferior mesenteric artery is infrequently involved. Aortic coarctation presents with localized obstruction at the junction of the aortic arch and descending aorta. Takayasu’s arteritis more frequently affects oriental young adult females and presents with systemic signs. In syphilitic aortitis there is a thick aortic wall with saccular aneurysmal dilatation. Neurofibromatosis can cause the described MR angiogram findings, but there should be other findings to point you to this diagnosis, for example, neurofibromas, cafe-au-lait spots, axillary/inguinal freckling, and so on. Lewis VD, Meranze SG, Mclean GK, et al. The midaortic syndrome: diagnosis and treatment. Radiology 1988; 167: 111–13.
965
A 54-year-old male presents with severe central chest pain that radiates through to the interscapular region. The superior mediastinum is widened on his admission chest radiograph. A CT angiogram (non-contrast and arterial phase) demonstrates an aortic dissection. There is a tear in the intima of the aorta 2 cm above the coronary vessels with retrograde dissection to the left coronary and antegrade dissection to the descending thoracic aorta. The clinician wants to know the location of the entry point of the dissection. What would you say? A. Aortic annulus B. Sinotubular junction C. Mid-ascending aorta D. Aortic sinuses of Valsalva E. High-ascending aorta
D. Aortic sinuses of Valsalva It is important to understand the location of the entry point of an aortic dissection because the location of the dissection impacts on potential treatment options. From proximal to distal, useful locations to be aware of are: * Aortic annulus: the base of the aortic valve, i.e. the most caudal point in the aorta and the junction between the left ventricular outflow tract and the aorta. * Aortic sinus of Valsava: this is where the coronary arteries arise and tends to be the widest point of the aortic root, just above the aortic valve. * Sinotubular junction: the point where the bulbous aortic sinus meets the more tubular aorta proper. * Mid-ascending aorta: the mid point between the sinotubular junction and the exiting branch vessels. * High-ascending aorta: the level where the brachiocephalic artery arises. Freeman LA, Young PM, Foley TA, et al. CT and MRI assessment of the aortic root and ascending aorta. AJR Am J Roentgenol 2013; 200: W581–92. doi: 10.2214/AJR.12.9531.
966
A 30-year-old male is involved in a road traffic collision at 70 mph. He is hypotensive and the clinical team are concerned for an aortic injury. Contrast extravasation at the time of the study results in a non-contrast study that demonstrates widening of the upper mediastinum with high-density material adjacent to the aorta. There is a preserved fat-plane between the area of increased density and the aorta. What is the most likely diagnosis? A. Aortic dissection B. Intramural haematoma C. Mediastinal haematoma D. Developmental remnant E. Aortic transection
D. Developmental remnant The ‘developmental remnant’ represents the insertion of the ligamentum arteriosum a remnant of the ductus arteriosus. It lies on the inferior aspect of the aortic arch at the isthmus. In the trauma situation, aortic transaction (traumatic aortic rupture), dissection and mediastinal haematoma should all be considered. Aortic transaction most commonly occurs at the isthmus and results in hyperdense haematoma around the site of injury. In mediastinal haematoma, there is usually preservation of the fat-plane between the haematoma and vessel suggesting that there is integrity of the vessel wall. Aortic dissection, results from damage to the intima and blood that enters the media and strips the intima from the vessel wall creating a flap and two lumens that contain blood. There should not be associated mediastinal haematoma. Intramural haematoma is due to hypertensive rupture of the vasa vasorum with haematoma that may remain localized or propagate to simulate a dissection. It is hyperdense on unenhanced CT and on a cross-sectional view of the vessel, it will extend over half way round the wall. Steenburg SD, Ravenel JG, Ikonomidis JS, et al. Acute traumatic aortic injury: imaging evaluation and management. Radiology 2008; 248: 748–62. doi: 10.1148/radiol.2483071416
967
A 78-year-old male presents with anaemia and shortness of breath. An admission chest radiograph reveals a widened upper mediastinum. At upper gastrointestinal endoscopy, a duodenal ulcer is found and the endoscopist also notes posterior indentation of the mid-oesophagus. Concern remains about the widened mediastinum and bronchoscopy and CT is arranged. At bronchoscopy, there is narrowing in the anteroposterior diameter of the trachea above the carina but no endobronchial lesion. What is subsequently diagnosed at CT? A. Sarcoidosis B. Double aortic arch C. Lymphoma D. Enlarged hemiazygous vein E. Oesophageal duplication cyst
B. Double aortic arch The double aortic arch is a congenital vascular anomaly where the right dorsal aortic arch (present in foetal life) does not regress. It represents a complete vascular ring and usually causes symptoms because of compression of local structures. Hence, the vast majority of double aortic arches are diagnosed in childhood. However, they may persist into adulthood without symptoms. This patient has a widened upper mediastinum, with posterior indentation of the oesophagus and narrowing of the trachea all of which point to this as the diagnosis. Sarcoidosis and lymphoma can both cause mediastinal lymphadenopathy that may cause widening of the mediastinum and local compression. An enlarged hemiazygous vein may cause posterior indentation of the oesophagus and duplication cysts may cause local compressive features, but none of these diagnoses fits the complete picture. Kypson AP, Anderson CA, Rodriguez E, et al. Double aortic arch in an adult undergoing coronary bypass surgery: a therapeutic dilemma? Eur J Cardiothorac Surg 2008; 34: 920–1. doi: 10.1016/j.ejcts.2008.07.015.
968
A 21-year-old male who is known to have Marfan syndrome is admitted with breathlessness. He is otherwise well and not clinically compromised. His left hemithorax is hyperresonant and a chest radiograph confirms an apical pneumothorax that is 1 cm deep. What treatment is recommended? A. It is a small pneumothorax: he can safely be discharged B. It is a small pneumothorax: but he should be admitted for observation C. It is a small pneumothorax: aspiration should be performed D. A small gauge chest drain (10–14 ch) should be inserted E. No drain should be inserted; he should be referred to the cardiothoracic team
E. No drain should be inserted; he should be referred to the cardiothoracic team The important factor here is that the male has Marfan syndrome. The underlying connective tissue disease results in a propensity for lung bullae that may leak resulting in pneumothoraces. Despite the small size, it is unlikely to respond to aspiration and in most centres early surgical treatment is advocated. Since he is well, a chest drain is not required providing he is monitored appropriately – surgical bullecomy is the treatment of choice. The BTS guidelines split the treatment of pneumothoraces into those that are simple and those that are secondary to underlying disease. If simple and less than 2 cm deep, patient can be discharged providing follow-up is arranged. If symptomatic, they should be observed. If over 2 cm, aspiration should be performed. If that fails, repeat aspiration should be attempted. If that fails, drain insertion is suggested. For secondary pneumothoraces, aspiration should only be attempted in small (less than 2 cm) pneumothoraces and drains should be inserted in the rest. Hall JR, Pyeritz RE, Dudgeon DL, et al. Pneumothorax in the Marfan syndrome: prevalence and therapy. Ann Thorac Surg 1984; 37: 500–4. Henry M, Arnold T, Harvey J, et al. BTS guidelines for the management of spontaneous pneumothorax. Thorax 2003; 58: ii39–52. doi: 10.1136/thorax.58.suppl_2.ii39.
969
A 64-year-old male with ischaemic heart disease and heart failure is admitted to hospital because of worsening breathlessness and lower limb oedema. A chest radiograph demonstrates a radiopaque device projected over the left upper chest. Three wires descend from the upper chest into the mediastinum. One terminates in the right atrium, one descends towards the apex and one deviates to the left and one sits to the left of the spine below the left pulmonary artery. Which of the following best describe the device? A. Cardiac resynchronisation device B. Dual lead pacemaker and remnant lead from a previous device C. Implantable cardiac defibrillator D. Implantable cardiac defibrillator and remnant lead from a previous device E. Dual lead pacemaker and an ectopic lead that became lodged and was left in situ
A. Cardiac resynchronisation device Cardiac resynchronization devices (also known as biventricular pacemakers) are devices used in the treatment of heart failure in patients with bundle branch block and severe left ventricular dysfunction. Their leads sit in the right atrium, at the apex in the right ventricle and in the coronary sinus over the left ventricle. Once deployed, the leads are triggered in sync to ensure that left and right ventricular systole occurs at the same time and appropriately timed following atrial contraction. Implantable cardiac defibrillators are used in patients with life-threatening arrhythmia and have two leads, one of which is thicker than the other. If the extra lead were one that was left in from a previous device, it should be sitting at the right ventricular apex, no crossing the midline in the coronary sinus. Becker M, Altiok E, Ocklenburg C, et al. Analysis of LV lead position in cardiac resynchronization therapy using different imaging modalities. JACC Cardiovasc Imaging 2010; 3: 472–81. doi: 10.1016/j.jcmg.2009.11.016.
970
A chest radiograph of a cyanotic 1-day-old male child shows cardiomegaly, splaying of the carina and narrowing of the superior mediastinum. What is the most likely diagnosis? A. Tetralogy of Fallot B. Ventricular septal defect C. Atrial septal defect D. Transposition of the great arteries E. Pulmonary stenosis
D. Transposition of the great arteries The commonest cause of early presentation cyanosis is transposition of the great arteries. On chest X-ray (CXR), the classic appearance is of “egg on its side” with enlargement of the RA, right ventricle and left atrium from 2 weeks of age (left atrium enlargement displaces the left main bronchus superiorly, splaying the carina). Superior mediastinal narrowing is caused by hypoplasia of the thymus, a posterior and midline location of the pulmonary artery (PA), absence of the pulmonary trunk and midline/right-sided aorta. In the absence of atrial septal defect, venticular septal defect (VSD) or patent ductus arteriosus, transposition of the great arteries is fatal. In tetralogy of Fallot deoxygenated blood is delivered to the aorta through the VSD (high right heart pressures secondary to pulmonary stenosis), but cyanosis may be delayed depending on the severity of the abnormality. CXR classically shows a boot-shaped heart, which is of NORMAL size. There is an elevation of the apex due to right ventricular hypertrophy, pulmonary oligaemia, and a right aortic arch. VSD and atrial septal defect in the neonatal period will cause left to right shunt and therefore, no cyanosis. CXR is often normal. Pulmonary stenosis results in RV enlargement, an enlarged left PA and pulmonary trunk and a normal right PA on CXR. Ferguson EC, Krishnamurthy R, Oldham SA. Classic imaging signs of congenital cardiovascular abnormalities. Radiographics 27: 1323–34. doi: 10.1148/rg.275065148
971
An 18-year-old white male presents with sudden onset chest pain and shortness of breath. A chest X-ray reveals a left-sided pneumothorax and extensive subcutaneous emphysema as well as a few well-defined, high-attenuation subpleural nodules. He had an operation on his left thigh 9 months ago but has missed all of his follow-up appointments. What is the most appropriate next investigation? A. Bone scintigraphy B. CT chest, abdomen and pelvis C. Skeletal survey D. High-resolution chest CT E. MRI of the left thigh
A. Bone scintigraphy The underlying diagnosis here is metastatic osteosarcoma, which commonly metastasizes to the lungs and causes pneumothoraces. Other metastases that can cause pneumothoracies are pancreatic, adrenal and Wilms’ tumour. Osteosarcoma commonly presents in young males, with a vague history of pain, or pathological fracture. Prognosis is significantly worsened if metastases are present at diagnosis. Bone scintigraphy is the first step in detecting any further bony metastases, and should be followed by staging CT. Lung nodules are not specific for malignancy and therefore CT staging is inadequate as an isolated staging investigation. A CT chest, abdomen and pelvis would be used in conjunction with other investigations to stage osteosarcoma. Skeletal survey comprises a series of plain films. In paediatrics, it is used in the investigation of dysplasias, non-accidental injury and staging in Langerhans’ cell histiocytosis. In adults, they are used for the staging of multiple myeloma. High-resolution CT would provide more information about the lung disease, chest and the pneumothorax, but nothing about the rest of the body or bones. If this was a first presentation, MRI would be used at presentation of the original disease, to locally stage the tumour and plan for biopsy. Athanasou N, Hogendoorn PCW, Bielack S, et al. Bone sarcomas: ESMO Clinical Practice Guidelines for diagnosis, treatment and follow-up. Ann Oncol 2010; 21: v204–13.
972
A 16-year Asian girl has a chest X-ray after her uncle is diagnosed with tuberculosis. The X-ray shows a well-demarcated, 3 cm soft tissue lesion in the left lower lobe. She is fit and well and has been fully immunized. The mass is further investigated with CT, which reveals a round, thin-walled cystic lesion of slightly higher attenuation than water. There is no lymphadenopathy and the lungs are otherwise clear. What is the most likely diagnosis? A. Active pulmonary tuberculosis B. Intralobar sequestration C. Extralobar sequestration D. Bronchogenic cyst E. Congenital pulmonary airway malformation
D. Bronchogenic cyst This is a bronchogenic cyst, the commonest of the thoracic foregut malformations. They are nearly always incidental findings, although they can cause stridor, haemoptysis or dysphagia if large. Their contents are often of water density but can be mucoid and slightly higher attenuation. These cysts are usually mediastinal, but can occur within pulmonary parenchyma (as in this case). In acute and active tuberculosis in children, clinical features of sepsis are pronounced. An active focus of tuberculosis would also characteristically cavitate and be thick walled. Multiple foci are common. In sequestration, a focus of unaerated lung tissue does not communicate with the tracheobronchial tree and is supplied by systemic vasculature. The intralobar type present later (usually with recurrent infection), account for 75%, are usually in the left lower lobe and show multiple cysts and fibrosis on CT. The extralobar type account for the remainder and are usually diagnosed within the first few weeks of life, with an accessory lobe of unaerated normal lung tissue is invested in its own pleura. These are commonly located between the lower lobe and the hemidiaphragm and are more solid than cystic on CT. Congenital pulmonary airways malformation is usually symptomatic and presents as respiratory failure in neonates. Yoon Y, Lee K, Kim TS, et al. Intrapulmonary bronchogenic cyst: CT and pathologic findings in five adult patients. AJR Am J Roentgenol 2002; 179: 167–70. doi: 10.2214/ajr.179.1.1790167
973
A 28-year-old female with a history of spontaneous pneumothorax complains to her general practitioner of a long standing dry cough and increasing shortness of breath on exertion. Her chest X-ray shows hyperinflation and fine, bilateral reticular shadowing throughout both lungs. High-resolution CT shows extensive, thin-walled parenchymal cysts throughout both lungs and pleural thickening. What is the most likely diagnosis? A. Langerhans cell histiocytosis B. Lymphangiomyomatosis C. Asthma D. Bronchiectasis E. Tuberous sclerosis
B. Lymphangiomyomatosis Lymphangiomyomatosis is caused by abnormal smooth muscle proliferation in the lungs and exclusively affects females, usually of reproductive age. Hyperinflation is due to obstructive ventilatory deficiency and reticular shadowing is apparent because of the conglomeration of innumerable thin parenchymal cyst walls centrally and peripherally throughout both lungs. Spontaneous pneumothorax and chylous pleural effusions are common. Langerhans’ cell histiocytosis is most common in male smokers and although it presents with bilateral widespread parenchymal cysts, the costphrenic angles are spared and the cysts have varying wall thickness. Hyperinflation and pneumothorax are common in asthma, but patients usually present at a younger age and reticular shadowing is not a feature. In bronchiectasis, you would expect to see ring shadows and air bronchograms due to bronchial wall thickening, and volume loss with chronicity. Tuberous sclerosis is diagnosed in childhood and commonly presents with seizures, low IQ and facial angiofibromas. In the chest, lower zone predominant fibrosis (volume loss) differentiates tuberous sclerosis from lymphangiomyomatosis, although chylothorax and spontaneous pneumothorax make it an important differential diagnosis. Aberle DR, Hansell DM, Brown K, et al. Lymphangiomyomatosis: CT, chest radiographic, and functional correlations. Radiology 1990; 176: 381–7.
974
A 52-year-old male with a 8 cm abdominal aortic aneurysm undergoes endovascular aneurysm repair. The procedure is uncomplicated, but post-operative CT angiography reveals a type I endoleak. Haemoglobin measured on the same day as the CT is 2 g/dl lower than the preoperative sample. What is the best treatment option? A. Simple IV fluids and observe B. Transfuse and observe C. Angiographic branch vessel embolization D. Angiographic coil deployment or glue sac embolization E. Surgical repair
D. Angiographic coil deployment or glue sac embolization An endoleak is defined as persistent flow of blood within the aneurysm sac following endovascular aneurysm repair. The covered stent-graft should exclude the aneurysm from the circulation by allowing blood to bypass it via the conduit. Endoleak is a common complication of endovascular aneurysm repair (40%) and is seen at follow-up imaging with flow out with the stent graft. Endoleaks can be split into five groups: * Type I – leak at graft attachment site. * Type II – aneurysm sac filling via branch vessel (most common). * Type III – leak through a defect in graft. * Type IV – leak through the graft fabric as a result of graft porosity. * Type V – continued expansion of aneurysm sac without demonstrable leak. Types II and IV usually resolve spontaneously. Types I and III do not, and therefore require immediate treatment. Endoleak may become evident intra-operatively, or months (or years) later. Lifelong surveillance imaging is required, usually in the form of CT angiography. Type I endoleaks can be treated by coil deployment, the injection of a sclerosant, or with glue sac embolization. Brach vessel embolization may be used in type II endoleaks that do not thrombose spontaneously. Lu Q, Feng J, Yang Y, et al. Treatment of type I endoleak after endovascular repair of infrarenal abdominal aortic aneurysm. J Endovasc Ther 2010; 17: 687–93. Rosen RJ, Green RM. Endoleak management following endovascular aneurysm repair. J Vasc Interv Radiol 2008; 19: S37–43. doi: 10.1016/j.jvir.2008.01.017. Valenti D. Endoleak: Definition, diagnosis, and management. Vasc Embolother 2006; 235–51. doi: 10.1007/3-540-33256-1_19.
975
A 30-year-old female is admitted to the accident and emergency department with headache, dizziness and neck pain. She is neurologically intact, but noted to be hypertensive. Inflammatory markers are normal. A CT angiogram of the neck vessels is requested because of clinical concern for vertebral artery dissection: serum creatinine is 120 mmol/l. The intracranial component of the study is normal. The extracranial internal carotid arteries are dilated and have a string-of-beads appearance. Given the likely diagnosis, what is the next best radiological investigation? A. Diagnostic angiogram B. MR angiogram renal arteries C. MR angiogram neck D. Ultrasound abdomen E. MRI brain
B. MR angiogram renal arteries The underlying diagnosis here is fibromuscular dysplasia, a heterogeneous group of vascular disorders that are characterized by a non-inflammatory, non-atherosclerotic angiopathy of medium-sized arteries. It tends to affect women aged 30–50. The most common arteries to be involved are the renal arteries and their involvement is characterized by mild renal impairment and hypertension. Other arteries involved include the extracranial internal carotid arteries, vertebral arteries, iliac and mesenteric vessels. The string-of-beads appearance is typical of the disease. A CT angiogram has been performed that does not show any evidence of dissection (one of the complications of fibromuscular dysplasia). Her creatinine is elevated and she is hypertensive, suggesting renal artery involvement and hence, the next best test is one to assess them. Of the tests offered, an MR angiogram of the renal arteries would be most appropriate and should be followed by therapeutic angioplasty of the affected vessel. These lesions respond well to angioplasty and stent insertion is not required. Similar treatment of the carotids should be considered. Furie DM, Tien RD. Fibromuscular dysplasia of arteries of the head and neck: imaging findings. AJR Am J Roentgenol 1994; 162: 1205–9.
976
A 43-year-old female with a history of previous deep venous thrombosis presents with pleuritic chest pain. D-dimer is positive and she undergoes a CT pulmonary angiogram. No pulmonary embolism is demonstrated, but while tracing the pulmonary vessels, you note the left-sided pulmonary veins drain into the superior vena cava and the right atrium. The left-sided pulmonary veins are normal. Given the likely diagnosis, what coexisting abnormality would you expect? A. Atrial septal defect (sinus venosus) B. Ventricular septal defect C. Anomalous pulmonary arterial supply D. Atrial septal defect (ostium secundum) E. Pulmonary sequestration
A. Atrial septal defect (sinus venosus) This female has partial anomalous pulmonary venous return in which one or more of the pulmonary veins has an abnormal insertion into the right atrium, superior vena cava, inferior vena cava or coronary sinus. It usually affects the pulmonary veins from the same lung and usually on the right. It results in a left to right shunt with oxygenated blood returned to the right side of the heart. It does not usually result in symptoms and is often found incidentally following a chest X-ray or cross sectional imaging. Associations include atrial septal defect (usually sinus venosus type). In some cases, an anomalous vein that drains to the inferior vena cava appears as a vascular structure adjacent to the right heart border (the scimitar syndrome). In these patients, there is no associated atrial septal defect but instead, there is an association with pulmonary sequestration or anomalous arterial supply. It is not associated with ventricular septal defect. Ho ML, Bhalla S, Bierhals A, et al. MDCT of partial anomalous pulmonary venous return (PAPVR) in adults. J Thorac Imaging 2009; 24: 89–95. doi: 10.1097/RTI.0b013e318194c942
977
A 65-year-old male who was initially admitted with ventricular fibrilation has a permanent pacemaker inserted. The lead is seen to descend to the left of the cardiac contour within the mediastinum before making a sharp turn to the right. After further advancing the lead, the patent has a run of ventricular tachycardia, which resolves without treatment. After the arrhythmia has settled, the lead tip sits near the cardiac apex. Where is the lead tip? A. Pericardial space B. Coronary sinus C. Accessory hemiazygous D. Left superior vena cava E. Right ventricle
E. Right ventricle This patient has a patent left-sided superior vena cava (SVC) and this is why the lead descends to the left of the cardiac contour in the mediastinum. A patent left SVC is a remnantof the left anterior cardinal vein that has failed to regress. It more commonly occurs as a duplicated SVC than an isolated patent left SVC. When it exists with the normal right SVC, it tends to be the larger of the pair (two-thirds of cases). Drainage is variable, but can be directly into the right atrium, or via the oblique vein of Marshall, the coronary sinus or, in some cases, into the left atrium. In this case, the change in direction suggests that we have made it back to the right side of the heart and from there, into the right ventricle (which is where the lead would have been headed anyway). It is not unusual to have runs of ventricular tachycardia as the pacing wire traverses the tricuspid valve. It is very unusual to perforate the myocardium during line insertion, although possible where the myocardium is damaged following infarction. The accessory hemiazygous vein drains the left-sided posterior intercostal veins and does not usually join up to the left inominate vein. Kellman GM, Alpern MB, Sandler MA, et al. Computed tomography of vena caval anomalies with embryologic correlation. Radiographics 1988; 8: 533–56.
978
A 51-year-old female presents to the respiratory clinic with a 6-month history of worsening dyspnoea. It initially improved with corticosteroids, but has since relapsed. She is hypoxic but her chest radiograph is normal. A CT pulmonary angiogram is performed, but no pulmonary emboli are demonstrated. On lung windows, there is mosaic attenuation, particularly in the middle zones with more focal opacification at the lung bases. During the patient’s hospitalization, all radiographic and clinical manifestations resolved in several days and the patient was discharged. A. Allergic bronchopulmonary aspergillosis B. Hypersensitivity pneumonitis C. Eosinophilic pneumonia D. Usual interstitial pneumonia E. Respiratory bronchiolitis interstitial lung disease
B. Hypersensitivity pneumonitis This female has subacute hypersensitivity pneumonitis. She has worsening symptoms (dyspnoea) and signs (hypoxia) over 6 months with some response to steroids (typical of hypersensitivity). Despite being hypoxic, the chest radiograph is normal and the CT pulmonary angiogram does not demonstrate any pulmonary emboli. Moreover, after admission, symptoms and signs resolve suggesting an environmental component. The best fitting diagnosis here is hypersensitivity pneumonitis, which occurs as a result of response to an external allergen and can have acute, subacute or chronic (fibrotic) forms. Eosinophilic pneumonia presents with acute symptoms, ground glass change, septal thickening and rapidly improves with steroids. Allergic bronchopulmonary aspergillosis often presents similarly to asthma with centrilobular nodules and bronchiectasis at high-resolution CT. Usual interstitial pneumonia is the radiological term for the pathological process of idiopathic pulmonary fibrosis and is not described here. Respiratory bronchiolitis-associated interstitial lung disease is another interstitial pneumonia that is strongly linked to smoking and causes centrilobular nodules and ground glass change. None would spontaneously improve days after admission. Matar LD, Mcadams HP, Sporn TA. Hypersensitivity pneumonitis. AJR Am J Roentgenol 2000; 174: 1061–6.
979
A 87-year-old ex-plumber is referred to the respiratory clinic with worsening shortness of breath. He is a smoker and has an abnormal chest radiograph with a moderate sized left-sided pleural effusion. A CT confirms the pleural effusion and demonstrates areas of pleural thickening, particularly on the mediastinal pleural surface. There is no pleural calcification. What is the most likely diagnosis? A. Mesothelioma B. Bronchial carcinoma C. Asbestosis D. Benign pleural effusion E. Pleural plaque disease
A. Mesothelioma This patient has mesothelioma and is of the correct age and previous profession to have been exposed to asbestos. Features that suggest malignant pleural disease (mesothelioma or metastases) include nodular pleural thickening (greater than 1 cm) that involves the mediastinal pleural surface or is concentric. Only 20–30% of pleural plaques in mesothelioma are calcified. Benign effusion would not have associated pleural thickening and benign pleural plaques (that may or may not be calcified) should not be associated with pleural effusions or symptoms. Asbestosis is a chronic fibrotic lung disease caused by the inhalation of the tiny asbestos fibres, but this does not fit the description here. While bronchial carcinoma does have an increased incidence amongst asbestos workers, there is no mass or lymphadenopathy mentioned here. Leung AN, Müller NL, Miller RR. CT in differential diagnosis of diffuse pleural disease. AJR Am J Roentgenol 1990; 154: 487–92.
980
A 67-year-old female with a history of postmenopausal bleeding and radiological proven fibroids undergoes fibroid embolization. Several weeks following the procedure, she develops severe pain and has several episodes of bleeding per vagina. What is the most likely diagnosis? A. Tumour lysis syndrome B. Tubo-ovarian abscess C. Postembolization syndrome D. Fibroid passage E. Endometritis
D. Fibroid passage Complications following uterine fibroid embolization (UFE) are not uncommon and can be stratified into major and minor complications. Fibroid passage, as in this case, is the commonest major complication following UFE and occurs in ~2% of cases. It is associated with severe pain, infection or recurrent bleeding. Fibroids in contact with the endometrial surface are most commonly affected. Assessment of fibroid following embolization should be performed using MRI. The commonest complication is postembolization syndrome. It usually occurs within hours of the procedure, but may present several days later. It is caused by tissue hypoxia and cell death that causes the release of breakdown products and inflammatory mediators which induce pain, fever, a raised white cell count, nausea and vomiting. Supportive care and exclusion of an underlying infection is required and symptoms settle within 24–48 hours. Fever is low grade and there should not be vaginal discharge or uterine tenderness. Endometritis, tubo-ovarian abscess and tumour-lysis syndrome can all occur following UFE, but are much less likely. Kitamura Y, Ascher SM, Cooper C, et al. Imaging manifestations of complications associated with uterine artery embolization. Radiographics 2005; 25: S119–32.
981
A 62-year-old male with a history of head and neck squamous cell carcinoma has a CT scan 15 years after radical surgery, which was followed by radiotherapy. On lung windows, there is a fibrotic change in the right upper lobe with linear delineation between it and the surrounding normal lung. Within it is an area of consolidation with distal collapse. What is the underlying pathology? A. Atypical infection B. Radiation pneumonitis C. Radiation induced fibrosis D. Radiation induced tumour E. Lymphangitic carcinomatosis
D. Radiation induced tumour Radiation-induced lung disease (RILD) following radiotherapy is common. The resultant manifestation of disease is usually confined to the radiation field. In the acute phase, RILD tends to manifest as pneumonitis with ground-glass opacification or consolidation. In later phases, traction bronchiectasis, volume loss, and scarring becomes apparent. Traditionally, radiation-induced injury was though as having linear delineation between normal and abnormal lung (because of the field lines). However, radiotherapy advances and the use of oblique beam angles and the development of three-dimensional conformal radiotherapy mean that the distribution of these findings can be abnormal. Differential diagnosis includes infection, recurrent malignancy, lymphangitic carcinomatosis, and radiation-induced tumours. Findings not associated with RILD include pleural effusions (that appear after a period of time or enlarge); development of consolidation, masses or cavitation; and occlusion of bronchi within an area of radiation-induced fibrosis. Choi YW, Munden RF, Erasmus JJ, et al. Effects of radiation therapy on the lung: radiologic appearances and differential diagnosis. Radiographics 2004; 24: 985–97.
982
A 22-year-old male with a history of poorly controlled asthma is admitted with pleuritic chest pain and breathlessness. His initial chest radiograph is normal. His clinical risk score is low, d-dimer is raised and he proceeds to CT pulmonary angiogram. No emboli are demonstrated. However, there is an incidental vascular anomaly with a vessel passing between the trachea and oesophagus. What is the diagnosis? A. Total anomalous pulmonary venous return B. Aberrant left pulmonary artery C. Scimitar syndrome D. Right aortic arch and aberrant left subclavian artery E. Double aortic arch
B. Aberrant left pulmonary artery This patient has an aberrant left pulmonary artery, the so-called pulmonary sling. The left pulmonary artery arises from the right and passes posterior to the trachea en route to the left lung. As it passes between the trachea and oesophagus, an incomplete vascular ring is created around the trachea, which may cause compression (and his wheeze, misdiagnosed as asthma). Bilateral atelectasis in the upper lobes is seen, as is abnormal branching within the bronchial tree in some patients. With a right aortic arch and aberrant left subclavian artery, both limbs of the pulmonary tree pass anterior to the trachea. In a double aortic arch, a complete vascular ring around the trachea and oesophagus is seen. Scimitar syndrome is partial anomalous pulmonary venous return from the right lung and neither it nor total anomalous pulmonary venous return fit the vignette. Goo HW, Park IS, Ko JK, et al. CT of congenital heart disease: normal anatomy and typical pathologic conditions. Radiographics 2003; 23: S147–65. doi: 10.1148/rg.23si035501
983
A 25-year-old Asian male with a 3-month history of weight loss and abdominal discomfort underwent a contrast-enhanced CT examination of the abdomen and pelvis. This revealed multiple enlarged and peripherally enhancing but centrally hypodense portal and peripancreatic lymph nodes, ascites with a density of 30 HU, and concentric thickening of the terminal ileum and caecum. What is the most likely cause for these abnormalities? A. Whipple’s disease B. Tuberculosis C. Lymphoma D. Crohn’s disease E. Ulcerative colitis
B. Tuberculosis Lymphadenopathy is the most common manifestation of abdominal tuberculosis, with a characteristic pattern of mesenteric, portal and peripancreatic nodal enlargement, compared with lymphoma in which para-aortic nodal enlargement is more typical. Peripheral and homogeneous enhancement patterns are recognized features. Ileocaecal concentric thickening with an incompetent ileocaecal valve is typical of gut tuberculosis. When advanced, caecal retraction occurs due to mesenteric fibrosis, and multiple small bowel strictures give rise to a “napkin ring stenosis” appearance. Omental and mesenteric thickening, as well as ascites with a raised density (due to high protein and cellular content) are also found. Burrill J, Williams CJ, Bain G, et al. Tuberculosis: A radiologic review. Radiographics 2007; 27: 1255–73.
984
A 35-year-old male presented with a 24-hour history of abdominal pain and vomiting. He underwent contrast-enhanced CT of the abdomen and pelvis revealing dilated small bowel loops (diameter >3 cm) leading to a collapsed loop lateral to the lateral margin of the right rectus abdominis muscle within the anterior abdominal wall. A small volume of free fluid was also present but no free intraperitoneal gas. The large bowel appeared normal. What is the most likely cause of these abnormalities? A. Spigelian hernia B. Richter’s hernia C. Littre’s hernia D. Petit’s hernia E. Grynfeltt’s hernia
A. Spigelian hernia Spigelian hernias occur through the linea semilunaris, which lies between the lateral edge of the rectus abdominis muscle and the medial edge of the condensed sheaths of the lateral abdominal wall muscles. The hernial necks are usually small so the risk of strangulation is high and they are often bilateral. A hernia involving an incarcerated Meckel’s diverticulum is known as a Littre’s hernia. In a Richter’s hernia, only one edge of the bowel is trapped, such that although there is no obstruction, there is a risk of strangulation and subsequent perforation secondary to ischaemia. Petit’s hernia occurs through the inferior lumbar triangle, and Grynfeltt’s hernia through the superior lumbar triangle. Sinha R, Verma R. Multidetector row computed tomography in bowel obstruction. Part 1. Small bowel obstruction. Clin Radiol 2005; 60: 1058–67.
985
A 76-year-old patient is investigated for jaundice and is found to have intrahepatic biliary dilatation but no gallstones. A subsequent CT of the abdomen demonstrates biliary dilatation in both lobes of the liver with an abrupt cut off at the hilum. MRI and subsequent biopsy confirmed the presence of a cholangiocarcinoma. Which of the following is not considered to be a risk factor for cholangiocarcinoma? A. Ulcerative colitis B. Primary sclerosing cholangitis C. Clonorchis sinensis infection D. Caroli disease E. Primary biliary cirrhosis
E. Primary biliary cirrhosis Cholangiocarcinoma is a primary tumour arising from the bile duct epithelium and has a wide variety of clinical and imaging manifestations. Risk factors include: inflammatory bowel disease (10× increased risk), biliary lithiasis, primary sclerosing cholangitis, clonorchis infection (mainly in the Far East but most common cause worldwide), choledochal cyst, Caroli disease and thorotrast exposure. Cholangiocarcinoma can be confused with other primary hepatic tumours, but colour Doppler can help differentiation as cholangiocarcinomas are usually hypovascular whereas hepatocellular carcinomas are typically hypervascular. On unenhanced CT, tumours are usually hypoattenuating compared with normal hepatic parenchyma and show enhancement on delayed phase contrast enhanced imaging. Tumours are hypointense compared to liver parenchyma on T1 weighted imaging and hyperintense on T2 weighted scans and show delayed enhancement on T1 weighted, fat saturated post contrast imaging. MRI with MR cholangiography is better than CT at demonstrating intraductal lesions. Sainani NI, Catalano OA, Holalkere N, et al. Cholangiocarcinoma: current and novel imaging techniques. Radiographics 2008; 28: 1263–87.
986
Contrast-enhanced CT of the abdomen of a 45-year-old patient with lower abdominal pain and diarrhoea demonstrates circumferential bowel wall thickening of the caecum and terminal ileum with pericolonic inflammation. Clinical examination and blood tests show multiple cutaneous lesions and a CD4 count of <200 cells/µl. What is the most likely diagnosis? A. Typhlitis B. Ulcerative colitis C. Diverticulitis D. Appendicitis E. Ischaemic colitis
A. Typhlitis Typhlitis is also known as neutropenic colitis and is seen in neutropenic or immunocompromised patients. CT shows circumferential thickening of the caecum and ascending colon with involvement of the terminal ileum in some cases. The bowel wall can be of low attenuation due to oedema. Pericolonic mesenteric fat stranding is a common associated finding. CT can also demonstrate the complications such as pneumatosis, perforation and intra abdominal collections. Ulcerative colitis is typically left sided and rarely involves terminal ileum. Diverticulitis commonly affects the descending and sigmoid colon. CT findings include segmental colonic wall thickening with hyperaemia and pericolic fat stranding in the presence of diverticula in the involved segment. The CT appearances of appendicitis include a dilated appendix >6 mm, appendicolith and focal caecal inflammation. Ischaemic colitis typically affects elderly patients with a history of ischaemic heart disease. Watershed areas of the colon are particularly at risk of ischaemia secondary to hypoperfusion (splenic flexure and rectosigmoid junction). Younger patients usually have a history of trauma with hypovolaemia leading to right-sided colonic ischaemia. CT demonstrates circumferential wall thickening with fold enlargement. Low or high attenuation of the bowel wall can be seen indicating oedema or intramural haemorrhage respectively. Horton KM, Corl FM, Fishman EK. CT evaluation of the colon: inflammatory disease. Radiographics 2000; 20: 399–418.
987
A 65-year-old male patient is referred for transjugular intrahepatic portosystemic shunt (TIPS) insertion. Which of the following statements regarding TIPS is true? A. The principal indication is treatment of Budd-Chiari B. Covered stents are more likely to fail secondary to stenosis than uncovered stents C. Portal vein occlusion is a contra-indication to TIPS D. Access is obtained via the external jugular vein into the right or middle hepatic vein E. TIPS is unhelpful in the treatment of refractory ascites
C. Portal vein occlusion is a contra-indication to TIPS The main indications for transjugular intrahepatic portosystemic shunt (TIPS) are variceal haemorrhage resistant to medical or endoscopic ligation and refractory ascites. Other indications include hepatorenal and Budd-Chiari syndromes. Uncovered stents have a complication rate of 50% at 1 year. Covered stent patency at 1 year is up to 90%. It is important to ensure the hepatic vein is covered to the junction with the inferior vena cava to maintain patency. Portal vein patency has to be proven before undertaking TIPS. Percutaneous access is achieved via the right internal jugular vein or middle hepatic vein. Review of TIPS versus paracentesis in the treatment of refractory ascites shows that although TIPS achieves better control of ascites in 3–12 months, over 2 years there is an increased risk of encephalopathy in patients who have TIPS rather than paracentesis. Owen AR, Stanley AJ, Vijayananthan A, et al. The transjugular intrahepatic portosystemic shunt (TIPS). Clin Radiol 2009; 64: 664–74.
988
A neonate presents with vomiting after each feed. An abdominal X-ray is performed. The “double bubble” sign on a plain abdominal film of a neonate is concerning for all but one of the following? A. Duodenal atresia B. Annular pancreas C. Gastric duplication cyst D. Midgut volvulus E. Ladds band
C. Gastric duplication cyst The double bubble sign indicates duodenal obstruction with gas within the stomach and dilated proximal duodenum but no gas seen within the distal bowel. Common causes to be considered include duodenal atresia, midgut volvulus and annular pancreas with Ladd bands, duodenal web and duodenal duplication cysts being less common causes. Duodenal atresia typically presents after the baby begins feeding with frequent, bilious vomiting but little abdominal distension and is associated with Down’s syndrome in roughly 30% of cases. Annular pancreas is a congenital band of pancreatic tissue arising from the pancreatic head and surrounding the second part of the duodenum. Complete rings usually cause obstruction. Midgut rotation refers to malrotation of the intestine which can be distinguished from other causes using ultrasound and barium studies which demonstrate “corkscrewing” of the small intestine around the superior mesenteric artery with the superior mesenteric vein lying on the left of the artery on colour Doppler. Gastric duplication cysts are seen in 7% of gastrointestinal duplications. Frequently patients are asymptomatic but infants can present with abdominal pain and vomiting. Plain film and barium studies show a paragastric mass displacing the stomach and bowel, centred on the greater curve. Ultrasound shows a double layered cyst with an echogenic inner layer and hypoechoic outer layer of muscle. Berrocal T, Torres I, Gutiérrez J, et al. Congenital anomalies of the upper gastrointestinal tract. Radiographics 1999; 19: 855–72.
989
A 70-year-old female patient presents with abdominal pain and jaundice. Examination reveals tenderness in the right upper quadrant associated with a palpable right-sided abdominal mass. Ultrasound of the abdomen demonstrates a cystic structure at the porta hepatis separate from the gall bladder communicating with the common hepatic duct and intrahepatic ducts. Which of the following statements is true with regards to this patient’s diagnosis? A. The classic triad of abdominal pain, obstructive jaundice and right upper quadrant mass is seen in 75% of adult patients B. Majority of patients present in their third decade C. 99mTc pertechnetate is used to demonstrate connection between the cyst and biliary ducts D. There is an increased risk of cholangiocarcinoma E. Bile peritonitis is a frequent presenting complaint
D. There is an increased risk of cholangiocarcinoma Choledochal cyst is a congenital cystic dilatation of the common bile duct without involvement of gall bladder or cystic duct. Sixty percent of patients present in childhood but patients can present in their 70s. The classic triad of abdominal pain, obstructive jaundice and right upper quadrant mass is observed in 20–30% of adult patients. Other presentations include recurrent fevers, weight loss and pruritus. Ultrasound shows a cystic mass separate from the gall bladder and communicating with the common hepatic duct. Sixteen percent of patients have intrahepatic biliary dilatation secondary to stenosis. Seven percent of cases are detected antenatally with a right-sided cystic mass and dilated hepatic ducts within the foetal abdomen. Nuclear imaging using 99mTc acetanilide iminodiacetic acid analogues shows the cyst as a photopaenic area that gradually fills in over time. Radionuclide activity within the bowel is not always seen due to stasis within the dilated ducts. Complications of choledochal cysts include: stones in 8–50% (gallbladder, common bile duct, cyst itself, intrahepatic biliary ducts and pancreatic ducts), cancer of the gallbladder or biliary ducts, recurrent pancreatitis (33%), cholangitis/cholecystitis, cyst rupture leading to peritonitis (1.8%). Dahnert W. Liver Bile Ducts, Pancreas and Spleen. In: Radiology Review Manual. 6th edn. Philadelphia, PA: Lippincott Williams and Wilkins; 2007. pp. 703–4.
990
A 1-year-old boy is brought to the accident and emergency department by his parents who have noticed intermittent episodes of the child passing black stools. Which of the following statements is incorrect regarding Meckel’s diverticula? A. Intussusception occurs in 30% of patients with a Meckel’s diverticulum B. 99mTc pertechnetate scintigraphy is used to identify Meckel’s diverticula C. They occur in approximately 2% of the general population D. It results from incomplete involution of the omphalomesenteric duct E. Angiography can demonstrate the residual vitello-intestinal artery
A. Intussusception occurs in 30% of patients with a Meckel’s diverticulum Meckel’s diverticulum is the most common congenital abnormality of the small bowel occurring in 1–4% of the general population. They are found within the last 100 cm of small bowel and result from incomplete involution of the omphalomesenteric duct. The majority are asymptomatic in adults. Complications include bleeding, intussusception, obstruction, diverticulitis and perforation. Angiography shows the persistent vitellointestinal artery and can detect active bleeding at greater than 0.5 ml/minute. Scintigraphy (99mTc pertechnetate) shows activity within ectopic gastric mucosa within the diverticulum. False positives are caused by duplication cysts containing ectopic gastric mucosa, intussusception, volvulus and inflammatory bowel disease. False negatives can be caused by recent barium investigation or lack of ectopic gastric mucosa. Red cell scintigraphy can show gastrointestinal bleeding with rates of as little as 0.1 ml/min but is not specific for Meckel’s diverticula. Thurley PD, Halliday KE, Somers JM, et al. Radiological features of Meckel’s diverticulum and its complications. Clin Radiol 2009; 64: 109–18.
991
A 40-year-old patient presented with abdominal pain, diarrhoea and an urticarial rash. Chest radiograph revealed scattered, well-defined sclerotic lesions within the ribs. Abdominal ultrasound demonstrated hepato-splenomegaly. A barium follow through showed nodular, thickened folds within the small bowel. What is the most likely diagnosis? A. Amyloidosis B. Eosinophilic enteritis C. Mastocytosis D. Whipple’s disease E. Giardiasis
C. Mastocytosis Mastocytosis is a systemic disease with mast cell proliferation affecting the skin and reticuloendothelial system. Urticaria pigmentosa is the cutaneous form, seen in 80–90% of patients. Hyperpigmented skin lesions show a wheal and flare response when irritated. Skeletal involvement is seen in 70% of patients with symptoms and signs including bone and joint pain with osteoporosis. Scattered, well-defined sclerotic foci occur due to histamine release by mast cells promoting increased osteoblastic activity (skull, spine, ribs, pelvis). Splenomegaly is seen in 43–61%. Abdominal lymphadenopathy and hepatomegaly is also seen. The small bowel demonstrates “sand-like” mucosal nodules causing fold thickening within non-dilated bowel due to infiltration by mast cells. Amyloidosis, eosinophilic enteritis, Whipple’s disease and giardiasis are differential diagnoses for nodular thickening of the small bowel but they do not fit with the other findings in this patient. Dahnert W. Gastrointestinal Tract. In: Radiology Review Manual. 6th edn. Philadelphia, PA: Lippincott Williams and Wilkins; 2007. pp. 854–5. Davies S. Abdomen and Gastrointestinal Tract. In: Chapman and Nakielny’s Aids to Radiological Differential Diagnosis. London, UK: Saunders; 2009.
992
A 53-year-old male patient is referred by his general practictioner with a prolonged history of joint pain, recurrent abdominal pain and diarrhoea. An oesophago-gastro-duodenoscopy reveals pale shaggy yellow plaques in the post-bulbar duodenum. CT scan of the abdomen and pelvis shows bulky, low-density mesenteric and retro-peritoneal adenopathy with mucosal thickening of non-dilated jejeunum and duodenum, splenomegaly and ascites. Diagnosis was made following duodenal biopsy. What is the most likely diagnosis? A. Lymphoma B. Amyloidosis C. Sprue D. Whipple’s disease E. Intestinal lymphangiectasia
D. Whipple’s disease Whipple’s disease is a chronic, multisystem disease which is thought to be caused by infection with gram positive Tropheryma whippelii, typically affecting patients in their fourth to sixth decades with a male predominance. It can affect any organ system and presents with migratory arthralgia in 65–95% along with malabsportion, diarrhoea, steatorrhoea, weight loss, low-grade fever, generalized peripheral adenopathy and hyperpigmentation of the skin. CT findings include the above plus pleural and pericardial effusions and sacroiliitis. Treatment includes long term tetracyclines. Sprue shows marked bowel dilatation without mucosal thickening. Intestinal lymphangiectasia shows mucosal thickening throughout the small bowel. Amyloidosis causes small bowel dilatation and jejunalization of the ileum. Lymphoma rarely affects the duodenum (2%) with 51% affecting the ileum. Fantry GF, James SP. Whipple’s disease. Dig Dis 1997; 131: 108–18.
993
A 26-year-old female presented with right upper quadrant pain. A solid lesion was demonstrated in the liver on ultrasound. CT scan of the abdomen showed a peripheral well-defined, hypodense mass with avid contrast enhancement in the arterial phase and an isodense mass on the delayed phase in segment 8. Out-of-phase T1 weighted MRI showed homogenous decreased signal in the mass. What is the most likely diagnosis? A. Focal nodular hyperplasia B. Hepatic adenoma C. Hepatocellular carcinoma D. Focal fatty infiltration of the liver E. Hepatic lipoma
B. Hepatic adenoma Hepatic adenoma is a benign, encapsulated neoplasm that shows a propensity to haemorrhage and rarely undergoes malignant change. It most commonly occurs in young women taking oral contraceptives. Other risk factors include type I glycogen storage disease and use of anabolic steroids. The adenoma cells may be filled with glycogen and fat, giving rise to a hypodense lesion on unenhanced CT scan. Hepatic adenomas show transient avid enhancement in the arterial phase and are iso/hypodense on delayed phase images. On MRI, the lesion has T1 hyperintense areas due to the presence of fat/ haemorrhage and usually shows homogenous signal loss with out-of-phase imaging. At CT and MR imaging, focal nodular hyperplasia demonstrates brisk, intense, homogeneous contrast enhancement with rapid contrast material washout and may have a characteristic central scar that is bright on T2 weighted images and shows delayed contrast. Unlike the uniform fat deposition in adenomas, fat deposition in hepatocellular carcinomas is usually patchy. Hepatic lipomas are extremely uncommon and appear as well-circumscribed, uniformly hyperechoic lesions on ultrasound with negligible enhancement at contrast-enhanced CT and MR imaging. Focal steatosis is often easily recognized on the basis of the typical periligamentous or periportal location and the presence of non-distorted, traversing blood vessels. Prasad SR, Wang H, Rosas H, et al. Fat-containing lesions of the liver: Radiologic-pathologic correlation. Radiographics 2005; 25: 321–31.
994
A 50-year-old female was referred by her general practitioner for investigation due to mild derangement of her liver function tests. She is otherwise well. Ultrasound of the liver showed a lesion in the right hepatic lobe, which was further characterized with MRI. Which of the following features on imaging would not support a diagnosis of focal fatty infiltration? A. Well-defined homogenously hyperechoic lesion B. Periportal location of the lesion C. Distortion of traversing blood vessels D. Homogenously hyperintense lesion on T1 weighted MRI E. Homogenously hypointense lesion on out-of-phase T1 weighted MRI
C. Distortion of traversing blood vessels Liver steatosis can be focal or diffuse. Focal steatosis typically occurs in periligamentous or periportal areas and has nondistorted, traversing blood vessels. Patchy focal fat deposition or sparing can mimic infiltrating malignancy, as can multifocal nodular steatosis. In-phase and out-of-phase MRI can differentiate multifocal nodular steatosis from metastatic disease. Prasad SR, Wang H, Rosas H, et al. Fat-containing lesions of the liver: Radiologic-pathologic correlation. Radiographics 2005; 25: 321–31.
995
A 58-year-old male presented with a 2-week history of dysphagia and weight loss. Upper gastrointestinal endoscopy showed an ulcerating lesion in the distal oesophagus. The following statements are true regarding imaging in oesophageal carcinoma except: A. Fludeoxyglucose positron emission tomography (FDG-PET)/CT is used as standard practice in the initial staging of oesophageal carcinoma B. The N (nodal) stage in the TNM classification for oesophageal carcinoma is an important prognostic factor C. Standard FDG-PET/CT technique in the United Kingdom involves the administration of intravenous contrast media D. FDG-PET/CT is used to identify unsuspected metastatic disease E. Patient preparation for FDG-PET/CT includes a 6-hour fast
C. Standard FDG-PET/CT technique in the United Kingdom involves the administration of intravenous contrast media Oesophageal malignancy is commonly staged according to the TNM classification. Endoscopy remains the most effective method for the early detection of oesophageal tumours. PET/CT with 18F-FDG is a standard of investigative care for all patients pre-resection and is important in the evaluation of unsuspected metastatic disease, which is present in up to 30% of patients at initial diagnosis. Patient preparation includes a 6-hour fast, with an optimal blood glucose level of less than 10 mmol/l. The oesophagus is drained by a submucosal network of lymphatic channels, which drain to lymph nodes in the neck, upper abdomen, and retroperitoneum. The N-stage is an important prognostic factor as patients with nodal disease have significantly worse survival rates. Chowdhury FU, Bradley KM, Gleeson FV. The role of 18F-FDG PET/CT in the evaluation of oesophageal carcinoma. Clin Radiol 2008; 63: 1297–309.
996
A 45-year-old male with a history of alcohol abuse presented with acute abdominal pain. His abdomen was distended with marked tenderness in the epigastrium on clinical examination. Biochemical tests revealed an amylase level >1000, deranged liver and renal function tests. Contrast-enhanced CT of the abdomen and pelvis was performed. Which of the following findings is most in keeping with acute necrotizing pancreatitis? A. Ascites, enlarged pancreatic head with ill-defined, poorly enhancing body and tail of the pancreas B. Enlarged pancreatic gland with peripancreatic fat stranding and uniform enhancement of the pancreas C. Enlarged pancreatic gland with a low-attenuation collection and an enhancing capsule D. Ascites, enlarged pancreatic gland with extensive mesenteric and peripancreatic fat stranding E. Enlarged pancreatic gland with peripancreatic fat stranding and pancreatic calcification
A. Ascites, enlarged pancreatic head with ill-defined, poorly enhancing body and tail of the pancreas The most common cause of acute pancreatitis in the United Kingdom is attributed to alcohol misuse. The 2012 revision of the 1992 Atlanta classification of the disease divides acute pancreatitis into acute interstitial oedematous and acute necrotizing pancreatitis. It further divides acute pancreatitis into an early phase, during the first week, and a late phase, which occurs thereafter. Necrotic areas of the pancreas will be poorly enhancing. Local complications include the formation of acute collections, which are classified according to the presence or absence of necrosis and thus termed acute necrotic collections or acute peripancreatic fluid respectively. Peripancreatic fluid collections that develop capsular enhancement are termed pseudocysts in the absence of necrosis. Severity of acute pancreatitis is classified into mild, moderately severe, and severe to reflect extent of multi-organ failure. Thoeni RF. The revised Atlanta classification of acute pancreatitis: Its importance for the radiologist and its effect on treatment. Radiology 2012; 262: 751–64.
997
A 58-year-old male who recently returned from a holiday in New Zealand presented with abdominal pain. Ultrasound of the abdomen was performed and showed a hypoechoic lesion with echogenic internal septae in the liver. MRI performed showed this lesion to have a hypointense rim on T1 and T2 weighted MRI with peripheral cysts which are hypointense relative to the matrix of the lesion on T1 weighted imaging. What is the most likely diagnosis? A. Pyogenic hepatic abscess B. Cat-scratch disease C. Hydatid cyst D. Schistosomiasis (Schistosoma mansoni) E. Miliary tuberculosis
C. Hydatid cyst Hydatid disease is a parasitic disease that is endemic to the Mediterranean basin and other sheep-raising areas. Ultrasound findings range from purely cystic to solid-appearing pseudotumours. Wavy bands of delaminated endocyst may be noted internally and daughter cysts are frequently seen. Calcifications are often present peripherally. At CT, it appears as a well-defined, hypodense lesion with a distinguishable wall. Coarse wall calcifications are present in 50% of cases, and daughter cysts are identified in approximately 75%. On MRI, the pericyst is seen as a T1 and T2 hypointense rim, the matrix appears hypointense on T1 weighted images and markedly hyperintense on T2 weighted images and daughter cysts are T1 and T2 hypointense relative to the matrix. Pyogenic abscesses may manifest as either discrete hypoechoic nodules or ill-defined areas of distorted hepatic echogenicity at ultrasound and as well-defined hypoattenuating lesions with faint rim enhancement and perilesional edema at contrast-enhanced CT. Miliary tuberculosis is usually not detected at imaging, and hepatomegaly may be the only abnormality. In the healing stages, CT may show diffuse hepatic calcification. In chronic schistosomiasis due to Schistosoma mansoni, a “bull’s-eye” appearance represents an anechoic portal vein with wall thickening surrounded by an echogenic mantle of fibrous tissue. At CT, periportal fibrosis manifests as low-attenuation rings around the portal vein branches throughout the liver, with marked enhancement following intravenous administration of contrast material. At MRI, the periportal bands are T1 isointense relative to normal liver with marked enhancement following contrast administration. The hallmark of cat scratch disease is lymphadenopathy. In disseminated disease, multiple granulomata may be seen in the liver and spleen which are hypodense on CT with a variable enhancement pattern. Mortelé KJ, Segatto E, Ros PR. The infected liver: radiologic-pathologic correlation. Radiographics 2004; 24: 937–55.
998
A 35-year-old female was found to have an incidental lesion in the liver on CT scan of the abdomen. The following statements are true regarding focal nodular hyperplasia except? A. It is the second most common benign solid hepatic lesion B. It is often discovered incidentally C. It has a central scar which is hyperintense on T2 weighted MRI D. It has a central scar which is hypointense on T2 weighted MRI E. Ultrasound colour Doppler shows a “spoke-wheel” pattern of internal vascularity
D. It has a central scar which is hypointense on T2 weighted MRI Focal nodular hyperplasia (FNH) is the second most common benign hepatic lesion following haemangioma. It occurs predominantly in young asymptomatic women and is often discovered incidentally. In contrast to adenomas, which haemorrhage and have malignant potential, FNH exhibits benign behaviour. At ultrasound, FNH is typically isoechoic to liver parenchyma. Color and power Doppler ultrasound often show a characteristic “spoke-wheel” pattern of internal vascularity. At CT and MR imaging, FNH demonstrates brisk, intense, homogeneous contrast enhancement with rapid return to background parenchymal density/signal. A central scar that is hyperintense on T2 weighted MRI and shows delayed contrast enhancement is a characteristic feature, but not always present. A central scar which is hypointense on T2 weighted MRI is a feature of fibrolamellar carcinoma. Prasad SR, Wang H, Rosas H, et al. Fat-containing lesions of the liver: radiologic-pathologic correlation. Radiographics 2005; 25: 321–31.
999
A 45-year-old male with a liver transplant was found to have deteriorating liver function tests on his third postoperative day. A liver ultrasound with colour Doppler was performed. Which of the following is the most likely vascular cause of his deterioration? A. Hepatic artery thrombosis B. Hepatic artery stenosis C. Portal vein thrombosis D. Portal vein stenosis E. Inferior vena cava thrombosis
A. Hepatic artery thrombosis The commonest causes of graft failure are related to vascular complications, which usually occur in the early post-operative period and are well-evaluated with gray-scale and Doppler ultrasound. Hepatic artery thrombosis occurs in up to 60% of all post-transplantation vascular complications and is associated with a mortality of 20–60%. Early hepatic artery thrombosis occurs within 15 days of transplantation. Portal vein thrombosis and stenosis occur in 1–2% of transplant recipients. Inferior vena cava thrombosis and stenosis are diagnosed in less than 1% of transplant cases and seen in cases of re-transplantation and in paediatric cases. Crossin JD, Muradali D, Wilson SR. US of liver transplants: Normal and abnormal. Radiographics 2003; 23: 1093–114.
1000
A 65-year-old male presented with a 1-month history of worsening dysphagia. He is known to have gastro-oesophageal reflux disease. Upper gastrointestinal barium examination was performed. Which of the following findings are least likely to be associated with a benign oesophageal stricture? A. Hiatus hernia B. Shouldering with nodular narrowing of the distal oesophagus C. Short concentric narrowing in the distal oesophagus D. Fixed transverse folds in the distal oesophagus E. Pseudodiverticula of the oesophagus
B. Shouldering with nodular narrowing of the distal oesophagus Benign strictures of the oesophagus commonly arise from gastro-oesophageal reflux disease. These “peptic” strictures typically appear as smooth, tapered, concentrically narrowed lesions in the distal oesophagus, up to 4 cm in length. Asymmetric scarring and narrowing is often associated with sacculations. Longitudinal scarring from reflux oesophagitis can cause fixed transverse folds with barium trapped inbetween, producing a “stepladder” appearance. Hiatal hernias are very often seen in patients with peptic strictures. If there is a distal oesophageal stricture in the absence of a hernia, a malignant tumour should be considered. Malignant strictures are usually more irregular and nodular with more “shouldered” proximal and distal margins compared with benign peptic strictures. Luedtke P, Levine MS, Rubesin SE, et al. Radiologic diagnosis of benign esophageal strictures: a pattern approach. Radiographics 2003; 23: 897–909.
1001
A 78-year-old female patient presented with weight loss, general malaise and abdominal swelling. An abdominal CT was performed that revealed ascites and multiple calcified metastases within the liver. Which of the following is least likely to be the primary tumour? A. Colorectal B. Breast C. Thyroid D. Lung E. Melanoma
D. Lung Calcified hepatic metastases are most frequently associated with mucin-producing neoplasms such as colon, breast, stomach and ovary. Although these are the most common, a wide variety of other primary tumors are associated with calcified liver metastases such as melanoma, thyroid carcinoma, chondrosarcoma, carcinoid, leiomyosarcoma and neuroblastoma. These calcifications may be central or peripheral in location and are found in areas of reduced attenuation on CT scans. Calcification within liver metastases may also be seen as a result of radiation treatment or systemic chemotherapy. Stoupis C, Taylor HM, Paley MR, et al. The rocky liver: radiologic-pathologic correlation of calcified hepatic masses. Radiographics 1998; 18: 675–85.
1002
A 46-year-old Asian female who has recently arrived from India presents with acute abdominal pain and fever. On examination, she is febrile and tender in the right iliac fossa. Ultrasound examination was inconclusive and a CT abdomen was performed. This demonstrated appendiceal mural thickening, peri-appendiceal fat stranding and thickening of the pole of the caecum, with a calcific density seen within the appendix. The most likely diagnosis is: A. Tuberculosis B. Acute appendicitis C. Acute diverticulitis D. Right ureteric calculus E. Crohn’s disease
B. Acute appendicitis The most likely diagnosis is acute appendicitis. Appendiceal wall thickening and increased enhancement with periappendiceal fat stranding are important signs of appendicitis as the normal appendix can measure up to 11 mm in diameter due to physiological distension and use of appendiceal diameter without secondary signs is unreliable. A calcified appendicolith may be seen in up to 30% of cases. Localized tuberculosis of the appendix is rare and inflammation is typically in association with changes in the ileum and right colon, and less frequently an isolated finding. Isolated Crohn’s disease of the appendix is very rare and clinically may mimic acute appendicitis. On CT appendiceal Crohn’s disease shows wall thickening and periappendiceal fat stranding similar to acute appendicitis. Reactive thickening of terminal ileum and adjacent loops of bowel in the latter may be difficult to differentiate from adjacent Crohn’s disease, although the presence of skip lesions can help. Acute diverticulitis commonly affects the sigmoid and descending colon. Whitley S, Sookur P, McLean A, et al. The appendix on CT. Clin Radiol 2009; 64: 190–9.
1003
An 82-year-old female patient presented with gradually worsening abdominal pain and nausea followed by bilious vomiting and right upper quadrant pain. Abdominal ultrasound demonstrated a focus of increased echogenicity with posterior acoustic shadowing in the gallbladder with multifocal punctate areas of increased echogenicity with ring-down artifact noted scattered throughout the liver parenchyma, thought to represent pneumobilia. CT confirmed the presence of a dilated, fluid-filled stomach and pneumobilia. In addition, a high-attenuation focus was noted in the duodenum. What is the most likely diagnosis? A. Bouvret syndrome B. Mirizzi’s syndrome C. Gallstone ileus D. Perforated peptic ulcer E. Pancreatitis
A. Bouvret syndrome Bouveret syndrome is a gastric outlet obstruction produced by a gallstone impacted in the distal stomach or proximal duodenum. It occurs most commonly in elderly women. Classic gallstone ileus often refers to an obstructing stone localized to the terminal ileum. A duodenal location accounts for only 2–3% of cases. Gallstone ileus occurs in 15% of patients with a biliary-enteric fistula. The presenting clinical situation is variable but often includes nausea, vomiting and epigastric pain. Mirizzi’s syndrome is when inflammation related to a stone impacted in the cystic duct causes narrowing of the common hepatic duct and subsequent biliary obstruction. Brennan GB, Rosenberg RD, Arora S. Bouveret syndrome. Radiographics 2004; 24: 1171–5.
1004
A 59-year-old male presents with epigastric pain, fatigue, dry mouth and jaundice. A CT abdomen was performed which revealed diffuse enlargement of the pancreas, moderate enhancement, and sharp borders, with the absence of normal pancreatic clefts (featureless). Peripancreatic fat stranding was minimal. MR cholangiopancreatography showed narrowing of the main pancreatic duct and biliary strictures. The most likely diagnosis is: A. Autoimmune pancreatitis B. Pancreatic adenocarcinoma C. Primary sclerosing cholangitis D. Primary biliary cirrhosis E. Acute pancreatitis
A. Autoimmune pancreatitis Chronic pancreatitis caused by an autoimmune mechanism has been termed autoimmune pancreatitis. There is an association between autoimmune pancreatitis and other autoimmune disorders such as Sjogren syndrome, primary sclerosing cholangitis, primary biliary cirrhosis, ulcerative colitis, and systemic lupus erythematosus. Clinical features are non-specific and include upper abdominal pain, easy fatigability and jaundice. There is a strong male preponderance. Imaging features include focal or diffuse enlargement of the pancreas at endoscopic ultrasound, CT and MR imaging. Peripancreatic fat stranding is minimal suggesting a low-grade inflammatory reaction. In the more common diffuse form, the classic appearance is a mildly enlarged pancreas (sausage shaped), with homogeneous attenuation, moderate enhancement, and sharp borders, with the absence of normal pancreatic clefts (featureless) and a peripheral rim of hypoattenuation possibly representing inflammatory exudates. Some patients have biliary and pancreatic ductal strictures on MR cholangiopancreatography, with minimal thickening and irregularity of the ducts. The involvement of the main pancreatic duct is characteristic. Sahani DV, Kalva SP, Farrell J, et al. Autoimmune pancreatitis: Imaging features. Radiology 2004; 233: 345–52.
1005
A 68-year-old female with a recent history of diarrhoea presents with right upper quadrant pain, high fever and rigors. There is no history of travel. An ultrasound revealed multiple hypoechoic lesions. A contrast-enhanced CT showed multiple small, well-defined hypoattenuating lesions with faint rim enhancement and perilesional oedema. The most likely diagnosis is: A. Hydatid cysts B. Pyogenic abscesses C. Amoebic abscesses D. Miliary Tuberculosis E. Echinococcus multilocularis cyst
B. Pyogenic abscesses Pyogenic abscesses may be caused by haematogenous dissemination (of either gastrointestinal infection via the portal vein or disseminated sepsis via the hepatic artery), ascending cholangitis, or superinfection of necrotic tissue. At ultrasound, pyogenic microabscesses (<2 cm) may manifest as either discrete hypoechoic nodules or ill-defined areas of distorted hepatic echogenicity. At contrast-enhanced CT, they appear as multiple small, well-defined hypoattenuating lesions with faint rim enhancement and perilesional oedema. Patients with amoebic abscess are usually more acutely ill, younger and are from high-prevalence areas or recent travelers to such areas. In Hydatid disease, ultrasound findings are variable, ranging from purely cystic to solid-appearing pseudotumours. At CT, they appear as a well-defined, hypoattenuating lesion with a distinguishable wall, coarse wall calcifications in 50% and daughter cysts in approximately 75%. Echinococcus multilocularis lesions manifest as multiple echogenic nodules with irregular and indistinct margins on ultrasound. CT images typically display multiple irregular, ill-defined lesions scattered throughout the involved liver that are generally hypoattenuating with little or no enhancement after bolus administration of contrast. Miliary tuberculosis is usually not detected at imaging, and hepatomegaly may be the only radiologic abnormality. Mortelé KJ, Segatto E, Ros PR. The infected liver: radiologic-pathologic correlation. Radiographics 2004; 24: 937–55.
1006
A 72-year-old male presents to the accident and emergency department with painless obstructive jaundice. A trans-abdominal ultrasound is performed showing a mass in the pancreatic head. A CT is performed that shows a hypoattenuating solid mass in the head of the pancreas. Which of the following features would be considered an absolute contraindication to curative surgery? A. 2 cm tumour B. Peripancreatic lymphadenopathy C. 30% contiguity between tumour and vessel D. Vascular encasement of 270° E. Invasion of splenic vessels
D. Vascular encasement of 270° Pancreatic adenocarcinoma appears as a hypo-attenuating solid mass on CT. Metastases, malignant ascites and gross local invasion are contraindications to curative surgery. Tumour size has direct correlation with outcome and masses greater than 3 cm in diameter are viewed with caution. Arterial and venous involvement may also be signs of inoperability. Vascular encasement of 270° or more and wall irregularity are considered as unequivocal signs of unresectability. Direct involvement of the coeliac, superior mesenteric and gastroduodenal arteries and the superior mesenteric and portal veins is evidence of advanced local disease. Invasion of the spleen and splenic vessels is not usually seen as an absolute contraindication as these can be resected at laparotomy. Smith SL, Rajan PS. Imaging of pancreatic adenocarcinoma with emphasis on multidetector CT. Clin Radiol 2004; 59: 26–38.
1007
A 23-year-old female presents to the accident and emergency department with severe abdominal pain and diarrhoea. She had a previous admission 1 year before when she was diagnosed with a left renal calculus. Her serum corrected calcium is elevated at 3.1 mmol. An ultrasound of the neck was performed revealing two well-defined, oval hypoechoic masses posterior to the thyroid gland with corresponding increased uptake on sestamibi scan. A CT scan of the abdomen showed a 1 cm lesion in the head of the pancreas, with avid enhancement in the arterial phase. Which of the following is the most likely cause of the pancreatic lesion? A. Insulinoma B. Gastrinoma C. Glucagonoma D. Adenocarcinoma E. VIPoma
B. Gastrinoma The features described are consistent with a diagnosis of multiple endocrine neoplasia type 1 (MEN 1). Parathyroid, pancreatic and pituitary tumours are the major components of the disease. Primary hyperparathyroidism occurs in more than 95% of cases. In MEN 1, most pancreatic tumours are functional. Gastrinomas account for 60%, with major comorbidity due to associated Zollinger–Ellison syndrome. Insulinomas account for 30% and coexist with gastrinomas in 10%. Other tumour types are rare. The majority of pancreatic tumours in MEN 1 are small (<2 cm) and may be multiple. They are typically isoattenuating at unenhanced CT with avid enhancement in the arterial phase. Scarsbrook AF, Thakker RV, Wass JA, et al. Multiple endocrine neoplasia: spectrum of radiologic appearances and discussion of a multitechnique imaging approach. Radiographics 2006; 26: 433–51.
1008
A 35-year-old male with a history of ulcerative colitis presents with a 3-month history of pruritis, fatigue, abdominal pain and diarrhoea. On examination, he is jaundiced. His liver function tests are deranged with raised bilirubin and alkaline phosphatase. Which of the following MRI sequences would help to confirm the diagnosis? A. Heavily T2 weighted B. Heavily T1 weighted C. STIR D. HASTE E. FLAIR
A. Heavily T2 weighted Primary Sclerosing Cholangitis is an idiopathic, chronic, fibrosing inflammatory disease of the bile ducts which has a strong association with inflammatory bowel disease, especially ulcerative colitis (70% of cases). Seventy-five percent of patients present with progressive fatigue, pruritus, or jaundice. MR cholangiopancreatography is used in the detection of biliary disease. By using heavily T2 weighted sequences, the signal of static or slow-moving fluid-filled structures such as the bile and pancreatic ducts is greatly increased, resulting in increased duct-to-background contrast. Vitellas KM, Keogan MT, Freed KS, et al. Radiologic manifestations of sclerosing cholangitis with emphasis on MR cholangiopancreatography. Radiographics 2000; 20: 959–75; quiz 1108–9, 1112.
1009
A 50-year-old male presents to the general practictioner with a 2-month history of intermittent palpitations and arthralgia. He is noted to be hyperpigmented with hepatomegaly. Blood tests reveal a raised glucose. Abdominal ultrasound reveals an enlarged liver. Which of the following would not be consistent with a diagnosis of primary haemachromatosis on MR? A. Significant signal loss in liver on T2 weighted imaging B. Normal bone marrow signal C. Signal intensity of spleen less than muscle D. Normal pancreatic signal intensity in non-cirrhotics E. Normal signal intensity of spleen
C. Signal intensity of spleen less than muscle Primary haemochromatosis is a hereditary autosomal recessive disease with increased intestinal absorption and normal dietary intake of iron. Iron deposits preferentially in the liver, pancreas, heart, pituitary gland, thyroid, and synovium. Secondary haemochromatosis occurs secondary to parenteral administration of iron (e.g., repeated blood transfusions) with iron predominantly deposited in the reticuloendothelial system (spleen and Kupffer cells). In secondary haemachromatosis, the signal intensity of spleen is therefore less than muscle. Tissue accumulation of iron ions results in shortening of the longitudinal relaxation time (T1), the transverse relaxation time (T2), and particularly the transverse relaxation time as affected by magnetic field inhomogeneity (T2*) due to the superparamagnetic properties of the ions. This effect causes loss of signal intensity in the affected organs proportional to iron deposition. Queiroz-Andrade M, Blasbalg R, Ortega CD, et al. MR imaging findings of iron overload. Radiographics 2009; 29: 1575–89.
1010
A 28-year-old male presented with a 2-week history of epigastric abdominal pain, vomiting, diarrhoea and swelling of his legs. He had a history of gastroesophageal reflux disease. On chest radiographs and abdominal ultrasound images, bilateral pleural effusions and ascites were seen. Blood tests revealed hypoproteinaemia and hypoalbuminaemia. CT images revealed diffusely enlarged rugal folds, projecting into the lumen of the gastric fundus and body with relative sparing of the antrum. No lymph node enlargement was observed. A barium swallow upper gastrointestinal series with small bowel follow-through showed markedly thickened and lobulated mucosal folds, predominantly seen in the gastric fundus and body and, to a lesser extent, in the antrum. No discrete mass, stricture, ulceration or obstruction was seen. The small bowel was normal. The most likely diagnosis is: A. Zollinger–Ellison B. Menetriers C. Lymphoma D. Amyloidosis E. Gastric varices
B. Menetriers Patients with Ménétrier disease most often present with epigastric pain and hypoalbuminemia secondary to a loss of albumin into the gastric lumen. On enhanced CT, the gastric mucosa is thickened with intervening areas of normal mucosa. On barium studies, the rugal folds appear enlarged with contrast seen as intervening linear strands, trapped between the folds. Coating of the mucosa may be impaired by hypersecretion. Ulcers are common in Zollinger–Ellison syndrome. Fold thickening tends to be most significant along the greater curvature in Ménétrier disease. Fold thickening in lymphoma is more often seen in the distal stomach and lesser curvature as well as splenomegaly or extrinsic compression of the gastric lumen by enlarged nodes. A serpentine appearance is more suggestive of varices and usually seen in the cardia and fundus. Nodular thickening is more in keeping with amyloidosis. Friedman J, Platnick J, Farruggia S, et al. Ménétrier disease. Radiographics 2009; 29: 297–301.
1011
A 19-year-old male with right iliac fossa pain and weight loss underwent MR enterography. Abnormality of the terminal ileum was present. Which of the following findings is least suggestive of active inflammatory bowel disease? A. Linear high signal protrusion from mucosa into bowel wall B. Increased enhancement of mesenteric fat C. Prominent distended arcade of vessels in mesentery D. Intermediate submucosal signal intensity on fat suppressed T2 images E. Reduced submucosal signal intensity on fat suppressed T2 images
E. Reduced submucosal signal intensity on fat suppressed T2 images MR enterography is useful for the evaluation of small bowel disease, allowing multiplanar imaging without using ionizing radiation which is of particular relevance given the relatively young age of patients with Crohn’s disease. Signs of active inflammatory disease include bowel wall thickening, mural and mesenteric oedema (high or intermediate signal on fat suppressed T2 images), intense transmural or a stratified pattern of mural enhancement, prominent mesenteric vessels (“comb sign”) and enhancement of mesenteric fat and lymph nodes. Reduced submucosal signal intensity on fat suppressed T2 images is found in chronic disease due to submucosal fat deposition or fibrosis. Endoscopy and double contrast barium techniques remain superior in evaluation of early mucosal changes, but ulceration is sometimes depicted as linear signal change extending into the bowel wall. MR enterography also allows evaluation of extra-luminal pathology such as sinus tracks, fistulae and abscesses. Tolan DJ, Greenhalgh R, Zealley IA, et al. MR enterographic manifestations of small bowel Crohn disease. Radiographics 2010; 30: 367–84.
1012
A 79-year-old male with a background of ischaemic heart disease presents with sudden onset abdominal pain. Intravenous contrast-enhanced CT of the abdomen and pelvis was performed. Diffuse thickening of the distal small bowel and proximal large bowel was observed, as well as gas within the wall of the terminal ileum. What is the most common cause of bowel ischaemia? A. Superior mesenteric artery thrombosis B. Radiation injury C. Superior mesenteric artery embolus D. Superior mesenteric vein thrombosis E. Bowel obstruction
C. Superior mesenteric artery embolus Colonic ischaemia and infarction has many different causes with similar imaging signs. Findings include bowel wall thickening, a layered or target-like appearance to the wall due to submucosal haemorrhage or oedema, mesenteric venous engorgement or mesenteric oedema, and bowel dilatation. Enhancement of the bowel wall may be increased or decreased. Filling defects may be observed in the supplying arteries or draining veins. Intramural gas is caused by mucosal breakdown and dissection of gas across the bowel wall; this is sometimes followed by translocation of gas into the draining veins. The most common cause is superior mesenteric artery embolus in approximately 50% of patients. Other causes include superior mesenteric artery or superior mesenteric vein thrombosis, transient hypoperfusion, bowel obstruction, vasculitis, involvement of vessels by intra-abdominal inflammatory processes, chemotherapy, radiotherapy, and trauma. Rha SE, Ha HK, Lee SH, et al. CT and MR imaging findings of bowel ischemia from various primary causes. Radiographics 2000; 20: 29–42.
1013
A 55-year-old male with abdominal pain underwent intravenous contrast-enhanced CT of the abdomen and pelvis. An exophytic 10 cm solid lobulated mass was noted arising from the stomach wall with multiple irregularly marginated hypodense liver lesions, in keeping with metastatic gastrointestinal stromal tumour, confirmed with biopsy. He commenced treatment with imatinib mesylate and a follow-up intravenous contrast-enhanced CT was performed at 3 months. Which of the following imaging signs in relation to the liver disease is most suggestive of disease progression? A. Stable size of liver lesions with reduced density B. New low-density liver lesions C. Increased size of liver lesions with reduced density D. Reduced size of liver lesions with reduced density E. New enhancing nodule within hypodense lesion
E. New enhancing nodule within hypodense lesion Gastrointestinal stromal tumours are the most common mesenchymal tumours of the gastrointestinal tract, with 60–70% arising from the stomach. Most common sites of metastases are liver and mesentery. Conventional objective criteria used for response, such as response evaluation criteris in solid tumours, do not apply to gastrointestinal stromal tumours as response is not simply manifest as a reduction in size. With response, lesions typically reduce in density, and in the early course of treatment are either stable in size or increase, with a reduction in size after 3–6 months. The appearance of new hypodense liver lesions is known as pseudoprogression; these are actually responding lesions which were inconspicuous on the pretreatment CT. Limited progression is defined by the presence of new enhancing nodules within hypodense lesions, and general progression includes an increase in tumour size and density. Kochhar R, Manoharan P, Leahy M, et al. Imaging in gastrointestinal stromal tumours: Current status and future directions. Clin Radiol 2010; 65: 584–92.
1014
A 45-year-old male with a body mass index of 43 kg/m2 and diabetes mellitus underwent laparoscopic Roux-en-Y gastric bypass surgery. He presents 6 weeks later with abdominal pain and vomiting. A plain abdominal radiograph revealed dilated small bowel loops clustered in the left side of the abdomen. What is the most likely cause for this? A. Jejuno-jejunal anastomosis stricture B. Small bowel obstruction secondary to adhesions C. Gastro-jejunostomy anastomosis leak D. Gastro-gastric fistula E. Small bowel obstruction secondary to internal hernia
E. Small bowel obstruction secondary to internal hernia Small bowel obstruction following Roux-en-Y gastric bypass surgery is reported in 4–5% of cases. Adhesions and internal hernias are the most common causes: since the increased uptake of laparoscopic surgery, the incidence of adhesional obstruction has decreased but that of internal hernias has increased. Common sites include through the defect created in the transverse mesocolon when the alimentary (or Roux) limb is brought up towards the gastric pouch, this is known as a Petersen’s hernia. On plain radiography, clustering of small bowel loops in the left upper quadrant is more suggestive of internal hernia as a cause of small bowel obstruction than adhesions. Other important complications include leak from the gastro-jejunal anastomosis with subsequent abscess formation and narrowing at the gastro-jejunal anastomosis. Shah S, Shah V, Ahmed AR, et al. Imaging in bariatric surgery: Service set-up, post-operative anatomy and complications. Br J Radiol 2011; 84: 101–11.
1015
A 25-year-old female presents with intermittent colicky abdominal pain and rectal bleeding. She is also noted to have pigmentation of the lips, perioral region and buccal mucosa. In this condition, which is the commonest site for polyps? A. Stomach B. Jejunum C. Oesophagus D. Descending colon E. Duodenum
B. Jejunum The patient has Peutz Jeghers syndrome, an autosomal dominant condition with typical features of pigmentation of the lips, peri-oral region and buccal mucosa. It is usually diagnosed early in life in childhood or early adulthood and may present with intermittent colicky pain due to intussusception at the site of a polyp. Presentation may also be with rectal prolapse or bleeding. Polyps are hamartomatous and are most commonly seen in the jejunum and ileum followed, less frequently, by stomach and colon. The oesophagus is spared. Patients are at increased risk of both gastrointestinal and extra-gastrointestinal malignancies such as pancreas, breast and reproductive organs. Buck JL, Harned RK, Lichtenstein JE, et al. Peutz-Jeghers syndrome. Radiographics 1992; 12: 365–78.
1016
A 34-year-old female presents with a 3-week history of right upper quadrant pain and abdominal distension. She does not drink or smoke and is on no other medication apart from the oral contraceptive pill. On examination, she is jaundiced and has gross ascites. Abdominal ultrasound shows hepatosplenomegaly, caudate lobe hypertrophy and ascites with non-visualization of the hepatic veins. On Doppler ultrasound, there are no portal vein velocity variations with breathing. The most likely diagnosis is: A. Liver cirrhosis B. Budd-Chiari syndrome C. Portal vein thrombosis D. Hepatic artery thrombosis E. Lymphoproliferative disease
B. Budd-Chiari syndrome Budd–Chiari syndrome is caused by hepatic venous thrombosis. This leads to elevation of sinusoidal pressure which causes reversal of portal venous inflow and ascites. The caudate lobe of the liver drains directly into the inferior vena cava; thrombosis of the hepatic veins causes compensatory hypertrophy of the caudate lobe. Hepatosplenomegaly is an early sign due to hepatic congestion. Blood flow in the hepatic veins may be absent, decreased or reversed. Portal vein thrombosis occurs in 20% of cases. Causes of Budd–Chiari syndrome include hypercoagulable states, abdominal trauma, hepatocellular carcinoma and myeloproliferative disease. The cause is idiopathic in 66% of cases. Dahnert W. Live Bile Ducts, Pancreas and Spleen. In: Radiology Review Manual. 6th edn. Philadelphia, PA: Lippincott Williams and Wilkins; 2007. p. 694.
1017
A 49-year-old diabetic male with a 2-day history of substernal pain and fever was referred for an urgent abdominal ultrasound. The ultrasound showed a diffusely echogenic gallbladder wall with multiple tiny echogenic reflectors within the gallbladder lumen. What is the most likely diagnosis? A. Acute cholecystitis B. Porcelain gallbladder C. Emphysematous cholecystitis D. Focal biliary lipomatosis E. Bowel gas overlying the gallbladder
C. Emphysematous cholecystitis Emphysematous cholecystitis is a rare form of acute cholecystitis affecting men twice as commonly as women. The majority of patients have underlying diabetes mellitus and peripheral atherosclerotic disease. It is associated with an increased prevalence of acalculous disease and gallbladder perforation. There are three radiographic stages of emphysematous cholecystitis, with stage 1 characterized by gas within the gallbladder lumen; stage 2, by gas within the gallbladder wall; and stage 3, by gas within the pericholecystic tissues. Ultrasonography demonstrates highly echogenic reflectors with low-level posterior shadowing and reverberation artifact emanating from the gallbladder wall or lumen. A more specific, though less common, finding consists of small, non-shadowing echogenic foci rising up from the dependent portions of the gallbladder lumen, which is called the “champagne sign”. Grayson DE, Abbott RM, Levy AD, et al. Emphysematous infections of the abdomen and pelvis: A pictorial review. Radiographics 2002; 22: 543–61.
1018
A 50-year-old female patient presented with acute abdominal pain. On CT, an incidental cystic lesion was seen in the pancreas. What is the modality of choice for sampling in patients with cystic pancreatic lesions and no definitive diagnosis on cross sectional imaging? A. Percutaneous ultrasound-guided aspiration B. Percutaneous CT-guided aspiration C. Endoscopic ultrasound-guided sampling D. Endoscopic retrograde cholangiopancreatography E. MR-guided sampling
C. Endoscopic ultrasound-guided sampling Endoscopic ultrasound is currently recommended for cyst aspiration and fine needle aspiration biopsy of cystic pancreatic lesions. This technique reduces the potential for tumour seeding along the needle tract. Cytology and biochemical and tumour markers can differentiate mucinous from non-mucinous lesions, and prevent unnecessary surgical resection of benign lesions. Sahani DV, Kadavigere R, Saokar A. Cystic pancreatic lesions: a simple imaging-based classification system for guiding management. Radiographics 2005; 25: 1471–84.
1019
A 30-year-old female with a history of recurrent epigastric pain presented to the accident and emergency department. The patient had no history of fever or jaundice and liver function tests were normal. An ultrasound of the abdomen revealed saccular dilatation of the intra-hepatic bile ducts with bridge formation across dilated bile ducts and portal radicles surrounded by dilated bile ducts. There was communication between the cystic ductal dilatation and the bile duct system. Several cysts were also noted in both kidneys. The most likely diagnosis is: A. Primary sclerosing cholangitis B. Autosomal dominant polycystic kidney disease C. Recurrent cholangitis D. Caroli’s disease E. Cholangiocarcinoma
D. Caroli’s disease Caroli’s disease is an autosomal recessive condition characterized by congenital saccular dilatation of intrahepatic bile ducts, high frequency of stone formation and cholangitis and an association with renal tubular ectasia or other forms of cystic disease of the kidneys. Typical ultrasound features are described. Primary sclerosing cholangitis and recurrent pyogenic cholangitis can be associated with duct dilatation, stenosis, intrahepatic calculi, and malignancy. Duct dilatation in primary sclerosing cholangitis is usually isolated and fusiform. Patients with pyogenic cholangitis have intra- and extrahepatic biliary dilatation. Patients with Caroli’s disease usually have saccular dilatation. Patients with polycystic liver disease may have hepatic and renal cysts. However, the hepatic cysts rarely communicate with the bile ducts, which are usually intrinsically normal. The imaging features are not consistent with cholangiocarcinoma. Levy AD, Rohrmann CA Jr, Murakata LA, et al. Caroli’s disease: Radiologic spectrum with pathologic correlation. AJR Am J Roentgenol 2002; 179: 1053–7.
1020
A 25-year-old Asian male with a 3-month history of weight loss and abdominal discomfort underwent a contrast-enhanced CT examination of the abdomen and pelvis. This revealed multiple enlarged and peripherally enhancing but centrally hypodense portal and peripancreatic lymph nodes, ascites with a density of 30 HU, and concentric thickening of the terminal ileum and caecum. What is the most likely cause for these abnormalities? A. Whipple’s disease B. Tuberculosis C. Lymphoma D. Crohn’s disease E. Ulcerative colitis
B. Tuberculosis Lymphadenopathy is the most common manifestation of abdominal tuberculosis, with a characteristic pattern of mesenteric, portal and peripancreatic nodal enlargement, compared with lymphoma in which para-aortic nodal enlargement is more typical. Peripheral and homogeneous enhancement patterns are recognized features. Ileocaecal concentric thickening with an incompetent ileocaecal valve is typical of gut tuberculosis. When advanced, caecal retraction occurs due to mesenteric fibrosis, and multiple small bowel strictures give rise to a “napkin ring stenosis” appearance. Omental and mesenteric thickening, as well as ascites with a raised density (due to high protein and cellular content) are also found. Burrill J, Williams CJ, Bain G, et al. Tuberculosis: A radiologic review. Radiographics 2007; 27: 1255–73.
1021
A 28-year-old female underwent supra-diaphragmatic radiotherapy for Hodgkin’s lymphoma at the age of 15 years. She is concerned about her risk of developing breast cancer. She has no other past medical history of note. What would be the recommended method of surveillance? A. Annual MRI B. Annual mammography C. 3-yearly mammography D. No imaging E. Annual ultrasound
A. Annual MRI Women treated with supradiaphragmatic radiotherapy for Hodgkin’s disease under the age of 35 years are at increased risk of subsequent breast cancer. No screening method is of proven efficacy in these women. Women should be counselled to allow an informed decision about surveillance. For those who opt for surveillance, this should be offered at 8 years post radiotherapy or at age 30, whichever is the later. A recommended schedule to be performed by the NHS Breast Screening Programme has been developed: * <30 years: No imaging. * 30–39 years: Annual MRI but, if contraindicated, annual ultrasound (mammography is not recommended for this age group). * 40–49 years: Women should be divided according to breast density. Those with predominantly adipose breast tissue should have annual mammograms. Those with dense breast tissue should have annual mammograms plus MRI. If MRI is contraindicated, these women should have annual mammograms plus ultrasound. In some women, the breast tissue may become predominantly adipose before the age of 50 in which case they should move into the annual mammography group. * >50 years: Mammography + MRI according to breast density. Ralleigh G, Given-Wilson R. Breast cancer risk and possible screening strategies for young women following supradiaphragmatic irradiation for Hodgkin’s disease. Clin Radiol 2004; 59: 647–50. Chapman K, ed. NHSBSP Publication No: 74. Protocols for the surveillance of women at higher risk of developing breast cancer. London, UK: Public Health England; 2013.
1022
A 40-year-old female presented to breast clinic with a palpable mass at 2 o’clock in the right breast that had been noted to fluctuate in size. Mammograms demonstrate an oval mass with circumscribed and partially obscured margins in the right breast. Ultrasound demonstrates an anechoic mass with imperceptible walls and posterior acoustic enhancement. Which of the following is the most likely diagnosis? A. Simple cyst B. Fibroadenoma C. Radial scar D. Paget’s disease E. Ductal carcinoma in situ
A. Simple cyst Simple cysts are the most common breast mass in women. They have no malignant potential and may fluctuate in size due to the menstrual cycle. Cysts cannot be distinguished from circumscribed solid masses on mammography. Shah B, Fundaro G, Mandava S, eds. Breast imaging review—a quick guide to essential diagnoses. New York: Springer; 2010.
1023
A 19-year-old female presents to the breast symptomatic clinic with a firm well-defined mobile mass in the periphery of the breast. Ultrasound examination showed an ill-defined mass with multiple small cysts especially at the periphery. What is the most likely diagnosis? A. Fibroadenoma B. Juvenile papillomatosis C. Intraductal papilloma D. Phyllodes tumour E. Breast carcinoma
B. Juvenile papillomatosis Juvenile papillomatosis is a localized, proliferative disorder of young women and older adolescents. Typical clinical and ultrasound characteristics are described above. Although a benign condition, it is considered a marker for familial breast cancer with 33–58% of patients having a positive family history of breast cancer. Treatment is by surgical excision. Fibroadenoma is the most common breast mass in girls less than 20 years old, and more common in Afro–Caribbean women. The typical sonographic appearance is a well-circumscribed, round, oval or macrolobulated mass with fairly uniform hypoechogenicity. They are often indistinguishable from at phyllodes tumour at imaging. An intraductal papilloma is most commonly seen as a mass lying within an enlarged duct. Breast carcinoma is extremely rare in children or adolescents accounting for less than 1% of breast lesions. Chung EM, Cube R, Hall GJ, et al. Breast masses in children and adolescents: radiological–pathologic correlation. Radiographics 2009; 29: 907–31.
1024
A 27-year-old female presents to the general practitioner concerned about herself as her elder sister, aged 30 years has recently been diagnosed with breast cancer in both breasts. What would you recommend? A. Reassure and recommend breast self-examination yearly B. Recommend mammogram C. Recommend bilateral ultrasound D. Recommend MRI E. Recommend genetic testing for both
E. Recommend genetic testing for both Approximately 5–10% of breast cancer cases are hereditary, with the most common cause being due to an inherited mutation in the BRCA1 and BRCA2 genes. The risk of breast cancer may be as high as 80% for members of some families with BRCA mutations. These cancers tend to occur in younger women and are more often bilateral than cancers in women who do not have one of these gene mutations. Referral to a specialist genetic centre in order to offer genetic testing would be recommended in this case. If the BRCA1 or BRCA2 gene mutation was found, strategies would include annual surveillance, risk-reducing surgery and chemoprophylaxis would be considered as she is at high risk of developing breast cancer. Women with the BRCA mutation are also high risk of developing ovarian cancer. NICE clinical guidelines CG164. Familial breast cancer. 2013. Available from: http://www.nice.org.uk/CG164
1025
A patient with a suspicious mass in the left breast has an axillary ultrasound. Which of the following ultrasound features is not consistent with a pathological axillary lymph node? A. Thickened cortex B. Absent hilum C. Hyperaemic blood flow in the hilum on Doppler colour ultrasound D. Lymph node >5 mm E. Eccentrically bulging cortex
D. Lymph node >5 mm Ultrasound features of an abnormal axillary lymph node include: * thickened of eccentrically bulging cortex * diminished or absent hilum * hyperaemic blood flow in the hilum and central cortex on colour Doppler ultrasound * abnormal (nonhilar cortical) blood flow. The size of the lymph node is not an indicator of benignity or malignancy, since normal lymph nodes larger than 5 cm can be present in the axilla and lymph nodes as small as 5 mm can contain metastases. Abe H, Schmidt RA, Sennett CA, et al. US-guided core needle biopsy of axillary lymph nodes in patients with breast cancer: why and how to do it. Radiographics 2007; 27: S91–9.
1026
A 63-year-old female underwent screening mammography. Branching linear and pleomorphic calcification was present in the upper outer quadrant of the left breast. On contrast-enhanced MRI examination, a plateau enhancement pattern was observed in a segmental, non-mass-like distribution in this region. What is the most likely cause for these abnormalities? A. Galactocoele B. Fat necrosis C. Fibroadenoma D. Ductal carcinoma in situ E. Fibrocystic change
D. Ductal carcinoma in situ Ductal carcinoma in situ accounts for 20–25% of breast cancers detected at screening mammography. The most common mammographic appearance is the presence of microcalcifications of the pleomorphic or linear branching type. Low-grade lesions may present as non-calcified mass-like abnormalities. The most common enhancement pattern on MRI is non-mass-like, which may be in a segmental distribution. The most common MRI appearance of fat necrosis is a lipid cyst. A round or oval mass with hypointense T1 weighted signal on fat-saturated images is typical with variable rim enhancement. An oil cyst or coarse calcifications are seen on mammography. Galactocoeles usually occur during pregnancy or lactation. Fibroadenomas are typically well defined and show mass-like enhancement on MRI and may also contain non-enhancing septations. In fibrocystic change, the pattern of calcification on mammography is benign and sedimented and may demonstrate “tea-cupping”. Yamada T, Mori N, Watanabe M, et al. Radiologic–pathologic correlation of ductal carcinoma in situ. Radiographics 2010; 30: 1183–98.
1027
A 45-year-old patient from a low-risk group presents to breast clinic with a palpable lump in the right breast. On ultrasound, a solitary 10 mm ill-defined hypoechoic mass (U5) is seen in the right upper outer quadrant which is core-biopsied. In the axilla, a node with cortical thickness of 4 mm is seen at level 1. Fine needle aspiration of the node was performed. An invasive ductal carcinoma was diagnosed with evidence of metastasis to the node. Which of the following is most appropriate management of this patient? A. Core biopsy of the axillary node B. Wide local excision and sentinel node biopsy C. Wide local excision and axillary node clearance D. Mastectomy E. Repeat axillary ultrasound and fine needle aspiration of a different node
C. Wide local excision and axillary node clearance Axillary lymph node status is an extremely important prognostic factor in the assessment of new breast cancer patients. If the image-guided sampled nodes are normal, sentinel lymph node biopsy is performed. Sentinel lymph node biopsy can be avoided if lymph node metastasis is diagnosed pre-surgically using image-guided biopsy of the node and patients undergo axillary node clearance. Ultrasound-guided needle biopsy (fine needle aspiration or core biopsy) is now a standard of care for pre-operative diagnosis of the axilla and a well-established procedure. However, there are anatomic challenges of performing axillary core biopsy. If metastasis is proven on cytology, a repeat core biopsy is unnecessary. Ultrasound findings in abnormal lymph nodes include cortical thickening and diminished or absent hilum. With a solitary 10 mm focus of disease in a single quadrant in a low risk patient, wide-local excision would be appropriate treatment. Abe H, Schmidt RA, Sennett CA, et al. US-guided core needle biopsy of axillary lymph nodes in patients with breast cancer: why and how to do it. Radiographics 2007; 27: S91–9.
1028
A 75-year-old male undergoes a triple-phase CT scan of the kidneys following an ultrasound scan which showed a renal lesion. Two cysts are identified in the left kidney with thin septa, nodular areas of calcification and solid non-enhancing areas. Which of the following would be the most appropriate Bosniak classification? A. I B. II C. IIf D. III E. IV
C. IIf The following is the Bosniak classification of renal cysts with CT features: * Type 1 – Benign simple cysts with hairline thin walls. These do not require follow-up. * Type II – Benign cystic lesions that may contain hairline-thin septa with perceived enhancement (not measurable). They may contain fine calcification in the walls or septa or a short segment of thickened calcification. * Type IIF – Cysts which may contain multiple hairline septae. There may be minimal thickening of wall or septa and these may contain thick, nodular calcification. There are no enhancing solid components. These are thought to be benign but require follow-up to prove that they have stable appearances. * Type III – Cystic masses with thickened, irregular or smooth walls or septa and in which measurable enhancement is present. Included in this category are complicated haemorrhagic or infected cysts, multilocular cystic nephroma, and cystic neoplasms. These masses need surgical intervention. * Type IV – Definitely malignant cystic masses which contain the features of III plus enhancing solid components which are independent of the wall or septae. Israel GM, Bosniak MA. How I do it: evaluating renal masses. Radiology 2005; 236: 441–50.
1029
A 76-year-old male with bladder cancer presents with anuria. The creatinine is raised at 400 and ultrasound shows bilateral hydronephrosis. The patient is referred for insertion of nephrostomies. Which of the following is the optimal site of entry to minimize risk of bleeding? A. Junction of anterior two-thirds and posterior one-third of the renal parenchyma B. Junction of anterior one-third and posterior two-thirds of the renal parenchyma C. No specific site D. At the inferior aspect of the kidney E. At the superior aspect of the kidney
A. Junction of anterior two-thirds and posterior one-third of the renal parenchyma During percutaneous nephrostomy, bleeding complications can be minimized by traversing a relatively avascular region which lies just posterior to the lateral convex border of the kidney. This demarcates the division of the major dorsal and ventral branches of the renal artery. The optimal site of entry lies postero-laterally at the junction of the anterior two-thirds and posterior one-third of the renal parenchyma. Dyer RB, Regan JD, Kavanagh PV, et al. Percutaneous nephrostomy with extensions of the technique: step by step. Radiographics 2002; 22: 503–25.
1030
A 46-year-old female with a history of polycystic kidney disease undergoes a right renal transplant. She presents after 6 weeks with a swollen right leg. An ultrasound scan is performed to rule out deep vein thrombosis. While assessing the veins, a rounded fluid collection is seen compressing the right external iliac vein. The abdomen is scanned and this shows hydronephrosis on the right. CT scan shows a mid-ureteric fluid collection with attenuation similar to water. A radionuclide scan is performed to clarify the diagnosis and shows a photopenic area compressing the transplanted kidney. The most likely diagnosis is: A. Urinoma B. Haematoma C. Abscess D. Lymphocoele E. Acute rejection
D. Lymphocoele Lymphocoeles occur in 15% of transplant patients and are usually an incidental finding. They occur owing to leakage of lymph from disrupted lymphatics along the iliac vessels or of the transplanted kidney. They are the most common fluid collection to cause hydronephrosis of the transplanted kidney. Attenuation on CT is usually less than abscess or haematoma. Urinomas and haematomas usually present soon after transplant whereas lymphocoeles develop between 4 and 8 weeks. For urinomas, radionuclide studies would show uptake of radio-isotope in an area that was previously cold. Akbar SA, Jafri SZ, Amendola MA, et al. Complications of renal transplantation. Radiographics 2005; 25: 1335–56.
1031
An asymptomatic 23-year-old male presented with deteriorating renal function. A renal ultrasound was performed, demonstrating markedly increased echogenicity in the renal pyramids within upper pole of the right kidney. The renal cortex was normal in echogenicity. The left kidney was unremarkable. No renal calculi were seen within either kidney. What is the most likely cause of these findings? A. Renal tubular acidosis B. Renal tuberculosis C. Medullary sponge kidney D. Renal papillary necrosis E. Hyperparathyroidism
C. Medullary sponge kidney Medullary sponge kidney is defined as dysplastic cystic dilatation of papillary and medullary portions of the collecting ducts. It affects young to middle-aged adults and is often asymptomatic. On ultrasound, the medulla are echogenic in the absence of stones and it may be unilateral in 25% of patients. The three leading causes for medullary nephrocalcinosis include hyperparathyroidism (40%), medullary sponge kidney (20%) and distal type 1 renal tubular acidosis (20%). If the calcification is noticeably segmental medullary sponge kidney is the top consideration as the other two main causes typically produce diffuse and bilateral renal involvement. Dahnert W. Urogenital tract. In: Radiology Review Manual. 6th edn. London, UK: Lippincott Williams and Wilkins; 2007.
1032
An elderly male is being investigated for painless haematuria. On ultrasound, a hypoechoic mass is seen within the bladder which appears as an enhancing mass on contrast-enhanced CT and a urothelial tumour is suspected . What would be the most likely signal characteristics on MRI? A. Hypointense T1 and hyperintense T2 B. Hypointense T1 and hypointense T2 C. Hyperintense T1 and hyperintense T2 D. Intermediate T1 and intermediate T2 E. Intermediate T1 and hyperintense T2
D. Intermediate T1 and intermediate T2 The layers of the bladder wall can be clearly distinguished on MRI allowing accurate staging of urothelial bladder tumours. On T1 weighted images bladder wall and tumour are of intermediate signal intensity and urine is dark. Extra-vesical infiltration, nodes and bone metastases are detected on T1 weighted sequences as fat has a high signal. On T2 weighted imaging, tumour is intermediate, urine has high signal and muscle has low signal. These sequences are used for evaluation of tumour depth, differentiation of tumour from fibrosis and for detecting invasion of surrounding organs and bone marrow metastases. Metastatic lymph nodes enhance early and simultaneously with the primary bladder tumour. Wong-You-Cheong JJ, Woodward PJ, Manning MA, et al. Neoplasms of the urinary bladder: radiologic–pathologic correlation. Radiographics 2006; 26: 553–80.
1033
A 70-year-old male presents with bilateral painless testicular enlargement with a history of weight loss and fever. Ultrasound reveals discrete hypoechoic lesions in both testes. What is the most likely diagnosis? A. Seminomatous germ cell testicular tumours B. Choriocarcinoma C. Testicular teratomas D. Chronic tubercular orchitis E. Lymphoma
E. Lymphoma Testicular lymphoma is the most common testicular neoplasm in men above the age of 60. The most common presentation is painless testicular enlargement. However, 25% of patients may present with systemic symptoms such as weight loss, fever, anorexia and weakness. Lymphoma accounts for 5% of all testicular tumours but testicular lymphoma occurs in less than 1% of patients with lymphoma. It is also the commonest bilateral testicular tumour, with bilaterality occurring in 38% of cases. Woodward PJ, Sohaey R, O’Donoghue MJ, et al. Radiographics 2002; 22: 189–216.
1034
A 14-month-old boy presents with an enlarged left hemiscrotum. Ultrasound demonstrates an enlarged heterogeneous left testis with no discrete focal lesion seen. On further investigations, serum alpha fetoprotein levels are elevated. What is the most likely diagnosis? A. Teratoma B. Seminoma C. Choriocarcinoma D. Yolk sac tumour E. Embryonal carcinoma
D. Yolk sac tumour Yolk sac tumours usually present before the age of 2 years and account for 80% of childhood testicular tumours. Imaging in children is non-specific and the only finding may be testicular enlargement without a mass. Greater than 90% of patients have elevated levels of serum alpha fetoprotein. Teratoma is the second commonest childhood testicular tumour. Imaging usually shows well-circumscribed complex masses. Cysts are a common feature. Cartilage, calcification, fibrosis and scar formation are also seen. Woodward PJ, Sohaey R, O’Donoghue MJ, et al. Tumors and tumorlike lesions of the testis: radiologic-pathologic correlation. Radiographics 2002; 22: 189–216.
1035
A 65-year-old male is diagnosed with renal pelvis transitional cell carcinoma on CT. Which of the following investigations is most appropriate for complete assessment before treatment? A. Retrograde pyelography B. Ultrasound C. MR D. Cystoscopy E. Excretory urography
D. Cystoscopy Patients with newly diagnosed upper tract transitional cell carcinoma should have a cystoscopy and patients with newly diagnosed bladder transitional cell carcinoma (TCC) should have CT urography to evaluate the upper tracts. This is because synchronous bilateral TCC has been reported in 1–2% of cases of renal TCC and 2–9% of ureteric TCC. Browne RF, Meehan CP, Colville J, et al. Transitional cell carcinoma of the upper urinary tract: spectrum of imaging findings. Radiographics 2005; 25: 1609–27.
1036
A 66-year-old presenting with flank pain and haematuria is being investigated with CT urography. On the precontrast images, a hyperattenuating mass (20–30 HU) is seen in the left renal pelvis. On the post-contrast images, the lesion shows enhancement less than the renal parenchyma and is seen as a filling defect in the renal pelvis on the excretory phase with normal peripelvic fat. What is most likely abnormality? A. Transitional cell malignancy of the renal pelvis B. Clot within the renal pelvis C. Calculus within the renal pelvis D. Papillary necrosis E. Inflammatory mass
A. Transitional cell malignancy of the renal pelvis On unenhanced scans, transitional cell carcinoma is has higher attenuation (5–30 HU) than urine and renal parenchyma and lower attenuation than clots (40–80 HU) and calculi (>100 HU). The typical appearance of renal transitional cell carcinoma is a sessile filling defect in the excretory phase which compresses the renal sinus fat. Pelvicalyceal irregularity, mural thickening and focally obstructed calyces are also features. Early tumours which are confined to the muscularis have normal peri-pelvic fat as they are separated from the renal parenchyma by renal sinus fat or excreted contrast material. In advanced tumours, there is infiltrative extension into the renal parenchyma which distorts the normal architecture but preserves the reniform shape unlike renal cell carcinoma. Browne RF, Meehan CP, Colville J, et al. Transitional cell carcinoma of the upper urinary tract: spectrum of imaging findings. Radiographics 2005; 25: 1609–27.
1037
A 75-year-old male with renal colic underwent a non-contrast CT of the abdomen and pelvis (“CT KUB”). A 2.8 cm right adrenal lesion was noted. To characterize this further, an “adrenal protocol” CT was performed, consisting of intravenous contrast enhancement and imaging at 60 seconds and 15 minutes. The Hounsfield unit measurements obtained from a region of interest placed in the centre of the lesion were pre = 20; 60 seconds = 50; and 15 minutes = 25. What is the most likely cause of the abnormality? A. Lipid-rich adenoma B. Lipid-poor adenoma C. Metastasis D. Adrenocortical carcinoma E. Phaeochromocytoma
B. Lipid-poor adenoma Adrenal adenomas are found in 7% of patients aged over 70 years. Most adenomas contain fat so a mean density measurement of <10 HU on a pre-contrast study has sufficient accuracy such that no further imaging is required. 10–40% of adenomas are lipid poor, and will therefore have a mean Hounsfield unit >10 on the pre-contrast study. In these cases, absolute and relative percentage washout (APW and RPW, respectively) values are calculated from the mean density measurements on portal venous phase and delayed phase images. APW >60% and RPW >40% have high sensitivity and specificity for lipid-poor adrenal adenomas when compared with non-adenomatous lesions, as adenomas typically display rapid washout. Johnson PT, Horton KM, Fishman EK. Adrenal mass imaging with multidetector CT: pathologic conditions, pearls, and pitfalls. Radiographics 2009; 29: 1333–51.
1038
A 65-year-old female under investigation for anaemia underwent a contrast-enhanced CT of the abdomen and pelvis revealing a 9 cm well-defined lesion of the right adrenal gland, with enhancing nodules of soft tissue as well as macroscopic lobules of fat. Small punctuate calcific foci were also noted. The left adrenal gland appeared normal, and there were no other abnormalities present. What is the most likely cause for the findings? A. Adrenal adenoma B. Adrenocortical carcinoma C. Adrenal lymphoma D. Adrenal metastasis E. Adrenal myelolipoma
E. Adrenal myelolipoma Adrenal myelolipomas are benign and usually incidental findings containing variable quantities of fat and soft tissue, and partly consisting of haematopoietic tissue. When isolated, they are on average 10 cm in size and composed of 50–90% fat. Calcification occurs in approximately 25% and 75% have a pseudocapsule. When they occur in conjunction with other adrenal lesions, they are smaller (mean 7 cm) with fat in <10%, calcification in approximately 50% and a pseudocapsule in 33%. Although usually asymptomatic, they may cause symptoms due to mass effect, necrosis or haemorrhage. Johnson PT, Horton KM, Fishman EK. Adrenal mass imaging with multidetector CT: pathologic conditions, pearls, and pitfalls. Radiographics 2009; 29: 1333–51.
1039
A 32-year-old male with refractory hypertension, reduced facial hair growth and gynaecomastia underwent intravenous contrast-enhanced CT of the abdomen and pelvis. This revealed a heterogeneously enhancing 10 cm mass arising from the right adrenal gland with small foci of calcification within and several irregular hypodense liver lesions. The most likely diagnosis is: A. Adrenal adenoma B. Adrenocortical carcinoma C. Adrenal lymphoma D. Adrenal metastasis E. Adrenal myelolipoma
B. Adrenocortical carcinoma Adrenocortical carcinoma has highest incidences in the first and fourth decades. Presentation may be due to hormonal activity (Cushing syndrome, feminization, virilization or a combination of these) or mass effect and pain due to the large size of lesions. Approximately 50% of lesions are functional. They have a heterogeneous appearance due to internal necrosis, with a thin enhancing capsule seen in some cases. Foci of calcification are found in 25% of cases. The liver, lungs and lymph nodes are the most common sites for metastases, and direct invasion into the inferior vena cava is a well known complication. Johnson PT, Horton KM, Fishman EK. Adrenal mass imaging with multidetector CT: pathologic conditions, pearls, and pitfalls. Radiographics 2009; 29: 1333–51.
1040
A 25-year-old male with right loin pain underwent intravenous urography revealing right hydronephrosis and proximal hydroureterosis. Beyond the dilated segment, the ureter turned sharply medially to overlie the pedicles of L3 and L4 before returning to a normal position. The left kidney and ureter had a normal appearance. No renal tract calcification was observed. What is the most likely cause of this appearance? A. Aortic aneurysm B. Pelvic lipomatosis C. Retroperitoneal fibrosis D. Retrocaval ureter E. Retroperitoneal haematoma
D. Retrocaval ureter Retrocaval ureter is due to an embryological defect of the inferior vena where the infrarenal inferior vena cava (IVC) develops from the right posterior cardinal vein, which lies anterior and lateral to the ureter, rather than the normal situation in which it develops from the right supracardinal vein which is posterior and medial to the ureter. The ureter becomes trapped posterior to the IVC, causing a transition in calibre and a sharp turn in its course medially at the level of L3–L4, from where it passes anteriorly between the IVC and aorta and returns to a normal course. Treatment consists of surgical relocation of the ureter. Aortic aneurysm and retroperitoneal haematoma more typically cause lateral deviation of the ureter, and retroperitoneal fibrosis is usually bilateral. Kandpal H, Sharma R, Gamangatti S, et al. Imaging the inferior vena cava: a road less traveled. Radiographics 2008; 28: 669–89.
1041
A 4-year-old boy with deranged renal function attends the radiology department for an ultrasound examination of his abdomen. The kidneys were noted to be enlarged with a hyperechoic parenchyma. The spleen was also enlarged. What is the most likely diagnosis? A. Medullary cystic kidney disease B. von Hippel–Lindau syndrome C. Autosomal dominant polycystic kidney disease D. Bilateral Wilm’s tumours E. Autosomal recessive polycystic kidney disease
E. Autosomal recessive polycystic kidney disease Autosomal recessive polycystic kidney disease is seen in 1/6000 to 1/14,000 births. The severe variety presents at birth with uraemia, Potter’s facies and pulmonary hypoplasia and is generally fatal. A milder variant presents in early childhood (3–5 years of age) with enlarged poorly functional kidneys and enlarged liver and spleen. The most common radiological finding is enlarged kidneys with hyperechoic parenchyma, with the cysts often being too small to be individually distinguished. Other findings include liver cysts, hepatic fibrosis, splenomegaly and portal hypertension. In medullary cystic kidney disease the kidneys are small or normal sized, which is an important distinguishing factor from Autosomal Recessive Polycystic Kidney Disease and Autosomal Dominant Polycystic Kidney Disease. Barbaric ZL. Renal cysts. In: Principles of genitourinary radiology. 2nd edn. Stuttgart, Germany: Thieme; 1994. pp. 185–200.
1042
A young male with a known autosomal dominant condition undergoes a follow-up contrast-enhanced CT examination of the neck, chest and abdomen, which shows an enhancing well-defined mass within the right parathyroid, a heterogeneous right-sided adrenal mass, and a cystic mass with an enhancing wall in the head of the pancreas. A recent CT head examination had been performed as the patient had been complaining of blurred vision. What is the most likely clinical syndrome associated with these symptoms and CT findings? A. Multiple endocrine neoplasia Type 1 B. Multiple endocrine neoplasia Type 2a C. Multiple endocrine neoplasia Type 2b D. von Hippel–Lindau syndrome E. Tuberous sclerosis
A. Multiple endocrine neoplasia Type 1 Multiple endocrine neoplasia (MEN) 1 is autosomal dominant. It is associated with parathyroid, pancreatic and pituitary tumours. Other features may include adrenal cortical tumours, carcinoid tumours, lipomatous tumours and collagenomas. 85–95% of MEN 1 patients have multiple facial angiofibromas. MEN 2 is autosomal dominant and associated with medullary thyroid carcinoma, benign or malignant phaeochromocytomas and parathyroid tumours or hyperplasia. MEN 2B is rare. Patients have a marfanoid appearance and develop mucosal neuromas. These are pathognomic. Scarsbrook AF, Thakker RV, Wass JA, et al. Multiple endocrine neoplasia: spectrum of radiologic appearances and discussion of multitechnique imaging approach. Radiographics 2006; 26: 433–51.1
1043
A 55-year-old female with a past medical history of diabetes mellitus presents to the emergency department with a 2-week history of worsening right flank pain and fever. Blood tests showed leucocytosis and anaemia. Xanthogranulomatous pyelonephritis was suspected and an urgent renal ultrasound was performed. What is the most likely finding on ultrasound? A. Staghorn calculus B. Psoas abscess C. Hydronephrosis D. Cortical renal atrophy E. Intrarenal collection
C. Hydronephrosis Xanthogranulomatous pyelonephritis is a chronic pyelonephritis causing destruction of the renal parenchyma which is replaced by lipid-rich, foamy macrophages. Middle-aged women are most commonly affected with predisposing factors including diabetes, recurrent urinary tract infections and renal calculi. It is most often a diffuse process that can extend beyond the kidney but can also be focal, with a differential diagnosis of primary renal tumour. Ultrasound and CT are the main radiological imaging modalities. 90% of patients have hydronephrosis on imaging. Other findings include renal calculi (72.7%), intraparenchymatous collection (45.5%), renal cortical atrophy (45.5%) and PSOAs abscess (36.4%). Loffroy R, Guiu B, Watfa J, et al. Xanthogranulomatous pyelonephritis in adults: clinical and radiological findings in diffuse and focal forms. Clin Radiol 2007; 62: 884–90.
1044
Following ultrasound within 24 hours of birth, a male term baby is diagnosed with megaureter. A micturating cystourethrogram does not demonstrate posterior urethral valves or reflux. What is the most appropriate next step in the patient’s management? A. Immediate surgical intervention B. 99mTc diethylene triamine penta-acetic acid to assess perfusion and diuresis renography C. No further investigations D. Repeat ultrasound in 3 months to monitor ureteric dilatation E. Intravenous urogram
B. 99mTc diethylene triamine penta-acetic acid to assess perfusion and diuresis renography 49% of cases of primary megaureter can be treated conservatively in the first instance once functional obstruction has been excluded by nuclear imaging. Functional obstruction requires surgical intervention. An intravenous urogram can also demonstrate functional obstruction but has a higher radiation dose to the patient than that of nuclear imaging. Follow-up imaging of non-obstructing primary megaureter is most commonly by nuclear imaging rather than ultrasound. Berrocal T, López-Pereira P, Arjonilla A, et al. Anomalies of the distal ureter, bladder, and urethra in children: embryologic, radiologic, and pathologic features. Radiographics 2002; 22: 1139–64.
1045
When performing a percutaneous nephrostomy with the potential need for antegrade ureteric stenting, which puncture site and approach is preferred? A. Upper pole, anterior calcyx with supracostal approach B. Lower pole, anterior calyx with subcostal approach C. Lower pole, posterior calyx with subcostal approach D. Upper pole, posterior calyx with subcostal approach E. Lower pole, posterior calyx with supracostal approach
D. Upper pole, posterior calyx with subcostal approach For simple percutaneous nephrostomy, the preferred approach is subcostal into a posterior lower pole calyx. For patients requiring ureteric intervention, a subcostal approach into an upper or middle pole posterior calyx is recommended to allow easier access to the ureteropelvic junction. The supracostal approach carries a greater risk of complications but allows greater visualization of the collecting system when removing complex stones. Dyer RB, Regan JD, Kavanagh PV, et al. Percutaneous nephrostomy with extensions of the technique: step by step. Radiographics 2002; 22: 503–25.
1046
A 38-year-old male patient is referred for investigation of poorly controlled hypertension. He is subsequently diagnosed with fibromuscular dysplasia of the renal artery. Imaging findings consistent with this diagnosis include all but one of the following: A. Stenosis at the ostium of the affected renal artery B. Angiotensin converting enzyme inhibitor scintigraphy shows diminished tracer uptake by the affected kidney C. Peak systolic velocity >150 cm per second on 60° angle Doppler ultrasound D. “String of beads” sign on angiogram E. Prolongation of cortical nephrographic phase on contrast-enhanced CT
A. Stenosis at the ostium of the affected renal artery Answers B, C and E are features of renal artery stenosis of any cause. Renal artery stenosis is the cause of hypertension in 1–5% of hypertensive patients. It is caused by atherosclerosis (60–90%), fibromuscular dysplasia (10–30%) and other causes including arterial dissection, arteriovenous malformation and vasculitis (<10%). Arteriosclerotic renal artery stenosis most commonly affects patients over 50 years old and involves the proximal 2 cm of the main renal artery in 93% of patients. Fibromuscular dysplasia is the most common cause of renovascular hypertension in children and adults under the age of 40 years old, involving the mid to distal main renal artery with sparing of the proximal third of the main renal artery in 98%. The “string of beads” sign is seen in medial fibroplasia with microaneurysm. Dahnert W. Radiology Review Manual. 6th edn. London, UK: Lippincott Williams and Wilkins; 2007. pp. 956–58. Soulez G, Olivia VL, Turpin S, et al. Imaging of renovascular hypertension: respective values of renal scintigraphy, renal Doppler US, and MR angiography. Radiographics 2000; 20: 1355–68.
1047
A 33-year-old male presents with episodes of flushing, palpitations and anxiety. He gives a history of episodic headaches. He also mentions that his general practitioner had some concerns recently about his blood pressure being raised. A contrast-enhanced CT scan of the abdomen shows an avidly enhancing, heterogeneous adrenal mass with flecks of calcification within it. What is the most likely diagnosis? A. Phaeochromocytoma B. Neuroblastoma C. Adrenal metastasis D. Myelolipoma E. Granulomatous disease
A. Phaeochromocytoma The clinical and imaging findings are most representative of an adrenal phaeochromocytoma which normally presents in the third/fourth decade with symptoms secondary to excess catecholamines. They are usually sporadic but can be associated with MEN 2, neurofibromatosis type 1, von Hippel–Lindau syndrome and tuberous sclerosis, familial phaeochromocytoma and Carney syndrome. Findings on CT are of an enhancing mass which, when large, can be complex with areas of haemorrhage, necrosis and non-specific calcification. Neuroblastoma is a childhood tumour usually arising from the adrenal medulla. On CT, it is heterogeneous with 85% containing calcification. Adrenal metastases are usually from lung, breast and melanoma and may be calcified. Myelolipomas are non-functioning, benign adrenal tumours which contain haemopoetic tissue, fat and may be calcified. Diagnosis is made by demonstrating macroscopic fat on CT or MRI. Granulomatous disease from tuberculosis or histoplasmosis can cause adrenal calcification. Hindman N, Israel GM. Adrenal gland mass and calcification. Eur Radiol 2005; 15: 1163–7.
1048
A 55-year-old male with a known history of poorly controlled diabetes mellitus presents to the accident and emergency department with general lethargy and right flank pain. Biochemical tests revealed raised urea and creatinine levels and hyperglycaemia. Emphysematous pyelonephritis is clinically suspected. Which of the following does not relate to this condition? A. Urinary collecting system obstruction is commonly present B. Escherichia coli is the most common causative organism C. Intravenous urography will demonstrate a persistent nephrogram on the affected side D. Ultrasound will show an enlarged kidney containing high-amplitude echoes within the collecting system E. CT is the best modality to assess the extent of the disease
D. Ultrasound will show an enlarged kidney containing high-amplitude echoes within the collecting system Emphysematous pyelonephritis represents a severe life-threatening infection of the renal parenchyma with gas-forming bacteria (Escherichia coli in 70%). In 90% of patients, there is a history of underlying poorly controlled diabetes. Abdominal X-ray may show gas bubbles over the renal fossa or a mottled kidney with radially orientated gas corresponding to the renal pyramids. Intravenous urography may show a persistent nephrogram due to delayed excretion of contrast. Ultrasound shows kidney enlargement with high amplitude echoes in the parenchyma. CT shows parenchymal gas and obstruction. Contrast enhancement may be asymmetrical with delayed excretion. In the nephrographic phase, focal abscesses or tissue necrosis may be identified. Grayson DE, Abbott RM, Levy AD, et al. Emphysematous infections of the abdomen and pelvis: a pictorial review. Radiographics 2002; 22: 543–61.
1049
A 40-year-old male presents to the urology clinic with haematuria and hypertension. Contrast-enhanced CT of the abdomen showed an expansile, heterogeneously enhancing mass in the right kidney with invasion of the renal vein. The left kidney is normal. What is the most likely diagnosis? A. Papillary renal cell carcinoma B. Clear cell renal cell carcinoma C. Multilocular cystic renal cell carcinoma D. Renal medullary carcinoma E. Chromophobe renal cell carcinoma
B. Clear cell renal cell carcinoma Renal cell carcinoma (RCC) can be classified into histologic subtypes. Clear cell RCC is the most common, accounting for 70% of all RCCs. They appear heterogeneous at imaging due to the presence of haemorrhage, necrosis and cysts, and typically exhibit an expansile growth pattern and are typically hypervascular on contrast-enhanced studies. Capsular or renal vein invasion is seen in up to 45% of tumours. Papillary RCC is the second most common and typically appears hypovascular and homogeneous on imaging studies, showing lesser degrees of contrast enhancement than clear cell RCC. Bilateral and multifocal tumours are more common in papillary RCCs than in other subtypes. Chromophobe RCC is the third most common subtype and are typically large (average size 8 cm) and demonstrate relatively homogeneous enhancement at CT and MR imaging. Multilocular cystic RCCs typically manifest as multilocular cystic tumours and may show asymmetric septal thickening, septal or wall calcification. Renal medullary carcinoma is extremely rare, occurring almost exclusively in patients with sickle cell trait and appears as an infiltrative, heterogeneous mass with a medullary epicentre. Prasad SR, Humphrey PA, Catena JR, et al. Common and uncommon histologic subtypes of renal cell carcinoma: imaging spectrum with pathologic correlation. Radiographics 2006; 26: 1795–806.
1050
A 45-year-old presents with a painless testicular lump and undergoes an ultrasound. Regarding testicular tumours, which of the following ultrasound characteristics is least suggestive of a seminoma? A. Hypoechoic solitary mass with areas of increased echogenicity and cystic areas B. A solitary mass with mixed echo texture with sonolucent and highly echogenic components C. Uniformly hypoechoic solitary mass D. Multiple hypoechoic masses E. Uniformly hyperechoic solitary mass
E. Uniformly hyperechoic solitary mass Seminoma is the most common pure germ cell tumour accounting for 35–50% of all germ cell tumours. The average patient age is 40.5 years, which is in contrast to non-seminomatous tumours, which usually occur in a younger population. The imaging characteristics reflect their uniform cellular nature, with these tumours being generally uniformly hypoechoic on ultrasound. Teratomas are of mixed echotexture with sonolucent and highly echogenic components, multiple hypoechoic masses are most likely to represent metastasis and a hypoechoic solitary mass with areas of increased echogenicity and cystic areas is suggestive of an embryonal cell carcinoma. Seminomas are highly radiosensitive. Woodward P, Sohaey R, O’Donoghue MJ, et al. Tumors and tumor like lesions of the testis: radiologic–pathologic correlation. Radiographics 2002; 22: 189–216. Dahnert W. Urogenital tract. In: Radiology Review Manual. 6th edn. London, UK: Lippincott Williams and Wilkins; 2007. p. 976.1
1051
A 40-year-old unconscious male was brought into accident and emergency following a road traffic accident. CT abdomen and pelvis was performed as part of the trauma series imaging. A displaced pelvic fracture involving the right pubic bone with widening of the left sacroiliac joint was observed. The following statements are true regarding bladder trauma except? A. The bladder is the second commonest urological organ to be injured after kidney B. Over 70% of patients with traumatic bladder injury have co-existing pelvic fractures C. Extraperitoneal bladder rupture is the commonest type of bladder injury D. Intraperitoneal bladder rupture occurs as a result of penetrating bladder injury at the bladder base E. Imaging findings are commonly normal in bladder contusion
D. Intraperitoneal bladder rupture occurs as a result of penetrating bladder injury at the bladder base Following abdominal trauma, the incidence of urological tract injury is approximately 10%. In traumatic bladder injury, over 70% have associated pelvic fractures. Bladder rupture is frequently associated with gross haematuria. 15% of bladder ruptures are intraperitoneal and frequently follow blunt trauma. A tear in the dome of the bladder results in intraperitoneal extravasation of urine. Contrast is seen outlining small bowel loops and filling the pouch of Douglas. 80% of bladder ruptures are extra-peritoneal and occur following penetrating injury. Contrast medium is seen in the peri-vesical space and may be seen tracking into the peritoneum, scrotum or thigh. Bent C, Iyngkaran T, Power N, et al. Urological injuries following trauma. Clin Radiol 2008; 63: 1361–71.
1052
The urology team refer a 45-year-old male presents with several weeks history of swelling of the scrotum and chronic cough, which started after returning from a holiday. Testicular ultrasound demonstrates a diffusely hypoechoic right testis with thickening of the tunica albuginea and scrotal wall. Doppler studies are equivocal. He was also successfully treated for lymphoma several years previously. What would be your next first step? A. Request a chest X-ray B. Request a staging CT C. Request testicular MRI D. Advise empirical orchitis treatment and follow-up ultrasound in 6 weeks E. Advise urgent urological review for orchidectomy assessment
A. Request a chest X-ray The suspected diagnosis is tuberculosis. A chest X-ray would be the most appropriate next investigation to exclude pulmonary tuberculosis in the context of cough, recent travel and with a previous history of immunocompromised status. A staging CT would be appropriate if suspecting lymphoma relapse. Testicular MRI is helpful to further characterize testicular lesions and narrow the differential diagnosis and can assist in planning surgery. Wise GJ, Marella VK. Genitourinary manifestation of tuberculosis. Urol Clin North Am 2003; 30: 111–21. Burrill J, Willams CJ, Bain G, et al. Tuberculosis: a radiologic review. Radiographics 2007; 27: 1255–73.
1053
A 58-year-old male presented to the urologist with nocturia and poor stream of urine. He was found to have a prostate specific antigen of 10. Histology from prostate biopsy showed Gleason 4 + 3. MRI of the prostate was performed as part of tumour staging. Which of the following is true regarding MRI of the prostate gland? A. The zonal anatomy of the prostate gland is well delineated on T1 weighted images B. Prostate cancer appears as an area of high signal on T2 weighted images C. The transitional zone is normally of high signal on T2 weighted images D. Prostate cancer shows a low apparent diffusion coefficient value compared with the surrounding tissues on diffusion-weighted images E. The choline and creatine to citrate ratio is reduced on MR spectroscopy in prostate cancer
D. Prostate cancer shows a low apparent diffusion coefficient value compared with the surrounding tissues on diffusion-weighted images Seventy percent of prostate cancers arise in the peripheral zone. On T2 weighted images, prostate cancer in the peripheral zone appears as an area of low signal intensity that is easily differentiated from high signal-intensity normal tissue. The zonal anatomy cannot be clearly identified on T1 weighted images as the prostate has uniform intermediate signal intensity. The central and transitional zones are of low signal intensity on T2 weighted images. On diffusion-weighted imaging, prostate cancer shows restriction of diffusion and reduction of apparent diffusion coefficient values compared with the surrounding tissues as the normal glandular architecture is disrupted and replaced by aggregated cancer cells and fibrotic stroma. MR spectroscopy provides metabolic information about prostate tissue by demonstrating the relative concentration of chemical compounds. The ratio of choline to citrate is increased in cancerous tissue. Choi JY, Kim JK, Kim NK, et al. Functional MR imaging of prostate cancer. Radiographics 2007; 27: 63–75.
1054
A 35-year-old female presented with palpitations, flushing and headache. On examination, she was found to be hypertensive. Which of the following statements is least suggestive of phaeochromocytoma? A. An association with multiple endocrine neoplasia Type 2 B. The diagnosis is made by using 24 hour urine vanillyl-mandelic acid C. Homogenous on unenhanced CT D. Classically non-enhancing on post-contrast CT E. Calcification is present in less than 40% of cases
D. Classically non-enhancing on post-contrast CT Phaeochromocytoma most commonly arises from the adrenal medulla and is typically solid and benign. However, in 10–15% of cases, they are malignant. Although most are sporadic, they may be associated with MEN 2, von Hippel-Lindau, neurofibromatosis, tuberous sclerosis and Sturge–Weber syndrome. The diagnosis can be made by measuring urinary vanillylmandelic acid levels. On unenhanced CT, they can be homogeneous (particularly if small) or heterogeneous (larger lesions with necrosis and haemorrhage). Calcification may be seen in up to 29% of cases. Phaeochromocytomas classically show marked avid contrast enhancement with variable washout. Johnson PT, Horton KM, Fishman EK. Adrenal mass imaging with multidetector CT: pathologic conditions, pearls, and pitfalls. Radiographics 2009; 29: 1333–51.
1055
A 50-year-old female patient presented with a 4-month history of dull ache in the lower back and in the right renal angle. She had recently noticed swelling in both legs and had become increasingly breathless. An intravenous urography was performed in the accident and emergency department which showed delayed renal contrast excretion with bilateral hydronephrosis and proximal hydroureter, medial deviation of the middle third of both ureters, and tapering of the ureteral lumen at the L4–L5 vertebral level. Which of the following is the most unifying diagnosis? A. Transitional cell carcinoma of both ureters B. Retroperitoneal fibrosis C. Lymphoma D. Nephrotic syndrome E. Radio-lucent ureteric calculi
B. Retroperitoneal fibrosis Retroperitoneal fibrosis is a rare collagen vascular disorder of unclear cause. Retroperitoneal fibrosis has been increasingly recognized as a systemic condition. Severe pain in the lower back, abdomen and flank areas, and unilateral or bilateral lower extremity swelling are most common presentations. Chest radiographs may show signs of non-cardiogenic pulmonary oedema secondary to nephrogenic fluid overload which would account for this patient’s increasing breathlessness. Mediastinal widening may also be seen and indicate the presence of mediastinal fibrosis. The intravenous urogram findings described are typical with bilateral hydronephrosis seen in 68%. On CT, it most often manifests as a paraspinal, well-demarcated but irregular retroperitoneal mass that is isodense to surrounding muscle. These typical features are not seen in the other answers. Cronin CG, Lohan DG, Blake MA, et al. Retroperitoneal fibrosis: a review of clinical features and imaging findings. AJR Am J Roentgenol 2008; 191: 423–31.
1056
A 63-year-old female with a history of diabetes presented with haematuria and left loin pain. She had a history of recurrent urinary tract infections. A renal stone was suspected and an intravenous urogram was performed. This revealed multiple, small 2 to 3 mm smooth filling defects within the ureter. The most likely diagnosis is: A. Ureteric calculi B. Transitional cell carcinoma C. Pyeloureteritis cystica D. Schistosomiasis E. Blood clots
C. Pyeloureteritis cystica Pyeloureteris cystica consists of multiple, small, fluid-filled, non-communicating cysts in the wall of the renal pelvis and ureter and may be associated with inflammation. The typical urographic findings include multiple small (2–3 mm), smooth filling defects in the ureter. It is most commonly seen in diabetics with recurrent urinary tract infections and more commonly affects females. Kawashima A, Vrtiska TJ, Leroy AJ, et al. CT urography. Radiographics 2004; 24: S35–54.
1057
A 35-year-old female presents with a prolonged history of recurrent urinary tract infections and urinary incontinence. An ultrasound is performed that shows a cystic intravesical mass, contiguous with a normally placed left dilated ureter. An intravenous urography demonstrates a “cobra head” appearance of contrast material within the bladder a surrounding radiolucent halo. The most likely diagnosis is: A. Simple ureterocele B. Bladder calculus C. Transitional cell carcinoma of bladder D. Ectopic ureterocele E. Urachal cyst
A. Simple ureterocele Ureteroceles represent cystic dilatation of the intravesical segment of the ureter and may be associated with either a single or a duplex ureter. Typical bladder ultrasound and intravenous urogram features are described. It may be as small 1 cm or it may fill the entire bladder and prolapse through the urethra. Ureteral duplication is present in about 75% of patients with ureteroceles. In simple ureteroceles, stenosis (usually congenital but may be inflammatory) of the distal end of the normally positioned ureteral orifice leads to ballooning of the segment immediately above it. An ectopic ureterocele is the cyst-like protrusion into the bladder lumen of the dilated submucosal distal portion of an ectopic ureter. It is almost invariably associated with a duplex collecting system and represents the distal portion of the ureter of the upper renal moiety. An ectopic ureterocele is more inferiorly located than a simple ureterocele. An uncomplicated urachal cyst appears as a collection of simple fluid localized in the midline of the anterior abdominal wall, between the umbilicus and the pubis and often contiguous with the bladder dome. Bladder calculus and transitional cell carcinoma appear as a filling defect on intravenous urogram. Berrocal T, López-Pereira P, Arjonilla A, et al. Anomalies of the distal ureter, bladder, and urethra in children: embryologic, radiologic, and pathologic features. Radiographics 2002; 22: 1139–64.
1058
A 44-year-old male complains of malaise and found to be positive for microscopic haematuria and pyuria. A renal ultrasound demonstrates right renal hydronephrosis but otherwise no focal urinary tract lesions. A plain KUB XR is deemed unremarkable. Blood biochemistry reveals hyponatraemia and there is a cavitating lung lesion seen on a concurrent chest X-ray. What is the most likely diagnosis? A. Metastatic squamous cell lung carcinoma B. Metastatic transitional cell carcinoma C. Urinary calculus with obstruction D. Tuberculosis E. Vasculitides
D. Tuberculosis x Genitourinary tuberculosis can cause ureteric strictures with secondary hydronephosis. Pulmonary tuberculosis can be associated with the syndrome of inappropriate antiduretic hormone secretion with secondary hyponatraemia. Tuberculosis, squamous cell carcinoma and vasculitic disorders can all lead to cavitating lung lesions. Abbara A, Davidson RN; Medscape. Etiology and management of genitourinary tuberculosis. Nat Rev Urol 2011; 8: 678–88. Eastwood JB, Corbishley CM, Grange JM. Tuberculosis and the kidney. J Am Soc Nephrol 2001; 12: 1307–14.
1059
A 67-year-old female presents with post-menopausal bleeding. She undergoes a transvaginal ultrasound which reveals an endometrial thickness of 7 mm. Subsequent biopsy confirms adenocarcinoma. Typical features on staging MRI are: A. Isointense to myometrium on T1, hypointense to endometrial lining on T2 B. Isointense to myometrium on T1, hyperintense to endometrial lining on T2 C. Hyperintense to myometrium on T1, hyperintense to endometrial lining on T2 D. Hyperintense to myometrium on T1, hypointense to endometrial lining on T2 E. Hyperintense to myometrium on T1, isointense to endometrial lining on T2
A. Isointense to myometrium on T1, hypointense to endometrial lining on T2 Endometrial cancer is most accurately imaged pre-operatively by MRI. However it can be difficult to differentiate endometrial carcinoma from a blood clot, endometrial hyperplasia and an endometrial polyp. Histological diagnosis prior to staging is essential. In most women, the normal zonal uterine anatomy is demonstrated on T2 weighted imaging. Typically, endometrial carcinoma is isointense to myometrium on T1 weighted sequences and of lower signal intensity than endometrial lining on T2 weighted sequences. After giving contrast, tumours are usually of lower signal intensity than normal myometrial tissue which enhances brightly. Tumours enhance more slowly than the myometrium on dynamic contrast-enhanced images. Barwick TD, Rockall AG, Barton DP, et al. Imaging of endometrial adenocarcinoma. Clin Radiol 2006; 61: 545–55.
1060
A 48-year-old female presented with pelvic discomfort. A trans-abdominal ultrasound revealed a large cystic mass in the pelvis. An MRI was performed showing a cystic ovarian mass. Which of the following features would favour a diagnosis of mucinous adenocarcinoma over serous adenocarcinoma? A. Unilocular B. Bilaterality C. Psammomatous calcification on histology D. Peritoneal carcinomatosis E. Pseudomyxomaperitonei
E. Pseudomyxomaperitonei The two most common types of epithelial ovarian neoplasms are serous and mucinous tumours. A unilocular or multilocular cystic mass with homogeneous CT attenuation or MRI signal intensity, a thin regular wall or septum, and no vegetations is considered to be a benign serous cystadenoma. A multilocular cystic mass with a thin regular wall and septa, no vegetations and containing liquids of different attenuation or signal intensity is considered to be a benign mucinous cystadenoma. Mucinous cystadenomas are often larger than serous cystadenomas. Psammomatous calcification is seen on histology in up to 30% of malignant serous tumours. Bilaterality and peritoneal carcinomatosis are seen more frequently in serous than in mucinous cystadenocarcinomas. Mucinous adenocarcinoma can rupture and is associated with pseudomyxoma peritonei. Jung SE, Lee JM, Rha SE, et al. CT and MR imaging of ovarian tumors with emphasis on differential diagnosis. Radiographics 2002; 22: 1305–25.
1061
A 48-year-old presents to the gynaecological out patients clinic with dysmenorrhoea, menorrhagia and abnormal uterine bleeding. A pelvic MRI was performed showing diffuse thickening of the junctional zone, with a poorly defined border with embedded bright foci on T1 and T2 weighted images. What is the most likely diagnosis? A. Endometrial cancer B. Leiomyoma C. Adenomatoid tumour of the uterus D. Metastasis to the uterus E. Adenomyosis
E. Adenomyosis Adenomyosis is a benign condition characterized by the invasion of ectopic endometrium into myometrium with associated hyperplasia of adjacent smooth muscle. Symptoms include menorrhagia, dysmenorrhoea and abnormal uterine bleeding. On MRI it appears as either diffuse or focal thickening of the junctional zone forming an ill-defined area of low signal intensity, with occasional embedded bright foci on T2 weighted images. These represent the islands of ectopic endometrial tissue and cystic dilatation of glands. Leiomyoma is the most common lesion that clinically resembles adenomyosis. Leiomyoma typically have a well-defined border, and are isointense to myometrium on T1 weighted images and hypointense on T2 weighted images. In addition dilated vessels maybe seen at the margin of the lesion. Tamai K, Togashi K, Ito T, et al. MR imaging findings of adenomyosis: correlation with histopathologic features and diagnostic pitfalls. Radiographics 2005; 25: 21–40. Chapman S, Nakielny R. Abdomen and gastrointestinal tract. In: Aids to Radiological Differential Diagnosis. 5th edn. London, UK: Saunders; 2009. pp. 306–7.
1062
A 31-year-old female with a history of acute lower abdominal pain was investigated with an ultrasound which demonstrated an enlarged left ovary with peripherally placed cysts, small amount of free fluid, peripheral arterial flow and absent central venous flow on Doppler imaging. What is the most likely diagnosis? A. Polycystic ovaries B. Ovarian hyperstimulation syndrome C. Ovarian torsion D. Serous cystadenoma E. Endometriosis
A. Polycystic ovaries Sonographic features of ovarian torsion are unilateral enlarged ovary with peripherally placed cysts, free fluid in the pelvis and an echogenic twisted vascular pedicle. It may be associated with a co-existent mass within the ovary, which predisposes to torsion. Colour Doppler findings are not consistent because of the variable degree of torsion. However, ovarian torsion rarely manifests with completely normal venous waveforms. Arterial flow can be reduced but typically only with a concomitant venous flow abnormality. Ovarian hyperstimulation syndrome presents with bilaterally enlarged ovaries with peripherally placed follicles, normal vascular flow and associated ascites and pleural effusions in severe cases. Patients are at greater risk for ovarian torsion. Polycystic ovaries present with bilaterally enlarged ovaries with small subcentimetre-sized peripherally placed follicles, echogenic stroma and normal flow on Doppler. Serous cystadenomas commonly presents as large, unilocular, thin-walled cystic lesions that may have thin septations or papillary projections. Chang HC, Bhatt S, Dogra VS. Pearls and pitfalls in diagnosis of ovarian torsion. Radiographics 2008; 28: 1355–68.
1063
A 46-year-old is being investigated for chronic pelvic pain. Ultrasound showed bilateral large complex adnexal masses with homogenous low-level echoes. On MRI, the lesions appear hyperintense on T1 weighted imaging. On T2 weighted imaging, the right adnexal mass remains hyperintense and the left adnexal mass appears markedly hypointense. What is the most likely diagnosis? A. Dermoid cysts B. Mucinous cystadenomas C. Endometriomas D. Haemorrhagic cysts E. Ovarian cysts with proteinaceous contents
C. Endometriomas Endometriomas commonly appear as multiple hyperintense cysts on T1 weighted images regardless of their signal intensity on T2 weighted images. Another characteristic feature is that they appear more conspicuous on T1 weighted fat suppressed sequence. Endometriomas demonstrate shading (i.e., loss of signal within the lesion), on T2 weighted images which can range from faint, dependent layering to complete signal void, reflecting the concentration of blood products. Endometriomas with degenerated blood products, including methaemoglobin and concentrated protein appear with high signal-intensity areas on T1 and T2 weighted images. Haemorrhagic cysts can have similar appearances, however, they are usually unilocular as opposed to endometriomas, which are frequently multilocular and bilateral. In addition, haemorrhagic cysts do not exhibit shading on T2 weighted images. Woodward PJ, Sohaey R, Mezzetti TP. Endometriosis: radiologic–pathologic correlation. Radiographics 2001; 21: 193–216.
1064
A 32-year-old female with acute lower abdominal pain and a palpable right-sided pelvic mass was referred for an urgent ultrasound scan which showed a cystic mass within the region of the right ovary which had multiple interdigitating fine strands within it. Colour Doppler showed no flow within the mass. What is the most likely diagnosis? A. Ovarian carcinoma B. Endometrioma C. Simple ovarian cyst D. Haemorrhagic ovarian cyst E. Ruptured ectopic pregnancy
D. Haemorrhagic ovarian cyst Most ovarian cysts are functional cysts resulting from failure of rupture or regression of an ovarian follicle. Different kinds of functional cysts can develop during a normal menstrual cycle such as follicular cysts, corpus luteal cysts and theca luteal cysts. Physiological ovarian cysts are found in all age groups and are extremely common in women of reproductive age group. These can become symptomatic when complicated by haemorrhage, rupture and torsion. Patients with haemorrhagic ovarian cysts clinically presents with an acute onset of pelvic pain or lower abdominal pain and pelvic mass. Common appearances of haemorrhagic ovarian cysts on ultrasound include a cystic mass with multiple interdigitating fine strands. A similar multiseptate appearance is described in neoplasms, however, the septae in neoplasms are thicker, more reflective and visualized in a continuous manner than the fibrin strands of a haemorrhagic cyst. Another common appearance is a retracting blood clot in an anechoic cyst. When the blood clot is retracted and small it can simulate a mural nodule in a cystic neoplasm. The blood clot of a cyst will, however be triangular or curvilinear in shape where as a mural nodule will have convex margins. Levin D, Brown DL, Andreotti RF, et al. Management of asymptomatic ovarian and other adnexal cysts imaged at US: Society of Radiologists in Ultrasound Consensus Conference Statement. Radiology 2010; 256: 943–54.
1065
A 40-year-old female has a pelvic ultrasound for menorrhagia and dysmenorrhoea demonstrating several well defined, intramural lesions of low echogenicity in keeping with fibroids. The adenexal appearances are normal and there is no free fluid within the pelvis. What are the most likely appearances on pelvic MRI when compared to the normal myometrium? A. High signal intensity on T1 and T2 weighted imaging on the pre-contrast imaging with no enhancement following intravenous contrast B. Low signal intensity on T1 and high signal intensity on T2 weighted imaging on the pre-contrast imaging with avid enhancement on T1 weighted imaging post-intravenous contrast C. Isointense signal on T1 and low signal intensity on T2 weighted imaging with no enhancement following intravenous contrast on T1 weighted imaging D. Isointense signal on T1 and low signal intensity on T2 signal with enhancement following intravenous contrast on T1 weighted imaging E. High signal intensity on T1 and mixed signal intensity on T2 weighted imaging with heterogeneous enhancement following intravenous contrast on T1 weighted imaging
D. Isointense signal on T1 and low signal intensity on T2 signal with enhancement following intravenous contrast on T1 weighted imaging Fibroids (leiomyomas) are benign, smooth muscle tumours. They commonly present with abnormal vaginal bleeding and abdominal pain and have a prevalence of 20–30% in women over the age of 30 years. On MRI, they are generally of low signal on T2 weighted imaging but can have a variable appearance depending on the amount of haemorrhage, necrosis and cellular degeneration. They are isointense on pre-contrast T1 weighted imaging and demonstrate enhancement following intravenous contrast. Szklaruk J, Tamm EP, Choi H, et al. MR imaging of common and uncommon large pelvic masses 1. Radiographics 2003; 23: 403–24.
1066
A 28-year-old female is investigated for multiple miscarriages. Amongst the investigations she undergoes, she has a pelvic MRI to assess her pelvic anatomy. This is reported as showing a heart shaped uterus with widely separated uterine horns and a single cervix. Which uterine abnormality is being described? A. Didelphus B. Bicornuate C. Septated D. Unicornuate E. Diethylstilbesterol related
B. Bicornuate Anatomical uterine abnormalities are related to müllerian duct anomalies and are also associated with urinary tract anomalies and may be classified as follows: * Class I – Hypoplasia/agenesis, presenting with primary amenorrhoea in adolescence. Normally, the ovaries and Fallopian tubes are present. * Class II – Unicornuate uterus with a banana-shaped uterus and single Fallopian tube. Women present with spontaneous abortion or early miscarriage. * Class III – Didelphus uterus with two uteri and two cervices. * Class IV – Bicornuate uterus with abicornuate, heart-shaped uterus with one cervix. Diagnostic criteria are divergence of uterine horns of >4 cm; concavity of fundal contour or an external fundal cleft of >1 cm. * Class V – Septate where the septum is a combination of fibrous tissue and muscle. It is of low T2 signal when predominantly fibrous but can be of intermediate signal if highly muscular. It differs from bicornuate uterus in that the external contour of the uterus is normal or indented <1 cm. * Class VI – Related to diethylstilbesterol, used historically to prevent miscarriage. Exposed female foetuses have a 50% incidence of uterine anomalies including uterine hypoplasia and “T shaped” uterus where the fundus is thinner than the cervix. No association with renal anomalies. Imaoka I, Wada A, Matsuo M, et al. MR imaging of disorders associated with female infertility: use in diagnosis, treatment and management. Radiographics 2003; 23: 1401–21.
1067
A 35-year-old female presents to the emergency department with epigastric pain. An abdominal ultrasound is performed which shows a pelvic cystic lesion with a mural nodule causing posterior acoustic shadowing. The patient is treated conservatively, her symptoms resolve and an outpatient MRI pelvis is arranged. What are the most likely features shown on MRI? A. A cyst with a fluid–fluid level showing low T1 weighted signal with no change in the dependent layer on fat saturated imaging B. A cyst with fluid–fluid level showing high T1 weighted signal with signal dropout in the non-dependent layer on fat saturated imaging C. A cyst with a fluid–fluid level showing high T2 weighted signal with dependent lipid laden cyst fluid D. A cyst with a fluid–fluid level showing dependent fluid hyperintensity compared with adjacent muscle on in phase gradient echo imaging E. A cyst with a fluid–fluid level showing a thin white outline between water and fat interface on out of phase gradient echo imaging
B. A cyst with fluid–fluid level showing high T1 weighted signal with signal dropout in the non-dependent layer on fat saturated imaging Findings are that of an ovarian dermoid or mature cystic teratoma. On ultrasound, it is commonly a cystic mass with a nodule arising from the inner surface of the cyst. This is known as a Rokintansky nodule containing hair, teeth and fat with acoustic shadowing. The presence of calcification is not diagnostic, but the diagnosis is made if the presence of fat is confirmed. T1 weighted imaging with fat saturation is necessary for demonstrating the presence of fat. MRI findings are of a cyst with a fluid–fluid level with hyperintense lipid laden, non-dependent cyst fluid on T1 weighted imaging. On in-phase gradient echo imaging the dependent fluid is hypointense compared with muscle. The non-dependent fluid is slightly hypointense compared with surrounding fat. On out-of-phase gradient echo imaging, the dependent fluid remains hypointense compared with muscle and the interface between the water and fat content of the cyst is a thin black line. Park SB, Kim JK, Kim K, et al. Complications and unusual manifestations of ovarian teratomas. Radiographics 2008; 28: 969–83.
1068
A 33-year-old female, who had previously had three children all delivered by caesarean section, experiences vaginal bleeding during the third trimester of her fourth pregnancy. She underwent transabdominal ultrasound and subsequently MRI of the pelvis. A normal placenta–myometrium interface was present, with the lowest margin of the placenta 0.5 cm from the internal cervical os. Which of the following is the most likely abnormality? A. Abruptio placentae B. Placenta praevia C. Placenta accreta D. Placenta percreta E. Placenta increta
B. Placenta praevia Placenta praevia is abnormal implantation of the placenta in the lower uterine segment, near to or overlying the cervical os. The lower placental margin should be at least 2 cm from the internal os. Subtypes based on proximity to the os include low-lying, marginal, complete and central. The normal decidua forms a barrier to deep invasion of chorionic villi into the uterus. In placenta accreta, there is a loss of the normal border between placenta and myometrium. Placenta increta and percreta are more severe forms of accreta in which villi invade and penetrate through the myometrium, sometimes into the urinary bladder. Abruptio placentae represents premature separation of the placenta from the uterine wall. Although rare, third-trimester abruption is associated with an increased risk of preterm delivery and foetal death. Elsaye KM, Trou AT, Friedki AM, et al. Imaging of the placenta: a multimodality pictorial review. Radiographics 2009; 2, 1373–91.
1069
A 40-year-old female presented with a 2-year history of pelvic pain. On clinical examination, a palpable lump is seen protruding into the vagina. The mucosa overlying the lump is normal in appearance. MRI of the pelvis was performed and showed a 2 cm cystic lesion in the anterolateral right paravaginal region. This lesion demonstrates low signal on T2 weighted imaging and high signal on T1 weighted imaging. It does not demonstrate any contrast enhancement. Which of the following is the most likely diagnosis? A. Gartner cysts B. Nabothian cysts C. Adenomyosis D. Endometriosis E. Fibroid
A. Gartner cysts Gartner cysts are embryological remnants of mesonephric (Wolffian) ducts. Most are small (<3 cm), paravaginal and in an anterolateral position. They are high signal on T2 weighted images when they contain simple fluid. If they contain blood or protein, they may be high signal on T1 weighted and low signal on T2 weighted images. Nabothian cysts are common, benign lesions seen on the surface of the cervix. They occur when cervical mucous glands are obstructed after new tissue regenerates on the cervix after childbirth. They have high signal intensity on T2 weighted MR images, Adenomyosis is a benign condition in which there is migration of glands from the endometrium to into the myometrium. Endometriosis is a condition where functional endometrial tissue implants outside the uterine musculature onto the surface of other organs and responds to hormonal stimuli. Fibroids are generally of low signal on T2 weighted imaging but can have a variable appearance depending on the amount of haemorrhage, necrosis and cellular degeneration. Kuligowska E, Deeds L 3rd, Lu K 3rd, et al. Pelvic pain: overlooked and underdiagnosed gynecologic conditions. Radiographics 2005; 25: 3–20. Chaudhari VV, Patel MK, Douek M, et al. MR imaging and US of female urethral and periurethral disease. Radiographics 2010; 30: 1857–74. doi: 10.1148/rg.307105054
1070
A 65-year-old female was found to have a palpable pelvic mass during a routine examination by her general practitioner. She has no other symptoms and does not report any weight loss. Trans-abdominal ultrasound demonstrated a mass in the pelvis. MRI of the pelvis was performed. Which of the following statements is least suggestive of ovarian fibroma? A. Patients with ovarian fibromas are usually asymptomatic B. Associated with ascites and pleural effusion C. Commonly seen on ultrasound as a solid hypoechoic mass with sound attenuation D. On contrast-enhanced CT scan, the mass is well circumscribed and shows homogenous intense enhancement E. On T2 weighted MRI, the mass show very low signal intensity similar to that of muscle
D. On contrast-enhanced CT scan, the mass is well circumscribed and shows homogenous intense enhancement Ovarian fibromas are ovarian tumours of gonadal stromal origin and account for approximately 4% of all ovarian neoplasms. Patients are generally asymptomatic, and masses are typically detected in middle-aged women at palpation during routine examination. Ovarian fibromas appear as solid masses on imaging, mimicking malignant neoplasms. They are associated with ascites in 40% of cases, particularly in larger lesions, and with pleural effusions (Meig syndrome) in a minority. At ultrasound, they most commonly manifest as solid, hypoechoic masses with sound attenuation. At CT, they manifest as diffuse, slightly hypoattenuating masses with poor and very slow contrast enhancement. On T1 weighted MR images, they demonstrate homogeneous, relatively low signal intensity. On T2 weighted MRI, they are of very low signal intensity similar to that of muscle, a finding that is characteristic of an ovarian fibroma. Jeong YY, Outwater EK, Kang HK. Imaging evaluation of ovarian masses. Radiographics 2000; 20: 1445–70.
1071
A 14-year-old boy presents to his general practitioner with worsening pain in his left thigh. The pain tends to occur at night and has not been relieved by paracetamol. A plain radiograph reveals a lucent lesion in the proximal femur with associated periosteal reaction. What is the likely diagnosis? A. Cortical desmoid B. Osteoid osteoma C. Stress fracture D. Brodie abscess E. Enostosis
B. Osteoid osteoma An osteoid osteoma is a painful, benign tumour of bone that predominantly occurs in adolescents and children. They classically present with nocturnal pain that is relieved by non-steroidal anti-inflammatory agents, for example, aspirin. The plain radiograph may reveal a small lytic lesion with periosteal reaction and a central nidus, but this is not always the case. Typically there is reactive sclerosis around a focal lucency often referred to as the nidus. Cortical desmoids are avulsive injuries of the femoral insertion of the medial head of gastrocnemius or adductor magnus aponeurosis caused by repetitive stress and are located in the posteromedial distal femoral diaphysis. Stress fractures tend to have a more linear appearance and would cause more pain during activity, not during rest at night. Brodie abscess is a focus of chronic osteomyelitis which tends to be lytic and is often oval in shape within the metaphysis of tubular bones. Enostosis (bone island) is a benign dense cortical island within the bone rather than a lucent lesion as described and is asymptomatic Motamedi D, Learch TJ, Ishimitsu DN, et al. Thermal ablation of osteoid osteoma: overview and step-by-step guide. Radiographics 2009; 29: 2127–41. doi: 10.1148/rg.297095081 Chai JW, Hong SH, Choi JY, et al. Radiologic diagnosis of osteoid osteoma: from simple to challenging findings. Radiographics 2010; 30: 737–49. doi: 10.1148/rg.303095120
1072
A 5-year-old child has recurrent urinary tract infections and the general practitioner requests an ultrasound of the urinary tract to exclude a structural abnormality. Both kidneys are structurally normal on ultrasound, but on the right side, a cystic area is associated with the bladder wall and there is dilatation of the distal ureter on that side. What is the most likely diagnosis? A. Simple ureterocele B. Bladder diverticulum C. Pseudoureterocele D. Ectopic ureterocele E. Transitional cell carcinoma
A. Simple ureterocele A ureterocele is a cystic dilatation of the ureter at the vesico-ureteric junction (VUJ). It may be simple (occur at the site of the normally placed VUJ), or ectopic. Ectopic ureteroceles account for 75% of all ureteroceles and are almost exclusively seen with a duplicated collecting system. In this case, the kidneys are normal and there is no sign of a duplicated system. Simple ureteroceles usually occur in adults, but when they do occur in children, they tend to cause symptoms. Bladder diverticula are seen as focal outpouchings of the bladder wall. A pseudoureterocele is a dilated distal ureter caused by an obstructing tumour or calculus. An ectopic ureterocele has a similar appearance but occurs at a site other than the native VUJ. Berrocal T, López-pereira P, Arjonilla A, et al. Anomalies of the distal ureter, bladder, and urethra in children: embryologic, radiologic, and pathologic features. Radiographics 2002; 22: 1139–64.
1073
At a prenatal foetal anatomy scan, there is concern about the intracranial appearances of the foetus. There is a large cystic area intracranially with no cortical mantle but a normal cerebellum. The remainder of the scan confirms additional abnormalities including congenital heart disease and polyhydramnios. What is the most likely diagnosis for the intracranial appearances? A. Hydrocephalus B. Holoprosencephaly C. Hydranencephaly D. Schizencephaly E. Porencephaly
C. Hydranencephaly Hydranencephaly is a rare encephalopathy that occurs in utero and results in complete destruction of the cerebral hemispheres. There is transformation of the cerebral hemispheres into a membranous sac that contains cerebrospinal fluid and remnants of the cortex with the brainstem and basal ganglia. There are a number of causes that include infection, leukomalacia, diffuse hypoxic–ischaemic brain necrosis, infarction and thromboplastic material from a deceased co-twin. Holoprosencephaly is a rare congenital brain malformation that results in incomplete separation of the cerebral hemispheres and is associated with facial abnormalities and trisomy 13. In schizencephaly, a cleft extends through the cerebral hemisphere from the ependyma of the ventricular system to the pia mater; there should be relatively preserved cortical matter apart from the cleft. Porencephaly refers to focal encephalomalacia that communicates with the ventricles; there may be localized cystic degeneration with malformation of the skull as a result of cerebrospinal fluid pulsation. The lack of cortical mantle makes hydrocephalus unlikely. Kurtz AB, Johnson PT. Diagnosis please. Case 7: hydranencephaly. Radiology 1999; 210: 419–22.
1074
A term baby is delivered at 40 + 4 weeks to a diabetic mother. The labour is prolonged and forceps delivery is required. Following delivery, the baby develops respiratory distress and is intubated. The chest radiograph is initially normal, but 24 hours later there are features of hyperinflation with atelectasis and extensive bilateral nodular airspace shadowing. What is the most likely diagnosis? A. Transient tachypnoea of the newborn B. Meconium aspiration syndrome C. Neonatal pneumonia D. Pulmonary haemorrhage E. Respiratory distress syndrome
B. Meconium aspiration syndrome Meconium aspiration syndrome occurs most commonly in babies who are large (macrosomia being associated with maternal diabetes), and have prolonged and difficult deliveries. Aspiration of meconium causes a severe inflammatory pneumonitis and may also cause mechanical obstruction. The radiograph may initially be normal, but classically there is development of bilateral and extensive airspace opacities and hyperinflation due to air trapping. Treatment is supportive and in severe cases will include mechanical ventilation, suction and antibiotics. Transient tachypnoea of the newborn is characterized by a pulmonary oedema type lung injury with fluid within the interstitium and airspaces; it is most commonly seen in term babies delivered by caesarean section. Neonatal pneumonia may have similar radiographic appearances to meconium aspiration syndrome (bronchial obstruction and atelectasis may occur in both conditions). Respiratory distress syndrome affects premature infants who do not have appropriate surfactant in their lungs at delivery; relatively stiff lungs fail to inflate adequately and oxygenation is insufficient. Gooding CA, Gregory GA. Roentgenographic analysis of meconium aspiration of the newborn. Radiology 1971; 100: 131–40. doi: 10.1148/100.1.131
1075
A 6-year-old girl presents to the accident and emergency department (A&E) with increasing abdominal pain and a right upper quadrant mass. She has had problems with constipation for the last couple of months and the A&E registrar asks for an ultrasound. A chest radiograph performed in A&E is normal. The ultrasound demonstrates a large soft tissue mass in the right hypochondrial region which is inseparable from the kidney and extends towards the midline. Staging investigations are performed on the mass. Which of the following features does not fit with the presumed diagnosis? A. A rim of normal renal tissue B. Extension into the spinal canal C. Renal vein involvement D. Pulmonary metastases E. Compression, not invasion of local tissues
B. Extension into the spinal canal The diagnosis here is of a Wilms’ tumour. These are renal tumours which may grow to be large and present with abdominal pain or symptoms related to compression of local structures. Wilms’ tumours tend to compress rather than invade local structures. Lung metastases occur via the renal vein and inferior vena cava meaning that careful assessment of these vessels is key to correct staging of the tumour. They originate in the renal parenchyma and should have a rim of normal renal tissue peripherally. 15% of Wilms’ tumours calcify. The other common childhood tumour in this region is neuroblastoma. Neuroblastomas originate from sympathetic nervous tissue and commonly from within the adrenal gland. They invade local structures, including the ipsilateral kidney and calcify heavily. Metastases to bone are common and this may be the presenting feature. When staging neuroblastoma it is essential to identify extension into the spinal canal and describe the degree of cord displacement. Gylys-Morin V, Hoffer FA, Kozakewich H, et al. Wilms tumor and nephroblastomatosis: imaging characteristics at gadolinium-enhanced MR imaging. Radiology 1993; 188: 517–21.
1076
A 9-year-old boy is admitted to the paediatric cardiology ward following an episode of ventricular tachycardia with cardiovascular instability terminated with defribillation in the accident and emergency department. He has a history of congenital heart disease that presented with cyanosis in infancy and has had a number of operations to repair the abnormality. His ECG reveal right bundle branch block which is longstanding and ventricular ectopy. His chest radiograph shows a boot-shaped heart and an implantable cardiac defribillator. What is the likely underlying diagnosis? A. Aortic coarctation B. Tetralogy of Fallot C. Atrial septal defect D. Transposition of the great vessels E. Ventricular septal defect
B. Tetralogy of Fallot Tetralogy of Fallot is one of the cyanotic congenital heart diseases. It is composed of four co-existing anomalies: ventricular septal defect (VSD), over-riding aorta, pulmonary atresia and right ventricular hypertrophy. As long as the ductus arteriosis remains patent, there is collateral pulmonary flow and cyanosis may not present for several days or weeks. The commonest cause of right bundle branch block in children is repair of a VSD or other cardiac abnormality. The ventricular ectopics and episode of ventricular tachycardia, as well as the implantable defibrillator point to an underlying ventricular disease. Both atrial septal defect and aortic coarctation do not cause cyanosis and the vignette describes features of ventricular disease. Transposition of the great vessels does cause cyanosis, but this is present from birth. A simple VSD should not require multiple operations and do not usually cause ventricular ectopy and VT. Haider EA. The boot-shaped heart sign. Radiology 2008; 246: 328–9. doi: 10.1148/radiol.2461041673
1077
A 13-year-old girl complains of severe dysmenorrhoea. She started menstruating 6 months previously and her periods are regular. She has no significant past medical history apart from obesity and clinical examination is unremarkable. Transabdominal ultrasound is normal except that the uterus is noted to lie in a relatively lateral position. What is the most appropriate next investigation? A. Laparoscopy B. Transvaginal ultrasonography C. MRI pelvis D. Hysteroscopy E. Hysterosalpingography
C. MRI pelvis MRI is usually the most helpful imaging modality in demonstrating the female genital tract. This particular presentation is most suggestive of an endometrium-lined, non-communicating rudimentary uterine horn associated with a unicornuate uterus. Unicornuate uterus is one of the commoner congenital uterine anomalies, comprising a single-horned uterus which opens normally into the vagina. It is usually associated with a non-communicating contralateral rudimentary uterine horn; if this is lined with endometrium, dysmenorrhoea will result. Classical ultrasound appearances are of a laterally displaced, small, rounded uterus. MRI is superior to ultrasound in demonstrating the rudimentary horn, particularly in an obese patient. Transvaginal ultrasonography is usually avoided in this age of patient and hysteroscopy and hysterosalpingography will not demonstrate a non-communicating rudimentary horn directly. Laparoscopic resection of the rudimentary horn may be the appropriate treatment, but MRI should precede laparoscopy to confirm the diagnosis. Junqueira BL, Allen LM, Spitzer RF, et al. Mullerian duct anomalies and mimics in children and adolescents: correlative intraoperative assessment with clinical imaging. Radiographics 2009; 29: 1085–103.
1078
An ultrasound scan is performed on an 18-month-old infant with scrotal swelling. The right testis is markedly enlarged, predominantly hyperechoic with several small hypoechoic areas. The left testis and both epididymides are normal. What is the most likely diagnosis? A. Teratoma B. Gonadoblastoma C. Rhabdomyosarcoma D. Lymphoma E. Yolk sac tumour
E. Yolk sac tumour Yolk sac tumour (also known as endodermal sinus tumour) is the commonest testicular tumour in young boys. It usually presents as a painless scrotal mass before the age of 2 years. Ultrasound features are non-specific, but typically, a solid mass replaces the testis and contains hypoechoic areas secondary to central necrosis. In general, non-germ cell tumours are less common than germ cell tumours and may arise from sex cord cells (Sertoli cell tumours), stroma (Leydig cell tumours) or a combination of sex cord cells and stroma (gonadoblastoma). Teratoma is the principal differential diagnosis, but is uncommon in this age group. Gonadoblastoma is a mass that contains a combination of sex cord cells and stroma. They are considered a dysgenetic lesion associated with intersex syndromes rather than a true neoplasm. Rhabdomyosarcoma do not arise from the testis but are the commonest paediatric extratesticular scrotal tumour. Lymphoma is uncommon in this age group, but may involve the epididymis and is often bilateral. Aso C, Enríquez G, Fité M, et al. Gray-scale and color Doppler sonography of scrotal disorders in children: an update. Radiographics 2005; 25: 1197–214.
1079
An ultrasound scan is performed on a 6-month-old boy with a left flank mass. It demonstrates a multicystic mass arising from the left kidney, containing multiple anechoic spaces separated by thin echogenic septae. The radiologist considers a diagnosis of cystic nephroma. Which of the following imaging features would suggest an alternative diagnosis? A. A peripheral claw of normal renal parenchyma on ultrasound B. Septal enhancement on MRI C. A corresponding photopenic area on DMSA scintigraphy D. A corresponding abnormality on antenatal ultrasound E. Cystic change within the renal pelvis and proximal ureter
D. A corresponding abnormality on antenatal ultrasound Cystic nephroma is an uncommon benign renal tumour, usually presenting as an abdominal mass in male infants but with a second peak incidence in middle-aged women. It is typically a multicystic mass arising from the lower renal pole, containing multiple anechoic spaces separated by thin echogenic septae. Presentation in the antenatal or neonatal periods is not recognised. This is in contradistinction to mesoblastic nephroma which is usually solid (but can be multicystic). Typical features of cystic nephroma include: * surrounding claw of normally enhancing renal parenchyma * septal enhancement * corresponding photopenic area on DMSA scintigraphy * herniation of the cysts into the renal pelvis and proximal ureter. Hopkins JK, Giles HW Jr, Wyatt-Ashmead J, et al. Best cases from the AFIP: cystic nephroma. Radiographics 2004; 24: 589–93.
1080
A term male neonate requires early intubation for respiratory failure and is discovered to have an elevated serum creatinine level. Portable ultrasound at 12 hours demonstrates markedly enlarged kidneys that contain multiple cysts measuring up to 4 mm. The renal parenchyma between the cysts is hyperechoic and the cortex cannot be clearly distinguished from the medulla. What is the most likely diagnosis? A. Autosomal dominant polycystic kidney disease B. Autosomal recessive polycystic kidney disease C. Multicystic dysplastic kidneys D. Cystic renal dysplasia E. Acute cortical necrosis
B. Autosomal recessive polycystic kidney disease The ultrasound features described are typical of autosomal recessive polycystic kidney disease. When this condition presents in the perinatal period, associated pulmonary hypoplasia accounts for much of the morbidity and mortality. As the age of presentation increases, renal failure, and later periportal fibrosis, become more prominent clinical features. Autosomal dominant polycystic kidney disease would have similar ultrasound appearances but rarely presents at such an early age. Multicystic dysplastic kidneys contain variably sized cysts that are often large and have limited intervening renal parenchyma. They are rarely bilateral and are not usually associated with renal failure. Cystic renal dysplasia may have similar appearances (if bilateral), although the kidneys are not enlarged. Acute cortical necrosis results in normally sized or small kidneys with increased cortical echogenicity and relatively prominent hypoechoic medullary pyramids. Mercado-Deane MG, Beeson JE, John SD, et al. US of renal insufficiency in neonates. Radiographics 2002; 22: 1429–38.
1081
A 3-year-old boy is investigated for a palpable abdominal mass. Ultrasound demonstrates a large, echogenic left paravertebral mass containing multiple small hyperechoic areas. There is inferolateral displacement of the adjacent kidney and anterior displacement of the aorta and inferior vena cava. Given the likely diagnosis, which of the following describes the most appropriate series of investigations? A. CT chest; MRI head, neck, abdomen and pelvis; MIBG; skeletal scintigraphy B. CT chest, abdomen and pelvis; MRI whole spine; octreotide scan C. Chest X-ray; MRI abdo/pelvis; octreotide scan; skeletal scintigraphy D. CT chest, abdomen and pelvis; MRI whole spine; MIBG E. Chest X-ray; CT head, neck, abdomen and pelvis; MIBG; skeletal scintigraphy
A. CT chest; MRI head, neck, abdomen and pelvis; MIBG; skeletal scintigraphy A large retroperitoneal solid mass, with calcification, causing displacement of the kidney and major blood vessels, is strongly suggestive of neuroblastoma in this age of patient. It is worth remembering that imaging protocols vary between centres. Cross-sectional imaging of the head, neck, chest, abdomen and pelvis is recommended for local staging and the detection of metastases. MRI reduces ionizing radiation and is superior to CT in evaluating organ invasion, liver metastases and bone marrow involvement. MIBG (123I-MIBG scintigraphy) may assist characterization of the primary tumour, but also detects metastases and helps when monitoring treatment response. Skeletal scintigraphy (99mTc-MDP skeletal scintigraphy) has an established role in the identification of skeletal metastases, although its value has diminished somewhat with the more widespread application of MIBG. Neither chest radiography nor dedicated MRI of the whole spine is indicated for staging purposes in this case. Kushner B. Neuroblastoma: a disease requiring a multitude of imaging studies. J Nuc Med 2004; 45: 1172–88.
1082
An ultrasound is performed to investigate an abdominal mass in a 2-year-old male child. Adjacent to the right kidney upper pole is a lobulated, heterogeneous mass that measures 18 cm in diameter. The radiologist notes that the child has a facial rash and a large tongue. What is the most likely diagnosis for the abdominal mass? A. Neuroblastoma B. Phaeochromocytoma C. Adrenal haemorrhage D. Adrenocortical carcinoma E. Rhabdomyosarcoma
D. Adrenocortical carcinoma Adrenocortical carcinoma is an aggressive adrenal tumour which is associated with Beckwith–Wiedemann syndrome (as are neuroblastoma and hepatoblastoma). Beckwith-Wiedemann syndrome is one of the commonest causes of macroglossia in a child of this age. Furthermore, adrenocortical neoplasms in children are usually functioning, resulting in pseudo-precocious puberty and/or Cushing syndrome. Apart from acne, the patient may also have pubic hair and penile enlargement. Neuroblastoma is the commonest suprarenal mass in this age of patient, but the acne does not fit and there is no description of calcification or other features of neuroblastoma. Phaeochromocytoma (a tumour of the adrenal medulla that secretes noradrenaline) results in hypertension, headaches and haemorrhage. Adrenal haemorrhage would cause adrenal enlargement but not result in a lobulated mass. Rhabdomyosarcoma is a subtype of sarcoma which may present as a suprarenal mass, but is not the most likely option here. Agrons GA, Lonergan GJ, Dickey GE, et al. Adrenocortical neoplasms in children: radiologic–pathologic correlation. Radiographics 1999; 19: 989–1008.
1083
Urine is noted to leak from the umbilicus of a term male neonate, whose mother had declined antenatal ultrasound scanning. Clinical examination is unremarkable and both ultrasound and micturating cysto-urethrography are requested. Which of the following would be a surprising imaging finding? A. Vesicoureteric reflux B. A dilated, elongated prostatic urethra C. Bladder wall thickening D. A tubular channel from the trigone of the bladder to the umbilicus E. Bilateral hydronephrosis
D. A tubular channel from the trigone of the bladder to the umbilicus The clinical presentation is diagnostic of a persistent urachus, which extends from the anterosuperior dome of the bladder (and not the trigone) to the umbilicus. There are many associations. Vesicoureteric reflux, bladder wall thickening and bilateral hydronephrosis are associated with a urachal remnant abnormalities. There is also an association between urachal abnormalities and posterior urethral valves that typically result in a dilated, elongated posterior urethra with an abrupt transition to normal calibre at its junction with the anterior urethra. Berrocal T, López-Pereira P, Arjonilla A, et al. Anomalies of the distal ureter, bladder and urethra: embryologic, radiologic and pathologic features. Radiographics 2002; 22: 1139–64.
1084
A phenotypically female term neonate is noted to have a single perineal opening through which both urine and meconium pass. Otherwise, clinical examination is unremarkable. Under fluoroscopic guidance, contrast is instilled through this perineal opening and outlines three distinct chambers. What is the diagnosis? A. Cloacal malformation B. Persistent urogenital sinus C. Imperforate anus D. Vaginal agenesis E. Cloacal exstrophy
A. Cloacal malformation A cloacal malformation comprises a single perineal opening through which the genital, urinary and gastrointestinal tracts drain. It results from incomplete segregation of the cloaca by the urorectal septum into the urogenital sinus anteriorly and the intestinal canal posteriorly. With a persistent urogenital sinus, the bladder and vagina drain through a common opening, but the anal canal drains separately. In imperforate anus, the anus is not formed and may be associated with other malformations of the bowel. In vaginal agenesis, the urethral and anal meati should be normal although associated malformation may occur. Cloacal exstrophy is a major embryonic defect of the lower anterior abdominal wall where urine and meconium drain through the abdominal wall defect. Berrocal T, López-Pereira P, Arjonilla A, et al. Anomalies of the distal ureter, bladder and urethra: embryologic, radiologic and pathologic features. Radiographics 2002; 22: 1139–64.
1085
A 9-year-old boy is taken to his general practitioner complaining of a 3-week history of weight loss, vomiting and headaches, worst in the morning. Contrast-enhanced CT reveals a homogeneously enhancing, hyperdense mass in the posterior fossa. On MRI, the lesion returns low signal on T1 and mixed T2 signal, restricted diffusion on diffusion-weighted imaging, and no blooming artefact on susceptibility-weighted imaging. What is the most likely diagnosis? A. Ependymoma B. Astrocytoma C. Meningioma D. Medulloblastoma E. Acoustic neuroma
D. Medulloblastoma Medulloblastoma is a primitive neuroectodermal tumour and is highly malignant (early seeding into the central nervous system via subarachnoid metastatic spread). CT reveals a hyperdense mass (highly cellular), centred on the cerebellar vermis, often displacing or compressing the fourth ventricle. Secondary hydrocephalus (as in this case) leads to raised intracranial pressure or an increasing head size in younger children. MRI typically shows low T1 and mixed T2 signal due to mixed areas of cellularity and necrosis, and restricted diffusion. Intratumoural haemorrhage is rare. Ependymoma may be seen in the midline of the posterior fossa, and is associated with raised intracranial pressure. They are usually isodense on CT, more likely to have calcification and intratumoural haemorrhage and are far less common than medulloblastoma. Cerebellar pilocytic astrocytomas are common in childhood but more likely to cause visual disturbance. Meningioma is much less common in childhood unless associated with neurofibromatosis type 2. Acoustic neuromas are found at the cerebellopontine angle. Koeller KK, Rushing EJ. Medulloblastoma: a comprehensive review with radiologic–pathologic correlation 1. Radiographics 2003; 23: 1613–37. Chawla A, Emmanuel JV, Seow WT, et al. Paediatric PNET: pre-surgical MRI features. Clin Radiol 2007; 62: 43–52.
1086
On routine renal ultrasound following an uncomplicated urinary tract infection, a 7-year-old girl is seen to have multiple, bilateral, simple renal cortical cysts. Her mother reports that the girl has been falling over a lot and suffering frequent headaches. Her father is registered blind. What imaging would be most appropriate? A. No further imaging B. Follow-up abdominal ultrasound C. Contrast-enhanced CT brain D. Follow-up abdominal ultrasound and MRI brain and spine E. MRI brain
D. Follow-up abdominal ultrasound and MRI brain and spine The girl has von Hippel–Lindau disease, a multisystem neurocutaneous disease. Abdominal ultrasound should be performed to look for multiple organ cysts, in particular pancreatic cysts (diagnostic with a positive family history), liver haemangioma and phaechromocytoma. Presentation is frequently in later life, but this girl has cerebellar signs (ataxia) and multiple renal cysts that are frequently mistaken for polycystic kidney disease if they are an isolated finding. Regular ultrasound avoids radiation exposure and can detect other associated benign pancreatic and renal tumours. MRI is the imaging modality of choice in search of a diagnostic central nervous system haemangioblastoma, which can occur in the brain or spinal cord (cystic mass with an enhancing mural nodule). Retinal haemangioblastoma should be assessed by ophthalmology and are frequently multiple and bilateral. In any child with neurological signs and an abnormality of any organ, multisystem disease should be excluded. The likelihood of multiple neural abnormalities means that isolated imaging of the brain is not advised. Leung RS, Biswas SV, Duncan M, et al. Imaging features of von Hippel–Lindau disease. Radiographics 2008; 28: 65–79.
1087
A 1-day-old baby boy is rushed to the emergency department after a home birth with failure to feed, increasing tachypnoea and hypoxia. Chest X-ray shows cardiomegaly. The mother mentions a “cyst on the brain” that was seen antenatally. The clinicians suspect Vein of Galen malformation with associated high-output cardiac failure. What are the most likely imaging findings? A. A focal, fluid-filled well-defined cavity communicating with the left lateral ventricle with septations seen on transcranial ultrasound B. A well-defined cystic structure in the midline, posterior to the third ventricle and superior to the cerebellar vermis with rim calcification on unenhanced CT C. A well-defined cystic lesion at the right cerebello-pontine angle with signal characteristics identical to cerebrospinal fluid on all MRI sequences performed D. A well-defined oval, low-attenuation lesion in the pineal region, which is isointense on T1 weighted MRI, and high signal on T2 relative to cerebrospinal fluid E. Globally dilated ventricles with surrounding hypoechoic white matter
B. A well-defined cystic structure in the midline, posterior to the third ventricle and superior to the cerebellar vermis with rim calcification on unenhanced CT Vein of Galen malformation is a large central arteriovenous malformation, which causes dilatation of the draining Vein of Galen. It is often missed antenatally because it is frequently only demonstrable after 25 weeks gestation. In the neonate, the first 24 hours of life and delivery are usually unremarkable. Thereafter, high-output cardiac failure ensues as a result of the large shunt of oxygenated blood into the venous system. Diagnosis is often delayed by the search for a cardiac cause. Imaging includes transcranial ultrasound with Doppler, which demonstrates a high flow, midline cystic lesion associated with abnormal serpentine feeding and draining vessels. On unenhanced CT, rim calcification differentiates it from other causes of intracranial cystic masses. (A) describes a porencephalic cyst, (C) is typical of a simple arachnoid cyst, (D) describes a pineal cyst and (E), obstructive hydrocephalus with cerebrospinal fluid transudation. Jones BV, Ball WS, Tomsick TA, et al. Vein of Galen aneurysmal malformation. AJNR Am J Neuroradiol 2002; 23: 1717–24.
1088
A 3-year-old girl attends epilepsy clinic. She has a low intelligence quotient, frequent seizures and has a facial rash. On CT, “tram track” cortical calcifications are seen in the left parietal region, and the left parietal bone is thickened. Abdominal ultrasound is unremarkable. What is the most likely diagnosis? A. Sturge–Weber syndrome B. Tuberous sclerosis C. Klippel–Trenaunay syndrome D. Von Recklinghausen disease E. Wyburn–Mason syndrome
A. Sturge–Weber syndrome Sturge–Weber syndrome is a congenital syndrome characterized by a unilateral congenital port-wine stain in the distribution of the trigeminal nerve and cerebral angiomatosis. Leptomeningeal angiomas are confined to the pia mater (parietal lobes) and result in underlying gyriform cortical “tram track” calcifications. The overlying, ipsilateral skull and orbital bone are frequently thickened with underlying cortical atrophy and widening of the diploic space. If the port-wine stain affects V1, ipsilateral glaucoma, abnormal intraocular vessels and choroidal haemangiomas may be seen. Angiomas of the intra-abdominal and pelvic viscera may also be seen. Tuberous sclerosis is the main differential and is associated with seizures, low IQ and facial angiofibromas. Intracerebral cortical/subcortical tubers (hamartomas) are often calcified, but seen as more thickened, misshapen gyrae with central low attenuation. Kilppel–Trenaunay syndrome usually affects one limb in isolation and is diagnosed by a dermatomal port wine naevus of the affected limb. Von Recklinghausen disease (NF-1) is an autosomal dominant phakomatosis with axillary freckling, cafe-au-lait spots and cutaneous neurofibromas. Wyburn–Mason syndrome is associated with skin telangectasia and a vascular malformation of the ipsilateral optic tract. Vilanova JC, Barceló J, Smirniotopoulos JG, et al. Hemangioma from head to toe: MR imaging with pathologic correlation. Radiographics 2004; 24: 367–85.
1089
A 12-month-old boy has a plain anteroposterior skull radiograph following a simple fall at home. There is no concern for non-accidental injury. He is drowsy and vomiting. Multiple bones show prominent convolutional markings and he has an increased craniofacial ratio. The cranial sutures appear normal. His anterior fontanelle is bulging and his head circumference is in the 75th centile for his age. Which is the most appropriate next examination? A. Unenhanced CT brain B. Transcranial ultrasound C. MRI brain D. No further imaging E. Angiography
A. Unenhanced CT brain The appearance described is that of “copper (or silver, or brass) beaten” skull which can be seen in infants with raised intracranial pressure and can also be a normal variant. Common causes are craniosynostosis and obstructive hydrocephalus. In this context, concern is for hydrocephalus, as the head size is at the upper limit of normal and the fontanelle is bulging. There is also concern for intracranial injury following injury. Unenhanced CT brain will allow optimum assessment of the bones to exclude any bony injury secondary to the fall, exclude intracranial haemorrhage, allow assessment of the ventricular system, and exclude an intracranial mass. Although this skull appearance can be normal, there are clinical signs of a significant head injury and intracranial injury should be excluded with further imaging. Transcranial ultrasound could assess for hydrocephalus and intracranial haemorrhage and would avoid radiation exposure, but would not assess the skull bones. MRI brain would allow excellent assessment of the cranial contents, but not a good bony review. Tuite GF, Evanson J, Chong WK, et al. The beaten copper cranium: a correlation between intracranial pressure, cranial radiographs, and computed tomographic scans in children with craniosynostosis. J Neurosurg 1996; 39: 691–9.
1090
A 6-week-old child presents with persistent jaundice. An ultrasound shows a normal sized echogenic liver. The portal tracts are difficult to visualize, particularly peripherally. The gallbladder appears normal. The radiologist notices multiple small soft tissue nodules in the left upper quadrant. Given the most likely differential diagnosis, what is the most appropriate investigation and what is it likely to show? A. HIDA (hepatobiliary iminodiacetic acid) scan – good hepatic activity but no bowel activity at 6 or 24 hours B. HIDA scan – good hepatic activity, no bowel uptake at 6 hours (faint at 24 hours) C. Liver biopsy – reduced bile in bile canaliculi D. MRI abdomen – dilated cystic structure at porta with normal hepatic ducts E. CT scan – multiple lymph node masses throughout the chest and abdomen
A. HIDA (hepatobiliary iminodiacetic acid) scan – good hepatic activity but no bowel activity at 6 or 24 hours The most likely diagnosis is biliary atresia. The other main differential is neonatal hepatitis and the ultrasound features of these two conditions may be similar. Features that suggest biliary atresia include: * absent gallbladder, although 10–20% of patients will have a normal gallbladder * triangular cord sign – an echogenic tubular/triangular structure adjacent to the main portal vein that represents the common bile duct remnant (pathognomonic). Typically, both conditions present with persisting neonatal jaundice of more than 4 weeks. Biliary atresia is associated with other conditions, for example, polysplenia in 10%. If ultrasound is not diagnostic, the next step is to perform a HIDA (hepatobiliary iminodiacetic acid) scan. Biliary atresia is the most likely diagnosis if there is no bowel uptake after 24 hours. Treatment is surgical – the Kasai procedure (portoenterostomy) is often performed initially but ultimately liver transplant is required in most cases. A cystic structure at the porta communicating with normal ducts describes a choledochal cyst. Humphrey TM. US diagnosis. Radiology 2007; 244: 845–51.
1091
A 1-week-old boy born at term (and diagnosed with trisomy 21) presents to the paediatricians with constipation. Meconium was passed at birth and the child is breast-fed. Abdominal X-ray shows general gaseous distension of bowel and a barium enema is performed. It shows an abnormal corrugated appearance to the rectum with a normal sigmoid proximally, the remaining colon is dilated. What is the most likely diagnosis? A. Meconium ileus B. Immature left colon syndrome C. Hirschsprung disease D. Necrotizing enterocolitis E. Milk allergy
C. Hirschsprung disease Hirschsprung disease is characterized by an absence of parasympathetic nerve fibres in the muscular and submucosal layers of the colon. Bowel innervation develops cranio-caudally and the rectum is therefore the most commonly affected segment. It tends to affect boys, is rare in premature infants and usually presents within 6 weeks. Down’s syndrome is associated in 5%. Definitive diagnosis is achieved by suction rectal biopsy. Meconium ileus usually presents with microcolon and small bowel dilatation because the obstruction is usually at the level of the terminal ileum. It is nearly always secondary to cystic fibrosis. Immature left colon syndrome is a temporary phenomenon secondary to failure to pass meconium. Necrotizing enterocolitis tends to occur in premature infants and the clinical picture is usually that of sepsis with diarrhoea. Cow’s milk associated allergic colitis presents with constipation and rectal involvement, seen at contrast enema. Amiel J, Sproat-Emison E, Garcia-Barcelo M, et al. Hirschsprung disease, associated syndromes and genetics: a review. J Med Genet 2008; 45: 1–14.
1092
A 7-year-old boy presents with abdominal pain and irritability. On examination he is generally tender across the whole abdomen with a rash on his legs and a sore left knee. An abdominal ultrasound shows a mass in the right upper quadrant with concentric circles of hypo- and hyperechoic tissue. An ileocolic intussusception is diagnosed but pneumatic reduction fails and an emergency laparotomy is performed. What is the most likely underlying diagnosis? A. Meckel’s diverticulum B. Hypertrophic Peyer’s patches C. Lymphoma D. Henoch schonlein purpura E. Ileal duplication cyst
D. Henoch schonlein purpura Intussusception most commonly presents between 6 months and 2 years (peak incidence at 5–9 months). In these patients over 95% will be idiopathic. In older children presenting with similar symptoms, a lead point should be considered. This is also true if good quality pneumatic reduction fails. Henoch schonlein purpura is a common small vessel vasculitis seen in children. It may be precipitated by a preceding bacterial or viral infection, insect bites or stings and even certain drugs. It is most common between 3 and 10 years and its features include purpuric rash over the legs and extensor surfaces of the arms, haematuria, large joint arthralgia and intussusception. Meckel’s diverticulum is a recognised cause of intussusception but usually occurs at a younger age, and the rash does not fit. Hypertrophic Peyer’s patches are thought to be the cause of idiopathic intussusception. Lymphoma and ileal duplication cysts are recognised lead points in older children, but the description does not support either. Martinez-Frontanilla LA, Silverman L, Meagher DP. Intussusception in Henoch-Schonlein purpura: diagnosis with ultrasound. J Pediatr Surg 1988; 23: 375–6.
1093
A 6-month-old female is admitted by the paediatric service with progressive respiratory distress and multiple bruises. The child is failing to thrive and is below the 25th centile for age. On examination, there is right upper quadrant swelling. An ultrasound is performed that demonstrates a massively enlarged liver with a multilobulated complex hypoechoic mass and distal aortic narrowing. What is the most likely diagnosis? A. Mesenchymal hamartoma B. Choledochal cyst C. Hepatoblastoma D. Metastatic neuroblastoma E. Infantile haemangioendothelioma
E. Infantile haemangioendothelioma Infantile haemangioendothelioma is the commonest benign liver tumour in children under 6 months with girls being more frequently affected. They can cause extra-cardiac heart failure and platelet consumption (Kasabach–Merrit syndrome). Ultrasound features include a complex mass with an enlarged coeliac axis and distal aortic narrowing. CT characteristics mimic those of a liver haemangioma with centripetal enhancement. They tend to rapidly enlarge for 6 months before gradually involuting. Treatment is only necessary in symptomatic patients. Mesenchymal hamartoma are more common in males and present in slightly older infants. Choledochal cysts communicate with the intra-hepatic ducts. Hepatoblastoma is the most common primary malignant liver tumour in children (tends to occur in slightly older children). Presents with jaundice and precocious puberty secondary to sex hormone production. Neuroblastoma, a malignant tumour of the sympathetic chain, but does metastasize to liver in approximately 15% of cases. Woodward PJ, Sohaey R, Kennedy A, et al. From the archives of the AFIP: a comprehensive review of fetal tumors with pathologic correlation. Radiographics 2005; 25: 215–42.
1094
A radiographer phones you and asks you to review an abdominal radiograph for a 3-year-old male who has been sent in by the paediatric surgeon for the investigation of constipation and bouts of abdominal pain. The image show gas-filled loops of small bowel but no colonic gas. You notice prominent calcifications over the right upper quadrant and within the scrotum. You phone the radiographer back to give your diagnosis and explain your plan: A. Meconium peritonitis – probably incidental, but surgical review is required B. Small bowel perforation – urgent surgical assessment is required C. Gallstones – arrange for an abdominal ultrasound D. Small bowel atresia – contact the surgeons for urgent assessment E. Constipation – organize a water-soluble enema for further evaluation
A. Meconium peritonitis – probably incidental, but surgical review is required Meconium peritonitis occurs secondary to an antenatal bowel perforation with the spillage of meconium into the peritoneum. This results in (sterile) chemical peritonitis. In most cases, the perforation heals, the bowel returns to normal and the diagnosis is incidental (as in this case). However, in some cases, the peritonitis may result in small-bowel atresia or adhesions that result in small bowel obstruction with presentation soon after birth. Meconium commonly collects around the free edge of the liver. In boys, a patent processus vaginalis allows meconium accumulation in the scrotum and resultant calcification. There are no features described here point to a diagnosis of small bowel perforation. Gallstones are rare in children unless there is a pre-disposing condition and the diagnosis of gallstones does not explain the scrotal calcification. Small bowel atresia is very uncommon in a child of 2 years of age and no proximal small bowel obstruction is described. Constipation is suggested by the clinical presentation, but there are no radiographic features support the diagnosis. Berrocal T, Lamas M, Gutiérrez J, et al. Congenital anomalies of the small intestine, colon, and rectum. Radiographics 1999; 19: 1219–36.
1095
A 2-day-old boy with abnormal facies presents with bilious vomiting and a rapid clinical deterioration. An abdominal radiograph shows a massively dilated stomach and a further gas bubble immediately medial to it with a small amount of gas in the right iliac fossa. What is the most likely diagnosis? A. Duodenal atresia B. Annular pancreas C. Small bowel obstruction with perforation D. Midgut volvulus E. Hirschsprung’s disease
B. Annular pancreas Annular pancreas is a common cause of duodenal stenosis. It presents in a similar fashion to duodenal atresia and the characteristic double-bubble sign on abdominal radiograph can be seen. It is also associated with Down’s syndrome as is duodenal atresia (though less so). The diagnosis can be confirmed with endoscopic retrograde cholangiopancreatography/MR cholangiopancreatography. The key feature here is gas in the right iliac fossa which is most likely to represent gas within the bowel – this essentially excludes duodenal atresia since the duodenum must be patent to allow passage of gas. No gas is seen within the vast majority of the small bowel, so any obstruction would have to be very proximal. The gas in the right iliac fossa is in an unusual place to represent extraluminal gas secondary to perforation. Midgut volvulus may cause obstruction of the proximal small bowel, but the story here is of obstruction in the proximal duodenum. Hischsprung’s disease is a colonic pathology that results from aganglionosis of a portion of bowel (usually in the rectum) and results in large bowel obstruction and no passage of meconium. O’Donovan AN, Habra G, Somers S, et al. Diagnosis of Hirschsprung’s disease. AJR Am J Roentgenol 1996; 167: 517–20.
1096
A 5-year-old boy presents to his general practitioner with a urinary tract infection. He has never had a urinary infection before. Urine cultures are performed and reveal a proteus infection. The general practitioner treats him with a course of antibiotics and when the patient returns 2 days later, the symptoms have resolved. The general practitioner contacts you to discuss the further management with respect of imaging. What is your recommendation? A. Request a non-urgent ultrasound for 6 weeks time B. Suggest the patient attends for your afternoon ultrasound session C. No imaging is required D. Organize a micturating cystourethrogram for 6 weeks once the acute infection has resolved E. Arrange a DMSA scan
A. Request a non-urgent ultrasound for 6 weeks time Children over 3 years of age with a first episode of urinary infection, who respond to antibiotics within 48 hours, do not need any imaging. When the causative organism is not Escherichia coli an ultrasound is usually required during the acute infection. However, the child has responded to antibiotics and the ultrasound can be performed within 6 weeks. NICE guidelines—Urinary tract infection in children. Available from: http://www.nice.org.uk/CG54
1097
A 2-day-old male who was initially well after birth is brought back to hospital. There is evidence of respiratory distress while feeding as well as an elevated respiratory rate and a blue tinge around his lips. The paediatrician suspects congenital heart disease and an echocardiogram is performed. It demonstrates a small left ventricle and absent mitral valve. There is clinical improvement following prostaglandin administration. Given the likely diagnosis, which of the following is the most likely association? A. Intrauterine growth retardation B. Oligohydramnios C. Tricuspid atresia D. Aortic coarctation E. Twin to twin transfusion syndrome
D. Aortic coarctation Hypoplastic left heart syndrome is the most likely diagnosis. It is a severe congenital heart malformation with a very small left ventricle and severely stenotic, or completely atretic, mitral and aortic valves. Venous return to the left atrium must pass through an atrial septal defect to the right heart and then through a patent ductus arteriosus to the systemic circulation. Presentation is usually with cyanosis and respiratory distress as the ductus closes. Prostaglandin limits ductus closure and is used as a temporizing measure before surgical correction. Coarctation is present in 80% of patients. Oligohydramnios is associated with pulmonary hypoplasia but this would present at the time of delivery, not days later. Although tricuspid atresia may be associated with hypoplastic left heart syndrome, the baby was initially well suggesting normal right heart anatomy. Twin to twin transfusion does have an association with severe and lethal cardiomyopathy, but that is not the case here. Bardo DM, Frankel DG, Applegate KE, et al. Hypoplastic left heart syndrome. Radiographics 2001; 21: 705–17.
1098
A 10-year-old boy presents with apathy, hypotonia, irritability and diffuse body pain. He has frontal bossing and wrist swelling. A wrist radiograph reveals metaphyseal fraying of both the ulnar and radius with threadlike shadows into epiphyseal cartilage, osteopenia with visible coarsening of trabeculae, and a green stick fracture. What is the most likely diagnosis? A. Juvenile rheumatoid arthritis B. Rickets C. Blount’s syndrome D. Scurvy E. Metaphyseal dysostoses
B. Rickets Rickets is osteomalacia in children. It causes the epiphyses to become flared and irregular and as the long bones become progressively softer, they bend and may fracture. Juvenile rheumatoid arthritis is equivalent to rheumatoid arthritis in patients under 16. They present with joint pain, skin rash (50%), fever and lymphadenopathy. Radiological signs include wide metaphysis, ankylosis, periarticular soft tissue swelling, juxta-articular osteoporosis, articular erosions and joint destruction. In Blount’s syndrome, there is leg bowing, but normal biochemistry. Scurvy is a multisystemic condition caused by vitamin C deficiency. Radiographs show characteristic ground-glass osteopenia with cortical thinning. Metaphyseal dysostoses are a group of conditions that result from defective osteoid formation. The Schmid type presents with waddling gait, bowed legs, short stature and metaphyseal changes similar to rickets. However, biochemistry is normal and it only affects the lower limbs. Ozonoff MB. Metaphyseal dysostosis of Jansen. Radiology 1969; 93: 1047–50. doi: 10.1148/93.5.1047
1099
A 14-year-old girl presents with pain in her left knee that has been increasing in severity over 6 weeks. There is no history of trauma. A radiograph of her left knee reveals a lytic eccentric expansile lesion in the femoral metaphysis that spares the epiphysis. The cortex is thin and intact with no periosteal reaction. MRI reveals multiple fluid–fluid levels. What is the most suitable method of treatment? A. Radiation therapy B. Currettage with polymethylmethacrylate filling C. Chemotherapy D. Percutaneous radiofrequency ablation E. Preoperative selective arterial embolization and complete resection
E. Preoperative selective arterial embolization and complete resection The lesion described is an aneurysmal bone cyst, and the appearances are characteristic: metaphyseal expansile mass with fluid–fluid levels. Histologically, they represent intraosseous arteriovenous malformations that contain thin-walled cystic cavities filled with chronic blood products. Treatment depends on whether there is a pathological fracture through the lesion. If, as in this case, there is no fracture, aggressive curettage and bone grafting is performed. Selective arterial embolization performed 48 hours before surgery may reduce the amount of haemorrhage. If there is a fracture, non-operative fracture management is followed by curettage after healing. Radiation therapy can be used in inoperable aneurysmal bone cyst cases. It carries the risk of recurrence (39–63%). Curettage with polymethylmethacrylate filling is the treatment of choice for giant cell tumour. Percutaneous radiofrequency ablation is the treatment option for symptomatic osteoid osteomas. Kransdorf MJ, Sweet DE. Aneurysmal bone cyst: concept, controversy, clinical presentation, and imaging. AJR Am J Roentgenol 1995; 164: 573–80.
1100
A general practitioner refers a 6-year-old boy for a pelvic X-ray. The young boy has a 3-month history of left hip pain and limping. A pelvic radiograph shows the left femoral epiphysis is smaller than the asymptomatic right side and it appears sclerosed with slight widening of the left hip joint space. The articular cortex is intact. What is the most likely diagnosis? A. Septic arthritis B. Transient synovitis C. Legg–Calve–Perthes disease D. Juvenile rheumatoid arthritis E. Slipped upper femoral epiphysis
C. Legg–Calve–Perthes disease This is typical for Legg–Calve–Perthes disease with avascular necrosis of the head of femur and sclerosis because of sequestration and compression. Widening of the joint space is due to thickening of cartilage, failure of epiphyseal growth and presence of an effusion. Other radiographic features include bone demineralization and local atrophy. Slipped upper femoral epiphysis usually occurs in patients who are overweight and older than those with Perthes (mean age 13 years). Latterly, there is posteromedial displacement of the femoral head (the line of Klein fails to intersect the femoral head). Transient synovitis is a non-specific inflammatory reaction (may be post viral) but the pelvic radiograph is normal. In septic arthritis, radiologic imaging may be normal, but there is typically an acute presentation with additional systemic features of infection. Juvenile rheumatoid arthritis is a multisystem disease that presents with fever, rash and joint symptoms. Radiological signs are similar to rheumatoid arthritis. Weishaupt D, Exner GU, Hilfiker PR, et al. Dynamic MR imaging of the hip in Legg–Calvé–Perthes disease: comparison with arthrography. AJR Am J Roentgenol 2000; 174: 1635–7.
1101
In sonography of the normal neonatal hip, the normal α and β angles in a coronal view of a hip joint in a 6-week-old infant are as follows: A. α < 60°, β > 77° B. α > 60°, β < 77° C. α > 60°, β > 77° D. α < 60°, β < 77° E. α < 77°, β > 60°
B. α > 60°, β < 77° The normal angles of a mature hip on ultrasound in the coronal view are α > 60°, β < 77°. The α angle is the angle between straight lateral edge of ilium and bony acetabular margin. The β angle is the angle between straight lateral edge of ilium and fibrocartilaginous acetabulum. If the angles fall out with the normal range, they represent the following: * α < 60°, β > 77°: these are the opposite of normal and represent dysplastic hips. * α > 60°, β > 77°: the β angle of >77° indicates a decentred, subluxed hip. * α < 60°, β < 77°: the α angle of <60° indicates a deficient bony acetabulum. Wientroub S, Grill F. Ultrasonography in developmental dysplasia of the hip. J Bone Joint Surg Am 2000; 82-A: 1004–18.
1102
An 8-year-old boy presents with a limp and complains of joint pain and stiffness. On examination, he is noted to have a skin rash with fever and lymphadenopathy. Pelvic radiographs reveal a wide femoral metaphysis, periarticular soft tissue swelling, juxta-articular osteopenia and joint space narrowing. What is the most likely diagnosis? A. Metaphyseal dysostoses B. Blount’s syndrome C. Scurvy D. Juvenile rheumatoid arthritis E. Rickets
D. Juvenile rheumatoid arthritis Juvenile rheumatoid arthritis is equivalent to rheumatoid arthritis in patients who are less than 16 years old. It represents a non-infectious arthritis lasting 6 weeks to 3 months. Joint involvement (knee > finger > wrist > ankle > hip > C-spine) is classified as polyarticular where more than five joints are involved. When compared with rheumatoid arthritis in adults, there is earlier involvement of larger joints and a greater degree of ankylosis. In the hand, phalangeal periostitis and carpal crowding due to ankylosis can be seen. Many patients have systemic features including skin rash (50%), fever and lymphadenopathy. Metaphyseal dysostosis (defective osteoid formation, short stature and waddling gait) and Blount’s syndrome (dyschondrosis of the medial physis of the proximal tibia) both result in bowed legs but have normal biochemistry and no systemic symptoms. Scurvy results in multisystem dysfunction because of a lack of vitamin C with ground-glass osteopenia with cortical thinning. Patients often present with haemorrhage. Rickets is osteomalacia in children and typically causes flaring of the metaphyses. Reed MH, Wilmot DM. The radiology of juvenile rheumatoid arthritis. A review of the English language literature. J Rheumatol Suppl 1991; 31: 2–22.
1103
The surgical registrar is concerned about a child with symptoms and signs of intussusception. They are pale, tachycardiac and peripherally shutdown. What is the first line of investigation for this child? A. Diagnostic air enema B. Abdominal ultrasound C. Chest radiograph D. Abdominal radiograph E. CT abdomen
B. Abdominal ultrasound The suspected diagnosis here is intussusception, and as such, ultrasound examination is always going to be the investigation of choice. Ultrasound examination allows confirmation of the diagnosis and allows assessment of the intussusception. Primarily, this assessment confirms that the intussusception is ileocolic, that is, that ileum has been drawn into the caecum and up the ascending colon. Secondary assessment can be made of the vascularity of the intussusceptum (central component) and whether there is fluid between the intussusceptum and intussuscipitens (the outer segment of bowel). In the patient where intussusception is not confirmed, an alternative diagnosis can be actively sought. Diagnostic air enema and CT could all be used to reach the diagnosis, but they involve the use of ionizing radiation and are not the first-line investigation of choice. Abdominal radiographs may demonstrate intussusception but they should not be used as a first-line investigation. A chest radiograph would be unlikely to help here and should not be performed routinely. del-Pozo G, Albillos J, Tejedor D, et al. Intussusception in children: current concepts in diagnosis and enema reduction. Radiographics 1999; 19: 299–319.
1104
A 5-year-old boy has recurrent urinary tract infection and a mass in the right flank on physical examination. At ultrasound, the right kidney is large and bilobed with two renal pelves. There is no kidney on the left. Both ureteral orifices are in normal position in the trigone of the urinary bladder and ureteric jets are visualized bilaterally. These abnormal findings represent: A. Renal fusion B. Longitudinal renal ectopia C. Renal malrotation D. Horseshoe kidney E. Crossed fused renal ectopia
E. Crossed fused renal ectopia Crossed renal ectopia refers to a single renal mass with two collecting systems, located on one side of the abdomen. It occurs because of improper renal ascent during embryogenesis. Formation of the perirenal fascial planes occurs during normal ascent of the kidneys. Ectopia (or renal agenesis) causes failure of fascial plane development and without the restraining fascia, bowel may be malpositioned in the empty renal fossa. Renal fusion refers fusion of kidneys, for example, horseshoe kidney, discoid/pancake kidney. There is association with obstruction, stone formation and vesicoureteric reflux. Longitudinal renal ectopia describes the location of a kidney that has failed to ascend normally. They are usually located within the pelvis and do not cross the midline. In renal malrotation, the kidney can sit in any axis with the collecting structures positioned ventrally (most common), laterally or dorsally. Associated with abnormal looking calyces due to developmental ectasia. Dyer RB, Chen MY, Zagoria RJ. Classic signs in uroradiology. Radiographics 2004; 24: S247–80. doi: 10.1148/rg.24si045509
1105
A 5-month-old male with a patent ductus arteriosus presents with progressive respiratory distress and cyanosis. The chest radiograph reveals a hyperlucent expanded left upper lobe with compression of adjacent lung tissue and contralateral mediastinal shift. What is the most likely diagnosis? A. Congenital lobar emphysema B. Cystic fibrosis C. Congenital lymphangiectasia D. Congenital pulmonary venolobar syndrome E. Cystic pulmonary airway malformation
A. Congenital lobar emphysema Congenital lobar emphysema causes progressive overdistension of the left upper lobe (~40%), right middle lobe (~30%), right upper lobe (20%) or multiple lobes (5%). Causes include bronchial compression (patent ductus arteriosus, aberrant left pulmonary artery, pulmonary artery dilatation), endobronchial obstruction and bronchial cartilage deficiency. Cystic fibrosis typically causes cystic bronchiectasis, atelactasis with right upper lobe predominance and hilar adenopathy. Congenital lymphangiectasia is characterized by anomalous dilatation of pulmonary lymph vessels and usually manifests at birth. Congenital pulmonary venolobar syndrome is a form of lung hypoplasia affecting one or more lobes. Cystic pulmonary airway malformation was previously known as congenital cystic adenomatoid malformation. It is a congenital cystic abnormality of the lung characterized by an intralobar mass of disorganized pulmonary tissue. Chest X-ray shows expansile mass with well-defined margins and multiple air or fluid-filled cysts. Berrocal T, Madrid C, Novo S, et al. Congenital anomalies of the tracheobronchial tree, lung, and mediastinum: embryology, radiology, and pathology. Radiographics 2004; 24: e17. doi: 10.1148/rg.e17
1106
Routine prenatal ultrasound is performed during the normal pregnancy of a 32-year-old female. It is her first pregnancy. The ultrasound reveals abnormalities with the foetal right-sided chest cavity which contains echogenic material. The sonographer also suspects a neural tube defect and cardiac defect. What is the most likely diagnosis? A. Morgagni hernia B. Bochdalek hernia C. Congenital cystic adenomatoid malformation type 3 D. Pulmonary sequestration E. Teratoma
B. Bochdalek hernia Prenatal imaging demonstrates echogenic material within the chest, for example, fluid within a cystic structure or bowel (as in this case). 90% of congenital herniae are of the Bochdalek type and tend to be big, occur at the back of the diaphragm and tend to have a poorer (bad) prognosis. Early neonatal films may reveal multiple gas-filled loops of bowel in the chest with fewer loops within the abdomen. When large, solid organs such as the liver and spleen may herniate. Bochdalek herniae are associated with neural tube defects, malrotation and cardiac abnormalities. Morgagni hernias are rarer (3–5%) occurring in the middle of the diaphragm. They tend to be smaller and are more common. Congenital cystic adenomatoid malformation, pulmonary sequestration and teratoma can all be included in the differential diagnosis for diaphragmatic hernia, but the associated abnormalities suggest an alternative diagnosis here. Leung JW, Coakley FV, Hricak H, et al. Prenatal MR imaging of congenital diaphragmatic hernia. AJR Am J Roentgenol 2000; 174: 1607–12.
1107
A 6-year-old child falls over in the playground at school. He complains of left elbow pain and is unable to extend his elbow. An elbow radiograph is obtained. The radial head and capitellum ossification centres are present and there is ossification in the expected location of the trochlea. The anterior humeral line bisects the capitellum, as does a line drawn down the radial neck. What is the diagnosis? A. Radial head dislocation B. Supracondylar fracture C. Lateral condylar fracture D. Medial epicondyle avulsion E. Lateral epicondyle avulsion
D. Medial epicondyle avulsion When assessing the paediatric elbow, alignment and ossification is key to accurate diagnosis. The anterior humeral line should intersect the middle third of the capitellum (which it does) – posterior displacement of the capitellum signifies a supracondylar fracture although minimally displaced fractures may have a normally located anterior humeral line. A line drawn down the radial neck should intersect the capitellum which suggests that the radial head is correctly located. Ossification progresses in order: capitellum, radial head, internal epicondyle, trochlea, olecranon, external epicondyle and can be remembered using the mnemonic (CRITOE). The ossification centres should appear in order and in particular, if there is ossification in the region of the trochlea, the internal epicondyle should be seen. In this case, the ossification in the region of the trochlea is the displaced internal epicondyle following avulsion. This needs to be highlighted so that it can be fixed operatively. Cheng JC, Wing-Man K, Shen WY, et al. A new look at the sequential development of elbow-ossification centers in children. J Pediatr Orthop 1998; 18: 161–7.
1108
An 8-year-old boy is seen in the paediatric outpatient clinic with symptoms of wheezing and coughing that are worsened on lying flat. He also describes generalized tiredness that has developed over the last year. On examination, he is overweight with a central abdominal fat distribution and striae. Pigmentation is seen around his mouth and a number of soft tissue masses are seen in the skin over his thighs. Auscultation reveals a pansystolic murmur with radiation to the axilla and a systolic “plop” heart sound. What is the most likely diagnosis? A. Peutz–Jeghers syndrome B. Carney complex C. Holt–Oram syndrome D. Atrial myxoma E. Cushing’s syndrome
B. Carney complex The wheeze, cough and added heart sound are characteristic of a cardiac myxoma. Symptoms are typically due to the effect of the mass of the tumour obstructing normal flow of blood. Pedunculated myxomas may cause symptoms when the patient is in a particular position. The Carney complex is an autosomal dominant syndrome that accounts for 7% of all cardiac myxomas. Carney complex findings include cardiac myxomas, cutaneous myxomas, spotty pigmentation of the skin, endocrinopathy, and both endocrine and nonendocrine tumours. Peutz–Jeghers syndrome and Cushing’s syndrome may cause pigmentation alteration, but they do not account for the characteristic added heart sound. Holt–Oram syndrome is a condition that causes congenital deformity of the upper limbs and heart and does not fit the clinical picture described here. Radin R, Kempf RA. Carney complex: report of three cases. Radiology 1995; 196: 383–6.
1109
A 3-week-old child is brought into the accident and emergency department (A&E) in extremis. The child is in shock and passing redcurrant jelly stools. For the past 2 days she has also been profusely vomiting green liquid. After appropriate stabilization of the patient, the paediatrician phones you to request an urgent ultrasound in A&E. What do you do? A. Go to A&E and perform the ultrasound B. Organize an urgent lower gastrointestinal contrast study C. Get the patient to the department for consideration of intussusception reduction D. Arrange an urgent CT abdomen E. Organize an urgent upper gastrointestinal contrast study
E. Organize an urgent upper gastrointestinal contrast study The key diagnosis to exclude in this patient is midgut volvulus secondary to malrotation. The child’s age, the duration of onset and bilious vomiting are all in favour of this. The passing of jelly-like red stools suggests vascular compromise (not the diagnosis of intussusception) and necessitates urgency of investigation. An upper gastrointestinal study allows accurate depiction of the position of the duodenojejunal flexure which is the key diagnostic feature. Going to the accident and emergency department to perform the ultrasound allows you to look for the relationship of the superior mesenteric artery and superior mesenteric vein, but accuracy is reduced on a portable study, and the vessel relationship may be normal despite volvulus. At CT, the whirlpool sign is a classic feature of volvulus and the vascular relationship is readily demonstrated. However, in a young child, the high radiation dose of CT precludes it as a first-line investigation. A lower gastrointestinal contrast study may demonstrate the position of the caecum, but its position is variable and cannot be relied upon for an accurate diagnosis of volvulus. Peterson CM, Anderson JS, Hara AK, et al. Volvulus of the gastrointestinal tract: appearances at multimodality imaging. Radiographics 2009; 29: 1281–93. doi: 10.1148/rg.295095011
1110
A 3-year-old child is taken to the accident and emergency department by his father after an injury while playing football. The ball hit him on the forehead and he immediately cried, complaining of neck and back pain. On examination, the child is small for age with relatively short limbs, but has normal intelligence. Midline neck tenderness with pain radiating to the arms and midline lumbar spine tenderness is present. C-spine X-ray shows decreased height of the odontoid peg with a thick C2 body and a slight narrowing of the cervical canal. L-spine X-ray shows slight increase in lordosis with anterior beaking of the lumbar vertebrae. What is the most likely diagnosis? A. Hurler syndrome B. Morquio syndrome C. Odontoid peg fracture D. Sickle cell disease E. Juvenile chronic arthritis
B. Morquio syndrome Morquio syndrome is type IV mucopolysaccharidosis, associated with normal intelligence and musculoskeletal manifestations after 18 months of age. Hypoplasia of the odontoid peg and anterior beaking of the vertebral bodies is present. Hurler syndrome is similar but associated with mental retardation. Odontoid peg fracture will not usually have lumbar spine beaking or C2 vertebral body changes, but needs to be carefully differentiated from odontoid hypoplasia. Sickle cell disease presents with bone pain and the lumbar vertebrae show a H-Shape pattern. Juvenile chronic arthritis presents with cervical spine apophyseal fusion in an older age group (5–10 years). Hendriksz CJ. Inborn errors of metabolism for the diagnostic radiologist. Pediatr Radiol 2009; 39: 211–20. doi: 10.1007/s00247-008-1072-x
1111
A 10-year-old child presents to the orthopaedic fracture clinic with back pain after a 2-day walking holiday with his scout troop. He initially presented to the accident and emergency department where the nurse practitioner could see no vertebral fracture, but described generalized increased bone density and “general bony abnormality”. The child is well, and there is generalized sclerosis of all bones including lumbar vertebrae which has a is a “sandwich appearance”. There is a history of several previous fractures and he wears a hearing aid. What is the unifying diagnosis? A. Osteopetrosis type II B. Idiopathic juvenile osteoporosis C. Autosomal recessive osteoporosis D. Rickets E. Secondary hyperparathyroidism
A. Osteopetrosis type II Osteopetrosis is a rare hereditary bone disorder associated with generalized bone sclerosis and loss of cortical and medullary differentiation. There are two forms, infantile (type I) and adult (type II; benign). The adult type is named so because those with the condition survive into adulthood, unlike the infantile type (who tend to die in childhood). Even though there is bony overgrowth, the bone that is formed is not as strong as normal bone and therefore is prone to fracture. Additionally, the bony overgrowth may cause compression of local structures including nerves, for example, cranial nerves (leading to deafness). Crowding of the marrow by bone thickening may also cause anaemia. In the infantile form thrombocytopaenia and recurrent infections are seen. Andersen PE, Bollerslev J. Heterogeneity of autosomal dominant osteopetrosis. Radiology 1987; 164: 223–5.
1112
A neonate is admitted to the paediatric cardiology ward with right heart failure and cyanosis. Pregnancy and delivery had been normal (40 + 5) and the only maternal history was depression for which she was taking medication. Chest X-ray reveals pulmonary oligaemia and an elevated apex. Echo confirms right atrial enlargement and tricuspid regurgitation with abnormal leaflet position and separation. A. Pulmonary stenosis B. Ebstein’s anomaly C. Tetralogy of Fallot D. Uhl anomaly E. Fatty infiltration of right ventricle
B. Ebstein’s anomaly Ebsteins anomaly is uncommon and characterized by abnormal tricuspid valve development. Presentation is often antenatally with development of hydrops foetalis and tachyarrhythmias although in less severe cases presentation may be in the neonatal period. Depending on the degree of atrial right to left shunting, the infant may or may not be cyanosed. Associations include maternal lithium carbonate ingestion (the reference to the depressed mother). The tricuspid valve is displaced downwards into the right ventricle resulting in atrialization of the superior portion of the ventricle. Findings depend on the severity of the abnormality and the position of the valve. There is often severe right-sided cardiomegaly due to an elongated and enlarged right atrium which may result in an elevated apex. Classically, the heart is described as having a “box shape” on a frontal chest radiograph. Pulmonary stenosis would cause right-sided chamber enlargement, but the tricuspid position should not be affected. Fatty infiltration of the right ventricle may cause right ventricular failure, but should not have structural defects. Uhl anomaly is an exceptionally rare defect with partial or total loss of right ventricular myocardial muscle. Tetralogy of Fallot includes defects of the right heart with RVH, but again, the tricuspid valve is involved here. Choi YH, Park JH, Choe YH, et al. MR imaging of Ebstein’s anomaly of the tricuspid valve. AJR Am J Roentgenol 1994; 163: 539–43.
1113
A 10-year-old boy is referred to the orthopaedic outpatient with a 3-month history of right knee pain. The pain is made worse by walking and relieved by rest and non-steroidal anti-inflammatory drugs. On examination, the knee is swollen but the range of movement is normal. Plain radiographs demonstrates a permeative, lytic metaphyseal lesion with a wide zone of transition and associated lamellated periosteal reaction. MRI demonstrates a bone tumour with large soft tissue component and central areas with fluid–fluid levels. What is the most likely diagnosis? A. Aneurysmal bone cyst B. Langerhanns cell histiocytosis C. Osteosarcoma D. Ewing’s sarcoma E. Chondrosarcoma
C. Osteosarcoma The lesion described has aggressive appearances: permeative, wide zone of transition, periosteal reaction and soft tissue mass. The description of fluid–fluid levels on MRI limits the differential here since Langerhans’ cell histiocytosis, Ewing’s sarcoma and chondrosarcoma do not have this appearance. The majority of osteosarcomas do not have fluid–fluid levels. However, telenagiectatic osteosarcomas (10% of all osteosarcomas) classically have fluid–fluid levels and would be the most likely diagnosis here. Typical appearances are of an aggressive lesion with a wide zone of transition, bone destruction, periosteal reaction (lamellar, sunburst and Codman’s triangle) and an associated soft tissue mass. Aneurysmal bone cysts do contain fluid–fluid levels, the radiographic features described are of an aggressive permeative lesion which would not fit with an aneurysmal bone cyst. Discepola F, Powell TI, Nahal A. Telangiectatic osteosarcoma: radiologic and pathologic findings. Radiographics 2009; 29: 380–3. doi:10.1148/rg.292085150
1114
A 5-year-old boy presents to the accident and emergency department after falling at home and smashing through a glass door. He sustains a laceration to his hand, as well as a scalp wound. Skull radiographs are performed to exclude a radiopaque foreign body. There is no visible foreign body, but incidental note is made of persistence of the metopic suture and the presence of wormian bones. Overall, the anterior skull and mandible are large but the paranasal sinuses are small. What is the most likely underlying diagnosis? A. Cleidocranial dysostosis B. Craniosynostosis C. Turricephaly D. Non-accidental injury E. Gardner syndrome
A. Cleidocranial dysostosis Cleidocranial dysostosis is an autosomal dominant condition in which there is delayed ossification of midline structures. Skull features include large head, widened fontanelles, wormian bones, persistent metopic sutures and hypoplasia of nasal sinuses. Hypoplasia or absence of the clavicle may also be present. Turricephaly is usually due to premature fusion of the coronal or lambdoid sutures. Craniosynostosis describes premature fusion of the skull bones and usually causes abnormality of skull shape. Non-accidental injury should be considered but is dependent on the results from clinical examination and any other imaging that has been performed. Gardner syndrome results in skull osteomas and gastrointestinal polyps. Eisen D. Cleidocranial dysostosis with a report of four cases. Radiology 1953; 61: 21–31. doi: 10.1148/61.1.21
1115
A young child with a history of learning difficulties is being reviewed in clinic. He has a facial abnormality with pulsating proptosis as well as a thoracolumbar scoliosis. Additionally, there are multiple small (~1 cm) non-mobile subcutaneous soft tissue masses and several brown lesions on the iris of the right eye during ocular examination. CT head confirms the cause of proptosis with a cranial bony abnormality with partial protrusion of the temporal lobe and associated dura through the defect. What is the diagnosis? A. Sturge–Weber syndrome B. Neurofibromatosis type 1 C. Fibrous dysplasia D. Gorlin–Goltz syndrome E. Juvenile nasopharyngeal angiofibroma
B. Neurofibromatosis type 1 Neurofibromatosis type 1 or Von Recklinghausen disease is a neurocutaneous disorder characterized by neurofibroma, intracranial hamartomata and other tumours. In this case, the patient has multiple cutaneous neurofibromas, Lisch nodues (iris hamatomas), scoliosis and sphenoid wing dysplasia. Sphenoid wing dysplasia can occur as an isolated finding or in patients who have neurofibromatosis type 1. It can cause pulsatile exopthalmos or enopthalmos. Presenting features include visual impairment, extra-ocular muscle impairment or inflammation of the conjunctiva. There is a defect in a part or all of the greater wing, elevation of the lesser wing and distortion of the sella. There may be elevation of the superior orbital fissure, enlargement of the temporal fossa in all directions and lifting of the sphenoid ridge out of the orbit. Sturge–Weber and Gorlin–Goltz syndrome are both neurocutaneous disorders, but the features here do not fit. Fibrous dysplasia and juvenile nasopharyngeal angiofibroma and can affect the base of skull with bony defects and proptosis. However, soft tissue masses and Lisch nodules point away from these as the diagnosis. Jacquemin C, Bosley TM, Liu D, et al. Reassessment of sphenoid dysplasia associated with neurofibromatosis type 1. AJNR Am J Neuroradiol 2002; 23: 644–8.
1116
A 12-year-old boy presents with headache and a developing visual field disturbance. On examination, there is a homonymous hemianopia. An MRI reveals a 2 cm irregular mass in the midline in the region of the sella. It returns high signal on T1 and T2 with signal dropout on T2* and enhancement following gadolinium. What is the most likely diagnosis? A. Central nervous system dermoid B. Rathke’s cleft cyst C. Intracranial teratoma D. Craniopharyngioma E. Pituitary adenoma
D. Craniopharyngioma The diagnosis here is craniopharyngioma is a type of relatively benign (WHO grade I) neoplasm which typically arises in the sellar/suprasellar region. They account for 1–5% primary brain tumours. They are derived from remnants of the craniopharyngeal duct and can occur anywhere along the infundibulum, from the floor of the third ventricle to the pituitary gland. There are two types: adamantinomatous (children) and papillary (adult), the former of which calcify in 90% of cases and cause signal dropout on gradient sequences (T2*). High T1 signal is secondary to haemorrhage or proteinaceous fluid. Presentation is with headache, raised intracranial pressure, visual disturbance (position at the optic chiasm) and hormonal upset. While 80% of adults with craniopharyngioma present with visual disturbance, only 20% of children do. Rathke’s cleft cyst may have similar imaging characteristics and location, but tend affect girls and not calcify. Pituitary adenomas tend to have a more heterogeneous signal characteristic on both T1 and T2. Moreover calcification is rare. Teratomas are rare in this location tend to have mixed signal because of numerous different tissue types in the mass. Calcification can occur. Eldevik OP, Blaivas M, Gabrielsen TO, et al. Craniopharyngioma: radiologic and histologic findings and recurrence. AJNR Am J Neuroradiol 1996; 17: 1427–39.
1117
A 4-year-old boy is found to have an anterior neck lump that measures 1.5 cm clinically and does not move with tongue protrusion. An ultrasound reveals a heterogeneous mass with internal echoes in the region of the thyroid cartilage, 2 cm from the midline. There is no internal colour on Doppler, although colour is returned from the wall of the mass. The child is otherwise well and blood tests are normal. What is the diagnosis? A. Ranula B. Second branchial cleft cyst C. Thyroglossal duct cyst D. Delphian node E. Epidermoid cyst
C. Thyroglossal duct cyst While not a typical case, this is a thyroglossal duct cyst. They are epithelial lined cysts that result from a failure of the normal developmental obliteration of the thryoglossal duct. The majority are midline and infrahyoid, and they are associated with ectopic thyroid tissue in 40% of cases. In some cases, the position is off midline (more common, the more inferior the cyst), but it is generally accepted that this should be no more than 2 cm. Unless infected, these are painless, fluctuant masses. Previous infection may result in debris and a more heterogeneous appearance. There may be associated inflammatory change in the area of the cyst following an episode of infection. A ranula is a rare, benign, acquired cystic lesion that occurs in the floor of the mouth that results from the obstruction of a sublingual gland. Second branchial cleft cysts tend to occur just below the angle of the mandible anterior to sternocleidomastoid. Delphian nodes are anterior midline nodes that are associated with laryngeal cancer and epidermoid cysts are extremely rare neck lesions that are most often located in a submental region. Ahuja AT, King AD, King W, et al. Thyroglossal duct cysts: sonographic appearances in adults. AJNR Am J Neuroradiol 1999; 20: 579–82.
1118
A 6-year-old boy presents with a mass in his abdomen and is subsequently diagnosed as having a Wilms’ tumour. At CT, there is hepatosplenomegaly and a number of skeletal abnormalities including localized gigantism. His heart appears abnormal on initial cross sectional imaging although a subsequent echocardiogram reveals a small ventricular septal defect. What is the underlying diagnosis? A. Beckwith–Wiedemann syndrome B. Drash syndrome C. Perlman syndrome D. WAGR syndrome E. Isolated association with hemihypertrophy
A. Beckwith–Wiedemann syndrome Wilms’ tumour is the most common paediatric renal mass and accounts for over 85% of cases of renal masses. It typically occurs in childhood and while most cases are sporadic, a number of associations with syndromes and other isolated abnormalities (including isolated hemihypertrophy) are recognised. The syndromes can be split into overgrowth and non-overgrowth syndromes. Beckwith–Wiedemann and Perlman syndromes are two overgrowth syndromes. WAGR (Wilms’, aniridia, genital abnormalities, mental retardation) and Drash syndromes are non-overgrowth syndromes. Beckwith–Wiedemann syndrome is a congenital overgrowth disorder that is characterized by a number of abnormalities that include, macroglosia, omphalocoele, localized gigantism, hemihypertrophy, cardiac abnormalities, nephromegaly and hepatosplenomegaly. The vignette clearly describes an overgrowth disorder. Perlman syndrome presents differently with polyhydramnios, foetal overgrowth, neonatal macrosomia, macrocephaly, dysmorphic facial features, visceromegaly and nephroblastomatosis with an increased incidence of Wilms’. O’Connor C, Levine D. Case 49: Beckwith–Wiedemann syndrome. Radiology 2002; 224: 375–8. doi: 10.1148/radiol.2242010266
1119
A newborn is found to have blueness around his lips and mouth at birth that worsens whenever he cries. Within 2 hours of delivery, there is central and peripheral cyanosis. The paediatrician describes a pansystolic murmur and an ECG shows left axis deviation. A chest X-ray shows a rounded contour to both the right and left heart borders, and the pulmonary arteries appear flattened. There is generalized decreased pulmonary vascularity. What is the most likely diagnosis? A. Tetralogy of Fallot B. Transposition of the great vessels C. Tricuspid atresia D. Eisenmenger syndrome E. Hypoplastic left heart syndrome
C. Tricuspid atresia Tricuspid atresia results in complete right-sided atrioventricular dissociation. An atrial septal defect must be present to shunt blood from the right to left atria. A subsequent ventricular septal defect allows blood to be pumped from the left ventricle into both the aorta and pulmonary trunk. The right ventricle is tiny in most cases since it has had no function in foetal life. Tetralogy of Fallot is the most common cyanotic heart disease and presents weeks to months after birth with progressive cyanosis. Transposition of the great vessels does present at birth, but there is usually right ventricular hypertrophy with right axis deviation on the ECG. Eisenmenger syndrome occurs when there is reversal of a left-to-right shunt secondary to raised pulmonary pressures and presents later. Hypoplastic left heart syndrome is reliant on a patent ductus to maintain circulation and therefore presents as the duct closes between 2 and 10 days following delivery. Fletcher BD, Jacobstein MD, Abramowsky CR, et al. Right atrioventricular valve atresia: anatomic evaluation with MR imaging. AJR Am J Roentgenol 1987; 148: 671–4.
1120
A 12-year-old girl presents with headaches and on examination has a left sixth nerve palsy and papilloedema. An MRI demonstrates a mass in the region of the third ventricle. It is largely cystic with an enhancing nodule in its wall but no calcification. The lateral ventricles are dilated with appearances of hydrocephalus. What is the diagnosis? A. Medulloblastoma B. Haemangioblastoma C. Ependymoma D. Pilocytic astrocytoma E. Ganglioglioma
D. Pilocytic astrocytoma A sixth cranial nerve palsy can be a false localizing sign and occurs in raised intracranial pressure, as does papilloedema. This young girl has hydrocephalus secondary to an obstructing tumour near the third ventricle. Pilocytic astrocytoma is a low grade astrocytoma found in young patients (75% under the age of 20). They tend to be midline, for example, optic chiasm, third ventricle or brainstem. There are a range of appearances, the commonest (~70%) being a large cystic mass with a brightly enhancing mural nodule with or without cyst-wall enhancement. They may also appear as mixed solid/cystic masses with central necrosis (~15%) or completely solid masses. About 20% calcify. Medulloblastoma typically occur in young children and arise in the midline, especially the vermis and roof of the fourth ventricle. Ependymoma will usually not have a large cystic component and tends to fill the fourth ventricle and protrude from its foramina. Haemangioblastoma usually occur in the adult population and are associated with von Hippel–Lindau disease. Gangliogliomas are uncommon central nervous system tumours with a variable appearance that occur in children and are associated with temporal lobe epilepsy. Koeller KK, Rushing EJ. Pilocytic astrocytoma: radiologic–pathologic correlation. Radiographics 2004; 24: 1693–708. doi: 10.1148/rg.246045146
1121
A neonate born to a teenage mother who had concealed the pregnancy develops respiratory distress in the first few hours of life. A chest radiograph reveals mediastinal shift to the right with an abnormal cystic lesion in the left hemithorax. An abdominal radiograph demonstrates a normal gas pattern. The mass is multicystic with fluid–fluid levels. The left hemi-diaphragm appears to be inverted. What is the diagnosis? A. Staphylococcal infection with abscess formation B. Congenital pulmonary airway malformation C. Bronchogenic cyst D. Congenital lobar emphysema E. Congenital diaphragmatic hernia
B. Congenital pulmonary airway malformation The nomenclature around congenital lung lesions is complex and there have been relatively recent changes in the pathological understand of congenital lung lesions. Congenital pulmonary airway malformation is an umbrella term given to a number of lung lesions including what were originally called congenital cystic adenomatoid malformation (CCAM). CCAM was originally divided into type I (small cysts), type II (larger cysts) and type III (solid with microscopic cysts). Moreover, there is association with sequestration and other lung malformations. Type III lesions are now known to be a result of lung hyperplasia and not a true cystic malformation. The lesion described here is a cystic lesion that expands the lung and is most in keeping with a type II CCAM/congenital pulmonary airway malformation. Diagnosis is usually either made on antenatal ultrasound, or in the neonatal period as a result of progressive respiratory distress. Chest radiographs demonstrate a multicystic lesion which, if large may cause mass effect with resultant mediastinal shift and depression (even inversion) of the diaphragm. In the neonatal period, they may be partially fluid filled. CT more accurately delineates the location and extent of the lesion. Infection and abscess formation seems unlikely given the clinical story and the absence of features of sepsis. Bronchogenic cysts are solitary. Congenital lobar emphysema refers to overinflation of a lobe. The main differential here is a diaphragmatic hernia, but the bowel pattern is described as normal and the diaphragm has been seen. Kao S, Zuppan C, Young L. Type 2 congenital cystic adenomatoid malformation (Type 2 congenital pulmonary airway malformation). Radiographics 2011; 31: 743–8.
1122
A pre-term infant is born at 32 + 6 to a diabetic mother. Within hours, he develops respiratory distress and is admitted to the neonatal unit. He is tachypnoic, but no pyrexial. Cyanosis and hypoxaemia worsen and he is intubated and ventilated. The chest radiograph reveals a large heart, narrow mediastinum and mild pulmonary plethora. What is the likely diagnosis? A. Respiratory distress syndrome B. Transient tachypnoea of the newborn C. Congenital heart disease D. Pulmonary haemorrhage E. Group B Streptococcus pneumonia
C. Congenital heart disease This is not respiratory distress syndrome, and while the child is preterm, he is cyanotic and tachypnoeic (a feature that completely excludes respiratory distress syndrome as the diagnosis). The diagnosis is transposition of the great vessels where the left- and right-sided circulations exist in parallel. Without a shunt, the condition is fatal. Some patients have a co-existing ventricular septal defect, while others rely on a patent ductus arteriosus. The neonate is unwell and hypoxic with a hyperdynamic precordium, tachycardia and tachypnoea. On the chest X-ray, there is moderate cardiomegaly and a narrow mediastinum resulting in bell-clapper or “egg-on-side” appearance. Pulmonary plethora is mild. Pulmonary haemorrhage does not fit the vignette and infection would be unlikely to cause such a severe physiological response in the absence of other signs. Transient tachypnoea of the newborn is a pulmonary oedema-type lung injury caused when the amniotic fluid usually expressed from the lungs during vaginal delivery builds up causing respiratory distress. It is commoner amongst caesarian section delivered babies. Ferguson EC, Krishnamurthy R, Oldham SA. Classic imaging signs of congenital cardiovascular abnormalities. Radiographics 2007; 2: 132–4. doi: 10.1148/rg.275065148
1123
A 5-year-old boy is found to have hypoplasia of the aorta between the brachiocephalic artery and the level of ductus arteriosus. The chest radiograph demonstrates a contour abnormality of the aorta and Roesler sign is noted unilaterally on the left. There are no features of cardiac failure. What is the diagnosis? A. Coarctation with associated ventricular septal defect B. Coarctation with an aberrant subclavian artery C. Coarctation with associated transposition D. Coarctation involving the left subclavian artery E. Coarctation with associated bicuspid aortic valve
B. Coarctation with an aberrant subclavian artery Roesler sign is the eponymous name given to inferior rib notching. The patient has coarctation of the aorta, described here as diffuse hypoplasia of the aorta, that is, it is not completely stenotic. The mediastinal contour abnormality is the figure of 3 sign. Rib notching occurs when the intercostal vessels enlarge as they shunt blood via the subclavian vessels to the aorta distal to the hypoplastic/stenotic segment. The unilaterality of the rib notching suggests that there is a second vascular anomaly; the side without rib notching must have reduced flow in the subclavian vessel. If there is no rib notching on the left, it is usually because the left subclavian artery is distal to the coarctation. In this case, there is no rib notching on the right. This is because of an abberrant right subclavian artery that branches from the aorta distal to the narrowed segment. There are many associations with coarcation of the aorta including congenital heart disease (bicuspid aortic valves in 80% of cases). An associated ventricular septal defect may be present, but would not cause unilateral rib notching and a transposition would have presented in infancy – the child is 5 years old. Goo HW, Park IS, Ko JK, et al. CT of congenital heart disease: normal anatomy and typical pathologic conditions. Radiographics 2003; 23: S147–65. doi: 10.1148/rg.23si035501
1124
A neonate is admitted to the neonatal intensive care unit immediately after delivery. While pregnancy had been completely normal, labour had been long and after failure to progress, the child was born by emergency caesarian section. At initial assessment, the child displayed signs of respiratory distress with nasal flaring. A subsequent chest radiograph reveals bilateral perihilar interstitial infiltrate with a small right-sided pleural effusion. The heart is normal in size. What is the diagnosis? A. Meconium aspiration syndrome B. Respiratory distress syndrome C. Neonatal pneumonia D. Congenital heart disease E. Transient tachypnoea of the newborn
E. Transient tachypnoea of the newborn Transient tachypnoea of the newborn also known as retained foetal fluid or wet lung disease presents as tachypnoea for the first few hours of life. It may last up to 1 day and resolves completely by 2 days. Amniotic fluid is expressed from the lungs during vaginal delivery and absorbed after birth. However, in babies born by caesarian section, there is reduced mechanical squeezing of the thorax and reduced clearance of fluid from the lungs. Radiographically, this appears similar to pulmonary oedema with interstitial fluid and small pleural effusions. Heart size is normal and radiological appearances will improve rapidly with supportive treatment. While the labour was long, there was no suggestion of meconium at delivery and in meconium aspiration, areas of patchy opacification are expected on the chest X-ray. Respiratory distress syndrome occurs secondary to a reduction in lung surfactant in premature babies. In most congeniial heart disease, the heart is enlarged and there is no suggestion that the cause of the neonate’s respiratory distress is cardiac. There is no suggestion of neonatal pneumonia. Warren JB, Anderson JM. Newborn respiratory disorders. Pediatr Res 2010; 31: 487–96. doi: 10.1542/pir.31-12-487
1125
A 6-month-old child has a chest radiograph after admission for presumed respiratory syncytial virus infection. The upper mediastinum is widened with increased opacification in the right upper zone. The trachea is not displaced and the hila are seen through the mass. The lung markings are normal. The edges of the mass are irregular in contour. No bony destruction is seen. What is the diagnosis? A. Lymphadenopathy B. Normal thymus C. Bronchogenic cyst D. Thymic hyperplasia E. Teratoma
B. Normal thymus The vignette describes a normal thymus gland. It is usually seen as triangular sail (thymic sail sign) frequently towards the right of the mediastinum. It has no mass effect on vascular or airway structures. The size can vary with inspiration. Classically, it has an undulating contour where it makes contact with the chest wall. Position and size are highly variable and it may even be retrocaval or cervical. Thymic hyperplasia is most commonly seen in younger children after the cessation of chemotherapy and occurs in the first 3-4 months after treatment is stopped. There are other causes of thymic hyperplasia, but the only clinical information given is that of presumed bronchiolitis. Bronchogenic cysts may be filled with simple or complex (proteinaceous or haemorrhagic) fluid. When large, they cause mass effect and are commonly seen within the middle mediastinum. Middle mediastinal masses are more likely to disrupt the normal anatomy of the bronchial tree. Teratomas are rare and would be likely to contain denser and more heterogeneous material. Nasseri F, Eftekhari F. Clinical and radiologic review of the normal and abnormal thymus: pearls and pitfalls. Radiographics 2010; 30: 413–28. doi: 10.1148/rg.302095131
1126
A 45-year-old male with a history of oligodendroglioma has been treated with radiation therapy. Post-radiotherapy the patient undergoes MRI with MR spectroscopy as part of his follow-up. On axial fluid attention inversion recovery images there is a heterogeneous mass with a cyst-like component anteriorly and hyperintense soft tissue component posteriorly. MR spectroscopy with the region of interest centred on the enhancing soft tissue component is evaluated. Which metabolite finding is most in keeping with tumour recurrence rather than radiation necrosis? A. Decreased N-acetylaspartate level B. Increased choline level C. Increased lactate level D. Increased creatine level E. Decreased lipid level
B. Increased choline level MR spectroscopy provides a measure of brain chemistry. The MR spectroscopy must be interpreted with the imaging characteristics. NAA is a neuronal marker which is present in normal tissue and is decreased in most cases of pathology. Creatine is a marker of energy metabolism. Lactate is a product of anaerobic glycolysis, while lipids are products of brain destruction. Choline is a cell membrane marker which is raised with tumour activity, the more hypercellular the tumour the higher the choline level. Generally, as malignancy increases, NAA and creatine decrease, and choline, lactate, and lipids increase. NAA decreases as abnormal tumour tissue displaces or destroys neurons. High-grade tumours (WHO grades III–IV) have high metabolic rates depleting creatine levels. Lipids are found in areas of necrosis, and lactate levels are raised when tumours outgrow their blood supply. Ross BD, Colletti P, Lin A. MR spectroscopy of the brain: neurospectroscopy. In: Edelman RR, Hesselink J, Zlatkin M, Crues J, eds. Clinical magnetic resonance imaging. 3rd edn. Philadelphia, PA: Saunders–Elsevier; 2006. pp. 1840–910.
1127
An 18-month-old male presents with vomiting, increased lethargy and increasing gait unsteadiness for 1 month. An MRI brain is performed which reveals a solid tumour in the left cerebellar hemisphere which extends into the left cerebellopontine angle. The mass is hypointense on T1 weighted and hyperintense compared with white matter on T2 weighted with surrounding vasogenic oedema, and demonstrates diffusion restriction. Post contrast administration there is intense enhancement. The imaging features are most typical for which of the following? A. Medulloblastoma B. Atypical teratoid/rhabdoid tumour C. Pilocytic astrocytoma D. Haemangioblastoma E. Meningioma
B. Atypical teratoid/rhabdoid tumour The MRI imaging characteristics are compatible with both medulloblastoma and atypical teratoid/rhabdoid tumour (AT/RT). The distinguishing feature is the presence of cerebellopontine angle involvement which is rare in medulloblastoma. In children particularly under the age of 3 years, AT/RT is an important consideration particularly as correct diagnosis of AT/RT is important since owing to its aggressive nature (WHO grade IV) intensive therapy rather than standard treatment for medulloblastoma could alter the natural course of the tumour. Pilocytic astrocytomas are a common cause of posterior fossa masses in children and adolescents however commonly they are multicystic with diffusion facilitation. Haemangioblastomas are the most common primary cerebellar tumour in the adult population, in up to 20% of cases they are multiple and occur as part of von Hippel–Lindau syndrome. Meningiomas are extra-axial tumour which are more common in adults. Ho DM, Hsu CY, Wong TT, et al. Atypical teratoid/rhabdoid tumor of the central nervous system: a comparative study with primitive neuroectodermal tumor/medulloblastoma. Acta Neuropathol 2000; 99: 482–8.
1128
A 15-year-old suffers from neck injury falling off a horse. A CT scan of the neck shows a 2 cm hyperdense soft tissue mass at the base of the tongue in the midline. Following contrast administration, the entire mass demonstrates prominent enhancement. This is most likely to represent: A. Thyroglossal cyst B. Branchial cleft cyst C. Lingual thyroid D. Haemangioma E. Dermoid cyst
C. Lingual thyroid A lingual thyroid is a type of ectopic thyroid, and results from lack of normal caudal migration of the thyroid gland from the foramen caecum down to its normal location anterior to the larynx and upper trachea. Thyroid tissue can be seen at any point along the course of the thyroglossal duct; however, complete arrest at the base of tongue is the most common site of ectopic thyroid. The condition is congenital and it is more common in females. Grainger RG, Allison DJ. Diagnostic Radiology. 5th edn. London, UK: Churchill Livingstone; 2007.
1129
A 40-year-old female presents with a 3-month history of gradually increasing proptosis of her left eye. An MRI scan reveals an intraconal well circumscribed, round mass that on T1 weighted is isointense compared with muscle and on T2 weighted is hyperintense to muscle. It shows a “capsule” of low intensity. Post contrast there is gradual irregular enhancement with delayed wash out. What is the most likely diagnosis? A. Orbital pseudotumour B. Graves’ disease C. Orbital venous varix D. Orbital lymphoma E. Orbital cavernous haemangioma
E. Orbital cavernous haemangioma Cavernous haemangiomas are the most common orbital vascular lesions in adults. They are well-circumscribed masses encircled within a fibrous pseudocapsule, and are without prominent arterial supply which accounts for the relatively slow enhancement. Areas of thrombosis within the mass are common owing to slow flow. Orbital lymphoma is solidly and homogeneously enhancing, demonstrates diffusion restriction and is of intermediate to high T2 signal. Grainger RG, Allison DJ. Diagnostic Radiology. 5th edn. London, UK: Churchill Livingstone; 2007.
1130
A 37-year-old male with no past medical history is brought to the accident and emergency department of a busy district general hospital in an acute confusional state. His partner states that he had a flu-like illness and has become increasingly confused in the last 5 days. Clinical examination did not show any neurological deficit and his Glasgow Coma Scale on admission was 13. On initial unenhanced CT of the brain, you see hypodense areas in the temporal cortex and insula without mass effect. Which of the following diagnostic tests are most likely to confirm the diagnosis? A. Lumbar puncture B. MRI of the brain C. Electroencephalgram D. CT brain with contrast E. Neurologist review
A. Lumbar puncture The initial non-contrast CT findings and clinical history are suggestive of herpes encephalitis, which is the most frequent cause of viral encephalitis. It is often fatal without treatment. It characteristically affects the limbic system. The next most important step is probably lumbar puncture rather than MRI as this will give the definitive answer; MRI is often also quite specific. Unenhanced CT scan can appear normal up to 3–5 days of onset. Thereafter low attenuation changes are demonstrated affecting the temporal lobes, insula and cingulate gyrus, most often bilaterally, but asymmetrically. MRI is sensitive and but demonstrates low signal on T1 weighted and high signal on T2 weighted images usually from the second day of onset. The T2 weighted high signal typically spares the putamen. Mortality is 30–70% but is reduced with early antiviral treatment. Danhert W. Radiology Review Manual. 6th edn. Philadelphia, PA: Lippincott Williams and Wilkins; 2007.
1131
The following is not a typical feature of a carotid body tumour on imaging: A. Splaying of the internal and external carotid arteries B. Low signal intensity on T2 weighted images C. Multiple flow voids D. Isointense signal intensity on T1 weighted images E. Malignant transformation occuring in less than 10%
B. Low signal intensity on T2 weighted images High signal intensity is seen on T2 weighted images. Weissleder R. Primer of Diagnostic Imaging. 5th edn. Maryland Heights, MO: Mosby; 2011.
1132
While reviewing an MRI of the brain you see bright signal on T1 weighted images in the basal ganglia. Which of the following is least likely to cause this appearance? A. Haemorrhage B. Wilson’s disease C. Long-term parenteral nutrition D. Neurofibromatosis-1 E. Parkinson’s disease
E. Parkinson’s disease Causes of bright signal on T1 weighted images involving basal ganglia include: * calcification * paramagnetic substances – haemorrhage, Wilson’s disease, haemorrhagic infarction, long term total parenteral nutrition (due to manganese deposition) * hamartomas of neurofibromatosis type 1 may be high on T1 and T2 weighted images and may also involve globus pallidus, cerebellum, internal capsule and brainstem * chronic liver disease with portocaval shunt. Chapman S, Nakielny R, eds. Aids to Radiological Differential Diagnosis. 5th edn. London, UK: Saunders Ltd; 2009.
1133
Which of the following combination of blood break down products and their signal appearances on MRI is incorrect? A. Oxyhaemoglobin – hypointense on T1 weighted; hyperintense on T2 weighted B. Extracellular methaemoglobin – hyperintense on T1 weighted; hyperintense on T2 weighted C. Haemosiderin – hypointense on T1 weighted; hypointense on T2 weighted D. Intracellular methaemoglobin – hypointense on T1 weighted; hyperintense on T2 weighted E. Deoxyhaemoglobin – isointense T1 weighted; hypointense on T2 weighted
D. Intracellular methaemoglobin – hypointense on T1 weighted; hyperintense on T2 weighted Signal characteristics are variable on MRI owing to different magnetic properties of blood and blood breakdown products. * <24 hours – oxyhaemoglobin is iso/hypointense on T1 and hyperintense on T2. * 1–3 days – deoxyhaemoglobin is iso/hypointense on T1 and hypointense on T2. * 3–7 days – intracellular methaemoglobin is hyperintense on T1 weighted and hypointense on T2. * 1–2 + weeks – extracellular methaemoglobin is hyperintense on both T1 and T2. Haemosiderin is iso/hypointense on T1 and hypointense on T2. Brant WE, Helms CA. Fundamentals of Diagnostic Radiology. Volume I. 3rd edn. Philadelphia, PA: Lippincott Williams and Wilkins; 2007.
1134
A 48-year-old female with a previous history of breast cancer is admitted with confusion and blurring of vision. Clinical examination showed a superior right quadrantonopia. Contrast-enhanced CT confirms the presence of a metastatic lesion. Where is the metastasis most likely to be present? A. Left parietal white matter B. Optic chiasm C. Right visual cortex D. Right anteroinferior temporal mass E. Left posteriorinferior temporal mass
E. Left posteriorinferior temporal mass Temporal lobe lesions cause contralateral superior quadrantanopia. Homonymous hemianopia is a characteristic field defect caused by lesions (vascular, neoplastic or uncal herniation) in the contralateral visual cortex. A person with stroke involving the right occipital lobe will have same field defect with macular sparing. Lesions of the optic chiasm cause bitemporal hemianopia. Optic nerve lesion causes ipsilateral anopsia. Grainger RG, Allison DJ. Diagnostic Radiology. 5th edn. London, UK: Churchill Livingstone; 2007.
1135
A 31-year-old male with HIV presents with lethargy, confusion and seizures. On examination, he has a Glasgow Coma Scale of 14/15 with no focal neurological deficit. CT demonstrates multiple small ring enhancing lesions in the basal ganglia and thalami. What is the most likely diagnosis? A. Progressive multifocal leukoencephalopathy B. Lymphoma C. Toxoplasmosis D. Tuberculosis E. Cytomegalovirus infection
C. Toxoplasmosis Toxoplasmosis is the most common cause of ring enhancing mass lesions in patients with AIDS. It may mimic lymphoma but multiple lesions are more suggestive of toxoplasmosis. It results from reactivation of latent infection by Toxoplasma gondii. Imaging shows multiple 1–4 cm lesions in the corticomedullary junction and basal ganglia. Single lesions are uncommon. The lesions typically show ring enhancement, surrounding oedema and mass effect. In the severely immunocompromised the peripheral enhancement may be minimal or absent. Central nervous system lymphoma can appear identical in immunocompromised patients and may coexist. Progressive multifocal leukoencephalopathy is a central nervous system demyelinating disorder caused by reactivation of latent infection by JC polyoma virus. Limb weakness is the commonest presentation. Progressive multifocal leukoencephalopathy lesions are most commonly seen in subcortical white matter, most typically in the parieto occipital regions. MRI show multifocal asymmetrical bilateral white matter lesions which are low signal on T1 and high signal on T2 without mass effect. In cases of cerebral meningeal tuberculosis there is often obliteration of the basal cisterns as the basal exudates are isodense with adjacent brain on CT. These exudates demonstrate diffuse contrast enhancement. Grainger RG, Allison DJ. Diagnostic Radiology. 5th edn. London, UK: Churchill Livingstone; 2007.
1136
A 28-year-old cyclist involved in a road traffic accident presents to accident and emergency department with Glasgow Coma Scale of 9/15. An urgent CT of the brain and C-spine showed a large lentiform lesion with mixed attenuation overlying the left frontoparietal convexity with surrounding oedema and effacement of the left lateral ventricles. The temporal horn of the right lateral ventricle is dilated. There is also subtle loss of grey–white matter differentiation in the ipsilateral frontal lobe. The features are most likely to represent: A. Acute subdural haematoma with subfalcine herniation B. Acute extradural haematoma with subfalcine herniation C. Acute extradural haematoma with uncal herniation D. Acute subdural haematoma with uncal herniation E. Acute extradural haematoma with transtentorial herniation
B. Acute extradural haematoma with subfalcine herniation Extradural haemorrhage is usually arterial caused by traumatic middle meningeal artery rupture. The haematoma is under arterial pressure but limited by the sutures giving rise to a biconvex lentiform lesion seen on CT. Subdural haemorrhage is venous in nature giving rise to a crescenteric biconcave collection over the cerebral hemispheres. The secondary effects of haemorrhage include herniation, cerebral oedema, ischaemia, coning and embolic complications. Herniation can be subfalcine, uncal or transtentorial. Subfalcine herniation causes midline shift giving rise to ischaemia in the distribution of the anterior cerebral artery, contralateral hydrocephalus with obstruction at the foramen of Munro. In uncal herniation, the third cranial nerve is compressed by the medial temporal lobe causing fixed dilated pupil. Transtentorial herniation causes brainstem distortion and posterior cerebral arterial territory ischaemia. Chapman S, Nakielny R, eds. Aids to Radiological Differential Diagnosis. 5th edn. London, UK: Saunders Ltd; 2009.
1137
An 80-year-old male underwent a CT brain as part of investigation for memory problems. Intracranial appearances were normal. However note was made of a rounded 3 cm soft tissue mass in the scalp which contained some speckled calcification and no evidence of underlying bone erosion. On MRI, the lesion was isointense to brain on T1 with heterogeneous signal on T2 due to internal calcification and no significant enhancement post contrast. Which of the following options best fits with the above description? A. Meningioma B. Trichilemmal cyst C. Haematoma D. Basal cell carcinoma E. Cortical tuber
B. Trichilemmal cyst Trichilemmal cysts are an occasional finding on routine CT imaging of the brain. They are slow growing superficial nodules and may be solitary or multiple. The imaging description above is said to be pathognomonic for these lesions. They do not have malignant potential. Gossner J, Larsen J. Incidental subcutaneous nodules on the scalp of patients undergoing CT of the brain; frequency of appearance and differential diagnosis. Clin Radiol 2010; 65: 427–9.
1138
A 30-year-old renal transplant patient presents with increasing confusion, headaches and right arm weakness. He had a grand mal seizure while in the emergency department. A CT of the brain showed some low-density lesions in the white matter of the right parieto-occipital region without mass effect. MRI of the brain demonstrates bilateral asymmetrical areas of white matter abnormality which extend all the way to the cortex, are hypointense on T1 weighted, hyperintense on T2 weighted and do not enhance post-gadolinium. The most likely diagnosis is: A. Metastasis B. Cortical tubers C. Tuberculosis D. Progressive multifocal leukoencephalopathy E. Meningitis
D. Progressive multifocal leukoencephalopathy Progressive multifocal leukoencephalopathy is a demyelinating disease which is commonly seen in immunosuppressed patients following reactivation of the JC virus. The hall mark of this disease is extensive demyelination in the absence of significant inflammatory response. Typical imaging findings are multifocal white matter lesions typically involving the arcuate U-fibres which may become more confluent with progressive disease. There is usually no mass effect. Shah R, Bag AK, Chapman PR, et al. Imaging findings of progressive multifocal leukoencephalopathy. Clin Radiol 2010; 65: 431–9.
1139
A 35-year-old HIV positive male presents with headaches and cognitive impairment which has been progressive over the last few weeks. MRI imaging of the brain was done. Which of the following features on imaging would favour a diagnosis of HIV encephalitis over progressive multifocal leukoencephalopathy? A. Multifocal lesions with asymmetric bilateral involvement of the white matter B. Lack of global brain atrophy C. Subcortical U fibre involvement D. Confluent symmetrical and bilateral white matter signal abnormality with associated global atrophy E. Mixed diffusion pattern within the abnormal white matter, with areas of reduced and areas of increased diffusion, compared to normal white matter
D. Confluent symmetrical and bilateral white matter signal abnormality with associated global atrophy HIV encephalitis on MRI would show bilaterally symmetrical white matter lesions with associated global cerebral atrophy and sparing of the subcortical U-fibres. The rest of the answers above favour progressive multifocal leukoencephalopathy. Shah R, Bag AK, Chapman PR, et al. Imaging findings of progressive multifocal leukoencephalopathy. Clin Radiol 2010; 65: 431–9.
1140
A 60-year-old male presents to the casualty with a 2-week history of mild cognitive impairment, memory problems and headaches. A CT head done on admission showed a 4 cm lesion in the right temporal lobe which demonstrated ring enhancement post contrast. An MRI of the brain was done which showed the lesion to be hyperintense on T2 and hypointense on T1 with ring enhancement post contrast. Diffusion-weighted imaging (DWI) was carried out. Which of the following patterns would favour an abscess rather than tumour? A. Hyperintensity on DWI and dark on apparent diffusion coefficient (ADC) map B. Hypointensity on DWI and dark on ADC map C. Hypointensity on DWI and bright on ADC map D. Hyperintensity on DWI and bright on ADC map E. Isointensity on DWI and intermediate on ADC map
A. Hyperintensity on DWI and dark on apparent diffusion coefficient (ADC) map The content of brain abscesses is usually hyperintense on diffusion-weighted imaging due to a combination of restricted diffusion and high T2 signal. The former is reflected in the low calculated apparent diffusion coefficient (ADC; dark on ADC map). Tumours usually have serous necrotic components and are hypointense to brain parenchyma on diffusion-weighted imaging and bright on the ADC map. Mortimer A, O’Leary S, Bradley M, et al. Pitfalls in the discrimination of cerebral abscesses from tumour using diffusion weighted imaging. Clin Radiol 2010; 65: 488–92.
1141
The following feature is not typical of Chiari II malformation: A. Enlarged tentorial incisura and dysplastic tentorium B. Small posterior fossa C. Beaked tectum D. Small foramen magnum E. Low-lying medulla and cerebellar vermis
D. Small foramen magnum A large foramen magnum is typical. Chapman S, Nakielny R, eds. Aids to Radiological Differential Diagnosis. 5th edn. London, UK: Saunders Ltd; 2009.
1142
A 6-month infant is brought to the neurology clinic with low birth weight and developmental delay. MRI brain was performed which showed periventricular and basal ganglia calcification, cerebral and cerebellar volume loss with reduced sulcation, ventriculomegaly and abnormal high signal white matter changes. The most likely diagnosis is: A. Encephalitis B. Tuberous sclerosis C. Sturge–Webber syndrome D. Congenital cytomegalovirus infection E. Meningitis
D. Congenital cytomegalovirus infection Congenital cytomegalovirus infection occurs through foetomaternal transmission in the intrapartum period. Neonates may present with low birth weight, microcephaly and hearing loss. Patients infected at a younger gestational age present with more severe disabilities and have a poorer outcome. Imaging findings include intracranial calcification, cerebral and cerebellar volume loss, ventriculomegaly, lisencephaly and abnormal high signal white matter changes. Fink KL, Thapa MM, Ishak GE, et al. Neuroimaging of paediatric central nervous system cytomegalovirus infection. Radiographics 2010; 30: 1779–96.
1143
A 30-year-old male was brought into the emergency room with a Glasgow Coma Scale of 3/15 following a suicide attempt involving carbon monoxide poisoning. A CT brain done in the emergency department was unremarkable and he went on to have an MRI brain. Which of the following signal characteristics is seen in acute carbon monoxide poisoning? A. T2 prolongation in the basal ganglia with restricted diffusion on diffusion-weighted imaging B. T1 prolongation and T2 shortening in the globus pallidus C. Decreased signal on proton density images D. Low signal in the basal ganglia on fluid attention inversion recovery E. None of the above
A. T2 prolongation in the basal ganglia with restricted diffusion on diffusion-weighted imaging Carbon monoxide affects the basal ganglia due to the high mitochondrial content of these structures. The globus pallidus is typically affected with high signal changes on T2 and restricted diffusion on diffusion-weighted imaging. Delayed leukoencephalopathy may also occur. Hegde A, Mohan S, Lath N, et al. Differential diagnosis for bilateral abnormalities of the basal ganglia and thalamus. Radiographics 2011; 31: 5–30.
1144
A 28-year-old presents with dysmetria, dysdiadochokinesia and a positive Romberg sign. MR head reveals a 2 cm cystic lesion in the cerebellum with an enhancing mural nodule. The following feature is not typical manifestation of the underlying disease: A. Pancreatic cysts B. Retinal haemangioblastoma C. Cystadenoma of the ovaries D. Renal cell carcinoma E. Pheochromocytoma
C. Cystadenoma of the ovaries The underlying multisystem disorder is von Hippel–Lindau which is characterized by the developmental of numerous benign and malignant tumours as well as several non-malignant lesions. Patients present with some of the following associated lesions: * pancreatic cysts (50–91%) * cerebellar haemangioblastoma (44–72%) * renal cysts (59–63%) * retinal haemangioblastoma (45–59%) * renal cell carcinoma (24–45%) * spinal cord haemangioblastoma (13–59%). Leung RS, Biswas SV, Duncan M, et al. Imaging features of von Hippel–Lindau disease. Radiographics 2008; 28: 65–79.
1145
A 60-year-old female presents to the accident and emergency department complaining of a 2-month history of frontal headache. An unenhanced CT head reveals no acute intra-parenchymal pathology. Incidental note is made of bilateral basal ganglia calcification. The following is not a pathological cause of basal ganglia calcification: A. Paget’s disease B. Hypothyroidism C. Fahr disease D. Carbon monoxide poisoning E. Chemotherapy
A. Paget’s disease Differentials for basal ganglia calcification include (normal aging is by far the most common): * Toxic – carbon monoxide, lead, radiation therapy, chemotherapy. * Infectious – TORCH (Toxoplasma gondii, others like HIV, rubella, cytomegalovirus and Herpes simplex), central nervous system tuberculosis, AIDS, neurocysticercosis, central nervous system toxoplasmosis. * Metabolic – hypothyroidism, hypoparathyroidism, pseudohypoparathyroidism, hyperparathyroidism. * Inherited – Fahr disease, mitochondrial disease, for example, mitochondrial encephalopathy, lactic acidosis and stroke-like episodes. Chapman S, Nakielny R, eds. Aids to Radiological Differential Diagnosis. 5th edn. London, UK: Saunders Ltd; 2009.
1146
A 68-year-old male presents with increasing unsteadiness over the past 6 months. MRI head demonstrates a well-defined low T1 weighted and high T2 weighted 4 cm mass in the right cerebellar lobe and right fronto-parietal lobe. Your working diagnosis is metastases from an occult primary. Which of the following sites is most likely to be the source of the primary? A. Breast B. Colon C. Lung D. Renal E. Head & neck
C. Lung The sources of primary tumour in brain metastases are approximately: 48% lung, 15% breast, 9% melanoma, 1% lymphoma (mainly non-Hodgkin), 3% gastrointestinal (3% colon and 2% pancreatic), 11% genitourinary (21% kidney, 46% testes, 5% cervix, 5% ovary), 10% osteosarcoma, 5% neuroblastoma, and 6% head and neck tumour Grainger RG, Allison DJ. Diagnostic Radiology. 5th edn. London, UK: Churchill Livingstone; 2007.
1147
A neonate presents to the neonatal intensive care unit in respiratory distress. She is found to be apyrexial with normal blood parameters. The lead clinician notices that the neonate becomes intermittently cyanosed, most markedly during feeding and on crying the respiratory distress is alleviated. What would be the procedure of choice for further evaluation of the underlying condition? A. MRI neck with gadolinium B. CT nasal passage C. Ultrasound neck D. Rhinography E. Clinical examination is sufficient
B. CT nasal passage Bilateral choanal atresia is depicted in the scenario above. It refers to a lack of formation of the choanal openings and this can be unilateral or bilateral. Clinical presentation is dependent on whether choanal atresia is uni- or bilateral, or if there has been failure to pass a nasogastric tube. Unilateral choanal atresias presents late and can be asymptomatic while bilateral atresias can present with neonatal respiratory distress and cyclical cyanosis (infants are obligate nose breathers). CT scanning is the radiographic procedure of choice in the evaluation of choanal atresia. It confirms the diagnosis (unilateral or bilateral) and also allows further evaluation. It can also delineate other abnormalities in the nasal cavity and nasopharynx. Rhinography is a procedure that involves the administration of radiopaque dye into the nasal cavity to demonstrate choanae blockage. Grainger RG, Allison DJ. Diagnostic Radiology. 5th edn. London, UK: Churchill Livingstone; 2007.
1148
A 44-year-old male presents with a hard lump over his right cheek. He does not give a history of pain, inflammation or discomfort during eating. He is anxious that he has a cancer as his mother died of laryngeal carcinoma. MRI of the neck reveals a well-circumscribed, homogeneous 3.5 cm mass in the superficial lobe of the right parotid lobe with low intensity on T1 weighted and high intensity on T2 weighted images. There is also a rim of decreased signal intensity on T2 weighted images and homogenous enhancement on the post-gadolinium sequences. The most likely diagnosis is: A. Warthin’s tumour B. Pleomorphic adenoma C. Intraductal papilloma D. Oncocytoma E. Parotid duct calculusB. Pleomorphic adenoma Pleomorphic adenoma is the most common benign salivary gland tumour, accounting for nearly 80% of benign tumours. On CT, they are smooth, well-marginated tumours, which appear homogenous and of higher attenuation value than that of the surrounding gland. On MRI, they are well-circumscribed, homogeneous masses with low intensity on T1 weighted and high intensity on T2 weighted images. They commonly have a rim of decreased signal intensity on T2 weighted images that represents the surrounding fibrous capsule. After gadolinium, the tumours enhance avidly and homogeneously (unlike Warthin’s tumours). Warthin’s tumours have mixed geographic high and low T1 signals, and oncocytomas are generally isointense to parotid on T2/short T1 inversion recovery imaging. Grainger RG, Allison DJ. Diagnostic Radiology. 5th edn. London, UK: Churchill Livingstone; 2007.
1149
A 67-year-old male was referred by general practitioner for a CT scan of the brain owing to poor memory, hyponatraemia and possible seizures. Initial CT of the brain was unremarkable. Owing to persistent symptoms, an MRI of the brain was arranged. You are asked to review the MRI and you see slight swelling and high signal intensity coronal fluid attention inversion recovery limited to both hippocampi and amygdala. Which of the following is unlikely to be useful in the patient’s management? A. Chest radiograph B. CT angiogram C. PET/CT scan D. Antivoltage-gated potassium channel complex antibodies E. Lumbar puncture
B. CT angiogram The case describes features consistent with a limbic encephalitis which occurs as both an autoimmune or paraneoplastic syndromes. When autoimmune, voltage-gated potassium channel complex antibodies are the most common cause. When paraneoplastic, patients often develop it before the diagnosis of the systemic malignancy is made. The primary is often small as it induces an immune response which hold the tumour in check, but also induces the paraneoplastic syndrome – so a PET/CT or presence of paraneoplastic antibodies helps confirm the diagnosis. Lumbar puncture is useful for excluding infectious causes of a limbic encephalitis. Lung cancer, particularly small cell lung cancer is the most common malignancy associated with neurologic paraneoplastic syndromes which could be quickly confirmed with a chest radiograph. There is no indication for a CT angiogram. Loener LA, Yousem DM. Case Review Series: Brain Imaging. Philadelphia, PA: Mosby Books; 2008.
1150
The following is not a typical feature of Sturge–Weber syndrome: A. Port wine stain typically follows the distribution of the ophthalmic division of the trigeminal nerve B. Hemispheric atrophy C. Enlarged choroid plexus glomus D. Diffuse visceral angiomas E. Dystrophic cortical calcification
D. Diffuse visceral angiomas The cortical venous angioma gradually calcifies through infancy. This results in tram track calcification on imaging studies. The associated vascular stasis and congestion results in cortical hypoxia and progressive atrophy. Brant WE, Helms CA. Fundamentals of Diagnostic Radiology. Volume I. 3rd edn. Philadelphia, PA: Lippincott Williams and Wilkins; 2007.
1151
A 60-year-old male presents with headaches and visual symptoms. CT head without contrast demonstrates a large suprasellar heterogeneous solid-cystic lesion with specks of calcification present within it. MRI shows heterogeneous avid enhancement following IV gadolinium. The most likely diagnosis is: A. Rathke cleft cyst B. Craniopharyngioma C. Pituitary macroadenoma D. Intracranial tertoma E. Meningioma
B. Craniopharyngioma Craniopharyngioma is a benign, extra-axial, slow-growing tumour that involves the sella and suprasellar space. On MRI, it appears as a cystic suprasellar mass with a solid enhancing component. Gradient-echo images may show susceptibility effects from calcified components. Cystic areas with high cholesterol content appear hyperintense on T1 weighted, T2 weighted and fluid-attenuated inversion recovery images with heterogeneous isointense to hypointense solid components. A Rathke cleft cyst can usually be differentiated from craniopharyngioma, as they are typically entirely cystic, rarely calcify and do not enhance. Meningiomas demonstrate the dural tails, which is absent with craniopharyngiomas. Macroadenomas rarely calcify, usually enhance homogeneously and often have a snowman appearance when large. Brant WE, Helms CA. Fundamentals of Diagnostic Radiology. Volume I. 3rd edn. Philadelphia, PA: Lippincott Williams and Wilkins; 2007.
1152
A 42-year-old male presents with a 6-month history of tinnitus. MRI with high-resolution heavily T2 weighted sequences demonstrates hyperintense masses in both cerebellopontine angles. Further MRI of the whole spine shows a well-marginated intensely enhancing lesion within the thoracic cord which displays isointense signal on T` weighted and hyperintense signal on T2 weighted images. There is a hint of a peripheral rim of decreased T2 weighted signal. The most likely diagnosis of the cord lesion is: A. Meningioma B. Astrocytoma C. Ependymoma D. Haemorrhage E. None of the above
C. Ependymoma Features described above are very suggestive of Neurofibromatosis type 2 with bilateral cerebellopontine angle CN VIII schwannomas. They are typically isointense to hypointense on T1 weighted and hyperintense on T2 weighted images. Following contrast administration, vestibular schwannomas enhance avidly. In larger tumours this is often heterogeneous owing to cystic degeneration. Spinal ependymomas found in Neurofibromatosis type 2 can occur in the upper cord and conus. They may be isointense or hypointense relative to the spinal cord on T1 weighted and always hyperintense on T1 weighted images. Evans GR, Sainio M, Baser ME. Neurofibramatosis type 2. J Med Genet 2000; 37: 897–904.
1153
A 30-year-old male was admitted after being found in a coma. Admission CT head was normal. The clinical diagnosis was carbon monoxide poisoning. MRI obtained 4 weeks post admission however showed bilateral abnormalities particularly on T2 weighted and fluid attention inversion recovery sequences. Which part of the brain is most likely to be involved? A. Globus pallidus B. Cerebellum C. Substantiaanigra D. Thalamus E. Corpusmcallosum
A. Globus pallidus Carbon monoxide poisoning typically causes abnormalities in the basal ganglia, with a predilection to affect the globus pallidus. Haemorrhagic necrosis of the putamen is seen in cyanide and methanol poisoning, and white matter oedema can be an additional feature of methanol poisoning. Hegde AN, Mohan S, Lath N, et al. Differential diagnosis for bilateral abnormalities of the basal ganglia and thalamus. Radiographics 2011; 31: 5–30.
1154
A 65-year-old female with previous history of schizophrenia develops tremor of her hands for which she is seen by a neurologist. Owing to atypical symptoms, he refers her for a dopamine transporter scan which shows symmetrical comma shaped uptake in the caudate nucleus/lentiform nuclei. The most likely diagnosis is: A. Parkinson’s disease B. Drug-induced Parkinsonism C. Multiple system atrophy D. Progressive supranuclear palsy E. Alzheimer’s disease
B. Drug-induced Parkinsonism Dopamine transporter (DAT) scan is useful in evaluating patients with suspected Parkinson’s disease when the clinical features are not typical and diagnosis is uncertain. Depletion of dopaminergic nigrostriatal neurons and dopamine transporters on the pre-synaptic dopaminergic neurons is seen in Parkinson’s disease and also conditions such as multiple system atrophy and progressive supranuclear palsy. In these conditions, DAT scan is abnormal and shows a dot or full stop sign due to lack of tracer uptake in the lentiform nuclei. DAT scan is normal in drug induced parkinsonism which is an important differential diagnosis as these drugs act by blocking the post-synaptic D2 receptors whereas the pre-synaptic Dopamine transporters are preserved. DAT Scan is also normal in essential tremor and Alzheimer’s disease. Biersack HJ, Freeman LM. Clinical Nuclear Medicine. 4th edn. Berlin, Germany; Springer-Verlag; 2007. pp. 415–20.
1155
A 24-year-old female presents with sudden onset severe headaches that are increasing in intensity over the past 24 hours. CT and MR imaging demonstrate bilateral areas of thalamic swelling and low attenuation on CT and increased signal on T2 weighted images on MR. There is effacement of the third ventricle resulting in hydrocephalus affecting only the lateral ventricles. These appearances are suspicious for the following underlying process: A. Acute arterial infarct B. Chronic arterial infarct C. Metastases with associated oedema D. Subarachnoid haemorrhage E. Venous oedema and infarction
E. Venous oedema and infarction Appearances are in keeping with acute venous infarction. Bilateral infarcts due to occlusion of the midline venous sinuses may result in above appearances. Thrombosis of the deep cerebral veins may involve the basal ganglia, thalami, midbrain and mesial temporal lobes in a relatively symmetrical fashion. Chapman S, Nakielny R. Aids to Radiological Differential Diagnosis. 4th edn. London, UK: Elsevier Ltd; 2007.
1156
A 62-year-old male presents with loss of consciousness and headache after an unwitnessed fall in his garden. CT head demonstrates a hyperdense focus in the right parietal lobe that shows minor contrast enhancement. MRI shows a central core of high intensity on T1 weighted and T2 weighted images with a surrounding rim of low signal on T2 weighted. No surrounding oedema is detected. The most likely diagnosis is: A. Metastasis B. Developmental venous anomaly C. Acute haemorrhage D. Cavernoma E. Arteriovenous malformation
D. Cavernom The peripheral rim of haemosiderin (very low signal on T2 weighted images) makes the diagnosis. MRI is the modality of choice demonstrating a characteristic “popcorn” or “berry” appearance with a rim of signal loss due to haemosiderin, which demonstrates prominent blooming on susceptibility weighted sequences. Developmental venous anomalies are associated with cavernomas, but appear as medusa head veins draining via a large anomalous trunk. Chapman S, Nakielny R. Aids to Radiological Differential Diagnosis. 4th edn. London, UK: Elsevier Ltd; 2007.
1157
The following is not a major frequent feature in tuberous sclerosis: A. Facial angiofibromas B. Cortical tubers C. Subependymal nodules D. Hypomelanotic macules E. Gingival fibromas
E. Gingival fibromas Tuberous sclerosis is a rare autosomal dominant neuro-cutaneous syndrome characterized by the presence of benign congenital tumours in multiple organs. The classical triad of epilepsy, mental retardation and adenoma sebaceum is uncommonly seen at clinical examination so radiological examinations can play an important role in the diagnosis of tuberous sclerosis and in treatment. Umeoka S, Koyama T, Miki Y, et al. Pictorial review of tuberous sclerosis in various organs. Radiographics 2008; 28: e32.
1158
A 4-year-old male is brought to the emergency department with symptoms of flu by his mother. When interpreting the chest radiograph, a radiology trainee notices that there is aplasia of the lateral clavicles presence of supranumerary ribs. Further radiographs show the presence of wormian bones and persistent metopic suture. The most likely diagnosis is: A. Alexander’s disease B. Cleidocranial dysplasia C. Canavan’s disease D. Brachycephaly E. Rubinstein–Taybi syndrome
B. Cleidocranial dysplasia Cleidocranial dysostosis is a skeletal dysplasia which predominately affects bones which have membranous ossification. It is inherited in an autosomal dominant pattern and is characterized by incomplete ossification of skeletal structures, particularly the clavicle. Clinical features include a large head, large fontenelles with delayed closure, supernumerary teeth and high arched palate. Plain films demonstrate multiple wormian bones, widened sutures, hypoplasia/aplasia of the clavicle, supranumerary ribs and hemivertebrae. Brant WE, Helms CA. Fundamentals of Diagnostic Radiology. Volume I. 3rd edn. Philadelphia, PA: Lippincott Williams and Wilkins; 2007.
1159
A 40-year-old female presents with altered consciousness, seizures and visual loss. CT head demonstrates multiple hyperattenuating calcific nodules throughout the cerebral parenchyma (involving the grey–white matter junction) with no significant oedema or enhancement. MRI demonstrates signal dropout on T2 weighted and T2* images. The most likely diagnosis is: A. Cerebral metastases B. Pyogenic cerebral abscesses C. Neurocysticercosis D. Tuberculoma E. Perivascular spaces
C. Neurocysticercosis Neurocysticercosis is a neurologic parasitic disease that results due to the encysted larva of the tapeworm Taenia solium and is one of the most important parasitic diseases of the human central nervous system. Imaging findings depends on the stage of the life cycle of T. solium at presentation; the number and location of parasites; and associated complications such as arteritis/infarction, oedema and degree of obstruction. The calcified nodular stage is the end-stage quiescent calcified cyst remnant. On CT this demonstrates calcified lesion as described above with no oedema or enhancement. On MR there is signal dropout on T2 and T2* sequences, some intrinsic high T1 signal may be present and long-term enhancement may be evident, and may predict ongoing seizures. Kimura-Hayama ET, Higuera JA, Corona-Cedillo R, et al. Neurocysticercosis: radiologic–pathologic correlation. Radiographics 2010; 30: 1705–19.
1160
A 35-year-old Barbadian male presents with a 3-month history of intermittent headaches and blurred vision and a nodular rash on his shins. On examination, he has mild right-sided weakness and papilloedema. MRI shows a sessile low T2 signal durally based mass in the left middle cranial fossa and free edge of the tentorium. Following contrast there is diffuse dural enhancement most marked in the region of the left middle cranial fossa dural mass. Which of the following is most likely? A. Tuberculosis B. Sarcoidosis C. Chronic subdural haematoma D. Disseminated malignancy E. Meningitis
B. Sarcoidosis Sarcoidosis is a multisystem disease of unknown cause. It is usually diagnosed in patients between 20 and 40 years of age. The disease is more common in women and in people of West African descent. The characteristic pathologic lesion is a non-caseating granuloma. MRI findings include solitary or multiple dural-based masses, pachymeningeal enhancement (nodular or diffuse) and leptomeningeal enhancement (nodular or diffuse). Tuberculosis and sarcoid can be very similar on imaging, but sole dural involvement is more likely to be due to sarcoid. Koyama T, Ueda H, Togashi K, et al. Radiologic manifestations of sarcoidosis in various organs. Radiographics 2004; 24: 87–104.
1161
A 15-year-old male presents with recurrent epistaxis. On examination, there is a large pale reddish blue mass seen in the posterior choanal space. CT imaging demonstrates a lobulated non-encapsulated soft tissue mass centred on the sphenopalatine foramen and bowing of the posterior wall of the maxillary antrum anteriorly. There is marked contrast enhancement. Which of the following next steps is contraindicated? A. Surgical resection B. Core biopsy C. Pre-operative embolization D. Irradiation E. None of the above
B. Core biopsy A juvenile nasopharyngeal angiofibroma) is a rare locally aggressive vascular tumour that occurs almost exclusively in males and usually in adolescence. These lesions are very vascular and so biopsy is contraindicated. Brant WE, Helms CA. Fundamentals of Diagnostic Radiology. Volume I. 3rd edn. Philadelphia, PA: Lippincott Williams and Wilkins; 2007.
1162
A 62-year-old male with a past medical history of uncontrolled hypertension presents with a single episode of a generalized tonic–clonic seizure, and 24-hour history of fluctuating headache and visual disturbance. On neurological examination, he was drowsy but had no focal neurological deficit. The fundoscopic examination was normal but his blood pressure was recorded at 210/95 mmHg. MRI demonstrated bilateral symmetric vasogenic oedema involving the subcortical white matter in the posterior temporal and posterior occipital lobes. The most likely diagnosis is: A. Posterior reversible encephalopathy syndrome B. Acute venous infarction C. Acute arterial infarction D. Microangiopathic changes due to chronic hypertension E. Acute venous thrombosis
A. Posterior reversible encephalopathy syndrome Posterior reversible encephalopathy syndrom) primarily affects the posterior occipital and parietal lobes. It is characterized by headaches, altered mental status, seizure, and visual loss. Posterior reversible encephalopathy syndrome occurs in severe hypertension, such as in acute renal injury or pre-eclampsia, as well as being associated with several immunosuppressant and cytotoxic medications. Hinchey J, Chaves C, Appignani B, et al. A reversible posterior leukoencephalopathy syndrome. N Engl J Med 1996; 334: 494–500.
1163
A mother brings her 1-year-old child into the accident and emergency department with a 1-week history of cold and headache. The examining junior doctor finds the head shape of the child unusual and requests a skull radiograph. The radiograph shows an elongated narrow “boat-shaped” skull. Which of the best terms describe the head shape? A. Plagiocephaly B. Scaphocephaly C. Brachycephaly D. Cloverleaf skull E. Trigonocephaly
B. Scaphocephaly Craniosynostosis involves premature closure of one or more of the cranial sutures. It can occur as an isolated primary abnormality, or as part of a more complex syndrome/systemic disease. The skull shape undergoes characteristic changes depending on the suture(s) that close early. * scaphocephaly/dolichocephaly – sagittal suture * plagiocephaly – unilateral coronal (harlequin orbit) and lambdoid sutures * brachycephaly – bilateral coronal sutures (short head) * cloverleaf skull – synostosis of multiple paired sutures produces “trilobular skull” (most severe) * trigonocephaly: metopic suture (triangular-shaped head). Chapman S, Nakielny R. Aids to Radiological Differential Diagnosis. 4th edn. London, UK: Elsevier Ltd; 2007.
1164
A 2-year-old boy presents with a 4-day history of redness, swelling and discharge from the right eye. His general practitioner had treated him with gentamicin eye drops but the redness and discharge persisted. He was noted to be apyrexial and although his right eye was oedematous on examination, there was no purulence. There was reduction in visual acuity with that eye and the right cornea was hazy and oedematous. Specialist examination showed poor vision far lateral and lower quadrants unilaterally and a progressive strabismus. CT imaging demonstrated a calcified right orbital mass in the medial aspect of the right globe. The most likely diagnosis is: A. Orbital cellulitis B. Choroidal osteoma C. Orbital pseudotumour D. Retinoblastoma E. Rhabdomyosarcoma
D. Retinoblastoma Retinoblastoma is the most common intraocular tumour of childhood. Leukocoria, which has been reported in 3–72% of patients, is the most common presenting symptom for retinoblastoma. Atypical presentations occur 2–10% of the time, primarily with inflammatory signs such as endophthalmitis; secondary glaucoma with a red, painful eye; and uveitis. Over 90% of tumours show calcification on CT which may be small, large, single or multiple. Intraocular calcification in children under the age of 3 years is highly suggestive of retinoblastoma. Abramson DH, Frank CM, Susman M, et al. Presenting signs of retinoblastoma. J Pediatr 1998; 132: 505–8.
1165
A 32-year-old female presents with rapid onset unilateral left orbital pain over 24-hour period. On presentation to the accident and emergency department the next day, she had developed proptosis and diplopia. CT imaging showed enhancing enlargement of the medial and inferior rectus muscles with involvement of the tendinous insertions as well. MRI demonstrated low signal on T1 weighted and T2 weighted of the affected muscle bellies. The most likely diagnosis is: A. Orbital lymphoma B. Orbital pseudotumour C. Orbital cellulitis D. Orbital sarcoidosis E. Thryroid opthalmopathy
B. Orbital pseudotumour Orbital pseudotumour is an idiopathic inflammatory condition that presents as an enhancing soft-tissue mass which may involve the extraocular muscles and tendinous insertions. Patients typically present with rapid onset usually unilateral (90% of cases) orbital pain, proptosis and diplopia. CT imaging demonstrates enlargement of the muscle belly of one (or more) ocular muscles with involvement of the tendinous insertion. Involvement of the tendinous insertion distinguishes it from thyroid ophthalmopathy in which the insertion point is spared. Lymphoma and sarcoid can have similar imaging appearances, but the onset is much slower. Chapman S, Nakielny R. Aids to Radiological Differential Diagnosis. 4th edn. London, UK: Elsevier Ltd; 2007.
1166
During a routine plain film reporting session on an orthopantogram, you notice a lucent lesion adjacent to the crown of an unerupted left lower crown tooth in the mandible. On further examination, this lesion is well-defined, unilocular and expansile. The most likely diagnosis is: A. Dentigerous cyst B. Periapical cyst C. Developmental cyst D. Aneurysmal bone cyst E. Ameloblastoma
A. Dentigerous cyst Dentigerous cysts are benign odontogenic cysts found adjacent to the crown of an unerupted tooth. Majority are located in the mandible (75%) and they tend to be painless and discovered during routine radiographic examination. They can also grow and displace adjacent teeth. Dentigerous cysts are well-defined pericoronal radiolucencies centred on an impacted or unerupted tooth. They have a thin regular sclerotic margin and expand the overlying cortex without cortical breach. They are typically unilocular, but can be multilocular. Chapman S, Nakielny R. Aids to Radiological Differential Diagnosis. 4th edn. London, UK: Elsevier Ltd; 2007.
1167
During a routine plain film reporting session on an orthopantogram, you notice a “floating” tooth. Which of the following diagnoses is associated with this finding? A. Ameloblastoma B. Aneurysmal bone cyst C. Langerhans cell histiocytosis D. Hyperthyroidism E. Cementoma
C. Langerhans cell histiocytosis The appearance of “floating teeth” demonstrates no obvious supporting bone for the teeth. Found in severe periodontal disease, Langerhans cell histiocytosis, hyperparathyroidism, metastases and multiple myeloma. Chapman S, Nakielny R. Aids to Radiological Differential Diagnosis. 4th edn. London, UK: Elsevier Ltd; 2007.
1168
A 7-year-old female presents with a lump in her neck that is tender and painful on swallowing. On ultrasound, there is a 3 cm anechoic lesion that contains debris in the midline of the infrahyoid neck. The most likely diagnosis is: A. Branchial cleft cyst B. Lingual cyst C. Thymic cyst D. Ranula E. Thyroglossal cyst
E. Thyroglossal cyst Thyroglossal cysts are the most common congenital neck cysts found in the midline position. They tend to be painless, round and midline unless they are infected. The latter can present with a red warm painful lump. On ultrasound, they are fluctuant masses which spread the strap muscles. The fluid is anechoic and can contain debris within it. Chapman S, Nakielny R. Aids to Radiological Differential Diagnosis. 4th edn. London, UK: Elsevier Ltd; 2007.
1169
A 55-year-old male presents with central abdominal pain. On examination, the clinician finds a pulsatile mass and suspects a leaking abdominal aortic aneurysm. When assessing an abdominal aortic aneurysm for endovascular repair which parameter makes endovascular aneurysm repair less suitable? A. Abdominal aortic aneurysm diameter: 7 cm B. Left common iliac artery diameter: 6 mm C. Cylindrical aneurysm neck of length: 17 mm D. Inferior renal artery to aortic bifurcation: 100 mm E. Proximal neck angulation: 40°
B. Left common iliac artery diameter: 6 mm There are a number of parameters to consider when assessing suitability for endovascular aneurysm repair. Of first importance is being able to get the device into place and to leave it in a secure position. In order for the device to be placed, the iliac vessels must be of reasonable diameter and not excessively tortuous: a commonly quoted minimum common iliac artery dimension is 8 mm. An abdominal aortic aneurysm diameter of 7 cm is suitable for endovascular aneurysm repair. A cylindrical aneurysm neck of length 17 mm would be suitable also: a conical neck will promote stent migration and typically the neck must be >15 mm in length to allow the proximal end of the stent to be safely deployed. While the distance from the inferior renal artery to the aortic bifurcation is important when sizing the device, it does not determine suitability for stenting. Excessive angulation of the neck of the proximal aorta (typically >60°) creates a number of problems, including device deployment and creating an adequate seal. It may also promote strut fractures. Matsumoto AH. What randomized controlled trials tell us about endovascular repair of abdominal aortic aneurysms. J Vasc Interv Radiol 2008; 19: S18–21. doi: 10.1016/j.jvir.2008.01.028
1170
A 49-year-old female with stable rheumatoid arthritis diagnosed 12 years previously and who is currently taking methotrexate, complains of worsening shortness of breath. Following assessment and initial investigations, a high-resolution CT is performed. There is a small left-sided pleural effusion and patchy areas of ground glass opacification in the lower zones with scattered reticulation and honeycombing peripherally. Traction bronchiecatsis was also visualized. What is the diagnosis? A. Pulmonary hypertensive change B. Bronchiolitis C. Methotrexate induced pulmonary toxicity D. Non-specific interstitial pneumonia E. Organising pneumonia
D. Non-specific interstitial pneumonia Four types of lung disease pattern occur in rheumatoid disease: non-specific interstitial pneumonia (NSIP), usual interstitial pneumonia (UIP), organizing pneumonia and bronchiolitis. NSIP is by far the commonest picture in rheumatoid arthritis and while it has a wide variety of features, high-resolution CT findings are similar to those in the vignette. The key differential here is UIP (which is not one of the possible answers). In UIP, there is ground glass change, reticulation and honeycombing. There is considerable overlap between the two disease patterns, but bronchiectasis is less likely. Honeycombing is commoner in UIP, but can occur in NSIP. In the bronchiolitis picture, there would be predominance of nodular change and bronchiectasis. In organizing pneumonia, there is airspace opacification and ground glass change, which tends to be in a subpleural distribution. Methotrexate-induced pulmonary toxicity occurs about 6 months after starting methotrexate and it would be unusual to change medications in stable disease. Tanaka N, Kim JS, Newell JD, et al. Rheumatoid arthritis-related lung diseases: CT findings. Radiology 2004; 232: 81–91. doi: 10.1148/radiol.2321030174
1171
A 76-year-old with a history of asbestos exposure is admitted with cardiac chest pain. A chest radiograph reveals a small right-sided pleural effusion and a mass in the periphery of the right lung. A CT confirms the effusion and the mass in the right lower lobe. There is pleural thickening and right lower lobe volume loss. The bronchovascular bundle is pulled towards the lesion in a curvilinear pattern. What is the diagnosis? A. Bronchial carcinoma B. Mucoid impaction C. Round pneumonia D. Mesothelioma E. Round atelectasis
E. Round atelectasis This patient has an area of round atelectasis, an unusual form of lung collapse and one of the commonest causes of psuedotumour. It is intimately related to the pleura and while there are a number of postulated theories of pathogenesis, most agree that pleural disease and irritation is involved. There is increased incidence in patients with previous asbestos exposure. It has been seen in ages between 20 and 90 years and predominates in men (80%). It is almost always asymptomatic and found when a chest radiograph is performed for another reason (in this case, an admission for cardiac chest pain). Characteristic features include a subpleural location, pleural effusion, associated volume loss and the comet tail sign (bronchopulmonary bundle pulled towards the mass in a curvilinear pattern). Clearly, bronchial carcinoma is in the differential, but the classical features here suggest round atelectasis. Mesothelioma is related to asbestosis exposure, but not described here. Round pneumonia is a disease of childhood and mucoid impaction would present with atelectasis distal to an area of bronchial obstruction. Partap VA. The comet tail sign. Radiology 1999; 213: 553–4.
1172
A neutropenic 47-year-old female is recovering from a bone marrow transplant. She develops a chest infection, which is unresponsive to standard therapy. A CT is performed that demonstrates widespread nodular change with surrounding ground glass change. A fungal infection is suspected. What is the diagnosis? A. Angioinvasive aspergillosis B. Aspergilloma C. Allergic bronchopulmonary aspergillosis D. Semi-invasive aspergillosis E. Airway-invasive aspergillosis
A. Angioinvasive aspergillosis This patient has angioinvasive aspergillosis, the most severe type of infection with Aspergillus organisms. It occurs in patients who are severely immunocompromised. Classically, there are nodules surrounded by a halo haemorrhage (often seen as ground glass change). Peripheral wedge shaped haemorrhagic areas of consolidation may also be seen. There are five patterns of disease in patients with Aspergillus infection: * Aspergilloma: a soft-tissue mass within a lung cavity that is separated from the cavity wall by an airspace (“air crescent” sign); there may be some wall thickening. * Allergic bronchopulmonary aspergillosis: mucoid impaction and bronchiectasis involving upper lobe segmental and subsegmental bronchi. * Semi-invasive aspergillosis: chronic symptoms and segmental areas of consolidation with or without cavitation or pleural thickening and with multiple nodules. * Airway-invasive aspergillosis: characterized by acute tracheobronchitis, bronchiolitis, and bronchopneumonia. * Angioinvasive aspergillosis: as above. Franquet T, Müller NL, Giménez A, et al. Spectrum of pulmonary aspergillosis: histologic, clinical, and radiologic findings. Radiographics 2001; 21: 825–37.
1173
A 56-year-old male presents with copious sputum production and a relative paucity of other symptoms. He is Asian and a non-smoker. His chest radiograph demonstrates a mass, peripherally in the left upper lobe. A CT is requested that shows the peripherally located mass with tiny internal lucencies and some surrounding ground glass change. What is the likely diagnosis? A. Eosinophilic pneumonia B. Obstructing bronchogenic carcinoma C. Pulmonary lymphoma D. Lepidic predominant adenocarcinoma E. Alveolar proteinosis
D. Lepidic predominant adenocarcinoma Lepidic predominant adenocarcinoma used to be called bronchoalvelolar carcinoma and is characterized by non-invasive lepidic growth along the walls of the airways. It is more common in Asian populations and non-smokers and copious mucous production may be a feature of the disease. There are three recognized radiographic patterns: (1) single mass, (2) consolidative and (3) multinodular. The single mass type is the commonest appearance and is characterized by a peripheral nodule that is solitary and well circumscribed. Cavitation is rare, but pseduocavitation (the apparent presence of tiny bubble-like lucencies within the mass) is recognized, as is a surrounding halo ground glass change (the fried egg sign). Peripheral bronchial carcinomas tend to be adenocarcinomas, but the radiographic features and sputum production are not typical. Bronchogenic associated lymphoid tissues are the commonest intrapulmonary lymphoma and tend to be solitary masses with or without tiny lucencies and ground glass change. However, they are far less common and the sputum production points towards lepidic predominant adenocarcinoma. Alveolar proteinosis and eosinophilic pneumonia tend to present with ground glass change. Lee KS, Kim Y, Han J, et al. Bronchioloalveolar carcinoma: clinical, histopathologic, and radiologic findings. Radiographics 17: 1345–57.
1174
A 23-year-old with severe combined immunodeficiency who is on prophylactic aerosolized antibiotics has a high-resolution CT as an outpatient. It demonstrates a bilateral process in the upper lobes with thickening of the interlobular septa and cystic lucent areas and ground glass change. What is the diagnosis? A. Pneumocystis jiroveci pneumonia B. Sarcoidosis C. Tuberculosis D. Mucinous lepidic predominant adenocarcinoma E. Lipoid pneumonia
A. Pneumocystis jiroveci pneumonia The high-resolution CT demonstrates thickening of the interlobular septa with interspersed ground glass change caused by a partial alveolar filling process. It is termed “crazy paving” and was initially described in patients with alveolar proteinosis. Most patients with pulmonary alveolar proteinosis do have a crazy paving appearance at CT. However, it is rare and much commoner causes exist, for example, bronchial pneumonia, adult respiratory distress syndrome, acute interstitial pneumonia. In this patient with known immunodeficiency, the most likely answer is Pneumocystis jiroveci pneumonia (the correct term for what used to be known as Pneumocystis carinii pneumonia, still termed PCP). Pneumocystis jiroveci pneumonia is an atypical infection that predominantly affects the immunocompromised with a CD4 count below 200. There is a predominant ground glass pattern in the perihilar regions. However, if the patient is on prophylatic aerosolized medication, the relatively poorly ventilated upper zones are affected. Reticular opacities and septal thickening are also common (causing the crazy paving appearance). Pneumatocoeles are seen in 30% and pleural effusions are rare. Lee CH. The crazy-paving sign. Radiology 2007; 243: 905–6. doi: 10.1148/radiol.2433041835
1175
A 42-year-old presents with shortness of breath. A chest radiograph reveals no consolidation, but a difference in the transradiancy of the hemithoracies is noted. A high-resolution CT shows predominantly left-sided thickening of the interstitium and a left-sided pleural effusion. The background architecture of the lung is normal. What is the likely diagnosis? A. Sarcoidosis B. Lymphangitic carcinomatosis C. Pulmonary oedema D. Radiation pneumonitis E. Lymphocytic interstitial pneumonitis
B. Lymphangitic carcinomatosis This female has lymphangitic carcinomatosis secondary to breast cancer (with a previous mastectomy). Lymphangitic carcinomatosis describes tumour spread through the lymphatics of the lung, most commonly secondary to adenocarcinoma, for example, breast (commonest), gastrointestinal, bronchogenic, prostate and ovarian. Clinical presentation is variable. High-resolution CT shows interlobular septal thickening which is nodular and irregular and manifests as tessellating polygons with changes seen extending towards the hilum giving a characteristic “dot in box” appearance. Other features include sub-pleural nodules and thickening on the interlobar fissures, pleural effusion and lymphadenopathy. Generally, overall lung and lobular architecture is preserved. Pulmonary oedema would cause interstitial fluid and an effusion, but there is more going on here. There is no mention of lymphadenopathy, which makes sarcoidosis less likely. Radiation pneumonitis can present similarly, as can lymphocytic interstitial pneumonitis. The latter is a benign lymphoproliferative disorder that tends to present with ground glass change and a history of autoimmune disease would be key. Ikezoe J, Godwin JD, Hunt KJ, et al. Pulmonary lymphangitic carcinomatosis: chronicity of radiographic findings in long-term survivors. AJR Am J Roentgenol 1995; 165: 49–52.
1176
A 40-year-old female with a history of recurrent pulmonary infection is admitted with cough, pyrexia and dizziness. A chest radiograph displays a persistent area of pulmonary consolidation in the left lower lobe, which had not changed despite adequate antibiotic treatment over a period of 6 weeks. CT demonstrates a consolidative mass in the posterior basal segment of the left lower lobe with brightly contrast-enhancing vessels, but no air bronchograms. What is the most likely diagnosis? A. Bronchigenic cyst B. Lymphoma C. Bronchial carcinoma D. Pulmonary arteriovenous malformation E. Sequestration
E. Sequestration This female has intralobar pulmonary sequestration. There are two types (intra- and extra-lobar) with the intralobar type accounting for 75% of all sequestrations (tend to occur in adolescents and young adults). The lung parenchyma lies within the visceral pleura of the lobe in which it occurs but there is no connection with the bronchial tree and following air drift, the area of lung becomes deaerated. Arterial supply is derived from the lower thoracic aorta with venous drainage into the left atrium via pulmonary veins (left to right shunt). The sequestration is often located in the posterior segment of the left lower lobe. Presentation is usually with recurrent infections after bacteria migrate through the pores of Kohn. The bright contrast enhancement of the vessels occurs because they are derived from the aorta, not the pulmonary tree (which is in a different contrast phase). The main differential here would be obstruction secondary to a central lesion, but the description here does not fit. Molinari F, Paolantonio G, Valente S, et al. Images in cardiovascular medicine. Intralobar pulmonary sequestration in a 46-year-old woman. Circulation 2009; 119: e368–70.
1177
A 41-year-old male undergoes a routine insurance medical examination during which he has a chest radiograph. Heart and mediastinal contours are normal and the lungs are clear apart from a small (7 mm) nodule located peripherally in the right lower lobe. There is popcorn calcification, but no fat within the lesion. What is the diagnosis? A. Carcinoid B. Granuloma C. Bronchogenic carcinoma D. Pulmonary hamartoma E. Breast cancer metastasis
D. Pulmonary hamartoma A pulmonary hamartoma is a localized benign neoplasm of an abnormal amount of normal tissue, for example, cartilage, connective tissue, muscle, fat and bone. They account for approximately 8% of all lung neoplasms and 6% of solitary pulmonary nodules, and are commonly diagnosed in the fourth or fifth decade and more commonly occur in males (2–3:1). They typically occur in the periphery of the lung and contain fat and calcification. However, one or both of these constituents may not be apparent when imaging and the absence of fat does not exclude the diagnosis. Popcorn calcification is classically used to describe the appearance of calcification in a hamartoma (it is also used to describe chondroid lesions, fibrous dysplasia and fibroadenomas in the breast!). The differential here is that of a calcified solitary pulmonary nodule and all the answers are potential diagnoses. The chief differential in this case is a granuloma, the commonest cause a calcified solitary pulmonary nodule. These often occur secondary to a previous insult and during the resultant inflammatory process undergo calcification. They are usually discovered incidentally. Chai JL, Patz EF. CT of the lung: patterns of calcification and other high-attenuation abnormalities. AJR Am J Roentgenol 1994; 162: 1063–6.
1178
A 32-year-old male presents with haemoptysis but without history of previous lung disease. The chest radiograph demonstrates a solitary lesion in the left upper lobe that is confirmed at CT. It is sited peripherally, but there is no local invasion, and no other parenchymal abnormality, lymphadenopathy or effusion. A CT-guided biopsy is undertaken that is uncomplicated and histology reveal gland-like sheets with calcification and amyloid deposition. What is the most likely diagnosis? A. Small-cell lung carcinoma B. Carcinoid tumour C. Intermediate cell neuroendocrine carcinoma D. Adenocarcinoma E. Hamartoma
B. Carcinoid tumour While bronchial carcinoid is uncommon, it is certainly the most likely diagnosis in this setting. Carcinoid tumours cause a wide variety of symptoms including haemoptysis, have no gender predilection, tend to affect younger patients than bronchial neoplasms. The mean age of presentation is in the mid-40s. Histologically, there is variation, but sheets of glands with calcification and amyloid deposition is highly suggestive of the disease. This tumour is sited peripherally, which is less common than a central position (80% are central). However, there is no invasion, no lymphadenopathy and it is solitary – all key features of carcinoids. Carcinoids can less frequently present as atypical carcinoids with differing histology. A large cell neuroendocrine variant is intermediate cell neuroendocrine carcinoma and at the far end of the spectrum is small cell carcinoma. While a hamartoma may calcify, it is not the likely diagnosis here. Jeung MY, Gasser B, Gangi A, et al. Bronchial carcinoid tumors of the thorax: spectrum of radiologic findings. Radiographics 2002; 22: 351–65.
1179
A 23-year-old male presented with shortness of breath and pleuritic chest pain. He is apyrexial. A chest radiograph revealed a cavitating mass in his left mid zone and a moderate sized pneumothorax. There is no other presenting symptom. What is the most likely diagnosis? A. Squamous cell carcinoma B. Wegner’s granulomatosis C. Adenocarcinoma D. Osteosarcoma E. Staphylococcal infection
D. Osteosarcoma This young male has presented with a spontaneous pneumothorax secondary to a pulmonary metastasis from an osteosarcoma. The important differential here is that of a pulmonary mass with cavitation, and can include infection, inflammatory conditions, malignancy, trauma, vascular malformations and congenital abnormalities. Both osteosarcoma pulmonary metastases and Wegner’s granulomatosis cavitate and both may be associated with pneumothorax. While uncommon, presentation of osteosarcoma with lung metastasis and pneumothorax is recognized, spontaneous pneumothorax in Wegner’s without any other symptoms is even rarer. Squamous cell carcinoma is the classical example of bronchial neoplasia with cavitation, and while other bronchial neoplasms may cavitate, they would both be unlikely in a male of this age. There are no other features of infection that would make it the likely cause. Hoag JB, Sherman M, Fasihuddin Q, et al. A comprehensive review of spontaneous pneumothorax complicating sarcoma. Chest 2010; 138: 510–18. doi: 10.1378/chest.09-2292.
1180
A 26-year-old with fulminant hepatic failure presents with shortness of breath and the diagnosis of hepatopulmonary syndrome is considered. A chest radiograph is performed that shows parenchymal nodules and a subsequent CT demonstrates hepatic nodules also. What is the most likely diagnosis? A. Hereditary haemorrhagic telangectasia B. Alpha-1 antitrypsin deficiency C. Amiodarone toxicity D. Sarcoidosis E. Cystic fibrosis
D. Sarcoidosis Hepatopulmonary syndrome is a recognized complication of end-stage liver disease with pulmonary manifestations caused by the alteration in the production or clearance of circulating cytokines and other mediators. Pulmonary vasodilatation results in hypoxemia with arteriovenous shunting and ventilation-perfusion mismatch. Many conditions, including all the given answers, are associated with hepatopulmonary syndrome. Those associated with nodular parenchymal change are sarcoidosis and hereditary haemorrhagic telangectasia and the hepatic parenchymal nodules point towards sarcoidosis (a multisystem granulomatous disease). Hereditary haemorrhagic telangectasia (Osler–Weber–Rendu disease) is a group of autosomal dominant conditions that result in a range of vascular disorders and were there a history of haemorrhage or features suggestive of a vascular malformation, this would be the more likely diagnosis. Cystic fibrosis would be associated with bronchiectasis and not nodular disease. Alpha-1 antitrypsin deficiency is associated with panlobular emphysema. Amiodarone use in a young patient is unlikely given its cumulative side effect profile and it would not result in lung nodules. Meyer CA, White CS, Sherman KE. Diseases of the hepatopulmonary axis. Radiographics 2000; 20: 687–98.
1181
A 41-year-old female smoker presents to her general practitioner with weight loss and a dry cough for 3 months. On examination, her fingers are clubbed and she is short of breath at rest. Her chest X-ray is unremarkable. Because she fails to respond to beta-agonists, she is referred by the respiratory physicians for high-resolution CT. This demonstrates widespread ground glass opacity sparing the upper zones bilaterally with early features of fibrosis. What is the likely diagnosis? A. Respiratory bronchiolitis interstitial lung disease B. Asthma C. Cardiac failure D. Lymphoid interstitial pneumonia E. Desquamative interstitial pneumonia
E. Desquamative interstitial pneumonia Desquamative interstitial pneumonia is an interstitial lung disease primarily affecting middle aged male smokers but can affect females. Macrophages infiltrate the alveoli and thus high-resolution CT features include confluent bibasal airspace consolidation, which progresses to fibrosis with traction bronchiectasis if untreated with corticosteroids. Symmetrical basally and peripherally predominant centrilobular nodules are also seen. Respiratory bronchiolitis-associated interstitial lung disease is an interstitial lung disease affecting younger, heavy smokers and considered a precursor to desquamative intersitial pneumonia. Ground glass opacification and centrilobular nodules are patchy rather than widespread and fibrosis is NOT a feature. Asthma is an important differential clinically but at this age, the diagnosis should be established, and hyperinflation may be seen on chest X-ray. Cardiomegaly, pulmonary plethora, upper lobe diversion, interstitial lines and pleural effusions may be seen but nodules and fibrosis will not. Lymphocytic interstitial pneumonitis shows ground glass opacification but deep parenchymal cysts are a diagnostic feature. Lynch DA, Travis WD, Müller NL, et al. Idiopathic interstitial pneumonias: CT features. Radiology 2005; 236: 10–21. doi: 10.1148/radiol.2361031674.
1182
A 28-year-old male presents with cardiac chest pain and ST depression on his electrocardiogram. Physical examination and chest radiography are normal. Troponin is elevated. CT pulmonary angiogram demonstrates no coronary calcification, no pulmonary embolism and no other cause for chest pain. Echocardiography demonstrates left ventricular dysfunction. A coronary angiogram confirms the likely diagnosis: A. Atheromatous plaque B. Vasospasm C. Dissection secondary to cocaine D. Myocardial bridging E. Anomalous coronary supply
C. Dissection secondary to cocaine Myocardial bridging describes an abnormal course of the coronary arteries. Rather than having an epicardial course, they dip into the myocardium and during systolic contraction of the cardiac muscle, there is relative compression of the vessel. However, cardiac CT has demonstrated this is a very common finding, rarely important. Cocaine use and its sequelae should be considered in young patients with cardiac ischaemia although it is important to exclude other non-cardiac causes. Cocaine use may cause vasospasm or dissection, both of which are unlikely to be demonstrated on a non-gated study. The investigation of choice here is a formal coronary angiogram. Atheromatous plaque disease is much more prevalent in older patients, typically those over the age of 40. Anomalous coronary vessels may cause ischaemia, but diagnosis would have been expected by this age. Bayés A, Marti V, Augé JM. Coronary stenting for symptomatic myocardial bridging. Heart 1998; 80: 102–3. doi: 10.1136/hrt.80.1.102.
1183
A 28-year-old male presents to the accident and emergency department with chest pain and breathlessness after falling from standing onto an occasional table. A chest radiograph reveals a significant amount of surgical emphysema, a number of fractured ribs, but no pneumothorax. He is admitted for observation. On day 2, there is rapid deterioration in his condition, with a new effusion and a CT shows large volume fluid in the chest with mediastinal shift. There is small volume extravasation of contrast into the chest from a low intercostal. What is the best next course of action? A. Perform a delayed phase CT chest B. Insert a wide bore chest drain C. Immediate cardiothoracic referral D. Suggest vascular radiology embolization E. Chest drain insertion and cardiothoracic referral
C. Immediate cardiothoracic referral There has been rapid deterioration in this male’s clinical condition and active extravasation from a lacerated intercostal artery. Chest CT is performed in the arterial phase unless otherwise stated (clearly non-contrast and pulmonary phase examinations can be performed). At present, there is mediastinal shift, and with mediastinal shift comes the possibility of vascular compromise, so immediate action is required. However, the large volume of blood in the pleural space may be tamponading the bleeding vessel and insertion of a chest drain could cause clinical deterioration. Thus, discussion with the cardiothoracic team, by the radiologist or the referring clinical is paramount. While embolization of vessels following trauma can be performed by the vascular radiologist, but the mediastinal shift is concerning and should be treated as soon as possible to prevent vascular compromise and deterioration. A delayed phase study would not be helpful. Ross RM, Cordoba A. Delayed life-threatening hemothorax associated with rib fractures. J Trauma 1986; 26: 576–8.
1184
A 45-year-old female presents with breathlessness that has increased insidiously over the last 6–12 months. She has a history of asthma, but increases in her inhaled medication have not helped. A chest radiograph demonstrates non-segmental consolidation, particularly in the upper lobes. A high-resolution CT is performed that demonstrates ground-glass opacities and non-segmental peripheral consolidation, in the upper lobes, with similar, but less severe appearances peripherally in the lower lobes bilaterally. What is the most likely diagnosis? A. Churg–Strauss syndrome B. Chronic eosinophilic pneumonia C. Loeffler syndrome D. Usual interstitial pneumonia E. Non-specific interstitial pneumonia
B. Chronic eosinophilic pneumonia This female has chronic eosinophilic pneumonia. It usually affects middle-age people and women are more commonly affected than men. In 50% of cases, there is an association with asthma. Increasing respiratory symptoms are insidious in onset. There is associated eosinophilia. Radiographically, non-segmental peripheral airspace consolidation is typical. This usually affects the upper lobes and may appear as the reverse-bats-wing pattern, that is, the reverse of classical pulmonary oedema. CT confirms this pattern of consolidation and in some cases, there may be areas of ground-glass change, nodules and reticulation. The latter are more common in the later stages of the disease when there are additional band-like opacities parallel to the pleural surface. Pleural effusions may also be apparent. Churg–Strauss syndrome may present with peripheral consolidation and is associated with asthma. However, it tends to present with lower lobe predominant disease. Löeffler syndrome has an identical radiological appearance, but is an acute condition with transient changes that alter over days. Usual interstitial pneumonia and non-specific interstitial pneumonia tend not to present with peripheral consolidation. Peripheral subpleural honeycombing is the classical appearance of UIP and patchy ground-glass change are the dominant finding in NSIP. Jeong YJ, Kim KI, Seo IJ, et al. Eosinophilic lung disease: a clinical, radiologic and pathologic overview. Radiographics 2007; 27: 617–37. doi: 10.1148/rg.273065051.
1185
A 42-year-old post-menopausal female presented a year earlier with heavy vaginal bleeding requiring transfusion. Subsequent investigation revealed uterine fibroid disease and she was commenced on a gonadotrophin-releasing hormone analogue. She has been on it for 5 months and a decision is made to proceed to therapeutic embolization. Which of the following is the best solution? A. Use non-heparinized saline to flush catheters B. Use end-hole only catheters when using absolute alcohol C. Slightly oversize the coil relative to the target vessel D. Plan embolization for 3 months time E. Use autologous blood clots for the treatment of high-flow priapism
D. Plan embolization for 3 months time The medical treatment of fibroids using gonadotropin-releasing hormone analogues shrinks the uterine arteries making them difficult to catheterize and delaying intervention for 3 months after their use is suggested. Gonadotropin-releasing hormone analogues are usually only used prior to surgical management and work by reducing oestrogen levels. They tend not to be used for more than 6 months at a time because of the risk of osteoporosis. The other options are all appropriate in the treatment of fibroid disease, but only after appropriate delay. Using non-heparinized saline to flush catheters is useful because heparin hampers the attempted embolization. End-hole only catheters should be used when using absolute alcohol because liquid sclerosants can be difficult to control and direct. A coil that reforms perfectly is probably under-sized and a coil that is too small may embolize to a distal position. Too large and it will not form the coil at all leaving a thrombogenic site upstream. While autologous blood clots are rarely used in general intervention, they may be appropriate in this condition. Stein K, Ascher-Walsh C. A comprehensive approach to the treatment of uterine leiomyomata. Mt Sinai J Med 2009; 76: 546–56. doi: 10.1002/msj.20145.
1186
A 75-year-old on intensive care unit has a chest radiograph following robotic prostatectomy. The endotracheal tube tip is 2.5 cm above the carina and a right-sided central line is in situ and appropriately sited with the tip at the cavoatrial junction. The lungs are clear and the cardiomediastinal contour is normal. The right hemidiaphragm is elevated, but clearly seen. The right hilum is normal. A previous chest radiograph is normal. What is the cause of this abnormality? A. Eventration B. Right lower lobe consolidation C. Right upper lobe collapse D. Morgagni hernia E. Phrenic nerve palsy
E. Phrenic nerve palsy This question is related to the many causes of diaphragmatic elevation and how to narrow the differential. The lungs are clear and there is no mention of collapse, hilar abnormality of mediastinal shift. This means that consolidation and collapse cannot be the correct answer. Diaphragmatic eventration cannot be the correct answer because of the previously normal chest radiograph. When a Morgagni hernia occurs, the diaphragmatic contour is usually difficult to clearly trace because of interposed bowel or hernial contents. In addition, a previously normal chest radiograph make this an unlikely cause. The cause of diaphragmatic elevation here is phrenic nerve palsy. This has been reported as the result of irritation from central venous catheters. Takasaki Y, Arai T. Transient right phrenic nerve palsy associated with central venous catheterization. Br J Anaesth 2001; 87: 510–11.
1187
A 64-year-old female who has a history of systemic lupus erythematosus and who has been on long-term treatment with methotrexate presents with increasing breathlessness. A chest radiograph is performed and followed by further investigation with high-resolution CT. Which pattern of lung disease is unlikely to be evident on the high-resolution CT that is performed? A. Diffuse alveolar disease B. Eosinophilic pneumonia C. Non-specific interstitial pneumonia D. Pulmonary haemorrhage E. Usual interstitial pneumonia
E. Usual interstitial pneumonia Many drugs can cause lung toxicity and the patterns of disease manifestation is protean. There are a number of histopathological manifestations of pulmonary drug toxicity, the commonest patterns of which are: * diffuse alveolar disease * non-specific interstitial pneumonia * cryptogenic organising pneumonia * eosinophilic pneumonia * pulmonary haemorrhage. A usual interstitial pneumonia pattern of disease would not be expected in this clinical situation. Rossi S, Erasmus JJ, McAdams HP, et al. Pulmonary drug toxicity: radiologic and pathologic manifestations. Radiographics 2000; 20: 1245–59.
1188
A 25-year-old active female dislocates her patella for the first time. On MRI, she has disruption of her medial patellofemoral ligament. Which constellation of abnormalities are you most likely to see? A. Disruption of the patellar tendon with bony bruising of the superior aspect of the patella B. Tear of the anterior cruciate ligament with destabilization of the lateral ligaments C. Small avulsion fracture involving the superolateral surface of the proximal tibia D. Deformity or oedema of the inferomedial patella and the lateral femoral condyle E. Increase in width and oedema involving the deep portion of the proximal end of the patellar tendon
D. Deformity or oedema of the inferomedial patella and the lateral femoral condyle MR imaging can be used to diagnose prior patellar dislocation on the basis of typical injury patterns. In general, deformity or oedema of the inferomedial patella and the lateral femoral condyle in conjunction with medial patellofemoral ligament disruption and patellar lateralization is diagnostic for recent patellar dislocation. Answer C describes a Segond fracture that is frequently associated with lateral capsular ligament and anterior cruciate ligament tears. Answer E describes patella tendonosis (Jumper’s knee). Diederichs G, Issever AS, Scheffler S. MR imaging of patellar instability: injury patterns and assessment of risk factors. Radiographics 2010; 30: 961–81.
1189
Which of these is not a recognised risk factor for patellar dislocation? A. Trochlear dysplasia B. Patella alta C. Patella baja D. Marfan syndrome E. Excessive lateralization of the tibial tuberosity
C. Patella baja Patella baja (low-riding patella) is associated with achondroplasia, quadriceps tendon rupture, and juvenile rheumatoid arthritis. Risk factors for patellar dislocation include trochlear dysplasia, patella alta (high-riding patella), and excessive lateralization of the tibial tuberosity. Other recognized anatomical risk factors include femorotibial malrotation, genu recurvatum (back-knee), and ligamentous laxity (Ehlers–Danlos and Marfan syndromes). Diederichs G, Issever AS, Scheffler S. MR imaging of patellar instability: injury patterns and assessment of risk factors. Radiographics 2010; 30: 961–81.
1190
A 55-year-old male patient presents with back pain and is seen to have bilateral sacroiliac erosions. Which of the following diseases should not be included in the differential diagnosis? A. Ankylosing spondylitis B. Hyperparathyroidism C. Inflammatory bowel disease D. Psoriatic arthritis E. Rheumatoid arthritis
E. Rheumatoid arthritis Involvement of the sacroiliac (SI) joints is common in the HLA-B27 spondyloarthropathies. Ankylosing spondylitis and inflammatory bowel disease typically produce bilateral symmetrical joint disease, which is initially erosive and progresses to sclerosis and fusion. Hyperparathyroidism also can have bilateral SI joint erosions. Reactive arthritis (Reiter’s syndrome) and psoriatic arthritis can be either unilateral or bilateral. Rheumatoid arthritis can involve large joint such as the hip, knee and shoulder but does not typical cause erosions at the SI joints. Helms C. Fundamentals of Skeletal Radiology. 3rd edn. Philadelphia, PA: Elsevier Saunders; 2004.
1191
A 45-year-old male presents with knee pain. MRI is performed. On the fat suppressed T2 weighted para-sagittal images a high signal line extending horizontally in the posterior horn of the medial meniscus is seen to just breach the articular surface of the meniscus on a single 3 mm slice. This should be reported as: A. A horizontal tear of the medial meniscus B. A radial tear of the medial meniscus C. A bucket handle tear of the medial meniscus D. Intrasubstance degeneration only E. Meniscocapsular separation
D. Intrasubstance degeneration only Meniscal signal that does not disrupt the articular cartilage represents intrasubstance degeneration and high signal must be seen to reach the articular cartilage before a tear is definitely diagnosed. However, if breach of the articular surface of the meniscus is only seen on a single thin slice then the chances of this abnormality being identified at arthroscopy is low, and these lesions are therefore best not reported as tears. De Smet AA, Tuite MJ. MR imaging and MR arthrography for diagnosis. AJR Am J Roentgenol 2006; 187: 911–14.
1192
A 78-year-old female presents following a low energy fall onto her right arm. Radiographs show a fracture and CT is performed for preoperative planning. In addition to the fracture, multiple lucencies are seen in her humerus. On closer inspection, it is confirmed that these are within the cortex of her humerus. Which of the following diagnoses is most likely? A. Angiomatosis B. Osteosarcoma C. Primary lymphoma D. Aggressive osteoporosis E. Metastatic disease
D. Aggressive osteoporosis It is important to differentiate multiple bony lucencies as either medullary or cortical in origin. In young people with a permeative pattern that does not involve the cortex, differentials include Ewing’s sarcoma, infection and eosinophilic granuloma. In older people the differential includes multiple myeloma, metastatic carcinomatosis, and primary lymphoma. If the lucencies can be confirmed to be purely cortical in position then include benign condition such as aggressive osteoporosis, haemangioma and radiation-induced changes should be considered. Helms C. Fundamentals of Skeletal Radiology. 3rd edn. Philadelphia, PA: Elsevier Saunders; 2004.
1193
A 21-year-old female presents with wrist pain 3 weeks following a fall. On radiographs, no fracture is seen but on the lateral view the scapholunate angle is significantly increased – the scaphoid is pointing more than 70° volarly in comparison to the lunate. The capitolunate angle is also abnormally increased with the capitate pointing more than 20° volarly in comparison to the lunate. These findings are most consistent with: A. Scapholunate dissociation B. Volar intercalated segment instability C. Dorsal intercalated segment instability D. Lunate dislocation E. Perilunate dislocation
C. Dorsal intercalated segment instability Dorsal intercalated segment instability is most due to ligamentous injury of the scapholunate ligament as a result of trauma or arthritis. The normal scapholunate angle is 30°–60° and the capitolunate angle is less than 20°. The scenario in this question describes dorsal intercalated segment instability where there is a dorsal tilt of the lunate. With the rarer volar intercalated segment instability, which is due to an incompetent triquetrolunate ligament, the scapholunate angle is decreased (<30°) and there is a volar tilt of the lunate. Weissleder R, Wittenburg J, Harisinghani M, et al. Primer of Diagnostic Imaging. 4th edn. Maryland Heights, MO: Mosby; 2007.
1194
A 62-year-old female presents for an ankle X-ray after stepping on a small piece of metal. There is no evidence of a foreign body on the X-ray but there is an area of relative radiolucency seen in the anterio-inferior portion of the calcaneus. This area is poorly defined. There is no associated fracture or evidence of periosteal reaction. This is most likely to represent: A. Unicameral bone cyst (simple bone cyst) B. Non-ossifying fibroma C. Intraosseous lipoma D. Pseudolucency of the calcaneum E. Bone infarct
D. Pseudolucency of the calcaneum Pseudolucency (sometimes referred to as pseudotumour or pseudocyst) of the calcaneus is a result of diminished stress through this region with resultant atrophy of the bony trabeculae. The region is identical to the region where simple bone cysts occur in the calcaneum. Pseudolucency can also occur in the region of the greater tuberosity of the humerus. Simple bone cysts and non-ossifying fibromas are almost exclusively found in the under 30 seconds. Bone infarcts are sclerotic while intraosseous lipomas are well circumscribed. Helms C. Fundamentals of Skeletal Radiology. 3rd edn. Philadelphia, PA: Elsevier Saunders; 2004.
1195
A 29-year-old male is referred by the general practitioner for a lumbar spine X-ray for long-standing back pain not relieved by aspirin. Plain radiograph shows scoliosis along with an expansile lytic lesion in the posterior elements of the vertebral body of L4. This has a soap bubble appearance with some speckled calcification. The radiological abnormality is most likely to represent: A. Aneurysmal bone cyst B. Non-ossifying fibroma C. Osteoblastoma D. Metastasis E. Osteoid osteoma
C. Osteoblastoma Osteoblastoma is a rare tumour accounting for 1% of all primary bone neoplasms. Majority of the patients are under the age of 30 years. It is two to three times more common in males. Presentation differs from osteoid osteoma in that pain is not usually acute and rarely relieved by aspirin. The spine including the sacrum is the commonest location. Over 90% involve the neural arch eccentrically typically leading to painful scoliosis. Other major sites are diaphysis and metaphysis of long bones. Radiologically the lesion is predominantly lytic measuring between 1 and 10 cm. CT often reveals occult calcification which may be punctuate, nodular or generalized. Larger lesions may result in bony expansion with or without surrounding shell of reactive bone. Grainger RG, Allison DJ. Diagnostic Radiology. 5th edn. London, UK: Churchill Livingstone; 2007.
1196
A previously healthy 53-year-old driver involved in a high-speed road traffic accident is admitted to the accident and emergency department. On the initial trauma series, there is a completely collapsed L3 vertebral body. The following does not cause this appearance: A. Scheuermann’s disease B. Plasmocytoma C. Thalassaemia D. Myeloma E. Osteoporosis
C. Thalassaemia The above description fits “vertebra plana” when a vertebral body has lost its entire height, both anteriorly and posteriorly. It occurs in a variety of settings, including trauma, osteoporosis, Langerhan’s cell histiocytosis, myeloma and metastases. Thalasseamia does not cause this appearance – it typically causes a classic “hair on end” appearance (skull), rodent facies (facial bones) and “rib within rib” appearance (ribs). Other skeletal manifestations include extramedullary haematopoiesis and premature fusion of epiphyses. Danhert W. Radiology Review Manual. 6th edn. Philadelphia, PA: Lippincott Williams and Wilkins; 2007.
1197
A plain radiograph of the hand of a 48-year-old female demonstrates subperiosteal bone resorption along the radial aspect of the middle phalanges along with a lytic lesion in the distal phalanx of the left index finger. What is the most likely diagnosis? A. Osteopetrosis B. Hyperparathyroidism C. Sarcoidosis D. Pseudohypoparathyroidism E. Pseudopseudohypoparathyroidism
B. Hyperparathyroidism The described features are pathognomonic for hyperparathyroidism, especially subperiosteal erosions in the phalanges, particularly on the radial aspect. It may also involve other sites such as distal phalanges (acro-osteolysis), outer end of clavicles, sacroiliac joints, proximal humerus shaft, femur and ribs. Other features include intracortical bone resoprtion, chondrocalcinosis, browns tumours, osteosclerosis, osteoporosis and metastatic calcification. Hypoparathyroidism causes skull vault thickening, basal ganglia calcification, extraskeletal ossification. Pseudohypoparathyroidism exhibits end organ defect in parathyroid hormone responsiveness. Radiographic features include premature epiphyseal fusion, calvarial thickening, bone exostoses, basal ganglia and soft tissue calcification. Metacarpal shortening is seen in fourth and fifth digits. Pseudo-pseudohypoparathyroidism is the same as pseudohypoparathyroidism radiologically, however, with absent parathyroid or biochemical abnormalities. Osteopetrosis is result of defective osteoclastic resorption radiologically showing dense bones with lack of corticomedullary differentiation. Grainger RG, Allison DJ. Diagnostic Radiology. 5th edn. London, UK: Churchill Livingstone; 2007. Helms C. Fundamentals of Skeletal Radiology. 3rd edn. Philadelphia, PA: Elsevier Saunders; 2004.
1198
A plain radiograph of a knee of an otherwise healthy 29-year-old female demonstrated a lucent defect in the patella. She underwent an MRI, which showed the lesion in the upper outer quadrant of the patella with low signal on T1 weighted imaging and high signal on T2 weighted imaging. What is the most appropriate next step in management? A. Ultrasound B. Bone scan C. Bone biopsy D. Reassure E. CT
D. Reassure This is a normal variant – dorsal defect of the patella with characteristic MR appearances. Helms C. Fundamentals of Skeletal Radiology. 3rd edn. Philadelphia, PA: Elsevier Saunders; 2004.
1199
A 59-year-old male presents to the accident and emergency department after falling off a 5-foot wall. As part of the skeletal trauma series, the radiologist reports that there is an incidental finding of anterior scalloping of multiple vertebral bodies in the lumbar spine. Which of the following is not associated with this finding? A. Down’s syndrome B. Hurler’s syndrome C. Tuberculous spondylitis D. Aortic aneurysm E. Chronic leukaemia
B. Hurler’s syndrome Anterior vertebral scalloping is a concavity to the anterior aspect of the vertebral body, typically seen on lateral projections of the spine. Hurler’s syndrome is a cause of posterior vertebral body scalloping. Chronic leukaemia presents as retroperitoneal lymphadenopathy thereby causing anterior scalloping. The rest are usual causes of anterior scalloping. Grainger RG, Allison DJ. Diagnostic Radiology. 5th edn. London, UK: Churchill Livingstone; 2007.
1200
In ankylosing spondylitis which of the following imaging features is not a typical: A. Squaring of the vertebral body B. “Trolley track” sign C. Diffuse syndesmophytic ankylosis D. Bilateral asymmetric sacroilitis E. Romanus lesion
D. Bilateral asymmetric sacroilitis Ankylosing spondylitis is the most common seronegative spondyloarthropathy and has an overwhelming male predilection. The onset of the disease occurs between the ages of 15 and 35 years, and more than 90% of afflicted are HLA-B27 positive. Radiographically, the earliest sign is indistinctness of the joint. The joints initially widen before they narrow. Subchondral bony erosions on the iliac side of the joint are seen; these are followed by subchondral sclerosis and bony proliferation. At end-stage, the sacroiliac joint may be a thin line or not visible at all. Sacroilitis is typically symmetrical. Bilateral asymmetrical sacroilitis is typical of psoriatic arthropathy, although it has been noted in the early stages of ankylosing spondylitis. Other spinal features include early spondylitis characterized by small erosions at the corners of vertebral bodies with reactive sclerosis, squaring of the vertebral body, diffuse syndesmophytic ankylosis (“bamboo spine” appearance), interspinous ligament calcification (“dagger spine” appearance), ossification of spinal ligaments, joints and discs, enthesophyte formation from enthesopathy, Romanus lesions of the spine (shiny corner sign) and trolley track sign. Helms CA. Arthritis. In: Fundamentals of Skeletal Radiology. 3rd edn. Philadelphia, PA: Elsevier Saunders; 2004.
1201
A 9-year-old boy presents with increasing clicking and locking of his left knee. He also reports pain over the lateral aspect of the left knee. On MRI, there is absence of the normal triangular meniscal morphology of the meniscus. The lateral meniscus has a “bow-tie” appearance on four consecutive sagittal sections. The most likely diagnosis is: A. Meniscal tear B. Normal variant C. Discoid meniscus D. Bucket-handle tear E. Meniscal haematoma
C. Discoid meniscus Discoid meniscus is a morphological variant characterized by meniscal thickening and widening, and is most frequently seen affecting the lateral meniscus. It is bilateral in 20%. The typical findings are that instead of a normal narrow crescent shape, a discoid meniscus appears thickened. It does not taper as much towards the centre of the joint and is shaped like a disc. The thickness of the meniscus and its reduced vascular blood supply makes it more prone to tears compared to a normal meniscus. The transverse diameter of a normal meniscus is approximately 10–11 mm; therefore a normal meniscus body should be visible on up to two slices of a MR with 4–5 mm sagittal slices. A discoid meniscus needs be considered if more than two contiguous body segments are present. Children with discoid meniscus usually present with a snapping knee joint, especially those around 7 years old. If the child remains otherwise asymptomatic, only observation is necessary. Grainger RG, Allison DJ. Diagnostic Radiology. 5th edn. London, UK: Churchill Livingstone; 2007.
1202
A 10-year-old boy fell onto his right shoulder while playing football. Plain radiograph of the right humeral head demonstrated a lucent, well-demarcated geographical lesion arising from the metaphysis, with its long axis parallel to the axis of the bone. A small detached fragment of bone is noted in the dependent portion of the lesion. The most likely diagnosis is: A. Giant cell tumour B. Unicameral bone cyst C. Aneurysmal bone cyst D. Eosinophilic granuloma E. Non-ossifying fibroma
B. Unicameral bone cyst A simple or unicameral bone cyst is a common benign lucent bone lesion. Most are discovered in people under the age of 20 years, with it being twice as likely in males. These lesions are usually asymptomatic and found incidentally, although pain, selling and stiffness of the adjacent joint also occur. The most frequent complication is pathological fracture, and this is frequently the cause of presentation. Unicameral bone cyst are well-demarcated lucent lesions with a narrow transitional zone and no adjacent periosteal reaction. If there is fracture through this lesion a dependent bony fragment may be seen, and this is known as the fallen fragment sign. CT and MRI adds little to the diagnosis, but can help in eliminating other entities that can potentially mimic a simple bone cyst. MR imaging of the lesion shows low signal T1 weighted and high signal T2 weighted images. Intervention is usually not required for asymptomatic lesion. If large then intra-lesional steroid injection (with or without curettage and bone grafting) can be performed. Helms CA. Fundamentals of Skeletal Radiology. 3rd edn. Philadelphia, PA: Elsevier Saunders; 2004.
1203
A 69-year-old male presents with a 1-year history of increasing lower back pain. Lumbar spine radiograph demonstrates an ivory vertebra affecting multiple vertebral bodies. Differentials include the following except: A. Paget’s disease B. Osteoblastic metastasis C. Lymphoma D. Carcinoid E. Leukaemia
E. Leukaemia Ivory vertebra sign refers to an increase in opacity of a vertebral body that retains its size and contours, with no change in the opacity and size of adjacent intervertebral disks. This sign can be seen in both adults and children, with it being much less common in the latter group, where it is typically the result of lymphoma, usually Hodgkin lymphoma. In adults, there are multiple differential diagnoses, including associations with metastatic disease, especially carcinoma of the prostate or breast, and occasionally with osteosarcoma, carcinoid, Paget’s disease and lymphoma, particularly Hodgkin’s lymphoma. Graham TS. The ivory vertebra sign. Radiology 2005; 235: 614–15.
1204
A 20-year-old male presents with low back pain. MRI demonstrates a bamboo spine along with discal calcification. Disc spaces are considerably narrowed. The radiographer at the time makes note of overlying skin pigmentation. The most likely diagnosis is: A. Calcium pyrophosphate deposition disease B. Ankylosing spondylitis C. Alkaptonuria D. Rheumatoid arthritis E. Psoriatic arthropathy
C. Alkaptonuria Alkaptonuria is a rare inborn metabolic disorder characterized by pigmentation of connective tissue. Patients often have pigmentation of auricular cartilages and sclera. Urine colour tends to be dark. Imaging findings include multilevel intervertebral disc calcification, syndesmophytic formation (similar appearance to bamboo spine) and multilevel disc space narrowing. Radiographic findings within joints include early osteoarthritis, subchondral sclerosis and chondrocalcinosis. Complications include aortic stenosis and chest radiography is advised to assess for the possible involvement of aortic or mitral valves. Jacobson JA, Girish G, Jiang Y, et al. Radiographic evaluation of arthritis: degenerative joint disease and variations. Radiology 2008; 248: 737–47.
1205
A 56-year-old male presents with increased left elbow stiffness. Accompanying radiograph displays joint disorganization and dislocation, with significant intra-articular debris formation. The most common cause of these appearances in the upper limb is: A. Peripheral neuropathy B. Syringomyelia C. Diabetes mellitus D. Tabes dorsalis E. Alcohol excess
B. Syringomyelia Neuropathic osteoarthropathy tends to affect patients with diabetes mellitus, syringomyelia and syphilis. It is associated with neurosensory deficit and loss of proprioception. Best reminded by Mnemonic 6Ds: dense bones, degeneration, destruction of articular cartilage, deformity, debris and dislocation. Diabetes is the foremost for neuropathic joint disease, and the foot is the most affected region. Patients with neurosyphilis tend to have knee involvement, and patients with syringomyelia of the spinal cord may demonstrate shoulder deformity. Jones EA, Manaster BJ, May DA, et al. Neuropathic osteoarthropathy: diagnostic dilemmas and differential diagnosis. Radiographics 2000; 20: S279–93.
1206
A 34-year-old male with characteristic appearance of acromegaly has back pain and fatigue. Which of the following features does not support the diagnosis on spinal imaging? A. Increased intervertebral height B. Posterior scalloping C. Diffuse idiopathic skeletal hyperostosis-like condition D. Elongation and widening of the vertebral bodies E. Picture frame appearance of vertebral body
E. Picture frame appearance of vertebral body Acromegaly is caused by excess growth hormone generated secondary to pituitary hyperplasia or an adenoma. It most commonly affects adults in middle age and can result in severe disfigurement, serious complications and even premature death. Its insidious onset and slow progression make it difficult to diagnose in the early stages. It can be diagnosed when the external facial features become prominent. The key to diagnosis is in the skull. The sella turcica is enlarged with erosion of a portion of the posterior wall. The cranial vault is thickened, and there is mild enlargement of the frontal sinuses. In the hands, the terminal phalangeal tufts become hypertrophied and have a “spade appearance”. Joint spaces may be minimally enlarged. Premature osteoarthritis can set in the advanced stages of acromegaly. In the feet, heel pad thickness may be increased. In the vertebral column, there is a diffuse idiopathic skeletal hyperostosis – like condition, posterior scalloping, increased intervertebral height, elongation and widening of the vertebral bodies. Grainger RG, Allison DJ. Diagnostic Radiology. 5th edn. London, UK: Churchill Livingstone; 2007.
1207
A 54-year-old boat yard worker presents with bilateral wrist pain. He recalls dropping a heavy load onto his left hand over a year ago but reports no other history of trauma. He also reports increasing difficulty in breathing over the last year. Plain film radiograph demonstrates periostitis of both radius and ulna. Chest radiograph confirms a large right-sided pleural effusion. What feature on bone scan confirms the diagnosis? A. “Parallel track” sign B. Cold spots C. Focal increased uptake over epiphysis D. Normal scan E. None of the above
A. “Parallel track” sign Hypertrophic osteoarthropathy is secondary to cortical periostitis that arises in connection with various intra-thoracic tumors, pulmonary diseases, cardiac conditions, and intra-abdominal diseases. The most common scintigraphic pattern is non-uniform, irregular cortical uptake involving the long bones and giving rise to the “parallel track sign”. Scintigraphic findings tend to appear earlier than radiographic findings and correlate better with the clinical findings. Love C, Din AS, Tomas MB, et al. Radionuclide bone imaging: an illustrative review. Radiographics 2003; 23: 341–58.
1208
A 60-year-old alcoholic presents with increasing pain over the fifth metacarpophalangeal joint. A radiograph demonstrates juxta-articular erosions and overhanging edges. What MRI features are most typical of the diagnosis of gout? A. High T1 weighted and low T2 weighted B. Isointense T1 weighted and low T2 weighted C. Low T1 weighted and high T2 weighted D. High T1 weighted with increased uptake post gadolinium E. High T2 weighted and high short T1 inversion recovery image
B. Isointense T1 weighted and low T2 weighted Gout is a peripheral arthritis that results from the deposition of sodium urate crystals in one or more joints. Articular manifestations occur in the different stages of the disease and include acute arthritis, an intermediate phase, and chronic tophaceous gout. The most common sites of abnormality are the feet, hands, and wrists. Risk factors include obesity, hyperuricaemia, alcohol and myeloproliferative disorders. Characteristic joint features include joint effusion (earliest sign), preservation of space until late stages of disease, absence of periarticular osteopenia and juxta-articular erosions. Periarticular soft tissue swelling due to crystal deposition in tophi around the joints is common, but presence of tophi is pathognomonic. Chondrocalcinosis can also be seen in ~5%. The typical appearance is the presence of well-defined “punched-out” erosions in the juxta-articular distribution, with overhanging edges. On MR imaging, most lesions are isointense relative to muscle on T1 weighted images. On T2 weighted images, most lesions show low to intermediate heterogeneous signal intensity. Llauger J, Palmer J, Rosón N, et al. Nonseptic monoarthritis: imaging features with clinical and histopathologic correlation. Radiographics 2000; 20: S263–78.
1209
An 18-year-old male sustains an injury over the medial aspect of his thigh while playing football. Within 2 hours he notices a fluctuant mass over the site of injury. Which of the following features on MRI supports the diagnosis of haematoma? A. Hypointense T1 weighted and T2 weighted images B. Hyperintense T1 weighted and hypointese T2 weighted images C. Isointense T1 weighted and hyperintense T2 weighted images D. Hyperintense T1 weighted and hyperintense T2 weighted images E. Hyperintense T1 weighted and T2 weighted images
C. Isointense T1 weighted and hyperintense T2 weighted images The imaging characteristics of blood on MRI are variable and change with the age of the blood. The above appearance is in keeping with a hyperacute phase. * hyperacute phase T1 isointense/hypointense T2 hyperintense * acute phase T1 hypointense T2 hypointense * early subacute phase T1 hyperintense T2 hypointense * late subacute phase T1 hyperintense T2 hyperintense * chronic phase T1 hypointense T2 hypointense. Grainger RG, Allison DJ. Diagnostic Radiology. 5th edn. London, UK: Churchill Livingstone; 2007.
1210
The following is not a common location for bone resorption in hyperparathyroidism? A. Radial head B. Sacroiliac joint C. Middle phalanx D. Distal clavicles E. Distal phalanx
A. Radial head Hyperparathyroidism is the effect of excess parathyroid hormone in the body. It can be primary, secondary or tertiary. The radiological hallmark of hyperparathyroidism is bone resorption. Other features include osteopenia, brown tumours, terminal tuft erosions, osteosclerosis and soft tissue calcification. The sacroiliac joints are a common location for subchondral resorption (others include steroclavicular and acrominoclaviclar joints) and may mimic the radiographic findings of sacroiliitis. The iliac side is more severely involved. Radiographically, this produces ill-defined joint margins that may appear widened. Subperiosteal bone resorption characteristically affects the distal phalangeal tufts and the radial aspect of the proximal and middle phalanges of the fingers. Intracortical bone resorption, which results in a lacy appearance, involves the cortex of the metacarpals. Endosteal resorption may involve the phalanges of the digits. Yu J. Case Review Series: Musculoskeletal Imaging, Case 79. St Louis, MO: Mosby; 2001.
1211
A 59-year-old Indian male presents with worsening back pain, stiffness and localized tenderness. On MRI, there is low signal intensity in the subchondral region of the endplate on T1 weighted which becomes moderately bright on T2 weighted image indicating oedematous infected marrow. Your differential includes tuberculous and pyogenic spondylitis. Which of the following features makes tuberculous spondylitis less likely? A. Epidural and paraspinal abscess B. Subligamentous spread to L2–L3 and L3–L4 disks anteriorly C. Acute angular kyphosis D. Dural spread to S2 posteriorly E. Intervertebral disc destruction
E. Intervertebral disc destruction Tuberculous spondylitis refers to vertebral body and intervertebral disc involvement with tuberculosis. The spine is the most frequent site of osseous involvement, with the upper lumbar and lower thoracic spine being involved most frequently. There is usually a slow collapse of one or more vertebral bodies, which spreads underneath the longitudinal ligaments. This results in an acute angular kyphotic or “gibbus” deformity, which can lead to cord compression. Unlike pyogenic infections, the discs can be preserved. In late-stage spinal tuberculosis, large epidural and paraspinal abscesses without severe pain or frank pus are common, leading to the formation of “cold abscess”. The diagnosis of tuberculosis is favoured if a large, calcified paravertebral mass and absence of sclerosis or new bone formation are noted. Conversely, a rapid loss of height in the disc with destruction, along with extensive sclerosis, the absence of gibbus deformities and the absence of calcified paravertebral masses are in favour of a pyogenic spondylodiscitis. Harisinghani MG, McLoud TC, Shepard JA, et al. Tuberculosis from head to toe. Radiographics 2000; 20: 449–70.
1212
A 21-year-old male presents with increasing pain over the posterior aspect of his right knee. He indicates the presence of a mass that has been increasing in size over the past 6 months. A radiograph demonstrates an osteoblastic, exophyic, broad-based mass arising from the posterior surface of the distal femur. Differentials for these appearances include all the following except: A. Myositis ossificans B. Parosteal osteosarcoma C. Chondrosarcoma D. Osteochondroma E. Tumoral calcinosis
E. Tumoral calcinosis Parosteal osteosarcoma (OS) is the most common type of juxta-cortical OS, accounting for approximately 5% of all cases. It typically manifests in the second to fourth decades of life, usually occurring in the metaphyses of long bones, with the posterior aspect of the distal femur being the most frequent site (62% of cases). The prognosis for parosteal OS is better than conventional OS, with the 5-year overall survival rate 86–91% and 53–61%, respectively. At radiography, the classic appearance is a lobulated and exophytic mass with central dense ossification adjacent to the bone. The “string sign” portrays a thin radiolucent line separating the tumour from cortex. At MR imaging, the ossified tumor is predominantly low in signal intensity on both T1 weighted and T2 weighted images, similar to the appearance of the cortex. Differentials include benign entities such as osteochondroma, myositis ossificans, and periosteal chondroma and malignant entities such as fibrous malignancy and periosteal chondrosarcoma. Yarmish G, Klein MJ, Landa J, et al. Imaging characteristics of primary osteosarcoma: nonconventional subtypes. Radiographics 2010; 30: 1653–72.
1213
The following statement about calcium pyrophosphate deposition disease is not true: A. Primary hyperparathyroidism is a known cause B. It affects the metacarpophalangeal and radioulnar joints C. It typically causes hyaline cartilage calcification, which is irregular and thick D. It can exhibit symmetrical polyarthritis similar to rheumatoid arthritis E. It can exhibit subchondral cyst formation similar to osteoarthritis
C. It typically causes hyaline cartilage calcification, which is irregular and thick The calcification associated with calcium pyrophosphate deposition disease is typically fine and linear, following the contour of the bone. Irregular and thick calcification tends to be seen in fibrocartilage calcification. Danhert W. Radiology Review Manual. 6th edn. Philadelphia, PA: Lippincott Williams and Wilkins; 2007.
1214
A 21-year-old female had an inversion injury of her right ankle during a night out with friends. She was imaged with a radiograph initially in the accident and emergency department followed by an MRI a few weeks later. The following injury is not compatible with this mechanism of trauma: A. Oblique impaction fracture of medial malleolus B. Transverse avulsion fracture of lateral malleolus C. Injury to peroneus longus tendon D. Fracture base of fifth metatarsal E. Avulsion injury of deltoid ligament
E. Avulsion injury of deltoid ligament The first four answers are compatible with inversion injury. Eversion injury of the ankle can cause oblique impaction fracture of the lateral malleolus and avulsion of the medial deltoid ligament with or without avulsion fracture of the medial malleolus. Eversion injuries can also cause the force to be dissipated upwards through the interosseous ligament and cause a fracture of the proximal fibula (Maisonneuve fracture). Grainger RG, Allison DJ. Diagnostic Radiology. 5th edn. London, UK: Churchill Livingstone; 2007. pp. 1008–9.
1215
A 35-year-old male was brought into the accident and emergency department after a motor vehicle accident involving the car he was driving. The cervical spine radiograph showed an extension teardrop fracture of C2 vertebral body. He underwent CT imaging for further assessment. The following is the most likely associated injury in keeping with the above mentioned mechanism of injury and radiological findings: A. Jefferson’s fracture B. Hangman’s fracture C. Type I Dens fracture D. Type II Dens fracture E. Type III Dens fracture
B. Hangman’s fracture Hangman’s fracture is a traumatic spondylolyis of the posterior arch of C2 vertebra secondary to hyperextension injury. This is as a result of the head striking the steering wheel or the windscreen. It is associated with the hyperextension tear drop fracture which is an avulsion fracture of the anteroinferior margin of C2 vertebral body. The isolated hyperextension teardrop is a neurologically benign injury but associated Hangman’s fracture should be excluded. Brant WE, Helms C. Fundamentals of Diagnostic Radiology. 4th edn. Philadelphia, PA: Lippincott Williams and Wilkins; 2012. pp. 1105–11.
1216
A 4-year-old child presented with an episode of fever and generalized bone pain. On clinical examination he had a large skull and bowing of the both legs. Biochemical examination revealed normal inflammatory markers. However, alkaline phosphatase was markedly raised. A radiograph of his skull showed widening of the diploic space with areas of sclerosis. A radiograph of his femur showed cortical thickening and coarse trabecular pattern similar to Paget’s disease. Subsequent technetium-99m-methylene diphosphonate bone scan showed a superscan appearance. The most likely diagnosis is likely to be: A. Rickets B. Tumour induced rickets C. Vitamin D intoxication D. Hyperphosphatasia E. Hypervitaminosis A
D. Hyperphosphatasia Hyperphosphatasia is a genetic disorder characterized by increased rate of bone turnover. The radiographic features resemble Paget’s disease except that the whole skeleton is affected. Paget’s disease is rare before 40–50 years of age. Hypervitaminosis A in children can cause cupping and splaying of the metaphyses. Vitamin D intoxication causes metastatic calcification and bone sclerosis. Grainger RG, Allison DJ. Diagnostic Radiology. 5th edn. London, UK: Churchill Livingstone; 2007. p. 1109.
1217
The following soft tissue masses is not typically low signal on T2 weighted MRI: A. Pigmented villonodular synovitis B. Melanoma metastases C. Intramuscular abscess D. Amyloid deposits E. Morton’s neuroma
C. Intramuscular abscess Intramuscular abscess is typically low signal on T1 weighted images and high signal on T2 weighted images. Pigmented villonodular synovitis is low signal on T2 due to its haemosiderin content. Danhert W. Radiology Review Manual. 6th edn. Philadelphia, PA: Lippincott Williams and Wilkins; 2007.
1218
Which of the following is not associated with carpal tunnel syndrome? A. Diabetes B. Alcoholism C. Calcium pyrophosphate deposition disease D. Amyloidosis E. Acromegaly
B. Alcoholism Carpal tunnel syndrome is the most common peripheral neuropathy of the upper limb. It occurs predominantly in middle-aged women and results from compression of the median nerve beneath the flexor retinaculum. It has many associations as above and others like rheumatoid arthritis, gout, pregnancy, hypothyroidism, and mass lesions such as ganglia, lipomas, and neurofibromas. Alcoholism is not an association. Linda DD, Harish S, Stewart BG, et al. Multimodality imaging of peripheral neuropathies of the upper limb and brachial plexus 1. Radiographics 2010; 30: 1373–400.
1219
A routine screening isotope bone scan in a 66-year-old male with carcinoma of the prostate showed diffuse intense uptake in the proximal femur with a sharply demarcated lower edge. In addition there was similar intense uptake in L3 vertebral body and posterior elements and diffuse uptake in part of the frontal bone. The most likely diagnosis is: A. Metastases from prostate cancer B. Multifocal osteomyelitis C. Lymphoma D. Osteosarcoma E. Paget’s disease
E. Paget’s disease Bone scan is more sensitive than plain radiograph in detecting Paget’s disease and is useful to assess the extent of disease. It is also very useful in assessing the response to treatment. The typical appearances help differentiate it from the other options. Prostatic metastases are usually more focal and do not typically involve the whole of a vertebral body including the posterior elements. However it can occasionally be difficult to differentiate in the absence of typical findings. Peters AM. Nuclear Medicine in Radiological Diagnosis. London, UK: Informa Healthcare; 2003.pp. 87–99.
1220
A 70-year-old female presents to the accident and emergency department with left-sided chest pain. A chest X-ray done in casualty showed multifocal expansile lytic lesions involving the left sixth and seventh ribs. A large soft tissue mass was seen on the left lateral chest wall at the level of the rib lesions. She was also found to have an expansile lytic lesion on the proximal diaphysis of left femur on a previous X-ray diagnosed as fibrous dysplasia. She had also noticed some associated soft tissue swelling in the inner left thigh at the level of the proximal femoral lesion. MRI of the soft tissue lesion showed signal characteristics resembling a cyst, but with patchy internal enhancement following intravenous gadolinium. Which of the following is the most likely diagnosis? A. Ollier’s disease B. Mazabraud’s syndrome C. Maffucci syndrome D. Multiple enchondromatosis E. Myositis ossificans
B. Mazabraud’s syndrome The patient had polyostotic fibrous dysplasia with associated soft tissue myxomas. Monostotic or polyostotic fibrous dysplasia associated with single or multiple soft tissue myxomas is described in Mazabraud syndrome. Case DB, Chapman CN Jr, Freeman JK, et al. Best cases from the AFIP: atypical presentation of polyostotic fibrous dysplasia (Mazabraud syndrome). Radiographics 2010; 30: 827–32.
1221
The following is not a typical cause of Erlenmeyer flask deformity: A. Gaucher’s disease B. Fibrous dysplasia C. Paget’s disease D. Lead poisoning E. Pyle’s disease
C. Paget’s disease Erlenmeyer flask deformity has typical radiographic appearances of relative constriction of the diaphysis and flaring of the metaphysis. There are many causes for it but Paget’s disease is not one of them. Danhert W. Radiology Review Manual. 6th edn. Philadelphia, PA: Lippincott Williams and Wilkins; 2007.
1222
Which of the following options describes the possible appearances of the epicondylitis (tennis elbow) on ultrasound? A. Intratendinous thickening with calcification and diffuse heterogeneity of the common extensor origin B. Avulsion fracture of the medial epicondyle C. Hypervascularity and swelling of the common flexor origin D. Debris within the elbow joint E. Hypoechoic and hypervascular triceps tendon, with swelling of the olecranon bursa
A. Intratendinous thickening with calcification and diffuse heterogeneity of the common extensor origin Lateral epicondylitis is said to result from repetitive stress injury to the common extensor origin. Typical findings on ultrasound include intra-tendinous thickening with calcification and diffuse heterogeneity as described above. Walz D, Newman JS, Konin GP, et al. Epicondylitis: pathogenesis, imaging and treatment. Radiographics 2010; 30: 167–84.
1223
A 15-year-old boy with right arm pain has an X-ray of the humerus that shows a 4 cm lucent lesion in the humeral diaphysis, which demonstrates cortical expansion and reactive sclerosis. Which of the following options is the most likely diagnosis? A. Simple bone cyst B. Melorheostosis C. Paget’s disease D. Aneurysmal bone cyst E. Osteoblastoma
E. Osteoblastoma Osteoblastomas have similar histological characteristics to osteoid osteomas but are larger, and tend to be more destructive. An osteoid osteoma is a highly vascular, painful benign tumour, which usually affects the long bones (8–10% spine involvement with >95% cases involving the neural arch). Typical CT appearance of a small hypodense nidus, with or without central calcification and surrounding bone sclerosis. Osteoblastoma is also benign. Chai JW, Hong SH, Choi JY, et al. Radiologic diagnosis of osteoid osteoma: from simple to challenging. Radiographics 2010; 30: 737–49. Kurugoglu S, Adaletli I, Mihmanli I, et al. Lumbosacral osseous tumors in children. Eur J Radiol 2008; 65: 257–69.
1224
A 70-year-old diabetic male underwent an MRI of the foot to exclude osteomyelitis. Which of the following characteristics favours osteomyelitis rather than neuropathy on MRI? A. Marrow signal abnormality involving the tarsometatarsal and metatarsophalangeal joints B. Marrow signal abnormality involving the bones distal interphalangeal joints and calcaneus C. Presence of a joint effusion D. Presence of periarticular involvement E. Rockerbottom foot deformity
B. Marrow signal abnormality involving the bones distal interphalangeal joints and calcaneus Osteomyelitis of the foot mostly occurs distal to the tarsometatarsal joints, in the calcaneus and both malleoli. The presence of ulceration also favours osteomyelitis rather than neuropathy. Rockerbottom foot can be seen in both conditions. Donovan A, Schweitzer M. Use of MRI in diagnosing diabetes-related osteomyelitis. Radiographics 2010; 30: 723–36.
1225
A 65-year-old noninsulin dependent diabetic presents to the clinic with an ulcer in his left great toe. He is also pyrexial with raised inflammatory markers. An MRI of the left foot is carried out to exclude underlying osteomyelitis. Which of the following findings is the most reliable sign for diagnosing osteomyelitis rather than mean reactive oedema. A. Confluent low signal marrow changes on T1 weighted B. Confluent high signal marrow changes on T2 weighted fat sat C. Patchy low signal marrow changes on T1 weighted D. Marked low signal marrow changes on T2 weighted E. Confluent high signal marrow changes on gradient echo
A. Confluent low signal marrow changes on T1 weighted Confluent low signal marrow change on T1 is indicative of osteomyelitis. High signal marrow changes on T2 and T2 fat sat are also seen in osteomyelitis but are less specific for this condition. Marrow may be hyperintense on T2 in osteitis arising from overlying soft tissue or cortical infection; however, T1 low signal marrow change is not seen in osteitis. Osteomyelitis in the diabetic foot tends to occur distal to the tarsometatarsal joints, the calcaneus and both malleoli. Donovan A, Schweitzer M. Use of MRI in diagnosing diabetes-related osteomyelitis. Radiographics 2010; 30: 723–36.
1226
A three month old male is brought to A&E by their parents. A history is given of apparent reduced use of the right arm for 48 hours with inconsolable crying. The child is alert and appears systemically well with no neurological deficit. There is skin bruising over the right shoulder and scapula. X-ray reveals a spiral fracture of the right proximal humerus. What is the most appropriate subsequent imaging protocol to undertake? * A - X-ray of the right radius and ulna * B - Skeletal survey * C - Skeletal survey and unenhanced CT head * D - Skeletal survey and MRI brain * E - Skeletal survey, MRI brain and whole spine
C - Skeletal survey and unenhanced CT head
1227
An 8-year-old boy complains of pain following a sports injury where he received a kick to the scrotum. On ultrasound the scrotal skin appears thickened, the epididymis swollen and on colour Doppler imaging the flow within the left testicle is markedly increased. What is the most likely diagnosis? * A - Bell-clapper effect * B - Testicular torsion * C - Epididymo-orchitis * D - Non seminomatous germ cell tumour * E - Reverse torsion
E Reverse torsion
1228
A preterm neonate is admitted to the intensive care unit in an unstable condition. The surgical team suspect an intestinal perforation. A supine AP x- ray of the abdomen is undertaken but the appearances are equivocal. What further imaging would be most appropriate in confirming the clinical suspicion? * A - Ultrasound abdomen * B - Right lateral decubitus abdominal radiograph * C - Horizontal beam lateral shoot through abdominal radiograph * D - CT abdomen * E - Erect chest x-ray
C - Horizontal beam lateral shoot through abdominal radiograph
1229
A 12 month old child is being referred for multiple abdominal ultrasound studies annually to exclude an underlying malignancy. You notice the child has hemihypertrophy and macroglossia, and on imaging a solitary heterogenous echogenic mass in the liver is present. What is the most likely diagnosis? * A. Hepatic adenoma in Familial Adenomatous Polyposis * B. Hepatoblastoma in Beckwith-Wiedemann Syndrome * C. Hepatic haemangioma in PHACES * D. Hepatic angiosarcoma in Li Fraumeni Syndrome * E. Hepatic AML in Tuberous Sclerosis
* B. Hepatoblastoma in Beckwith-Wiedemann Syndrome
1230
An 8 year old is brought to A&E by air ambulance after being hit by a car at 30 mph. Their GCS is 15. They have vomited four times since the incident. They are complaining of midline neck tenderness, abdominal pain and shortness of breath. There is bruising across the chest and abdomen suggesting significant blunt injury. The patient’s observations are stable. Which of these investigations should not form part of this child’s imaging protocol? * A - CT Head * B - CT Chest * C - CT Abdomen and Pelvis * D - Chest x-ray * E - Cervical spine x-ray
* B - CT Chest
1231
A child presents to A&E after trauma to the elbow. Which configuration of the following ossification centres is the most concerning for traumatic injury? * A - Internal epicondyle before trochlea * B - Capitellum before radial head * C - Olecranon before trochlea * D - Trochlea before internal epicondyle * E - Olecranon before lateral epicondyle
* D - Trochlea before internal epicondyle
1232
A 12 year old female presents to A&E with lower abdominal pain. She is known to have uterine didelphysis from a previous study. Ultrasound reveals a large rounded echogenic mass-like lesion within the midline of the lower abdomen, associated with the right uterine cavity. Incidental note is made of an absent right kidney. MRI confirms presence of a T1-bright rounded lesion within the right uterus and hemivagina. What is the most likely diagnosis? * A - OHVIRA Syndrome * B - Wunderlich Syndrome * C - VACTERL association * D - Endometriosis * E - Mayer-Rokitansky-Kuster-Hauser Syndrome
* A - OHVIRA Syndrome Also known as OHVIRA syndrome * Obstructed Hemi Vagina with Ipsilateral Renal Anomalies * Uterine didelphys * Usually associated with a transverse septum across the hemivagina, resulting in obstruction of the vagina, which can extend to the uterine cavity and fallopian tubes * Renal agenesis is usually the associated renal abnormality
1233
A 3 month old child presents with a palpable abdominal mass. Ultrasound reveals a large left-sided flank mass demonstrating ‘claw sign’ with the left kidney. The lesion is mixed cystic and solid, with areas of colour Doppler flow within. There is contact within the main renal vasculature. Whole body MRI does not reveal any metastatic disease. What is the most likely diagnosis? * A - Nephroblastoma * B - Mesoblastic nephroma * C - Rhabdoid tumour * D - Renal Cell Carcinoma * E - Cystic nephroma
* B - Mesoblastic nephroma
1234
You are reporting a plain film from the GP reporting pile. A 7 year old has a lower leg x-ray due to shin pain. This demonstrates a lobulated cortically-based lucent lesion within the anterior tibial diaphysis. This has a thick sclerotic margin measuring 7 cm in craniocaudal length. There is no significant periosteal reaction or cortical thickening. MRI demonstrates the lesion encroaches upon the medullary cavity, but does not infiltrate it completely. What is the most likely diagnosis? * A - Osteoid osteoma * B - Osteofibrous dysplasia * C - Adamantinoma * D - Non-ossifying fibroma * E - Fibrous cortical defect
* B - Osteofibrous dysplasia Age * Adamantinoma tends to be second and third decades * OFD before the age of 10 * Size * Adamantinoma often 10 cm or more * OFD mean 6-7 cm * Medullary involvement * Adamantinoma more likely to involve the medullary cavity
1235
A 7 month old baby is seen in the general paediatric clinic with failure to thrive and irritability. Few clinical signs are present but the child cries on manipulation of the lower limbs. X-ray was performed for suspected fracture but reveals generalised osteopenia and a sclerotic rim surrounding the distal femoral epiphysis, which is abnormally lucent. There is high density material running along the metaphysical cortices. What is the most likely diagnosis? * A - Hypoparathyroidism * B - Scurvy * C - Rickets * D - Hypothyroidism * E - Hypophosphatasia
* B - Scurvy Hypovitaminosis C * Uncommon in the UK, but typically seen between 6 to 9 months of age if present * Wimberger Ring Sign * Metaphyseal cortical haematomas * Pelkan Spurs * Osteopenic changes
1236
A 16 year old boy presents for MR urogram after recurrent urinary tract infections. This demonstrates duplicated ureters. Which of the following statements is correct regarding ureteric duplication? * A - The lower pole moiety inserts inferior to the upper pole moiety * B - The lower pole moiety is the more likely to obstruct * C - The upper pole moiety is often associated with an ectopic ureterocele * D - The upper pole moiety inserts horizontally and is associated with reflux * E - Calyceal dilatation is usually seen in the lower pole
C - The upper pole moiety is often associated with an ectopic ureterocele
1237
A 5 month old infant presents with one episode of urinary tract infection. The patient recovers and urinalysis reveals E.coli as the causative organism. There is no family history of vesicoureteric reflux or renal disease. What are the next imaging steps? * A - Urgent urinary tract US * B - Routine urinary tract US within six weeks * C - Routine urinary tract US with DMSA in 4-6 months * D - Routine US, DMSA and MCUG * E - MR cystourethrogram
* B - Routine urinary tract US within six weeks
1238
A 4 year old child falls in the playground. X-ray demonstrates no cortical break, but presence of a posterior fat pad along the distal humerus. What is the most likely underlying pathology? * A - Supracondylar fracture of the humerus * B - Osteomyelitis * C - Radial head fracture * D - Septic arthritis with effusion * E - Clinoid process fracture of the ulna
* A - Supracondylar fracture of the humerus
1239
A neonate is referred to the imaging department for further assessment of an incidentally detected sacral dimple at the natal cleft during initial ‘baby check’. This is blind ending without any associated hair growth or palpable mass. What is the next most appropriate imaging step? * A - No further imaging required * B - US of the spine * C - US of the cranial contents and spine * D - MRI of the spine * E - MRI of the brain and spine
* A - No further imaging required
1240
A neonate presents with non-bilious vomiting and a palpable abdominal lump. Which of the following US findings would not be in keeping with pyloric stenosis? * A - Single pyloric muscular wall measurement of 4 mm * B - Target sign in the transverse plane * C - Pyloric canal length of 14 mm * D - Antral nipple sign * E - Cervix sign
* C - Pyloric canal length of 14 mm
1241
A 2 year old boy presents to ED with acute wheeze. Foreign body aspiration is suspected. Which of the following imaging should be performed? Frontal CXR and decubitus views Frontal CXR and right anterior and left anterior oblique views Frontal CXR and lordotic view Frontal CXR and lateral CXR Frontal CXR and horizontal beam lateral CXR
Frontal CXR and decubitus views Frontal CXR and right anterior and left anterior oblique views – NAI Frontal CXR and lordotic view – rarely used in paediatrics, for lung apices Frontal CXR and lateral CXR – rarely used at our institution Frontal CXR and horizontal beam lateral CXR – for anterior pneumothorax
1242
A 6 month old boy presents with difficulty swallowing and failure to thrive. A barium contrast study demonstrates anterior indentation of the thoracic oesophagus. What is the most likely diagnosis? A. Innominate artery compression syndrome B. Right sided aortic arch with aberrant left subclavian artery C. Pulmonary sling D. Patent ductus arteriosus E. Coarctation of the aorta
C. Pulmonary sling Innominate artery compression syndrome – no oesophageal impression, indents anterior trachea Right sided aortic arch with aberrant left subclavian artery – posterior indentation on oesophagus Patent ductus arteriosus – no oesophageal impression Coarctation of the aorta – no oesophageal impression
1243
An 8 week old term neonate with an antenatally diagnosed abnormality has a CXR demonstrating a multicystic gas-filled lesion of the left lung, with mediastinal shift. The diaphragm is intact. A contrast-enhanced CT is performed. What is the most common association with this lesion? A. Partial anomalous pulmonary venous drainage B. Congenital lobar overinflation C. Bronchogenic cyst D. Scimitar syndrome E. Pulmonary sequestration
E Pulmonary sequestration
1244
An 8 year old girl with a chronic condition has a yearly ultrasound abdomen and CXR. She is currently well. CXR demonstrates widespread bronchiectasis and a Port-A-Cath in situ. What is the most likely abdominal finding? A. Adrenal calcification B. Free fluid C. Hyperechoic pancreas D. Absent spleen E. Thickened bowel loops
C. Hyperechoic pancreas
1245
A premature neonate presents with tachycardia ,expiratory grunting, nasal flaring and intercostal recession. A diagnosis of respiratory distress syndrome is given. What would the CXR show? Increased lucency within the left upper zone with mediastinal displacement to the right. Hyperinflated lungs with asymmetrical patchy opacities. Reduced lung volumes and widespread granular lung opacities. Perihilar interstitial oedema and small pleural effusions Hyperinflated lungs. Cystic and linear radiolucencies radiating from the hilum.
Reduced lung volumes and widespread granular lung opacities. A Increased lucency within the left upper zone with mediastinal displacement to the right – Congenital lobar emphysema B Hyperinflated lungs with asymmetrical patchy opacities – Meconium aspiration D. Perihilar interstitial oedema and small pleural effusions – Transient tachypnoea of the newborn E. Hyperinflated lungs. Cystic and linear radiolucencies radiating from the hilum – Pulmonary interstitial emphysema
1246
A 4 year old child with a known malignancy has a CT Chest which demonstrates multiple pulmonary metastases. Which of the following best describes the most likely primary tumour? Midline fourth ventricle tumour on MRI A large solid left sided mass which demonstrates stippled calcification on CT Permeative lesion of the distal femur with sunburst periosteal reaction on plain radiograph A large solid left sided mass which demonstrates the renal ‘claw sign’ on US Well defined lytic skull lesion on plain radiograph
A large solid left sided mass which demonstrates the renal ‘claw sign’ on US A. B. C. D. Midline fourth ventricle tumour on MRI – posterior fossa tumour (medulloblastoma, ependymoma etc) A large solid left sided mass which demonstrates stippled calcification on CT – neuroblastoma Permeative lesion of the distal femur with sunburst periosteal reaction on plain radiograph – osteosarcoma Well defined lytic skull lesion on plain radiograph – LCH
1247
A full term 3 week old infant, born by forceps delivery, presents with a left-sided hard neck mass, and torticollis. Otherwise the child is well. What is the most likely next imaging study and diagnosis? A MRI and Rhabdomyosarcoma B US Neck and Fibromatosis Colli C US Neck and Branchial cleft cyst D MRI and Neuroblastoma E MRI and Thyroglossal duct cyst
B US Neck and Fibromatosis Colli
1248
A 6 year old boy presents with a posterior fossa mass on MRI. A diagnosis of medulloblastoma is made. What is the most likely set of MRI findings? A Midline fourth ventricular tumour extending to the left cerebellopontine angle, heterogenous signal on T1 and T2 and heterogenous enhancement B Cystic left cerebellar hemisphere tumour with enhancing solid nodule C Midline fourth ventricular tumour, T2 and FLAIR hyperintense with diffusion restriction and avid enhancement D Diffuse infiltrating pontine mass with heterogenous enhancement E Heterogenous ‘popcorn’ lesion with T2 hypointense rim in the left cerebellar hemisphere
C Midline fourth ventricular tumour, T2 and FLAIR hyperintense with diffusion restriction and avid enhancement A Midline fourth ventricular tumour extending to the left cerebellopontine angle, heterogenous signal on T1 and T2 and heterogenous enhancement – ependymoma B Cystic left cerebellar hemisphere tumour with enhancing solid nodule – pilocytic astrocytoma D Diffuse infiltrating pontine mass with heterogenous enhancement – diffuse intrinsic pontine glioma E Heterogenous ‘popcorn’ lesion with T2 hypointense rim in the left cerebellar hemisphere – cavernoma
1249
A 4 year old boy presents with headache. A CT reveals a complex partially cystic partially calcified suprasellar lesion. What is the most likely diagnosis? A Subependymal giant cell astrocytoma (SEGA) B Rathke’s cleft cyst C Pituitary macroadenoma D Craniopharyngioma E Pituitary apoplexy
D Craniopharyngioma
1250
A 3 year old presents with decreased level of consciousness following a bout of viral gastroenteritis. A diagnosis of Acute Disseminated Encephalomyelitis (ADEM) is made on MRI. Which of the following best describes the radiological features? cB Several high T2 lesions in the deep white matter and callososeptal interface C Ill defined area of T2 hyperintensity of the left temporal lobe with haemorrhagic components, sparing the deep grey matter D Bilateral vasogenic oedema of the parieto-occipital lobes E T2 hyperintensity and diffusion restriction of the lower pons
A Several asymmetric high T2 lesions in the deep white matter and thalami B Several high T2 lesions in the deep white matter and callososeptal interface – MS C Ill defined area of T2 hyperintensity of the left temporal lobe with haemorrhagic components, sparing the deep grey matter – HSV encephalitis D Bilateral T2 hyperintensity of the parieto-occipital lobes and posterior limb of the internal capsule –hypoglycaemia E T2 hyperintensity and diffusion restriction of the lower pons – central pontine myelinolysis
1251
A 3 year old presents with a painful subgaleal mass, painful right arm and increased thirst and polyuria. A radiograph demonstrates well defined lytic lesions of the skull and humeral diaphysis. What would MRI brain demonstrate? A Well defined enhancing suprasellar mass touching the optic chiasm B Thickening of the pituitary stalk and loss of the posterior pituitary T1 bright spot C Multiple well defined enhancing brain metastases D Fourth ventricular mass with diffuse leptomeningeal thickening and enhancement E Scattered T2 bright lesions with fusiform enlargement and kinking of the optic nerves
B Thickening of the pituitary stalk and loss of the posterior pituitary T1 bright spot A Well defined enhancing suprasellar mass touching the optic chiasm – pituitary macroadenoma C Multiple well defined enhancing brain metastases – rare in LCH, does not explain polyuria and polydipsia D Fourth ventricular mass with diffuse leptomeningeal thickening and enhancement – medulloblastoma E Scattered T2 bright lesions with fusiform enlargement and kinking of the optic nerves – NF1
1252
A 4 week old baby presents with seizures. A CT is performed which demonstrates mild right cerebral atrophy, subcortical tram-track calcification and ipsilateral choroid plexus enlargement. What diagnosis do these features suggest? A Tuberous Sclerosis B Sturge Weber syndrome C Von Recklinghausen disease D Neurofibromatosis Type 2 E Von Hippel Lindau disease
B Sturge Weber syndrome A Tuberous Sclerosis – cortical and subcortical tubers, subependymal nodules, SEGA C Von Recklinghausen disease – focal T2 lesions in deep WM and basal ganglia, optic nerve glioma, sphenoid wing dysplasia D Neurofibromatosis Type 2 – meningioma, schwannoma, ependymoma E Von Hippel Lindau disease – Haemangioblastoma (cystic posterior fossa tumour with enhancing mural nodule)
1253
A 2 year old boy presents with multiple bony metastases. What is the next most likely imaging modality and diagnosis? A US Abdomen and LCH B US Abdomen and Wilms tumour C Plain radiograph and Osteosarcoma D US Abdomen and Neuroblastoma E MRI Brain and Medulloblastoma
D US Abdomen and Neuroblastoma
1254
A 3 year old boy presents with left sided limp. An XR reveals slight flattening and irregularity of the left upper femoral epiphysis. Blood tests are normal. What is the most likely diagnosis? A Legg-Calve-Perthes disease B Slipped upper femoral epiphysis C Developmental dysplasia of the hip D Septic arthritis E Osteomyelitis
A Legg-Calve-Perthes disease
1255
A 10 year old child presents with an ankle fracture. The fracture involves the distal tibial metaphysis, physis and epiphysis. Which of the following best describes the fracture? A. Salter Harris I B. Salter Harris II C. Salter Harris III D. Salter Harris IV E. Salter Harris V
D. Salter Harris IV
1256
A 9 year old child presents with a chest pain. A CXR demonstrates a destructive mass of the 1st rib with a large soft tissue component. What is the most likely diagnosis? A. LCH B. Osteosarcoma C. Rhabdomyosarcoma D. Caffey’s disease E. Ewing’s sarcoma
E. Ewing’s sarcoma
1257
A 36 year old female patient originally presented to her GP with difficulty swallowing solids and liquids, associated chest discomfort and occasional episodes of regurgitation. Barium swallow helps to obtain the diagnosis. There is smooth distal oesophageal tapering with proximal oesophageal dilatation and tertiary contractions. This is successfully treated at the time but 19 years later the same patient presents with dysphagia again. The barium swallow now demonstrates an irregular, shouldered narrowing with proximal oesophageal dilatation. Endoscopy confirms malignancy. Where is the new narrowing most likely to be sited? a. Cervical oesophagus b. Distal oesophagus c. Mid-oesophagus d. Mid and distal oesophagus e. Proximal stomach
c) Mid-oesophagus The patient was originally suffering with achalasia leading to dysphagia to both solids and liquids, which can be relieved with warm fluids. Barium swallow typically causes a characteristic ‘rat-tail’ narrowing of the distal oesophagus with associated oesophageal dilatation, an oesophageal air-fluid level and tertiary contractions. Warm water can be administered during the barium study to demonstrate relaxation of the lower oesophageal sphincter. Complications from achalasia include aspiration, pneumonia, oesophagitis and also malignancy. Approximately 5% of patients with achalasia develop oesophageal carcinoma and these strictures tend to occur in the mid-oesophagus. Most idiopathic oesophageal carcinomas are squamous cell carcinomas and these similarly arise in the mid-oesophagus. Oesophageal adenocarcinoma is less common but frequently related to Barrett oesophagus and arises distally at the gastro-oesophageal junction, therefore more commonly involving the proximal stomach.
1258
A 42 year old female patient, undergoing long-term peritoneal dialysis, has an abdominal ultrasound for left-sided flank pain with haematuria. The kidneys measure up to 5.5 cm in bipolar length with cortical thinning. There is no hydronephrosis or renal calculi. There are several bilateral renal lesions. These are small, exophytic, anechoic and well defined. One lesion on the left side has internal echoes and dependent debris. There is a moderate amount of free fluid in the abdomen and pelvis. What is the most likely underlying diagnosis causing the renal lesions? a. Acquired cystic kidney disease b. Autosomal dominant polycystic kidney disease c. Autosomal recessive polycystic kidney disease d. Idiopathic bilateral renal cysts e. Tuberous sclerosis
(a) Acquired cystic kidney disease Acquired cystic kidney disease is associated with end stage renal failure and can affect patients having either peritoneal or haemodialysis. The condition becomes much more common the longer a patient has been on dialysis, with 90% of patients estimated to have developed it after a decade. The typical features include small kidneys and >3–5 cysts in each kidney. Unfortunately, there can be complications such as cyst haemorrhage – as has happened to this patient. There is also a risk of renal cell carcinoma; findings such as solid nodules and septations would warrant further assessment and this risk continues in the native kidneys following transplantation. Autosomal dominant polycystic kidney disease is the most common hereditary cause of end stage renal failure; however the kidneys usually appear enlarged due to the innumerable cysts. The cysts can be variable in size and can become very large rather than the small cysts described in the question. Autosomal recessive polycystic kidney disease is a paediatric condition which tends to cause enlarged kidneys with a lot of small cysts and is also associated with hepatic fibrosis. Tuberous sclerosis is associated with renal cysts as well as angiomyolipomas, which can also haemorrhage; however the most likely cause is acquired cystic kidney disease in the context of end stage renal failure.
1259
A CT head is performed on a toddler who has tripped at home, hitting his head. The paediatric team are concerned about a possible seizure following the event and a couple of episodes of vomiting. The scan shows no intracranial haemorrhage. However, there is a hypodense posterior fossa lesion. An MRI more clearly demonstrates that this is in the posterior midline of the posterior fossa displacing the cerebellum anteriorly. The tentorium cerebelli and cerebellar vermis appear intact and have normal appearances. There is no hydrocephalus and the fourth ventricle is within normal limits. There is no significant enhancement following contrast administration. There is no restricted diffusion. On a FLAIR sequence the lesion is isointense to cerebrospinal fluid. Which diagnosis is most likely? a. Dermoid cyst b. Epidermoid cyst c. Ependymal cyst d. Pilocytic astrocytoma e. Mega cisterna magna
e) Mega cisterna magna Mega cisterna magna is usually an incidental diagnosis which causes the appearances of a cystic midline posterior fossa lesion located posteriorly and displacing the cerebellum. It follows cerebrospinal fluid (CSF) on all MRI sequences. An arachnoid cyst would be another possible differential as it has the same signal characteristics, although the position is suggestive of mega cisterna magna. This appearance can be detected antenatally and in these cases can be associated with infections such as cytomegalovirus or chromosomal abnormalities. An epidermoid cyst has similar appearances to an arachnoid cyst on imaging apart from on DWI and FLAIR sequences where there will be restricted diffusion and FLAIR hyperintensity compared to CSF. Dermoid cysts are usually in the midline but they characteristically contain fat and will therefore have high T1 signal. If they rupture they may cause a chemical meningitis. Ependymal cysts also have similar imaging features to arachnoid cysts but are usually deep within the parenchyma or associated with the ventricles, being either peri or intraventricular. Pilocytic astrocytoma is an important differential for midline paediatric posterior fossa lesions. They usually have a large cystic component with an enhancing solid nodule.
1260
A 53 year old diabetic patient presents with right shoulder pain and reduced range of movement which does not improve despite community physiotherapy. The patient is reviewed in the orthopaedic outpatient clinic and an MRI shoulder arthrogram is requested. Which of the options below is most consistent with adhesive capsulitis?
(a) Adhesive capsulitis is also known as frozen shoulder and classically presents with restriction in shoulder elevation and external rotation. It is most common in middle-aged women, particularly diabetic patients, and can also be associated with previous trauma. Classical radiological features are a thickened joint capsule and coracohumeral ligament. The subscapularis bursa is small, and lymphatic filling is a feature. The subcoracoid fat triangle between the coracohumeral ligament and coracoid process can be obliterated. The joint volume is reduced and therefore there is limited filling capacity during arthrogram injection.
1261
A 56 year old male presents with gradual onset right upper quadrant pain. An ultrasound examination is performed which demonstrates absence of Doppler flow in the hepatic veins. Budd-Chiari syndrome is suspected. Which of the following imaging features differentiates chronic from acute Budd-Chiari? a. Hepatomegaly b. Hypertrophied caudate lobe c. Heterogeneous hepatic echotexture d. Splenomegaly e. Ascites
(b) Hypertrophied caudate lobe In the chronic phase of Budd-Chiari there is compensatory enlargement of the caudate lobe as it drains directly into the inferior vena cava. There is associated atrophy of the peripheral segments, and regenerative nodules may be a feature. In the acute presentation there is classically hepatosplenomegaly and ascites. CT can demonstrate a ‘flip-flop’ pattern of enhancement due to hyperenhancement of the central liver segments on early-phase images, with peripheral hypoenhancement. This appearance is reversed on delayed phase images.
1262
A 5 year old girl has an obvious deformity affecting the right side of her upper back and shoulder with a visible ‘bump’. She has spinal imaging which detects fusion of the C2-C4 vertebrae. The patient has also had an MRI brain and spine. Given the other findings, which of the following is most likely to be found on the MRI brain? a. Chiari I malformation b. Haemangioblastoma c. Holoprosencephaly d. Optic glioma e. Polymicrogyria
(a) Chiari I malformation The patient has characteristic features of Klippel-Feil syndrome with Sprengel deformity of the shoulder and cervical vertebral fusion. Chiari I malformation is associated with Klippel-Feil syndrome and is when the cerebellar tonsils descend ≥5 mm below the level of the foramen magnum. It is often asymptomatic and found incidentally in adults. As well as being associated with Klippel- Feil syndrome, it is also associated with craniosynostosis, hydrocephalus, syrinx and a tethered cord. Haemangioblastomas are a cause of predominantly posterior fossa cystic lesions and are associated with von Hippel-Lindau. They often have an enhancing solid nodule and flow voids. Optic gliomas are associated with neurofibromatosis type I, along with pilocytic astrocytoma, sphenoid wing dysplasia and focal areas of parenchymal T2 signal hyperintensity. Holoprosencephaly is a cleavage failure and appearances range from severe (alobar) to less severe (lobar). Polymicrogyria is associated with a number of infections during pregnancy including toxoplasmosis, rubella and cytomegalovirus.
1263
An oncology patient has an MRI spine due to increasing back pain. Apart from mild generalised motor weakness he has no significant neurology. The MRI identifies multiple areas of abnormal T1 and T2 signal which are hypointense compared to the adjacent disc. There is heterogenous high STIR signal in these regions and restricted diffusion. There is no significant narrowing of the canal or neural foramina. The cord returns normal signal. What is the most likely underlying primary site of malignancy? a. Colorectal carcinoma b. Melanoma c. Non-small cell lung carcinoma d. Prostate carcinoma e. Renal cell carcinoma
(d) Prostate carcinoma Vertebral metastases, when diffuse, can be difficult to appreciate. It is always important to compare the vertebral body signal to the adjacent disc. The signal of metastases depends on whether they are sclerotic or lytic. STIR signal is usually high and they demonstrate restricted diffusion. Lytic metastases would be low on T1 and high on T2 compared to the disc. However, sclerotic metastases would be low on both sequences. The metastases in this case are consistent with sclerotic lesions and therefore the most likely primary site is prostate cancer. The other cancers are more commonly associated with lytic metastases. Melanoma is unusual in that it can demonstrate high T1 signal due to melanin.
1264
A 35 year old male patient presents to the emergency department with atraumatic right hip pain for 10 days; he is otherwise systemically well. A radiograph shows mild osteopenia in the right femoral head. A previous abdominal radiograph from 2 years ago included the right hip, which looked normal at that time, but you notice foci of calcification projected over the upper abdominal quadrants bilaterally and generalised osteosclerosis. What MRI finding in the proximal femur would be most specific for the correct diagnosis? a. Adjacent subchondral hypo and hyperintense T2 linear signal b. Low T1 signal at the medial aspect of the femoral neck c. Low T1 signal line parallel to the subarticular cortex d. Synovial enhancement with gadolinium e. Subchondral high STIR signa
a) Adjacent subchondral hypo- and hyperintense T2 linear signal The patient has features of sickle cell disease on abdominal radiograph with osteosclerosis, likely calcified gallstones in his right upper quadrant and a calcified spleen in the left upper quadrant. Therefore, the most likely diagnosis is avascular necrosis of the hip. Radiographic features are often not visible, or subtle in the early stages, and osteopenia may be present. It is only later in the disease process that sclerosis, lucency, irregularity and flattening of the femoral heads develop. MRI is the most sensitive test for diagnosis and the most specific sign is the ‘double line’ sign which is paired high and low T2 linear signal. The low T1 signal parallels the subarticular cortex, also known as the ‘crescent’ sign − representing a subchrondral fracture. This would likely be associated with subchrondral high STIR signal in the acute phase. Low signal at the medial aspect of the femoral neck can be seen in stress fractures, which typically affect young patients; however the clinical findings are more in keeping with sickle cell anaemia and subsequent avascular necrosis. Synovial enhancement with gadolinium is not typical for avascular necrosis but would be seen in septic arthritis, which patients with sickle cell disease are at higher risk of; however you would also expect systemic illness.
1265
A 22 year old male medical student reports to his GP with recurrent spontaneous epistaxis as well as fatigue and increasing dyspnoea. The patient is found to be tachycardic and tachypnoeic. The GP is concerned and in view of the patient’s significant family history refers him to be seen by the on-call medical team at hospital. The medical team organise an urgent chest radiograph and perform an arterial blood gas which demonstrates hypoxia. They refer the patient to you for a CT pulmonary angiogram to exclude a pulmonary embolus. On review of the plain film, you confirm the presence of multiple pulmonary nodules and the heart appears large. There is no other significant abnormality. What is the most likely underlying diagnosis? a. Caplan syndrome b. Goodpasture's syndrome c. Granulomatosis with polyangiitis d. Multiple metastases e. Osler-Weber-Rendu syndrome
(e) Osler-Weber-Rendu syndrome The most likely diagnosis is Osler-Weber-Rendu syndrome, or hereditary haemorrhagic telangiectasia. It is characterised by multiple arteriovenous malformations (AVMs) with a triad of telangiectasia, epistaxis (due to nasal telangiectasia) and a positive family history (autosomal dominant). The pulmonary nodules seen on plain film are multiple AVMs. The AVMs cause cyanosis and can cause high output heart failure. Although multiple pulmonary nodules are seen in granulomatosis with polyangiitis (previously known as Wegener granulomatosis), the associated upper respiratory tract symptom is sinusitis rather than epistaxis, and cyanosis is not usually a feature. Caplan syndrome is a combination of rheumatoid arthritis and pneumoconiosis. Although lung nodules are seen in this condition, there is no relevant history provided in the main stem, and there is no evidence of interstitial abnormality provided on plain film. Goodpasture syndrome may present with haemoptysis; however, pulmonary nodules are not a typical feature. Multiple metastases is a possible answer; however it is not the most likely from the information provided, especially given the patient’s age.
1266
A 55 year old female patient has a lateral cervical spine radiograph requested by the rheumatology clinic. On the lateral view, the tip of the odontoid process of C2 sits at the basion-opisthion line. Which of the conditions below is NOT associated with this appearance? a. Achondroplasia b. Fibrous dysplasia c. Osteopetrosis d. Paget disease e. Rheumatoid arthritis
(c) Osteopetrosis Basilar invagination is where the odontoid process of C2 sits at or above the level of the basion- opthision line which is also known as the McRae line. It should sit >5 mm below this line and it can lead to compression of the brainstem and subsequent hydrocephalus, syringomyelia and neurological symptoms. Causes can be acquired or congenital, and a common mnemonic is PF ROACH: Paget disease Fibrous dysplasia Rheumatoid arthritis, rickets Osteogenesis imperfecta, osteomalacia Achondroplaisa Chiari I/II, cleidocranial dysostosis Hyperparathyroidism Osteopetrosis is not associated with basilar invagination.
1267
A GP registrar calls for advice regarding an 11 month old girl following the diagnosis of her first urinary tract infection. This has been confirmed on urine culture as positive for Escherichia coli which is sensitive to trimethroprim. The patient has had 2 days of oral antibiotics at home and on review again today at the surgery she is improving. The GP registrar asks for your advice regarding further management. What is the most appropriate advice? a. Paediatric referral b. Ultrasound within 6 weeks c. Ultrasound and DMSA within 6 weeks d. Ultrasound and DMSA if recurrent infection e. Urgent ultrasound
(d) Ultrasound and DMSA if recurrent infection NICE guidelines on paediatric urinary tract infections (UTI) are intermittently updated and the current guidelines are listed for review in December 2024. In both the previous and current guidance for a typical Escherichia coli infection, in a child of 11 months who is responding to treatment, the guidance suggests no imaging is required. US and DMSA only becomes necessary if the UTI is recurrent. If the infection is atypical (non-E. coli) or there are other concerning features such as sepsis, abdominal mass, unresponsiveness to treatment, raised creatinine or poor urine flow, then urgent US is indicated. Similarly, if a child is less than 3 months old and has a UTI, then paediatric referral and imaging would be indicated.
1268
A 23 year old male presents with fever, loose stools and watery diarrhoea for the last 2 weeks. He has recently returned from a trip to southeast Asia. Blood tests show raised inflammatory markers. An abdominal radiograph is performed which shows mucosal oedema and thumb printing affecting the ascending colon. Stool cultures are sent. Given the distribution of the abnormality, what is the most likely organism responsible for the appearances? a. Salmonella b. Shigella c. Cytomegalovirus d. Herpes simplex virus e. Clostridioides difficile
(a) Salmonella The underlying organism in infectious colitis may be predicted by the segment of the bowel affected. The typical inpatient and antibiotic history associated with Clostridioides difficile (previously known as Clostridium difficile) colitis does not fit with the history in this case. Furthermore, it tends to cause a diffuse colitis with gross mural thickening. Salmonella - Ascending colon Shigella - Sigmoid colon CMV - Ileocolic HSV - Proctitis
1269
A 54 year old female patient has an abdominal ultrasound for right upper quadrant pain. This demonstrates a homogenous, echogenic lesion in the upper pole of the left kidney. It is well defined, exophytic and measures 46 mm with posterior acoustic shadowing. There is no hydronephrosis and the kidney otherwise has normal appearances. Appearances of the right kidney are within normal limits. What would be the next most appropriate step? a. CT chest, abdomen and pelvis b. CT abdomen as next available outpatient appointment c. CT abdomen in 3–6 months d. Discussion at the urology multidisciplinary team meeting e. Repeat ultrasound in 3–6 months
(b) CT abdomen as next available outpatient appointment The ultrasound features are of an angiomyolipoma – an echogenic, often cortically based exophytic lesion containing fat, as well as vessels and muscle. The figure 4 cm is important to remember; below this size these lesions should be followed up to assess for interval growth. Once greater than 4 cm in size, the risk of haemorrhage increases and embolisation or partial nephrectomy should be considered. Although ultrasound imaging demonstrates typical features, these lesions need prompt CT to characterise fully and confirm the diagnosis. It can sometimes be difficult to exclude renal cell carcinoma (RCC), particularly those containing microscopic fat or if the angiomyolipoma is lipid poor. MRI may be helpful in these cases. Features such as calcification and necrosis would tend to favour RCC. In this instance, further staging imaging could then be completed.
1270
The urology team request an urgent testicular ultrasound on a teenager with left sided scrotal pain and swelling. The ultrasound demonstrates scrotal wall thickening with associated increased Doppler flow. The testicles have a homogenous echotexture with symmetrical vascularity. The left testicle is marginally bigger than the right testicle. There is a trace amount of fluid around the left testicle. There is a 3-mm thin-walled, avascular, anechoic structure with posterior acoustic enhancement at the left epididymal head adjacent to the upper pole of the testicle. The left epididymal tail is swollen with a heterogenous echotexture and diffusely increased vascularity compared to the right side. The right epididymis has a homogenous echotexture. There is no evidence of varicocele. The urology registrar comes to discuss the findings after the scan. What is the most appropriate report conclusion? a. Epididymitis b. Epididymo-orchitis c. Orchitis d. Testicular torsion e. Torsion of the appendix testis
(a) Epididymitis Testicular torsion is always a concern with scrotal pain and swelling. Although an urgent ultra- sound is frequently requested, this may not reveal a significant abnormality, especially in cases of intermittent torsion. Therefore appropriate clinical assessment is vital. In this case, the left epididymis is swollen and heterogenous with increased vascularity, and this is consistent with epididymitis. Torsion of the appendix testis has similar symptoms to testicular torsion. Specific findings may not be evident on ultrasound and it is sometimes a diagnosis of exclusion; however occasionally the appendix may be visible as a small structure at the upper pole of the testicle with internal echoes which is hypoechoic compared to the adjacent testicle. A hydrocele and scrotal wall oedema may also be present. The small, thin-walled lesion at the epididymal head has features of a cyst with anechoic echotexture and posterior acoustic enhancement. In this position, this is likely to be an epididymal cyst. Findings that would be suggestive of orchitis would be hypoechoic echotexture and hypervascularity within the testicle itself. There may also be swelling and scrotal wall oedema. Orchitis is most commonly associated with epididymitis but can occur in isolation; for example in mumps. Although the left testicle is marginally bigger, there is frequently physiological asymmetry and without the other findings, orchitis cannot be confidently diagnosed.
1271
You are the radiology registrar in a major trauma centre. An intubated and ventilated 35 year old motorcyclist has an MRI whole spine following a road traffic collision earlier the same day. The initial trauma CT identified intracranial haemorrhage and fractures of the T4, T5 and L1 vertebrae. The MRI reveals a thoracic epidural collection which is isointense on T1 sequences and mildly hyperintense on T2 sequences. The radiology consultant agrees with your observations. What is the most appropriate next step in management? a. Inform neurosurgical team b. Request immediate T2* MRI sequence c. Request immediate additional contrast enhanced MRI sequences d. Repeat MRI in 24 hours e. Urgent discussion with microbiology
(a) Inform neurosurgical team The epidural collection has the signal characteristics of hyperacute blood; T1 isointense to cord and T2 bright. In the context of recent trauma this is consistent with epidural haemorrhage. This is a surgical emergency and therefore the neurosurgeons should be informed urgently. Haemorrhage causes blooming on T2* MRI sequences and although this may be helpful to confirm the diagnosis, it could also delay treatment. If the neurosurgeons want any further clarification then this could always be performed following discussion with them. With a history of fever and other infective symptoms then an epidural abscess could be considered and microbiology input may be required. Epidural abscess usually have low T1 signal, high T2 signal and peripheral enhancement. Contrast enhanced imaging would not be indicated in this case as the history and radiological findings are consistent and additional sequences may delay appropriate management.
1272
A 67 year old diabetic male patient presents to hospital with a 4-day history of erythema and swelling of his left elbow which has not improved despite 2 days of antibiotics from his GP. There is no history of preceding trauma. Inflammatory markers are raised and he has a temperature of 38.5 °C. A blood culture is negative. A radiograph of the left elbow demonstrates soft tissue swelling but the bones are normal in appearance. The MRI scanner is undergoing maintenance; therefore a nuclear medicine triple phase bone scan is performed. Which of the below patterns of tracer uptake would be most consistent with osteomyelitis?
(c) Increased tracer uptake on all three phases Bone scans are sensitive but not specific for osteomyelitis. The bone scan will often be abnormal before any radiographic changes are evident. The three phases are angiographic, tissue and osseous phases. Osteomyelitis will demonstrate increased tracer uptake on all three phases due to increased blood flow to the region, increased tracer uptake within the adjacent inflamed soft tissues and then increased uptake within the bone itself. This is in contrast to cellulitis where uptake would only be increased on the angiographic and tissue phases. The most sensitive test for osteomyelitis is MRI which can reveal bone marrow oedema as the earliest change. After several days a low T1 signal area, representing intraosseous or subperiosteal abscess, may be evident with surrounding intermediate T1 signal which enhances (penumbra sign). Radiographic features are often delayed and include soft tissue swelling, periosteal reaction and soft tissue gas.
1273
A 58 year old male patient has a barium swallow and meal study for heartburn and symptoms of gastritis which have not responded to treatment. This demonstrates a malignant appearing gastric ulcer and the patient subsequently has an endoscopy that confirms malignancy. CT chest, abdomen and pelvis confirms no distant spread of the disease. With regard to the ulcer, what imaging finding on barium meal best correlates with this diagnosis? a. Carmen meniscus b. Hampton’s line c. Location on the lesser curve d. Extending beyond the gastric wall e. Gastric folds reach the edge of the ulcer
(a) Carmen meniscus It is important to know the different imaging characteristics of benign and malignant gastric ulcers. Carmen meniscus is a sign caused by malignant ulcers; there are heaped-up margins causing an inner ulcer margin, which is convex towards the lumen and causes subsequent lenticular pooling of barium in the ulcer, whereas a benign ulcer has a smooth concave shape without heaped-up margins. The gastric folds therefore do not reach the edge of a malignant ulcer as they do in a benign ulcer. Hampton’s line, associated with benign ulcers, is a radiolucent line seen at the edge of the ulcer. A way to remember these two signs is Carmen = Carcinoma and Hampton = Harmless. Lesser curve ulcers are more typically associated with benign ulcers, whereas malignant ulcers tend to be found on the greater curve. Benign ulcers tend to be exoluminal, extending beyond the gastric contour, whereas malignant ulcers are endoluminal and do not. Similarly smooth gastric folds are benign features, whereas irregular folds are more concerning.
1274
A 2 month old boy has an abdominal ultrasound scan which identifies small kidneys bilaterally with bilateral hydronephrosis and hydroureter. The urinary bladder is thickened and trabeculated. On micturating cystourethrogram there is bilateral vesicoureteral reflux and dilatation of the posterior urethra. Contrast also fills a small diverticulum, continuous with the bladder dome, at the bladder’s anterosuperior aspect. Assessment of the scrotum finds that the testicles are undescended. Which one of the following conditions is most consistent with these radiological findings? a. Bladder exstrophy b. Eagle-Barrett syndrome c. Edwards syndrome d. Patau syndrome e. Posterior urethral valve
(b) Eagle-Barrett syndrome Eagle-Barrett syndrome, also known as prune-belly syndrome due to hypoplasia of the abdominal wall muscles, causes a constellation of findings including urinary tract anomalies and cryptorchidism. The urinary tract can be affected by renal dysplasia, posterior urethral valve, urachal diverticulum, an enlarged trabeculated bladder and vesicoureteral reflux. Posterior urethral valves are indicated by the presence of a thickened bladder and sometimes bilateral hydronephrosis and hydroureter due to vesicoureteral reflux. A micturating cystourethrogram will find a dilated posterior urethra and a valve may be visualised. The other findings in this case of small kidneys and cryptorchidism make Eagle-Barrett syndrome more likely than just a solitary posterior urethral valve. Edwards and Patau syndrome can be associated with Eagle-Barrett syndrome; however there are no other features to suggest that these are more likely. Patau syndrome is also associated with cryptorchidism, and Edwards syndrome is associated with ureteral duplication and horseshoe kidney. Bladder exstrophy leads to herniation of the bladder through the anterior abdominal wall and it is also associated with cryptorchidism in males.
1275
The abdominal radiograph of an 18 month old child reveals a large soft tissue density mass in the left upper quadrant displacing the adjacent bowel loops. An abdominal ultrasound confirms a vascular, heterogenous, solid and cystic left upper quadrant mass. There are highly reflective foci which exhibit posterior acoustic shadowing. The left kidney is displaced anteroinferiorly and the mass extends posteriorly and is seen adjacent to the vertebral column, with the aorta seen pulsating within the mass. The liver and spleen have normal appearances. The case is discussed at the paediatric oncology multidisciplinary team meeting with the paediatric surgeons. Which imaging test would be most appropriate for assessing the local disease extent? a. CT chest, abdomen and pelvis b. MIBG c. MRI abdomen d. Technetium 99-m bone scan e. 18F-FDG PET/CT
(c) MRI abdomen The characteristics of the case in this question are suggestive of neuroblastoma; the patient is <2 years old and the mass is large, displacing the bowel and kidney and encasing the vasculature. On ultrasound it has areas which likely represent calcification. The mass extends adjacent to the vertebral column, and neural foraminal involvement is important to establish as it may impact management if neurosurgical input is required. Neuroblastoma frequently metastasises, commonly to the liver and bone. CT is frequently employed in the diagnosis and staging of neuroblastoma. To assess distant extent, MIBG has traditionally been the investigation of choice; however PET/CT has been used in some centres. Technetium-99m bone scans can also be helpful. In this case, MRI abdomen would be the best modality to assess local disease extent and particularly neuroforaminal involvement.
1276
A 55 year old male patient is referred by his GP due to pain in his non-dominant left wrist, particularly at night, and associated pins and needles in his thumb, index and middle fingers. Carpal tunnel syndrome is suspected and the patient has an MRI of the left wrist. What is the most specific sign on MRI? a. Abrupt change in median nerve diameter b. Bowing of the flexor retinaculum c. Enhancement of the median nerve d. Flattening of the median nerve e. Oedema within the carpal tunnel
(a) Abrupt change in median nerve diameter Carpal tunnel syndrome can be associated with conditions such as acromegaly, hypothyroidism, pregnancy, rheumatoid arthritis and diabetes. It presents with pain and paraesthesia in the hand in the distribution of the median nerve. Tinel test (tapping the median nerve) and Phalen test (paraesthesia exacerbated by wrist flexion) are used to aid diagnosis. The most specific sign on MRI is enlargement of the nerve. Other findings include bowing of the flexor retinaculum, flattening and/or enhancement of the nerve and oedema within the nerve or carpal tunnel and loss of fat within the carpal tunnel. Ultrasound can be used and can also show bowing of the flexor retinaculum and nerve compression causing flattening distally and enlargement of the nerve just proximal to the flexor retinaculum.
1277
An MRI spine for a patient with back pain radiating down the left leg has a report which states that there is a broad-based disc protrusion at L2/3 causing significant stenosis of the left exit foramen and impingement of the exiting nerve at this level. Which of the following descriptions fits this observation most accurately? a. Disc material extending >90° circumference and >3 mm beyond the vertebral body margin impinging the left L2 nerve b. Disc material extending >90° circumference and >3 mm beyond the vertebral body margin impinging the left L3 nerve c. Disc material extending >90° circumference and >3 mm beyond the vertebral body margin impinging the right L3 nerve d. Disc material extending >180° circumference and >3 mm beyond the vertebral body margin impinging the left L2 nerve e. Disc material extending >180° circumference and >3 mm beyond the vertebral body margin impinging the left L3 nerve
(a) Disc material extending >90° circumference and >3 mm beyond the vertebral body margin impinging the left L2 nerve A broad based protrusion is when the disc material extends >3 mm beyond the vertebral body margin over an area extending >90° but <180° over the circumference of the vertebral body. This is in contrast to a focal protrusion which is <90° and a broad based bulge which is >180°. In the lumbar spine the nerve roots exit below the matching pedicle, so at L2/3, the L2 nerve exits. In the cervical spine there is a mismatch so at C2/3 the C3 nerve root exits below the C2 pedicle.
1278
A renal ultrasound on a 1 year old girl being followed up following an abnormal antenatal scan finds an enlarged right kidney. There are several thin-walled, large anechoic lesions which are variable in size but demonstrate posterior acoustic enhancement. The morphology of the kidney is distorted with no visible normal renal parenchyma. It is difficult to assess if these lesions are continuous with the renal pelvis. There is no associated increased Doppler vascularity. There is no uptake in this kidney on a MAG3 study. The left kidney has normal appearances. What is the most likely diagnosis? a. Autosomal recessive polycystic kidney disease b. Congenital mesoblastic nephroma c. Hydronephrotic right kidney d. Multicystic dysplastic kidney e. Multilocular cystic nephroma
(d) Multicystic dysplastic kidney Multicystic dysplastic kidney is an obstructed non-functioning kidney which manifests as an enlarged multicystic kidney lacking normal renal parenchyma. The cysts may have rim calcification. If it is bilateral it is incompatible with survival. When unilateral, the contralateral side is commonly associated with vesicoureteral reflux. The appearance may be difficult to differentiate from a hydronephrotic kidney; however, the absence of tracer uptake on the MAG3 study indicates there is no functioning renal tissue and therefore helps to differentiate the two. Congenital mesoblastic nephroma are most commonly predominantly solid lesions. Autosomal recessive polycystic kidney disease is associated with bilateral enlarged kidneys and multiple tiny cysts. Hepatic fibrosis is also a feature. Multilocular cystic nephromas typically have a thick fibrous capsule and multiple septations which may demonstrate vascularity. The lesion may compress surrounding normal renal parenchyma. A typical characteristic is herniation into the renal pelvis, although with large lesions this may be difficult to appreciate.
1279
A 52 year old female patient presents to hospital feeling unwell with fever, fatigue and generalised aches and pains causing difficulty mobilising. There is a rash on her back and upper arms. The emergency department team request blood tests and chest and abdominal radiographs. The chest radiograph shows some air space opacification and bibasal atelectasis. The abdominal radiograph demonstrates a normal bowel gas pattern with subcutaneous calcification in a linear distribution projected over the soft tissues of the pelvis and upper thigh. What is the most appropriate next investigation? a. Barium swallow b. CT chest c. Oesophageal endoscopy d. Nuclear medicine cardiac perfusion study e. Ultrasound Doppler lower limbs
(b) CT chest The patient has signs of dermatomyositis with a rash and inflammation of striated muscle. It tends to affect patients around the age of 50 years. Inflammatory changes of the muscles can lead to soft tissue calcification which typically is sheet-like in appearance, although can manifest as non-specific subcutaneous calcification. On MRI the muscles can have high T2 and STIR signal, indicating oedema with low signal related to any calcification. With long-standing disease the muscles may undergo fatty infiltration. Dermatomyositis is associated with underlying malignancy, most commonly oesophageal, melanoma, genitourinary and lung. However, the majority have interstitial lung disease with a cryptogenic organising pneumonia type pattern causing peripheral subpleural consolidation and nodularity. Patients also have high risk of myocardial ischaemia and thromboembolic events. Therefore, all the answers could potentially be correct; however the most suitable answer is a CT chest to look for underlying interstitial lung disease, as this is the most common association.
1280
An abdominal radiograph is performed for a 51 year old patient with abdominal pain presenting to hospital. Bilateral renal calcification is evident in a medullary distribution. Which of the following is unlikely to be a cause for this appearance? a. Hyperparathyroidism b. Hypothyroidism c. Multiple myeloma d. Sarcoidosis e. Tuberculosis
(e) Tuberculosis Chronic end-stage renal tuberculosis tends to manifest as amorphous foci of calcification forming throughout the kidney sometimes associated with the process of autonephrectomy. The kidney often appears shrunken with a thinned cortex. The other available answers − hyperparathyroidism, hypothyroidism, sarcoidosis and multiple myeloma − are all associated with medullary calcification, which is the most common form of nephrocalcinosis. The most common causes of medullary nephrocalcinosis are hyperparathyroidism, renal tubular acidosis and medullary sponge kidney.
1281
A 20 year old man is assaulted sustaining several heavy blows to the face. Facial bone radiographs are performed in the emergency department followed by CT facial bones. There are bilateral complex facial bone fractures. Which structure needs to be involved to classify the injury as a Le Fort fracture? a. Zygomatic arch b. Inferior orbital rim c. Pterygoid plates d. Nasal septum e. Lateral walls of maxillary sinuses
(c) Pterygoid plates Le Fort fractures are midface fractures involving the pterygoid plates. The fractures are categorised as Type I–III. Type I is known as a floating palate, Type II a floating maxilla and Type III is the most severe and is separation of the base of skull and midface, otherwise known as ‘floating face’ or ‘craniofacial disjunction’. When facial fractures are bilateral there may be asymmetry in the Le Fort fracture types.
1282
An athletic 45 year old man is seen in a follow-up orthopaedic clinic following surgery for a prolapsed L3/4 disc 12 months ago as an emergency case for cauda equina. He recovered well immediately postoperatively but is concerned because he has been suffering from increasing back pain for 3 months radiating down his left leg. The patient has no neurological symptoms on examination. The orthopaedic team request an MRI of the lumbar spine. At the L3/4 disc space there is moderate low T1 signal narrowing the left subarticular zone. Which finding in the left subarticular zone is most in keeping with post-surgical fibrosis versus recurrent disc prolapse? a. Early enhancement following gadolinium b. High T2 signal c. Smooth border d. Late enhancement following gadolinium e. Restricted diffusion
(a) Early enhancement following gadolinium Patients who have previously had back surgery, for example discectomy or laminectomy, and develop back pain often require repeat MRI to distinguish between recurrent disc prolapse and postoperative epidural fibrosis. This is an important distinction to make as recurrent prolapse would be treated surgically whereas surgeons would be reluctant to intervene for postoperative fibrosis. It can be difficult to distinguish between the two entities on non-contrast imaging as both have low T1 signal, although fibrosis tends to have irregular borders and evidence of thecal retraction towards the area of interest. In contrast to this, a disc would typically have a smoother border. Following gadolinium administration, scarring enhances early whereas a disc will either not enhance or enhance late. Enhancement of the fibrosis diminishes after a few years. DWI and ADC imaging are not routinely performed for spinal imaging. High T2 signal in a disc is associated with discitis.
1283
A 7 year old girl with urinary incontinence is referred for a micturating cystourethrogram. The patient has an ultrasound followed by an intravenous urogram. Both kidneys have a normal position and size. The left kidney has two ureters draining it and one of the ureters drains into the vagina with the other ureter draining into the urinary bladder. Which of the following is the most accurate statement? a. The anatomy in this case represents a bifid ureter b. The lower pole moiety is frequently obstructed by a ureterocoele c. The upper pole moiety can have a ‘drooping lily’ appearance d. The ureter draining into the vagina is associated with the upper pole moiety e. Vesicoureteral reflux is associated with the ureter draining into the vagina
(d) The ureter draining into the vagina is associated with the upper pole moiety The anatomy in this case represents a double collecting system. A bifid ureter is two ureters that unite before the urinary bladder. A duplicated system is associated with a normally inserting ureter which arises from a lower pole moiety and an ectopic ureter which arises from an upper pole moiety. The ectopic ureter can insert elsewhere in the urinary bladder, usually more inferiorly and medially, or as in this case into nearby structures such as the vagina, which can lead to incontinence. The upper pole moiety can be obstructed due to the presence of a ureterocoele. This can inferiorly displace the lower pole moiety, leading to the ‘drooping lily’ appearance. The lower pole ureter is frequently affected by vesicoureteral reflux.
1284
A 60 year old woman presents following a traumatic head injury. She is noted to have haemotympanum. A CT head is performed which shows a transverse fracture extending through the left petrous temporal bone. What likely complication is most associated with this type of fracture? a. Tympanic membrane disruption b. Sensorineural hearing loss c. Conductive hearing loss d. Carotid artery dissection e. Sigmoid sinus injury
(b) Sensorineural hearing loss Temporal bone fractures are split into longitudinal, transverse and mixed fractures. Transverse fractures are less common but more likely to lead to vertigo and sensorineural hearing loss secondary to CN VIII injury. CN VII can also be affected causing facial paralysis. Longitudinal fractures are more likely to lead to conductive hearing loss with involvement of the ossicles and tympanic membrane.
1285
A 31 year old patient comes to the emergency department with worsening lower lumbar back pain over the past 7 weeks. The patient has no fixed abode and has a recent history of intravenous drug use. Inflammatory markers are raised and the patient has a temperature of 37.5 °C. Chest radiograph is normal and urine dip is negative. Blood cultures are sent to the laboratory. Radiograph of the spine shows reduction in disc space at L3/4. MRI reveals high T2 signal and post contrast enhancement in the L3/4 disc with low T1 and high T2 signal in the adjacent vertebral bodies. There is also an enhancing paravertebral collection which is inseparable from the left psoas muscle. What is the most likely causative organism? a. Burkholderia pseudomallei b. Candida c. Mycobacterium tuberculosis d. Salmonella e. Streptococcus viridans
(e) Streptococcus viridans The MRI features are typical for pyogenic discitis and associated vertebral osteomyelitis. S. viridans discitis is associated with the immunocompromised and intravenous drug users and can have accompanying endocarditis. The most common causative organism for discitis in the general population is Staphylococcus aureus, and blood cultures are positive in around 50–70% of cases. In patients with sickle cell disease, Salmonella should be considered. Candida is less common. Burkholderia pseudomallei is more commonly encountered in locations such as southeast Asia and Australasia. Spondylodiscitis caused by Mycobacterium tuberculosis (TB) (also known as Pott disease) would also be within the differential for this patient, although this is a relatively acute presentation and usually TB discitis tends to have a longer history without much systemic upset. It is not unusual for the chest radiograph to be normal in TB discitis. There can be reduction in vertebral body height causing a ‘gibbus’ deformity. Subligamentous spread is also a feature with the potential for skip lesions. The lower thoracic/upper lumbar spine is more commonly involved, whereas the lumbar spine is most frequently affected in pyogenic discitis. Intervertebral disc height is usually quite well preserved in TB discitis compared to pyogenic discitis. Paravertebral collections are possible in both pyogenic and TB discitis but they are more likely to calcify in the latter.
1286
A 75 year old male patient is admitted with fever, reduced appetite, shortness of breath and chest pain. Contrast enhanced CT chest, abdomen and pelvis identifies no consolidation or intrabdominal collections however there are multiple low attenuation hepatic lesions consistent with metastases and subsegmental bilateral lower lobe pulmonary emboli. An echocardiogram reveals tricuspid valve vegetations. Blood cultures obtained upon presentation to hospital have grown Streptococcus bovis. Which is the most likely primary site of malignancy? a. Biliary tree b. Colon c. Oesophagus d. Pancreas e. Prostate
(b) Colon The patient has metastatic liver lesions and spread from a cholangiocarcinoma, colonic, pancreatic or oesophageal carcinoma are all possible. Liver metastases in prostate cancer would be more unusual. The presence of endocarditis and blood cultures which have grown Streptococcus bovis are highly suspicious for an underlying colonic malignancy. Patients with colorectal cancer may present with metastatic disease, particularly if the colonic primary is right sided. Left sided tumours tend to present earlier with altered bowel habit.
1287
A 57 year old woman has a breast assessment following a positive CT pulmonary angiogram which reported a small, enhancing, solid mass in the right breast and two similar lesions in the left breast. The lesions are assessed in the clinic and biopsied. The pathology reports they are metastatic lesions rather than primary breast carcinoma. Which of the following is the most likely underlying malignancy? a. Colonic carcinoma b. Lung carcinoma c. Lymphoma d. Ovarian carcinoma e. Renal cell carcinoma
(c) Lymphoma The most common metastatic lesion to the breast is associated with lymphoma, followed by melanoma, choriocarcinoma and renal cell carcinoma. In men, prostate cancer is commonly a cause of breast metastases. It is also possible with other malignancies such as colon, ovarian and gastric cancers. Metastases are usually multiple and bilateral. They are frequently well-defined, solid lesions on imaging and may demonstrate posterior acoustic enhancement. In contrast to this, primary breast malignancy is typically ill defined and hypoechoic with posterior acoustic shadowing, although appearances can be variable and hence biopsy is vital to determine the cause of most solid breast lesions.
1288
A 67 year old female patient has a chest radiograph following presentation to hospital with chest pain. The lungs and pleural spaces are clear and appearances of the mediastinum are within normal limits; however, there are multiple osseous lytic lesions. The patient is investigated further with a CT chest, abdomen and pelvis and a nuclear medicine bone scan. The CT scanner is currently undergoing maintenance awaiting a new part and therefore the nuclear medicine bone scan occurs first and is reported as normal osseous uptake. What is the next best available test to confirm the diagnosis? a. Breast clinic triple assessment b. Bone marrow biopsy c. Blood culture d. Serum alkaline phosphatase levels e. Serum parathyroid hormone levels
(b) Bone marrow biopsy The patient has an abnormal radiograph with a normal bone scan. The differentials for this include metabolically inactive bone lesions such as bone cysts and islands, osteoporosis, new fractures (<48 hours old), multiple myeloma or metastases with no osteoblastic activity. Therefore, the most likely cause is multiple myeloma and the most appropriate answer is a bone marrow biopsy. Most bony breast cancer deposits do demonstrate tracer uptake on nuclear medicine bone scans; however some breast cancers and other osteolytic metastases, such as from renal, thyroid or lung cancer, can be non-avid. However, these cancers tend to create photopenic foci. Osteomyelitis can cause lytic lesions on radiographs but the multiplicity is not typical and therefore blood culture would not be appropriate. Similarly Paget’s disease and hyperparathyroidism can cause abnormal radiographic appearances with lytic lesions but the bone scan would usually be abnormal in these conditions, demonstrating focal and generalised increased uptake, respectively.
1289
A 22 year old student develops cognitive decline and then worsening ataxia and spasticity over a period of a few weeks. Following admission she has an MRI head which demonstrates subcortical and deep white matter T2 hyperintensity affecting the majority of the right parietal lobe, the left temporoparietal region and minimal signal changes in the left putamen and globus pallidus. There is evidence of adjacent oedema and patchy white matter enhancement. There is also mild volume loss in the frontotemporal region. Past medical history is unremarkable apart from appendicectomy as a child. Upon further questioning she recalls an episode of feeling unwell about 6 months ago with fever and flu- like symptoms before developing white spots on the inside of her cheeks and then a more generalised rash. What is the most likely diagnosis? a. Acute demyelinating encephalomyelitis b. Acute haemorrhagic leukoencephalopathy c. Herpes simplex encephalitis d. Japanese encephalitis e. Subacute sclerosing panencephalitis
(e) Subacute sclerosing panencephalitis The patient has a history consistent with measles infection. The buccal spots are Koplik spots and are highly characteristic for measles. The associated asymmetrical subcortical and deep white matter changes and volume loss are consistent with subacute sclerosing panencephalitis. This typically affects the temporoparietal region but can become diffuse, affecting the basal ganglia and corpus callosum. Acute demyelinating encephalomyelitis (ADEM) is within the differential, especially with a history of infective illness; however this rarely involves the basal ganglia and is not associated with atrophy. Acute haemorrhagic leukoencephalopathy is a severe variant of ADEM characterised by significant oedema and small white matter haemorrhages. Herpes simplex encephalitis affects the temporal lobes, often bilaterally, as well as commonly involving the frontal lobes and insular cortex with gyral and leptomeningeal enhancement rather than the patchy white matter enhancement described in this case. Japanese encephalitis typically demonstrates bilateral thalamic involvement, which can lead to haemorrhage, as well as involvement of other both supra and infra tentorial structures.
1290
A 34 year old male is under investigation for a newly diagnosed malignancy. Lymph node biopsy demonstrates Reed-Sternberg cells. Where is the most likely site of thoracic lymph node enlargement on the CT chest? a. Hilar b. Internal mammary c. Paratracheal d. Peri-oesophageal e. Subcarinal
(c) Paratracheal The presence of Reed-Sternberg cells is in keeping with a diagnosis of Hodgkin lymphoma. Over 80% of patients with Hodgkin lymphoma have involvement of the thorax at the first presentation. Most commonly, the anterior mediastinum and paratracheal nodes are involved. Less frequently the subcarinal, perioesophageal and internal mammary nodes may also be involved. Hilar lymph node enlargement usually occurs in the context of mediastinal lymphadenopathy.
1291
A 37 year old semi-professional footballer attends the orthopaedic clinic due to left hip pain which is causing him difficulty when playing. Following clinical review, radiographs and subsequent MRI, a diagnosis of cam femoroacetabular impingement is made. Which radiological findings are most consistent with this diagnosis?
(d) Lateral osseous bump, 65°, Labral tear There are two subtypes of femoroacetabular impingement (FAI); cam FAI is more common in young men versus pincer FAI which is associated more with middle-aged females. Cam FAI is caused by a non-spherical femoral head impacting against the acetabulum; typically there is a flattening at the junction between the femoral head and neck with an osseous bump just lateral to the physeal scar. This is associated with subsequent degenerative change and labral tears. Cartilage and labral injuries tend to be positioned anterior-superiorly. The alpha angle is frequently cited and an angle >55–60° (depending on the article) is considered abnormal. Pincer FAI is due to a deep acetabulum; therefore the coverage of the femoral head is increased and leads to abnormal contact between the femoral head/neck junction and the overhanging acetabulum. Chondral injury is more typically circumferential versus the anterior-superior position in cam FAI. It is also possible to have a combined type of FAI.
1292
A 54 year old male patient with no significant past medical history has an enhanced CT abdomen pelvis requested by the emergency department for right sided abdominal pain which confirms appendicitis. There is an incidental finding of a 23 mm exophytic right renal lesion which has a density of 2HU, multiple thin septae and nodular calcifications. On the left kidney there is another exophytic lesion which is 15 mm, homogenous and has a density of 72HU; there is thin wall calcification but no septations. Both lesions are classified as per the Bosniak classification system. The CT report is forwarded to the urology multidisciplinary team meeting co-ordinator. What is the most appropriate management for this patient? a. CT chest b. Follow-up CT in 3–6 months c. No further follow-up required d. Surgical excision of the right renal lesion e. Unenhanced CT abdomen
e) Unenhanced CT abdomen The patient has bilateral renal lesions. The lesion on the right kidney is consistent with a Bosniak 2F cyst with thin septae and nodular calcification; this will require CT follow-up. Once septae or walls become thickened and nodular with measurable enhancement then the cyst falls into the Bosniak 3 category and many patients will undergo invasive management, either surgery or radiofrequency ablation. The lesion associated with the left kidney is hyperdense. This CT is enhanced, likely portal venous phase imaging for this clinical indication, so we need to establish whether this hyperdensity is enhancement or due to the inherent density of the lesion. A hyperdense renal cyst should not significantly change density between the unenhanced and the enhanced sequences. A soft tissue mass would demonstrate a significant difference being lower density on the unenhanced and then increasing following contrast administration. The patient should be fully worked up prior to being discussed at the multidisciplinary team meeting and therefore although the patient will require follow-up for the right Bosniak 2F lesion, the unenhanced CT should be performed in the first instance to complete the imaging and complete characterisation of the lesion.
1293
A 7 year old boy with no significant past medical history is being investigated for increased thirst. An MRI head demonstrates mild exophthalmos but no intracranial abnormality. A chest radiograph shows that the lungs are clear. Hepatosplenomegaly is reported on an ultrasound abdomen. Imaging 1 month previously for a painful right leg identified a lucent lesion in the distal right femoral metadiaphysis with a narrow zone of transition and a fine sclerotic border. There is minimal adjacent periosteal reaction but no matrix calcification and no soft tissue component evident. What is the most likely underlying diagnosis? a. Diabetes mellitus b. Graves disease c. Langerhans cell histiocytosis d. Sarcoidosis e. Tuberculosis
(c) Langerhans cell histiocytosis The constellation of clinical signs and symptoms are consistent with Hand-Schüller-Christian disease which is the chronic systemic form of Langerhans cell histiocytosis. It can manifest with bone lesions, hepatosplenomegaly, diabetes insipidus, proptosis and dermatitis. The majority of bone lesions in Langerhans cell histiocytosis have variable appearances, with both lucency and sclerosis depending on the stage of the disease. However, skull lesions typically have a bevelled ‘hole within a hole’ appearance and vertebral involvement leads to vertebra plana, which is the most common cause of vertebra plana in children. Sarcoidosis and tuberculosis can cause diabetes insipidus with other systemic signs and symptoms, including musculoskeletal manifestations. Sarcoidosis can affect the eyes too but usually affects the lacrimal glands rather than causing proptosis. Graves disease can cause exophthalmos and skeletal changes, such as thyroid acropachy, but it is not associated with the other clinical signs in this case. Diabetes mellitus is a cause of increased thirst in children but does not account for the other changes.
1294
A 6 year old patient has a chest radiograph for cough which is not responding to antibiotics. There is hyperinflation of the lungs and mild bronchial wall thickening suggestive of asthma. Incidental note is made of complete collapse of the T8 vertebral body. What other finding would best correlate with the most common paediatric cause of this vertebral appearance? a. Abdominal mass on ultrasound abdomen arising adjacent to aorta b. Enlarged rounded lymph nodes on ultrasound neck and axilla c. Multiple lytic skull lesions d. Permeative lytic lesion with laminated periosteal reaction in the right femur e. Raised inflammatory markers
c) Multiple lytic skull lesions The case describes vertebra plana. The most common cause in children is Langerhans cell histiocytosis (LCH) which among other findings can cause lytic skull lesions. LCH can cause aggressive looking osseous lesions with a laminated or ‘onion skin’ periosteal reaction, as described in Answer D. However, the skull is more commonly affected in LCH than the long bones. The femoral appearances in the available answer are concerning for Ewing’s sarcoma which is another, less common cause of vertebra plana. The other available answers describe other potential causes of vertebra plana in children including neuroblastoma which could cause the finding of an abdominal mass adjacent to the aorta on abdomen ultrasound. Raised inflammatory markers are non-specific but would be associated with infection, including osteomyelitis and tuberculosis. Multiple abnormal lymph nodes would be suspicious for lymphoma; however in lymphoma there may be multi-level spine involvement.
1295
A 6 year old boy has a chest radiograph requested by his GP following a week of fever and productive cough. There is a small region of air space opacification in the right lower zone. Asymmetry is noted between both sides of the chest; the left hemithorax is smaller and more lucent compared to the right with reduced vascular markings. There is no significant mediastinal shift. The heart size is normal. What is the most likely cause? a. Congenital lobar over inflation b. MacLeod syndrome c. Poland syndrome d. Pulmonary agenesis e. Scimitar syndrome
(b) MacLeod syndrome MacLeod syndrome, also known as Sywer-James syndrome, is thought to occur secondary to childhood infectious bronchiolitis. Previous infections with mycoplasma pneumoniae and viral infections are thought to play a role. It causes a unilateral, often small, lucent lung with reduced vascularity and expiratory air trapping. It can be an incidental finding in adulthood. The history in this case is not typical for congenital lobar over inflation (CLO), which was previously known as congenital lobar emphysema. CLO usually presents in neonates with respiratory distress. It most commonly affects the left upper lobe, followed by the right middle and upper lobes. It tends to cause marked mediastinal shift. Pulmonary agenesis causes an absent lung on one side with no vasculature. Therefore, this usually causes a dense hemithorax with mediastinal shift towards the affected side due to cross-herniation from the contralateral side. Poland syndrome is a congenital unilateral absence of the pectoralis muscles. It would not cause a small unilateral hemithorax and would also not impact the vasculature. Scimitar syndrome does not typically cause a lucent lung; however, the lung is commonly small, with associated ipsilateral mediastinal shift and anomalous venous drainage. This is most commonly into the inferior vena cava which causes a ‘scimitar’ appearance in the right lower zone.
1296
A neonate has an abdominal ultrasound for a right sided abdominal mass. The patient is 3 days old and was born by emergency caesarean section following an unsuccessful induction of labour. The pregnancy was uncomplicated and antenatal scans unremarkable. The ultrasound finds there is a hypoechoic mass in the right upper quadrant containing internal echoes without Doppler vascularity. This is posterior to the liver and the right kidney is seen inferiorly. What is the most appropriate next step in management? a. MIBG study b. MRI abdomen c. No further management required d. Repeat ultrasound in 3 weeks e. Ultrasound-guided biopsy
(d) Repeat ultrasound in 3 weeks The radiological findings are suggestive of adrenal haemorrhage. This often presents as an asymptomatic abdominal mass, and depending on the age of the haemorrhage it can have variable sonographic appearances. Acutely they may appear more solid with internal echoes but over time blood products liquify and they become more heterogenous with cystic components. Chronically they may calcify. The main differential is neuroblastoma which can have similar appearances of a solid appearing mass. One distinguishing feature is the history, and in this case there is a history of a traumatic birth. Secondly, the lack of vascularity is indicative of adrenal haemorrhage rather than neuroblastoma. On follow-up ultrasound, adrenal haemorrhage will decrease in size over time, whereas neuroblastoma will increase in size. Therefore prompt follow-up ultrasound is important. Ultrasound guided biopsy would not be indicated and MRI is usually reserved for challenging cases. MIBG is often used in the staging of neuroblastoma.
1297
A 56 year old male presents with increasing shortness of breath and persistent cough. He has never smoked and has a medical history of rheumatoid arthritis. His GP requests a plain film of the chest which is abnormal. Therefore, a non-contrast CT chest is suggested to assess further. The CT chest reveals volume loss in the lower zones. There is diffuse ground glass opacification seen in the mid and lower zones which demonstrates immediate subpleural sparing with fine subpleural reticulation and traction dilatation of the small airways. What is the most likely diagnosis? a. Cryptogenic organising pneumonia b. Hypersensitivity pneumonitis c. Lymphocytic interstitial pneumonitis d. Non-specific interstitial pneumonia e. Usual interstitial pneumonia pattern of disease
(d) Non-specific interstitial pneumonia (NSIP) Rheumatoid arthritis can be associated with both usual interstitial pneumonia (UIP) pattern of disease as well as NSIP. The features described on the CT are more typical for NSIP. Radiological overlap between the two diseases can occur; however the UIP pattern is associated with honeycomb destruction of the parenchyma compared to the predominantly ground glass opacification seen in NSIP. The immediate subpleural sparing is considered specific for NSIP and traction dilatation occurs due to the fibrotic process pulling the small airways open. Although UIP is associated with an apicobasal gradient, the parenchymal changes are typically peripheral rather than diffuse, as can be the case in NSIP. Subacute hypersensitivity pneumonitis can manifest as ground glass subpleural nodularity with subpleural sparing, representing infiltrate into the small airways. There may also be mosaicism secondary to air trapping. In the chronic stage, features of lung fibrosis can occur, although this is more typically in the upper zones. The predominant imaging finding in cryptogenic organising pneumonia is ground glass change and dense consolidation. Lymphocytic interstitial pneumonia is very rare and more typically associated with Sjogren syndrome or HIV. The main imaging finding is ground glass opacification and perivascular cysts.
1298
A 25 year old male patient has a CT head and sinuses after review in the Ear, Nose and Throat clinic for symptoms of recurrent sinusitis. The CT shows the paranasal sinuses are lacking normal aeration and obliterated due to medullary expansion of the adjacent bones. An abdominal ultrasound from the previous year described the spleen as measuring 17 cm in craniocaudal extent and the inferior border of the liver extending to 6 cm below the lower pole of the right kidney. A recent radiograph of the left shoulder demonstrates normal bone density with metaphyseal flaring at the proximal humerus. What is the most likely diagnosis? a. Gaucher disease b. Osteopetrosis c. Sarcoidosis d. Thalassaemia e. Granulomatosis with polyangiitis (previously known as Wegener granulomatosis)
(a) Gaucher disease The patient is around the typical age for presentation and has classical radiological features of Gaucher disease: hepatosplenomegaly, paranasal sinus obliteration due to medullary expansion and Erlenmeyer flask deformity of the proximal humerus. Erlenmeyer flask deformity has a limited number of differentials which can be remembered with the mnemonic ‘Lead GNOME’: Lead poisoning Gaucher disease Niemann-Pick disease Osteopetrosis Metaphyseal dysplasia Ematological conditions such as thalassaemia Osteopetrosis can cause medullary hyperexpansion and hepatosplenomegaly but the bones are typically sclerotic and fracture easily. Similarly, Niemann-Pick disease can cause hepatosplenomegaly; however, Gaucher disease is more common than Niemann-Pick. Granulomatosis with polyangiitis (previously known as Wegener granulomatosis) is a cause of paranasal sinus obliteration and sarcoidosis is a cause of hepatosplenomegaly but neither cause Erlenmeyer flask deformities.
1299
One of the gastroenterology consultants discusses a case with you of a patient with weight loss and suspicious gastric findings on a recent CT abdomen pelvis. The patient has declined endoscopy. Based on imaging findings, the differential lies between gastric lymphoma and primary gastric carcinoma. Which of the following is more consistent with gastric lymphoma than carcinoma? a. Associated with a history of atrophic gastritis b. Duodenal involvement c. Large polypoid mass d. Gastric outlet obstruction e. Perigastric fat invasion
(b) Duodenal involvement The appearances of gastric lymphoma can be varied; however, wall thickening tends to be more diffuse and thicker than with gastric carcinoma. Extensive wall thickening and submucosal infiltration can extend into the duodenum. There is less likely to be perigastric fat invasion and gastric outlet obstruction than with gastric carcinoma. Atrophic gastritis is a risk factor for gastric carcinoma, along with pernicious anaemia. Helicobacter pylori infection is associated with both gastric carcinoma and lymphoma. Both gastric carcinoma and lymphoma tend to metastasise to the liver first. Regional adenopathy is common in both malignancies but the lymph nodes in gastric lymphoma are usually larger and more extensive than with gastric carcinoma, potentially extending below the level of the renal vein.
1300
You are on-call overnight and a 10 year old rear seat passenger is brought in following a road traffic collision at 30 mph. The child was wearing a seatbelt. Glasgow Coma Scale is 15/15 at the scene and the patient is haemodynamically stable. There is a 2-cm bruise on the forehead but there is no focal neurology and the patient has not vomited. The patient is complaining of neck pain and is immobilised. The chest is clear on auscultation and the abdomen is soft. There is no thoracic or lumbar spine pain on secondary survey. The trauma team discuss with you regarding imaging the child. Based on the available information what is the most appropriate form of initial imaging for this child? a. CT head and cervical spine b. CT head, cervical spine, chest, abdomen and pelvis c. CT cervical spine d. MRI head, cervical spine and chest radiograph e. Radiographs of the cervical spine
(e) Radiographs of the cervical spine The NICE indications for CT head in a paediatric patient of this age include suspicion of non-accidental injury, post traumatic seizure, initial GCS < 14 or GCS < 15 at 2 hours following the injury, suspected open or depressed skull injury, suspected basal skull fracture or focal neurological deficit. Based on the provided information the patient does not fulfil these criteria. The NICE indications for CT cervical spine in paediatric patients include initial GCS < 13, other areas being scanned, for example for head injury, focal neurological signs and/or paraesthesia and if there was an urgent definitive diagnosis of cervical spine injury required, for example prior to surgery. In this case, the answer is no to all of these and therefore due to the neck pain and mechanism of injury the patient would be suitable for three-view cervical spine radiographs in the first instance. If there is still a strong suspicion of injury or if the films are suboptimal, then CT should be considered. The ALARA principal is important when considering imaging in paediatric trauma patients and the Royal College of Radiologists have published useful guidance based on the NICE algorithms. The Royal College
1301
The colorectal nurse specialist calls for advice regarding a 48 year old male patient who underwent total mesorectal excision 23 months ago for T3 N1b M0 rectal carcinoma. More recently he has had a stoma reversal and anastomosis. The patient’s serum carcinoembryonic antigen (CEA) level has increased over the past 3 months. What would be the most appropriate course of action? a. CT cologram and CT chest b. Ultrasound abdomen c. FDG-PET/CT study d. MRI liver with liver specific contrast agent e. Re-check serum CEA in 3 months
(c) FDG-PET/CT study Serum CEA is used to monitor patients following treatment for colorectal cancer but it can also be raised in other cancers and in conditions such as inflammatory bowel disease and liver disease. This patient is young and although asymptomatic it would be important to ascertain if there is evidence of disease recurrence so prompt management can be administered if appropriate. A CT cologram is used for asymptomatic screening and following a failed colonoscopy but its use is not appropriate in this setting. Furthermore, rectal cancer is better visualised and locally staged using MRI rather than CT. An MRI pelvis could be helpful in the identification of recurrence more than 6 months following surgery, with the latter demonstrating higher T2 signal intensity compared to post-surgical fibrosis. FDG-PET/CT is also helpful in recurrence as there will be increased tracer uptake compared to post-surgical changes. An MRI liver may be helpful in assessing for metastatic spread; however with T3N1bM0 disease it is important to exclude local recurrence, and an FDG-PET/CT should also identify any distant disease, for example in the liver.
1302
A chest radiograph is performed for a neonate followed repeated episodes of aspiration. The paediatric team feel tracheo-oesophageal fistula ± oesophageal atresia is likely due to cardiac abnormalities detected on antenatal scans. The chest radiograph demonstrates right lower zone air space opacification. The gastric air bubble is visible under the left hemidiaphragm. Which of the following is most likely? a. Double fistula with intervening oesophageal atresia b. Isolated fistula without oesophageal atresia c. Isolated oesophageal atresia d. Proximal fistula with distal oesophageal atresia e. Proximal oesophageal atresia with distal fistula
(e) Proximal oesophageal atresia with distal fistula Tracheo-oesophageal fistula and oesophageal atresia are associated with other VACTERL anomalies. The typical finding which indicates this problem is a coiled nasogastric tube on a chest radiograph. The most frequent type (85% of cases) is Type C which is proximal oesophageal atresia with a distal tracheo-oesophageal fistula. The second most common type is Type A which is isolated oesophageal atresia. The three other types are rare. The finding of gas in the stomach suggests the presence of a distal fistula or an ‘H-type’ fistula where there is no oesophageal atresia.
1303
A 43 year old man with a three day history of headache, fever and vomiting is admitted. Neurological assessment finds he has a GCS of 12. The right eyelid is drooping and the eye is deviated outwards and downwards. A non-contrast CT head is normal. The scan acquired following contrast injection shows evidence of leptomeningeal enhancement with thickening and enhancement of the basal meninges extending into the sylvian fissures. The brain parenchyma returns normal signal. The leptomeninges overlying the cerebral convexities are not thickened or enhancing. There is mild dilatation of the temporal horns of the lateral ventricles. What is the most likely underlying cause for these changes? a. Enterovirus b. Group B streptococcus c. Mycobacterium tuberculosis d. Neurosarcoidosis e. Streptococcus pneumoniae
(c) Mycobacterium tuberculosis The distribution of findings is typical for tuberculous meningitis. It typically affects the basal cisterns with leptomeningeal enhancement and purulent material in the basal cisterns which can lead to hydrocephalus, the early signs of which are evident in this case. There is also the potential for cranial nerve palsies, particularly the third, fourth, and sixth nerves. The patient in this case has a right third nerve palsy. A differential with a similar distribution of changes is neurosarcoidosis. Neurosarcoidosis most commonly causes parenchymal changes but it can cause leptomeningeal enhancement and cranial nerve palsies, particularly the facial and optic nerves. Fever also makes an infective cause more likely in this case. Serum angiotensin converting enzyme is frequently raised in sarcoidosis. Streptococcus pneumoniae is one of the most common causes of bacterial meningitis in adults. CT imaging is often normal and lumbar puncture should be considered if clinical suspicion is high. Imaging can be helpful in complicated cases when there is development of hydrocephalus, abscess or subdural effusion. Thin, generalised leptomeningeal enhancement may be seen rather than the thickened basal enhancement in this case. Group B streptococcus is the most common cause of meningitis in neonates and therefore does not fit with the patient in this case. Viral meningitis is often self limiting and not associated with reduced conscious level or cranial nerve palsies. It most commonly affects children or immunocompromised adults.
1304
A young patient has a diagnosis of cleidocranial dysostosis; he is reviewed regularly in the paediatric outpatient clinic. The clinical team are concerned as he has a 1-week history of productive cough and increased shortness of breath. The paediatric junior doctor asks you to review his chest radiograph. There is consolidation in the left mid-lower zone which obscures the left heart border. The clavicles are absent. What other feature would correlate with the patient’s underlying congenital skeletal abnormality? a. Decreasing interpedicular distance b. Incompletely ossified sternum c. Pectus excavatum d. Platyspondyly e. Shortened horizontally orientated ribs
b) Incompletely ossified sternum Cleidocranial dysostosis is associated with completely absent clavicles or absence of the lateral portion of them. There is delayed ossification of midline structures causing delayed skull suture closure, enlarged fontanelle and an incompletely ossified sternum. The pubic symphysis may appear widened due to delayed or absent ossification of the pubic bone. Small, high scapulae, supernumerary ribs and a narrowed thorax are also features. There may be hemivertebrae and hypoplasia of the iliac bones. Other bones may be short or absent, such as the radius, fibula and terminal phalanges. Wormian bones and dental abnormalities are also features. Decreasing interpedicular distance is associated with achondroplasia and thanatophoric dysplasia. Pectus excavatum is associated with various conditions including Marfan syndrome, Ehlers-Danlos, neurofibromatosis 1 and osteogenesis imperfecta. Platyspondyly is a feature of Morquio syndrome, osteogenesis imperfecta and thanatophoric dysplasia. Shortened horizontally orientated ribs are associated with asphyxiating thoracic dysplasia.
1305
A 45 year old female patient has been referred to the orthopaedic service with anterior right knee pain, exacerbated by climbing stairs. A radiograph is unremarkable. Chondromalacia patellae is suspected clinically. The patient is mildly claustrophobic and the radiographer from the MRI scanner asks you which sequences you would like to prioritise in case the patient cannot tolerate the whole scan. What sequence would be most helpful in assessing the cartilage? a. Axial T1 weighted b. Axial proton density fat saturation c. Coronal short tau inversion recovery (STIR) d. Sagittal T1 post gadolinium e. Sagittal T1 weighted
(b) Axial proton density fat saturation Chondromalacia patellae is caused by patella cartilage degeneration leading to knee pain, often in young women, which is exacerbated by certain movements such as squatting, kneeling or climbing stairs. Knee radiograph may demonstrate degenerative changes or effusion, but is often normal. MRI is the best modality to assess for the condition; for assessing cartilage either T2 or proton density fat saturation sequences are most helpful. Focal increased signal in the cartilage along with cartilage irregularity is abnormal. T1 weighted sequences may show low signal in the adjacent patella if there is marrow oedema. STIR imaging is best for assessing inflammatory changes such as occult fracture, marrow oedema and soft tissue inflammation. Post gadolinium sequences can be helpful when assessing the synovium.
1306
A 25 year old woman has a simple, partial seizure and goes on to have a CT head. This shows a focal, 9-mm hyperdense mass in the left centrum semiovale. There is speckled calcification within the lesion. The patient is suspected to have a cerebral cavernoma and a MRI brain is requested. What is the best MRI sequence to diagnose cerebral cavernomas? a. T2* b. T2 c. T1 post contrast d. Diffusion weighted imaging e. FLAIR
(a) T2* Cavernomas are venous malformations with a low risk of haemorrhage and therefore many are incidental lesions. However, when symptomatic they can present with headaches and seizures. T2* or gradient echo are the best sequences to help diagnose cavernomas as they demonstrate blooming or susceptibility artefact. On T1 post contrast they can have a ‘popcorn’ appearance with central enhancement. On T2 weighted imaging they show peripheral low signal intensity due to haemosiderin deposition.
1307
You are asked to review a plain film by a junior doctor working in the emergency department who is concerned about the appearance of the mediastinum. The case is that of a 25 year old male who has presented with a productive cough and a fever. You review the chest radiograph; there is consolidation in the left lower zone which obscures the left hemidiaphragm, the airways appear dilated and abnormally thick walled and there is evidence of dextrocardia. You look back at the patient’s previous imaging and see the patient had a CT for investigation of sinusitis. What is the most likely unifying diagnosis? a. Cystic fibrosis b. Hypogammaglobulinaemia c. Post-infective bronchiectasis d. Primary ciliary dyskinesia e. Recurrent aspiration
d) Primary ciliary dyskinesia Kartagener syndrome is a type of primary ciliary dyskinesia, typically consisting of a triad of dextrocardia, bronchiectasis and sinusitis. It has an autosomal recessive mode of inheritance. It is also associated with infertility in males, reduced fertility in females, transposition of the great vessels, pyloric stenosis, post-cricoid web and epispadias. The patient has presented with pneumonia which is caused by ciliary dysfunction and can be a recurrent problem in patients with this condition. Post infective bronchiectasis is the most common acquired cause of bronchiectasis, including allergic bronchopulmonary aspergillosis (ABPA). Recurrent aspiration and immunodeficiency such as hypogammaglobulinaemia also cause bronchiectasis, but are not associated with dextrocardia. Cystic fibrosis is the most common congenital cause of bronchiectasis.
1308
A 12 year old girl presents to the emergency department with pain weight bearing on her right leg after a school hockey tournament. Pelvic radiograph demonstrates a small linear flake of bone adjacent to the right ischial tuberosity. Which tendinous avulsion is most likely to cause this appearance? a. Adductor magnus b. Gracilis c. Iliopsoas d. Rectus femoris e. Tensor fascia lata
(a) Adductor magnus The ischial tuberosity is the origin of the hamstring muscles. These include biceps femoris, semimembranosus and semitendinosus. The ‘hamstring portion’ of the adductor magnus muscle also arises from this site. Avulsion fractures are more common in children due to their immature skeleton, and appearances may be subtle with just small bone flakes visible on close inspection. Avulsion fracture sites of the other tendons around the pelvis and proximal femora are listed below. Gracilis – inferior pubic ramus Iliopsoas – lesser trochanter of the femur Rectus femoris – anterior inferior iliac spine Tensor fascia lata – anterior superior iliac spine Sartorius – anterior superior iliac spine
1309
An 84 year old care home resident is brought into hospital with fever, abdominal pain, distension and vomiting. The patient is referred to the surgeons, made nil by mouth and an attempted nasogastric tube insertion is unsuccessful. Chest radiograph demonstrates right lower lobe consolidation and an air-fluid level projected over the heart extending just to the right of the midline. A chest radiograph taken 1 year previously demonstrates a smaller retrocardiac air-fluid level. A current abdominal radiograph demonstrates no gas-filled dilated small or large bowel loops. Inflammatory markers, lactate and renal function tests are raised. Liver function tests are normal. Which is the most likely diagnosis? a. Epiphrenic diverticulum b. Hiatus hernia with aspiration pneumonia c. Lung abscess d. Mesentero-axial gastric volvulus e. Organo-axial gastric volvulus
e) Organo-axial gastric volvulus The vomiting, abdominal pain, retrocardiac air-fluid level, inability to pass a nasogastric tube and raised lactate is consistent with gastric volvulus. The elevated lactate is concerning for ischaemia. Organo-axial gastric volvulus is more common in adults and often associated with a diaphragmatic defect, sometimes following trauma or with a para-oesophageal (rolling) hiatus hernia. The previous chest radiograph demonstrating an air-fluid level is suggestive of this. This type of volvulus can be chronic and asymptomatic; however, the appearance of the air-fluid level increasing and its position, along with the other clinical symptoms and signs, suggests that the stomach has volved and become trapped above the diaphragm. Mesentero-axial gastric volvulus is more common in children and also more frequently associated with vascular compromise due to twisting of the mesentery. Epiphrenic diverticulum can occur secondary to conditions with oesophageal dysmotility and raised oesophageal pressure, such as achalasia. The presentation in this case is not typical as they often present with dysphagia and regurgitation. This patient does likely have a hiatus hernia and subsequent aspiration pneumonia could cause raised inflammatory markers and a raised lactate; however, this would not account for the abdominal pain, vomiting and inability to pass a nasogastric tube. A lung abscess is a differential for an air-fluid level in the thorax; however the other features in this case are not typical.
1310
A 59 year old male patient has an MRI spine for back pain which incidentally identifies an exophytic right sided renal mass. This measures up to 35 mm and is homogenously low signal on both T1 and T2 weighted images. A subsequent unenhanced CT followed by a contrast enhanced scan visualises the renal mass. This has a density of 35HU which increases to 40HU following contrast. Additional MRI sequences are performed which confirm restricted diffusion of the mass and similar enhancement characteristics to the CT. Based on the imaging characteristics, what is the most likely diagnosis? a. Clear cell carcinoma b. Haemorrhagic renal cyst c. Oncocytoma d. Papillary carcinoma e. Renal lymphoma
(d) Papillary carcinoma The mass has imaging features of papillary carcinoma of the kidney which is the second most common malignant renal tumour following the clear cell subtype. Papillary tumours are often hypovascular and poorly enhancing, hence the minimal change in density following contrast administration during the CT. The MRI findings of homogenous hypointensity on T1 and T2 weighted images are also consistent with this diagnosis. Clear cell carcinomas are usually T2 hyperintense; they enhance more and can be more heterogenous in appearance. Similarly, oncocytomas are usually heterogenous and can also demonstrate a central scar and pseudocapsule. Renal lymphoma frequently causes multiple masses and diffuse renal infiltration rather than a discrete mass. Like papillary carcinoma it can be poorly enhancing; however it is usually T1 hypointense and T2 iso- or hyperintense. A haemorrhagic renal cyst would typically be T1 hyperintense and T2 hypointense. An uncomplicated cyst would not enhance and, in contrast to the lesion in this case, it would not demonstrate restricted diffusion. Layering of debris may be seen within a haemorrhagic cyst.
1311
A 67 year old male patient with a history of hypertension and diabetes is brought to hospital by ambulance with sudden onset chest pain radiating to his back. A CT aorta confirms a Stanford Type B and Debakey Type IIIB aortic dissection. Following the CT scan he develops bladder and bowel dysfunction and bilateral lower limb motor impairment. Neurological assessment reveals bilateral proprioception is intact but pain sensation is absent. Which of the below is/are most likely to be affected? a. Artery of Adamkiewicz b. Artery of Percheron c. Posterior spinal arteries d. Recurrent artery of Heubner e. Vertebral arteries
(a) Artery of Adamkiewicz The distribution of neurological symptoms suggests an anterior spinal cord infarct. The artery of Adamkiewicz can have a variable origin usually arising between T9 and L2. It is the dominant arterial supply to the lumbosacral cord segments. Occlusion of this artery should be considered in patients with sudden onset chest pain caused by aortic dissection and evolving neurological symptoms. It can also be damaged during spinal surgery or abdominal aortic aneurysm repair. The artery of Percheron is variant anatomy of the posterior cerebral circulation which is characterised by a single artery arising from the posterior cerebral artery supplying the bilateral thalami and midbrain. The posterior spinal arteries are paired arteries arising from the vertebral or posterior inferior cerebellar arteries and they supply the posterior columns of the spinal cord. Occlusion of these arteries can lead to contralateral loss of vibration, proprioception and two-point discrimination. The recurrent artery of Heubner is also known as the medial striate artery and can be seen on angiography as the largest perforating branch of the anterior cerebral artery, arising from the A1 or A2 segments.
1312
An athletic 28 year old man presents with intermittent claudication affecting the right leg particularly when running or playing sports. Examination is normal with present lower limb pulses. The vascular team query popliteal artery entrapment syndrome and request further imaging to evaluate further and help plan surgical management. What would be the most appropriate test? a. Angiography b. AP and lateral right knee radiographs c. Arterial phase CT d. MRI e. Ultrasound
(d) MRI Popliteal artery entrapment syndrome is caused by symptomatic compression or occlusion of the popliteal artery by adjacent structures. It commonly presents in athletic males and is most frequently caused by the medial head of gastrocnemius or occasionally popliteus. It presents as intermittent claudication, which may be exacerbated by plantar flexion, or with thrombosis and can be bilateral in two-thirds. MRI is the favoured imaging modality because it demonstrates the underlying anatomy and aids surgical planning. Arterial phase CT and angiography would help to delineate the popliteal artery but the underlying soft tissue definition is better on MRI than CT. Similarly, ultrasound can help to demonstrate the artery and plantar flexion whilst scanning may reveal arterial compression, but MRI would still be favoured for surgical planning. A knee radiograph would not provide much benefit in this scenario.
1313
A 49 year old dialysis patient is referred to the rheumatology clinic with back and joint pain involving bilateral shoulders and hips. Radiographs of the hips and shoulders demonstrate erosions and subchondral cyst formation. Joint space is well preserved. An MRI spine reveals normal alignment. There is a low T1 and T2 signal lesion at the inferior endplate of the L3 vertebral body adjacent to the L3/4 disc. This lesion enhances following contrast. The L3/4 disc returns normal MRI signal. Similar low signal intra-articular nodules are seen on MRI of the right shoulder. Which finding would indicate the most appropriate diagnosis? a. Congo red staining of synovial fluid b. Cartilage pigmentation on arthroscopy c. Growth of Staphylococcus aureus on blood culture d. Serum parathyroid hormone levels e. Weakly positive birefringent crystals on polarised light microscopy of synovial fluid
(a) Congo red stain following biopsy The case describes amyloid arthropathy caused by amyloid deposition in the joints. This can be primary, or secondary to dialysis-dependent renal failure or multiple myeloma. It often presents as a large joint, bilateral, symmetrical arthropathy. Shoulder pain and carpal tunnel syndrome are common. Radiographic findings include preservation of joint space with subchondral cyst formation and well circumscribed erosions. MRI can reveal low T1 and T2 intra-articular nodules and bone lesions which enhance following contrast. Amyloid protein stains with Congo red. Cartilage pigmentation is seen in ochronosis, otherwise known as alkaptonuria. Homogentisic acid accumulates within cartilage leading to dark pigmentation and premature degeneration. Synovial pigmentation is a feature of pigmented villonodular synovitis due to haemosiderin staining. Staphylococcus aureus is associated with spondylodiscitis. Amyloid lesions in the spine can resemble discitis; however, the disc would be expected to have high T2 signal rather than the low signal described in the question. Secondary hyperparathyroidism is associated with chronic renal failure too but features include metastatic calcification and osteosclerosis with the ‘rugger jersey’ spine appearance. Weakly positive birefringent crystals are associated with calcium pyrophosphate deposition disease (CPPD), also known as pseudogout. This can be commonly misdiagnosed as osteoarthritis due to overlapping features but the distribution is more unusual and chondrocalcinosis is frequently present.
1314
A 55 year old man reviewed in the Infection and Immunity clinic has marked cachexia and is generally unwell. He is known to have HIV and has poor compliance with antiretrovirals; his CD4 count is 154 cells/mm3 (500–1400 cells/mm3). A chest radiograph is abnormal, and the patient is referred for a CT chest abdomen pelvis to investigate further. CT reveals multiple perihilar and peribronchovascular ill-defined nodular opacities with surrounding ground glass opacification and marked bilateral enhancing hilar lymph node enlargement. The abdomen and pelvis are normal. He is considered too unwell for a bronchoscopy and so you suggest a gallium scan to aid diagnosis. The thoracic abnormalities are not tracer avid. Which diagnosis is most consistent with the above clinical information? a. Cytomegalovirus b. Kaposi sarcoma c. Lymphoma d. Mycobacterium avium e. Pneumocystis pneumonia
(b) Kaposi sarcoma The perihilar and peribronchovascular nodular opacities describe the ‘flame-shaped’ nodular opacities seen in Kaposi sarcoma, accompanied by the lymph node enlargement. This condition is seen in patients with HIV and CD4 count <200 cells/mm3 . The main differential diagnosis in this patient is lymphoma; however lymphoma associated with HIV (mostly non-Hodgkin) is usually associated with disseminated extra-nodal disease involving the CNS, GI tract and bone marrow. Lack of avidity on gallium scan is more in favour of Kaposi sarcoma. Lymphoma is gallium avid. Pneumocystis pneumonia can occur in patients with CD4 count <200 cells/mm3. The imaging appearances are of diffuse ground glass opacification but enhancing lymph node enlargement is not a feature. Cytomegalovirus (CMV) can be seen in patients with CD4 >200cells/mm3. The predominant imaging features of pulmonary CMV are multiple ground glass pulmonary nodules and consolidation. Mycobacterium avium is seen in patients with CD4 count <50 cells/mm3
1315
A 28 year old female patient with frequent abdominal discomfort, diarrhoea and weight loss has a positive faecal calprotectin test. Following a recent exacerbation of symptoms, an MRI abdomen is performed which demonstrates multiple radiological features of Crohn’s disease. Which MRI sequence is most helpful in identifying fistulae? a. T1 weighted sequence b. T2 weighted sequence c. Fat supressed T1 weighted sequence d. Fat supressed T2 weighted sequence e. Diffusion weighted imaging
d) Fat supressed T2 weighted sequence MRI is helpful in Crohn’s disease for monitoring disease activity and has the added benefit of no radiation, particularly for younger patients with the condition. Fistulae are possible in Crohn’s disease due to transmural bowel inflammation. Bowel wall ulceration can progress to full thickness fistulae which can communicate between bowel loops as well as between bowel and bladder, perineum or sometimes the vagina. Although all the sequences in the question may be employed for an MRI looking for manifestations of Crohn’s disease, fistulae can be visualised particularly well with fat supressed T2 weighted imaging, where they appear as a high signal tract. Following contrast administration the fistulae will enhance and post contrast enhancement can also highlight other areas of disease activity within the abdomen and pelvis.
1316
A neonate with dyspnoea has a chest radiograph which demonstrates a large lower zone opacity. An MRI scan helps confirm a congenital diaphragmatic hernia containing abdominal fat and viscera. Where is the diaphragmatic defect most likely to be? a. Left posterolateral b. Left posteromedial c. Right anterolateral d. Right anteromedial e. Right posterolateral
(a) Left posterolateral The majority of congenital diaphragmatic herniae are the Bochdalek variety. These are large, associated with other conditions such as pulmonary hypoplasia and usually in a left posterolateral position. Retroperitoneal contents such as fat, the spleen and left kidney may herniate into them. They may be diagnosed on antenatal scans. Morgagni herniae are less common, usually smaller and in a right anteromedial position. The transverse colon and stomach may herniate. BochdaLek are Left sided and MoRgagni are Right sided. BBBB can be used to remember the features of Bochdalek hernia: Bochdalek, big, bad, back and lateral.
1317
A 10 year old boy’s spinal radiograph demonstrates posterior vertebral body scalloping, beaking of the vertebral bodies at the anteroinferior corners and shortened pedicles with a decreased interpedicular distance. Which of the following underlying congenital skeletal anomalies is consistent with these findings? a. Achondroplasia b. Asphyxiating thoracic dysplasia c. Hurler syndrome d. Morquio syndrome e. Thanatophoric dysplasia
(a) Achondroplasia Features associated with achondroplasia on a spinal radiograph include an anteroinferior beak causing a bullet-shaped vertebrae and posterior vertebral body scalloping. Decreasing interpedicular distance and short pedicles increases the potential risk of canal stenosis. The mucopolysaccharidoses are associated with posterior vertebral body scalloping, and Hurler syndrome does also share a similar anteroinferior beak, but the other features make achondroplasia more likely. The vertebral body shape associated with Morquio syndrome is an anterior central beak (this could be remembered as Morquio – middle) and there is often platyspondyly. Thanatophoric dysplasia is also associated with platyspondyly and the typical ‘telephone handle’ appearance of the long bones. It overlaps with achondroplasia as both conditions cause narrowing of the interpedicular distance. Asphyxiating thoracic dysplasia, also known as Jeune syndrome, tends to cause more thoracic abnormalities leading to a long narrow thorax and high clavicles.
1318
A 36 year old woman presents with sudden onset headache and visual disturbance. On examination she is found to have bilateral homonymous hemianopia and left third cranial nerve palsy. A diagnosis of pituitary apoplexy is suspected. Regarding imaging in suspected pituitary apoplexy, which of the following statements is most accurate? a. CT is sensitive for the diagnosis b. High T1 signal helps differentiate it from other pituitary masses c. Macroscopic haemorrhage is uncommon d. The pituitary gland rarely demonstrates restricted diffusion on MRI e. The pituitary gland usually shows peripheral enhancement
(e) The pituitary gland usually shows peripheral enhancement Pituitary apoplexy is caused by pituitary necrosis, which may be haemorrhagic. This is frequently secondary to an underlying pituitary lesion, such as a macroadenoma. CT is insensitive for the diagnosis, and the pituitary fossa is better imaged with MRI. The pituitary may show high T1 signal if the cause is haemorrhagic; however, this will also depend on the age of blood, and other haemorrhagic/proteinaceous/necrotic masses can also have high T1 signal. Enhancement is typically peripheral and the infarcted centre commonly demonstrates restricted diffusion
1319
A 45 year old male is under investigation for haematuria and proteinuria. He also has episodes of chest pain, shortness of breath and new haemoptysis. The renal team perform a renal biopsy which shows focal glomerulonephritis and a serologic cytoplasmic ANCA test is positive. A CT chest has features supportive of the diagnosis. Which of the following findings is most commonly seen on CT chest in this condition? a. Airway wall thickening b. Increased mediastinal or hilar lymph node enlargement c. Nodules with cavitation d. Nodules without consolidation e. Pleural effusion
(c) Nodules with cavitation The condition described in the main stem is ANCA associated granulomatous vasculitis or granulomatosis with polyangiitis (previously known as Wegener granulomatosis). Cytoplasmic ANCA test is specific for the diagnosis of the condition if there are typical clinical features. Clinical features include upper airway symptoms (rhinitis, sinusitis, otitis media, subglottic or bronchial stenosis), lower respiratory tract infections (cough, chest pain, dyspnoea) and glomerulonephritis (haematuria/proteinuria). The most common imaging abnormality is cavitation of nodules or consolidation. The other imaging features provided in the question are also seen in this condition but are less frequent. Nodules, masses and focal areas of opacification are associated with active inflammation. Ground glass opacification may also be seen which can represent pulmonary haemorrhage.
1320
A 55 year old female patient presents with a short history of abdominal pain, vomiting and a distended abdomen. A contrast enhanced CT abdomen pelvis confirms small bowel obstruction secondary to intussusception. The patient undergoes laparotomy which identifies a polypoid lesion as the lead point. Note is made of multiple other polypoid lesions in the small bowel and areas of darker skin on her fingers and toes. The patient reports a positive family history for an underlying genetic condition with her sister, brother and father also affected. Her sister is currently undergoing treatment for cervical adenoma malignum. What is the most likely underlying diagnosis? a. Carney complex b. Cowden syndrome c. Cronkhite-Canada syndrome d. Neurofibromatosis 1 e. Peutz-Jeghers syndrome
(e) Peutz-Jeghers syndrome The patient has a condition with an autosomal dominant inheritance pattern. There are several polyposis syndromes; however, the one associated with cervical adenoma malignum is Peutz-Jeghers syndrome. Multiple gastrointestinal polyps can lead to presentations for haemorrhage and bowel obstruction secondary to intussusception. Patients also frequently have oral mucocutaneous pigmentation which can also affect the fingers and toes. Carney complex is a combination of cardiac myxomas and blue skin pigmentation tending to affect the trunk and face, including the eyes and lips. Gastrointestinal polyps are not a feature. Myxomas can occur elsewhere in the body and patients are also predisposed to testicular tumours and pituitary adenomas. Cowden syndrome causes multiple gastrointestinal hamartomatous polyps as well as mucocutaneous lesions and increases a patient’s risk of cancers such as breast and thyroid primaries. Cronkhite-Canada syndrome is more common in males and does not have a recognised familial inheritance pattern. As well as gastrointestinal polyps, patients may have brown skin pigmentation, nail atrophy and alopecia. Neurofibromatosis 1 can cause cutaneous café-au-lait spots, amongst other clinical signs; however, it is not associated with gastrointestinal polyps
1321
The clinical team come to discuss the case of a neonate with you who had abnormal antenatal scans showing an anterior abdominal wall defect. The patient has been diagnosed with Turner syndrome. Which of the following group of features is most likely associated with the anterior abdominal wall defect?
(b) Midline defect, peritoneal covering present, herniated liver, no associated bowel complications The headings in this question help to differentiate an omphalocele from gastroschisis. Turner syndrome, along with other chromosomal abnormalities, is associated with an omphalocele whereas gastroschisis is usually sporadic. An omphalocele is a midline defect with a peritoneal covering and therefore ascites may be present. In contrast to this, gastroschisis is usually to the right of the midline, has no peritoneal covering and therefore no ascites. The liver herniates infrequently in gastroschisis and is more common with an omphalocele. Unlike in gastroschisis, where the bowel is in contact with amniotic fluid, bowel complications are not a feature of an omphalocele.
1322
A 28 year old female non-smoker presents to the emergency department with acute onset shortness of breath and left sided chest pain. She had a chest radiograph which demonstrates a left sided pneumothorax. The acute medical team site a chest drain and she undergoes a high resolution CT chest. The CT shows the drain sited appropriately with almost complete resolution of the pneumothorax. Within the lungs there are multiple, bilateral thin-walled cysts. These are uniform in size, distributed in all zones and have normal intervening lung parenchyma. What is the most likely underlying diagnosis? a. Birt-Hogg-Dubé syndrome b. Lymphocytic interstitial pneumonia c. Pneumocystis jiroveciinfection d. Pulmonary Langerhans cell histiocytosis e. Tuberous sclerosis
e) Tuberous sclerosis The imaging features are that of lymphangioleiomyomatosis (LAM), which is a rare interstitial lung disease, more common in women of childbearing age. It manifests as thin-walled cysts of uniform size with normal intervening lung parenchyma. Patients with the condition can present with a pneumothorax and sometimes they develop chylous pleural effusion. It is associated with tuberous sclerosis (approximately 40% of patients with tuberous sclerosis have LAM). Pulmonary Langerhans cell histiocytosis occurs most commonly in smokers. Early in the disease it is seen as 3- to 10-mm nodules in the mid-upper zones; later in the process the nodules undergo cystic degeneration. The cysts are typically irregularly shaped and can coalesce to form ‘bizarre’- shaped cysts. The lung parenchyma between cysts may demonstrate emphysematous change. Birt-Hogg-Dubé syndrome is a genetic multisystem disease which is characterised by multiple lung cysts; these are predominantly in the lower zones with variable morphology and internal septation. Lymphocytic interstitial pneumonia is associated with HIV and other connective tissue disorders such as Sjögren syndrome. Interstitial infiltrate causes findings of ground glass opacification, consolidation, centrilobular nodules and scattered pulmonary cysts. Pneumocystis jiroveciiis an infection of immunocompromised patients. The imaging finding in this fungal infection are typically of bilateral ground glass densities. Cysts can occur within the ground glass density, usually in the upper zones.
1323
A 42 year old female patient without significant past medical history presents to her GP with a 3 week history of feeling increasingly tired and feverish with generalised aches and pains. On examination, the chest is clear and the abdomen is soft and non-tender. The GP notes characteristic tender swollen red patches on both her lower limbs. A chest radiograph is requested and reports bilateral prominent hila but the lungs are clear. Which diagnosis is most appropriate? a. Chronic lymphocytic leukaemia b. Löffler syndrome c. Löfgren syndrome d. Sapho syndrome e. Sever disease
(c) Löfgren syndrome Löfgren syndrome is an acute form of sarcoidosis characterised by fevers, malaise, arthritis, lymph node enlargement and erythema nodosum. Leukaemia can also cause similar symptoms along with erythema nodosum; however chronic lymphocytic leukaemia usually affects an older age group. Löffler syndrome is another term for simple pulmonary eosinophilia and although sounding similar to Löfgren syndrome should not be confused with it. Patients are not usually particularly unwell, and chest radiograph should show transient infiltrates rather than being completely clear. Hilar lymph node enlargement is also not a feature. Sapho syndrome is relatively rare and is a condition causing synovitis, acne, pustolosis, hyperostosis and osteitis. Skeletally, the sternoclavicular joint is most commonly involved. Sever disease causes pain in the posterior foot due to calcaneal apophysitis.
1324
A 56 year old man with a background of hepatitis C undergoes a routine surveillance liver ultrasound. There is a 3-cm hyperechoic lesion in segment VI of the liver. A CT liver is performed. The lesion is arterially enhancing with rapid contrast washout. The case is discussed at the regional hepatocellular carcinoma MDT meeting. Which of the following statements are true regarding the management of hepatocellular carcinoma? a. Transarterial chemoembolisation (TACE) is a curative treatment option b. TACE and thermal ablation cannot be used together c. TACE is indicated in Child-Pugh score C and D patients d. TACE may be used for large, unresectable tumours e. Thermal ablation is a curative treatment option
(d) Transarterial chemoembolisation (TACE) may be used for large, unresectable tumours. Transplant remains the definitive, curative treatment for hepatocellular carcinoma (HCC). TACE is a non-curative, life prolonging treatment of HCC. It is indicated in large, unresectable tumours for patients with Child-Pugh scores A and B. Thermal ablation (radiofrequency ablation or microwave ablation) is indicated in small hepatic tumours not amenable to surgery, which is life prolonging but not curative. TACE can be used to reduce tumour volume prior to ablation. It can also be used post ablation to reduce recurrence.
1325
Antenatal scans for a 3 day old baby demonstrated polyhydramnios. A chest radiograph following a difficult insertion of a nasogastric tube depicts a coiled tube projected over the mediastinum. The patient has surgical management for oesophageal atresia. A subsequent upper gastrointestinal water soluble contrast study demonstrates a good result but reveals a short narrowing of the duodenum at D2. There are normal calibre gas-filled distal small and large bowel loops. A couple of hemivertebrae are noted. What is the most likely cause? a. Annular pancreas b. Duodenal atresia c. Duodenal diverticulum d. Duodenal web e. Pancreas divisum
a) Annular pancreas An annular pancreas is when pancreatic tissue encircles the duodenum at D2 close to the ampulla of Vater. This can be a complete or incomplete ring. It can be diagnosed incidentally if asymptomatic but it can be symptomatic, causing duodenal obstruction. It is associated with other conditions such as oesophageal atresia, tracheo-oesophageal fistula and Down syndrome. Pancreas divisum is the commonest congenital pancreatic abnormality caused by failure of ductal fusion leading to two ducts draining into the duodenum. MRCP and MRI pancreas is the most helpful test to diagnose the condition. It does not cause duodenal luminal narrowing. A duodenal diverticulum would appear as an outpouching from the duodenal wall which may be filled with gas, fluid or debris; these are often asymptomatic. There can be intraluminal diverticulum which can cause obstruction, similar to a duodenal web. Duodenal webs typically cause a ‘windsock’ appearance due to the membranous web projected into the duodenal lumen rather than an area of focal narrowing. Duodenal atresia usually presents very early with bilious vomiting caused by complete obstruction just distal to the ampulla of Vater. There would be no distal bowel gas visible in this scenario and a ‘double bubble’ appearance on abdominal radiographs.
1326
A 72 year old male patient presents to the emergency department with facial bruising and swelling. Radiographs of his facial bones reveal a fracture of the nasal bone and soft tissue swelling. You also notice a couple of lucent areas in the partially imaged skull which are well- defined without surrounding sclerosis. In his right proximal ulna there is bony expansion and a lytic lesion extending along the cortex. Subsequent nuclear medicine bone scan reveals tracer uptake in all these regions, particularly peripherally in the skull lesions. There is also diffuse uptake throughout the L3 vertebral body. What is the most likely underlying diagnosis? a. Fibrous dysplasia b. Langerhans cell histiocytosis c. Myeloma d. Metastatic disease e. Paget’s disease
e) Paget’s disease Paget’s disease has various phases. The initial lytic phase leads to bone resorption followed by laying down of abnormal bone with coarsened trabeculae and then later sclerosis predominates. Paget’s disease typically affects the skull, pelvis, spine and long bones. The skull lesions described in the question are in keeping with osteoporosis circumscripta and are usually well-defined without a sclerotic margin. The humeral lesion which is extending along the cortex is consistent with the ‘blade of grass’ sign which describes a lucent leading edge when Paget’s affects long bones. During the lucent phase of Paget’s disease there is often increased tracer uptake on nuclear medicine bone scan due to the osteoblastic activity. Fibrous dysplasia typically causes a ‘ground glass’ appearance within the bones rather than lysis. With myeloma there would usually be more diffuse skull and skeletal lytic lesions causing a ‘raindrop’ appearance when affecting the skull. Metastatic disease is possible, although again, more diffuse lesions may be expected; the distal long bones are also not typically affected initially. When there is malignant vertebral body marrow infiltration the uptake on nuclear medicine bone scan would usually be focal rather than the diffuse uptake described. Similarly, the peripheral uptake in the skull lesions is typical for osteoporosis circumscripta. Langerhans cell histiocytosis can cause lytic skull lesions but is not typical in this age group and lesions also tend to have bevelled, sclerotic margins.
1327
A 35 year old male patient with no fixed abode is admitted with bilateral flank pain and haematuria. The flank pain is worse on the right side. An abdominal radiograph reveals no urinary tract calcification; however, there is coarse calcification in the left upper quadrant. The left femoral head is noted to be flattened and sclerotic. Renal ultrasound identifies mild right hydronephrosis but no calcified calculi. There are similar findings following an unenhanced CT urinary tract. A contrast enhanced CT urogram is then performed which identifies a few small filling defects in the right renal pelvis. Which of the following conditions accounts for the findings in this case? a. Acute tubular necrosis b. Indinavir calculi c. Papillary necrosis d. Pyelouretitis cystica e. Pyonephrosis
(c) Papillary necrosis The findings of radiolucent filling defects in the renal collecting system along with other radiological features of sickle cell disease – avascular necrosis of the femoral head and splenic calcification – is consistent with papillary necrosis. This often presents with flank pain and haematuria. It is usually bilateral except in ascending infection or renal vein thrombosis and causes include sickle cell disease, tuberculosis, NSAID use and diabetes. Indinavir, associated with antiretroviral drugs used in HIV treatment, is one cause of radiolucent calculi; however there are no other features of this in the case. Acute tubular necrosis can cause debris in the collecting system but the history is usually one of an ischaemic or nephrotoxic event preceding its development. Ultrasound can demonstrate enlarged echogenic kidneys and if contrast is administered for a CT or IVU there may be a persistent nephrogram with minimal or absent filling of the collecting system. Pyonephrosis usually presents with fever. Imaging can demonstrate layered echogenic/high density debris and/or gas within the collecting system. There may be perinephric stranding and renal abscess. The history in this case is not consistent with pyeloureteritis cystica which is associated with urinary tract infection and causes benign cysts in the renal pelvis and ureter.
1328
A trauma CT scan is completed for a child following a road traffic collision. There are multiple pelvic fractures and evidence of haemorrhage within the pelvis. The CT has features of hypoperfusion complex. Which of the following is NOT consistent with hypoperfusion? a. Hyper enhancing kidneys b. Increased adrenal gland enhancement c. Increased splenic enhancement d. Small calibre aorta e. Thickened jejunal wall with hyperdense mucosa
(c) Increased splenic enhancement Hypoperfusion complex may be evident on CT scans for patients with hypovolaemic shock. Hyperdense adrenals and kidneys are due to increased enhancement and in the kidneys due to decreased renal excretion. In contrast to this, the liver and spleen can demonstrate reduced contrast enhancement. Both the aorta and inferior vena cava may be small calibre and there may be a ring of low density fluid surrounding the inferior vena cava. The bowel loops may be thickened with hyperenhancing mucosa; this most commonly affects the jejunum.
1329
A 53 year old presents with a 2 day history of seizures, reduced conscious level and fever. There is no history of trauma or significant medical history. An admission CT head 2 days previously was normal. An MRI reveals signal abnormality in the cingulate gyrus and asymmetrical signal in the insula cortices and posterior temporal lobe white matter and cortex with predominantly T2 and FLAIR hyperintensity. There is restricted diffusion in the areas of FLAIR hyperintensity and subtle gyral enhancement. There are small punctate foci of T2 hypointensity which are high signal on T1 sequences. The basal ganglia have normal appearances. What is the most likely underlying cause? a. Autoimmune encephalitis b. Epstein-Barr virus encephalitis c. Gliomatosis cerebri d. Herpes simplex encephalitis e. Middle cerebral artery territory infarct
(d) Herpes simplex encephalitis Herpes simplex encephalitis is the most common cause of viral encephalitis. The typical features are bilateral temporal lobe involvement; however this can be asymmetrical, and involvement of the insular cortex, cingulate gyrus and frontal lobes is also common. There is frequently T2 and FLAIR hyperintensity involving the white matter and cortex and low T1 signal due to oedema. Areas of haemorrhage may develop, which will be high on T1 and low on T2 sequences. There is often restricted diffusion and gyral and leptomeningeal enhancement. Encephalitis caused by other viruses, such as Epstein-Barr virus, can have similar imaging appearances; however, they are much less likely as HSV accounts for 90% of all viral encephalitis. Autoimmune encephalitis is one differential; however, haemorrhage is more common in HSV encephalitis and sparing of the basal ganglia is also more suggestive of HSV. Neurosyphilis is usually seen in association with HIV infection and can cause bacterial meningitis, encephalitis or arteritis leading to infarcts, often in the brainstem, basal ganglia and middle cerebral artery territory. HSV encephalitis is much more common and the distribution of signal changes in this case is typical for HSV encephalitis. Middle cerebral territory infarcts are sometimes a differential for the changes associated with HSV encephalitis due to the territory involved; however, the bilateral abnormalities in this case and the sparing of the basal ganglia are less consistent with infarct. Gliomatosis cerebri would be less likely in an acute presentation, especially with fever. The cerebral changes are often much more diffuse and confluent.
1330
A 40 year old female patient has lumbar spine and pelvic radiographs following a trampolining accident. The emergency department junior doctor asks you to review it as he is unsure if there is abnormality of the sacrum. There are no acute fractures. There is dense subchondral sclerosis at the inferior aspects of the bilateral medial iliac bones. Sacroiliac and hip joint spaces are well preserved. There is no cortical irregularity. What recommendation is the most appropriate? a. MRI sacrum with T1, T2 and STIR sequences b. No further action required c. Nuclear medicine bone study d. Review other previous plains films to assess for other skeletal manifestations e. Suggest clinical correlation and consideration of rheumatology referral
(b) No further action required The features described are of osteitis condensans ilii, a benign appearance described typically in women who have had children but also seen in nulliparous females and males. Appearances affect the iliac side of the sacroiliac joint with subchondral sclerosis which is often triangular in shape. The sacroiliac joint space is preserved and normal in appearance. It is usually an incidental finding, although there are reports of it causing lower back pain. In a case such as this where it is an incidental finding, no further action is required. If there were features of sacroiliitis with changes affecting both sides of the sacroiliac joints and the joint itself, then review of other skeletal films and/or rheumatology referral may be indicated. An MRI or nuclear medicine bone scan would not be appropriate for this indication but can be helpful in detecting insufficiency fractures of the sacrum; however this would normally affect an older age group.
1331
A neonate is born at 38 weeks’ gestation following elective caesarean section and develops respiratory distress at 5 hours following delivery. A chest radiograph demonstrates normal lung volumes and heart size with perihilar airspace opacification, a small right pleural effusion and a trace of fluid in the fissures. The baby starts improving clinically after 48 hours. What is the most likely cause of this child’s illness? a. Beta-haemolytic streptococcal pneumonia b. Meconium aspiration c. Patent ductus arteriosus d. Surfactant deficiency disease e. Transient tachypnoea of the newborn
(e) Transient tachypnoea of the newborn The history of the illness, with a baby born at term (>37 weeks’ gestation) by caesarean section, onset within 6 hours following delivery and improvement after 2–3 days is suggestive of transient tachypnoea of the new born. The radiograph is also consistent with this due to normal lung volumes, perihilar air space opacification and pleural fluid which resolves with symptom resolution. The history is not typical for meconium aspiration which is usually associated with a traumatic delivery. Meconium aspiration is most common in term or post-term infants and is associated with hyperinflation rather than normal lung volumes; however, other radiographic features do include perihilar changes and pleural effusions. Beta-haemolytic streptococcal pneumonia more commonly affects premature infants. In contrast to this case, it is usually associated with reduced lung volumes and diffuse opacity. There may be pleural effusions. Infant respiratory distress syndrome due to surfactant deficiency is also a condition affecting premature infants and is associated with reduced lung volumes. Pleural effusions are uncommon. Patent ductus arteriosus (PDA) is most common in premature infants. The most common congenital heart defects are ventricular and atrial septal defects. Patent ductus arteriosus can be asymptomatic if small, but if large they present with signs of heart failure due to the left to right shunt. PDA is an acyanotic congenital cardiac anomaly and children may have signs of cardiomegaly and pulmonary oedema. (The Final FRCR Complete Revision Not
1332
An MRI head is performed for a trauma patient on neurosurgical intensive care who is 5 days post- surgical decompression of a large right subdural haematoma following multiple skull fractures. The MRI shows a small right subdural collection, much smaller than the collection prior to decompression. It has intermediate T1 and high T2 signal. The collection demonstrates restricted diffusion and peripheral enhancement. Anterior to the left frontal lobe there is a thin, crescentic, T1 hyperintense collection which was not evident on the admission CT. There is no significant mass effect. On susceptibility weighted imaging (SWI) there are a couple of small areas of hypointensity at the grey−white matter interface in both hemispheres and in the corpus callosum. In the posterior fossa at the left cerebellopontine angle there is a 25-mm extra-axial lesion which demonstrates high T2 and low T1 and FLAIR signal without diffusion restriction or enhancement. It was also evident on the admission CT. After reviewing sagittal sequences you note that the cerebellar tonsils descend 4 mm below the foramen magnum. In terms of clinical relevance, which finding is most important to prioritise in your report? a. Bilateral regions of parenchymal SWI signal hypointensity b. Cerebellar tonsil position c. Left cerebellopontine angle lesion d. New collection adjacent to left frontal lobe e. Right subdural collection
(e) Right subdural collection The right subdural collection has features of a subdural empyema with rim enhancement and restricted diffusion. This requires urgent surgical evacuation and is therefore the most urgent clinically relevant finding. The collection adjacent to the left frontal lobe is thin and not causing significant mass effect. In the context of recent trauma, the areas of hypointensity on susceptibility weighted imaging at the grey−white matter interface and in the corpus callosum are consistent with diffuse axonal injury. It is important for the clinical team to be aware of this but it does not need to urgently be relayed to the them. The left cerebellopontine angle lesion is consistent with an arachnoid cyst, which is a benign finding. Similarly, the cerebellar tonsil protrusion is within normal limits. At 3–5 mm below the foramen magnum this would be termed as benign tonsillar ectopia. Over 5 mm would correlate with a Chiari I malformation.
1333
A 46 year old male non-smoker presents with increased shortness of breath on exertion. Chest radiograph demonstrates hyperexpansion with bilateral predominantly lower zone lucency. The pleural spaces are clear and mediastinal appearances are within normal limits. Pulmonary function tests are abnormal and reveal an obstructive pattern. A blood test confirms the diagnosis. What other imaging would be important in this patient? a. Cardiac MRI b. Carotid Doppler ultrasound c. Echocardiogram d. Abdominal ultrasound e. Renal Doppler ultrasound
(d) Abdominal ultrasound This case describes alpha 1 antitrypsin deficiency. This is diagnosed with measurement of alpha 1 antitrypsin serum levels. Radiological features in the lungs include predominantly lower lobe emphysema and bronchiectasis in relatively young patients. The condition can also cause liver cirrhosis. Emphysema and cirrhosis are common causes of death. Therefore, abdominal ultrasound is the correct answer as the patient will likely have signs of liver disease and if chronic, may have features of portal hypertension with splenomegaly, varices and ascites. Other conditions associated with alpha 1 antitrypsin deficiency include asthma, pancreatitis and panniculitis.
1334
A 45 year old male presents with acute, severe abdominal pain. This is localised to the right lower abdomen. A CT abdomen pelvis demonstrates a well-defined 3.5-cm area of homogenous, high attenuation fat with stranding within the mesentery adjacent to the ascending colon. The adjacent colon and small bowel are normal. What is the most likely diagnosis? a. Desmoid tumour b. Diverticulitis c. Epiploic appendagitis d. Omental infarction e. Mesenteric vein thrombosis
(d) Omental infarction The differential lies between omental infarction and epiploic appendagitis. Epiploic appendagitis is most common in the rectosigmoid and ileocaecal regions; however, omental infarction is most likely adjacent to the ascending colon. The fact that this lesion is relatively large (>3 cm) is also more consistent with omental infarction. Both conditions cause high attenuation within the fat but epiploic appendagitis classically has additional features which may aid differentiation, including a hyperdense rim and a central hyperdense dot. Diverticulitis and mesenteric vein thrombosis are both unlikely when the adjacent bowel has been described as normal. Desmoid tumours are rare and usually well-defined soft tissue masses, most commonly occurring in relation to the anterior abdominal wall, retroperitoneum or root of the mesentery.
1335
A 24 year old patient has an MRI pelvis following an abnormal cervical smear and subsequent examination under anaesthetic and biopsy reveals squamous cell carcinoma. The patient tests positive for human papilloma virus. The patient is going to be discussed at the gynaecology MDT meeting. Which initial staging imaging is most appropriate? a. CT chest/abdomen and MRI pelvis b. CT chest and MRI abdomen/pelvis c. CT chest/abdomen/pelvis and US pelvis d. 18F-FDG PET/CT and CT chest/abdomen/pelvis e. 18F-FDG PET/CT and MRI pelvis
e) 18F-FDG PET/CT and MRI pelvis Royal College of Radiology and National Comprehensive Cancer Network guidelines suggest that PET/CT is indicated for locally advanced cervical cancer (≥FIGO stage Ib2 disease) considered for radical chemoradiotherapy. Based on the information provided in this case the patient has a FIGO Stage IIa1 tumour; therefore the most appropriate initial imaging from the available choices would be 18F-FDG PET/CT and MRI pelvis. Previously, CT and MRI were the modalities of choice, with PET/CT reserved for more challenging cases; however, recent studies have found that PET/CT can detect additional avid nodes which may change management, for example by extending the radiotherapy field to include the paraaortic region. US pelvis does not have a role in staging for cervical cancer.
1336
A 15 month old with abdominal distension has an abdominal ultrasound. This identifies a large right upper quadrant mass, inseparable from the liver. The lesion contains multiple anechoic, septated cysts. The septae demonstrate vascularity. The kidneys, pancreas and spleen have normal appearances. There is no free fluid. The aorta is normal calibre. Serum alpha-fetoprotein is normal. What is the most likely diagnosis? a. Hepatic adenoma b. Hepatoblastoma c. Infantile haemangioendothelioma d. Mesenchymal hamartoma e. Metastatic Wilms tumour
d) Mesenchymal hamartoma Mesenchymal hamartomas are benign. They are commonly multiloculated cystic lesions; some may contain vascular solid components and septae. They can have a very similar appearance to an infantile haemangioendothelioma; however unlike these, the aorta is normal calibre with a mesenchymal hamartoma. Alpha-fetoprotein helps to differentiate between mesenchymal hamartoma and hepatoblastoma. Hepatoblastomas are also usually solid masses, although they may contain regions of haemorrhage and necrosis. Wilms tumour, also known as nephroblastoma, can metastasise to the lungs, liver and lymph nodes. The normal appearance of the kidneys in this case makes metastases unlikely. The diagnosis of hepatic adenoma is usually made in an older age group than this patient, classically in young females taking oral contraceptives. Their appearance can vary, but they are typically solid masses which may be heterogenous due to haemorrhage and fat.
1337
A 30 year old male is referred by the neurologists with a several month history of headache. He was found to have papilloedema. An MRI brain shows a well-defined intraventricular mass. This appears to arise from the septum pellucidum. It is T1 isointense and T2 hyperintense with multiple cystic regions. There is blooming artefact on susceptibility weighted imaging. It shows mild, heterogenous T1 enhancement. What is the most likely diagnosis? a. Central neurocytoma b. Choroid plexus papilloma c. Ependymoma d. Intraventricular meningioma e. Subependymal giant cell astrocytoma
a) Central neurocytoma These are WHO grade II tumours with characteristic imaging appearances. They are typically seen in young adults. They appear as heterogenous intraventricular masses, typically attached to the septum pellucidum with a variable enhancement pattern. Calcification is common. Patients often present with headaches related to raised cerebrospinal fluid pressure. Ependymomas are typically seen in the fourth ventricle and in younger patients. Intraventricular meningiomas would normally show more homogenous enhancement. Subependymal giant cell astrocytomas (SEGAs) are usually seen in patients with tuberous sclerosis and show avid contrast enhancement. Choroid plexus papilloma is generally seen in children and show avid enhancement.
1338
An MRI brain is performed for a stroke patient confirming a right middle cerebral artery territory infarct. Whilst reviewing the acquired images you note a well-defined midline nasopharyngeal mass, just deep to and elevating the mucosa of the posterior nasopharynx between the heads of the longus colli muscles. The mass is homogenous, T1 isointense and T2 hypointense. There is no restricted diffusion. Looking at a previous MRI from 1 year earlier, this was present at that time and has not changed. What is the most likely underlying cause? a. Branchial cleft cyst b. Ranula c. Rathke cleft cyst d. Thyroglossal duct cyst e. Tornwaldt cyst
e) Tornwaldt cyst Tornwaldt cysts are commonly asymptomatic, incidental findings on imaging of the head and neck. They are midline nasopharyngeal mucosal cysts and can have variable MRI signal depending on their protein content. On CT they can exhibit fluid density or be slightly hyperdense. Ranula are cystic lesions that occur at the floor of the mouth in the sublingual space, although they can extend to the submandibular region (diving type). Rathke cleft cysts are midline cysts but they occur intracranially adjacent to the pituitary gland in the sella. Branchial cleft cysts are usually away from the midline and present as palpable neck masses occurring at a variety of locations, ranging from near the external auditory canal to adjacent to the thyroid gland. Thyroglossal duct cysts cause palpable midline neck masses, most commonly in the paediatric population. They can be located anywhere along the course of the thyroglossal duct but they are most commonly infrahyoid.
1339
A 7 year old male is under investigation for vitamin C deficiency. Wrist radiographs demonstrate sclerotic epiphyseal margins, loss of epiphyseal density, dense metaphyseal lines and an exuberant periosteal reaction. What additional feature might be present on cross sectional imaging? a. Cloaca b. Coarsened trabeculae c. Haemarthrosis d. Involucrum e. Sequestrum
(c) Haemarthrosis Scurvy is caused by vitamin C deficiency which leads to abnormal collagen and bone development and a bleeding diathesis. Features include ground glass osteoporosis, sclerosis of the margins of the epiphysis (Wimberger sign), metaphyseal spurs, dense metaphyseal lines (lines of Frankel), Trummerfield zone (radiolucent zone on the diaphyseal line of the Frankel line), pencil-point cortical thinning, corner fractures, an exuberant periosteal reaction and haemarthrosis. Coarsened trabeculae are not features of the condition. Cloaca, involucrum and sequestrum are features of osteomyelitis.
1340
A 56 year old female smoker has ongoing shortness of breath and reduced exercise tolerance. She is known to have a diagnosis of Sjögren syndrome. The GP requests a high resolution CT to assess for interstitial lung disease. The CT shows diffuse ground glass opacification. Which interstitial lung disease is most commonly associated with Sjögren syndrome? a. Acute interstitial pneumonia b. Desquamative interstitial pneumonia c. Lymphocytic interstitial pneumonitis d. Non-specific interstitial pneumonia e. Usual interstitial pneumonitis
(d) Non-specific interstitial pneumonia (NSIP) Sjögren syndrome is an autoimmune condition which affects the salivary glands and also has pulmonary associations. Whilst Sjögren syndrome is one of the only connective tissue diseases associated with lymphocytic interstitial pneumonitis (LIP), and the features described in the main stem could be in keeping with LIP. This condition is rare. The most common associated interstitial lung fibrosis is NSIP. This is demonstrated as predominantly ground glass change with reticular opacities and immediate subpleural sparing with traction dilatation of the small airways. Usual interstitial pneumonitis may also be seen in Sjögren syndrome and should be considered if honeycomb destruction of the lung parenchyma is the main abnormality. Desquamative interstitial pneumonia is more common in men and strongly associated with smoking. It is not associated with Sjögren syndrome. Acute interstitial pneumonitis is an acute form of the interstitial pneumonias; however it is of unknown aetiology and not associated with Sjögren syndrome.
1341
A 25 year old asymptomatic male doctor requires a chest radiograph to apply for a visa to work in Australia. The chest radiograph is performed which is of good quality, although slightly rotated to the left side. It demonstrates increased transradiancy of the whole right hemithorax. Lung markings are present; however there is paucity of the vascular markings on the right when compared to the left side. The hilum of the right side is thought to be marginally smaller than on the left; however this is difficult to determine accurately due to the rotation. What is the most likely aetiology? a. Large pulmonary embolus b. MacLeod syndrome c. Poland syndrome d. Pneumothorax e. Rotated chest radiograph
(b) MacLeod syndrome The finding is most likely due to MacLeod syndrome, also known as Swyer-James syndrome. It is often an incidental finding and is due to previous childhood infectious bronchiolitis. Radiographic findings are that of a hyperlucent lung with reduced lung markings and normal or small volume lung and hilum on the affected side. On HRCT there is also bronchiectasis and bronchial wall thickening. Poland syndrome is a congenital unilateral absence of the pectoralis major and minor muscles which causes increased transradiancy of the hemithorax. There may also be associated chest wall deformity. This is within the differentials for this case; however, it is a comparatively rare condition and not associated with the paucity of vascular markings and small hilum. Large pulmonary embolus may cause the imaging features of reduced lung markings. The ‘Westermark’ sign is a radiographic sign of focal increased lucency of the lung, thought to be secondary to occlusion of the pulmonary artery or due to vasoconstriction distal to the embolus. This sign is not commonly seen and the patient in the main stem is asymptomatic. Hyperlucency due to rotation occurs in the same hemithorax as the direction of rotation and so this cannot account for the findings in this stem. Pneumothorax is also an important consideration; however absent lung markings would be expected as well as presence of a lung edge
1342
A 4 year old girl is referred to the paediatric neurologists for investigation of recurrent seizures and global developmental delay. An MRI brain shows a thick band of abnormal T1 hyperintense, T2 hypointense signal intensity deep to the cerebral cortex. It parallels the cortex in contour and signal intensity. There is diminished overlying gyral sulcation and many of the cortical sulci are shallow. Which of the following conditions correlates best with these features? a. Band heterotopia b. Polymicrogyria c. Schizencephaly d. Subependymal heterotopia e. Subcortical heterotopia
(a) Band heterotopia Band heterotropia is a diffuse form of grey matter heterotopia almost exclusively affecting females. It is associated with seizures and developmental delay. On imaging, this condition is characterised by a band of grey matter located deep to and roughly paralleling the cortex. Polymicrogyria is another form of cortical maldevelopment where there are numerous small gyri. Schizencephaly is a rare cortical malformation that manifests as a grey matter−lined cleft extending from the ependyma to the pia matter. Subependymal and subcortical heterotopia are more nodular types of heterotopia rather than the diffuse types (band and lissencephaly
1343
An 82 year old, unkempt woman who lives alone is admitted with acute confusion. On clinical examination she is noted to be ataxic. A CT head is unremarkable. MRI brain demonstrates bilateral high T2 and FLAIR signal within the mammillary bodies and medial thalami. Further high signal is seen within the pons. Following gadolinium administration there is enhancement of the mammillary bodies bilaterally. What is the most likely diagnosis? a. Cerebral lymphoma b. Leigh syndrome c. Osmotic demyelination syndrome d. Pontine glioma e. Wernicke encephalopathy
(e) Wernicke encephalopathy Wernicke encephalopathy is caused by thiamine (vitamin B1) deficiency and is typically seen in alcoholics or those who self-neglect. It presents with the classic triad of confusion, ataxia and ophthalmoplegia. Korsakoff psychosis is the chronic form of the condition and is characterised by confabulation and memory loss. MRI findings include symmetrical high signal on T2 and FLAIR sequences with post-contrast enhancement in the mammillary bodies, basal ganglia, paraventricular/medial thalamic regions, brain stem and periaqueductal grey matter. Korsakoff syndrome is associated with mammillary body atrophy and dilatation of the third ventricle. Leigh syndrome can have similar imaging appearances but occurs in young children and the mammillary bodies are spared. Pontine glioma are also more common in children. Furthermore, the supra- and infratentorial abnormalities in this case are not consistent with this diagnosis. Osmotic demyelination is possible in patients with chronic alcohol misuse due to rapid correction of hyponatraemia; however, the radiological features are more consistent with Wernicke encephalopathy.
1344
A 27 year old male presents with a firm lump arising around the left elbow. Elbow radiographs demonstrate a pedunculated bony outgrowth from the humerus consisting of the cortex and medulla. MRI confirms the presence of a cartilage cap. The most likely diagnosis is considered to be a benign osteochondroma. Which of the following is most likely to be true? a. The bony protrusion is directed away from the joint b. The humerus is most commonly affected c. The cartilage cap is 2.2 cm in thickness d. There are likely to be multiple osteochondromas e. The stem is prone to fracture
(a) The bony protrusion is directed away from the joint An osteochondroma is a cartilage capped exostosis, classically directed away from the joint space. It most frequently affects the lower limbs. It is the most common benign bone tumour, seen between the ages of 2 and 60 years. Cartilage cap thickness of >2 cm on CT and >1 cm on MRI is indicative of chondrosarcomatous change. This occurs in less than 1% of cases. Fracture of the stem is rare; the lesion is usually identified incidentally or with an enlarging painless lesion. Rarely they may be multiple, such as in diaphyseal aclasis, which is also known as hereditary multiple osteochondromatosis.
1345
A 6 month old girl is referred for investigation for suspected congenital cytomegalovirus (CMV) infection due to delayed neurological development. Which of the following features is more typically associated with congenital rubella infection than with congenital CMV infection? a. Hepatomegaly b. Hydrocephalus c. Cardiac anomaly d. Microcephaly e. Ultrasound showing hyperechoic foci in a periventricular distribution
(c) Cardiac anomaly Cytomegalovirus (CMV) is the most common intrauterine infection and intracranially can cause hydrocephalus, microcephaly, delayed myelination and cerebral calcification. The calcification tends to be periventricular and will be hyperechoic on a cranial ultrasound. More generally, it can cause intrauterine growth restriction and in the abdomen can cause hepatomegaly. There is overlap between many of the features in the congenital TORCH infections; however, CMV is not usually associated with congenital cardiac anomalies, whereas rubella frequently is
1346
The MRI brain of a 47 year old female neurology inpatient is reviewed. The patient is cardiovascularly stable and immunocompetent. The MRI has features of multiple sclerosis. However, there is also a left temporal lobe lesion. It is T2 hyperintense but the central signal is T1 hypointense. There is surrounding white matter oedema, mass effect and a complete rim of peripheral enhancement. Restricted diffusion is present peripherally, but not centrally. MRI perfusion finds that relative cerebral blood flow is markedly increased both in and surrounding the lesion. What is the most likely diagnosis of the left temporal lobe lesion? a. Abscess b. Glioblastoma c. Lymphoma d. Metastases e. Tumefactive multiple sclerosis
b) Glioblastoma The key features that make glioblastoma more likely are the central fluid signal suggesting necrosis, the complete rim of enhancement, mild restricted diffusion and markedly increased relative cerebral blood flow both in and around the lesion. The patient has radiological features of multiple sclerosis (MS). The left temporal lobe lesion is different and the central fluid signal is concerning for necrosis and raises the possibility of a malignant lesion, infective process or tumefactive multiple sclerosis. Lymphoma usually enhances homogenously (the exception is in patients with AIDS where there may be central necrosis and rim enhancement). The relative cerebral blood flow (rCBF) may be slightly raised with lymphoma but not to the same degree as a glioblastoma. Differentiating ring enhancing lesions is important on an MRI brain. Tumefactive multiple sclerosis is an important differential in a patient with other signs of MS; however, unlike in this case, the rim of enhancement is usually incomplete and rCBF is usually reduced. Intracranial abscesses would usually have restricted diffusion centrally and this is a useful differentiating factor between them and other pathologies. rCBF is also usually reduced compared to glioblastoma. Metastases can have a similar appearance to glioblastoma; however the rCBF in the oedema surrounding the lesion is usually reduced.
1347
An 18 month old boy presents to the emergency department with his parents following two episodes of bright red blood in his nappy. He is cardiovascularly stable. There is no significant medical history and he was born at term by an uncomplicated vaginal delivery. The paediatric team suspect Meckel’s diverticulum and ask for your advice regarding imaging. Which is the most appropriate test to confirm the diagnosis? a. Angiography b. CT angiography c. Small bowel fluoroscopy d. 99mTc-Na-pertechnetate scintigraphy e. Ultrasound
(d) 99mTc-Na-pertechnetate scintigraphy Meckel’s diverticulum may be an incidental finding; however haemorrhage or small bowel obstruction are the most common causes of symptomatic presentations. Bleeding is often secondary to ulceration of ectopic gastric mucosa due to a persistent omphalomesenteric artery. It can be treated in the interventional radiology suite. In the paediatric population 95% of diverticulum presenting with bleeding will contain ectopic gastric mucosa and scintigraphy is most sensitive in this population. CT angiogram is of limited value and also radiation dose would need to be considered in a paediatric patient. Identifying a diverticulum with ultrasound can be challenging and similar to small bowel fluoroscopy, and cannot confidently exclude the diagnosis. Angiography may be employed; however, it would not typically be a first-line investigation.
1348
A 31 year old female patient has an MRI pelvis following a completed miscarriage. The sonographer in the early pregnancy assessment unit was concerned about a uterine anomaly and so an MRI was arranged. The uterine cavity is bisected by soft tissue extending to the internal cervical os. The external fundal uterine contour is convex. There is one cervix, one vagina and two ovaries and fallopian tubes. Which Müllerian duct anomaly does this patient have? a. Arcuate uterus b. Bicornuate uterus c. Septate uterus d. Unicornuate uterus e. Uterus didelphys
(c) Septate uterus Müllerian duct anomalies occur during foetal life and may not become apparent until the woman reaches childbearing age, with difficulty conceiving or recurrent miscarriage. All can be associated with renal anomalies and therefore it is important to always assess the kidneys too. The most common anomaly is an arcuate uterus which is considered a normal variant. There is mild indentation of the endometrium at the fundus. This is not thought to have an impact on the ability to conceive or maintain a pregnancy. A septate uterus is caused by the incomplete resorption of the uterovaginal septum and the septum can be ‘partial’ or ‘complete’, extending to either the internal or external os. The external fundal contour of the uterus helps to differentiate a bicornuate from a septate uterus. A bicornuate uterus has a septum too; however, in contrast to a septate uterus it cannot be resected due to the risk of uterine perforation. Uterus didelphys is complete duplication of the uterine horns leading to two separate uterine cavities and cervices. A unicornuate uterus is where there is just one uterine horn and fallopian tube and there may be a rudimentary horn on the contralateral side. Renal anomalies are most common with this subtype. Uterine agenesis is when the uterus has not formed at all and the upper two-thirds of the vagina will also be absent; however, the ovaries and fallopian tubes may or may not be present.
1349
A 4 year old girl presents with fever, rash, conjunctivitis and bilateral cervical lymph node enlargement. Chest radiograph demonstrates bilateral atelectasis and a small right pleural effusion. A diagnosis of Kawasaki disease is made. Which of the following radiological findings would be consistent with this diagnosis? a. Abdominal aortic wall thickening b. Calcified splenic artery aneurysm c. Enlarged kidneys with loss of corticomedullary differentiation d. Renal artery microaneurysms e. Thin-walled dilated gallbladder
(e) Thin-walled dilated gallbladder Kawasaki disease is a small or medium vessel vasculitis. Symptoms can include fever, rash, desquamation of the hands, a red tongue, conjunctivitis and lymph node enlargement. Coronary artery aneurysms can be encountered in some patients. A thin-walled dilated gallbladder is associated with the condition. Abdominal aortic wall thickening would be associated with a large vessel vasculitis such as Takayasu arteritis. Renal artery microaneurysms are seen in a small and medium size vessel vasculitis such as polyarteritis nodosa. Enlarged kidneys with loss of corticomedullary differentiation is suggestive of glomerulonephritis. This may be encountered with conditions such as Goodpasture syndrome
1350
A 25 year old man with a background of Cowden syndrome presents to the neurologist with left sided dysarthria and ataxia. An MRI brain is arranged. This shows a mass-like appearance of the left cerebellum with widened left cerebellar folia and a striated appearance. The affected area is T1 hypointense and T2 hyperintense and demonstrates high signal on both DWI and ADC. The lesion does not enhance. Supratentorial appearances are within normal limits. What is the most likely diagnosis? a. Cerebellitis b. Cerebellar infarction c. Cerebral lymphoma d. Demyelination e. Lhermitte-Duclos disease
(e) Lhermitte-Duclos disease This is a rare tumour of the cerebellum, associated with Cowden syndrome, which is likely hamartomatous in origin. It is a WHO grade I tumour. It has a classical ‘tigroid’, striated appearance on MRI, which has been described in the question. A cerebellar infarct would be unusual in this age group and also typically demonstrates restricted diffusion. Acute demyelination, for example in multiple sclerosis, can affect the cerebellum, causing ataxia; however, other lesions would be likely, including supratentorially, around the lateral ventricles for example. Cerebellitis is more common in a younger group of patients, frequently as a post-infective condition leading to ataxia. Radiological manifestations include cortical FLAIR hyperintensity, restricted diffusion, oedema and cortical and leptomeningeal enhancement. Lymphoma also usually enhances and has restricted diffusion. Furthermore, primary CNS lymphoma is most commonly supratentorial.
1351
A 56 year old female is under investigation for increasing shortness of breath. She has a CT chest which shows symmetrical honeycomb destruction of the parenchyma in the lower lobes with scattered ground glass opacification and basal volume loss. The oesophagus is markedly patulous. What is the most likely diagnosis? a. Achalasia b. Non-specific interstitial pneumonitis c. Recurrent aspiration d. Scleroderma e. Usual interstitial pneumonitis
d) Scleroderma The description is typical for scleroderma, also known as systemic sclerosis. This is a connective tissue disease affecting multiple organs. In this condition the oesophagus is commonly involved and looks patulous. The description of basal interstitial lung disease is common. The predominance of honeycombing is more in keeping with usual interstitial pneumonitis (UIP). Both UIP and non- specific interstitial pneumonitis can be seen in the condition. Recurrent aspiration can cause infection, bronchiectasis and scarring. Achalasia may cause oesophageal dilatation with hold-up of food debris; however, achalasia does not account for the UIP picture of fibrosis.
1352
A neonate who was diagnosed with truncus arteriosus in the antenatal period has a chest radiograph. Which of the following radiographic features is most likely to be observed in this child?
(b) Enlarged heart, plethoric lungs, left-sided aortic arch Truncus arteriosus is due to a failure of the normal division of the primitive truncus arteriosus into the aorta and pulmonary artery. A single vessel leaves the heart giving rise to the pulmonary, coronary and systemic arteries. The condition causes cyanosis and cardiac failure. Chest radiograph commonly demonstrates cardiomegaly, pulmonary plethora and forked ribs. The aortic arch is right-sided in around a third of cases.
1353
A 7 year old boy is referred to a paediatric neurologist with lower than normal IQ and recurrent seizures. He is noted to have a facial rash. A diagnosis of tuberous sclerosis is suspected and an MRI brain is performed which confirms the diagnosis. Which of the following imaging findings is most likely to be present in this child? a. Focal areas of signal intensity in the deep white matter b. Subcortical calcification c. Optic nerve gliomas d. Subependymal giant cell astrocytoma e. Prominent leptomeningeal enhancement
(d) Subependymal giant cell astrocytoma Tuberous sclerosis is an autosomal dominant disease which can present with seizures, development delay and, in around 75% of cases, a skin condition called adenoma sebaceum. Central nervous system features include subependymal hamartomas, cortical tubers, heterotopic grey matter islands and subependymal giant cell astrocytoma. Optic nerve gliomas and focal areas of signal intensity (FASIs) are associated with NF1. Subcortical calcification and prominent leptomeningeal enhancement is associated with Sturge-Weber syndrome.
1354
A middle-aged male patient with a low CD4 count, secondary to poorly controlled HIV infection, is admitted as an inpatient due to deteriorating neurological function. An MRI brain reveals normal cerebral volume and no evidence of hydrocephalus. There are multifocal white matter abnormalities with high T2 signal and low T1 signal. This is primarily affecting the right parieto-occipital region, and to a lesser degree the left parietal lobe and subcortical right frontal lobe. There is no associated mass effect, no enhancement and minor peripheral restricted diffusion. What is the most likely diagnosis? a. Cerebral toxoplasmosis b. Cytomegalovirus encephalitis c. HIV encephalopathy d. Posterior reversible encephalopathy syndrome e. Progressive multifocal leukoencephalopathy
(e) Progressive multifocal leukoencephalopathy Progressive multifocal leukoencephalopathy is associated with immunocompromised states due to reactivation of the JC virus. The key distribution is in the white matter; it is asymmetrical and typically parieto-occipital and subcortical. There is T2 hyperintensity and T1 hypointensity, minimal, if any enhancement and no mass effect. There may be restricted diffusion at the leading edge of the changes. In contrast to this, HIV encephalopathy is symmetrical and more typically frontal and focused on the periventricular white matter and centrum semi ovale. Cerebral atrophy is also a feature. Although there is similar T2 hyperintensity and no enhancement or mass effect, there are not usually the same T1 signal changes. Cytomegalovirus encephalopathy causes T2 hyperintense brainstem and periventricular white matter abnormalities. These do not cause mass effect or enhancement unless there is ventriculitis, in which case there will be ependymal enhancement. Posterior reversible encephalopathy syndrome classically also affects the parieto-occipital regions with minimal, if any enhancement or diffusion restriction; however, the history is the key factor and does not fit in this case. Toxoplasmosis typically causes ring-enhancing lesions in the basal ganglia, thalamus and corticomedullary junction with mass effect.
1355
A 56 year old male who retired from the mining industry 10 years ago has investigations for worsening shortness of breath. The CT chest shows tiny calcified nodules predominantly in the upper lobes. In the right mid-zone the nodules coalesce and there is upper lobe fibrosis with volume loss and elevation of the hila bilaterally. The hila lymph nodes demonstrate eggshell calcification. What is the most likely diagnosis? a. Caplan syndrome b. Coal workers’ pneumoconiosis c. Hypersensitivity pneumonitis d. Pulmonary alveolar proteinosis e. Silicosis
(e) Silicosis The correct underlying diagnosis is silicosis. This is a pneumoconiosis found in those in sandblasting or mining industries, secondary to inhaled silica. The features of tiny calcified pulmonary nodules and calcified hila lymph nodes is indicative of this condition, complicated by progressive massive fibrosis (PMF). PMF occurs when the nodules coalesce and form large opacities in the upper zones with upper zone fibrosis. Coal workers’ pneumoconiosis (CWP) is secondary to inhalation of coal dust and can also produce calcified pulmonary nodules and hilar/mediastinal lymph node calcification; however eggshell calcification is more typical in silicosis. Silicosis is more likely than CWP to be complicated by PMF. Caplan syndrome is CWP with features of rheumatoid arthritis. Pulmonary alveolar microlithiasis is a rare idiopathic condition, in which diffuse, dense, miliary calcification is seen in both lungs, more conspicuous in the mid-lower zones. Hypersensitivity pneumonitis, also known as extrinsic allergic alveolitis, is a response to inhaled antigen. In chronic cases it can cause fibrosis, typically in the upper zones. Centrilobular nodules seen in this condition are not typically calcified.
1356
A Meckel diverticulum is diagnosed incidentally on an MRI small bowel performed for a patient with Crohn’s disease. Where is the diverticulum most likely to have been visualised? a. Antimesenteric border of the distal jejunum b. Antimesenteric border of the distal ileum c. Antimesenteric border of the proximal ileum d. Mesenteric border of the distal ileum e. Mesenteric border of the proximal jejunum
b) Antimesenteric border of the distal ileum Meckel’s diverticulum are most common in the terminal ileum on the antimesenteric border. The ‘rule of 2s’ suggests that they present in 2% of the population, 20% have ectopic gastric mucosa, they are 2 feet from the ileocaecal valve and usually around 2 inches long. If they present symptomatically, most commonly with haemorrhage or small-bowel obstruction, it is most likely to be in the first 2 years of life. Other presentations may include intussusception, Meckel’s diverticulitis, perforation or neoplasm. Many patients remain asymptomatic, and the chance of them causing problems decreases with age.
1357
A 15 year old female with physical features of a short neck and low posterior hairline has restricted neck movement. She has an MRI of the spine, which demonstrates scoliosis, spinal stenosis within the cervical spine, partial fusion of C2 and C3 as well as the presence of hemivertebrae. What is the most likely diagnosis? a. Ankylosing spondylitis b. Homocysteinuria c. Juvenile idiopathic arthritis d. Klippel-Feil syndrome e. Turner syndrome
(d) Klippel-Feil syndrome Klippel-Feil syndrome is a congenital abnormality of vertebral segmentation. Features include partial or complete fusion of C1 with the occiput, rib fusion, thumb anomalies (triphalangeal thumb, hypoplasia and polydactyly) and Sprengel deformity. Fusion of the posterior column of C2/3 is common and causes a restriction of neck movement. Other causes of fusion of the cervical spine include ankylosing spondylitis and juvenile idiopathic arthritis. Homocysteinuria is an autosomal recessive inherited defect which causes defects in collagen and elastin structure. Imaging features include osteoporosis, carpal bone abnormalities, scoliosis and biconcave vertebrae. Features in Turner syndrome include squared lumbar vertebrae and kyphoscoliosis.
1358
A 72 year old female patient with right lower limb intermittent claudication has a CT angiogram of her lower limbs which identifies widespread vascular calcification with a short tight stenosis of her right common iliac artery. The patient is prepared for an interventional radiology procedure to stent the stenotic segment. Endovascular access is straightforward and the stenosis is confirmed with digital subtraction angiography. Following positioning of the stent and dilatation of the stenosis, the patient’s blood pressure suddenly drops and the patient becomes tachycardic. What is the most appropriate immediate management? a. CT angiogram of the lower limbs b. Deploy embolisation coils at the site of the stenosis c. Re-inflate the balloon at the site of stenosis d. Re-inflate the balloon proximal to the stenosis e. Remove the vascular access sheath, wire and balloon
(d) Re-inflate the balloon proximal to the stenosis The concern following sudden drop in blood pressure and tachycardia is one of rupture of the right common iliac artery causing significant pelvic haemorrhage. The most appropriate immediate management is to inflate the balloon just proximal to the stenosis which has just been dilated, and presumably the site of the haemorrhage. Once the patient is stabilised and the site confirmed with digital subtraction angiogram, then a covered stent could be deployed to cover the defect. A CT angiogram of the lower limbs would take too long to organise and moving an anaesthetised patient to the CT scanner quickly could waste precious time. The cause can be both confirmed and treated in the interventional radiology suite, so it is more appropriate the patient stays where she is. Embolisation coils are used to treat haemorrhage, but they would not be routinely used for this indication and there would be significant risk of distal ischaemia where there is not good collateralisation. Removing the vascular access sheath, wire and balloon would be contraindicated as maintaining vascular access is essential to treat this complication.
1359
A 14 year old boy is referred to the paediatricians for recurrent chest infections. A chest radiograph shows a wedge-shaped opacity within the medial right lower zone. Bronchopulmonary sequestration is suspected, and a CT thorax is arranged. Which of the following features favour a diagnosis of intralobar sequestration over extralobar sequestration? a. Associated diaphragmatic hernia b. Venous drainage to pulmonary veins c. Enclosed in its own pleura d. Presents in neonates e. Systemic arterial supply from the aorta
(b) Venous drainage to pulmonary veins Pulmonary sequestration tends to present in childhood and is more common in males. The extralobar subtype usually presents earlier than intralobar sequestration. The anomaly is a segment of lung with no communication to the pulmonary arteries or bronchial tree. The most common type is intralobar. This is supplied from the descending thoracic aorta and drains via the pulmonary venous system. Intralobar sequestration is enclosed in visceral pleura. It does not have any associated conditions. In contrast to this, extralobar sequestration is associated with other conditions such as duplication cysts and cardiac anomalies. The segment has its own pleura. The arterial supply is from the aorta or sometimes the splenic, intercostal or gastric arteries. The venous drainage is systemic and not via the pulmonary veins.
1360
A 36 year old male patient is under the care of the respiratory team. He has a high resolution CT chest which demonstrates bilateral hilar and mediastinal lymph node enlargement. There is nodular interlobular septal thickening in the right lower zone. The patient undergoes an endobronchial ultrasound and lymph node biopsy which confirms the diagnosis. What is the most likely diagnosis? a. Lymphoma b. Stage 1 sarcoidosis c. Stage 2 sarcoidosis d. Stage 4 sarcoidosis e. Stage 5 sarcoidosis
(a) Lymphoma Although the imaging findings could be in keeping with sarcoidosis, the features of hilar and mediastinal lymph node enlargement, as well as the parenchymal infiltration would be in keeping with stage 3 sarcoidosis (according to the Siltzbach classification), which has not been given as an option. Lymphoma is the most likely correct answer in this case. Hodgkin’s lymphoma is more commonly a cause for enlarged lymph nodes than non-Hodgkin’s lymphoma. The posterior mediastinum is not frequently involved; however it is an important site for recurrence of disease, as it may not be included in the radiotherapy field. The presence of nodular interlobular septal thickening raises the possibility of lymphatic infiltration causing lymphangitis carcinomatosis, although this is rare.
1361
A 42 year old man without significant past medical history has returned to the UK after 5 years teaching in Asia. He presents with headache and seizures. His MRI brain identifies hydrocephalus and there are multiple small bilateral parenchymal cysts. These are predominantly at the grey−white matter junction and some of them have a small amount of surrounding oedema and rim enhancement. There is also a cyst in the fourth ventricle, which appears as a small nodular focus of increased T1 signal compared to the cerebrospinal fluid. This enhances following contrast. What is the most likely diagnosis? a. Cryptococcus b. Hydatid cysts c. Neurocysticercosis d. Toxoplasmosis e. Tuberculosis
(c) Neurocysticercosis Neurocysticerosis is characterised by cysts at the grey−white matter junction and in the subarachnoid and intraventricular spaces. When intraventricular, they can affect the fourth ventricle and lead to hydrocephalus. The appearances of the cysts can vary depending on the life cycle of the larvae, and as they die can enhance. Breakdown of the cyst membrane leads to surrounding oedema. Calcification is seen in chronic cases. Intracranial toxoplasmosis and cryptococcus infection are not common unless a patient is immunocompromised, and there is no history to suggest this. Toxoplasmosis does also cause ring-enhancing lesions and they can have a similar distribution, affecting the corticomedullary junction as well as the basal ganglia. Cryptococcus has non-specific imaging features, although one described feature is prominent perivascular spaces and ring-enhancing cryptococcomas. Cerebral hydatid cysts often lie in the territory of the middle cerebral artery and are usually solitary. Enhancement and surrounding oedema is not typical unless there is superadded infection. Another differential for these appearances is intracranial tuberculosis infection causing tuberculomas at the corticomedullary junction. These may also enhance peripherally and cause surrounding oedema. The other features of an intraventricular lesion and hydrocephalus are more consistent with neurocysticercosis.
1362
A 64 year old female patient presents with shortness of breath on exertion. She smokes five cigarettes per day and her past medical history includes rheumatoid arthritis, hypertension, hypercholesterolaemia and atrial fibrillation. Initial bloods are normal apart from the arterial blood gas, which reveals a PaO2 of 9.5 kPa (normal range 10.5–13.5 kPa) with a restrictive pattern on spirometry. High resolution CT chest demonstrates bilateral hyperdense consolidation which is predominantly peripheral, affecting the bases more than the apices with patchy ground glass opacification and reticulation. What is the most likely diagnosis? a. Amiodarone lung disease b. Eosinophilic granulomatosis with polyangiitis (Churg-Strauss syndrome) c. Simple pulmonary eosinophilia d. Non-specific interstitial pneumonia e. Gold induced lung toxicity
(a) Amiodarone lung disease This woman has a history of atrial fibrillation, for which amiodarone is often used. Amiodarone lung disease commonly presents with shortness of breath on exertion caused by hypoxia. The hyperdense consolidation is typical and the distribution is often asymmetrical and peripheral. It can occur 1–12 months following a treatment course of ≥6 months. Other features include a hyperdense liver and heart. Eosinophilic granulomatosis with polyangiitis can cause peripheral consolidation however, it causes serum eosinophilia and this patient’s blood tests are normal. It also tends to present in a younger age group than this patient, around the third to fourth decade. Patients often present with asthma but can also suffer with sinusitis, diarrhoea and arthralgia. Simple pulmonary eosinophilia would also cause serum eosinophilia and although can cause peripheral fleeting ground glass opacification would not cause a restrictive pattern on spirometry. Usual interstitial pneumonia is more commonly associated with rheumatoid arthritis compared to non-specific interstitial pneumonia (NSIP). NSIP can cause predominantly basal and subpleural ground glass opacification. Hyperdense consolidation is not a feature. The treatment of rheumatoid arthritis with gold has reduced over the years due to the introduction of newer agents, and gold-induced lung toxicity is uncommon and would not cause the hyperdense consolidation typical of amiodarone lung disease.
1363
A 26 year old male presents to the GP with ongoing wrist pain that has not resolved with anti- inflammatory medications or physiotherapy. The GP requests a plain film. Amongst other findings, the report mentions positive ulnar variance and suggests further investigation with MRI to assess for complications. What is a likely complication of the condition described, which will be seen on the MRI? a. Avascular necrosis of the lunate bone b. Radioulnar convergence c. Scalloping of the distal radius by the ulna d. Tear of the triangular fibrocartilage complex e. Ulnar impingement syndrome
d) Tear of the triangular fibrocartilage complex Positive ulnar variance is a condition in which the ulnar-carpal articulation is more distal than the radial-carpal articulation. The condition causes pain and can result in ulnar impaction syndrome. Ulnar impaction syndrome occurs due to degenerative changes and causes thinning of the triangular fibrocartilage, which is then more easily torn. This is a separate entity to ulnar impingement syndrome, which occurs due to a shortened ulnar (negative ulnar variance). In ulnar impingement syndrome, radioulnar convergence causes impingement of the distal ulnar on the distal radius, proximal to the sigmoid notch. After a prolonged period of time, subchondral sclerosis and scalloping of the distal radius at the distal radio-ulnar joint may be seen. Avascular necrosis of the lunate bone, also known as Kienböck disease, is more associated with negative ulnar variance.
1364
A 12 year old boy is referred for a chest radiograph. He has recurrent pulmonary infections and shortness of breath on exertion. The radiograph shows a small right lung with ipsilateral mediastinal shift. There is a curvilinear tubular opacity adjacent to the heart border. What is the most likely diagnosis? a. Partial anomalous pulmonary venous return b. Pulmonary sequestration c. Right middle lobe atelectasis d. Tetralogy of Fallot e. Unilateral absence of the pulmonary artery
a) Partial anomalous pulmonary venous return (PAPVR) Scimitar or hypogenetic lung syndrome is a form of PAPVR. It is characterised by a hypoplastic right lung with an anomalous vein that drains the abnormal right lung segment into the systemic venous system. Most commonly, the anomalous vein drains into the inferior vena cava giving rise to the curvilinear tubular opacity described on radiographs. However, it can also drain into other structures including the right atrium, hepatic veins or portal vein. Its arterial supply can be variable. The description in this case is typical for scimitar syndrome; however, pulmonary sequestration also has alternative vascular supply with both intra- and extralobar types deriving their arterial supply from the systemic arteries. Intralobar sequestration has a pulmonary venous drainage, whereas extralobar drains into the systemic veins. Neither have a connection with the bronchial tree. Unilateral absence of the pulmonary artery is characterised by a small affected lung with oligaemia and contralateral lung compensatory over inflation. However, the tubular opacity adjacent to the right heart border is not consistent with this diagnosis. Tetralogy of Fallot is not associated with the findings in this case. The typical finding is a boot- shaped heart, pulmonary oligaemia, and a right-sided aortic arch may be present. Right middle lobe atelectasis may look similar with opacity adjacent to the right heart border, but the other findings are more consistent with scimitar syndrome.
1365
A 59 year old female presents with increasing shortness of breath and persistent cough. She has a 20 pack year smoking history. The GP requests a CT chest for further investigation. The CT shows volume loss in the lower lung zones. There is extensive honeycomb destruction of the lungs, predominantly in the dependent aspect of the lower zones but also at the lateral and anterior aspect of the mid and lower zones. The airways within the abnormal looking lung demonstrate traction dilatation. There is minimal ground glass opacification in both lungs. The upper and mid zones of the lung demonstrate mild background emphysematous change. What is the most likely diagnosis? a. Cryptogenic organising pneumonia b. Hypersensitivity pneumonitis c. Non-specific interstitial pneumonia d. Respiratory bronchiolitis-associated interstitial lung disease e. Usual interstitial pneumonia
(e) Usual interstitial pneumonia (UIP) The CT description provided is that of a fibrotic lung process and is typical of a UIP pattern of disease. A UIP pattern of disease is associated with predominantly honeycomb destruction of the lung which has an apicobasal gradient and tends to be peripheral. Ground glass opacification is the predominant finding in non-specific interstitial pneumonia. The imaging findings of chronic hypersensitivity pneumonitis can overlap significantly with UIP pattern. However, the fibrosis distribution is more typically basal and peripheral in UIP, whereas in hypersensitivity pneumonitis the fibrotic change is less likely to be peripheral and is typically upper zones, although all zones can be affected. Hypersensitivity pneumonitis may also be associated with mosaicism secondary to air trapping. The predominant imaging finding in cryptogenic organising pneumonia is ground glass change and dense consolidation. Respiratory bronchiolitis−associated interstitial lung disease represents a spectrum of disease associated with smokers. The findings on CT chest are more typically centred on the airways with airway wall thickening and evidence of air trapping. There may be small centrilobular nodules which represent occlusion of the small airways.
1366
A barium swallow study is performed on a 42 year old female patient on her third cycle of neoadjuvant chemotherapy for breast cancer with a plan to proceed to surgery and radiotherapy. The patient has been experiencing increasing odynophagia but no dysphagia, reflux or regurgitation. What appearances on the barium swallow study would be most likely in this clinical setting? a. Flask-shaped mucosal outpouchings b. Large areas of oesophageal ulceration c. Long, linear oesophageal filling defects d. Multiple, small, nodular oesophageal filling defects e. Smooth stricture in the mid-oesophagus
(c) Long, linear oesophageal filling defects The patient is likely immunocompromised due to the chemotherapy regimen and therefore opportunistic infections such as candida, herpes simplex and cytomegalovirus (CMV) should be considered. Candida oesophagitis is the most common infectious cause, and findings include long linear plaque-like lesions, ulceration and pseudomembrane formation. CMV (and HIV) cause massive ulcers, whereas herpes simplex oesophagitis tends to lead to multiple small ulcers. Flask-shaped mucosal outpouchings are typical for pseudo diverticulosis of the oesophagus caused by the mucous glands filling with contrast. It is associated with reflux, strictures and oesophagitis. Multiple small nodular oesophageal filling defects are seen in glycogenic acanthosis, a condition associated with the elderly due to glycogen deposition. Smooth oesophageal strictures infer benign causes such as previous caustic ingestion, radiation, Barrett oesophagus and skin diseases with oesophageal manifestations, such as epidermolysis bullosa and pemphigoid. A likely cause in a breast oncology patient would be radiation; however, this patient is currently only having neoadjuvant chemotherapy.
1367
A 15 year old female presents the emergency department with a fall onto an outstretched hand. The referring clinician asks you to review the plain film wrist, which demonstrates bullet-shaped metacarpals, a wide radius and ulna and metacarpal irregularity but no fracture. The referrer explains there are features of dwarfism but no other history is provided You review previous imaging and see a plain film of the spine, which shows posterior vertebral body scalloping, platyspondyly and anterior central beaking of the vertebral bodies. What is the most likely underlying aetiology? a. Achondroplasia b. Down syndrome c. Hurler syndrome d. Morquio syndrome e. Pseudoachondroplasia
(d) Morquio syndrome Morquio syndrome is the most common mucopolysaccharidosis, associated with multiple skeletal abnormalities and presenting within the first 18 months of life. There are several features seen on spinal radiographs; central beaking of the anterior vertebral bodies is relatively specific. Other features include platyspondyly, posterior vertebral body scalloping, exaggeration of the lumbar lordosis and atlantoaxial subluxation due to odontoid hypoplasia. Appendicular skeleton findings include lateral sloping of the tibial plateau, genu valgus, bullet-shaped metacarpals and short, wide tubular bones with metaphyseal irregularity. The other conditions provided as options are examples of anteroinferior vertebral body beaking.
1368
A 32 year old female patient with no significant past medical history is brought into hospital with at least three abdominal stab wounds for which she has an arterial and portal venous phase CT abdomen pelvis. This demonstrates three sites of skin breach without evidence of deep extension or peritoneal breach. Incidental note is made of alternating dilatation and stenoses of the distal renal arteries bilaterally. The kidneys both enhance symmetrically and uniformly. There is no vascular calcification. Appearances elsewhere on the CT are normal. The patient continues to have normal observations on the ward. Following discharge she attends the renal outpatient clinic for follow-up and has normal serum renal function tests. What is the most appropriate management for this condition? a. Angioplasty b. Angioplasty plus stenting c. Continued follow-up d. No further management required e. Steroid treatment
(c) Continued follow-up The case describes fibromuscular dysplasia with the typical appearances of alternating stenoses and dilatation of the renal arteries. This more commonly affects women between the ages of 30 and 50 years and often presents with hypertension. Although the renal arteries are most commonly affected, other sites such as the vertebral, iliac, coeliac and extracranial internal carotid arteries can also be involved. Therefore symptoms including headache, stroke, angina or mesenteric ischaemia are possible, depending on the site of involvement. If asymptomatic, patients are kept under observation but if presenting symptomatically the condition is amenable to angioplasty and responds well. Stenting is rarely required. Patients with fibromuscular dysplasia have weakened vascular walls and can encounter complications, including dissection and aneurysm formation.
1369
A 13 year old boy with an underlying congenital syndrome undergoes an MRI brain. This demonstrates a lesion within the left cerebellar hemisphere with widened folia and a striated/ tigroid appearance. The lesion is T1 hypointense and T2 hyperintense and does not enhance following contrast administration. No other lesions are identified. The child has a thyroid ultrasound recently which demonstrates a thyroid goitre. What underlying syndrome does the child likely have? a. Ataxia telangiectasia b. Cowden syndrome c. Neurofibromatosis type 1 d. Sturge-Weber syndrome e. Tuberous sclerosis
(b) Cowden syndrome The lesion described is typical for Lhermitte-Duclos disease (also known as dysplastic cerebellar gangliocytoma) which is associated with Cowden syndrome. Other features include thyroid goitres, skin lesions and gastrointestinal polyps. Although Cowden syndrome and dysplastic cerebellar gangliocytoma are rare, the answer to this question can also be deduced from knowledge of the other, more common conditions. Ataxia telangiectasia leads to cerebellar atrophy and on imaging low T2 signal foci likely represent haemosiderin deposition secondary to bleeds from abnormal telangiectatic vessels. Neurofibromatosis 1, Sturge-Weber syndrome and tuberous sclerosis have multiple central nervous system manifestations, none of which are present in this case. Cerebellar tumours and thyroid goitres are not typical features.
1370
A 30 year old female with a background of medullary sponge kidney presents with right upper quadrant pain. On examination blood tests show elevated bilirubin levels. A liver ultrasound demonstrates multiple, dilated cystic structures converging towards the porta hepatis. The cysts communicate with the bile ducts. No peripheral biliary duct dilatation is identified. MRCP shows ectatic intrahepatic ducts extending into the periphery. The common bile duct is dilated but no strictures are seen. What is the most likely diagnosis? a. Primary sclerosing cholangitis b. Polycystic liver disease c. Choledochocele d. Primary biliary cirrhosis e. Caroli disease
(e) Caroli disease Caroli disease is an autosomal recessive disease that is associated with medullary sponge kidney and renal cysts. Patients most often present in the second and third decades of life with recurrent cholangitis. Caroli disease is a type V choledochal cyst according to the Todani classification. The absence of strictures on MRCP excludes primary sclerosing cholangitis and primary biliary cirrhosis as diagnoses. In polycystic liver disease the cysts do not communicate with the biliary tree. A choledochocele is defined as a dilation of the duodenal part of the common bile duct.
1371
A 69 year old male patient with increasing shortness of breath is reviewed in the respiratory outpatient clinic following a GP referral. CT chest prior to the appointment demonstrates increased bilateral, predominantly lower lobe subpleural reticulation, interlobular septal thickening with associated bronchiectasis. Which is the most likely underlying diagnosis? a. Ankylosing spondylitis b. Asbestosis c. Chronic hypersensitivity pneumonitis d. Silicosis e. Sarcoidosis
(b) Asbestosis The case describes features of pulmonary fibrosis affecting the lower lobes. Of the available answers, asbestosis is the only one typically associated with lower lobe fibrosis. It can be very difficult to differentiate asbestosis and idiopathic pulmonary fibrosis and often the clinical history is critical. However, it is an important diagnosis to make, as patients can be awarded compensation if they have had previous occupational asbestos exposure. Other features that can be associated with asbestosis include honeycombing, pleural plaques and pleural effusions. Lymph node enlargement is not typical. Other causes of lower lobe fibrosis include systemic inflammatory conditions, such as scleroderma and rheumatoid arthritis, and drug reactions, for example secondary to amiodarone, methotrexate or bleomycin. The other available answers typically cause upper lobe fibrosis. Other causes of upper lobe fibrosis include cystic fibrosis, Langerhans cell histiocytosis, radiation-induced fibrosis and tuberculosis.
1372
A 5 year old girl with recurrent urinary tract infection is investigated as an outpatient by the paediatric team with a urinary tract ultrasound. The appearances are highly suggestive of a right duplex kidney. There are normal appearances of the left kidney. There is no hydronephrosis. Appearances of the urinary bladder suggest a right sided ureterocoele. The patient has an excretory phase CT further investigate. What is the most likely CT appearance? a. Two right ureters converging just proximal to the urinary bladder b. Two right ureters with upper moiety ureter inserting superolateral to lower moiety ureter c. Two right ureters with upper moiety ureter inserting inferomedial to lower moiety ureter d. Two right ureters with lower moiety ureter inserting superolateral to upper pole moiety e. Two right ureters with lower moiety ureter inserting inferomedial to upper pole moiety
(c) Two ureters with upper moiety ureter inserting inferomedial to lower moiety ureter A duplex kidney does not necessarily imply a complete ureteric duplication; however, with the ultrasound appearances of ureterocele there is suspicion of an ectopic insertion of the upper moiety ureter. Incomplete duplication is when the ureters converge above the level of the bladder and often this can exist with a degree of ‘yo-yo’ reflux from one ureter into the other. When there is complete duplication the upper pole moiety often obstructs due to the presence of a ureterocele. This can cause the ‘drooping lily’ sign due to hydronephrosis and lack of filling of the upper pole moiety and subsequent displacement of the opacifying lower pole moiety. The upper moiety ureter inserts ectopically, often into the urinary bladder, but this can insert into the urethra, vagina or seminal vesicles, potentially causing continuous wetting in older children. The lower moeity ureter inserts normally but is prone to reflux. The Weigert-Meyer rule states that the upper moiety ureter inserts inferomedial to the lower moiety ureter.
1373
An MRI small bowel is performed for a 39 year old female patient with abdominal pain and diarrhoea. This reveals small bowel fold thickening affecting the jejunum and ileum. There are several adjacent 11-mm short axis lymph nodes and a small amount of free fluid in the pelvis. The right lobe of the liver extends approximately 2 cm below the lower pole of the right kidney. The spleen measures 16 cm in craniocaudal extent. Both kidneys measure up to 10.5 cm in length. Patchy low T1 and T2 weighted signal is demonstrated in the imaged spine. What is the most likely unifying diagnosis? a. Amyloidosis b. Lymphoma c. Mastocytosis d. Tuberculosis e. Whipple disease
(c) Mastocytosis Causes of thickened mucosal small bowel folds include mastocytosis, amyloidosis, Whipple disease, radiotherapy and graft-versus-host disease. In this case the hepatosplenomegaly, lymph node enlargement and bone sclerosis all fit with mastocytosis. Mastocytosis can also cause ascites; however, a small amount of free pelvic fluid is common in women of reproductive age. Thickened mucosal folds caused by amyloidosis are most commonly seen in the duodenum, with the stomach being the second most frequently affected site. Whipple disease is caused by a bacterial infection and as well as the thickened valvulae conniventes, any associated lymph node enlargement typically demonstrates central low density (due to fat). The other findings in this case would not be typical. Tuberculosis (TB) is a cause of hepatosplenomegaly; however the combination of other findings is not typical. If TB affects the bowel it is frequently the ileocaecal region. Lymphoma can affect the bowel, causing wall thickening, lymph node enlargement and hepatosplenomegaly. Often the bowel lumen appears dilated due to involvement of the myenteric plexus.
1374
A 67 year old retired decorator presents to his GP following multiple episodes of chest pain during the past 2 weeks brought on by exertion. Past medical history includes hypertension and gallstones. The GP refers him to the rapid access chest pain clinic. As part of the investigations the patient has a cardiac MRI which demonstrates increased T2 weighted signal intensity in the mid anterior and septal walls, with delayed subendocardial hyperenhancement. What is the most likely diagnosis? a. Acute myocarditis b. Hibernating myocardium c. Myocardial infarct involving the left anterior descending artery d. Myocardial infarct involving the right coronary artery e. Myocardial stunning
(c) Myocardial infarct involving the left anterior descending artery The left anterior descending artery and its branches supply the anterolateral and apical walls of the left ventricle and the interventricular septum. The high T2 weighted signal in this region is secondary to oedema suggesting a relatively acute insult, and the delayed hyperenhancement is typical in infarcted myocardium and tends to be subendocardial or full thickness. Myocardial stunning and hibernation often have similar imaging findings. Stunning is caused following a transient period of ischaemia, whereas hibernation is thought to be related to more chronic ischaemia where the myocardial cells adapt to reduced perfusion by hibernating and reducing metabolic activity. Both these conditions lead to impaired function, manifesting as reduced contractility. Stunned myocardium tends to have preserved perfusion whereas it can be reduced in myocardial hibernation. Acute myocarditis may demonstrate increased myocardial T2 weighted signal however other findings such as a focal area of wall motion abnormality would also be expected. Enhancement in myocarditis tends to involve the epicardium and be early rather than the delayed subendocardial enhancement described in this case.
1375
A 7 year old female falls from a trampoline and develops wrist swelling. The parents take her to the emergency department and a plain film is performed. The imaging demonstrates buckling of the cortex; the cortex remains intact. What type of fracture is this? a. Greenstick fracture b. Lead pipe c. Plastic bowing d. Salter-Harris fracture e. Torus fracture
e) Torus fracture The features describe a buckle or torus fracture which occurs due to compression injury, for example falling on an outstretched hand. In a greenstick fracture, there is a cortical break on one side of the bone; the cortex on the other side remains intact, this is an unstable fracture. A lead pipe fracture is characterised by a torus fracture on one side of the bone and a greenstick fracture on the other side of the bone. In a plastic bowing fracture, there is no discernible cortical compression or break; however the bone appears deformed or bent. A Salter-Harris fracture is a fracture of the physis, which is divided into types 1–5.
1376
A trainee sonographer comes to you asking for advice about appearances on a transvaginal pelvic ultrasound examination of a 35 year old GP patient with intermittent lower abdominal pain. The sonographer could clearly see both ovaries. There are a couple of small (<10 mm), thin-walled, anechoic, avascular lesions associated with the right ovary. In the left ovary there is a 15 mm unilocular, anechoic cystic lesion which is thick walled with marked continuous peripheral vascularity. There is a trace of free fluid in the pouch of Douglas. The uterus is anteverted. The endometrium measures up to 8 mm. Which of the following is most appropriate? a. MRI pelvis b. No follow-up c. Recommend Ca-125 levels and gynaecology referral d. Ultrasound pelvis in 6 weeks e. Ultrasound pelvis in 3 months
(b) No follow-up The appearances are consistent with a corpus luteum cyst, they occur in premenopausal women following the release of an ovum and usually involute and resolve if fertilisation does not occur. There are described as thick walled with intense peripheral ‘ring of fire’ vascularity. Sometimes this vascularity can be described in association with ectopic pregnancies; however, these are usually extra-adnexal and not within the ovary. The anechoic, avascular lesions on the contralateral ovary likely represent follicles and are physiological. In premenopausal women simple ovarian cysts <5 cm do not require follow-up. If they are larger than this, then follow-up in at least 6 weeks (at a different phase in the menstrual cycle) is suggested. In postmenopausal women, follow-up of cysts >3 cm is suggested. If there are any concerns about vascular solid components, then an MRI pelvis can help to characterise further. Ca-125 and gynaecology referral may also be indicated; however Ca-125 is non-specific and can be raised in lots of benign conditions.
1377
A 10 year old child with a raised serum alpha-fetoprotein has an abdominal ultrasound. The liver has a coarse echotexture and reduced reflectivity. There is a 20-mm solid heterogenous lesion in the right hepatic lobe which is predominantly hypoechoic. There is no evidence of calcification. On MRI the lesion is T2 hyperintense and has mildly increased T1 signal. It enhances homogenously and rapidly following contrast. The lesion then becomes hypointense to the rest of the liver on portal venous phase images. Which of the following is the most likely diagnosis? a. Haemangioendothelioma b. Hepatoblastoma c. Hepatocellular carcinoma d. Mesenchymal hamartoma e. Regenerative nodule
(c) Hepatocellular carcinoma The patient has features of liver disease. This finding and the imaging characteristics of the lesion are consistent with hepatocellular carcinoma (HCC), which is the second most common malignant paediatric liver lesion. They have the same radiological features as HCC in adults with arterial phase enhancement followed by portal venous phase washout. In contrast to this, regenerative nodules do not display this type of enhancement and are typically hypointense on T2 weighted imaging. Hepatoblastoma is more common in a younger age group − typically children less than 5 years old. Similar to hepatocellular carcinoma, they are associated with raised alpha-fetoprotein; however the radiological features differ. Hepatoblastomas are usually T1 hypointense with heterogenous enhancement. Haemangioendothelioma and mesenchymal hamartomas have similar appearances to each other and are usually quite large lesions comprising solid and cystic elements with internal vascularity. The finding of reduced calibre aorta distal to the coeliac trunk is indicative of a haemangioendothelioma.
1378
Following a road traffic collision a 12 year old cyclist is brought into hospital by ambulance intubated and ventilated. A trauma CT is performed. Thoracic imaging reveals a right sided pneumothorax, bilateral rib fractures and a fractured right proximal humerus. The abdominopelvic imaging demonstrates abdominal free fluid with retroperitoneal free gas. There is a short segment of duodenal mural thickening with disruption of the wall and adjacent free gas locules. Where in the duodenum is the rupture most likely to be sited? a. At the level of the ampulla of Vater b. At the level of the ligament of Treitz c. D1–2 d. D2–3 e. D3–4
(d) D2–3 Duodenal trauma is most likely to affect the retroperitoneal D2–3 segments, as they are relatively fixed. Deceleration injuries in particular are associated with duodenal trauma. Duodenal rupture is suggested if there is retroperitoneal free gas, wall thickening, discontinuity of the wall, adjacent free fluid and fat stranding. This requires surgical management and may be associated with other intra-abdominal injuries. In the context of trauma, duodenal wall thickening and a heterogenous soft tissue attenuation mass adjacent to the duodenum without any evidence of perforation would be in keeping with a duodenal haematoma. This is usually managed conservatively unless there is evidence of active haemorrhage. The distinction between haematoma and rupture can be challenging especially with other concomitant trauma findings. The D1 segment of the duodenum is least likely to be affected. The ligament of Treitz at the duodenojejunal flexure is another possible site of injury due to the fixation the ligament provides.
1379
A 15 year old boy with epistaxis is reviewed by the ear nose and throat (ENT) team. There is a red/blue mass visible on nasoendoscopy and an MRI head confirms a diagnosis of juvenile angiofibroma. The ENT team request radiological guided embolisation prior to surgical management. Which artery is the most likely feeding vessel? a. Ascending pharyngeal b. Facial c. Internal maxillary d. Lingual e. Superficial temporal
(c) Internal maxillary Juvenile angiofibromas are benign lesions which typically present with epistaxis in adolescent males. Characteristic imaging findings include widening of the pterygopalatine fossa, erosion of the medial pterygoid plate and anterior bowing of the posterior wall of the maxillary sinus. They can spread through the skull base and typically enhance homogenously and avidly. Biopsy is contraindicated due to their high vascularity. Preoperative embolisation can be helpful prior to surgical management. These lesions are most commonly supplied by the internal maxillary artery, a branch of the external carotid artery.
1380
A 16 year old boy is referred for a MR angiogram for suspected aortic coarctation. He has a left sided aortic arch. The coarctation lies distal to the brachiocephalic trunk but proximal to the origin of the left subclavian artery. What rib notching pattern would be most likely? a. Bilateral first and second ribs b. Bilateral third to ninth ribs c. Left third to ninth ribs d. Right first and second ribs e. Right third to ninth ribs
(e) Right third to ninth ribs Rib notching occurs secondary to dilated intercostal collateral vessels which allow blood to bypass the coarctation and reach the descending aorta. The first and second ribs do not become notched because the first and second posterior intercostal arteries arise from the costocervical trunk, a branch of the subclavian artery. They therefore do not communicate with the aorta and so are not involved in collateral formation. Rib notching commonly affects the third to ninth ribs bilaterally due to the coarctation being distal to both subclavian arteries. If there is unilateral right rib notching, as in this case, then the coarctation lies distal to the brachiocephalic trunk but proximal to the origin of the left subclavian artery, or there may be a right sided aortic arch with aberrant left subclavian artery which is distal to the coarctation.
1381
A 16 year old female patient has a urinary tract ultrasound following several urinary tract infections. This shows the right kidney is atrophic and the left kidney is normal in appearance. Previous ultrasound from when the patient was younger demonstrated a large right kidney. There is no hydronephrosis and the renal hilum is normal; however, there are multiple anechoic right renal lesions varying in size. These are separated by hyperechoic linear tissue without increased vascularity. What is the most likely underlying diagnosis? a. Autosomal recessive polycystic kidney disease b. Juvenile nephronophthisis c. Mesoblastic nephroma d. Multicystic dysplastic kidney e. Cystic nephroma
(d) Multicystic dysplastic kidney Multicystic dysplastic kidney (MCDK) in infancy can demonstrate multiple cysts of varying sizes with intervening hyperechoic fibrous tissue and very little normal parenchyma. The kidney appears enlarged due to the multiple cysts. This condition is caused by ureteral obstruction in utero which stops normal nephron formation. The condition can be asymptomatic, going unrecognised and causing an atrophic kidney later in life. The condition is typically unilateral; bilateral involvement is fatal. Autosomal recessive kidney disease also causes large kidneys but is bilateral and symmetrical with small 1- to 2-mm cysts. It can also cause hepatosplenomegaly. Juvenile nephronopthisis causes normal or small kidneys cysts, atrophy and fibrosis. There is reduction in corticomedullary differentiation and multiple <15 mm cysts later in the disease process. Mesoblastic nephroma is a solid lesion but sometimes there can be areas of cystic degeneration or necrosis within it. Multilocular cystic nephroma, or simply cystic nephroma, also has imaging features of multiple cysts with fibrosis; however, it tends to be a discrete renal mass rather than affecting the whole kidney and is often described as herniating into the renal hilum. There can also be Doppler flow seen in the septations of these lesions
1382
A 4 year old boy with hypotonia and occasional seizures is under the care of neurology. A CT brain reveals subtle low density of the subcortical white matter and on a subsequent MRI brain there is associated T1 hypointensity, FLAIR hyperintensity and restricted diffusion. The appearances are symmetrical and diffuse. There is no abnormal enhancement following contrast administration. The internal capsule and caudate nucleus are spared and the cerebellum has normal appearances. MR spectroscopy reveals a high N-acetyl-aspartate to creatine ratio. What is the most likely diagnosis? a. Adrenoleukodystrophy b. Alexander disease c. Canavan disease d. Krabbe disease e. Pelizaeus-Merzbacher disease
(c) Canavan disease Canavan disease causes subcortical white matter abnormality with involvement of the deep grey matter. The caudate, corpus callosum and internal capsule may be spared. There is associated restricted diffusion but no enhancement. The characteristic feature is elevated N- acetyl-aspartate (NAA). Adrenoleukodystrophy is X-linked, affecting young male patients. It classically has a posterior distribution affecting the occipitoparietal region, particularly the periventricular area. There is involvement of the splenium of the corpus callosum. Peripheral enhancement is frequently seen. Alexander disease has a predominantly anterior distribution which progresses posteriorly and becomes more diffuse during the course of the disease. It causes abnormal white matter T2 hyperintensity and enhancement. NAA is normal. Pelizaeus-Merzbacher is also X-linked and causes subcortical white matter abnormalities. There is usually cerebellar atrophy. Importantly, the perivascular regions are usually spared, which leads to ‘tigroid’ appearance, similar to metachromatic leukodystrophy. NAA is usually reduced. Krabbe disease favours a more periventricular distribution and causes abnormal MRI signal in a more central and posterior position. It also leads to optic nerve hypertrophy.
1383
A 19 year old male who is usually fit and active has plain films of the right knee for ongoing knee pain. The radiograph demonstrates subtle flattening of the lateral surface of the medial femoral condyle with a loose osteochondral fragment. He has an MRI for investigation of suspected osteochondritis dissecans. Which finding is most likely to be seen on MRI? a. Blooming artefact on gradient echo sequence b. Cartilaginous defect on T1 c. Fat-fluid level within joint effusion d. Low signal material within the defect on T2 e. Synovial proliferation
(b) Cartilaginous defect on T1 Osteochondritis dissecans is an osteochondral fracture of the articular epiphysis thought most likely due to trauma and ischaemia. It is more common in males and most commonly affects the knee, talar dome, tibia, patella and femoral head. Features on plain film include flattening and cortical irregularity of the articular surface and a detached loose osteochondral defect may also be visible. On MRI a cartilaginous defect may be seen on T1, the defect may demonstrate high T2 signal in keeping with fluid and there may also be high signal seen within the articular cartilage. Synovial proliferation is a feature of pigmented villonodular synovitis, which may also show blooming on gradient echo imaging due to haemosiderin deposition. A fat-fluid level may be seen in an acute intra-articular fracture, not typically seen in osteochondritis dissecans.
1384
A renal transplant patient is 4 days post-surgery and the transplant team request an ultrasound Doppler due to increased tenderness in the right iliac fossa, decreased urine output and haematuria. It is a single artery, vein and ureter organ from an adult cadaveric donor. The unobstructed transplant measures 15 cm in bipolar length and there is a trace of perinephric fluid. Corticomedullary differentiation appears normal. There is generalised Doppler vascularity evident within the kidney. The arcuate arterial traces at the upper, lower and middle poles demonstrate a sharp upstroke and a resistive index of 0.71. The renal artery and vein are difficult to visualise but the main renal artery demonstrates reverse flow in diastole. The urinary bladder has normal appearances. What is the most likely diagnosis? a. Pseudoaneurysm b. Rejection c. Renal artery stenosis d. Renal artery thrombosis e. Renal vein thrombosis
(e) Renal vein thrombosis There is reverse flow in diastole on the arterial trace which is consistent with renal vein thrombosis; flow should always be flowing forwards even in diastole. The typical presentation is with tenderness and decreased urine output in the first week following transplant. The kidney can appear swollen and can be hypoechoic on ultrasound. The renal vessels can be difficult to clearly visualise immediately following transplant depending on position and body habitus. If the renal vein could be visualised and assessed with Doppler, there would be no venous flow. Renal artery thrombosis is also an early complication but thankfully quite rare. There would be no vascularity on the ultrasound Doppler, and diagnosis is imperative as it requires prompt return to theatre to try and salvage the graft. Pseudoaneurysm formation, rejection and renal artery stenosis are late complications. Pseudoaneurysms usually form as a result of biopsy, not due to the transplant surgery itself. On Doppler ultrasound there will be a focus of abnormal colour flow with very turbid flow. Rejection is also a late complication. Typically the kidney is enlarged with large renal pyramids and reduced corticomedullary differentiation. The resistive index tends to be high and there can be mild pelvicalyceal dilatation. Features of renal artery stenosis on ultrasound Doppler include increased resistive indices (>0.7) and a parvus-tardus wave form, in contrast to the sharp upstroke seen in a normal transplant trace. It usually occurs at the site of the anastomosis.
1385
You are reporting an MRI brain for a neurology outpatient with a history of epilepsy. There is unilateral partial opacification of the left maxillary sinus by a rounded lesion. The opacity demonstrates high T2 signal and intermediate T1 signal without restricted diffusion or enhancement. The maxillary ostia are symmetrical. On a CT brain from 2 months previously there is a similar appearance with a hypodense lesion in a partially aerated left maxillary sinus. What does this mass most likely represent? a. Allergic fungal sinusitis b. Antrochoanal polyp c. Mucocele d. Mucous retention cyst e. Papilloma
(d) Mucous retention cyst Mucous retention cysts are commonly seen in the maxillary sinus but can occur in other paranasal sinuses. They are commonly incidental findings. They typically have intermediate T1 and high T2 signal. They occur within partially aerated sinuses, in contrast to mucoceles, which are found in a non-aerated sinus. Antrochoanal polyps frequently arise from the maxillary sinus but protrude into the nasal cavity via a widened maxillary ostium. They can have similar MRI signal but usually demonstrate peripheral enhancement. Similarly, a papilloma usually demonstrates heterogenous avid enhancement. Papillomas are frequently described as having a cerebriform appearance due to alternating high and low signal. On CT, allergic fungal sinusitis typically appears as central hyperdensity surrounded by hypodense oedematous mucosa filling a sinus. This inflamed, oedematous mucosa is therefore T2 hyperintense and enhances. There may be bony expansion, thinning and erosion.
1386
A 33 year old female presents with left sided chest pain and shortness of breath. She has a chest radiograph in the emergency department which shows a left sided pleural effusion. She is under outpatient follow-up with the rheumatology team for systemic lupus erythematosus (SLE). She has a CT pulmonary angiogram as the team are concerned that she is at increased risk of pulmonary embolus. This shows no evidence of pulmonary embolus; however, there are radiological features consistent with her underlying diagnosis. What is the most common radiological chest finding associated with SLE? a. Interstitial lung disease b. Pericardial thickening c. Pleural effusion d. Pneumonia e. Pulmonary haemorrhage
(c) Pleural effusion Systemic lupus erythematous (SLE) is a multisystem collagen vascular disease which commonly affects the lungs. It is more commonly seen in women and can have a variety of manifestations. The presence of pleuritis is one of the many features which can be used in the diagnostic criteria. The most common thoracic finding in SLE is the presence of a pleural effusion (50% are bilateral) which are usually small and consisting of an exudate. Pericarditis is also with the diagnostic criteria for SLE, and is present in 17–50% of patients. It may be seen on CT chest as pericardial thickening and pericardial effusion. Patients commonly develop pulmonary disease in the form of pneumonia, pulmonary haemorrhage and acute lupus pneumonitis (diagnosis of exclusion). Patients are at significantly increased risk of developing pneumonia due to immunological deficiency, as well as immunotherapy treatment. Interstitial lung disease can be seen in SLE, however it is not the most common feature (only 3% of patients) and is less commonly seen in SLE than in other collagen vascular diseases.
1387
A patient is discussed at the upper gastrointestinal multidisciplinary team meeting. They presented with dysphagia and weight loss. A barium swallow revealed mid-oesophageal mucosal irregularity and endoscopic biopsies confirm moderately differentiated oesophageal squamous cell carcinoma. Staging imaging is required to aid management. What is the most appropriate staging imaging? a. Contrast enhanced CT chest abdomen and pelvis b. Endoscopic ultrasound c. Endoscopic ultrasound and 18F-FDG PET/CT d. Endoscopic ultrasound and contrast enhanced CT chest, abdomen and pelvis e. Thoracic MRI and contrast enhanced CT chest, abdomen and pelvis
(c) Endoscopic ultrasound and 18F-FDG PET/CT Oesophageal cancer is very FDG avid. Ideally it is locally staged with endoscopic ultrasound which differentiates T1 (limited to mucosa) from T2 (involving muscularis propria) disease and also assesses local lymph nodes. Locoregional and distant staging can be achieved with 18F- FDG PET/CT and this is more accurate than CT alone. PET/CT is also helpful in disease follow-up as it can help to distinguish between recurrence and fibrosis. Similarly, PET/CT is more accurate than PET alone. Thoracic MRI cannot differentiate between T1 and T2 disease and therefore other modalities are preferred.
1388
A 6 year old boy with developmental delay is referred to the paediatric neurologists for an MRI brain due to recurrent seizures. This reveals multiple subependymal nodules which return high T1 and intermediate T2 signal. Previous ultrasound of the abdomen demonstrated bilateral echogenic renal masses. Based on the above findings, what other feature is most likely on the MRI head? a. Cystic lesion in right cerebellum containing an enhancing solid nodule b. Homogenous intermediate signal durally based lesion adjacent to the right frontal lobe c. Intraventricular markedly enhancing lobulated mass d. Multiple foci of FLAIR hyperintensity in the cortical and subcortical regions e. Several foci of T2 hyperintensity in the deep white matter bilaterally
(d) Multiple foci of FLAIR hyperintensity in the cortical and subcortical regions The patient has features of tuberous sclerosis with subependymal nodules and renal angiomyolipomas. Other findings associated with tuberous sclerosis include white matter abnormalities, subependymal giant cell astrocystomas and cortical tubers. Tubers manifest as small areas of FLAIR and T2 hyperintensity in the cortical and subcortical regions. The cystic cerebellar lesion with the enhancing soft tissue nodule is suggestive of either a haemangioblastoma or pilocytic astrocytoma. The intraventricular lobulated enhancing mass is consistent with a choroid plexus papilloma. Both haemangioblastoma and choroid plexus papillomas are associated with von Hippel-Lindau. The homogenous durally based lesion is consistent with a meningioma in Neurofibromatosis (NF) 2 and the T2 hyperintense deep white matter lesions are typical for focal areas of signal intensity (FASI) which are seen in NF1.
1389
A 28 year old male patient, under the care of the gastroenterology team, has been admitted with an exacerbation of ulcerative colitis. After 48 hours in hospital he is stable but has some increased abdominal discomfort. You are asked to review his contrast-enhanced CT abdomen and pelvis. This demonstrates mural thickening and pericolic fat stranding affecting the entire colon and rectum. There is a rim of lower attenuation in the wall of the colon with a Hounsfield unit of −95 and minimal faecal residue in the colon. There is a tiny amount of peritoneal free fluid but no free gas. The colon measures up to 50 mm in diameter. Mesenteric lymph nodes measure up to 11 mm in short axis. What is the most appropriate recommendation to the clinical team? a. Abdominal radiograph in 24 hours b. Barium enema in 24 hours c. Colonoscopy within 48–72 hours d. Repeat CT abdomen pelvis in 48 hours e. Urgent surgical referral
(a) Abdominal radiograph in 24 hours The case demonstrates typical imaging findings of ulcerative colitis with the submucosal low attenuation consistent with fat deposition seen in chronic cases. The patient is at risk of toxic megacolon. This condition also affects patient with colitis of other causes but accounts for the majority of deaths related to ulcerative colitis and should therefore should be kept in mind when reporting acute imaging. Toxic megacolon does not always cause bowel dilatation; however, if the colon is dilated >5 cm, particularly the transverse colon, where gas tends to collect, it should be considered. Colonoscopy and barium studies are contraindicated due to the risk of perforation. Frequent (often daily) abdominal radiographs are suggested to monitor bowel dilatation. Repeat CT may be warranted during admission; however, this patient is relatively young and abdominal radiographs should be considered in the first instance if he is clinically stable. Urgent surgical referral is not indicated at this point.
1390
A 23 year old builder is seen in the emergency department following a fall from scaffolding. CT chest abdomen pelvis demonstrates several right rib fractures and a small amount of free fluid in the pelvis. The patient has haematuria and blood at the urethral meatus. The urology team request retrograde urethrography. This demonstrates extravasation of contrast into the retropubic space, but continuity of the urethra is maintained. Where is the urethral injury most likely to be sited? a. Above the urogenital diaphragm b. Below the urogenital diaphragm c. Bladder dome d. Bulbous urethra e. Penile urethra
(a) Above the urogenital diaphragm Signs of urethral injury include inability to void, haematuria and blood at the urethral meatus. The urethra is split into anterior and posterior sections, with anterior being penile and bulbous portions, and posterior being prostatic and membranous. The anterior urethra is more commonly injured in straddle type injuries and the posterior urethra in blunt trauma, sometimes with pelvic fractures. Iatrogenic causes are also important to consider. The Goldman classification helps to distinguish between the different types of injury. The important landmark is the urogenital diaphragm. If the contrast extravasates into the retropubic space it suggests injury above the urogenital diaphragm. Conversely, if the contrast leaks into the perineum it suggests injury below the urogenital diaphragm. Above the urogenital diaphragm relates to the posterior urethra. Incomplete urethral injuries, such as in this case, are often treated conservatively with catheterization; however complete transection may require surgery. Urethral stricture is the most common long-term complication of urethral injury, with post-traumatic strictures tending to be short. Post-infectious strictures are more likely to be long.
1391
A 4 year old female patient has splenomegaly, gallstones and bilateral enlarged kidneys on an abdominal ultrasound scan. A previous chest radiograph from the year before demonstrates consolidation; this is no longer present on the current radiograph. Which of the following findings may also be present in a patient with this condition? a. Anterior inferior vertebral body beaking b. H-shaped vertebrae c. Posterior vertebral scalloping d. Ribbon ribs e. Narrowed interpedicular distance
(b) H-shaped vertebrae The patient has features consistent with sickle cell disease. Young patients may have splenomegaly but over time the spleen becomes small and calcified. The kidneys are often enlarged in children but over the patient’s lifetime may become small due to renal failure. The previous consolidation which has now resolved may represent acute chest syndrome. An associated musculoskeletal finding is H-shaped vertebrae due to endplate infarction. Anterior inferior vertebral body beaking is a feature of conditions such as Hurler syndrome and achondroplasia. Posterior vertebral body scalloping is caused by a variety of pathologies including achondroplasia, mucopolysaccharidoses and dural ectasia. Ribbon ribs are classically associated with neurofibromatosis. A narrowed interpedicular distance is encountered in achrondroplasia and thanatophoric dysplasia.
1392
A 19 year old male presents with headache, and on examination he is found to have paralysis of upward gaze. CT head reveals dilatation of the lateral ventricles. There is a well-defined, hyperdense mass in the posterior aspect of the third ventricle. The mass has some central calcification present. On MRI the mass is isointense to grey matter on both T1 and T2 sequences with cystic high T2 signal foci. The mass enhances avidly on T1 post contrast sequence but there is no evidence of blooming on susceptibility weighted imaging. A further similar 2-cm mass is also seen in the midline within the suprasellar region. What is the most likely diagnosis? a. Pineoblastoma b. Pineocytoma c. Pineal germinoma d. Pineal yolk sac carcinoma e. Pineal teratoma
(c) Pineal germinoma As is the case with other pineal masses, germinomas tend to present with obstructive hydrocephalus and Parinaud syndrome (paralysis of upward gaze). Pineal germinomas are the most common cause of a mass in this region. They usually present in young adults and have a male predilection. The description in this case is typical for a pineal germinoma. Iso or hyperdense on CT with central calcification and isointense to grey matter on T1 and T2 weighted imaging with enhancement post contrast and no haemorrhage. Five to ten percent of patients will have synchronous tumours within the midline at the time of diagnosis. Pineoblastomas are aggressive lesions and therefore more infiltrative, less well-defined and any calcification is peripheral, causing an ‘exploded calcification’ appearance. As well as calcification, teratomas are also likely to contain fat and haemorrhage, both of which are not features of pineal germinomas. Pineocytomas have quite non-specific imaging findings; however, familiarity with the common features of the other pineal region tumours helps to exclude other causes. Pineal yolk sac tumours also have no specific imaging findings, but these tumours are unusual and therefore a pineal germinoma is much more likely
1393
A 57 year old female patient is referred for a CT chest by the respiratory clinicians. There is bronchial wall thickening with the airways having a similar diameter to the adjacent vessels at the lung peripheries and there is patchy ground glass opacification. The pleural spaces are clear. There are no size-significant hilar or mediastinal lymph nodes. Further images acquired following expiration demonstrate areas of lung which are comparatively low attenuation compared to the rest of the lungs. You note that the vessels are also smaller in the more lucent areas. What is the most likely diagnosis? a. Acute interstitial pneumonia b. Alveolar proteinosis c. Chronic eosinophilic pneumonia d. Chronic hypersensitivity pneumonitis e. Obliterative bronchiolitis
(e) Obliterative bronchiolitis Obliterative bronchiolitis causes bronchiolar inflammation and fibrosis leading to bronchiectasis and bronchial wall thickening with subsequent air flow obstruction. This causes the CT finding of mosaic perfusion, which is lucency created by trapped air during expiration. The vessels are decreased in calibre in the low attenuation lucent lung and comparatively increased calibre in the normal lung. Causes are varied and can be post-infective, following inhalation of toxic substances and due to aspiration. The changes associated with acute interstitial pneumonia are non-specific and include ground glass opacification and often dependent consolidation and fibrosis, which can lead to traction bronchiectasis. Alveolar proteinosis typically causes a ‘crazy paving’ appearance with ground glass opacification and thickened interlobular septa CT findings in chronic eosinophilic pneumonia are often described as the photographic negative of pulmonary oedema, with predominantly peripheral consolidation, and pleural effusions are not typical. The chronic form of hypersensitivity pneumonitis commonly causes mid and upper lobe fibrosis, subsequent traction bronchiectasis and honeycombing. It is often described as sparing the costophrenic angles.
1394
The clinical team ask your opinion on the best form of imaging for a 49 year old male with a family history of multiple endocrine neoplasia. He has symptoms of diarrhoea, abdominal pain and reflux. The patient has also presented twice during the past year to his GP with episodes of haematemesis. What test would you recommend to the team? a. Barium meal b. CT chest, abdomen pelvis with arterial and portal venous phase contrast c. 18F-FDG PET/CT d. MRI brain e. MRI small bowel
(b) CT chest abdomen pelvis with arterial and portal venous phase contrast Multiple endocrine neoplasia (MEN) 1 is associated with proliferative lesions in the pancreas, pituitary and parathyroid glands. MEN 2 is associated with medullary thyroid cancer, parathyroid hyperplasia and phaeochromocytomas. The patient likely has MEN 1, as he has symptoms of Zollinger-Ellison syndrome secondary to a gastrinoma. These commonly occur in the pancreas but are also seen elsewhere, for example in the duodenum. The most appropriate test is a CT, including arterial phase contrast, as the tumours are hypervascular and are most likely to be seen on this phase of imaging. A barium meal would likely reveal features of Zollinger-Ellison syndrome, such as thickened gastric folds and erosions/ulcers, but would likely not identify the gastrin-secreting lesion. An MRCP would be more helpful than an MRI small bowel, as the gastrinomas commonly occur in the pancreas. 18F-FDG PET/CT may be helpful in poorly differentiated metastatic disease with neuroendocrine tumours but would not be indicated in this case. A 111In-Octrotide scan may be more helpful in localising the gastrinoma and any metastatic spread. Patients with MEN 1 may also have pituitary lesions which an MRI brain would help to characterise, but in the first instance, the CT would be most helpful.
1395
A 57 year old male patient is investigated for painless haematuria. Past medical history includes hypertension and type 2 diabetes, for which he is on oral medication. On ultrasound the kidneys are unobstructed but eccentrically positioned lower than normal with poor visualisation of the lower poles. There are normal appearances of the urinary bladder. CT urogram is completed and confirms a horseshoe kidney. There are several small filling defects in the renal pelvises and proximal ureters; these have a density of 10HU on the non-contrast sequence and then 15HU following contrast administration. What is the most likely diagnosis? a. Leukoplakia b. Multiple calculi c. Pyeloureteritis cystica d. Transitional cell carcinoma e. Tuberculous urethritis
(c) Pyeloureteritis cystica Horseshoe kidneys are often asymptomatic and discovered incidentally. They are positioned lower than normal and their ascent into the upper abdomen is halted by the inferior mesenteric artery, usually around the level of L3. On ultrasound it may sometimes not be recognised, especially if the lower poles are not well seen and the fusion not appreciated. These abnormally positioned kidneys are more prone to trauma, calculi, pyelouretitis cystica, transitional cell carcinoma (TCC) and pelviureteric junction obstruction due to poor drainage. TCC is a concern in patients with horseshoe kidneys and potentially in patients with pyelouretitis cystica due to the chronic inflammation associated with the condition. However, with TCC the lesions would be expected to have a soft tissue density and enhance, although compared to the adjacent renal parenchyma this can be difficult to appreciate. Pyelouretitis cystica causes multiple small cysts and is often associated with diabetes and recurrent infection. This, along with the impeded drainage that can occur in a horseshoe kidney, further predispose patients to the condition. Multiple calculi are unlikely in the clinical setting of painless haematuria, and the density of these small lesions is not typical for calculi. Leukoplakia is also associated with recurrent infection but is more common in the urinary bladder than the upper tracts. Tuberculous urethritis causes intermittent stricturing and dilatation, as well as urinary tract calcification.
1396
A neonate is noted to be cyanotic from birth. A frontal chest radiograph is performed which shows a grossly dilated heart and right atrium with elevated apex. The lungs are oligaemic. What is the most likely diagnosis? a. Aortic coarctation b. Ebstein anomaly c. Tetralogy of Fallot d. Total anomalous pulmonary venous return (TAPVR) e. Transposition of the great arteries
b) Ebstein anomaly It can be helpful to categorise congenital heart disease into cyanotic versus acyanotic. Within the cyanotic group another important consideration is whether the lungs are oligaemic or congested. The use of categorisation helps to narrow the differentials. For this child, the presence of cyanosis and oligaemia suggests conditions such as tetralogy of Fallot, Ebstein anomaly or pulmonary atresia with an intact ventricular septum. The presence of a very enlarged heart excludes tetralogy of Fallot, when the heart is usually normal in size but classically described as boot shaped with an elevated apex. Pulmonary atresia with an intact ventricular septum is not an option for this question but can have very similar appearances to Ebstein anomaly. Total anomalous pulmonary venous return causes cyanosis and plethoric lungs. Transposition of the great arteries (the D-transposition subtype) is also a cause of cyanosis, but lung appearance can be variable. It typically cases an ‘egg-on-a-string’ appearance of the mediastinum due to narrowing of the superior mediastinum. Aortic coarctation is an acyanotic congenital cardiac disease with normal pulmonary flow.
1397
A 49 year old male patient has had his care transferred to the local ear, nose and throat team after relocating to the area. He has a history of thyroid cancer for which he has undergone total thyroidectomy 18 months previously. At the time of the ultrasound the patient is not sure about the type of thyroid cancer he had and the referral letter is not available. The scan demonstrates a 9-mm left level III lymph node with microcalcification. Early pathological assessment confirms malignant cells. Which underlying malignancy is most likely? a. Anaplastic thyroid carcinoma b. Follicular thyroid carcinoma c. Lymphoma d. Medullary thyroid carcinoma e. Papillary thyroid carcinoma
(d) Medullary thyroid carcinoma Nodules containing medullary thyroid carcinoma, along with their metastases frequently contain microcalcification. The age group it affects tends to be around 30–50 years of age and it is associated with multiple endocrine neoplasia 2 syndrome. The most common thyroid malignancy is papillary thyroid carcinoma, appearing as a hypoechoic nodule with ill-defined margins, and microcalcifications may be present. Regional lymph node metastases can occur early and may have a cystic component. Follicular thyroid carcinoma is more likely to have haematogenous metastases compared to regional lymph node involvement, and 20% of patients may have distant disease at presentation. Anaplastic thyroid carcinoma may contain microcalcification; however, it is usually associated with an older age group and has a poor prognosis, with nodal disease common at presentation. Thyroid lymphoma is usually the non-Hodgkins type. Microcalcification is uncommon and lymphoma would not usually be treated with total thyroidectomy.
1398
A 73 year old female patient is seen in the respiratory outpatient clinic with increasing shortness of breath over the past year. Subsequent CT chest shows right lower lobe volume loss with an enhancing, pleurally based lesion forming an obtuse angle with the pleura. Looking back at a previous CT 3 years ago, this has not increased in size. The vessels and bronchi seen extending from the right hilum towards the mass appear crowded and distorted. There is evidence of increased subpleural reticular markings in the lower lobes bilaterally. What additional feature would confirm the diagnosis? a. Cavitation b. Intralesional fat c. Pleural thickening and calcification d. Right hilum lymph node enlargement e. Water-lily sign
(c) Pleural thickening and calcification This is a typical description of round atelectasis associated with asbestos exposure. Although many men who have worked in the construction and manufacturing industries are at risk, their family members would have also potentially been exposed to asbestos fibres on their clothing. Round atelectasis can look mass-like − it is caused by collapsed infolded lung adjacent to calcified or non-calcified pleural thickening. Enhancement is a feature because of the presence of lung parenchyma. The bronchovascular crowding and distortion described is known as the ‘comet tail’ sign. Round atelectasis tends to be relatively stable over time but can sometimes demonstrate interval growth. Asbestos exposure can also lead to asbestosis − an interstitial lung disease typically associated with lower zone fibrosis. The other features listed in the question are not associated with round atelectasis. Causes of cavitating pulmonary nodules include tuberculosis, primary or metastatic squamous cell carcinoma, abscesses and septic emboli. Intralesional fat and popcorn calcification is typical for a pulmonary hamartoma. The water-lily sign is associated with pulmonary hydatid disease. Right hilum lymph node enlargement in the presence of a solitary lung lesion would be more indicative of a primary lung malignancy.
1399
A 57 year old woman with predominantly fatty breasts is recalled from screening for an area of new clustered microcalcification in the right breast. The radiographer asks whether you need any additional mammographic imaging prior to ultrasound. Which of the following is the most appropriate initial further investigation? a. Contrast-enhanced mammograms b. Ecklund technique c. Magnification views d. Paddle view e. Tomosynthesis
(c) Magnification views Following a screening recall for microcalcification, magnification views are obtained to further assess its characteristics. Clustered or branching pleomorphic microcalcification is suspicious for ductal carcinoma in situ. A true lateral view can also be helpful, along with the standard mediolateral oblique (MLO) and craniocaudal (CC) views, to aid localisation of the microcalcification in the breast. Ecklund technique is used for breast implants to displace the implants more posteriorly and make the breast tissue easier to assess. Paddle views apply focal compression to an area in an effort to assess if apparent distortion or spiculation may be real or caused by overlapping structures. Increasingly paddle views are superseded by tomosynthesis. Contrast-enhanced mammography is available at some breast units and can help detect abnormal areas of enhancement.
1400
A 53 year old male has ongoing cough and shortness of breath following a lung transplant 18 months earlier. He has a CT chest as part of his workup to look for complications. He has suffered intermittent infective episodes since the transplant, which have required treatment with antiviral therapy. He is increasingly symptomatic and the clinicians are concerned about chronic rejection. The CT scan shows hyperinflated lungs, bronchiectasis and airway wall thickening with mosaicism, worse in the lower zones. What is the most likely diagnosis? a. Acute transplant rejection b. Bronchiolitis obliterans c. Cytomegalovirus infection d. Post-transplant lymphoproliferative disease e. Reperfusion syndrome
b) Bronchiolitis obliterans Bronchiolitis obliterans is a manifestation of chronic rejection. It usually occurs 6–18 months after transplant, but can occur as early as 3 months. It occurs in approximately 50% of patients and is a major cause of mortality in these patients. Repeated episodes of acute transplant and cytomegalovirus infection are predisposing factors. The radiological features in this condition are that of bronchiectasis: mildly hyperinflated lungs, airway wall thickening and mosaicism (representing air trapping). Post-transplant lymphoproliferative disease can occur from 1 month to several years after lung transplantation. It represents lymphoid proliferation (B- or T-cell proliferation) on a spectrum of benign proliferation to high grade lymphoma. It usually occurs after Epstein-Barr viral infection. Radiographically it manifests as single or multiple nodules, less commonly consolidation and hilar or mediastinal lymph nodes. These do not match the description provided in the main stem. The description in the main stem is also not consistent with acute transplant rejection or reperfusion syndrome, as these complications occur in the acute setting. Infection can occur at any time following transplant, however the radiological features described, and the time frame provided is typical for bronchiolitis obliterans.
1401
A 54 year old patient with chronic hepatitis B infection and a high body mass index has an abdominal ultrasound to monitor her liver. Her previous scan was 12 months previously. The most recent ultrasound shows no sonographic evidence of cirrhosis; however, there is an indeterminate hypoechoic area in the left lobe. Further liver imaging is advised. With regard to liver MRI, which of the below is correct regarding normal liver appearances?
(c) T1: Spleen < liver, T2: Spleen > liver, No change between in- and out-of-phase Liver MRI is frequently employed to help clarify either CT or ultrasound appearances, especially for challenging cases such as in hepatitis B surveillance or in patients with a high body mass index, which can make ultrasound challenging. Normal liver parenchymal signal is hyperintense compared to the spleen on T1 and hypointense compared to the spleen on T2 and there should be no reduction of signal on out-of-phase imaging. Liver signal is also frequently compared to muscle – it should be a similar signal except on inversion recovery sequences. Diffuse signal reduction on out-of-phase imaging can be suggestive of a fatty liver and similarly the T1 signal may be increased in these patients.
1402
A 7 year old boy undergoes an MRI brain. This shows a dilated, high-riding third ventricle which appears to communicate with the interhemispheric cistern. The lateral ventricles appear widely spaced with small frontal horns. The interhemispheric fissure is also widened. Given this description, what is the most likely diagnosis? a. Cavum septum pellucidum b. Cavum vergae c. Cavum velum interpositum d. Corpus callosum agenesis e. Interhemispheric arachnoid cyst
(d) Corpus callosum agenesis Partial dysgenesis of the corpus callosum usually manifests as the posterior portion being absent and is frequently asymptomatic. However, this is not the case in complete agenesis. The description of the brain in agenesis is of a ‘racing car’ appearance due to widely separated lateral ventricles. The third ventricle is elevated, often located between the lateral ventricles; it is dilated and may communicate with the interhemispheric cistern. The interhemispheric fissure is widened and the splenium is absent. Cavum septum pellucidum, cavum vergae and cavum velum interpositum are all normal variants of an additional cerebrospinal fluid space in the midline. An interhemispheric arachnoid cyst is possible and associated with corpus callosum abnormality; however, the description in the question is typical for corpus callosum agenesis.
1403
A 48 year old patient has had surgical resection for a right frontal lobe glioblastoma, followed by radiotherapy and chemotherapy. The MRI brain following treatment demonstrates a lesion in the right frontal lobe with FLAIR hyperintensity and mass effect. The clinical team are concerned about tumour recurrence. Following further imaging, including MRI spectroscopy and MR perfusion studies, the multidisciplinary team conclude that it most likely represents cerebral radiation necrosis. Which of the following characteristics of the areas of FLAIR hyperintensity in the right frontal lobe fit with this diagnosis?
a) Enhancement present, Low choline level, Low relative cerebral blood flow (rCBF) Radiation necrosis can occur in the years following treatment with radiotherapy. The lesions can look very similar to tumour recurrence with rim enhancement; however, there are features which can help to differentiate between them. Whereas tumour recurrence would have a choline peak on MRI spectroscopy, radiation necrosis does not. Similarly, tumour recurrence causes an increase in relative cerebral blood flow, whereas radiation necrosis causes a reduction. 18F-FDG PET/CT can also be employed; in tumour recurrence there would be increased tracer uptake compared to radiation necrosis. Pseudoprogression also has the same features as radiation necrosis on MRI spectroscopy and perfusion studies.
1404
A 6 year old child with known scoliosis is referred for an MRI spine due to progressive bilateral lower limb leg weakness and sensory loss. The T2 weighted sagittal sequence shows an abnormally widened dural space from T10 to L2. The T2 weighted coronal shows increased interpedicular distance, cord widening and splitting at this level. Axial T2 weighted sequences demonstrate a thin hypointense septum between the cord. What is the most likely diagnosis? a. Diastematomyelia b. Diplomyelia c. Myelomeningocele d. Syringomyelia e. Tethered cord
(a) Diastematomyelia This is a congenital cord malformation causing a sagittal division of the spinal cord splitting it into two hemicords which usually join together again more caudally. Each hemicord has its own central canal, dorsal horn and ventral horn. This is in contrast to diplomyelia which is complete cord duplication. In diastematomyelia the lower thoracic and upper lumbar levels are most commonly affected. The condition is almost always symptomatic. Patients frequently have scoliosis and can present secondary to its association with a tethered cord with lower limb neurology and bowel and bladder dysfunction. Radiological signs include a widened interpedicular distance and vertebral anomalies. On MRI the cord can be seen splitting and there is sometimes a fibrous or bony spur between the two hemicords. Myelomeningocele is a type of spina bifida; not only are the meninges and cerebrospinal fluid present in the neural tube defect, but neural tissue too. This does not involve any splitting of the spinal cord. Syringomyelia, or syrinx, is a collection of fluid centrally in the cord and often occurs secondary to other conditions such as myelomeningocele, Chiari or Dandy-Walker malformations.
1405
A 70 year old woman presents with a two day history of left sided weakness. A CT brain shows acute intracerebral haematoma within the right frontal lobe with surrounding oedema. The patient goes on to have an MRI brain the same day. What is the most likely signal characteristic of the haematoma on MRI?
(d) Isointense T1 signal with low T2 signal It is helpful to know the appearance of haemorrhage on MRI both for exams and everyday reporting. One mnemonic is I-Be, Id-De, BiDy, BaBy, DoDo. Once you can remember the signal intensities and relevant time periods, these questions are quite straightforward. This patient’s symptoms suggest she is in the acute period and therefore the haematoma would be isointense on T1 weighted imaging and low signal on T2 weighted imaging. Hyperacute (up to 6 hours): T1 Isointense, T2 Bright Acute (8–72 hours): T1 Isointense, T2 Dark Early subacute (3–7 days): T1 Bright, T2 Dark Late subacute (Weeks to months): T1 Bright, T2 Bright Chronic (Months +): T1 Dark, T2 Dark
1406
A 29 year old female has a CT scan of the chest. This shows an incidental finding of lobulated soft tissue within the anterior mediastinum which is thought to represent a thymic abnormality. She undergoes an MRI scan to further characterise this lesion. Which of the following sequences is most useful to assess for thymic hyperplasia? a. Diffusion weighted imaging b. In- and out-of-phase imaging c. Post-contrast fat saturated T1 imaging d. T1 weighted imaging e. T2 weighted imaging
(b) In- and out-of-phase imaging Thymic hyperplasia can be divided into true hyperplasia (secondary to chemotherapy, radiotherapy or steroids) and lymphoid hyperplasia (associated with systemic lupus erythematosus/rheumatoid arthritis/Addison disease/Graves disease). Both types demonstrate diffuse symmetric enlargement of the thymus. Rebound hyperplasia can occur after the stressor has been removed, where the thymus grows even larger. On CT, the diffuse symmetric appearance of the thymus is the key feature in differentiating thymic hyperplasia from neoplasm; the latter is usually a focal mass. In addition to the morphologic features, MRI imaging with in- and out-of-phase sequences may be of use. On the out-of-phase sequences, thymic hyperplasia will demonstrate signal dropout due to chemical shift artefact, this is not seen in thymic neoplasms.
1407
A cranial ultrasound is performed on a premature neonate. Germinal matrix haemorrhage is identified. Which of the following radiological findings indicates the worst prognosis? a. Hyperechoic foci in the peritrigonal area b. Intraparenchymal haemorrhage c. Intraventricular haemorrhage d. Intraventricular haemorrhage with ventricular dilatation e. Subependymal haemorrhage
(b) Intraparenchymal haemorrhage Germinal matrix haemorrhage occurs in premature infants; after around 36 weeks gestation the germinal matrix is no longer present. It is frequently detected on neonatal cranial ultrasound as hyperechoic areas in the caudothalamic groove adjacent to the ventricles. Germinal matrix haemorrhage is graded I to IV in severity. Grade I is limited to the caudothalamic groove, grade II is intraventricular extension without hydrocephalus, grade III is intraventricular extension with ventricular dilatation and grade IV is parenchymal involvement, which is associated with high mortality.
1408
You are asked to review the MRI brain protocol for cases of suspected multiple sclerosis at your institution. Which of the following sequences is best for assessing for posterior fossa involvement? a. DWI and ADC map b. FLAIR c. Gradient echo d. T1 post gadolinium e. T2 spin echo
(e) T2 spin echo MRI is one of the tools that can help in the diagnosis and monitoring of multiple sclerosis. Lesions in the brain are typically T2 and FLAIR hyperintense and T1 hypointense. There can be a characteristic ‘Dawson’s finger’ pattern of lesions arranged perpendicular to the lateral ventricles; this is best appreciated on sagittal FLAIR images. An important feature is multiple white matter lesions separated in time and place, and the use of gadolinium can delineate active plaques versus inactive lesions. FLAIR is the best sequence for supratentorial assessment. T2 spin echo/STIR/proton echo/STIR/proton density sequences are best for imaging the posterior fossa. Active disease may have either increased or decreased diffusion.
1409
An 81 year old retired ship builder with a pacemaker has a CT pulmonary angiogram for pleuritic chest pain and shortness of breath. There is no evidence of pulmonary embolus; however you notice that he has small bilateral pleural effusions and widespread bilateral pleural thickening involving the hemidiaphragms. There are a couple of prominent, but not frankly enlarged, hila nodes. Ultrasound guided pleural biopsy is inconclusive. The clinical team call you asking for advice regarding further imaging to help distinguish malignant from benign pleural thickening. What is the most appropriate test? a. Contrast enhanced ultrasound b. CT abdomen and pelvis c. MRI d. 18F-FDG PET/CT scan e. Portal venous phase contrast CT chest
(d) 18F-FDG PET/CT scan 18F-FDG PET/CT scan is used to help differentiate benign from malignant pleural thickening and can also help to assess for nodal metastases in malignant mesothelioma. Contrast-enhanced ultrasound is not typically used for this indication. CT abdomen and pelvis would be more appropriate once malignancy was confirmed to exclude peritoneal involvement. On MRI, high T2 signal intensity when compared to muscle, and contrast enhancement of the pleural thickening is suggestive of malignancy; however, this patient has a pacemaker which would likely contraindicate MRI. MRI is sometimes used in the staging of malignant mesothelioma, particularly where there is a question regarding surgical resection and the extent of chest wall or diaphragmatic involvement. Portal venous phase contrast CT chest is better at demonstrating pleural enhancement than an arterial phase CT; however, 18F-FDG PET/CT is more sensitive for distinguishing between malignant and benign disease. CT features that favour a malignant process include pleural thickness >1 cm, nodularity and mediastinal pleural involvement.
1410
A 47 year old female patient is reviewed in the rheumatology clinic with joint pains in her hands. Radiographs are requested which demonstrate a bilateral symmetrical arthropathy with osteopenia, reduction in joint space, osteophytosis and flattening of the index and middle finger metacarpal heads. There are subchondral cysts and irregularity of the articular surface which is particularly affecting the metacarpal phalangeal joints and the carpal bones. What other imaging appearance would correspond with these findings? a. Dilated oesophagus on chest radiograph b. Hyperdensity in the sagittal sinus on CT head c. Pancreatic low signal on T1, T2 and T2* sequences on MRI of the upper abdomen d. Multiple small foci of subcortical T2* low signal on MRI head e. Symmetrical hilar and mediastinal lymph node enlargement on CT chest
(c) Pancreatic low signal on T1, T2 and T2* sequences on MRI of the upper abdomen Haemochromatosis causes a symmetrical arthropathy typically affecting the second and third metacarpal phalangeal joints causing osteopenia, reduction in joint space, subchondral cysts, flattening of the metacarpal heads and ‘hook-like’ osteophytes. Chondrocalcification is common and overlaps with calcium pyrophosphate dihydrate crystal deposition disease. Haemochromatosis is associated with iron deposition in the liver, spleen, heart, pancreas and central nervous system, causing low signal on T1, T2 and T2* MRI sequences. A dilated oesophagus would be associated with scleroderma – acro-osteolysis causing ‘pencil-in- cup’ deformities; soft tissue atrophy and calcification and contractures are also features. There is often marked ulnar deviation of the phalanges. Systemic lupus erythematosus (SLE) causes symmetrical muscle wasting, periarticular osteopenia and reversible deformities such as ‘swan-neck’ deformities and ulnar deviation. SLE is associated with hypercoagulability, and patients are at risk of venous thrombosis which could manifest as intracranial venous sinus hyperdensity. Multiple small foci of subcortical T2* low signal on MRI head are typical of microhaemorrhages in amyloidosis. Larger joints tend to be affected rather than the hands, and features include subchondral cysts, erosions and osteoporosis without joint space narrowing. Soft tissue deposition of amyloid can lead to periarticular nodularity. Sarcoidosis causes symmetrical hilar and mediastinal lymphadenopathy. The musculoskeletal manifestations include a typical ‘lace-like’ appearance along with soft tissue swelling, cyst formation and resorption of the terminal tufts.
1411
An adult male patient with abdominal pain undergoes a CT scan of the abdomen and pelvis under the surgical team. The bowel loops are unremarkable and there is no free gas. However, there is a 5.5-cm bulky tumour centred on the right adrenal gland which has an irregular margin, foci of low attenuation and demonstrates heterogeneous enhancement. The tumour does not contain calcification or haemorrhage, and abuts but does not invade the inferior vena cava. There are multiple low attenuation, ill-defined lesions seen within the liver and several lucent foci within the lumbar spine, which are suspicious for metastases. The lungs have not been included on the scan. What is the most likely cause of the appearances of the right adrenal gland? a. Adrenocortical carcinoma b. Collision tumour c. Metastasis from adenocarcinoma d. Myelolipoma e. Phaeochromocytoma
(c) Metastasis from adenocarcinoma The most likely diagnosis is metastatic adenocarcinoma. Metastases to the adrenal gland can be large with a heterogeneous appearance, central necrosis, haemorrhage and irregular borders. The most common sites of primary malignancy are lung, breast, melanoma, gastrointestinal tract and renal. The main differential diagnosis is adrenocortical carcinoma. This has peak incidences in early childhood and in the fourth and fifth decade of life. This tumour is usually large at diagnosis and typically has central necrosis or haemorrhage. There is calcification in 30% and it can invade the inferior vena cava or the renal vein. However, adrenocortical carcinoma is relatively rare compared to metastatic adenocarcinoma. A collision tumour is a rare occurrence of two different pathologies in the same place and can be benign or malignant. There are no specific features described to indicate that lesion could be a myelolipoma, such as fat density or punctate calcification. A phaechromocytoma would usually demonstrate homogeneous enhancement. They are prone to haemorrhage and then can appear heterogeneous. Most are benign but some may be malignant and demonstrate metastases; however, this occurs less commonly than adenocarcinoma metastases
1412
A usually fit and well 73 year old man is investigated for weight loss and anaemia. There is a history of left nephrectomy for organ donation 5 years previously. Contrast enhanced CT chest, abdomen, pelvis is performed as part of this workup and demonstrates multiple endophytic, homogenous right renal lesions. The renal lesions appear mildly hypodense to the surrounding parenchyma. There are several enlarged, rounded retroperitoneal and pelvic lymph nodes. The renal vessels opacify normally. There are homogenous appearances of the other solid upper abdominal viscera. The chest is clear. The spleen measures 15.2 cm in craniocaudal extent. Renal MRI is performed to help characterise further and the renal lesions exhibit intermediate T1 and intermediate T2 signal. What is the most likely diagnosis? a. Leiomyosarcoma b. Metastases c. Primary lymphoma of the kidney d. Renal cell carcinoma e. Secondary lymphoma of the kidney
(e) Secondary lymphoma of the kidney The imaging characteristics of the lesions, along with abdominopelvic lymphadenopathy and mild splenomegaly, make lymphoma most likely. Lymphoma of the kidney can be split into primary and secondary forms. The primary form is uncommon because the kidneys do not contain lymphoid tissue. The secondary form occurs because of haematogenous spread and is most likely to be non-Hodgkin’s lymphoma. Unlike renal cell carcinoma (RCC), typically lymphoma does not significantly affect the renal contour. Lymphomas often look slightly denser compared to adjacent normal renal tissue prior to contrast and are comparatively hypodense to the rest of the kidney following contrast. The MRI appearances are usually intermediate T1 and intermediate to low T2 signal. Lymphoma is typically more homogenous in appearance compared to RCC. Vascular invasion is an important feature when evaluating renal masses; RCC and leiomyosarcoma are more likely to invade the vessels, whereas lymphoma can encase them. Metastases do not typically invade the vessels either and are usually small lesions and cortically based. The most common primaries which metastasise to the kidneys are melanoma, lung, colorectal and breast. No other primary site is identified on the CT, although a melanoma primary is unlikely to be radiologically visible. The patient is male, making breast malignancy unlikely. CT cannot exclude gastrointestinal malignancy, and endoscopy could be considered, although if there was disseminated colorectal cancer, liver and lung metastases would also be expected. Leiomyosarcomas spread haematogenously; therefore lymphadenopathy is uncommon and the lesions are often positioned close to the renal pelvis and vessels.
1413
A young child has been diagnosed with an infantile haemangioendothelioma on imaging after presenting with a right upper quadrant mass and cardiac failure. An ultrasound confirmed a vascular heterogenous mass which was confirmed as a haemangioendothelioma following an MRI abdomen. Which of the following post contrast MRI findings correlates most closely with this diagnosis? a. Contrast washout on delayed phase imaging b. Early peripheral enhancement with delayed central enhancement c. Heterogenous arterial phase enhancement d. Heterogenous delayed phase enhancement e. Minimal enhancement on all phases
(b) Early peripheral enhancement with delayed central enhancement Infantile hepatic haemangioendothelioma are vascular lesions causing arteriovenous shunting which can lead to high-output cardiac failure. These lesions are also associated with thrombocytopaenia due to consumptive coagulopathy. They are heterogenous vascular masses which can contain calcification. The aorta above the level of the coeliac trunk may be enlarged compared to the aorta below this level. Enhancement of these lesions is similar to a hepatic haemangioma with peripheral early enhancement and central filling in on delayed phases.
1414
A 30 year old male who has multiple cutaneous nodules presents with recurrent episodes of right sided weakness. An intracranial CT angiogram shows bilateral occlusion of the intracranial portion of the internal carotid arteries. There are extensive leptomeningeal and dural arterial vessel collaterals. What is the most likely diagnosis? a. Radiation vasculitis b. Moyamoya syndrome c. Sickle cell disease d. Cerebral atherosclerosis e. Systemic lupus erythematous
(b) Moyamoya syndrome The description of multiple cutaneous nodules and the finding of bilateral internal cerebral artery (ICA) occlusion is suggestive of Moyamoya syndrome in the setting of neurofibromatosis (NF) 1. Two conditions containing the name Moyamoya exist − a disease and a syndrome. Moyamoya disease translates as ‘puff of smoke’ from Japanese and is a vasculo-occlusive angiopathy. The puff- of-smoke description arises from the appearance of the intracranial collateralisation that occurs secondary to congenital stenosis and occlusion of the distal ICAs; however, it can affect other vessels within the circle of Willis too. On imaging there are commonly watershed infarcts and it may present with intraventricular haemorrhage. Moyamoya syndrome gives the same appearance on angiography but is not the result of a congenital ICA occlusion. It is associated with NF1, sickle cell, radiation, chronic infection, tuberculosis and atherosclerosis.
1415
A junior radiology colleague asks you to review an upper gastrointestinal barium study that they have performed on a 57 year old male patient referred by his GP for difficulty swallowing and non-specific abdominal discomfort. The oesophagus and stomach outline normally with prompt gastric emptying. In the D2 part of the duodenum there is a posterior well-defined nodular protrusion into the duodenal lumen. Barium collects centrally within this protrusion. Transit through the duodenum is swift and the duodenojejunal flexure is positioned to the left of the vertebral column at the level of the duodenal bulb. What is the most likely diagnosis? a. Adenocarcinoma of the papilla of Vater b. Benign lymphoid hyperplasia c. Duodenal ulceration d. Ectopic pancreatic tissue e. Malrotation
(d) Ectopic pancreatic tissue The finding of a smooth nodular protrusion into the duodenum with a central depressed duct remnant is diagnostic for ectopic pancreatic tissue. Benign lymphoid hyperplasia typically demonstrates multiple filling defects and can be a normal variant in children but is indicative of hypogammaglobulinaemia in adults. Duodenal ulcers are most commonly more proximally in the bulbar part of the duodenum and are also more commonly anteriorly rather than posteriorly positioned. An adenocarcinoma of the papilla of Vater would be positioned laterally, expected to have ill-defined edges and may lead to obstruction. The duodenojejunal flexure is normally positioned in this patient and so malrotation would not be a concern.
1416
A 10 year old girl has a radiograph of her distal left arm following a fall at school. The radiographer shows you the film. There is no acute fracture but there is shortening and bowing of the radius with positive ulnar variance and distal dislocation of the distal ulnar with a ‘V’-shaped appearance of the proximal carpal row. What condition is NOT associated with these appearances? a. Diaphyseal aclasia b. Morquio syndrome c. Nail patella syndrome d. Ollier disease e. Turner syndrome
(b) Morquio syndrome Madelung deformity is a condition where the radius is bowed and shortened with subsequent positive ulnar variance causing a ‘V’-shaped appearance of the proximal carpal row. This can potentially lead to distal ulna dislocation. Reverse Madelung deformity has similar appearances but affects the ulna, causing negative ulnar variation. Causes include diaphyseal aclasia, nail-patella syndrome, Ollier disease and Turner syndrome. Morquio syndrome is not associated with it; however Hurler syndrome is. Some causes can be remembered with the mnemonic HIT DOC: Hurler syndrome Infection Trauma Dyschondrosteosis, for example Leri-Weill syndrome Osteochondromatosis (also known as diaphyseal aclasia) Congenital, for example Turner syndrome, nail-patella syndrome, achondroplasia
1417
A 42 year old patient is referred for imaging by his GP with a painless lump on his jaw. A radiograph demonstrates a lucent lesion at the mandibular ramus. This has a multilocular honeycomb-like appearance. It is well defined and corticated. There is evidence of root resorption affecting the adjacent teeth. An MRI reveals a lesion containing cystic and soft tissue signal elements. There are no fluid-fluid levels. The septations and solid components enhance avidly following contrast injection. What is the most likely diagnosis? a. Ameloblastoma b. Dentigerous cyst c. Metastases d. Odontogenic keratocyst e. Radicular cyst
(a) Ameloblastoma Ameloblastoma typically affect patients >40 years of age and presents as a painless hard lump. They are multilocular and characteristically have a ‘bubbly’ or ‘honeycomb’ appearance. On MRI they have both cystic and soft tissue components which enhance avidly. In contrast to this, odontogenic keratocysts often enhance poorly. Ameloblastomas are commonly expansile and can cause root resorption. Metastases are an important differential in this age group; however they usually have more ill-defined margins. Dentigerous and radicular cysts are more commonly unilocular lucencies related to dentition, the former related to the crown of an unerupted tooth and the latter related to the roots.
1418
A 64 year old male ex-smoker with a cough is provisionally diagnosed with a T2a N0 M0 primary lung cancer on CT. CT guided biopsy confirms the diagnosis of primary lung adenocarcinoma. Following multidisciplinary team discussion and surgical planning he is admitted for a right lower lobe lobectomy. Several chest radiographs in the week following surgery have shown an appropriately sited right chest drain but with a persistent moderate right pneumothorax. The clinical team report the chest drain continues to bubble. CT chest demonstrates similar findings to the chest radiographs. Which of the following could help to confirm the suspected diagnosis? a. CT pulmonary angiogram b. Insert a larger bore chest drain c. Remove the chest drain and repeat chest radiograph after 4 hours d. Ventilation-perfusion (VQ) scan e. Xenon ventilation study
(e) Xenon ventilation study The ongoing pneumothorax and bubbling chest drain are suspicious for a bronchopleural fistula. They can occur following surgery or as a sequelae of chemo- or radiotherapy and also sometimes with infections, such as tuberculosis. CT can often be used to diagnose this condition; however, in this case when further clarification is required, a Xenon ventilation study can confirm the diagnosis by demonstrating tracer activity in the pleural space. A CT pulmonary angiogram and a ventilation perfusion (VQ) study would not add anything further to the existing CT chest for this indication and would be more appropriate if pulmonary embolus was suspected. Inserting a larger bore chest drain is unlikely to clinically improve the situation and would not aid diagnosis. Larger bore chest drains can be indicated in the context of pleural effusions with thick fluid, for example empyema or haemothorax. Removing the chest drain when there is suspicion of a bronchopleural fistula could potentially lead to tension pneumothorax and therefore this is not an appropriate management plan.
1419
A 25 year old female presents to her GP with right upper quadrant pain. They refer her for an abdominal ultrasound. This shows a 10-cm mass in the right lobe of the liver which has a hyperechoic central scar. A MRI of the liver is arranged. The lesion is isointense relative to the liver on T1 and slightly T2 hyperintense. The central scar is hypointense on both T1 and T2. Following gadolinium administration, the lesion shows heterogeneous enhancement on arterial phase and is isointense on delayed phase. There is no loss of signal on out-of-phase imaging. The central scar does not show any significant enhancement. What is the most likely diagnosis? a. Focal nodular hyperplasia b. Hepatocellular carcinoma c. Haemangioma d. Fibrolamellar hepatocellular carcinoma e. Hepatic adenoma
(d) Fibrolamellar hepatocellular carcinoma (HCC) Fibrolamellar HCC is an uncommon variant of HCC affecting young patients with no risk factors. The tumour is typically large at diagnosis. The central scar is usually hyperechoic on ultrasound and hypointense on MRI sequences, which helps to differentiate it from the T2 hyperintense central scar in focal nodular hyperplasia (FNH). The central scar generally does not enhance in fibrolamellar HCC but does in FNH. The age of the patient in this case is highly suggestive of fibrolamellar HCC over standard HCC. Fibrolamellar HCC can calcify; however this is less likely in HCC. Furthermore, fibrolamellar HCC is commonly isointense on venous phase imaging, whereas HCC shows early washout. Hepatic adenoma can occur in a similar age group; however they do not contain a central scar. They frequently contain fat and haemorrhage. The former leads to a drop of signal on out-of-phase imaging. Haemangiomas can appear hyperechoic on ultrasound but their enhancement is quite characteristic with peripheral enhancement that progresses centrally with time. They do not contain a central scar.
1420
The consultant looking after a 41 year old female outpatient with normal renal function and urine dip comes to ask you for advice. The patient had an ultrasound abdomen for query gallstones due to intermittent epigastric and right upper quadrant pain. The ultrasound confirmed gallstones but the report also mentioned bilateral echogenic renal pyramids with otherwise normal kidneys. A CT urinary tract from an emergency department attendance 1 month previously also demonstrated hyperdense medullary pyramids with no hydronephrosis. A CT urogram confirmed a striated nephrogram. What is the most appropriate next step in management? a. Antinuclear antibody serum levels b. Blood culture and antibiotics c. No further investigation required d. Review patient medication e. Ultrasound renal Doppler§
c) No further investigation required The imaging appearances are consistent with medullary sponge kidney. On intravenous urograms the appearance used to be likened to a paintbrush; otherwise known as a ‘striated nephrogram’. Other causes for this appearance bilaterally include acute pyelonephritis, acute tubular necrosis, hypotension and autosomal recessive polycystic kidney disease. The latter is associated with paediatric patients. The history does not fit with hypotension, acute pyelonephritis or acute tubular necrosis. Urine dip would typically be abnormal in pyelonephritis and acute tubular necrosis, either showing signs of nitrites, blood or protein. If these conditions were suspected clinically, then answers B and D may be appropriate. If systemic lupus erythematous is suspected then an antinuclear antibody test would be helpful; however, this tends to be associated with glomerulonephritis, which causes swollen kidneys. Renal vein thrombosis would cause a unilateral striated nephrogram and is also unlikely in a patient without risk factors; therefore an ultrasound Doppler is unlikely to provide any additional information. Medullary sponge kidney does not progress to renal failure, and for most patients the condition is asymptomatic. It is often picked up incidentally and further follow-up is not required.
1421
A neonate with vomiting has an abdominal radiograph which demonstrates a distended stomach and duodenum causing a ‘double bubble’ appearance. There is no convincing small or large bowel gas and no free peritoneal gas. Duodenal atresia is suspected. An upper gastrointestinal contrast study is performed. Where is the level of obstruction most likely to lie? a. At D4 just proximal to the duodenojejunal flexure b. At the midpoint of D3 c. Just distal to the ampulla of Vater in D2 d. Just distal to the gastric antrum in D1 e. Just proximal to the ampulla of Vater in D2
(c) Just distal to the ampulla of Vater in D2 The case describes a classic case of duodenal atresia. This may be suspected antenatally if there is polyhydramnios and postnatally within the first week when infants experience vomiting. The radiographic appearance in the case is called the ‘double bubble’ sign. The majority of cases have an obstructive level which is just distal to the ampulla of Vater and hence the vomiting is bilious. Non-bilious vomiting would suggest a more proximal obstruction. The finding that there is no gas more distally in the bowel excludes other differentials such as duodenal stenosis or a duodenal web. Duodenal atresia is associated with an annular pancreas, VACTERL anomalies and Down syndrome. (The Final
1422
A female patient is discussed at the cancer of unknown primary multidisciplinary team meeting. An MRI lumbar spine incidentally detected multiple liver lesions and several paraaortic lymph nodes measuring up to 15 mm in short axis. A malignant looking gastric lesion was subsequently identified on endoscopy. Regarding malignant metastatic gastric lesions, what is the most common primary site? a. Breast b. Colon c. Endometrium d. Pancreas e. Skin
(e) Skin The most common metastatic lesion in the stomach is malignant melanoma. Other common primary sites of metastatic malignancy include breast, lung, cervix, prostate and renal. Spread can be contiguous from adjacent structures, for example via the gastrocolic ligament, or haematogenous. The latter is more common. On a barium meal haematogenous metastases can produce multiple nodules and a characteristic ‘bull’s-eye’ appearance, which is so called due to the central ulceration the lesions demonstrate. Metastases often affect the proximal and middle part of the stomach and can present with weight loss, melaena and haematemesis
1423
A 30 year old man describes a several month history of bilateral facial weakness. An MRI brain is performed. This shows diffuse, nodular thickening of the leptomeningeal layers predominantly within the basal cisterns. These enhance avidly following gadolinium administration. Further areas of high T2/FLAIR signal are seen involving the facial nerves bilaterally. What is the most likely diagnosis? a. Leptomeningeal metastases b. Lymphoma c. Multiple myeloma d. Neurosarcoidosis e. Tuberculous meningitis
(d) Neurosarcoidosis Central nervous system involvement of sarcoidosis can cause symptoms including bilateral facial nerve palsy, seizures and diabetes insipidus. It characteristically causes basal leptomeningeal thickening which can lead to hydrocephalus. Appearances can be similar to tuberculous meningitis; however, bilateral cranial nerve involvement, most commonly the facial or optic nerves, is suggestive of sarcoidosis. Leptomeningeal carcinomatosis can be due to a primary intracranial tumour however there would be evidence of this on the MRI. Lymphoma, leukaemia and metastatic disease, for example from a breast or lung primary, are possible; however this tends to cause diffuse thickening affecting all parts of the brain rather than having a basal predilection.
1424
A 14 year old boy presents with abdominal pain and vomiting. He is assessed by the paediatric team and found to be anaemic. He is considered to be short for his age and the paediatric team suspect a systemic aetiology, and amongst other investigations he has a plain film of the wrist. The radiograph demonstrates bands of increased density at the distal metaphysis of the radius. There is a ‘bone-in-bone’ appearance of the bones. What is the most likely underlying diagnosis? a. Rickets b. Lead poisoning c. Physiological d. Scurvy e. Trauma
(b) Lead poisoning Lead poisoning may present with loss of appetite, abdominal pain, constipation and vomiting; patients may also be anaemic. The presence of high-density bands within the metaphysis represents lead deposition. Patients may be short for their age, with skeletal immaturity present on imaging, and a ‘bone-in-bone’ appearance may also be seen. Metaphyseal bands can also be seen in healed rickets; however ‘bone-in-bone’ appearance is not a typically described feature of this condition. The presence of dense metaphyseal bands may be physiological in children aged less than 3 years old. Dense metaphyseal bands can be seen in scurvy, which is relatively uncommon. It is caused by dietary lack of vitamin C. Other imaging features include generalised osteopenia, cortical thinning, periosteal reaction, Wimberger ring sign, Frankel line and Trümmerfeld zone. Fracture at the distal radius may cause increased density at the metaphysis; however it does not account for the other clinical and radiographic features described.
1425
A 61 year old male patient attends hospital with a history of haematemesis. His observations are stable. During admission a contrast enhanced CT abdomen and pelvis reveals a 45-mm soft tissue and cystic density mass centred on the stomach with peripheral enhancement. There is no size-significant lymph node enlargement. Biopsy reveals that the mass expresses c-KIT (CD117) antigen. Apart from some mild lower lobe atelectasis, a CT chest is normal. An MRI head from 5 years ago demonstrated an abnormal left sphenoid wing and several areas of bilateral high T2 weighted signal in the basal ganglia and deep white matter. What is the most likely diagnosis affecting the stomach? a. Gastric hamartoma b. Gastrointestinal stromal tumour c. Lymphoma d. Metastatic small cell lung malignancy e. Gastric scirrhous adenocarcinoma
(b) Gastrointestinal stromal tumour The gastric soft tissue mass has imaging features in keeping with a gastrointestinal stromal tumour (GIST). They can be very large and have necrotic, haemorrhagic and cystic components. They may be asymptomatic or present with vague abdominal symptoms. Sometimes, similar to this case, they can present with haemorrhage. The most common site is the stomach, followed by the small intestine. They are usually benign but larger tumours, greater than 5 cm, are more likely to have malignant potential. Ninety percent of GISTs express c-KIT (CD117) antigen, a tyrosine kinase growth factor receptor. GISTs can be associated with several syndromes, including neurofibromatosis type I. The intracranial appearances are consistent with this – sphenoid wing dysplasia and focal areas of signal intensity (FASIs). Carney triad is also associated with GISTs. The lack of lymph node enlargement makes malignancy such as gastric adenocarcinoma, metastatic malignancy or lymphoma less likely. Gastric hamartomas can be seen in several polyposis syndromes such as Cowden and Peutz-Jegher syndrome.
1426
A 65 year old male patient with haematuria is sent by his GP for an ultrasound abdomen which describes a 5-cm isoechoic left renal mass. CT chest abdomen pelvis shows a heterogenous left renal lesion with coarse calcific foci and heterogenous enhancement. There is no enhancement centrally and no vascular invasion; however there is left perinephric stranding. The adrenal glands are normal, there is no size significant lymph node enlargement and the lungs are clear. Ultrasound guided biopsy was challenging due to the position of the lesion, and the patient did not tolerate it well. Limited tissue was acquired which could not differentiate between oncocytoma and renal cell carcinoma. The patient is referred to the urology multidisciplinary team meeting. From the following options, what is the most appropriate next step in management? a. MRI kidneys b. Referral to a sarcoma centre c. Surgical resection d. Repeat US guided biopsy e. Ultrasound guided ablation
c) Surgical resection The imaging description is of a heterogenous renal lesion with coarse calcification and a central scar. The differential lies between renal cell carcinoma and oncocytoma. Classical imaging characteristics for an oncocytoma include an isoechoic renal mass with a hypoechoic centre on ultrasound. CT of larger lesions such as this one can demonstrate a heterogenous mass and there can be perinephric fat stranding. Both CT and MRI demonstrate the typical non- enhancing central scar. Other MRI features include T1 hypointensity and T2 hyperintensity compared to the renal cortex. In this case an MRI is unlikely to add more information. Oncocytomas are benign but they are difficult to differentiate from renal cell carcinoma on imaging alone, and even a biopsy of a carcinoma can contain oncocytic components. Renal sarcomas are aggressive lesions. Often at this size there may be vascular invasion, and early haematogenous metastases are common. The CT scan on this patient does not demonstrate evidence of metastatic infiltration. Therefore, the most appropriate answer is surgical resection to help confirm what the lesion is and plan for the most appropriate management. Repeat ultrasound guided biopsy may not be successful given the issues during the first attempt. Ablation would not help with histological diagnosis and is therefore not indicated.
1427
A child presents with jaundice and a right upper quadrant mass. Sonographic appearances and MRCP is consistent with Caroli disease. Which choledochal cyst distribution is consistent with this diagnosis? a. Dilatation of distal extrahepatic duct within the duodenal wall b. Dilatation of the intrahepatic ducts c. Diverticulum of the extrahepatic duct d. Extrahepatic duct dilatation e. Intra- and extrahepatic duct dilatation
(b) Dilatation of the intrahepatic ducts The Todani classification helps differentiate between choledochal cysts, and ranges from I to V. Caroli disease is the cystic dilatation of intrahepatic ducts and consistent with Todani type V. Type I is fusiform cystic dilatation of the extrahepatic duct. Type II is an extrahepatic bile duct diverticulum. Type III is dilatation of the extrahepatic bile duct within the duodenal wall. Type IV is the presence of intra- and extrahepatic cysts
1428
A 29 year old patient with a known genetic condition has a chest radiograph for a new cough productive of sputum. You review the plain film with several previous films that are available. There is a large, well circumscribed opacity in the posterior mediastinum with associated focal thoracic scoliosis. There are also multiple nodules projected over the lungs with evidence of cutaneous soft tissue nodules. These findings are unchanged from the previous imaging. There is also new consolidation in the left lower lobe. Which of these findings are you LEAST likely to see on subsequent high resolution CT chest? a. Basal symmetrical lung fibrosis b. Pectus excavatum c. Posterior vertebral scalloping d. Ribbon ribs e. Tracheobronchomegaly
(e) Tracheobronchomegaly Basal lung fibrosis, pectus excavatum, posterior vertebral scalloping and ribbon ribs are all thoracic manifestations of neurofibromatosis (NF) type 1 (von Recklinghausen disease). This is the genetic condition described in the main stem. The cutaneous nodules seen on the plain film represent cutaneous neurofibromas. The posterior mediastinal mass described could represent a meningocele or neurofibroma (fluid content would favour meningocele). Posterior scalloping of the vertebral bodies and ribbon ribs can occur secondary to adjacent neurofibroma. Pectus excavatum is sometimes associated with other conditions including Marfan syndrome and Ehlers-Danlos syndrome. It can be seen on plain film as blurring of the right heart border and displacement to the left, mimicking middle lobe consolidation. The posterior ribs are also more horizontal and the anterior ribs are more vertical. Tracheobronchomegaly is not a feature of NF 1.
1429
A 10 year old girl is referred for an MRI spine due to lower limb sensory and motor deficit. This shows a single cord with termination at the L3 level. There is an intradural, extramedullary mass adjacent to the conus which is T1 and T2 hyperintense. There is no post- contrast enhancement or restricted diffusion. The mass does not extend into the dorsal subcutaneous tissues. Appearances of the vertebrae are within normal limits. What is the most likely diagnosis? a. Diastematomyelia b. Ependymoma c. Lipomyelomeningocele d. Paraganglioma e. Tethered cord syndrome
(e) Tethered cord syndrome The cord usually terminates around the level of L1/2; below this level it is low and is associated with a tethered cord. There are various associations with tethered cord including spinal lipoma, diastematomyelia, thickened filum terminale, Chiari malformation, myelomeningocele and dermal sinus. In this case, the cord is tethered due to a lipoma; it follows fat on all sequences and does not enhance. Ependymoma and paraganglioma are other causes of filum terminale masses. Ependymomas are the most common but they are often more heterogenous than the mass described in this question with haemorrhage, cystic change and calcification. Paragangliomas are also usually more heterogenous and they also intensely enhance. Lipomyelomeningocele is a form of spina bifida. Although they do contain fat, it extends within the subcutaneous tissues and there are other important features such as neural arch defects. The bones in this case are normal. Diastematomyelia is a split cord malformation however the question states that there is a single cord.
1430
A 50 year old man with a history of sickle cell anaemia is diagnosed with locally advanced prostate cancer. At his outpatient oncology appointment he reports he has been experiencing some lumbar back pain and therefore a lumbar spine MRI is requested. There is no canal stenosis, no evidence of significant intervertebral disc disease but there is multilevel central end plate depression. The marrow is mildly hyperintense on T2 weighted sequences and comparatively lower on T1 weighted imaging. T1 in- and out-of-phase imaging is performed which demonstrates signal reduction on the T1 out-of-phase sequence. What is the most likely diagnosis? a. Haemosiderosis b. Metastatic infiltration c. Myelofibrosis d. Red marrow reconversion e. Yellow marrow reconversion
(d) Red marrow reconversion In childhood red (haematopoietic) marrow predominates in the skeleton but as we age this is converted, distally to proximally, to yellow (fatty) marrow. In adulthood only the axial skeleton continues to contain red marrow. Some conditions promote the reconversion of yellow to red marrow, including conditions associated with anaemia, long distance running and altitude. Signal characteristics of yellow marrow follow subcutaneous fat on all sequences, demonstrating high signal on T1 and T2 sequences. Red marrow is mildly hyperintense on T2 sequences but although on T1 imaging will be hyperintense to disc and muscle, it can be comparatively lower than the T2 signal and can be difficult to differentiate from metastatic marrow infiltration. In- and out-of-phase imaging can be helpful as the fat in yellow marrow will cause a drop in signal on the out-of- phase sequence which will not occur in metastatic infiltration. Haemosiderosis may be possible in this case due to episodes of haemolysis or potentially repeated transfusion but it would be expected to cause reduced T1 and T2 signal. Myelofibrosis is unlikely in this case but causes low signal on T1 and T2 sequences.
1431
A neonate is born at 35 weeks’ gestation following premature rupture of membranes and emergency caesarean section. The child develops tachypnoea and nasal flaring within the first 24 hours. The clinical team are concerned about infant respiratory distress syndrome due to surfactant deficiency. A chest radiograph is requested. Which of the following radiographic features would be most likely to be associated with respiratory distress syndrome caused by surfactant deficiency? a. Interstitial oedema b. Lack of air bronchograms c. Normal chest radiograph at 6 hours d. Perihilar streaky opacities e. Reduced lung volumes
(e) Reduced lung volumes Respiratory distress syndrome in neonates is due to surfactant deficiency. It affects premature infants (<37 weeks) and is associated with low lung volumes, air bronchograms and granular opacities. If a chest radiograph is normal at 6 hours it excludes the diagnosis. Interstitial oedema is more commonly associated with transient tachypnoea of the newborn, which also affects babies relatively quickly following delivery and is more commonly associated with term infants delivered by caesarean section. It usually resolves within days. Meconium aspiration is typically associated with increased lung volumes, streaky perihilar opacities and lack of air bronchograms. Perihilar opacities may also be seen in neonatal pneumonia.
1432
A 44 year old male patient has had repeated GP attendances for mild fever, cough and breathlessness over the past 3 months. Several chest radiographs over this time have demonstrated patchy air space opacification varying in distribution with intervening normal chest films. Which investigation could confirm the diagnosis? a. Anti-basement membrane antibody b. Bronchoalveolar lavage c. cANCA d. Serum eosinophil count e. Urine 5-HIAA levels
(d) Serum eosinophil count Simple pulmonary eosinophilia, or Löffler syndrome, is associated with elevated serum eosinophil levels and presents with transient air space opacification. Anti-basement membrane antibody is associated with Goodpasture syndrome. This tends to cause pulmonary haemorrhage which may have similar appearances on chest radiograph but would be associated with more constitutional symptoms and haemoptysis. Bronchoalveolar lavage demonstrating eosinophils would help to diagnose acute or chronic eosinophilic pneumonia. Both are associated with raised eosinophils in bronchoalveolar fluid, and the latter also with serum eosinophilia. Serum cANCA positivity is found in vasculitic conditions such as eosinophilic granulomatosis with polyangiitis (Churg-Strauss) and microscopic polyangiitis. The presence of 5-HIAA in urine suggests increased serotonin metabolism, which may be a feature of a carcinoid tumour.
1433
A CT abdomen pelvis requested to assess for diverticulitis as a cause of upper abdominal discomfort in a 63 year old woman identifies a cystic right ovarian lesion. A subsequent MRI pelvis helps to evaluate this lesion further. It is predominantly high signal on T2 weighted imaging with a thin septation and a small amount of enhancing, nodular, peripheral, intermediate T1 and T2 signal. The lesion otherwise demonstrates low T1 signal. There are no size-significant lymph nodes within the pelvis. There is a trace of free fluid in the pouch of Douglas. The left ovary has normal post-menopausal appearances. What is the most likely diagnosis? a. Kruckenberg tumour b. Mucinous cystadenoma c. Mucinous cystadenocarcinoma d. Serous cystadenoma e. Serous cystadenocarcinoma
(e) Serous cystadenocarcinoma The ovarian lesion has features concerning for malignancy due to the enhancing, peripheral, intermediate T1 and T2 signal area. The most common malignant ovarian tumour is a serous cystadenocarcinoma. These often have a large cystic component and enhancing soft tissue which may be papillary in appearance and demonstrate restricted diffusion. In contrast to this, serous cystadenomas should be simple and cystic with no soft tissue component. Mucinous cystadenomas often have multiple septations, and due to their mucin content may have more varied signal than a serous cystadenoma. The lesion is frequently described as having a ‘stained glass’ appearance on MRI. A mucinous cystadenocarcinoma would have these features but also suspicious findings such as mural thickening or solid components. Kruckenberg tumours are metastatic ovarian lesions, commonly from the gastrointestinal tract, breast or lung. They are frequently bilateral and have complex appearances containing enhancing solid components and mucin
1434
A 3 year old boy is referred for an MRI brain. The request form says he has a facial cleavage abnormality and therefore a diagnosis of holoprosencephaly is suspected by the paediatric team. If holoprosencephaly is confirmed on the MRI, which structure will be absent? a. Corpus callosum b. Falx cerebri c. Olfactory tracts d. Septum pellucidum e. Third ventricle
(d) Septum pellucidum There are differing severities of holoprosencephaly, ranging from alobar to lobar depending on the degree of cleavage that has occurred. The unifying characteristic of all forms is the absence of the septum pellucidum. With the alobar subtype there is no cleavage of the cerebral hemispheres. There is no corpus callosum or falx cerebri, just one anterior cerebral artery and the thalami are fused. There is commonly an associated facial abnormality. In semilobar holoprosencephaly there is partial cleavage posteriorly and the thalami are partially separated, but anteriorly there is still no hemispheric cleavage. The corpus callosum may be absent or hypoplastic. In lobar forms of the condition it is just the frontal lobes and the frontal horns of the lateral ventricles which remain fused. There is either complete or partial cleavage of the thalami and the falx and interhemispheric fissure is formed. The corpus callosum may be hypoplastic.
1435
A barium swallow is performed for symptoms of regurgitation and dysphagia in a 42 year old female patient. There is a smooth, wide-based filling defect projecting into the lumen in the lower third of the oesophagus causing slow transit of barium. There is no evidence of mucosal ulceration. On review of the control images there is calcification in this region. What is the most appropriate next step? a. Breast triple assessment b. Endoscopic ultrasound and 18F-FDG PET/CT c. Follow-up in 1 year d. MRI head, physical assessment and family history e. Surgical referral
(e) Surgical referral The appearances of a single, broad-based intraluminal filling defect without mucosal irregularity is suggestive of a benign lesion. The calcification is typical of a leiomyoma − they are the only oesophageal masses that calcify. If these are asymptomatic, then they do not need treatment; however, this patient does have symptoms and therefore surgical referral would be indicated. Breast carcinoma is the most common metastatic lesion to affect the oesophagus and often causes multiple submucosal nodules. The appearance of this lesion is also not typical for an oesophageal carcinoma, which frequently causes shouldered stricturing, often with mucosal irregularity. Staging for this would include endoscopic ultrasound and 18F-FDG PET/CT. Neurofibromas can affect the oesophagus and can be difficult to differentiate from a leiomyoma; however they do not typically calcify and they are much rarer. If neurofibromatosis is suspected, then physical and family history assessment would be indicated along with neurological imaging.
1436
An 8 month old patient under the care of ophthalmology has an MRI orbits following clinical review for right proptosis. This demonstrates a unilateral right orbital mass. The mass spans both intra- and extraconal compartments. It is lobulated and septated with signal which is T1 hypointense to fat and T2 isointense to fat but hyperintense to muscle. The mass enhances intensely following contrast injection. There are thin, curvilinear, very low signal foci within the mass. What does this mass most likely represent? a. Capillary haemangioma b. Cavernous haemangioma c. Lymphangioma d. Retinoblastoma e. Venous varix
a) Capillary haemangioma Capillary haemangiomas often affect the periorbital region and are also called strawberry haemangiomas when they are external on the skin surface. However, they can cause intraorbital masses presenting with proptosis. They usually increase in size over the first few months of life and then slowly regress. On imaging they appear as lobulated, septated masses which can span both the intra and extraconal compartments. They have curvilinear flow voids and enhance intensely. Cavernous haemangioma are more common in the adult population and are usually intraconal. They are rounded and well defined with the appearance of a capsule. Their enhancement tends to be slower and more patchy than a capillary haemangioma. Lymphangioma are usually extraconal and are more heterogenous with solid and cystic components and fluid-fluid levels. They can cause sudden proptosis due to haemorrhage. Due to their proteinaceous cystic components they are commonly high on T1 and T2 with minimal enhancement. Retinoblastomas are related to the globe and are heterogenous solid masses containing necrosis and calcification. On imaging, a venous varix appears as a dilated intraconal vessel and can be congenital or acquired, following trauma for example. It presents with intermittent proptosis on coughing or straining, and therefore imaging before and after Valsalva manoeuvre can be helpful.
1437
A 10 year old boy presents with a couple of months history of intermittent left knee pain and is reviewed by the paediatric team. Radiograph demonstrates soft tissue swelling adjacent to his distal left femur. Within the bone, adjacent to the distal femoral metaphysis, there is a longitudinally orientated, ill-defined lytic lesion with surrounding sclerosis. Emanating from this is a linear lytic area extending towards the physis. What is the most likely diagnosis? a. Eosinophilic granuloma b. Giant cell tumour c. Osteomyelitis d. Osteoid osteoma e. Osteosarcoma
(c) Osteomyelitis The case describes a Brodie abscess, which is a form of chronic osteomyelitis and represents an intraosseous abscess. Typical radiographic appearances are described in the question, in particular the orientation along the long axis of the bone. They usually occur in the metaphysis, and the pathognomic sign is the lucent channel extending towards the physis in a patient with an unfused skeleton. They can have similar appearances to osteoid osteoma, although the latter are usually cortically based, but this can vary. Eosinophilic granulomas do occur in children and can affect the metadiaphysis; however they would not typically have a sclerotic rim. Osteosarcomas are more aggressive lesions and would usually cause more adjacent periosteal reaction and soft tissue extension without the sclerosis Giant cell tumours typically involve the metaphysis but tend to occur in an older age group with fused skeletons.
1438
A 62 year old male with known liver cirrhosis secondary to hepatitis C presents with an acute upper gastrointestinal bleed. The patient is fluid resuscitated and transfused. Endoscopy is performed which identifies large oesophageal varices. Unfortunately thrombin injection does not stop the bleeding. An emergency transjugular intrahepatic portosystemic shunt (TIPSS) procedure is planned. Regarding TIPSS, an iatrogenic communication is made between the portal vein and which vessel? a. Inferior vena cava b. Aorta c. Hepatic vein d. Hepatic artery e. Splenic vein
(c) Hepatic vein A transjugular intrahepatic portosystemic shunt (TIPSS) is performed to treat portal hypertension and can be utilised for bleeding oesophageal or gastric varices when endoscopic therapy has failed. Vascular access is usually via the right jugular vein and a shunt is created between the portal vein and a hepatic vein with a stent to allow blood to bypass the liver. Contraindications include severe right heart failure, severe encephalopathy, sepsis and severe chronic liver disease. Complications include haemorrhage, infection and specifically arteriovenous fistula, unintentional gallbladder puncture and hepatic infarct.
1439
A 26 year old male cyclist is involved in a serious road traffic collision and intubated at the scene prior to hospital transfer. A trauma CT of the head, neck, chest, abdomen and pelvis and targeted radiographs at the time of hospital admission identified a small right subdural haemorrhage, Malgaigne pelvic fracture, fracture of the right proximal femur, a stable L1 fracture and fracture dislocation of the right ankle joint. No neck or thoracic injuries were identified. The ITU team report increasing oxygen requirements 2 days following admission and difficulty ventilating the patient. CT chest demonstrates bilateral consolidation and ground glass opacity. What is the most likely diagnosis? a. Aspiration pneumonia b. Fat embolism c. Infective bronchopneumonia d. Lung contusion e. Pulmonary laceration
(b) Fat embolism Fat embolism can be encountered following trauma or other significant insults such as pancreatitis, burns, liposuction and severe sepsis. In the setting of trauma, long bone fractures are often the underlying cause due to fat from bone marrow entering the circulation. Initial imaging is normal, followed by delayed development at around 24–48 hours of consolidation, and ground glass opacity within the lungs, with fat density pulmonary artery filling defects also possible. In contrast to this, lung contusions and pulmonary lacerations would usually be present at the time of the initial trauma imaging. Lung contusions are often seen in the form of consolidation which is frequently peripherally sited at an interface between the lung and a firmer structure, for example the ribs or mediastinum. Pulmonary lacerations cause a defect in the lung which can fill with blood or air, in which case it is called a pneumatocele. Aspiration pneumonia is possible following trauma but the patient was intubated on admission to hospital so if it was not visible on the trauma CT it is unlikely to have developed since then in the presence of an inflated endotracheal tube. The short time interval between admission and the pulmonary abnormalities developing makes bronchopneumonia less likely
1440
A 58 year old woman with dysphagia has a barium swallow following GP referral. This demonstrates a thin transverse filling defect in the upper third of the anterior oesophagus. A diagnosis of Plummer-Vinson syndrome is made. Which other clinical feature is commonly associated with this diagnosis? a. Microcytic anaemia b. Low serum CD4 count c. Positive serum Helicobacter pylori test d. Raised serum gamma-glutamyl transferase e. Vitamin B12 deficiency
(a) Microcytic anaemia Plummer-Vinson syndrome is characterised by an oesophageal web, iron deficiency anaemia and dysphagia. The web is often anteriorly located in the upper third of the oesophagus, but it can be circumferential. Other causes of oesophageal webs include graft-versus-host disease, gastroesophageal reflux disease and pemphigoid. A low CD4 count is associated with an impaired immune system, most commonly secondary to HIV infection. On a barium swallow this may manifest with signs of herpes simplex infection causing small superficial ulcers, candidiasis causing large plaque-like abnormalities with pseudomembrane formation, or cytomegalovirus oesophagitis causing giant ulcers. Raised gamma-glutamyl transferase suggests excess alcohol intake. This may be associated with variceal formation in chronic abuse which cause serpiginous luminal filling defects. Helicobacter pylori is associated with gastro-oesophageal reflux disease and peptic and duodenal ulcers. Vitamin B12 deficiency can be secondary to gastric causes such as atrophic gastritis and following gastrectomy or due to ileal malabsorption, for example in Crohn’s disease.
1441
Following an ultrasound abdomen of a 32 year old patient which identified multiple hyperechoic bilateral renal lesions, the patient has a CT abdomen pelvis which confirms multiple bilateral exophytic renal masses. The largest measures up to 60 mm. All the lesions have similar appearances, with a soft tissue component as well as lower density areas measuring around −30HU. There is no calcification. The lung bases are included on the CT and there are a couple of small thin-walled cysts visible. What is the most appropriate management for the renal lesions? a. MRI kidneys b. No further management required c. Oncology referral d. Referral to urology surgeons e. Repeat CT in 6 months
(d) Referral to urology surgeons The multiple fat-containing renal lesions along with lung cysts are consistent with a diagnosis of tuberous sclerosis; therefore these lesions are likely angiomyolipomas. The main risk for angiomyolipoma is retroperitoneal haemorrhage and lesions that are >40 mm should be considered for embolisation or surgical removal. Lesions that are <20 mm generally do not require follow-up due to slow growth, and angiomyolipoma between these measurements can be followed up with interval imaging. A patient may require MRI kidneys if there is any doubt about the lesions, for example if they are fat poor. Fat-saturated and in- and out-of-phase MRI sequences can be helpful. Patients with tuberous sclerosis are more likely to have renal cysts and can develop renal cell carcinoma at a younger age than the general population.
1442
A 2 year old girl is brought into the emergency department by her parents with fever, cough and inspiratory stridor. The clinical team request a radiograph of the neck. There is subglottic tracheal narrowing and distension of the hypopharynx. The epiglottis and aryepiglottic folds do not appear thickened. The adenoid tonsils are enlarged. On the lateral view the retropharyngeal soft tissues are equivalent in thickness to approximately half a vertebral body width. What is the most likely diagnosis? a. Croup b. Epiglottitis c. Exudative tracheitis d. Pharyngitis e. Retropharyngeal abscess
(a) Croup Croup or laryngotracheobronchitis is a viral upper respiratory tract infection usually affecting children up to the age of 3 years. It causes a characteristic barking cough and can cause inspiratory stridor. The findings on radiograph include subglottic narrowing, referred to as the ‘steeple’ sign, and hypopharyngeal distension. Epiglottitis is an important and potentially life-threatening differential to exclude. This causes epiglottic and aryepiglottic fold swelling which can cause the epiglottis to resemble a thumb. Epiglottitis is less common now due to infant Haemophilus influenzae type B vaccination; however, Streptococcus A can also cause it. These patients should not be laid flat for imaging as this could lead to complete airway obstruction. Exudative tracheitis is less likely in this age group as it usually affects older children. Radiographic findings would include membranous tracheal filling defects and irregularity. Retropharyngeal abscess is suspected if the retropharyngeal soft tissues are thickened; however, they are normal in this case. The subglottic narrowing and hypopharyngeal distension in this case is below the pharynx and therefore not consistent with pharyngitis.
1443
A 49 year old smoker goes to the GP with persistent cough. On examination he is noted to have clubbed fingernails. He has a plain film of the chest which reports marked symmetrical perihilar ground glass opacification. He is referred to the respiratory team who request a CT chest, which shows a crazy paving appearance. The team perform a bronchial lavage. The overall findings are highly suggestive for pulmonary alveolar proteinosis. What is the most likely aetiology? a. Acute silicosis b. Congenital pulmonary alveolar proteinosis c. Haematological malignancy d. Infection with immunosuppression e. Primary (autoimmune) pulmonary alveolar proteinosis
(e) Primary (autoimmune) pulmonary alveolar proteinosis Pulmonary alveolar proteinosis (PAP) usually affects middle-aged men. It is strongly associated with smoking. It represents filling of airspaces with proteinaceous fluid with preservation of the interstitium. Imaging features are often more severe than the clinical symptoms. The ‘crazy paving’ description is typical for PAP but not pathognomonic. It refers to ground glass opacification in combination with smooth interlobular septal thickening. It is usually of primary (or autoimmune) origin (approximately 90%). Secondary PAP is less common. It can be precipitated by silica dust inhalation or haematological malignancy as well as immunodeficiency with infection, for example Cryptococcus, Nocardia or Aspergillus. The information provided in the main stem is most likely in keeping with primary PAP, as this is most common, and no information that could account for a secondary cause has been provided. Congenital PAP presents in the neonatal period and so is not considered a possible aetiology in this case.
1444
A 51 year old female patient is seen in the orthopaedic hand clinic with soft tissue swelling at the palmar aspect of the right middle finger which a radiograph confirms. The adjacent bone has normal appearances. Ultrasound identifies a well-defined hypoechoic solid lesion with mild internal vascularity around the flexor tendon, through which the tendon can be seen moving. On MRI this lesion is low signal on T1 and T2 sequences with evidence of susceptibility artefact on gradient echo images. What is the most likely diagnosis? a. Desmoid tumour b. Fibroma of the tendon sheath c. Ganglion cyst d. Glomus tumour e. Tenosynovial giant cell tumour
e) Tenosynovial giant cell tumour These tumours are pigmented villonodular synovitis affecting the tendon sheath and have similar imaging characteristics on MRI with low T1 and T2 signal and evidence of blooming on gradient echo sequences due to haemosiderin deposition. Although desmoid tumours are most common in the abdomen, they can affect limbs but tend not be in direct contact with the tendon sheath. They can look similar on ultrasound if well defined, and also demonstrate low signal on T1 and T2 sequences; however, they would not demonstrate susceptibility artefact. Fibromas can look similar but are not associated with susceptibility artefact either. Ganglion cysts are cystic rather than solid lesions and would not demonstrate internal vascularity. Glomus tumours are vascular tumours also known as glomangiomas, and present as painful small blue/red nodules under the fingernails. Due to their small size they are difficult to see on radiograph and there may just be mild soft tissue swelling. The lesions will be hypervascular if visible on ultrasound.
1445
A 70 year old woman undergoes a CT colonography which identifies an incidental pancreatic head mass. This measures 1.5 cm and has a HU density of 10. It contains multiple tiny cysts with a central area of calcification. The pancreatic duct measures 2 mm in diameter. What is the most likely diagnosis? a. Serous cystadenoma b. Main duct intraductal papillary mucinous neoplasm c. Mucinous cystadenoma d. Pancreatic adenocarcinoma e. Islet cell tumour
a) Serous cystadenoma The mass fits well with being a serous cystadenoma; it is in the pancreatic head, there are multiple small cysts, calcification is central and the patient is an elderly woman. A mucinous cystadenoma is more common in middle-aged women and the cysts are less numerous and larger. Calcification is peripheral and the majority of masses are in the pancreatic body or tail. Main duct intraductal papillary mucinous neoplasms (IPMNs) do occur in a more elderly age group and more commonly in the pancreatic head; however, they usually cause pancreatic duct dilatation (>3 mm). Calcification is not a typical feature. Islet cell tumours are neuroendocrine tumours, and when small are usually homogenous solid lesions rather than the cystic mass described in this question. The most common islet cell tumour is an insulinoma followed by gastrinoma. They exhibit arterial phase enhancement and so usually appear isodense to the pancreas on portal venous phase CT.
1446
Hydrops fetalis is identified on antenatal scans of a male fetus in the second trimester of pregnancy. Following close observation of the mother into the third trimester, the baby boy is delivered early by emergency caesarean section. The baby suffers with respiratory distress and is transferred immediately to the neonatal intensive care unit where a chest radiograph is obtained. The lungs appear clear; however there are bilateral pleural effusions. Drainage of the pleural fluid reveals chylous effusions. What is the underlying cause likely to be? a. Extralobar sequestration b. Lymphangioleiomyomatosis c. Pulmonary lymphangiectasia d. Thoracic duct atresia e. Turner syndrome
(d) Thoracic duct atresia Chylothorax is most commonly due to an iatrogenic injury of the thoracic duct, for example following thoracic surgery such as oesophagectomy. Chylothoraces can also be caused by traumatic injury or by malignancy. In a neonate these causes are less likely, although birth trauma is a possibility. The most likely causes are thoracic duct atresia, lymphangiectasia and pulmonary abnormalities such as congenital pulmonary airway malformation and extralobar sequestration. Certain congenital conditions such as Turner, Noonan and Down syndrome are also associated with chylothoraces. In this case, the child is male so this excludes Turner syndrome. The normal appearance of the lungs excludes lymphangiectasia and extralobar sequestration. The former causes dilated lymphatics, which leads to other radiological findings such as interstitial thickening and perihilar infiltrates. Lymphangioleiomyomatosis is not a diagnosis made in this age group, as it usually affects young women. Therefore the most likely cause in this patient is thoracic duct atresia.
1447
A 40 year old man with a background of previous renal cell carcinoma presents with nausea and headaches. On clinical examination he is noted to have papilloedema. An MRI brain shows a large cystic mass in the posterior fossa which is obstructing the fourth ventricle causing hydrocephalus. The mass also demonstrates a small enhancing nodule on the T1 post contrast sequence and serpiginous low T2 signal areas at the periphery. What is the most likely diagnosis? a. Cavernoma b. Ependymoma c. Haemangioblastoma d. Metastasis e. Pilocytic astrocytoma
(c) Haemangioblastoma The most common cause of a posterior fossa mass in adults is a metastasis; however the second most common is a haemangioblastoma, which fits the description in this case. The history of a relatively young man with a history of renal cell carcinoma is also suggestive of possible von Hippel-Lindau syndrome. On imaging, haemangioblastomas are most commonly located in the posterior fossa. They are well-defined cystic masses with an enhancing mural nodule. They may demonstrate flow voids, particularly at the periphery of the lesion. Pilocytic astrocytomas can have a similar appearance with a cystic mass and enhancing mural nodule; however, they usually occur in a much younger age group. A cavernoma is not a cystic mass and on MRI can have variable appearances depending on the presence of haemorrhage and haemosiderin. This causes blooming on susceptibility weighted imaging. Ependymomas often contain cystic components; however they also contain significant solid elements, calcification and haemorrhage
1448
A 57 year old patient presents to hospital with right flank pain and haematuria. The team request an unenhanced CT urinary tract as they are concerned about urinary tract calculi. The CT report describes a 6.2-cm exophytic soft tissue mass arising from the right kidney with perinephric fat stranding and a couple of 10-mm retrocaval and aortocaval nodes at the same level. The adrenal glands and other solid abdominal viscera have normal appearances. There is marked expansion of the right renal vein. What is the most appropriate test to confirm whether this is tumour thrombus or bland thrombus? a. Contrast enhanced MRI b. Digital subtraction angiography c. 18F-FDG PET/CT d. Portal venous phase CT abdomen pelvis e. Ultrasound Doppler right kidney
(a) Contrast enhanced MRI Renal cell carcinoma extending into the renal vein corresponds to at least T3 disease, with proximal extension an important factor in surgical planning. If it extends into the inferior vena cava, then a midline laparotomy is required, and if beyond the level of the hepatic veins, then thoracic surgeons will likely need to be involved. It can be difficult to assess tumour thrombus versus bland thrombus. Sometimes contrast enhancement is identified on post-contrast CT within the thrombus, consistent with tumour; however, MRI is superior to CT for assessment. Tumour thrombus on MRI would also enhance and on T1 weighted imaging the normal vascular flow voids would be replaced by comparatively high tumour signal. 18F-FDG PET/CT currently has a limited role in the staging of renal cell carcinoma due to renal excretion causing high renal tracer uptake and therefore making assessment difficult. However, there are reports of tracer uptake in tumour thrombi. Ultrasound Doppler may identify thrombus; however the assessment of bland thrombus versus tumour thrombus would be challenging on ultrasound. Digital subtraction angiography does not have a role in this case.
1449
A CT and MRI have been performed for an ear, nose and throat clinic patient with chronic sinus symptoms. The MRI confirms a mass centred on the right maxillary sinus. This is low signal on T1 and high signal on T2 sequences. There is peripheral enhancement. On CT the mass is slightly hyperdense and no air is visible within the sinus. The bony walls of the sinus are thinned. The mass passes through a widened maxillary ostium into the nasopharynx. What does this mass most likely represent? a. Antrochoanal polyp b. Esthesioneuroblastoma c. Inverting papilloma d. Mucocele e. Mucous retention cyst
(a) Antrochoanal polyp Antrochoanal polyps are benign lesions occurring in the maxillary sinus which fill the antrum and extend into the nasopharynx via an enlarged ostium. Typically they are low signal on T1 and high signal on T2 sequences with peripheral enhancement. Mucoceles may have similar MRI signal characteristics but they do not pass into the ipsilateral nasopharynx. Another differential is a mucous retention cyst. These are more likely when there is air visible in the sinus, whereas this finding would exclude a mucocele. Mucous retention cysts are also not associated with bony expansion. An inverting papilloma originates in the nose but can extend into the ipsilateral paranasal sinuses. The signal tends to be more isointense on T1 and T2 sequences. An esthesioneuroblastoma also arises from the nose, and these masses are more likely to affect the ethmoid air cells. They can invade the anterior cranial fossa and typically demonstrate marked contrast enhancement rather than the peripheral enhancement described in this case.
1450
A 34 year old patient suffers a high speed road traffic collision and is taken to the nearest trauma centre. A CT traumogram is performed which demonstrates, amongst other serious findings, the presence of pneumomediastinum, right pneumothorax and sagging of the right lung. Where is the most likely site of injury to cause these findings? a. Diaphragmatic rupture b. Fracture of the right first rib c. Right main bronchus at the site of bifurcation of the upper lobe bronchus and bronchus intermedius d. Right main bronchus near the carina e. Tracheal rupture
(d) Right main bronchus near the carina The description of the sagging right lung indicates that there is an underlying tracheobronchial injury. This injury is associated with high mortality. It usually occurs close to the carina which is fixed relative to the bronchi. Imaging features include presence of a pneumothorax which is refractory to drainage, as well as the presence of the ‘fallen lung’ sign, in which the lung sags towards the floor of the hemithorax, away from the hilum. There may also be a fractured first rib, indicating high impact force; however this is not the cause of pneumothorax in the case vignette. Diaphragm rupture can occur in cases of trauma and may be associated with complications such as herniation of abdominal viscera.
1451
A 32 year old has plain films of the right wrist to investigate ongoing pain. There is a well- defined lucent lesion in the distal radius, which appears lytic and mildly expansile with no matrix mineralisation, periosteal reaction or soft tissue swelling. Which of the following lucent bone lesions are most likely to have a non-sclerotic rim? a. Brown tumour b. Chondroblastoma c. Epidermoid cyst d. Fibrous dysplasia e. Fibrous cortical defect
(a) Brown tumour The features are in keeping with a Brown tumour, also known as an osteoclastoma. These are solitary lesions seen in primary and secondary hyperparathyroidism. They can mimic metastases and myeloma.
1452
A 22 year old woman is investigated for recurrent episodes of hypoglycaemia. Blood tests confirm endogenous insulin hypersecretion. A CT pancreas is performed. Which of the following is the most likely imaging finding on arterial phase imaging? a. Hypoenhancing 3-cm nodule in the tail of pancreas b. Hyperenhancing 5-cm nodule in the body of the pancreas c. Hypoenhancing 2-cm nodule in the head of pancreas d. Hyperenhancing 1-cm nodule in the tail of pancreas e. Hypoenhancing 4-cm nodule in the head of pancreas
(d) Hyperenhancing 1-cm nodule in the tail of pancreas Endocrine tumours of the pancreas, also known as islet cell tumours, are a type of neuroendocrine tumours. The most common type is an insulinoma followed by a gastrinoma, but other types include glucagonoma and VIPoma. Insulinomas tend to present when small due to the episodes of hypoglycaemia they induce. They are usually avidly arterially enhancing solid masses but they can be necrotic when large. They may also contain calcification.
1453
A GP requested a CT chest, abdomen and pelvis for a 58 year old post-menopausal female patient with type 2 diabetes and unintentional weight loss. The CT is performed without contrast due to mildly reduced renal function. There is mediastinal, hilar and upper abdominal lymph node enlargement which is noted to be relatively low attenuation compared to adjacent soft tissues. Regarding the causes of low attenuation lymph nodes, which one of these is NOT associated with this radiological finding? a. Coeliac disease b. Kaposi sarcoma c. Systemic lupus erythematosus d. Tuberculosis e. Whipple disease
(b) Kaposi sarcoma Low attenuation lymph nodes can be due to cystic or fatty components. Causes of cystic components include metastatic malignancy, typically squamous cell carcinoma, and sometimes lymphoma is associated with this. Tuberculosis and coeliac disease can also cause this appearance. Fat is deposited in lymph nodes in Whipple disease. It has also been described in systemic lupus erythematosus. High attenuation lymph nodes have been described in conditions such as Kaposi sarcoma, Castleman disease and carcinoid.
1454
An 11 year old girl is referred to the paediatric neurologist with refractory temporal lobe epilepsy. An MRI brain is performed. This shows a partially cystic mass in the left temporal lobe. The solid component is intermediate T1 signal and T2 hyperintense with heterogenous enhancement. There is no surrounding oedema and no dural tail sign. The mass demonstrates blooming artefact on T2*. What is the most likely diagnosis? a. Pilocytic astrocytoma b. Dysembryoplastic neuroepithelial tumour c. Ganglioglioma d. Oligodendroglioma e. Pleomorphic xanthoastrocytoma
(c) Ganglioglioma Gangliogliomas most commonly occur in the temporal lobes causing seizures in children and young adults. They can have variable appearances with both solid and cystic elements. They frequently demonstrate calcification which causes blooming on susceptibility weighted imaging. Oligodendrogliomas can have similar appearances to ganglioglioma and present with seizures; however they are uncommon in children. The other options are all likely in the temporal lobes, causing little if any adjacent oedema or seizures as a common presentation. Dysembryoplastic neuroepithelial tumours (DNETs) usually do not enhance much, if at all. Pleomorphic xanthoastrocytomas are rare tumours. Calcification is not a feature and although intra-axial, commonly demonstrate a dural tail sign. Pilocytic astrocytomas are not common in the temporal lobes and usually affect the cerebellum or optic pathways. They are also largely cystic with an enhancing solid nodule.
1455
A 35 year old male patient is diagnosed with asymptomatic hypertension following a medical examination at his place of work. Following hospital referral and further investigations he is found to have an aortic coarctation. Chest radiograph reveals unilateral inferior rib notching of the left third to eighth ribs. What is the most likely cause of this appearance? a. Anomalous origin of the left subclavian artery b. Anomalous origin of the right subclavian artery c. Right sided aortic arch with anomalous left subclavian artery d. Stenosed left subclavian artery e. Stenosed left costocervical trunk
(b) Anomalous origin of the right subclavian artery Inferior rib notching in aortic coarctation is caused by dilated intercostal vessels which act as collateral supply bypassing blood to the thoracic aorta distal to the coarctation. Unilateral left inferior rib notching is seen in aortic coarctation when there is an anomalous origin of the right subclavian artery distal to the coarctation. Anomalous origin of the left subclavian artery, a right-sided aortic arch with an anomalous left subclavian artery and a stenosed left subclavian artery are all potential causes of unilateral right inferior rib notching. The costocervical trunks are important anatomically as they arise directly from the subclavian arteries on both sides and supply the first and second intercostal arteries. The other intercostal arteries arise directly from the thoracic aorta, hence there is sparing of the first and second ribs from rib notching with aortic coarctation.
1456
A 38 year old female patient with no significant medical history apart from a high body mass index presents to hospital with abdominal pain. Bloods demonstrate slightly raised inflammatory markers. Urine human chorionic gonadotrophin test is negative. The solid abdominal viscera have normal size and appearances on ultrasound and there is no free fluid in the abdomen or pelvis. At the site of maximal tenderness there is an ovoid hyperechoic mass without increased vascularity. It is not compressible. A CT scan is performed to characterise this further. The mass is adjacent to the colon, peripherally enhances, has a density of −90 HU and there is adjacent fat stranding. Where is the most common location for this pathology? a. Anterior to the ascending colon b. Right iliac fossa c. Anterior to the sigmoid colon d. Adjacent to the transverse colon e. Adjacent to the umbilicus
(c) Anterior to the sigmoid colon The imaging characteristics are typical for epiploic appendagitis. This usually affects women more than men and is more common in patients with a raised body mass index. Differentials would include appendicitis and diverticulitis; however the patient is young and diverticulitis would be less likely. The imaging appearances are not characteristic for appendicitis. The epiploic appendages are small fatty appendages sited along the large bowel. When these twist they cause acute abdominal pain which can be difficult to clinically differentiate from other causes of pain. This most commonly occurs anterior to the rectosigmoid colon. On ultrasound the appearance of a hyperechoic mass indicates fat. Sometimes a slightly hypoechoic line can be seen peripherally, and there is no internal vascularity. CT is usually diagnostic and demonstrates a lesion of fat density adjacent to the colon with peripheral enhancement and surrounding fat stranding. Sometimes a hyperechoic dot centrally can be seen representing thrombosed vessels.
1457
A 52 year old man taking regular aspirin has noticed a change when examining the right side of his scrotum. He is referred by his GP for a scrotal ultrasound. The testicles are normal in size and homogenous in echotexture apart from a couple of tiny hyperechoic foci in the right testicle. Vascularity is symmetrical. The epididymides have normal appearances with no increased vascularity. The right pampiniform plexus is prominent, measuring up to 4 mm in several locations with flow reversal following Valsalva manoeuvre. The left pampiniform plexus has normal appearances. Which of the following is most appropriate? a. Abdominal ultrasound b. Repeat ultrasound in 12 months c. Suggest serum alpha-fetoprotein and human chorionic gonadotrophin levels d. Suggest referral for embolisation e. Suggest referral for surgical ligation
a) Abdominal ultrasound The patient has a right-sided varicocele. Varicoceles are more common on the left due to the drainage of the left testicular vein into the left renal vein. The right testicular vein drains directly into the lower pressure inferior vena cava. If a varicocele is just on the right side, then ultrasound of the abdomen, to focus on the retroperitoneal structures, should be performed to ensure there is not an underlying pathology, such as a mass, impeding venous return. If ultrasound cannot be confidently used to assess this, for example in patients with a high body mass index, then other modalities could be considered. Treatment for uncomplicated varicoceles include surgical ligation or embolisation. There are a couple of tiny hyperechoic foci in the right testicle of unlikely clinical significance; when >5 this is consistent with microlithiasis. Guidance from the European Society of Urogenital Radiology suggests annual follow-up until the age of 55 only if there are other risk factors present, such as a history of orchidopexy or maldescent. The presence of a testicular mass would prompt tumour marking testing but is not indicated in this case.
1458
A 67 year old neurology outpatient with cerebellar ataxia has an MRI brain. This reveals cerebellar atrophy and symmetrical high T2 signal in the pons and cerebellum. There is low T2 signal in the putamen with a rim of peripheral higher signal. The ventricles have normal appearances. There is mild generalised volume loss without lobar predilection. Which of these conditions are these radiological findings most consistent with? a. Huntington disease b. Lewy body dementia c. Multisystem atrophy d. Pick disease e. Progressive supranuclear palsy
c) Multisystem atrophy There can be overlap between the radiological appearances in neurodegenerative disease. Multisystem atrophy is classically associated with the ‘hot cross bun’ sign due to pontine T2 hyperintensity. Hyperintensity can also be seen in the cerebellum and cerebellar peduncles. Typically there is low signal in the basal ganglia, specifically the putamen, with a peripheral rim of T2 hyperintensity. Atrophy of the pons, cerebellum and midbrain is also reported. Progressive supranuclear palsy is associated with midbrain atrophy causing flattening of the superior midbrain leading to the characteristic ‘hummingbird’ appearance. T2 hyperintensities in the pons, midbrain and inferior olivary nucleus, as well as cisternal and ventricular dilatation, may be evident. Huntington disease is characterised by enlargement of the frontal horns of the lateral ventricles due to atrophy of the caudate nuclei. The basal ganglia may become T2 hypointense due to iron deposition. Features of Lewy body dementia include generalised volume loss with enlargement of the lateral ventricles. Hippocampal atrophy, which is frequently associated with Alzheimer disease, is not a feature. Pick disease classically has more focal volume loss which can be asymmetrical and affect the temporal and frontal lobes.
1459
A 24 year old female is investigated for ongoing malaise, chest pain on exertion and shortness of breath. Chest radiograph is unremarkable. She has a ventilation-perfusion (VQ) nuclear medicine scan which excludes a pulmonary embolus. She is subsequently referred to the cardiology team and on examination they discover that she has an absent left radial pulse. The team organise a CT angiogram of the chest. This shows an abnormal appearance of the aortic arch and proximal descending aorta, which is reduced in calibre and thick walled. Which of the following are most likely to seen in a patient with a diagnosis of Takayasu arteritis in the healed fibrotic phase? a. Delayed enhancement of the aortic wall b. Irregular contour of the descending aorta c. Linear calcification of the ascending aorta d. Linear calcification of the descending aorta e. Narrowing of the peripheral arteries
(b) Irregular contour of the descending aorta Takayasu arteritis is a vasculitis of large arteries, affecting predominantly the aorta and major branches. It can also affect the pulmonary arteries (in approximately 15%). It is divided into two phases, acute inflammatory phase and healed fibrotic phase, which have differing CT appearances. In the acute phase the aorta wall is thickened and demonstrates delayed enhancement (on imaging performed approximately 20 minutes after contrast injection). The most common feature in healed fibrotic phase is an irregular contour of the descending aorta. Absent enhancement of a thickened wall can also be seen as well as calcification of the arch and descending aorta; however these are less frequent. Whilst Takayasu arteritis has a predilection for the aortic arch and descending aorta, calcification of the ascending aorta may be seen as sequelae to syphilitic aortitis.
1460
A 65 year old patient has an enhanced CT abdomen pelvis at the request of the urology team to monitor recurrent renal calculi. There are bilateral renal calculi which have not significantly changed in size or position compared to the previous symptomatic study and the kidneys are unobstructed. You notice mildly increased density in the fat around the root of the jejunal mesentery, which has a hazy appearance. This has marginally increased compared to the previous CT. There are a couple of small sub-centimetre lymph nodes in this region. The urology team confirm that the patient is currently asymptomatic. Which other radiological finding would be consistent with mesenteric panniculitis? a. Displacement of adjacent vessels and bowel loops b. Low attenuation halo surrounding vessels c. Low attenuation lymph nodes d. Nodular shrunken liver e. Thickening of the adjacent bowel wall
b) Low attenuation halo surrounding vessels Mesenteric panniculitis is a chronic inflammatory condition of the small bowel mesentery, often affecting the jejunal root. It is described as haziness of the mesenteric fat which is not associated with adjacent bowel wall involvement and does not cause displacement of adjacent structures. A low attenuation perivascular halo is frequently described. Mesenteric oedema causes diffuse hyperdensity of the mesentery which can often make visualisation of the vessels difficult. One cause of this would be liver cirrhosis, which would be associated with a shrunken, nodular liver. Other causes include hypoalbuminaemia and cardiac failure. Low attenuation lymph nodes are not associated with mesenteric panniculitis but are seen in conditions such as tuberculosis, Whipple disease and coeliac disease.
1461
A 49 year old male patient is diagnosed with T2b N0 M0 right renal cell carcinoma of clear cell subtype. Also noted on his staging CT are two small left renal angiomyolipomas and several small pancreatic cysts. The patient reports a history of previous brain tumour although there is no imaging of his head available on your PACs system for review. What is the most likely underlying hereditary condition? a. Burt-Hogg-Dubé b. Gorlin-Goltz c. Osler-Weber-Rendu d. Tuberous sclerosis e. Von Hippel-Lindau
(e) Von Hippel-Lindau Von Hippel-Lindau is an autosomal dominant inherited condition which can exhibit multisystem involvement. The intracranial tumour described by the patient in the question likely represents a haemangioblastoma; however, choroid plexus papillomas are also possible. Patients can have cysts in the liver, pancreas and kidneys as well as renal angiomyolipoma, phaechromocytomas and pancreatic neuroendocrine tumours. This group of patients is at 217 THE FINAL FRCR increased risk of renal cell carcinoma, which can be bilateral and is typically of clear cell subtype. Tuberous sclerosis can cause angiomyolipomas. It carries an increased risk of renal cell carcinoma and is associated with intracranial tumours such as subependymal giant cell astrocytomas as well as other parenchymal brain abnormalities. However, the other features in the question are not typical. Pulmonary cysts, sclerotic bone lesions and cardiac rhabdomyomas are also encountered. Osler-Weber-Rendu is also known as hereditary haemorrhagic telangiectasia and is characterised by multiple arteriovenous malformations. Birt-Hogg-Dubé is associated with renal tumours such as renal cell carcinomas and oncocytomas as well as pulmonary cysts and cutaneous manifestations; however, the other lesions mentioned in the question are not consistent with this diagnosis. Gorlin-Goltz is characterised by multiple basal cell carcinomas and musculoskeletal and craniofacial abnormalities. Intracranial tumours are possible but the other features are not described.
1462
A 10 year old girl has a systolic murmur. A chest radiograph is performed which shows pulmonary plethora. Which of the following radiological signs will help differentiate between a diagnosis of patent ductus arteriosus versus ventricular septal defect? a. Enlarged aorta b. Enlarged left atrium c. Enlarged left ventricle d. Enlarged pulmonary vasculature e. Normal right atrium
a) Enlarged aorta With a patent ductus arteriosus, a left-to-right shunt occurs as high-pressure blood passes from the aorta to the pulmonary circulation via the patent ductus. The ductus arteriosus usually closes within 48 hours of birth. If this remains open, then eventually the left atrium and ventricle can become enlarged and the aorta dilated; however the right heart is unaffected. With a ventricular septal defect the left atrium and both ventricles may enlarge; however, the aorta remains normal in size.
1463
A 30 year old woman has been experiencing vague neurological symptoms including left- sided headaches and facial paraesthesia. An MRI brain reveals subtle loss of grey−white matter differentiation in the left temporal and parietal lobes with minor, diffuse gyral expansion. The affected area is extensive and involves the ipsilateral basal ganglia structures; it is T1 hypointense and T2 hyperintense to grey matter. There is moderate effacement of the left lateral ventricle. There is no enhancement following gadolinium administration and the affected area does not restrict on diffusion weighted imaging. What is the most likely clinical diagnosis? a. Encephalitis b. Gliomatosis cerebri c. Primary CNS lymphoma d. Progressive multifocal leukoencephalopathy e. Tumefactive demyelination
(b) Gliomatosis cerebri Gliomatosis cerebri is a diffuse infiltrative parenchymal process involving two or more lobes of the brain. The peak onset is 30–40 years of age. It affects large portions of the brain with relatively little mass effect considering the extent of involvement. Due to its isodense appearance relative to adjacent brain, it can be difficult to define on CT. On MRI there is often T1 hypointensity and T2 hyperintensity within the white matter and gyral thickening. There is limited enhancement and restricted diffusion. The history is not suggestive of progressive multifocal leukoencephalopathy (PML) which is encountered in the setting of immunosuppression, for example HIV and AIDS. There is also no mass effect in PML. Primary CNS lymphoma would usually demonstrate avid, uniform enhancement. Tumefactive demyelinating lesions also typically enhance with incomplete ring enhancement. Encephalitis has a different clinical presentation. The most common type is due to herpes simplex infection usually causing bilateral temporal lobe abnormalities.
1464
A 55 year old female smoker has an accident at work and comes to the emergency department with wrist pain. A radiograph demonstrates no fracture but there is an abnormality in the visualised hand; therefore formal views of the hands are also obtained. These demonstrate soft tissue swelling, cortical thickening and periosteal reaction at the metacarpals and proximal phalanges bilaterally. Joint spaces are well-preserved and no focal bone lesions are identified. The reviewing doctor reports that there is no pain in the hands. What other imaging finding would most appropriately correlate with these appearances? a. Evidence of scalp skin fold thickening on CT head b. Lung mass on CT chest c. Symmetrical enlargement of the inferior rectus muscles on MRI orbits d. Symmetrical enlargement of the lateral rectus muscles on MRI orbits e. Jejunoileal fold pattern reversal on small bowel follow through
c) Symmetrical enlargement of the inferior rectus muscles on MRI orbits The appearance of painless symmetrical periosteal reaction in the tubular bones, particularly of the hands and feet, is typical for thyroid acropachy. The thyroid itself does not have to be abnormal, as patients can be euthyroid or post-treatment. The condition is almost always associated with thyroid eye disease, which manifests as an increase in orbital fat and bilateral symmetrical enlargement of the extraocular muscles. There is a characteristic pattern of involvement, with the mnemonic ‘I’M SLOW’ being a helpful way to remember it: Inferior rectus Medial rectus Superior rectus Lateral rectus Obliques Another cause for periosteal reaction of the long bones is hypertrophic osteoarthropathy, but this tends to be painful. Secondary causes of hypertrophic osteoarthropathy include lung cancer, bronchiectasis and mesothelioma as well as non-pulmonary causes such as inflammatory bowel disease and coeliac disease. The latter causes reversal of the normal jejunoileal fold pattern with increased ileal folds and reduced jejunal folds. Primary hypertrophic osteoarthropathy, or pachydermoperiostosis, is most common in young black men and typically has skin changes with skin fold thickening, often on the scalp. The patient demographics do not fit with this diagnosis.
1465
A 42 year old patient has an ultrasound abdomen requested by his GP for non-specific lower abdominal discomfort. This partially visualises a hypoechoic mass in the pelvis with increased vascularity. Contrast enhanced CT abdomen and pelvis is performed which identifies the mass in the right side of the pelvis. It measures up to 50 mm, is ill defined and inseparable from the pelvic side wall musculature. There is a clear fat plane between the mass and the urinary bladder and the right external iliac vessels. The mass has areas of enhancing soft tissue as well as cystic components and regions measuring −20HU in density. This is within 20 mm of the anterior abdominal wall. There is no ascites. The kidneys are unobstructed. There are a couple of 8-mm right common iliac lymph nodes. What is the most appropriate next step in managing this patient? a. CT chest with contrast b. CT guided biopsy of pelvic mass c. Refer to tertiary sarcoma centre for multidisciplinary team discussion prior to biopsy d. Ultrasound guided biopsy of pelvic mass e. 18F-FDG PET/CT
(a) CT chest with contrast The imaging features are suspicious for a soft tissue sarcoma. The most appropriate next step is a CT chest to assess for any evidence of disease elsewhere. An MRI pelvis may also be helpful to more accurately delineate the extent of the mass. Discussion at a tertiary sarcoma centre prior to biopsy is important; however imaging should be completed prior to this referral. There are frequently concerns regarding tumour seeding with sarcomatous lesions, and sometimes sarcoma centres will request to biopsy the lesion themselves. Identifying the most appropriate biopsy site may also be aided by completing imaging, as a more superficial lymph node could be more easily accessible than an intrapelvic or abdominal mass. 18F-FDG PET/CT may be helpful but would not be clearly indicated at this point.
1466
A paediatric junior doctor calls you for advice over the weekend. They have seen a 5 month old girl who has presented to hospital with her parents. Urine dip confirms urinary tract infection. A urine sample taken at the GP surgery by the practice nurse the previous day has grown Escherichia coli. The paediatric team are prescribing a course of oral antibiotics with a plan to send home with a clinic review in 48 hours. The junior doctor enquires about the most appropriate imaging. According to NICE guidelines, what would the most appropriate recommendation be? a. Dimercaptosuccinic acid test b. Immediate ultrasound urinary tract c. Micturating cystourethrogram within 4 weeks d. No imaging required unless recurrent urinary tract infection e. Ultrasound urinary tract within 6 weeks
(e) Ultrasound urinary tract within 6 weeks NICE guidelines state that for children less than 6 months old an ultrasound within 6 weeks of a lower urinary tract infection (LUTI) is appropriate. If there are atypical features, such as raised creatinine, poor urine flow, failure to respond to treatment with 48 hours, sepsis or non-Escherichia coli bacteria, this would necessitate an immediate ultrasound. This child is being prescribed oral treatment with close outpatient follow-up and therefore atypical infection is unlikely. For children older than 6 months the recommendation is for immediate ultrasound if there are features of atypical LUTI and dimercaptosuccinic acid (DMSA) test within 4–6 months. Otherwise, imaging is not routinely indicated for a single episode of LUTI unless infections are recurrent (≥3 LUTI episodes) when an ultrasound within 6 weeks and DMSA test within 4–6 months is indicated. Micturating cystourethrograms are only advised if there is a family history of vesicoureteral reflux, a non-E. coli infection, poor urine flow or urinary tract dilatation on ultrasound.
1467
A patient is diagnosed with acute Hashimoto thyroiditis following clinical, serological, ultrasound and nuclear medicine assessment in a one-stop thyroid clinic. Which of the following profiles fits this diagnosis most accurately?
(b) Euthyroid, heterogenous hypoechoic gland, avascular, reduced tracer uptake Hashimoto thyroiditis can have variable ultrasound appearances but there is reduced tracer uptake on pertechnetate scans. The gland is usually enlarged and hypoechoic with reduced vascularity. Patients are commonly euthyroid or hypothyroid. There is an increased risk of non-Hodgkin’s lymphoma and therefore follow-up and biopsy of any focal nodularity is recommended. Chronic changes include a shrunken heterogenous gland. Graves disease is associated with hyperthyroidism and would be consistent with answer D in this case. Patients may present with extrathyroid signs such as ophthalmopathy. Ultrasound appearances are typically of an enlarged hyperechoic gland with hypervascularity and on nuclear medicine studies there is increased tracer activity.
1468
A 17 year old with a known chromosomal abnormality has an MRI of the whole spine which demonstrates a scoliosis, posterior vertebral body scalloping, enlarged neural foramina and dural ectasia as well as several well-defined dumbbell-shaped extra-axial lesions within the nerve exit foramina at multiple levels. All lesions demonstrate low signal on T1 pre-contrast, intermediate signal on T2 and there is T1 post contrast enhancement. A similar intra-axial lesion is seen within the canal at T12. You review the patient’s previous imaging. What other musculoskeletal finding is the patient most likely to have? a. Anterior vertebral body beaking b. Hypertrophy of the vertebral posterior elements c. Inferior rib notching d. Sclerotic bone lesions e. Superior rib notching
(c) Inferior rib notching The chromosomal abnormality described is neurofibromatosis type 1. The well-defined intra- and extra-axial lesions are neurofibromas. The description of enlarged neural foramina is secondary to the presence of neurofibromas. Lateral meningocoeles may also be present and can cause this finding. The posterior elements may be hypoplastic, also due to the presence of neurofibromas/meningocoeles. Inferior rib notching is more likely to occur for the same reason; superior rib notching is possible if the neurofibroma is large. Anterior vertebral beaking is a feature of mucopolysaccharidoses, Down syndrome and other conditions. Multiple non-ossifying fibromas may also be seen in neurofibromatosis type 1, which are typically lucent lesions with sclerotic margins. Other musculoskeletal manifestations include pseudarthrosis of the wrist, tibia, fibula and clavicle as well as other bony dysplasias.
1469
A neonate has a contrast enema after not passing meconium at 48 hours following delivery. Contrast passes quickly into the rectum and colon. The proximal large bowel is mildly dilated with a transition point to a narrower calibre in the sigmoid colon. The small bowel is normal calibre. The rectosigmoid ratio is 0.8. Which of the following would help diagnose the underlying condition causing these findings? a. Antenatal history of oligohydramnios b. Confirmed maternal history of diabetes c. Newborn heel prick test d. Rectal biopsy e. Sweat chloride test
d) Rectal biopsy The case describes a patient with Hirschsprung disease. There is a transition point, most commonly at the rectosigmoid colon, where the bowel transitions to a narrowed aganglionic segment. A longer segment can also be affected, with a transition at the splenic flexure, or less commonly the entire colon may be involved. There is frequently dilatation of the bowel proximal to this transition point. There may be a ‘sawtooth’ appearance of the narrowed segment. The rectosigmoid ratio is an important factor, as normally the rectum is larger than the sigmoid colon and should be >1. A ratio <1 is indicative of Hirschsprung disease and this is confirmed with rectal biopsy. If Hirschsprung disease was associated with an abnormal antenatal scan, the abnormality would lead to polyhydramnios rather than oligohydramnios. A sweat chloride test and the routine neonatal heel prick test could help in the detection of cystic fibrosis, which is commonly associated with meconium ileus. A maternal history of diabetes is associated with meconium plug syndrome. (The Final FRCR Complete
1470
A newborn undergoes a chest radiograph which shows a right sided aortic arch. What underlying condition are they most likely to have? a. Ebstein anomaly b. Tetralogy of Fallot c. Transposition of the great arteries d. Tricuspid atresia e. Truncus arteriosus
b) Tetralogy of Fallot A right-sided aortic arch with mirror imaging branching is the most common subtype of a right-sided aortic arch and is nearly always associated with congenital heart disease. Of these, 90% are associated with tetralogy of Fallot. Therefore other radiological findings to look for are a ‘boot-shaped’ heart and pulmonary oligaemia. A right-sided aortic arch with an aberrant left subclavian artery is the second most common subtype, and the persistent ductus ligament can cause tracheal compression. A Kommerell diverticulum is also a feature, which manifests as dilatation of the aberrant left subclavian artery at the right aortic arch origin.
1471
A 44 year old patient with a history of excess alcohol intake is admitted with acute severe pancreatitis. Initial chest radiograph shows bibasal atelectasis and a small left pleural effusion. Within 24 hours of admission the patient is transferred to intensive care. Following 48 hours on the intensive care unit the team report increased oxygen demand despite optimal therapy. CT scan demonstrates bilateral posterior consolidation with ground glass opacification more anteriorly. There is a small left pleural effusion. Appearances of the pancreas correlate with the clinical diagnosis of pancreatitis. What is the most likely explanation for the CT chest findings? a. Adult respiratory distress syndrome b. Aspiration pneumonia c. Cardiac failure d. Fat embolism e. Hospital-acquired pneumonia
(a) Acute respiratory distress syndrome Acute respiratory distress syndrome (ARDS) can be split into pulmonary causes (e.g. toxic inhalation, drowning, lung contusion, pneumonia and fat embolus) and extrapulmonary causes (e.g. burns, sepsis, blood transfusion and pancreatitis). This case describes an extrapulmonary cause. The features of symmetrical consolidation with a gradient from posterior to anterior with more ground glass changes anteriorly is typical. Pulmonary causes tend to lead to asymmetrical changes without a gradient. A hospital-acquired pneumonia is possible, although the patient has only had a brief admission so far. An aspiration pneumonia is also possible, especially if there has been vomiting with recent alcohol excess, but in an intensive care patient with pancreatitis and such typical features of ARDS, this is less likely. In cardiac failure, other features such as cardiomegaly, prominent vascular markings, interstitial lines and pleural effusions would be more common. This patient does have a small left pleural effusion but this is likely secondary to the pancreatitis. One of the causes of fat embolism is pancreatitis, and features can resemble ARDS. Typical appearances of fat embolism include geographic ground glass opacities rather than the gradient seen in ARDS. Interlobular septal thickening and nodularity are also features, which are less common in ARDS.
1472
An active 38 year old man has repeated GP visits for right hip pain which is stopping him from playing squash and does not improve despite several weeks of physiotherapy. A pelvic radiograph shows preserved joint space of both hips without significant degenerative change. There is relative reduction in the subchondral cortical thickness on the right and reduced bone density of the right proximal femur compared to the left. There is no periosteal reaction or soft tissue mass. An MRI scan reveals a joint effusion and low T1 and high T2 signal in the right femoral head which is particularly marked in the subchondral region with corresponding early post-contrast enhancement. What is the most likely diagnosis? a. Avascular necrosis of the proximal femur b. Complex regional pain syndrome c. Idiopathic transient osteoporosis of the hip d. Septic arthritis e. Stress fracture of the proximal femur
(c) Idiopathic transient osteoporosis of the hip This tends to be unilateral, affecting males more than females, although it is described in late pregnancy. The history of hip pain is long, and the classic radiological description is of osteopenia and subchondral cortical loss. An MRI will demonstrate bone marrow oedema, centred on the subchondral region, and early post-contrast enhancement of the abnormal marrow. The history and presentation is not characteristic for septic arthritis and there is no indication that the patient is unwell, as this has been managed as an outpatient. In avascular necrosis the early post-contrast enhancement would not be typical. Although bone marrow oedema is a feature, other signs such as subchondral linear low signal and the ‘double line’ sign are usually present. A stress fracture in an active young patient is possible but in the proximal femur this tends to occur at the femoral neck rather than at the femoral head. A cortical breach and a linear low T1 signal fracture line would also be expected. Complex regional pain syndrome commonly occurs following trauma. Due to sympathetic dysfunction, other symptoms such as alteration to skin blood flow, hyperalgesia and oedema are often described. This condition typically affects the extremities rather than a large joint such as the hip. There is some overlap in radiographic features, with joint space preservation and osteopenia being typical.
1473
A 37 year old female patient with poorly controlled type 1 diabetes has an MRI liver requested. During an inpatient admission for cholecystitis an ultrasound abdomen identified a 50-mm hyperechoic area in the right hepatic lobe. The MRI confirms a heterogenous lesion which is mildly T1 and T2 hyperintense with signal reduction on out-of- phase imaging. There is heterogenous arterial enhancement following gadolinium administration and the lesion washes out, becoming isointense on portal venous phase sequences. A sequence with a hepatocyte specific agent is also performed which reveals the lesion is hypointense. What is the most likely diagnosis? a. Adenoma b. Fibrolamellar carcinoma c. Focal nodular hyperplasia d. Haemangioma e. Metastatic deposit from breast primary malignancy
(a) Adenoma The appearances are typical for a hepatic adenoma. These lesions can be large and heterogenous due to areas of necrosis and haemorrhage. Despite being benign, they are often removed due to the risk of bleeding. They may contain fat, leading to the hyperechoic appearance on ultrasound, T1 hyperintensity and signal drop on out-of-phase imaging. Adenomas can demonstrate washout, and are the only benign lesions which may do so. The hypointensity following hepatocyte specific contrast helps to differentiate it from focal nodular hyperplasia (FNH) which would be iso to hyperintense due to their hepatocellular origin. Similarly, the absence of a high T2 signal intensity central scar differentiates it from FNH. A fibrolamellar carcinoma typically has a low signal intensity central scar and many cases demonstrate calcification. Hepatic metastases are often T1 hypointense and there is not usually fat present. Haemangiomas commonly cause incidental hyperechoic hepatic lesions on ultrasound but the other features in this case are not typical. They characteristically exhibit centripetal enhancement.
1474
A 13 week old baby boy is being investigated following in-utero diagnosis of bilateral hydronephrosis. Imaging identifies bilateral hydronephrosis and hydroureters with reflux of contrast from the urinary bladder into the ureters. The anterior abdominal wall is underdeveloped with characteristic ‘prune belly’ appearance. The paediatric team diagnose Eagle-Barrett syndrome. What other imaging should be performed in this patient? a. Echocardiogram b. MRI head c. MRI spine d. Neck ultrasound e. Testicular ultrasound
(e) Testicular ultrasound Eagle-Barrett, or prune belly syndrome is a congenital condition causing urinary tract abnormalities, a characteristic ‘prune belly’ appearance of the anterior abdominal wall and cryptorchidism. A testicular ultrasound would be able to confirm whether the testicles have descended into the scrotum, and if not, may be able to locate them in the inguinal canal. Undescended testes at birth may descend by around 3 months of age. Locating an intrapelvic or abdominal testicle with ultrasound can be more challenging, and sometimes MRI would be required. Eagle-Barrett syndrome is associated with cardiac anomalies in approximately 10%
1475
The MRI brain of a 3 year old boy suffering with seizures and spasticity is reviewed. It demonstrates symmetrical high T2 signal throughout the cerebellar white matter and the basal ganglia, including the caudate nucleus, with a similar pattern of abnormal enhancement. A previous MRI brain had less marked changes with symmetrical T2 hyperintensity and enhancement only affecting the frontotemporal lobes. Which of the following conditions best matches this radiological pattern? a. Adrenoleukodystrophy b. Alexander disease c. Canavan disease d. Krabbe disease e. Leigh syndrome
(b) Alexander disease Alexander disease is a progressive disease affecting the white matter in infants. It characteristically starts in the frontal lobes and progresses posteriorly, affecting the basal ganglia too. This causes white matter T2 hyperintensity and enhancement. In the late stages of the disease there can be cystic cavitation. X-linked adrenoleukodystrophy has a more posterior distribution of T2 hyperintensity with changes affecting the periventricular parieto-occipital white matter and splenium of the corpus callosum progressing to affect the visual and auditory pathways, with peripheral enhancement commonly seen. Canavan disease has a symmetrical, diffuse, bilateral involvement of subcortical white matter. However, there is usually sparing of the caudate nucleus, corpus callosum and internal capsule. In contrast to Alexander disease, there is no contrast enhancement. MR spectroscopy will reveal elevated N-acetyl-aspartate, which is a hallmark of the condition. Krabbe disease affects the white matter with a periventricular predilection but in a more central and posterior distribution with involvement of the centrum semiovale, thalami and basal ganglia. There is no contrast enhancement in these regions and unlike other leukodystrophies there is enlargement of the optic nerves. Leigh syndrome is a mitochondrial disorder. The changes seen on imaging tend to be symmetrical with T2 hyperintensity in the brainstem, medulla, midbrain and putamen. In contrast to the conditions already discussed above, cerebral white matter involvement is not commonly a feature.
1476
A 45 year old male smoker presents with chest pain and cough. After an abnormal chest radiograph, he has a CT which shows a 2-cm pleurally based soft tissue lesion in the left upper lobe with evidence of rib invasion. There is a further 11-mm pulmonary nodule in the left upper lobe. There are enlarged right hilar and mediastinal nodes including the left lower paratracheal lymph node as well as a left-sided pleural effusion. Biopsy of the pleural based lesion demonstrates primary lung cancer. What is the most likely type of lung malignancy in this case? a. Adenocarcinoma in situ b. Invasive adenocarcinoma c. Mesothelioma d. Small cell lung cancer e. Squamous cell carcinoma
b) Invasive adenocarcinoma The most common type of lung malignancy is non-small cell lung cancer which comprises adenocarcinoma and squamous cell carcinoma, of which adenocarcinoma is the most common. The imaging features are in keeping with lymph node involvement, and the effusion may also be malignant. Adenocarcinoma may also present as consolidation which is resistant to antibiotics, whilst adenocarcinoma in situ usually manifests as persistent predominantly ground glass opacification. Squamous cell carcinoma tends to cavitate. The common sites of metastasis are the skeleton, brain, adrenal glands, liver and soft tissues. Small cell lung cancer typically presents when it is systemically disseminated. Imaging commonly demonstrates extensive lymph node enlargement, occasionally without evidence of significant lung parenchymal abnormality. Mesothelioma is a relatively rare thoracic malignancy; however it is the most common primary pleural malignancy. It is usually nodular, but can be diffuse and is often associated with pleural effusion. It is strongly associated with previous asbestos exposure.
1477
A 34 year old woman attends the breast one-stop clinic with a right breast lump. She noticed it 1 month ago and she is unsure if it has changed over that time. There is no significant past medical or breast-related history. Her grandmother died from breast cancer at the age of 72 years. On examination, the lump is in the right upper outer quadrant at the 10 o’clock position, 6 cm from the nipple. It is firm, nontender and relatively mobile. At ultrasound the lesion measures 25 mm. It is horizontally orientated, well defined and slightly hypoechoic with a lobular contour. There are internal cystic spaces and posterior acoustic enhancement. There is vascularity in the solid component of the mass. An ultrasound guided biopsy is performed. Which of the following is the most likely diagnosis? a. Breast carcinoma b. Fat necrosis c. Fibrocystic change d. Papilloma e. Phyllodes tumour
(e) Phyllodes tumour A phyllodes tumour is difficult to differentiate from other lesions, such as fibroadenoma, without a biopsy. Typical features include a predominantly solid lesion with posterior acoustic enhancement which may contain cystic spaces and vascularity. Rapid growth is also a characteristic. Breast carcinoma is commonly described as an ill-defined, hypoechoic and spiculate mass with posterior acoustic shadowing. The mass is typically vertically orientated, disrupting tissue planes. High-grade cancers can appear as more focal well-defined masses and hence why biopsy of solid breast lesions is so important. Lobular cancers may be difficult to identify on ultrasound. Fat necrosis follows trauma to the breast, and eliciting this history is important. Its appearance depends on the age of the lesion. It may initially be a focal hyperechoic area in the fat, then become more cystic manifesting as an oil cyst, and finally it can appear as a spiculate area due to desmoplastic reaction. Eggshell calcification is typically associated with it on mammograms. Fibrocystic change is common in young women and can be painful. Simple cysts on ultrasound are anechoic and well defined. A cyst with solid components requires biopsy of the solid component. Prominent fibrous glandular tissue may be palpated and visualised at ultrasound. Papillomas are a cause of nipple discharge and may appear as a solid lesion within a dilated duct or as a solid and cystic mass.
1478
A 31 year old male patient has a long history of symptoms of early satiety, abdominal pain after eating and occasional vomiting. The discomfort is relieved when the patient lies down. The patient had been treated as an adolescent for anorexia nervosa and still has a low body mass index. An arterial and portal venous phase CT abdomen and pelvis helps establish the diagnosis and demonstrates an angle of 15° between the superior mesenteric artery (SMA) and aorta. Which other radiological findings correlate with this diagnosis? a. Aortomesenteric distance of 15 mm b. Dilatation of the proximal duodenum and stomach c. Reduced contrast enhancement of the small bowel in the distribution of the superior mesenteric artery d. Small bowel wall thickening e. SMA mural thickening and enhancement
b) Dilatation of the proximal duodenum and stomach The patient has symptoms and radiological signs of superior mesenteric artery (SMA) syndrome. This frequently affects patients with a low body mass index due to reduced intrabdominal fat; however, conditions such as lumbar lordosis can also predispose patients. The aortomesenteric angle is reduced to between 6 and 22° and the aortomesenteric distance is reduced to 2–8 mm. Sagittal reconstructions usually aid diagnosis. Another radiological sign is dilatation of the proximal duodenum and stomach, which fits with the symptoms of early satiety and vomiting. Reduced contrast enhancement of the small bowel in the distribution of the SMA and small bowel wall thickening are not features but would be more in keeping with SMA thrombus and ischaemia. SMA mural thickening and enhancement are findings consistent with a large vessel vasculitis such as Takayasu.
1479
A neonate, born at 28 weeks, has an ongoing need for ventilation and oxygenation. A chest radiograph performed at 30 days shows hyperinflation, coarse linear densities and focal areas of emphysema. What is the most likely diagnosis? a. Bronchopulmonary dysplasia b. Bronchopulmonary sequestration c. Congenital lobar emphysema d. Transient tachypnoea of the newborn e. Viral pneumonia
(a) Bronchopulmonary dysplasia The history of a baby born prematurely with long-term oxygen and ventilator requirements (>30 days) is consistent with bronchopulmonary dysplasia. The radiological appearances of hyperinflation, focal emphysema and coarse lung markings are also typical. Sequestration and congenital lobar overinflation are conditions affecting part of the lung rather than all of the lungs and therefore the radiological findings and history are not consistent with these diagnoses. Viral pneumonia can cause hyperinflation and perihilar densities but the emphysematous changes in this child are not consistent with this diagnosis. As the name suggests, transient tachypnoea of the newborn exists for a short time, usually resolving by 2–3 days following delivery
1480
A 66 year old female presents with a sudden decrease in GCS. A non-contrast CT head is performed which shows multiple bilateral, supratentorial, hyperdense intra-axial lesions (55HU). The lesions are predominantly distributed at the grey−white matter junction of the cerebral hemispheres. There is hypodensity surrounding the lesions consistent with oedema and associated mass effect. From which of the following primary malignancies are these lesions most likely to be disseminated? a. Cervical squamous cell carcinoma b. Colonic adenocarcinoma c. Endometrial endometroid carcinoma d. Pancreatic ductal adenocarcinoma e. Thyroid papillary carcinoma
(e) Thyroid papillary carcinoma The lesions described in the question are consistent with haemorrhagic metastases with a density of 55 HU and adjacent mass effect. Malignancies associated with haemorrhagic metastases include melanoma, thyroid (particularly papillary carcinoma), choriocarcinoma and renal cell carcinoma. The other available cancers do not typically cause haemorrhagic metastases.
1481
A 5 year old boy is referred to the paediatric neurologists having presented with visual field defects and diabetes insipidus. A CT head shows a partially calcified suprasellar mass. On MRI the mass is partially cystic and part solid. It demonstrates high signal on both T1 and T2 weighted imaging. There is heterogenous enhancement following gadolinium administration. What is the most likely diagnosis? a. Craniopharyngioma b. Germinoma c. Meningioma d. Pituitary macroadenoma e. Rathke cleft cyst
(a) Craniopharyngioma Craniopharyngiomas are the most common suprasellar mass in children, and the history in this question is typical with visual field defects and diabetes insipidus. Radiologically they are usually part cystic and solid and exhibit T1 and T2 hyperintensity due to proteinaceous content. Calcification is very common. Germinomas are typically solid and are neither cystic nor calcified. In contrast to this, meningiomas do commonly calcify, but a cystic component is not usually a feature. Pituitary macroadenomas occur in an older age group and calcification is rare. Rathke cleft cysts are thin walled, cystic and generally show no enhancement.
1482
A 42 year old patient with newly diagnosed ascending colon cancer is discussed at the colorectal multidisciplinary team meeting. He has a past medical history of hypertension and left nephroureterectomy for cancer 5 years previously. The patient was recently seen in outpatient clinic and reported a positive family history of colorectal cancer, with his father and two brothers affected. His father was successfully treated for colorectal cancer but later died from glioblastoma. His sister has recently had a hysterectomy following a diagnosis of endometrial cancer. Which of the following hereditary syndromes does the patient most likely have? a. Familial adenomatous polyposis b. Lynch syndrome c. Peutz-Jeghers syndrome d. Tuberous sclerosis e. Turcot syndrome
(b) Lynch syndrome Along with colorectal cancer, there is a patient history of nephroureterectomy, consistent with previous urothelial malignancy. The family history reveals endometrial cancer and glioblastoma. These all fit with Lynch syndrome, otherwise known as hereditary non- polyposis colorectal cancer. This is the most common cancer syndrome and increases the risk of urinary tract transitional cell carcinoma, endometrioid endometrial cancer and glioblastoma as well as ovarian, small bowel and gastric cancers. Apart from colorectal cancer, familial adenomatous polyposis is also associated with other conditions such as hepatoblastoma, osteomas, papillary thyroid cancer and desmoid tumours. Peutz-Jeghers syndrome is a condition which causes non-neoplastic hamartomas. These are not premalignant; however, the syndrome is associated with malignancies such as cervical adenoma malignum, breast, pancreas, ovarian and testicular tumours. Tuberous sclerosis predisposes patients to multiple gastrointestinal polyps but it is not associated with colorectal cancer. Associations with neurological tumours such as subependymal giant cell astrocytoma and renal angiomyolipoma, oncocytomas and renal cell carcinoma are documented. Turcot syndrome is characterised by colon cancer as well as primary brain tumours, such as glioblastoma and medulloblastoma. Although the father in this case died from glioblastoma, Turcot syndrome is rare compared to Lynch syndrome.
1483
Following an MRI of the auditory canal, an ear, nose and throat clinic patient with conductive hearing loss is diagnosed with a cholesteatoma located in the attic. Which of the following imaging characteristics is most consistent with this diagnosis?
e) T1 hypointense, T2 hyperintense, Restricted diffusion, Medial displacement Cholesteatoma are middle ear masses, which unlike other masses in this region demonstrate restricted diffusion. They are T1 hypointense and T2 hyperintense and do not typically enhance. They most commonly affect the superior tympanic membrane (pars flaccida) occurring in the attic, otherwise known as Prussak’s space. They can erode the scutum and displace the ossicles and in this location displace them medially. When cholesteatomas less commonly involve the pars tensa, the ossicles are displaced laterally.
1484
A 40 year old man presents with bright red rectal bleeding, hypotension and tachycardia. He is initially resuscitated. The colonoscopist is unable to negotiate the sigmoid colon and active bleeding is seen from proximal bowel. Endoscopy to the third part of the duodenum is normal. A 99m Tc RBC scan demonstrates accumulation across the upper abdomen, interpreted to be the proximal transverse colon. An angiogram is performed. Which vessel should be catheterised to maximise the chance of demonstrating the bleeding point and what would be the most likely cause of this? (a) coeliac axis and villous adenoma (b) coeliac axis and angiodysplasia (c) superior mesenteric artery and villous adenoma (d) superior mesenteric artery and angiodysplasia (e) inferior mesenteric artery and villous adenoma
D
1485
A 72 year old man, with a past history of coronary bypass surgery and transient ischaemic attack, comes to the clinic with a nine month history of weight loss and unwillingness to eat due to abdominal pain associated with meals. Investigation shows an iron deficiency anaemia and reduced serum albumin. What is most likely to be found on abdominal imaging? (a) caecal pole mass on double contrast barium enema (b) multiple polyps on small bowel enema (c) severe atheromatous disease of the aorta with small bowel wall thickening on abdominal CT (d) cavernous transformation of the portal vein on superior mesenteric angiography (e) increased uptake in the region of the terminal ileum on labelled white cell scan
C
1486
A 30-year-old man has been involved in an Road Traffic Accident (RTA). Aortic injury is suspected. CT angiogram shows a fusiform dilatation at the anteromedial aspect of the aortic isthmus with a steep contour superiorly, gently merging with the proximal descending thoracic aorta inferiorly. What is the likely diagnosis? A. Pseudoaneurysm B. Coarctation of the aorta C. Ductus diverticulum D. Aortic nipple E. Avulsed left subclavian artery
C. Ductus diverticulum Ductus diverticulum is a focal bulge at the anteromedial aspect of the aortic isthmus, visualised in 9% of adults. It is critical to identify this normal variant and distinguish it from a post-traumatic false aneurysm, which also occurs most commonly at the aortic isthmus (88%). The classic ductus diverticulum has smooth, uninterrupted margins and gently sloping symmetric shoulders; in contrast, false aneurysms have a variety of shapes and sizes with sharp margins and often contain linear defects. Compared with the classic ductus diverticulum, the atypical ductus diverticulum has a shorter and steeper slope superiorly and a more classic gentle slope inferiorly. However, both shoulders have smooth, uninterrupted margins, an important feature that distinguishes this variant from true injury. Other normal variants that can mimic injury include aortic spindle, which is a smooth circumferential bulge immediately distal to the aortic isthmus; infundibulum at the origin of aortic branches like the brachiocephalic and intercostal arteries, which are spherical or conical in shape but have a vessel at its apex, thereby differentiating them from false aneurysms.
1487
A contrast CT scan shows an incidental renal cyst that is hyperdense with thick septations and a mural nodule. What is the Bosniak classification? A. Type 1 B. Type 2 C. Type 2F D. Type 3 E. Type 4
D. Type 3 Type 3 cysts have thickened irregular/smooth walls or septa in which measurable enhancement is present. These need surgery in most cases, as neoplasm cannot be excluded. They include complicated haemorrhagic/infected cysts, multilocular cystic nephroma and cystic neoplasms. Type 2F (F denotes follow-up) cysts may contain multiple hairline-thin septa. Perceived (not measurable) enhancement of a hairline smooth septum or wall can be identified, and there may be minimal thickening of the wall or septa, which may contain calcification that may be thick and nodular. There are no enhancing soft-tissue components; totally intrarenal non-enhancing high-attenuation renal lesions (>3 cm) are also included in this category. These lesions are generally well marginated and are thought to be benign but need follow-up. Type 1 is a benign simple water attenuation cyst with a hairline-thin wall that does not contain septa, calcifications, or solid components and does not enhance. Type 2 is a benign cystic lesion that may contain a few hairline septa in which perceived (not measurable) enhancement might be appreciated; fine calcification or a short segment of slightly thickened calcification may be present in the wall or septa. Uniformly high-attenuation lesions (<3 cm) that are sharply marginated and do not enhance are included in this group. No intervention is needed. Type 4 are clearly malignant cystic masses that can have all of the criteria of Type 3 but also contain distinct enhancing soft-tissue components independent of the wall or septa; these masses need to be removed.
1488
A 33-year-old man with short stature and normal intelligence is being investigated for lower back pain. MRI of the thoracolumbar spine shows marked central stenosis with short pedicles. A comment of bullet-shaped vertebra with progressive narrowing of the lumbar interpedicular distance was noted on the report. Which of the following conditions is most likely? A. Hurler’s syndrome B. Congenital pituitary dwarfism C. Achondroplasia D. Thanatophoric dysplasia E. Hunter’s syndrome
C. Achondroplasia Spinal stenosis from congenital short pedicles along with reducing interpedicular distance towards the lumbar spine is a classic finding of achondroplasia. Other associated findings include the‘champagne glass pelvis’, bullet-shaped vertebra (cf. central vertebral beaking in Morquio syndrome and inferior vertebral beaking in Hurler’s and Hunter’s syndromes), trident hand and craniocervical stenosis from a small foramen magnum. Platyspondyly, loss of vertebral height, specially affecting lumbar vertebra by 2–3 years of age, is a typical feature of Morquio syndrome (cf. vertebral height is normal in Hurler’s syndrome).
1489
An obese 25-year-old man presents with atypical chest pain. Cardiac MR demonstrates asymmetrical hypertrophy of the interventricular septum, primarily affecting the anteroinferior portion. What is the most likely diagnosis? A. Hypertrophic obstructive cardiomyopathy B. Restrictive cardiomyopathy C. Myocardial infarction D. Dilated cardiomyopathy E. Constrictive pericarditis
A. Hypertrophic obstructive cardiomyopathy Hypertrophic cardiomyopathy (HCM) is defined as a diffuse or segmental left-ventricular hypertrophy with a non-dilated and hyperdynamic chamber, in the absence of another cardiac or systemic disease explaining the degree of cardiac muscle hypertrophy. Dyspnoea on exertion is the most common symptom because the key functional hallmark of hypertrophic cardiomyopathy is an impaired diastolic function with impaired LV filling in the presence of preserved systolic function. Systolic dysfunction occurs at end-stage disease. Asymmetric involvement of the interventricular septum is the most common form of the disease, accounting for an estimated 60%–70% of the cases of HCM. Other variants include apical, symmetric, midventricular, mass-like and non-contiguous HCM is typically associated with hypertrophy of the muscle to 15 mm or thicker and a ratio of thickened myocardium to normal left-ventricular basal myocardium of 1.3–1.5. With MRI and multidetector computed tomography (CT), apical HCM has a characteristic spadelike configuration of the LV cavity at end diastole, appreciated on vertical long-axis views.
1490
A 65-year-old diabetic with a history of alcohol excess is referred for a barium swallow following a history of dysphagia. The study shows several small, thin, flask-shaped structures along the cervical oesophagus oriented parallel to the long axis of the oesophagus. What is the most likely diagnosis? A. Feline oesophagus B. Pseudodiverticulosis C. Glycogenic acanthosis D. Traction diverticulum E. Idiopathic eosinophilic oesophagitis
B. Pseudodiverticulosis Oesophageal intramural pseudodiverticulosis is a condition of unknown cause characterised by flask-shaped outpouchings of the mucosa that extend into the muscular layer and show characteristic findings on oesophagograms. They are dilated excretory ducts of deep oesophageal mucous glands resulting from obstruction of excretory ducts by plugs of viscous mucus and desquamated cells or by extrinsic compression of the ducts by periductal inflammatory infiltrates and fibrotic tissue. It occurs in all age groups predominantly in the sixth and seventh decades with slight male preponderance. It has been reported as a separate entity or in association with diseases such as diabetes, peptic strictures and oesophagitis.
1491
An 11-year-old boy with left shoulder pain has a shoulder X-ray, which shows a lucent lesion in the metaphysis. This has distinct borders and lies in the intramedullary compartment. It is orientated along the long axis of the humerus. What is the most likely diagnosis? A. Aneurysmal bone cyst B. GCT C. Simple bone cyst D. Chondroblastoma E. Non-ossifying fibroma
C. Simple bone cyst SBC affects the young, aged 3–19 years, during the active phase of bone growth and has a slight male preponderance (M:F = 3:1). They are asymptomatic, unless fractured. They are commonly seen in the proximal femur or proximal humerus. They are solitary intramedullary lesions, centred at the metaphyses, adjacent to the epiphyseal cartilage (during the active phase) and migrating into diaphysis with growth (during the latent phase). They do not cross the epiphyseal plate. On a radiograph, they appear as an oval radiolucency with a long axis parallel to the long axis of the host bone, a fine sclerotic boundary and scalloping of the internal aspect of the underlying cortex. SBC appears as a photopenic area on a bone scan (if not fractured). Classic‘fallen fragment’ sign if fractured (20%); centrally dislodged fragment falls into a dependent position.
1492
A 58-year-old woman undergoes an echocardiogram followed by cardiac MRI for investigation of exertional dyspnoea. The cardiac MRI was reviewed at the X-ray meeting, and the radiologist diagnosed concentric hypertrophic cardiomyopathy. Which of the following did the radiologist see? A. Thickening of the interatrial septum at 7 mm B. Thickening of the entire LV wall measuring 17 mm at end diastole C. Nodular high signal in the interventricular septum on T2 D. Thickening of the LV wall measuring 14 mm with normal systolic function E. Thickened LV with delayed hyperenhancement of midwall
B. Thickening of the entire LV wall measuring 17 mm at end diastole HCM should be differentiated from other causes of symmetric increased thickness of the LV wall, including athlete’s heart, amyloidosis, sarcoidosis, Fabry disease and adaptive LV hypertrophy due to hypertension or aortic stenosis. HCM is associated with hypertrophy of the muscle to 15 mm or thicker. In cardiac amyloidosis, the amyloid protein is deposited in the myocardium, which leads to diastolic dysfunction and restrictive cardiomyopathy. Because amyloidosis is a systemic process, involvement of all four chambers is common; thus, an increase in the thickness of the interatrial septum and right atrial free wall by more than 6 mm is seen. Dynamic enhanced MRI shows late enhancement over the entire subendocardial circumference. Sarcoidosis is a non-caseating granulomatous disease that infiltrates any area of the body, but most of the morbidity/mortality is from involvement of the heart. MRI shows nodular or patchy increased signal intensity on both T2-weighted and enhanced images, which often involves the septum (more particularly, the basal portion) and the LV wall, whereas papillary and right-ventricular infiltration are rarely seen. Fabry disease is a rare X-linked autosomal recessive metabolic storage disorder. At MRI, the LV wall is seen to be concentrically thickened, and delayed hyperenhancement is typically seen mid-wall and has been reported in the basal inferolateral segment. Differentiation between compensatory hypertrophy and HCM is sometimes difficult. In comparison to HCM, patients with compensatory hypertrophy usually have normal systolic function, rather than hyperdynamic systolic function in HCM, and their LV wall rarely exceeds 15 mm in maximal thickness. Athlete’s heart can show increased LV wall thickness but end diastolic volume and ejection fraction are normal. Another feature of the cardiac remodelling in athletes is the lack of areas of delayed hyperenhancement within the LV myocardium at dynamic enhanced MRI.
1493
An elderly patient on long-term dialysis presents to the orthopaedic clinic with right shoulder pain. Plain films show juxta-articular swelling and erosions of the humerus, but the joint space is preserved. MRI shows a small joint effusion and the presence of low- to intermediate-signal soft tissue on all sequences covering the synovial membrane extending into the periarticular tissue. What is the likely diagnosis? A. Amyloid arthropathy B. Gout C. Calcium pyrophosphate deposition disease (CPPD) D. Pigmented villonodular synovitis (PVNS) E. Reticuloendotheliosis
A. Amyloid arthropathy Amyloid arthropathy most typically affects the shoulders, carpal bones and hips in a bilateral fashion. It is typically associated with long-term renal dialysis, which results in deposition of the beta-2 microglobulin. Affected joints demonstrate subchondral cystic lesions with juxta-articular swelling. The presence of low-to-intermediate signal soft tissue within and around the joint clinches the diagnosis, as this represents the signal characteristics of the deposited proteins (cf. other inflammatory/infectious arthropathies, which tend to produce higher water content than soft-tissue changes in the joint). Joint space is also typically preserved until the late stages of disease, similar to gout.
1494
A 30-year-old woman presents with shortness of breath and fatigue. CT shows enlargement of the right atrium, right ventricle and pulmonary artery and normal appearance of the left atrium. What is the most likely diagnosis? A. VSD– Ventricular Septal Defect B. ASD– Atrial Septal Defect C. Bicuspid aortic valve D. Coarctation of the aorta E. Mitral valve disease
1495
A 50-year-old man is referred to a gastroenterologist with a 6-month history of intermittent epigastric pain and nausea. He is referred for a barium meal test due to a failed OGD– oesophago-gastroduodenoscopy. The study shows an ulcer along the lesser curve of the stomach. Which of the following is a malignant feature of a gastric ulcer? A. The margin of the ulcer crater extends beyond the projected luminal surface. B. Carman meniscus sign. C. Hampton’s line. D. Central ulcer within mound of oedema. E. The ulcer depth is greater than the width.
1496
A 34-year-old woman with previous history of upper limb weakness that resolved spontaneously and optic neuritis was referred for an MRI brain. MRI confirms the presence of bilateral periventricular hyperintensities on FLAIR with abnormal signal in the corpus callosum and middle cerebellar peduncles. MRI also shows signal abnormality in the right optic nerve. Which portion of the optic nerve does Multiple sclerosis (MS) most commonly affect? A. Intra-orbital. B. Intracanalicular. C. Intracranial. D. Chiasmatic. E. All portions are equally susceptible.
A. Intra-orbital Typically, findings of optic neuritis in MS are seen in the retrobulbar intra-orbital segment of the optic nerve, which appears swollen, with high T2 signal. High T2 signal persists and may be permanent; chronically the nerve will appear atrophied rather than swollen. Contrast enhancement of the nerve is best seen with fat-suppressed T1-weighted coronal images, in >90% of patients if scanned within 20 days of visual loss.
1497
A newborn baby has US of the spine. At which level is the conus expected to be? A. Above L1 B. Above T12 C. L2 to L3 D. L3 to L4 E. S2
C. L2 to L3 The conus normally lies at or above the L2 disc space. A normal conus located at the mid-L3 level may be identified, especially in preterm infants; this position is considered the lower limits of normal but is usually without clinical consequence. However, in a preterm infant with a conus that terminates at the L3 mid-vertebral body, a follow-up sonogram can be obtained once the infant attains a corrected age between 40 weeks’ gestation and 6 months of age. In contrast, the thecal sac terminates at S2. In the preterm group, more than 90% of conus medullaris cases lie above L2; in the term group, more than 92% lie above L2.
1498
A nursing home resident is found to have a lung tumour and undergoes CT staging of the chest and abdomen. This reveals a discrete lesion medial to the second part of the duodenum with a fluid–fluid level. What is the most likely diagnosis? A. Duplication cyst B. Duodenal diverticulum C. Duodenal web D. Annular pancreas E. Adenocarcinoma of the duodenum
26. B. Duodenal diverticulum Duodenal diverticulosis is a common entity first described by Chomel in 1710. Its prevalence varies depending on the mode of diagnosis. Diverticula are found in 6% of upper gastrointestinal series, 9%–23% of ERCP procedures and 22% of autopsies. Its occurrence has no sex predilection, and the age range for detection varies from 26 to 69 years. Duodenal diverticula may be congenital or acquired, with the latter being more common. Congenital or true diverticula are rare, contain all layers of the duodenal wall, and may be subdivided into intraluminal and extraluminal forms. The CT appearance of a duodenal diverticulum includes a saccular outpouching, which may resemble a mass-like structure interposed between the duodenum and the pancreas that contains air, an air–fluid level, fluid, contrast material, or debris. A periampullary diverticulum may simulate a pseudocyst or tumour.
1499
A 3-year-old presents as acutely unwell with a maculopapular rash, lymphadenopathy and erythema of her palms. Her white cell count is normal, and a specific cause for her symptoms is not found. She improves on immunoglobulins and supportive treatment. A follow-up echocardiogram shows cardiomegaly and a coronary artery aneurysm. What is the likely diagnosis? A. Takayasu arteritis B. Kawasaki arteritis C. Moyamoya syndrome D. Henoch–Schonlein purpura E. Churg–Strauss syndrome
B. Kawasaki arteritis Kawasaki disease is a systemic vasculitis that is more severe in small and medium arteries, and veins to a lesser extent, with inflammatory lesions in virtually all organs. It is a leading cause of acquired heart disease in childhood. The aetiology of KD remains unknown, although the clinical presentation– self-limiting illness manifested by an abrupt onset of fever, rash, exanthema, conjunctival injection and cervical adenopathy– and the epidemiological features– a seasonal peak in winter and spring, age distribution and a geographic wave-like spread of illness during epidemics– strongly suggest an infectious cause. Fever is usually the first sign of KD. Rash is non-specific and mostly maculopapular. Cervical lymphadenopathy is the last common of the main manifestations. Cardiovascular complications include coronary artery aneurysms, myocarditis, pericarditis with pericardial effusion, systemic arterial aneurysms, valvular disease, mild aortic root dilatation and myocardial infarct. Takayasu arteritis (TA), also known as pulseless disease, is a granulomatous large vessel vasculitis that predominantly affects the aorta and its major branches, with increased prevalence in Asian women <50 years of age. Churg–Strauss syndrome is a small-to-medium vessel necrotising pulmonary vasculitis, affecting patients in the third and fourth decades with asthma, eosinophilia and systemic symptoms like purpura and arthralgia. Moyamoya disease is an idiopathic, non-inflammatory, non-atherosclerotic, progressive vasculo- occlusive disease involving the circle of Willis, typically the supraclinoid internal carotid arteries. It has a bimodal age distribution, affecting children and adults. In children, ischaemic strokes are most pronounced, whereas in adults haemorrhage from the abnormal vessels is more common.
1500
A 76-year-old male patient with chronic inflammatory disease and known history of secondary generalised multisystem amyloidosis showed an abnormal appearance of the heart on echocardiography. Dynamic enhanced cardiac MR imaging was advised for further characterisation. All of the following are imaging findings seen with cardiac amyloidosis, except A. Left ventricular wall hypertrophy B. Subendocardial delayed myocardial hyperenhancement C. Systolic dysfunction D. Granular echogenic myocardium E. Interatrial septal thickening
C. Systolic dysfunction In cardiac amyloidosis, the amyloid protein is deposited in the myocardium, which leads to diastolic dysfunction that progresses to restrictive cardiomyopathy. Because amyloidosis is a systemic process, involvement of all four chambers is common; thus, an increase in the thickness of the interatrial septum and right atrial free wall by more than 6 mm has been shown to be a specific finding for cardiac amyloidosis. Through the use of dynamic enhanced cardiac MRI, a distinct pattern of late enhancement, which was distributed over the entire subendocardial circumference, has been shown to have high specificity and sensitivity for cardiac amyloidosis Echocardiogram shows concentric LV hypertrophy, with hyperechoic granular sparkling of the ventricular wall.
1501
An elderly woman presents with progressive atraumatic pain within her right knee over the course of the last month, particularly on the medial aspect, associated with functional impairment. Her clinical history includes a meniscal tear, which was treated arthroscopically 10 years ago with a good outcome. An MRI reveals florid marrow oedema within the medial femoral condyle associated with mild flattening of the weight-bearing surface. What is the diagnosis? A. Perthe’s disease B. Sinding–Larsen’s disease C. Blount’s disease D. Spontaneous osteonecrosis of the knee E. Osteochondral defect
D. Spontaneous osteonecrosis of the knee Spontaneous osteonecrosis of the knee (SONK) is a rapid and painful condition in elderly patients that ultimately results in subchondral collapse of the weight-bearing portion of the medial femoral condyle. It is often idiopathic but can be associated with minor trauma. It is now also increasingly recognised as a subchondral insufficiency fracture resulting in rapid secondary subchondral collapse. Perthes disease is a childhood disease with avascular necrosis of the femoral head. Sinding–Larsen disease is essentially tendinosis of the proximal origin of the patella tendon. Blount’s disease is a growth disorder of the tibia resulting in a‘bow leg’ deformity from disturbance to the medial proximal tibial epiphysis. An osteochondral defect is a traumatic injury involving the articular cartilage and adjacent subchondral bone.
1502
A known MS patient has presented to the neurologist with clinical features of involvement of the spinal cord. An MRI of the whole spine has been requested with a view towards assessment of the cord for possible multiple sclerosis (MS) plaques. MS lesions in the spinal cord occur most commonly in the A. Cervical segment. B. Thoracic segment. C. Lumbar segment. D. Sacral segment. E. All segments are equally affected
A. Cervical segment MS can show multiple lesions in the spinal cord. Typical spinal cord lesions in MS are relatively small and peripherally located. They are most often found in the cervical cord and are usually less than two vertebral segments in length.
1503
A neonate presents with non-bilious vomiting with a palpable upper abdominal lump. Which of the following US findings would not be in keeping with pyloric stenosis? A. Pyloric muscle thickness 3.5 mm B. Target sign C. Pyloric canal length 14 mm D. Antral nipple sign E. Cervix sign
C. Pyloric canal length of 14 mm Ultrasound is the modality of choice because of its advantages of directly visualising the pyloric muscle and no ionising radiation. The hypertrophied muscle is hypoechoic, and the central mucosa is hyperechoic. Normal measurements of the pylorus are as follows: Pyloric muscle thickness (i.e., the diameter of a single muscular wall on a transverse image): <3 mm (most accurate) Length (i.e., longitudinal measurement): <15–17 mm Pyloric volume: <1.5 cc Pyloric transverse diameter: <13 mm Abnormal features on US includes target sign (hypoechoic ring of hypertrophied pyloric muscle around echogenic mucosa centrally on cross section), cervix sign (indentation of muscle mass on fluid-filled antrum on longitudinal section) and antral nipple sign (redundant pyloric channel mucosa protruding into gastric antrum). Other features include increased antral peristalsis and delayed gastric emptying. Infantile pyloric spasm also shows increased peristalsis and delayed gastric emptying with pyloric muscle thickness between 1.5 and 3 mm.
1504
A patient recently diagnosed with MS has been sent for an MRI of the whole spine to detect possible spinal plaques. All of the following are MR features of spinal cord lesions in MS, except A. The sole site of involvement (in some cases). B. Imaging features similar to those of MS lesions in the brain. C. Most lesions are centrally located. D. The length rarely exceeds two vertebral segments. E. Dorsal column involvement.
C. Most lesions are centrally located. Occurrence of spinal cord abnormalities is largely independent of brain lesions in MS. Both focal and disuse lesions affecting the cord are described, though multiple focal lesion (median 3) is the most common finding. Patients with focally involved spinal cords mostly show multiple small lesions. Focal lesions have an elongated configuration along the axis of the spinal cord and affect the peripheral part of the cord. Cervical cord is the most commonly affected segment and the lesions usually extend over fewer than two vertebral segments in length.
1505
A woman presents with infertility and undergoes a hysterosalpingogram. This demonstrates a uterus with two converging horns. A wide angle is seen at the roof of the uterus. Which uterine anomaly does the patient have? A. Uterine didelphys B. Septate uterus C. Arcuate uterus D. Bicornuate uterus E. Unicornuate uterus
D. Bicornuate uterus While the presence of a divided rather than triangular uterine cavity at Hysterosalpingogram (HSG) may suggest the presence of an Mullerian duct anomaly (MDA), it is not possible to differentiate between subtypes. MRI and US provide greater anatomic detail; both of these imaging methods provide information on the external uterine contour, which is an important diagnostic feature of MDAs. Furthermore, both MRI and US may be used to assess for concomitant renal anomalies; renal anomalies occur at a higher rate among MDA patients. Unicornuate uterus appears as a small, oblong, off-midline structure on US and MRI. Uterus didelphys results from complete failure of Müllerian duct fusion. Each duct develops fully with duplication of the uterine horns, cervix and proximal vagina. A fundal cleft greater than 1 cm has been reported to be 100% sensitive and specific in differentiation of fusion anomalies (didelphys and bicornuate) from reabsorption anomalies (septate and arcuate). Bicornuate uterus involves duplication of the uterus with possible duplication of the cervix (bicornuate unicollis or bicornuate bicollis). HSG demonstrates opacification of two symmetric fusiform uterine cavities (horns) and fallopian tubes. Historically, an intercornual angle of greater than 105° was used for diagnosis of bicornuate uterus. Septate uterus is the most common form of MDA, accounting for approximately 55% of cases. Historically, an angle of less than 75° between the uterine horns has been reported to be suggestive of a septate rather than bicornuate uterus. However, considerable overlap occurs between septate and bicornuate uteri; as such, the angle measurement is not a reliable diagnostic feature. Arcuate uterus at HSG shows a single uterine cavity with a broad saddle-shaped indentation at the uterine fundus.
1506
Plain X-ray of a newborn shows a large tubular air shadow behind the trachea. The lungs are clear. The bowels are grossly distended with air. What is the likely type of tracheo-oesophageal fistula? A. Type A B. Type B C. Type C D. Type D E. Type E
C. Type C Different types of oesophageal atresia are identified on the basis of the presence (and location) or absence of a tracheo-oesophageal fistula. Type A is pure oesophageal atresia without fistula, and Type B is oesophageal atresia with a fistula between the proximal pouch and the trachea. Type C is oesophageal atresia with a fistula from the trachea or the main bronchus to the distal oesophageal segment. Type D is oesophageal atresia with both proximal and distal fistulas, and Type E is an H-shaped tracheo-oesophageal fistula without atresia. Of these five types, Type C is by far the most common. Oesophageal atresia is generally suspected on the basis of polyhydramnios, inability to swallow saliva or milk, aspiration during early feedings, or failure to successfully pass a catheter into the stomach. Feeding difficulties with choking occur in infants with Type E (fistula without atresia), but the diagnosis may not be made until several years later when the patient presents with a cough while swallowing, recurrent pneumonia and a distended abdomen. In Types A and B, there is a complete absence of gas in the stomach and intestinal tract, whereas in Types C and D the gastrointestinal tract commonly appears distended with air.
1507
A 56-year-old woman known to the endocrinologist has been going to her family doctor with a funny sensation in her right hand and fingers for the last few months. An MRI was organised along with nerve conduction studies by her family doctor. MRI revealed fusiform swelling of the median nerve in the distal forearm just before the entrance into the carpal tunnel with increased signal on T2. What is the likely diagnosis? A. Cervical spondylosis B. Ulnar tunnel syndrome C. Carpal tunnel syndrome D. Cervical rib with brachial plexus impingement E. Neurofibroma of the median nerve
E. Alport’s syndrome Causes of medullary nephrocalcinosis include hyperparathyroidism, sarcoidosis, myelomatosis, primary or secondary hyperoxaluria (Crohn’s disease), hyperthyroidism, osteoporosis, idiopathic hypercalciuria, renal tubular acidosis, medullary sponge kidney and drug-induced (hypervitaminosis D, milk-alkali syndrome). Alport’s syndrome is an autosomal dominant condition also called chronic hereditary nephritis, associated with ocular abnormalities, deafness, small kidneys, cortical calcification and progressive renal failure without hypertension.
1508
A 66-year-old man with progressive shortness of breath and low-volume ECG shows an enlarged heart on chest X-ray. Echocardiogram confirms the presence of a moderately large pericardial effusion. The pericardial fluid is aspirated for symptomatic relief and sent off for cytology and culture. Cytology comes back as positive for malignant cells. Which of the following is the most common type of primary pericardial malignancy? A. Fibrosarcoma B. Pericardial angiosarcoma C. Fibromyxoid sarcoma D. Mesothelioma E. Epithelioid endothelioma
D. Mesothelioma Malignant mesothelioma is the most common primary pericardial malignancy. A causal relationship with asbestosis is uncertain because of low prevalence of this neoplasm. Mesothelioma may present as a well-defined single mass, multiple nodules, or diffuse plaques involving the visceral and parietal pericardium and wrapping around the cardiac chambers and great vessels. Other malignant primary tumours include lymphoma, sarcoma, pheochromocytoma and liposarcoma. Teratomas of the pericardium may also be malignant and are most commonly seen in children. Pericardial metastases are much more common than primary pericardial tumours. Breast and lung cancers are the most common sources of metastases in the pericardium, followed by lymphomas and melanomas.
1509
A plain lumbar spine radiograph of a 45-year-old woman shows marked posterior scalloping of the vertebral bodies extending over several vertebral lengths. All of the following are diseases associated with this finding, except A. Marfan B. Neurofibromatosis C. Ependymoma D. Achondroplasia E. Hypothyroidism
E. Hypothyroidism A common cause of localised posterior vertebral scalloping is increased intraspinal pressure secondary to an expanding mass. Widening of the interpediculate distance and alteration of the configuration of the pedicles are associated signs. Relatively large, slow-growing lesions that originate during a period of active skeletal growth (such as ependymomas) are most likely to give rise to posterior vertebral scalloping. Dural ectasia is thought to cause posterior vertebral scalloping due to loss of the normal protection provided to the vertebral body by a strong, intact dura. Dural ectasia classically occurs in association with inherited connective-tissue disorders such as Marfan syndrome (classical) and Ehlers–Danlos syndrome. Posterior vertebral scalloping is also commonly seen in patients with neurofibromatosis, most likely due to dural ectasia but also secondary to neurofibromas or a thoracic meningocoele. It has also been reported in patients with AS; in these cases, the development of associated arachnoid cysts may give rise to cauda equina syndrome. Acromegaly has been described as a further cause of diffuse posterior vertebral scalloping, probably because of a combination of soft-tissue hypertrophy in the spinal canal and increased bone resorption.
1510
A 90-year-old man is admitted following intermittent episodes of bright red rectal bleeding. He is haemodynamically stable on initial assessment. OGD and flexible colonoscopy are normal. He subsequently has another bleed on the surgical ward and is then referred for a CT mesenteric angiogram. Which of the following statements is false regarding CT mesenteric angiography? A. Severe bleeding episodes, such as those manifesting with hemodynamic instability, decrease the pretest probability of a positive result for active bleeding at CT angiography. B. Active bleeding must be present during the time contrast is injected into the vascular system in order to demonstrate the site of bleeding. C. Portal venous phase imaging depicts extravascular blushes with higher sensitivity than arterial phase imaging does. D. Retention of previously administered barium in colonic diverticula may be mistaken for, or may obscure, acute extravasation of contrast material. E. Hyperattenuating material within the bowel lumen on the unenhanced scan without additional findings in the contrast-enhanced phases indicates recent haemorrhage.
A. Severe bleeding episodes, such as those manifesting with haemodynamic instability, decrease the pretest probability of a positive result for active bleeding at CT angiography. Severe bleeding episodes, such as those manifesting with haemodynamic instability, increase the pretest probability of a positive result for active bleeding at CT angiography.
1511
A 25-year-old man undergoing abdominal CT shows the presence of bridging renal tissue across the midline at the level of the lower poles, consistent with a horseshoe kidney. All the following are recognised associations, except A. Bicornuate uterus B. Cardiac anomaly C. Undescended testis D. Tracheo-oesophageal fistula E. Anorectal malformation
D. Tracheo-oesophageal fistula Horseshoe kidney is the most common fusion anomaly of the kidneys. There is recognised association with cardiovascular, skeletal, CNS, genitourinary anomalies (undescended testes, bicornuate uterus, duplication of ureter, hypospadias, etc.), anorectal malformations, trisomy 18 and Turner syndrome. Vesico-ureteric reflux, hydronephrosis secondary to PUJ obstruction and increased frequency of complications like renal stones and infection are recognised.
1512
Neck US of a previously well 2-year-old girl shows a 3-cm thin-walled cystic structure with multiple septae of variable thickness in the left posterior triangle with extension into the mediastinum. The diagnosis is: A. Third branchial cleft cyst B. Cervical meningocoele C. Cystic teratoma D. Lymphangioma E. Second branchial cleft cyst
D. Lymphangioma A cystic hygroma is the most common form of lymphangioma and constitutes about 5% of all benign tumours of infancy and childhood. On US scans, most cystic hygromas manifest as a multilocular predominantly cystic mass with septa of variable thickness. The echogenic portions of the lesion correlate with clusters of small, abnormal lymphatic channels. Fluid–fluid levels can be observed with a characteristic echogenic, haemorrhagic component layering in the dependent portion of the lesion. Prenatal US may demonstrate a cystic hygroma in the posterior neck soft tissues. On CT images, cystic hygromas tend to appear as poorly circumscribed, multiloculated, hypoattenuated masses. They typically have characteristic homogeneous fluid attenuation. Usually, the mass is centred in the posterior triangle or in the submandibular space. A third branchial cleft cyst most commonly appears as a unilocular cystic mass centred in the posterior cervical space on CT and MRI. At US, a second branchial cleft cyst is seen as a sharply marginated, round to ovoid, centrally anechoic mass with a thin peripheral wall that displaces the surrounding soft tissues. The‘classic’ location of these cysts is at the anteromedial border of the sternocleidomastoid muscle. The first branchial cleft cyst appears as a cystic mass either within, superficial to, or deep to the parotid gland.
1513
A 5-year-old boy involved in an RTA is referred for a trauma CT scan. The reporting radiologist does not find any acute abnormality. However, there are other incidental findings on the scan suggestive of malrotation. Which of the following options is the most specific feature of gut malrotation on CT? A. SMV (superior mesenteric vein) anterior to the SMA (superior mesenteric artery) B. SMV to the right of the SMA C. Whirl sign around the SMA D. DJ (duodenojejunal) flexure to the right of the midline E. SMV to the left of the SMA
E. SMV to the left of the SMA SMV positioned to the left of SMA is the most specific sign of malrotation on CT (80%). Other signs on CT include the‘whirl sign’ around the SMA and large intestine on the left with small intestine on the right. Abnormal position of the caecum and duodenum with duodeno-jejunal junction over the right pedicle is the most specific sign of malrotation on barium meal studies.
1514
In order of frequency, the most common location of congenital lobar emphysema is as follows: A. LUL, LLL, RUL B. LUL, RML, RUL C. RUL, RML, LLL D. LUL, RUL, RLL E. LUL, RML, RLL
B. LUL, RML, RUL Congenital lobar emphysema represents a condition of progressive over-distension of one or multiple pulmonary lobes secondary to deficiency/immaturity of bronchial cartilage, endobronchial obstruction, or extrinsic compression. It is more common in boys. Preferential involvement is LUL (left upper lobe) > RML (right middle lobe) > RUL (right upper lobe) > two lobes. Initial chest X-ray shows opacification of lobe secondary to delayed clearing of pulmonary fluid; this is followed by progressive features of air trapping, hypertranslucent lung and mediastinal shift.
1515
A 17-year-old girl presents with a history of acute-on-chronic burning neck pain radiating into the right shoulder and arm. There is associated palmar paraesthesia, easy fatigability and loss of power, exacerbated by elevating the arm to the shoulder level. Sagittal T1W MRI obtained with the arm in the neutral position shows ample fat surrounding the subclavian vessels and brachial plexus. With the arm in abduction, there is compression of the subclavian vessels. What is the diagnosis? A. Subclavian artery stenosis B. Parsonage–Turner syndrome C. Median nerve entrapment D. Thoracic outlet syndrome E. Subclavian steal syndrome
D. Thoracic outlet syndrome Thoracic outlet syndrome involves the brachial plexus and the subclavian artery or vein at three anatomic levels where they are vulnerable to entrapment; the interscalene space, the costoclavicular space and the retropectoralis minor space. Thoracic outlet syndrome may result from post- traumatic fibrosis of the scalene muscles, compression secondary to activities like backpacking, and clavicular fractures with callus formation and exercise-related muscle hypertrophy affecting weightlifters, swimmers, tennis players and so on. Other causes include mass lesions such as lipomas, neurogenic tumours, accessory muscles and fibrous bands. MRI can help identify specific muscle denervation patterns. Muscle oedema may occur within 24–48 hours. In contrast, fatty atrophy reflects chronic denervation and manifests several months later. In this setting, MRI has an advantage over electromyography, which does not demonstrate signs of muscle denervation until 2–3 weeks after nerve impairment. Radiographs may reveal a cervical rib or a prominent C7 transverse process. Narrowing has been reported in the costoclavicular and retropectoralis minor spaces during imaging with postural manoeuvres during dynamic MRI. Sagittal T1-weighted MRI sequences are particularly useful in demonstrating the presence of denervation-related fatty atrophy of muscles, effacement of fat planes around the compressed plexus and an abnormal intramuscular course of the components of the brachial plexus. The retropectoralis minor space is not frequently affected by entrapment and is more often involved by mass lesions.
1516
The most common structure to herniate in Bochdalek hernia is A. Stomach B. Spleen C. Omentum D. Left lobe of liver E. Pancreas
C. Omentum Bochdalek hernia represents the commonest type of congenital diaphragmatic hernia. It is more common on the left. The most common structure to herniate on the left is omental fat; on the right is the liver.
1517
What type of labral injury is not associated with an anterior shoulder dislocation? A. Bony Bankart lesion B. Perthes lesion C. Glenolabral articular disruption injury (GLAD) D. Superior labral anterior–posterior tear (SLAP) E. Anterior labroligamentous periosteal sleeve avulsion injury (ALPSA)
D. Superior labral anterior–posterior tear (SLAP) The anteroinferior glenoid labrum is typically injured in an anterior shoulder dislocation. All of the aforementioned injuries except for the superior labral anterior–posterior (SLAP) tear involve the anteroinferior labrum. The glenolabral articular disruption (GLAD) lesion is a partial tear of the anteroinferior labrum with an associated glenoid cartilage injury. Perthes lesion is a complete tear of the labrum, which is still attached to the glenoid periosteum. An anterior labroligamentous periosteal sleeve avulsion (ALPSA) injury is similar to the Perthes lesion but with medial displacement of the torn labrum, which is still attached to the glenoid scapula periosteal sleeve.
1518
A 36-year-old woman with non-remitting headache is sent for an MRI brain by the neurologist to investigate the cause of her headache. The MRI brain is mostly unremarkable apart from showing areas of hyperintensity on FLAIR in subarachnoid spaces. All of the following conditions should be included in the differential diagnosis, except A. Pacchionian granulations B. Slow arterial flow due to vascular stenosis C. Subarachnoid haemorrhage D. Infectious meningitis E. Leptomeningeal melanosis
A. Pacchionian granulations Hyperintensity on FLAIR in subarachnoid spaces has been well described in a wide range of pathologic conditions, such as subarachnoid haemorrhage (SAH), infectious or malignant meningitis, leptomeningeal spread of malignant disease, Leptomeningeal melanosis (part of the neurocutaneous melanosis congenital phakomatosis), vascular hyperintensity in the subarachnoid space produced by severe (>90%) vascular stenosis or occlusion of major cerebral vessels with resulting slow flow and fat-containing tumours like lipoma of subarachnoid space. Retrograde slow flow of engorged pial arteries through leptomeningeal anastomoses is also seen as high signal intensity in the subarachnoid space on FLAIR in patients with Moyamoya disease, called ivy sign. Other, less common, causes of subarachnoid FLAIR hyperintensity are artefacts.
1519
Which of the following is false regarding bronchopulmonary sequestration? Intralobar Extralobar A. Pleural involvement Visceral pleura Own pleura B. Venous drainage Pulmonary veins Systemic veins C. Associated anomalies Less common More common D. Symptomatic First 6 months Adulthood E. Arterial supply Aorta Aorta
D. Symptomatic First 6 months Adulthood Bronchopulmonary sequestration
1520
A 10-year-old boy presents with fever and eosinophilia. MRI of the head shows thickening of the infundibular stalk and a markedly enhancing mass in the superior aspect of the stalk. There is also enhancement in the sella extending along the left petrous temporal bone with poorly defined borders. The features are consistent with A. Meningioma B. Petrous apicitis C. Histiocytosis X D. Craniopharyngioma E. Neuroblastoma metastasis
C. Histiocytosis X Space-occupying lesions affect the hypothalamic-neurohypophyseal axis, which is the central nervous system site most commonly and often earliest involved in Langerhans cell histiocytosis. MRI findings have been correlated with symptoms of diabetes insipidus, which is a clinical hallmark of the condition. Typically, the formation of Langerhans cell histiocytosis granulomas leads to a loss in the normally high signal intensity of the posterior neurohypophysis on T1-weighted images. Furthermore, the hypothalamus, the pituitary stalk or both are frequently enlarged and demonstrate gradually increasing homogeneous enhancement after an intravenous injection of gadolinium, without subsequent washout. The differential diagnosis includes other infundibular diseases, such as adenohypophysitis, which can be differentiated from Langerhans cell histiocytosis by a sharp increase in contrast enhancement and rapid washout after the administration of the intravenous contrast medium. Granulomatous diseases such as sarcoidosis, Wegener disease and leukaemia must also be considered in the differential. Rarer differentials are germ cell tumours (germinoma, teratoma) and haemangioblastoma. These produce the same MRI features, with the same pattern of enhancement at dynamic imaging. The second most frequent pattern of central nervous system involvement in Langerhans cell histiocytosis is characterised by intra-axial neuro-degenerative changes. Bilateral symmetric lesions in the cerebellum, especially the dentate nucleus, basal ganglia, or brainstem, are most often observed. The differential diagnosis includes ADEM, acute multiphasic disseminated encephalitis, disseminated encephalitis, various metabolic and degenerative disorders, leukoencephalopathy secondary to chemotherapy or radiation therapy, and paraneoplastic encephalitis Less frequently, Langerhans cell histiocytosis granulomas, which resemble tumours, are observed in the extra-axial space (in the meninges, pineal gland, choroid plexus and spinal cord)..
1521
Follow-up CT chest done on a 71-year-old man with previous history of malignancy shows a mass lesion in the heart. It is new compared to previous CT scans and is determined to represent a metastatic deposit. Which of the following types of malignancy is most likely to be the primary in this case? A. Colonic B. Oesophageal C. Bronchogenic D. Renal cell E. Astrocytoma
C. Bronchogenic Pericardial metastases are much more common than primary pericardial tumours. Breast and lung cancers are the most common sources of metastases in the pericardium, followed by lymphomas and melanomas.
1522
A 30-year-old weightlifter presents with swelling in the right groin. Which of the following is false?
A. Direct inguinal The hernial sac lies lateral to the inferior epigastric artery and above hernia the pubic tubercle. There are several sites on the abdominal wall prone to herniation. The first site is the deep inguinal ring, where an indirect inguinal hernia occurs. Here, herniated structures enter the inguinal canal lateral to the inferior epigastric artery and superior to the inguinal ligament, extending for a variable distance through the inguinal canal. A second site of herniation is at the inferior aspect of the Hesselbach’s triangle, where a direct inguinal hernia usually occurs. This weakened area is just lateral to the conjoint tendon and medial to the inferior epigastric artery, in contrast to the indirect inguinal hernia, which originates lateral to the inferior epigastric artery. A third weakened area is inferior in relation to the inguinal ligament and lateral to the lacunar ligament, where a femoral hernia occurs, typically medial and adjacent to the femoral vessels. The fourth area is at the lateral margin of the rectus abdominis muscle, superior to the inferior epigastric artery as it crosses the linea semilunaris, where a spigelian hernia occurs. Indirect inguinal hernias are most common regardless of sex; femoral hernias are more common in women.
1523
A 42-year-old man presents with a high-grade fever, splenomegaly and abdominal pain. CT chest and abdomen done to look for a source of sepsis show multiple small cavitating lesions in both lungs with areas of hypo-attenuation in the spleen and kidneys. Which of the following is the most likely diagnosis? A. Sarcoidosis B. Carcinoid heart disease C. Amyloidosis D. Infective endocarditis E. Rheumatic heart disease
D. Infective endocarditis Extracardiac Complications of Infective Endocarditis
1524
A 50-year-old woman has a CT abdomen and pelvis for non-specific abdominal pain. The scan shows a 7-cm low-density lesion in segment VII of the liver with heterogeneous enhancement in arterial and portal venous phase. An MRI liver is performed for further characterisation and shows a large lobulated mass with low signal on T1W and intermediate to high signal on T2W. On the dynamic post-contrast T1 scans, it shows enhancement in the arterial phase with a non-enhancing central scar, which later enhances in the delayed phase. What is the most likely diagnosis? A. Focal nodular hyperplasia B. Fibrolamellar HCC C. Adenoma D. Haemangioma E. Hepatocellular carcinoma
A. Focal nodular hyperplasia Focal nodular hyperplasia (FNH) is the second most common benign liver tumour after haemangioma. FNH is classified into two types: classic (80% of cases) and non-classic (20%). Distinction between FNH and other hypervascular liver lesions such as hepatocellular adenoma, hepatocellular carcinoma and hypervascular metastases is critical to ensure proper treatment. An asymptomatic patient with FNH does not require biopsy or surgery. MRI has higher sensitivity and specificity for FNH than does US or CT. Typically, FNH is iso- or hypointense on T1-weighted images, is slightly hyper- or isointense on T2-weighted images, and has a hyperintense central scar on T2-weighted images. FNH demonstrates intense homogeneous enhancement during the arterial phase of gadolinium-enhanced imaging and enhancement of the central scar during later phases.
1525
A 42-year-old woman is referred to the breast clinic and is due an ultrasound scan to evaluate a suspected lump in the breast. All of the following are ultrasonographic features of a benign breast mass, except A. Feeding central vessel on Doppler imaging B. Well-defined smooth margins C. Three or fewer lobulations D. Circumferential blood flow pattern on Doppler imaging E. Uniform hyperechogenicity
A. Feeding central vessel on Doppler imaging. US features characteristic of benign lesions have been described. These include hyperechogenicity compared to fat, an oval or well-defined, lobulated, gently curving shape and the presence of a thin echogenic pseudocapsule. Doppler examination of benign lesions shows displacement of normal vessels around the edge of the lesion. In contrast, malignant lesions show abnormal vessels that are irregular and centrally penetrating.
1526
All of the following are causes of a right-sided cardiac thrombus, except A. DVT B. Behcet’s syndrome C. Loffler syndrome D. Infective endocarditis E. Sarcoidosis
E. Sarcoidosis Cardiac thrombus tends to occur in older adults with a history of atrial fibrillation or ventricular aneurysm due to prior myocardial infarction. Right ventricular thrombus has been reported in patients with deep venous thrombosis, Behcet’s syndrome, Loffler syndrome, endocarditis, Churg–Strauss syndrome and right atrial aneurysms. Cardiac thrombus appears as a lobular, intracavitary mass. The density and signal intensity depend on the age of the thrombus. Cardiac thrombus does not enhance. Patients with cardiac thrombus are treated with anticoagulation.
1527
A 40-year-old woman with a known history of connective tissue disease presents to her gastroenterologist with non-specific upper abdominal pain and weight loss. She is referred for a barium follow-through, which shows that the small bowel folds are of normal morphology but distended and closely spaced together with delayed emptying of barium into the large bowel. There are also a number of jejunal diverticula. She had an X-ray of her left hand a few weeks earlier, which showed resorption of the distal tufts of her phalanges. What is the most likely unifying diagnosis? A. Hyperparathyroidism B. Whipple’s disease C. Scleroderma D. Amyloidosis E. Coeliac disease
C. Scleroderma Systemic sclerosis, or scleroderma, is characterised by excessive collagen production, autoimmune disease–induced inflammation, and microvascular injury. It is divided into two subtypes: limited cutaneous systemic sclerosis and diffuse cutaneous systemic sclerosis. Limited cutaneous systemic sclerosis typically manifests as CREST syndrome, which stands for calcinosis cutis, Raynaud phenomenon, oesophageal dysmotility, sclerodactyly and telangiectasia and is generally anticentromere–antibody positive. The systemic manifestations of systemic sclerosis are diverse. Abnormalities of the circulatory system (most notably Raynaud phenomenon) and involvement of multiple organ systems– such as the musculoskeletal, renal, pulmonary, cardiac and gastrointestinal systems– with fibrotic or vascular complications are most common. Nearly 90% of patients with systemic sclerosis have evidence of gastrointestinal involvement, which is, ultimately, a substantial cause of morbidity. The underlying pathologic change consists of smooth muscle atrophy and fibrosis caused by collagen deposition primarily in the tunica muscularis. Oesophageal involvement typically affects the distal two-thirds of the oesophagus because of the lack of striated muscle in the upper one-third. Findings of oesophageal involvement include decreased or absent oesophageal peristalsis combined with prominent gastroesophageal reflux from an incompetent lower oesophageal sphincter. Oesophagitis is frequently present, and associated complications such as oesophageal stricture or Barrett metaplasia are fairly common. Small bowel findings include hypomotility from smooth muscle atrophy and fibrosis, which leads to stasis, dilatation and pseudo-obstruction. The‘hide-bound’ sign of valvular packing is a fairly specific finding and may be seen in as many as 60% of patients with scleroderma.
1528
A 70-year-old man with a history of a scaphoid fracture several years ago is referred to the orthopaedic clinic for increasing wrist pain. A plain X-ray is ordered to check for developing OA and assess the mid-carpal joints. Measurements based on the true lateral view reveal the capitolunate angle is >30° and the scapholunate angle is >80°. What is your diagnosis? A. Volar intercalated segment instability (VISI) B. Dorsal intercalated segment instability (DISI) C. Refracture of scaphoid D. Perilunate dislocation E. Scapholunate advanced collapse (SLAC) deformity
B. Dorsal intercalated segment instability (DISI) DISI and VISI injuries refer to malalignment of the carpal rows with emphasis of the relation of the lunate to the capitate. In DISI (associated with tear of scapholunate ligament), the lunate is tilted dorsally with an increased scapholunate angle (>60 degrees) and capitolunate angle (>30 degrees). In VISI (associated with tear of the luno-triquetral ligament), there is a volar tilt of the lunate with decreased scapholunate angle (<30 degrees) and increased capitolunate angle (>30 degrees).
1529
A 30-year-old cab driver presents to his GP with malaise, jaundice and abdominal distension. Blood tests performed show deranged liver function tests. A provisional diagnosis of Budd–Chiari syndrome was made. All of the following are imaging features of Budd–Chiari syndrome, except A. Ultrasound demonstrates portal vein enlargement and change in flow dynamics. B. In acute Budd–Chiari, the liver is globally enlarged, with lower attenuation on CT. C. There is caudate lobe atrophy in chronic Budd–Chiari. D. CT shows non-homogenous liver enhancement with a predominantly central area of enhancement and delayed enhancement of the periphery. E. On MRI, the liver is low signal on unenhanced T1 and T2 and delayed enhancement post-contrast.
C. There is caudate lobe atrophy in chronic Budd–Chiari. Budd–Chiari syndrome is a heterogeneous group of disorders characterised by hepatic venous outflow obstruction at the level of the hepatic veins, the IVC, or the right atrium. Budd–Chiari syndrome has variable imaging features. Hepatic vein or IVC thrombosis, with resultant changes in liver morphology and enhancement patterns, venous collaterals, varices, and ascites may be directly observed. Duplex Doppler US is a useful method for detecting Budd–Chiari syndrome because it allows easy assessment of hepatic venous flow and detection of hepatic parenchymal heterogeneity. CT and MR imaging also can depict hepatic venous flow or thrombosis and IVC compression or occlusion. In the presence of acute disease, the imaging features correspond with histologic findings of liver congestion and oedema. The liver is globally enlarged, with lower attenuation on CT images, decreased signal intensity on T1-weighted MRIs, and heterogeneously increased signal intensity on T2-weighted MRIs, predominantly in the periphery. Differential contrast enhancement between the central and peripheral areas of liver parenchyma is a feature of acute Budd–Chiari syndrome. The more oedematous and congested peripheral regions demonstrate decreased contrast enhancement, whereas stronger enhancement is seen in the central parenchyma. After the administration of contrast material, increased enhancement is seen in areas of venous drainage that are less affected, such as the caudate lobe. The development of intra- and extrahepatic collateral veins in subacute Budd–Chiari syndrome permits the egress of venous flow, producing a more homogeneous enhancement pattern with persistent signs of oedema. In chronic Budd–Chiari syndrome, there is atrophy of the affected portions of the liver, and the parenchymal oedema is replaced by fibrosis, which results in decreased T1- and T2-weighted signal intensity at unenhanced MRI and in delayed enhancement in contrast-enhanced studies. Hypertrophy of the caudate lobe, irregularities of the liver contour, and regenerative nodules are prominent features of chronic Budd–Chiari syndrome.
1530
A 1-year-old boy is brought to the A&E department by his parents with head injury after falling off a sofa. The on-call paediatrician strongly suspects non-accidental injury. Which of the following features on unenhanced CT is most consistent with this? A. Bilateral occipital extradural haemorrhage B. Bilateral occipital subdural haemorrhage C. Subarachnoid haemorrhage D. Parietal skull fracture E. Bilateral frontal subdural haemorrhage
B. Bilateral occipital subdural haemorrhage Subdural haemorrhage (SDH) and subarachnoid haemorrhage (SAH) are common abusive injuries. Epidural haematoma is much more often accidental. Probably the most common location of inflicted SAH, and SDH diagnosed radiologically is a layer of hyperattenuating material adjacent to the falx; bleeding at this site represents an interhemispheric extra-axial haemorrhage. In this location, it is often difficult, radiologically, to distinguish SAH from SDH, and SDH and SAH may coexist.
1531
A 40-year-old man undergoes a CT scan of the abdomen for recurrent abdominal pain. The precontrast scan showed bilateral renal calculi. A post-contrast scan showed several pancreatic lesions, measuring between 1 and 2 cm. What is the likely unifying diagnosis? A. MEN I B. MEN II A C. Insulinoma D. Glucagonoma E. NF1
1532
A 45-year-old woman presents with a rapidly enlarging mildly painful breast mass over a period of few months. An urgent ultrasound is performed. The ultrasound shows that the mass measures 7 cm, filling up almost the entire breast with fluid-filled clefts in the tumour. What is the diagnosis? A. Inflammatory carcinoma B. Cystosarcoma phylloides C. Complex breast cyst D. Invasive lobular carcinoma E. Breast lymphoma
B. Cystosarcoma phyllodes Phylloides tumour (PT) is a rare breast fibroepithelial neoplasm. It is now generally accepted that PTs can be classified as benign, borderline or malignant. Mammography and ultrasound are notorious for their inability to distinguish the benign or malignant histologic nature of PTs. On US, they can be indistinguishable from fibroadenoma. They appear as an inhomogeneous, solid-appearing mass. A solid mass containing single or multiple, round or cleft-like cystic spaces and demonstrating posterior acoustic enhancement strongly suggests a diagnosis of PTs. Solid components of the tumour show vascularity on Doppler. On MRI, well-defined margins with a round or lobulated shape and a septate inner structure have been described as characteristic morphologic signs. They are usually low on T1-weighted images and vary from low to very high signal on T2-weighted images. Some have described a silt-like pattern on MRIs of benign PTs; these appear as hyperintense slit-like fluid-filled spaces on T2-weighted images, with a low signal after enhancement. Solid areas of the tumour show enhancement with contrast.
1533
A child presents with intermittent abdominal pain, vomiting, and a right upper-quadrant mass. On clinical examination, blood is noted on rectal examination. A clinical diagnosis of intussusception is made. Where is the most common site of intussusception in this population group? A. Ileoileal B. Ileocolic C. Ileoileocolic D. Colocolic E. Jejunoileal
B. Ileocolic Intussusception is one of the most common causes of acute abdomen in infancy. Intussusception occurs when a portion of the digestive tract becomes telescoped into the adjacent bowel segment. This condition usually occurs in children between 6 months and 2 years of age. In this age group, intussusception is idiopathic in almost all cases. The vast majority of childhood cases of intussusception are ileocolic. US is highly accurate in the diagnosis of intussusception with a sensitivity of 98%–100% and a specificity of 88%–100%. US is also good at demonstrating alternative pathology. Hence, enema could be reserved for therapeutic purposes when US is available.
1534
7. A 29-year-old woman with fever, malaise, fatigue, intermittent pain and numbness in both hands and feet, and normal chest radiograph is referred for MRI thorax. MRI shows wall thickening of the origin of the right subclavian artery and both carotid arteries. What is the diagnosis? A. Moyamoya disease B. Takayasu arteritis C. Churg–Strauss disease D. PAN E. Wegener’s granulomatosis
B. Takayasu arteritis Takayasu arteritis is a form of granulomatous vasculitis affecting large and medium-sized arteries, characterised by ocular disturbances and weak pulses in the upper extremities (pulseless disease). It is associated with fibrous thickening of the aortic arch with narrowing of the origins of the great vessels at the arch. Takayasu arteritis can be limited to the descending thoracic and abdominal aorta. It is seen in young and middle-aged patients, especially Asian and women. The diagnosis is confirmed by a characteristic arteriographic pattern of irregular vessel walls, stenosis, post-stenotic dilatation, aneurysm formation, occlusion and evidence of increased collateral circulation. Polyarteritis nodosa is a fibrinoid necrotising vasculitis that mainly involves small and medium- sized arteries of the muscles. Multiple aneurysm formation is a characteristic finding. The kidney is most commonly involved, followed by the GI tract, liver, spleen and pancreas. Positive ANCA titres (usually pANCA type) are found in variable percentages of patients. Wegener’s granulomatosis is a distinct clinicopathologic entity characterised by granulomatous vasculitis of the upper and lower respiratory tract together with glomerulonephritis. Churg–Strauss syndrome is characterised by granulomatous vasculitis of multiple organ systems, particularly the lung, and involves both arteries and veins as well as pulmonary and systemic vessels. Moyamoya disease is a progressive vasculopathy leading to stenosis of the main intracranial arteries. Characteristic angiographic features of the disease include stenosis or occlusion of the arteries of the circle of Willis, as well as the development of collateral vasculature that produces a typical angiographic image called‘clouds of smoke’ ‘ or puff of cigarette smoke’ .
1535
8. A slimly built 60-year-old woman presents with anorexia, diarrhoea, and weight loss. Barium meal shows multiple filling defects in the stomach with thickened gastric rugae. Colonoscopy shows multiple colonic polyps. The top differential is A. Peutz–Jeghers syndrome B. Familial adenomatous polyposis C. Cronkhite–Canada syndrome D. Cowden syndrome E. Turcot’s syndrome
C. Cronkhite–Canada syndrome Cronkhite–Canada syndrome occurs in older patients with an average age of 60 with no familial predisposition. The histologic appearance of the GI polyps resembles that of juvenile polyps, and they are characteristically distributed throughout the stomach. They are commonly small, sessile and characterised by cystic dilatation of the glands and inflammation of the lamina propria. Patients commonly present with abdominal pain, protein-losing diarrhoea, anorexia and weight loss. Dystrophic nail changes and alopecia usually appear after the onset of GI symptoms.
1536
A 16-year-old boy with progressive extra-pyramidal symptoms, dementia and positive family history was sent for an MRI brain by his neurologist. MRI showed bilaterally symmetric hyperintense signal changes in the anterior medial globus pallidus with surrounding hypointensity in the globus pallidus on T2W images, commonly described as‘eye of the tiger’ sign. Caudate was normal and no other areas of signal change was demonstrated. What is the diagnosis? A. Wilson disease B. Huntington disease C. MELAS D. Hallervorden–Spatz disease E. CADASIL
D. Hallervorden–Spatz disease The‘eye of the tiger’ sign represents marked low signal intensity of the globus palladi on T2-weighted MRI, surrounding a central, small hyperintense area. The sign is seen in what was once known as Hallervorden–Spatz (HS) syndrome but is now called neurodegeneration with brain iron accumulation (NBIA) or pantothenate kinase II (PANC2)-associated neurodegeneration. The low signal is a result of excessive iron accumulation and the central high signal is attributed to gliosis, increased water content and neuronal loss with disintegration. Iron levels in blood and CSF are normal. HS is a neurodegenerative disorder associated with extrapyramidal dysfunction and dementia. The sign can also be seen in other extrapyramidal Parkinsonian disorders such as cortical-basal ganglionic degeneration, Steele–Richardson–Olszewski syndrome, and early onset levodopa-responsive Parkinsonism. High signal in the basal ganglia, thalamus and midbrain is seen in Wilson disease. Caudate is atrophic in Huntington disease. CADASIL shows extensive white matter signal change and MELAS shows multiple focal white matter signal changes.
1537
A 70-year-old man presents with rectal bleeding. Flexible sigmoidoscopy shows a circumferential tumour in the upper third of the anal canal. An MRI performed for staging shows locoregional lymphadenopathy. The lymph node group most likely to be involved is A. Superficial inguinal B. Common iliac C. Pudendal D. External iliac E. Paraortic
A. Superficial inguinal Metastatic spread to regional lymph nodes represents the most common mode of tumour spread from cancer of the anal canal and margin. Nodal metastasis is more likely in cases of larger tumour size or a poorly differentiated anal tumour. Metastasis most commonly occurs to the perirectal nodes, with inguinal nodal spread being the second most common location of nodal metastasis.
1538
A 35-year-old weightlifter presents to the orthopaedic clinic with pain in the right shoulder. An initial radiograph is normal and no abnormality is identified on US. An MRI is suggested for further evaluation; it reveals increased T2W signal changes with fatty atrophy of the teres minor muscle. What is the likely diagnosis? A. Parsonage–Turner syndrome B. Spinoglenoid notch paralabral cyst C. Duchenne’s muscular dystrophy D. Quadrilateral space syndrome E. Acute rotator cuff tear
D. Quadrilateral space syndrome The anatomy of the suprascapular nerve renders it particularly susceptible to compression at the suprascapular notch and spinoglenoid notch. The pattern of muscle denervation provides information about the duration of entrapment and can identify the site of neurologic compromise. Acute denervation presents as hyperintensity of the supraspinatus and infraspinatus or of the infraspinatus muscle alone on fluid-sensitive sequences. Chronic compression is shown as a reduction in muscle bulk and fatty infiltration of the involved muscles. Involvement of both the supra- and infraspinatus muscles reflects proximal compression at the suprascapular notch, whereas isolated infraspinatus denervation suggests compression at the spinoglenoid notch. Quadrilateral space syndrome is a rare condition referring to an isolated compressive neuropathy of the axillary nerve. It generally results in isolated atrophy of the teres minor and, less commonly, of the deltoid, which appears as a reduction in muscle bulk and fatty infiltration with chronic compression. Parsonage–Turner syndrome is an uncommon, self-limiting disorder characterised by sudden onset of non-traumatic shoulder pain associated with progressive weakness of the shoulder girdle musculature. MRI is the technique of choice in patients with shoulder pain and weakness. It is sensitive for the detection of signal abnormalities in the shoulder girdle musculature related to denervation injury. MRI is also useful in excluding intrinsic shoulder abnormalities that can produce symptoms similar to Parsonage–Turner syndrome such as rotator cuff tears, impingement syndrome and labral tears None of the findings or history would be compatible with an acute rotator cuff tear or Duchenne’s muscular dystrophy
1539
A 56-year-old man with a history of PSC has an orthotopic liver transplant. He becomes unwell with a fever and acutely deranged LFTs on Day 4 post-operative. An ultrasound is subsequently performed. What is the likely cause of his symptoms? A. Hepatic vein thrombosis B. Portal vein thrombosis C. Hepatic artery thrombosis D. CBD ligation and cholangitis E. Gangrenous cholecystitis with perforation
C. Hepatic artery thrombosis In the early post-operative period (<72 hours after transplantation), increased hepatic artery resistance (resistive index of >0.8) is a frequent finding, but resistance ordinarily returns to a normal level within a few days. Increased hepatic artery resistance is associated with older donor age and a prolonged period of ischaemia. The estimated incidence of hepatic artery thrombosis among liver transplant recipients is 4%–12% in adults and 42% in children. This is one of the most feared complications, as it may lead to fulminant hepatic necrosis. In addition, in liver grafts, biliary ducts are supplied exclusively by small branches of the hepatic artery; therefore, arterial thrombosis may lead to biliary ischaemia and necrosis. Prompt diagnosis of hepatic artery thrombosis is extremely important because early intervention (with thrombectomy, hepatic artery reconstruction or both) may allow graft salvage. However, most patients ultimately require retransplantation. Even after retransplantation, the mortality rate approaches 30%. Risk factors for hepatic artery thrombosis include a significant difference in hepatic artery calibre between the donor and the recipient, an interpositional conduit for the anastomosis, a previous stenotic lesion of the celiac axis, excessive duration of cold ischaemia time, ABO blood type incompatibility, cytomegalovirus infection and acute rejection.
1540
A 66-year-old woman presents with back pain that radiates down the left anterior thigh towards the medial aspect of the knee. What are the MRI lumbar spine findings that you may expect based on the clinical history? A. L1/2 generalised disc bulge with left lateral recess stenosis B. L1/2 disc bulge with left foraminal stenosis C. L2/3 generalised disc bulge with left lateral recess stenosis D. L2/3 generalised disc bulge with left neural foraminal stenosis E. L3/4 generalised disc bulge with left lateral recess stenosis
C. L2/3 generalised disc bulge with left lateral recess stenosis Medial aspect of the knee corresponds to the L3 dermatome. Lateral recess stenosis at L2/3 will affect the transiting L3 nerve root, whereas foraminal stenosis will affect the exiting L2 nerve root.
1541
B. Normal trachea and anterior oesophageal indentation Aberrant right subclavian artery Aberrant right subclavian artery with a left-sided aortic arch or aberrant left-sided subclavian artery with a right-sided aortic arch both result in posterior impression on the oesophagus on a barium swallow, with normal appearance of the trachea. An aberrant left pulmonary artery or pulmonary vascular sling runs in between the trachea and oesophagus, resulting in an anterior indentation on the oesophagus and a posterior indentation on the trachea. Other entities that can result in anterior indentation on the oesophagus are a bronchogenic cyst, trachea-oesophageal node or a tracheal neoplasm extending posteriorly. Anterior tracheal, posterior tracheal and lateral oesophageal impression occurs with double aortic arch. The right arch is higher than the left, resulting in an S-shaped oesophagogram on AP view. Reverse‘3’ indentation of the oesophagus and normal trachea occurs with coarctation of the aorta.
1542
An 8-week old boy presents with profound cyanosis with associated congestive cardiac failure. Imaging demonstrates a localised concurrent aortic coarctation. The likeliest underlying diagnosis would be A. Tetralogy of Fallot B. Truncus arteriosus C. Transposition of the great arteries D. Hypoplastic left heart syndrome E. Tricuspid atresia
D. Hypoplastic left heart syndrome Hypoplastic left heart syndrome presents with early onset (days) of cyanosis and heart failure, leading to collapse and death in a few weeks of life. Associated cardiac malformations include pre- and post-ductal coarctation of the aorta, PDA, VSD, patent foramen ovale and so on. Truncus arteriosus presents with minimal cyanosis in newborn infants; signs of heart failure are usually absent. Heart failure is evident in older infants. Tetralogy of Fallot presents in early infancy with cyanosis, usually not present in early infancy, leading to clubbing; dyspnoea, heart failure, failure to thrive and paroxysmal hypercyanotic spells. X-ray shows a boot-shaped heart with oligaemic lungs. Transposition of the great arteries is a medical emergency. Infants usually present in the first few hours or days with worsening duct-dependent cyanosis. Hypoxia is severe, but heart failure is not a feature. X-ray shows an‘egg on end’ or ‘egg on string’ appearance. Tricuspid atresia presents in the first few days of life with increasing cyanosis; other clinical features are dependent on associated PDA or VSD.
1543
A child presents with an abnormally shaped cranium. There is hypertelorism and the skull vault appears short and widened. The anterior cranial fossa appears shortened. Which sutures are likely to have fused early? A. Sagittal B. Coronal C. Metopic D. Lambdoid E. Unilateral coronal and lambdoid
B. Coronal The appearance here describes brachycephaly. Craniosynostosis is the premature fusion of cranial sutures and may be isolated or may present as part of a craniofacial syndrome. It typically alters the shape of the cranial vault. Broad categories include simple craniosynostosis, involving only one suture, or compound craniosynostosis, where two or more sutures are involved.
1544
A 47-year-old woman presents to the breast clinic with a palpable lump. Mammography shows a well-circumscribed round mass with mixed dense and radiolucent areas surrounded by a thin radiopaque capsule. Targeted ultrasound performed in the clinic shows a sharply defined, encapsulated, round, heterogeneous mass with echo texture similar to surrounding breast. MRI breast reveals a lesion heterogeneous on T1W and T2W images. What findings on MRI confirm the diagnosis? A. Non-enhancing internal septations B. Slowly enhancing peripheral rim C. Solid homogenous enhancing lesion with well-defined margins D. Peripheral hyperintense cystic spaces on T2W images E. Thin hypointense pseudocapsule and fat content on T1W images
E. Thin hypointense pseudocapsule and fat content on T1W images Fat necrosis appears as well circumscribed with translucent areas in the centre (homogenous fat density of the oil cyst). Occasionally, it shows curvilinear or eggshell calcification in the wall. On US, it appears as a hypoechoic or anechoic mass with ill- or well-defined margins, with or without acoustic shadowing or as a complex cyst. On MRI, even if the lesion appears irregular in both shape and margin, with a rim enhancement, the key to diagnosis is a fat internal signal on unenhanced sequences without fat suppression (high on T1-weighted images).
1545
A 30-year-old woman who is 36 weeks pregnant is being evaluated with targeted ultrasound to investigate a recently noticed breast lump. Ultrasound images are reported to show a well-circumscribed, wider than tall, hypoechoic, solid mass corresponding to the abnormality. Follow-up imaging to assess stability shows progressive regression in size of the mass post-partum. What is the likely diagnosis? A. Fibrocystic change B. Fat necrosis C. Inflamed intramammary node D. Lactating adenoma E. Breast hamartoma
D. Lactating adenoma Lactating adenomas are newly discovered painless lumps that appear during the third trimester or lactation. They are a freely mobile, homogenous hypoechoic or isoechoic mass with posterior acoustic enhancement (most common) and septa. The lesion regresses after breastfeeding.
1546
A 16-year-old girl presents to a gastroenterologist with intermittent episodes of diarrhoea, weight loss and abdominal discomfort over the past 2 years. A full blood count done shows macrocytic anaemia. She is referred for a barium small bowel follow-through examination to assess the small bowel. The barium study reveals several discontinuous ileal strictures with alternate areas of dilatation. The terminal ileum is distinctly abnormal, with ulceration with‘cobblestoning’. What is the most likely diagnosis? A. Ulcerative colitis B. Coeliac disease C. Whipple’s disease D. Crohn’s disease E. Tuberculosis
D. Crohn’s disease In Crohn’s disease involving either the small bowel or colon, broad linear ulcers that crisscross in longitudinal and transverse directions can produce a pseudopolypoid appearance usually termed cobblestoning
1547
Scaphoid waist fracture healing is often complicated by avascular necrosis, non-union, or delayed union. Which of the following is the primary reason for this? A. The blood supply to the distal pole enters at the proximal pole. B. The blood supply of the proximal pole enters at the waist. C. It is a difficult fracture to immobilise. D. Fractures of the scaphoid are difficult to reduce. E. Fractures are often comminuted.
B. The blood supply of the proximal pole enters at the waist. The blood supply to the proximal pole of the scaphoid enters at the waist and courses proximally. A waist fracture can therefore interrupt this tenuous blood supply, leading to avascular necrosis of the proximal pole or delayed/non-union. Non-displaced fractures are often treated conservatively, whereas displaced fractures usually require reduction and internal fixation. The scaphoid bone is the most commonly fractured carpal bone and tends to result from a fall on outstretched hand in a younger population rather than the common dorsally angulated distal radial fractures (Colles fracture). A high index of suspicion should be used when reporting trauma plain films of the wrist, and follow-up imaging following a period of conservative management should be employed when there is clinical concern but unremarkable presentation radiographs.
1548
A 24-year-old man presents to his family doctor with chronic hindfoot pain. Plain radiographs reveal elongation of the anterior dorsal calcaneus on lateral projection of the foot, often described as anteater’s nose. What is the diagnosis? A. Talocalcaneal coalition B. Calcaneonavicular coalition C. Talonavicular coalition D. Calcaneocuboid coalition E. Cubonavicular coalition
B. Calcaneonavicular coalition Tarsal coalition is best confirmed with CT. CT and MRI can show direct bony continuity (osseous coalition) or fluid/cartilage intensity on MR/irregular serrated articular surface in case of cartilaginous coalition. A calcaneonavicular coalition shows the classic anteater’s nose appearance on lateral radiograph. The C sign is seen on a lateral radiograph in a patient with talocalcaneal coalition. Prominent talar beak is also associated with talocalcaneal coalition. Both calcaneonavicular and talocalcaneal coalition are equally common.
1549
All of the following tumours are associated with drop metastasis, except A. Medulloblastoma B. PNET C. Ependymoma D. Melanoma E. Pineocytoma
D. Melanoma Drop metastasis refers to CSF seeding of intracranial neoplasm. Post-contrast images show ‘sugar coating’ of the brain and spinal cord in patients with leptomeningeal drop metastases or leptomeningeal carcinomatosis. It can occur secondary to CNS involvement by distant primary tumours as well as primary CNS neoplasms. CNS tumours with drop metastasis include PNET, medulloblastoma, anaplastic glioma, ependymoma, germinoma, pineoblastoma, pineocytoma and rarely choroid plexus carcinoma and angioblastic meningioma. Non-CNS primaries include breast, lung, melanoma and lymphoma.
1550
Plain radiograph of a 9-month-old baby girl shows a large soft-tissue mass in the pelvis with punctate calcification. MRI reveals a large, lobulated, sharply demarcated tumour with extremely heterogeneous signal on T1W images. What is the diagnosis? A. Anterior sacral meningocoele B. Sacrococcygeal teratoma C. Caudal regression syndrome D. Rhabdomyosarcoma E. Rectal duplication
B. Sacrococcygeal teratoma Sacrococcygeal teratoma is the most common presacral germ cell tumour in children and the most common solid tumour in neonates. The benign form accounts for 60% of all sacrococcygeal teratomas. Benign teratomas are predominantly cystic; have attenuation similar to fluid on CT; and may include bone, fat and calcification. Cystic areas appear low on T1-weighted and high on T2-weighted MRI. Fatty tissue demonstrates high signal intensity on T1-weighted images, whereas calcification is depicted as a signal void. The coccyx is always involved, even in benign sacrococcygeal teratoma, and must be resected with the tumour. Malignant teratomas are more solid, and haemorrhage and necrosis are common. Approximately 50% of benign teratomas contain calcification, whereas it is seldom seen in malignant tumours. Malignant teratomas may metastasise. Anterior sacral meningocoele is a congenital abnormality that arises from herniation of the CSF-filled dura mater through a sacral foramen or a defect in the sacral bone. Eccentric defect in sacrum results in a scimitar appearance on plain film.
1551
A 32-year-old runner presents to the sports clinic with acute exacerbation of right heel pain. He is referred for an MRI study to investigate a potential underlying cause. The MRI reveals a poorly defined hypointense lesion inferior to the calcaneum on both T1W and T2W sequence. A further smaller lesion is noted beneath the first metatarsal head. There is marked contrast enhancement on the post-gadolinium sequences. What is the likely diagnosis? A. Haemangioma B. Mortons neuroma C. Plantar fibromatosis D. Abscess E. Neurofibroma
C. Plantar fibromatosis Plantar fibromas are benign fibrous nodules commonly affecting the medial plantar fascia, with typical anatomical location, ultrasound and MRI findings. Plantar fibromas are well defined and hypoechoic, appearing along the plantar fascia on ultrasound; some are vascular on Doppler. On MRI they are mainly low on T1-weighted and T2-weighted sequences, although sometimes T2-weighted signal is high secondary to an increased cellular component. Variable enhancement post-gadolinium is noted. They are bilateral in 10%–25%, and association wit
1552
An unenhanced CT brain performed on a young man with sudden severe occipital headache shows acute subarachnoid haemorrhage with most of the blood at the foramen magnum. Considering that the source is a ruptured aneurysm, what is the most likely location for the aneurysm in this patient? A. ACOM B. PCOM C. ACA D. AICA E. PICA
E. PICA The location of blood in cases of subarachnoid haemorrhage from a ruptured aneurysm can pinpoint the site of aneurysm in 70% of cases. Anterior chiasmatic cistern ACOM Septum pellucidum ACOM Anterior interhemispheric fissure ACOM Prepontine cistern Basilar artery Foramen magnum PICA Anterior pericallosal cistern ACA, ACOM Sylvian fissure MCA, ICA, PCOM Intraventricular MCA, ICA, ACOM
1553
A 4-year-old girl presents with progressive enlargement of her right thigh, with episodes of unprovoked bleeding from pigmented lesions over her right thigh, which have been present since birth. A lower limb venogram of the right leg demonstrates absence of the deep venous system, with varicose veins on the lateral aspect of the right leg. Which of the following is the most likely diagnosis? A. Klippel–Trénaunay syndrome B. Neurofibromatosis C. Beckwith–Wiedemann syndrome D. Macrodystrophia lipomatosis E. Maffuci syndrome
A. Klippel–Trénaunay syndrome Klippel–Trénaunay syndrome is a sporadic, rare, mesodermal abnormality that usually affects a single lower limb. It is characterised by a triad of a port-wine naevus (unilateral cutaneous capillary haemangioma often in a dermatomal distribution on the affected limb), overgrowth of distal digits/entire extremity (involving soft tissue and bone) and varicose veins on the lateral aspect of the affected limb. Although the other options can produce limb hypertrophy, they would not be expected to show all the features of the triad described.
1554
A 9-year-old girl was taken to her family doctor with fever and painful knee and wrists. The GP noticed a skin rash, hepatosplenomegaly and lymphadenopathy. Plain X-ray of the knee and wrist shows expansion of bones around the knee and advanced carpometacarpal arthritis. What is the likely diagnosis? A. Still disease B. Haemophilia C. Sickle-cell disease D. Psoriasis E. Lyme disease
A. Still disease Still disease is a clinical manifestation of polyarticular juvenile rheumatoid arthritis characterised by fever, rash, hepatosplenomegaly and pericarditis. There is periosteal reaction of the hand phalanges and broadening of bones with cortical thickening. The presence of advance arthropathy in the hands at such a young age along with the other clinical findings would be compatible with this condition. Lyme disease tends to follow a monoarticular pattern with involvement of the large joints, usually the knee, with the radiological findings not as profound as that of juvenile rheumatoid arthritis.
1555
A 7-year-old boy undergoes a plain CT brain for recent trauma, which reveals a cystic lesion in the cerebellum with a mural nodule laterally. No calcification is evident, but enhancement of the mural nodule is seen in post-contrast images. What is the most likely diagnosis in this patient? A. Haemangioblastoma B. Pilocytic astrocytoma C. Giant cell astrocytoma D. Pleomorphic xanthoastrocytoma E. Lymphoma
A. Haemangioblastoma Haemangioblastoma (HB) is a vascular tumour of the CNS. It occurs most often in the cerebellum, where it is the most common primary neoplasm in adults. Single tumours may be sporadic, but multiple tumours are almost always associated with VHL disease. The most common MR pattern of HB is an enhancing solid mural nodule with an adjacent non-enhancing cyst. The cyst is typically low on T1-weighted and high on T2-weighted images, but it can have areas of high T1 signal from fat or haemorrhage. HB can also look purely cystic, solid or a mural nodule with enhancing cystic wall. HB almost never calcifies. Pilocytic astrocytoma are cystic, with larger mural nodule, calcification, thick walls and no contrast blush to the mural nodule on angiography.
1556
All of the following are associations of Chiari II malformation, except A. Dysgenesis of corpus callosum B. Klippel–Feil deformity C. Syringomyelia D. Meningomyelocele E. Tectal beaking
B. Klippel–Feil deformity The hallmarks of Chiari II malformation include caudally displaced fourth ventricle (the fourth ventricle is in normal position in Chiari I malformation), caudally displaced brain stem, and tonsillar or vermian herniation through the foramen magnum. Associations include lumbar myelomeningocele, syringohydromyelia, dysgenesis of the corpus callosum, obstructive hydrocephalous, absent septum pellucidum and excess cortical gyration. It is not associated with any of the bony abnormalities described in Chiari I malformation, like basilar impression, occipitalisation of the atlas, platybasia and Klippel–Feil anomaly.
1557
A 1-year-old infant is admitted with acute stridor. A viral cause is suspected. On AP chest radiography no foreign body is identified, but there is an inverted V appearance of the subglottic trachea. Which of the following is the most likely diagnosis? A. Foreign body B. Acute laryngotracheobronchitis C. Whooping cough D. Tracheobronchomalacia E. Epiglottitis
B. Acute laryngotracheobronchitis Croup (laryngotracheobronchitis) most commonly affects children between 6 months and 3 years and presents with acute stridor, usually following viral infection. A subglottic inverted V sign is seen on plain film, but the epiglottis and aryepiglottic folds are usually normal. In contrast, epiglottitis is a life-threatening condition affecting 3–6-year-olds, with a lateral soft-tissue neck radiograph showing thickening of the epiglottis and aryepiglottic folds described as the‘thumb sign’
1558
A. No Yes No The chest radiographic appearance of fat embolism syndrome is non-specific. Normal radiographs can also be seen. Most patients presenting with a normal initial radiograph develop radiographic- evident abnormalities within 72 hours of injury, and most cases show radiographic resolution within 2 weeks of hospitalisation.
1559
A 25-year-old man presents with progressive increase in knee pain. Plain films show features of osteoarthrosis with increased soft-tissue swelling and density. An MRI shows diffuse low signal on T2W images to the synovium of the knee on the background of large joint effusion. What is the likely diagnosis? A. Amyloid arthropathy B. Haemophilic arthropathy C. PVNS D. Primary synovial osteochondromatosis E. Rapidly destructive articular disease
B. Haemophilic arthropathy Radiographic findings vary greatly with the different stages of haemophilic arthropathy (acute, subacute or chronic haemarthrosis) and reflect the presence of haemarthrosis (joint effusion), synovial inflammation and hyperaemia (osteoporosis and epiphyseal overgrowth), chondral erosions and subchondral resorption (osseous erosions and cysts), cartilaginous denudation (joint space narrowing), intraosseous or subperiosteal haemorrhage (pseudotumours) and osseous proliferation (sclerosis and osteophytosis). Some abnormalities of osseous shape, such as widening of the intercondylar notch, flattening of the condylar surface or squaring of the patella, are very characteristic of chronic haemarthrosis of the knee. At MR imaging, hypertrophied synovial membrane resulting from repetitive haemarthrosis has characteristic low signal intensity with all pulse sequences, especially with gradient-echo sequences, due to the magnetic susceptibility effect caused by haemosiderin. As in pigmented villonodular synovitis, the signal intensity of the subarticular defects varies and may indicate the presence of fluid (high signal intensity on T2-weighted images), soft tissue (intermediate signal intensity) or synovial tissue with haemosiderin (low signal intensity). Rapidly destructive articular disease is an unusual form of osteoarthritis that typically involves the hip. The disease is almost always unilateral, but bilateral lesions and involvement of shoulder have also been reported. Serial radiographs show progressive loss of joint space and loss of subchondral bone in the femoral head and acetabulum, resulting in marked flattening and deformity of the femoral head (‘hatchet’ deformity). Superolateral subluxation of the femoral head or intrusion deformity within the ilium can be observed. Most cases demonstrate subchondral defects and mild sclerosis. However, osteophytes are small or absent.
1560
A 53-year-old woman with chronic renal failure and polycystic renal disease had renal transplantation surgery 5 weeks ago. Initial recovery was uneventful and she is regularly being followed up by the transplant and the renal team. A follow-up graft ultrasound done at 5 weeks post-surgery reveals a large simple fluid collection in relation to the graft. What is the most likely explanation? A. Abscess B. Resolving haematoma C. Lymphocele D. Urinoma E. Seroma
C. Lymphocele Urine leaks and urinomas are relatively rare complications and are usually found in the first 2 weeks post-operative between the transplanted kidney and the bladder. They appear as a well-defined, anechoic fluid collection with no septations that increases in size rapidly. Antegrade pyelography is necessary to provide detailed information about the site of origin of the urinoma and in planning appropriate intervention. Haematomas are common in the immediate post-operative period, but they may also develop spontaneously or as a consequence of trauma or biopsy. At US, haematomas demonstrate a complex appearance. Acute haematomas are echogenic and become less echogenic with time. Older haematomas even appear anechoic, more closely resembling fluid, and septations may develop. Lymphoceles are the most common peritransplant fluid collections that may develop at any time, from weeks to years after transplantation. However, they usually occur within 1–2 months after transplantation. At US, lymphoceles are anechoic and may have septations. Similar to other peritransplant fluid collections, they can become infected and can develop a more complex appearance. Abscesses have a complex, cystic, non-specific appearance at US. Peritransplant abscesses are an uncommon complication and usually develop within the first few weeks after transplantation.
1561
A 5-year-old boy with a large head has widened sutures and Wormian bones on a skull radiograph. Review of other examinations performed earlier shows bilateral hypoplastic clavicles and delayed ossification of symphysis pubis. A chest radiograph shows supernumerary ribs. What is your diagnosis? A. Hypothyroidism B. Primary hyperparathyroidism C. Cleidocranial dysostosis D. Ehler–Danlos syndrome E. Down’s syndrome
C. Cleidocranial dysostosis Cleidocranial dysplasia (CCD) is characterised by aplasia or hypoplasia of the clavicles, characteristic craniofacial malformations, and the presence of numerous supernumerary and unerupted teeth. Cranial abnormalities include wide-open sutures, patent fontanelles and the presence of Wormian bones. Delayed closure of cranial sutures and fontanelles leads to frontal, parietal and occipital bossing. Additionally, there may be poor or absent pneumatisation of paranasal, frontal and mastoid, and sphenoid sinuses. Pelvic features include delayed ossification with wide pubic symphysis, hypoplastic iliac wings, widened sacroiliac joints and a large femoral neck resulting in coxa vara. The differential diagnosis of CCD includes Crane–Heise syndrome (CCD with cleft lip and agenesis of cervical vertebra), mandibuloacral dysplasia (CCD plus hypoplastic mandible), pyknodysostosis (CCD and osteopetrosis), Yunis–Varon syndrome (CCD with hypoplastic thumb and big toe), CDAGS syndrome (craniosynostosis, anal anomalies and genital hyplasia).
1562
A child presents following a witnessed first seizure. An MRI scan is arranged. This shows a well-demarcated, T1 hypointense, T2 hyperintense supratentorial mass with a bright rim on FLAIR. There was no enhancement post-contrast. Which of the following is the most likely diagnosis? A. Ependymoma B. Dysembryoplastic neuroepithelial tumour C. Pilocytic astrocytoma D. Oligodendroglioma E. Ganglioglioma
B. Dysembryoplastic neuroepithelial tumour History of seizure that may be medically refractory makes dysembryoplastic neuroepithelial tumour (DNET) more likely. DNET is a benign, supratentorial and predominantly cortical intra-axial lesion, characterised by a multinodular architecture. Although DNETs are usually located in the temporal lobe, any lobe within the brain lobes may be involved. They have a cortical base and an apex pointing towards the lateral ventricle. They are homogeneously hyperintense on T2-weighted images and hypointense on T1-weighted images. Some delicate septa-like structures are visible within the lesions. Despite their size, neither mass effect nor surrounding parenchymal oedema is present. On FLAIR images, the lesions show a hyperintense ring. Susceptibility-weighted images do not depict any hypointense signal in the lesion, which indicates the absence of calcium or blood products. Very high ADC values are measured inside the mass. No contrast enhancement is noted and there is scalloping of overlying bone. The differential diagnosis includes other brain tumours, such as ganglioglioma (cyst with a strongly enhancing mural nodule, frequent calcification), angiocentric glioma (hyperintense on T1 and star-like extension to ventricle), low-grade astrocytoma (similar to DNET but no scalloping of bone or rim of FLAIR) and pleomorphic xanthoastrocytoma (cyst with mural nodule, enhancement and dural tail).
1563
A 2-year-old girl with headache shows a large cyst in the posterior fossa with communicating hydrocephalus; bone windows reveal scalloping of the petrous pyramids. What is the likely diagnosis? A. Dandy–Walker malformation B. Dandy–Walker variant C. Astrocytoma D. Haemangioblastoma E. Chiari malformation
A. Dandy–Walker malformation The characteristic triad of the Dandy–Walker malformation includes (1) complete or partial agenesis of the vermis, (2) cystic dilatation of the fourth ventricle and (3) an enlarged posterior fossa with upward displacement of the lateral sinuses, tentorium and torcula. The triad is usually associated with hydrocephalus (most common presentation 80%), but this condition should be considered a common complication and not as part of the malformation itself. Depending on the degree of hydrocephalus, the age at diagnosis varies from the neonatal period to later childhood. The skull is enlarged, with characteristic thinning and bulging of the occiput. Pressure from the massively dilated fourth ventricle, along with cerebrospinal fluid pulsations, causes erosive scalloping of the occiput and petrous temporal bones. The cerebellar hemispheres are typically hypoplastic, and in extreme cases only a small nubbin of compressed cerebellar tissue is identified contiguous laterally with the wall of the posterior fossa cyst. Anomalies of the posterior inferior cerebellar arteries, especially absence of the inferior vermian branches and absence of the inferior vermian vein, help in the angiographic differentiation of a Dandy–Walker cyst from an arachnoid cyst, in which the vessels are displaced but present. The Dandy–Walker variant is used to describe a cystic posterior fossa malformation with varying degrees of agenesis of the vermis associated with expansion (often considerable) of the fourth ventricle, which communicates freely with the perimedullary subarachnoid space.
1564
A 56-year-old woman shows an 8-cm solid enhancing mass in the left kidney with a central area of low attenuation on contrast-enhanced CT scan. US study performed earlier showed a large heterogeneous solid mass in the left kidney with a central stellate hypoechoic area. The most likely diagnosis based on the imaging finding would be A. Renal cell carcinoma B. Clear cell carcinoma C. Rhabdomyoma D. Adenoma E. Oncocytoma
E. Oncocytoma Oncocytomas are tubular adenomas with a specific histological appearance. They were previously considered benign but have now been recognised to metastasise. They vary from 1 to 20 cm in diameter and tend to be large. They are usually solitary and unilateral. US shows a solid mass with internal echoes, which occasionally have a stellate hypoechoic centre. CECT demonstrates a well-defined solid mass, with a low-attenuation central scar, when large. Large lesions can extend into and engulf perinephric fat. RCC and oncocytoma look similar on MRI.
1565
A 25-year-old man has injured his knee. Sagittal T2W MRI sequences show a double posterior cruciate ligament (PCL) sign. Which of the following is the most likely injury? A. Ruptured PCL B. Bucket-handle tear of meniscus C. Osteochondral fragmentation D. Radial tear of meniscus E. Anterior cruciate ligament (ACL)
B. Bucket-handle tear of meniscus Bucket-handle tears of the meniscus involve displacement of the free edge of the meniscus into the intercondylar notch. The free edge can be seen adjacent to the PCL on sagittal images giving a double PCL sign. On coronal imaging, the meniscal fragment is displaced medially. A radial meniscal tear manifests as a linear collection of high signal in the meniscus that extends to the superior or inferior articular surfaces. Rupture of the ACL or PCL results in the ligament losing the normal position and morphology but not a double PCL sign. When assessing the PCL, be aware that the ligaments of Humphrey and Wrisberg can give the impression of a tear to the inexperienced eye. These ligaments are extensions of the meniscofemoral ligament; the ligament of Humphrey passes anterior to the PCL and the ligament of Wrisberg posteriorly.
1566
An 11-day-old child presents with duct-dependant cyanosis and congestive cardiac failure. Chest X-ray shows an enlarged heart with a figure of eight pattern and prominent veins. What is the likely diagnosis? A. Transposition of the great vessels (TGA) B. Truncus arteriosus C. Tetralogy of Fallot D. Vein of Galen aneurysm E. Total anomalous pulmonary venous return (TAPVR)
E. Total anomalous pulmonary venous return (TAPVR) TAPVR occurs when the pulmonary veins fail to drain into the left atrium and instead form an aberrant connection with some other cardiovascular structure. On chest radiographs, this cardiovascular anomaly resembles a snowman (figure of eight appearance). In infants affected by TAPVR, cyanosis and congestive heart failure typically develop in the early neonatal period. TA, TGA typically does not present with heart failure. Vein of Galen aneurysm presents with features of heart failure on a chest X-ray but without any specific pattern
1567
A middle-aged man presents with easy fatigability. CT shows an anterior mediastinal mass with areas of calcifications, invading the mediastinal structures. There are multiple small pleural masses. What is the most likely diagnosis? A. Thymoma B. Thymic lipoma C. Lymphoma D. Teratoma E. Asbestosis
A. Thymoma Thymomas are classified as encapsulated, infiltrative and metastasising, with pulmonary and pleural deposits and thymic carcinoma. Half the thymomas are asymptomatic and 30% are associated with myasthenia gravis. At CT a benign thymoma appears round, oval or lobulated. Focal calcification is seen in 25%, which may be dense, irregular or coarse. Benign thymomas show mild homogenous enhancement; cystic changes are also described. Invasive thymoma are heterogeneous in appearance; pericardial and pleural nodules suggest malignancy. Egg-shell calcification is described in invasive thymoma. Absent fat planes between thymoma and mediastinum does not necessarily reflect invasion.
1568
A previously well 29-year-old presents with inversion injury. An X-ray demonstrates no fracture. However, there is a well-circumscribed lesion in the tibia with a ground-glass matrix and a narrow zone of transition. There is no periosteal reaction or associated soft-tissue mass. Which of the following is the most likely diagnosis? A. Polyostotic fibrous dysplasia B. Osteosarcoma C. Osteoid osteoma D. Adamantinoma E. Monostotic fibrous dysplasia
E. Monostotic fibrous dysplasia Fibrous dysplasia is an uncommon, benign disorder characterised by a tumour-like proliferation of fibro-osseous tissue. It may either present as monostotic (affecting one bone) or polyostotic (affecting many bones). Fibrous dysplasia is usually found in the proximal femur, tibia, humerus, ribs and craniofacial bones in decreasing order of incidence. Polyostotic cases can affect multiple adjacent bones or multiple extremities. Men and women are equally affected by the disorder. Fibrous dysplasia is usually asymptomatic, although pain and swelling may accompany the lesion. Radiographically, fibrous dysplasia appears as a well-circumscribed lesion in a long bone with a ground-glass or hazy appearance of the matrix. There is a narrow zone of transition and no periosteal reaction or soft-tissue mass. The lesions are normally located in the metaphysis or diaphysis. There is sometimes focal thinning of the overlying cortex, called‘scalloping from within’. The radiological appearance can also be cystic, pagetoid, or dense and sclerotic. Repeated fractures through lesions in the proximal femur can result in the formation of a so-called shepherd’s crook deformity. The Tc-99m bone scan uptake may be normal or increased. Bone scans are not helpful in diagnosing these lesions but can be useful in identifying asymptomatic lesions. MRI or CT scans can be helpful in delineating the extent of the lesion and identifying possible pathological fractures. Sarcomatous change within the lesion can be identified by MRI or CT.
1569
A 2-year-old boy presents with bowing of the left leg. There is no history of trauma and the right leg appears normal. Standing anteroposterior radiography is performed. This shows a varus deformity of the left knee with fragmentation of the posteromedial tibial metaphysis and absence of the medial epiphysis. The right leg appears radiographically normal and bony density is preserved throughout. Which of the following is the most likely cause? A. Neurofibromatosis B. Blount disease C. Congenital bowing D. Developmental bowing E. Osteogenesis imperfecta
B. Blount disease All of the answers can lead to leg bowing in paediatric groups. However, the description is that of Blount disease, otherwise known as tibia vara. This is a common condition that is unilateral or asymmetrical and is thought to arise as a result of abnormal stress (such as obesity and walking at an early age) on the posteromedial proximal tibial physis. There are three types: infantile (the most common), juvenile and adolescent. Anteroposterior radiography of both legs is necessary. Neurofibromatosis may cause anterolateral tibial bowing, possibly with fibular hypoplasia, and one or both bones may fracture to give a pseudarthrosis. Developmental or physiological bowing is where there is exaggeration of varus angulation between the ages of 12 and 24 months, which again may be caused by obesity or early walking. There is metaphyseal beaking but no fragmentation, and it is usually symmetrical and bilateral. Congenital bowing is usually convex posteromedially (unlike neurofibromatosis). Marked dorsiflexion of the foot is evident at birth, as this condition is thought to arise from an abnormal intrauterine position. Radiography shows thickening of the cortex of the concavity of the curvature. Osteogenesis imperfecta causes bowing of multiple long bones as a result of osteoporotic softening and fractures; hence bone density reduction and bilateral bowing would be more likely.
1570
A 30-year-old man recently treated with bone marrow transplant for acute myeloid leukaemia 12 weeks ago presents with cough and fever. HRCT demonstrates multiple micronodules with areas of consolidation and ground-glass attenuation. What is the likeliest diagnosis? A. Pneumocystis carinii pneumonia B. Cytomegalovirus pneumonitis C. Drug toxicity D. Acute rejection E. Graft versus host disease
B. Cytomegalovirus pneumonitis Early complications include interstitial pneumonitis (infective and non-infective types), infection, oedema, haemorrhage, thromboembolism and calcification. COP is a rare complication that may occur early or late. Cytomegalovirus (CMV) is the most important viral pathogen that causes pneumonia in transplant recipients. RSV, HHV6, pneumocystis jiroveci and adenovirus are less common. Idiopathic interstitial pneumonia has various causes including acute graft-versus- host disease. Unfortunately, CT appearances of both infectious and non-infectious interstitial pneumonitis are non-specific. Notable features include increased interstitial markings, multilobar infiltrates, areas of ground-glass opacity and nodules. Biopsy is frequently undertaken to identify the cause.
1571
Regarding prostatic MRI, which of the following is true? A. B. C. D. E. Zonal anatomy is best seen on T1. The central and transition zones show similar low signal. The transition zone is heterogeneous in the young. The peripheral zone maintains a uniform shape throughout the length of the gland. US is better than MRI for assessing prostate volume.
B. The central and transition zones show similar low signal. On T1-weighted images, the prostate shows homogenous intermediate signal intensity, and the zones cannot be differentiated. The zonal anatomy is best seen on T2-weighted images with the peripheral zone showing a high signal compared to both the central and the transitional zones. The central and transitional zones have similar low signal. At a young age, the transitional zone is homogenous low signal; it gets increasingly heterogeneous with age and BPH changes. On T2-weighted images, the shape of the peripheral zone changes from the base to apex. At the base, the peripheral zone surrounds the posterolateral aspect of the central zone, while at the apex it concentrically surrounds the central zone. The prostatic capsule separates the peripheral zone from the periprostatic tissue, whereas the surgical pseudocapsule separates the peripheral zone from the transitional zone in the older population. MRI is recognised to be more accurate than US and CT in assessment of prostate volume.
1572
A 20-year-old man with a history of thrombophlebitis and recurrent mouth and genital ulcers undergoes a CXR. A well-defined opacity in the right hilum is noted. Contrast-enhanced CT confirms that the opacity has sharp borders, with intense enhancement in the arterial phase. What is the most likely diagnosis? A. Pulmonary artery aneurysm B. Pulmonary vein varix C. Necrotic hilar node D. Endobronchial carcinoid E. Pulmonary AVM
A. Pulmonary artery aneurysm One of the most common findings of Behcet’s disease at chest radiography is a lung mass attributed to a pulmonary artery aneurysm. The pulmonary artery is the second most common site of arterial involvement, with the aorta being the most common, and aneurysms are more common than thromboembolism. Behcet’s disease is the most common cause of pulmonary artery aneurysms. Hughes–Stovin syndrome is a rare disorder of unknown aetiology that is characterised by the combination of multiple pulmonary artery aneurysms with mural thrombi and deep venous thrombosis. Another well-known finding is the mediastinal widening caused by thrombosis of the SVC and accompanying mediastinal oedema. Thrombosis of the brachiocephalic, subclavian and axillary veins may also accompany SVC occlusion. Arterial involvement may occur in the ascending thoracic aorta and the aortic arch, as well as in the coronary artery and subclavian artery in the thorax. Aneurysm formation occurs more frequently than arterial occlusion.
1573
Antenatal ultrasound shows an abnormal facial contour with a large cyst without any cortical mantle of cerebral tissue anteriorly. Septum pellucidum, falx cerebri and optic tracts are not identified with evidence of a fused midline thalamus. A single large ventricle is identified. Normal brain stem, midbrain and cerebellum are noted. What is the diagnosis? A. Alobar holoprosencephaly B. Lobar holoprosencephaly C. Hydranencephaly D. Anencephaly E. Congenital hydrocephalous
A. Alobar holoprosencephaly Holoprosencephaly (HPE) is considered the most common malformation of the brain and face in humans. In alobar HPE, prosencephalic cleavage fails, resulting in a single midline forebrain with a primitive monoventricle often associated with a large dorsal cyst. The olfactory bulbs and tracts, the corpus callosum and anterior commissure, the cavum septum pellucidum and the interhemispheric fissure are absent, whereas the optic nerves may be normal, fused or absent. The basal ganglia, hypothalamic and thalamic nuclei are typically fused in the midline, resulting in absence of the third ventricle. In lobar HPE, the interhemispheric fissure is present along nearly the entire midline, and the thalami are completely or almost completely separated. The corpus callosum may be normal or incomplete, but the cavum septum pellucidum is always absent. Hydranencephaly is the result of a vascular insult (anterior circulation) with the cerebral hemispheres variably replaced by fluid covered with leptomeninges and dura. Falx cerebri is present. The cerebellum, midbrain, thalami, basal ganglia, choroid plexus and portions of the occipital lobes, all fed by the posterior circulation, are typically preserved. It is differentiated from hydrocephalus by absence of an intact rim of cortex (seen with even the most severe hydrocephalus).
1574
A 60-year-old man presents with a long-standing, painless mass in his left thigh, which has been increasing in size over recent months. Plain film demonstrates a soft-tissue mass with poorly defined curvilinear calcification and cortical erosion of the underlying femur. MRI demonstrates an inhomogeneous, poorly defined lesion that is isointense to muscle on T1W images and hyperintense on T2W images. Which of the following is the most likely diagnosis? A. Liposarcoma B. Pleomorphic undifferentiated sarcoma C. Rhabdomyosarcoma D. Lipoma E. Osteomyelitis
B. Pleomorphic undifferentiated sarcoma Pleomorphic undifferentiated sarcoma was previously known as malignant fibrous histiocytoma. Malignant fibrous histiocytoma is the most common soft-tissue sarcoma of late adult life. It typically presents as a painless, soft-tissue mass, which is often located in the thigh and measures 5–10 cm. X-ray demonstrates a non-specific soft-tissue mass with calcification/ossification detected in 5%–20% of patients. Secondary osseous involvement is uncommon. It can be identified as periosteal reaction, cortical erosion and pathological fracture. CT findings include a non-specific, large, lobulated, soft-tissue mass of predominantly muscle density with nodular and peripheral enhancement of solid portions. There are often central areas of low attenuation, which represent myxoid change, old haemorrhage or necrosis. The lesion does not contain fat. MRI typically reveals an intramuscular mass with heterogeneous signal intensity on all pulse sequences. As with other soft-tissue neoplasms, the signal intensity pattern is non-specific, usually low to intermediate on T1-weighted images and intermediate to high on T2-weighted images. Regions of prominent fibrous tissue (high collagen content) may demonstrate low signal intensity on both T1-weighted and T2-weighted images and calcification may present as foci of low signal on both T1-weighted and T2-weighted sequences.
1575
A middle-aged woman with a history of right-sided breast cancer underwent adjuvant radiotherapy following surgery several years ago. She has been symptom free with no features to suggest local recurrence. Over the last few weeks she has developed worsening right-sided chest pain. Chest radiograph shows a partially destroyed right rib with a large eccentric soft-tissue component. Which one of the following is the most likely histology of the lesion? A. Osteosarcoma B. Chondrosarcoma C. Ewing’ s sarcoma D. Angiosarcoma E. Fibrosarcoma
A. Osteosarcoma The most common radiation-induced sarcoma is pleomorphic undifferentiated sarcoma, previously described as malignant fibrous histiocytoma (which is a soft-tissue sarcoma), followed by osteosarcoma and lastly by fibrosarcoma. In this case, the pathology is centred on the bone, making osteosarcoma the most probable diagnosis.
1576
A 43-year-old man with a known diagnosis of AIDS presents with headache, personality change and seizure. Unenhanced CT brain shows a 5 cm, slightly hyperdense lesion in the right frontal lobe with significant peritumoural oedema with little mass effect. Contrast-enhanced MRI brain shows an irregular ring-enhancing lesion in the corresponding location in a sea of oedema. What is your diagnosis? A. Primary CNS lymphoma B. Systemic NHL C. PML D. Toxoplasmosis E. CMV encephalitis
A. Primary CNS lymphoma Toxoplasmosis typically manifests on CT scans and MRIs as nodular (small encephalitis) and/or ring-enhancing (large abscess) lesions within the brain parenchyma. The enhancing ring, when present, may be somewhat thicker and more ill-defined than that seen in association with a typical bacterial abscess. The lesions are associated with surrounding oedema and tend to be multiple at presentation. However, a significant percentage of patients present with solitary lesions. Toxoplasmic lesions are most often seen in the basal ganglia and cerebral hemispheres. On non-enhanced T1-weighted MR images, the lesions are of low signal intensity. On T2-weighted MR images, the lesions are mildly to moderately hyperintense in relation to the brain parenchyma and can be difficult to separate from the surrounding oedema. The presence of small haemorrhages may be a sign of toxoplasmosis, and calcifications can occasionally be seen in treated lesions. On CT scans and MR images, lymphoma most commonly manifests as an enhancing, space-occupying mass with surrounding oedema. However, much of the time the lesions undergo central necrosis and present as ring-enhancing masses; on non-contrast-enhanced T1-weighted images, typical lesions are isointense in relation to brain parenchyma, whereas on T2-weighted images the lesions are isointense to hyperintense. On non-enhanced CT, a small percentage of the lesions are of increased attenuation with respect to the brain parenchyma (as is commonly observed in cases of primary CNS lymphoma in patients who do not have AIDS). The imaging characteristics of lymphoma and toxoplasmosis overlap to such a significant degree that it is nearly impossible to differentiate the lesions on the basis of their appearance on CT scans or MR images alone. Patients with a few solid lesions or ring-enhancing subependymal or periventricular lesions (particularly those with subependymal extension) tend to have lymphomas, whereas patients with multiple ring-enhancing lesions (particularly those that are haemorrhagic) in the basal ganglia and cerebral hemispheres are more likely to have toxoplasmosis.
1577
A 5-year-old girl presents with left-sided abdominal pain. On examination, there is a palpable mass in the left flank. CT demonstrates a well-circumscribed multiseptated cystic renal mass replacing the lower pole of the left kidney. The intervening septae are thick and enhanced post-contrast, and the cysts appear to be herniating into the renal pelvis. What is the most likely diagnosis? A. Multicystic dysplastic kidney B. Multilocular cystic nephroma C. Nephroblastomatosis D. Polycystic kidney disease E. Mesoblastic nephroma
B. Multilocular cystic nephroma Multilocular cystic nephroma is a benign renal tumour that occurs in children and, less commonly, adult women. There is no known association with Wilms tumour. It is usually a unilateral abnormality that replaces an entire renal pole and presents as a large mass, often around 8–10 cm in diameter. Radiological appearances, while not entirely specific, can help to differentiate this lesion from other renal mass lesions. A sharply well-circumscribed, multiseptated cystic mass is typical with a thick surrounding capsule. The cysts appear to herniate into the renal pelvis– an appearance that is relatively specific for mesoblastic nephroma. Unsurprisingly, these lesions are excised, as definitive radiological differentiation from malignancy is often not possible. Multilocular cystic nephroma can be differentiated from multicystic dysplastic kidney by the presence of normal functioning renal parenchyma and symmetrical renal excretion. Polycystic kidney disease involves the entire kidney, unlike multilocular cystic nephroma, which tends to be localised around a renal pole. Multicystic dysplastic kidney is the most common cystic renal disease affecting infants and is twice as common in boys. It is a common cause for abdominal masses in this age group and typically involves one kidney. There are strong associations with a range of genitourinary abnormalities, including vesico-ureteric reflux, horseshoe kidney and ureteric anomalies. The classical ultrasound features are as described above, with near total replacement of the normal renal parenchyma by cysts of varying size and shape. The presence of thin septations can help to differentiate this condition from multilocular cystic nephroma, in which thick septations are typical. Wilms tumour typically presents in children of 3–4 years of age.
1578
A 74-year-old man with orthopnoea, ankle swelling and paroxysmal nocturnal dyspnoea has had US to evaluate right upper-quadrant pain. The scan shows hepatomegaly with a coarse liver. He subsequently has CT of the liver. All of the following are CT findings of hepatic congestion secondary to congestive heart failure, except A. Late enhancement of the IVC and central hepatic veins. B. Heterogeneous mottled mosaic pattern of enhancement on the portal phase. C. Hepatomegaly and ascites may be present. D. Dilated hepatic veins. E. Periportal oedema.
A. Late enhancement of the IVC and central hepatic veins. Passive congestion occurs with the stasis of blood within liver parenchyma as a result of impaired hepatic venous drainage secondary to cardiac disease. Elevated central venous pressure is directly transmitted from the right atrium to the hepatic veins. Passive hepatic congestion may occur with congestive heart failure, constrictive pericarditis, pericardial effusion, cardiomyopathy or right-sided valvular disease involving the tricuspid or pulmonary valve. Symptoms of congestive heart failure mask gastrointestinal symptoms. Patients may present with asymptomatic elevation of liver enzymes, jaundice, right upper-quadrant pain, hepatomegaly and increased abdominal girth. In the arterial phase, there is early enhancement of a dilated IVC and central hepatic veins because of the reflux of contrast material from the right atrium into the IVC. Parenchymal phase images show a heterogeneous, mottled mosaic pattern of enhancement, with linear and curvilinear areas of poor enhancement due to delayed enhancement of small and medium-sized hepatic veins. There may be peripheral large patchy areas of poor delayed enhancement due to stagnant flow within the periphery of the liver. Perivascular lymphedema may be seen as linear low-attenuation regions encircling the intrahepatic IVC or portal veins and should not be confused with venous thrombosis. Hepatomegaly and ascites may be present. Chest images may show cardiomegaly, congestive heart failure and pericardial and pleural effusion.
1579
A 56-year-old man undergoes US of the scrotum to investigate painless swelling of both testes. He does not recall any previous trauma and has been systemically well apart from chronic anaemia, for which he is being investigated by the haematologists. US reveals bilateral solid testicular masses. What is the most likely diagnosis? A. Metastases from renal cell carcinoma B. Leukaemia C. Seminoma D. Fractures E. Lymphoma
E. Lymphoma Lymphoma can occur in the testis in one of three ways: as the primary site, as the initial manifestation of occult disease or as the site of recurrence. It is the most common bilateral tumour, and the epididymis and spermatic cord are commonly involved. The sonographic appearance of testicular lymphoma is variable and indistinguishable from that of germ cell tumours. Testicular lymphoma generally appears as discrete hypoechoic lesions, which may completely infiltrate the testicle. Primary leukaemia of the testis is rare. However, the testis is a common site of leukaemia recurrence in children. Seminoma is the most common pure germ cell tumour but affects unilateral testis. Metastases are rare but are reported most commonly in cases of primary prostate and lung cancer. Testicular fracture usually appears as a linear hypoechoic band extending across the testicular parenchyma.
1580
A 23-year-old woman with multiple skin lesions presents to the ophthalmologist with worsening of vision. MRI brain and orbits reveal a thickened enhancing right optic nerve with further focal areas of T2W hyperintensities in the brainstem and basal ganglia. What is the diagnosis? A. NF2– optic tract meningioma B. TS– optic neuritis C. MS– optic neuritis D. NF1– optic glioma E. VHL– optic haemangioblastoma
D. NF1– Optic glioma NF1 is also called peripheral neurofibromatosis or von Recklinghausen disease. The classic triad includes cutaneous lesions, skeletal abnormalities and mental deficiency. CNS lesions include optic pathway glioma (thickened enhancing optic nerve), cerebral gliomas, hydrocephalus due to aqueduct stenosis, vascular dysplasia including Moyamoya disease, cranial nerve neurofibromas or craniofacial plexiform neurofibromatosis, CNS hamartomas (unidentified bright objects on T2-weighted MRI) and vacuolar/spongiotic myelinopathy. Spinal lesions include cord neurofibroma or neurofibroma of peripheral nerves. Optic nerve meningioma, seen as ring enhancement in cross section, would be suggestive of NF2 or central neurofibromatosis, classically associated with bilateral vestibular/acoustic schwannomas and ependymomas. Optic neuritis would also show nerve swelling, high signal on T2 and enhancement post-contrast on T1-weighted FS images; however, enlargement of the nerve on imaging may not be as pronounced as in glioma, and MS is not associated with skin lesions. Note that tuberous sclerosis can cause skin lesions but is associated with optic nerve hamartoma rather than glioma or optic neuritis.
1581
A 7-month-old baby is seen at the general paediatric clinic with failure to thrive and irritability. Few clinical signs are present on examination, although the child cries on manipulation of his lower limbs. A plain radiograph of the left leg was performed a few weeks ago for a suspected fracture, but no acute bony injury was identified. The doctor reviews this radiograph again. It shows generalised osteopaenia of the distal femur and proximal tibia and fibula. A sclerotic rim surrounds the distal femoral epiphysis, which itself is abnormally lucent. A bony spur can be seen arising from the distal femoral metaphysis. What is the most likely diagnosis? A. Hypoparathyroidism B. Scurvy C. Rickets D. Hypothyroidism E. Hypophosphatasia
B. Scurvy Scurvy is a dietary deficiency of vitamin C. It typically affects babies from 6 to 9 months and is characterised by non-specific symptoms of irritability, lower limb tenderness and reluctance to move legs normally. Bleeding gums can also occur. The condition usually manifests at the distal femur and proximal and distal ends of the tibia and fibula, and it can also affect the upper limb bones and the ribs. There are several key radiographic findings: bony spurs arising from the metaphysis of long bones are eponymously termed Pelkan spurs. Wimberger line refers to the appearance of a sclerotic line running around the perimeter of the epiphyses, which reflects osteopaenic change. Ground-glass osteoporosis and cortical thinning are also recognised findings. In contrast, rickets manifests as cupping and fraying of the metaphyses, periosteal reaction, bowing of the long bones and widening of the growth plates. Hypophosphatasia is often indistinguishable from rickets on imaging. Hypothyroidism in infancy also results in osteopaenia, with fragmentation of the epiphyses
1582
A 43-year-old patient with known diagnosis of AIDS presented with ataxia and progressive neurological deficits. MRI brain revealed patchy high signal on T2W images in the parieto-occipital white matter. No mass effect or contrast enhancement was evident. What is the diagnosis? A. Primary CNS lymphoma B. AIDS dementia complex C. Progressive multifocal leukoencephalopathy D. Periventricular leukomalacia E. Encephalitis
C. Progressive multifocal leukoencephalopathy The most common imaging manifestation of HIV infection is global atrophy that is out of proportion to age. The finding of diffuse cerebral atrophy can be accompanied by diffuse, confluent, ill-defined areas of abnormally increased signal intensity on T2-weighted MRI of the periventricular white matter. No enhancement is noted. The findings may be accompanied by encephalopathy (AIDS dementia complex). These global abnormalities were previously attributed to a subacute encephalitis caused by HIV. It now seems likely that both HIV and CMV can cause subacute encephalitis and encephalopathy; these conditions have an identical non-specific imaging appearance. CMV encephalopathy appears to manifest itself late in the illness, whereas HIV dementia (although usually presenting late) can occasionally be the AIDS-defining illness. Progressive multifocal leukoencephalopathy is the most serious focal lesion without significant mass effect. It is caused by the JC virus. On CT, the lesions have low attenuation. On MRI, they are of low signal intensity on TI-weighted images and high signal intensity on T2-weighted images. They exhibit little or no mass effect. Although ring enhancement has been reported, these lesions generally do not enhance on CT or MRI. They often occur at the interface between the grey matter and the white matter and have a scalloped contour secondary to involvement of peripheral U-fibres. The parietal lobe is predominantly affected, but these lesions also occur in the periventricular white matter, posterior fossa, brainstem, spinal cord and even the basal ganglia.
1583
A 6-year-old with spina bifida has a chest X-ray performed for possible lower respiratory tract infection. The lungs are clear but there is a well-defined, round paraspinal mass with an air–fluid level. What is the most likely diagnosis? A. Bronchogenic cyst B. Morgagni hernia C. Oesophageal duplication cyst D. Cystic teratoma E. Oesophageal tumour
C. Oesophageal duplication cyst Oesophageal duplication cysts are rare congenital anomalies. They are associated with vertebral anomalies (spina bifida, hemivertebrae, fusion defects). There is also an association with oesophageal atresia and small bowel duplication. Most cysts develop in the right posteroinferior mediastinum. CT demonstrates a well-marginated round, oval or tubular-shaped fluid-filled cystic structure that has a well-defined, thin wall. The cyst is of water attenuation with no enhancement of contents and no infiltration of surrounding structures. Malignant degeneration is rare. Bronchogenic cyst is the most common cystic mediastinal mass that typically lies in the middle mediastinum, not in a paraspinal location; in addition, you would not expect an air–fluid level. Cystic teratoma is an anterior mediastinal mass. Morgagni hernia would be unlikely to cause a solitary round lesion; multiple structures would be expected.
1584
Enhanced CT scan of an otherwise healthy motor vehicle accident victim demonstrates no enhancement of the right kidney on Day 0. A repeat contrast-enhanced CT scan obtained on Day 3 demonstrates a thin marginal rim of subcapsular enhancement of uniform thickness, described as the rim sign. The collecting system was mildly prominent. What is the most likely explanation? A. Analgesic overuse B. Renovascular compromise C. Diabetes mellitus D. Pyelonephritis E. Developing hydronephrosis
B. Renovascular compromise The rim sign is associated with major vascular compromise in the kidney. This sign is most commonly seen with renal artery obstruction from thrombosis, embolus or dissection. On contrast-enhanced CT or MRI, a 1- to 3-mm rim of subcapsular enhancement, paralleling the renal margin, can be seen as a result of preserved perfusion of the outer renal cortex by capsular perforating vessels. The finding may be accompanied by an abrupt termination of contrast material in the renal artery, referred to as the arterial cut-off sign. The rim sign of vascular compromise has also been described with renal vein thrombosis and acute tubular necrosis.
1585
A male infant is born at 39 + 3 weeks gestation. Prenatal ultrasound demonstrated a partly cystic, partly echogenic mass in the right upper lobe. Shortly after delivery the infant is in respiratory distress. Initial chest X-ray demonstrates dense lungs bilaterally with increased volume on the right. On Day 2, a repeat chest X-ray demonstrates multiple air-filled cystic masses of varying sizes within the right upper lobe with mediastinal shift to the left. What is the most likely diagnosis? A. Bronchogenic cyst B. Morgagni hernia C. Congenital cystic adenomatoid malformation D. Congenital lobar emphysema E. Hyaline membrane disease
C. Congenital cystic adenomatoid malformation Congenital cystic adenomatoid malformation is a developmental hamartomatous abnormality of lung with adenomatoid proliferation of cysts resembling bronchioles. It is thought to be caused by focal arrest in foetal lung development before the seventh week of gestation. Congenital cystic adenomatoid malformation represents 25% of all congenital lung lesions. A CXR on Day 1 of life usually demonstrates dense lungs with increased volume on the affected side. On Day 2, a CXR usually demonstrates resorption of fluid from affected areas of lung, which are then replaced with air-containing spaces. Communication with the tracheobronchial tree is maintained and the vascular supply and drainage are to the pulmonary circulation. There is a slight predilection for the upper lobes. Newborns often present with respiratory distress secondary to mass effect and pulmonary compression or hypoplasia. The chest is dull to percussion with decreased air entry. Prenatal ultrasound shows a partly cystic, partly echogenic mass.
1586
A 44-year-old man presents with increasingly severe and disabling pain in the left inguinal and anterior thigh region. The pain is exacerbated by weight-bearing and relieved by rest. The patient mentions that the pain began 2 months ago, was acute in onset, and there was no preceding trauma. A radiograph of his left hip reveals a focal osteopaenic region within the left femoral head; this is no longer evident on a follow-up film 8 months later. Which of the following is the most likely diagnosis? A. Osteomalacia B. Avascular necrosis C. Transient osteonecrosis D. Transient osteoporosis E. Occult fracture
D. Transient osteoporosis Transient osteoporosis is a rare, self-limiting condition that usually affects the hip. Classically, it is characterised by disabling pain in the hip without preceding trauma, and there is radiographic evidence of a focal region of osteopenia isolated to the hip. Although avascular necrosis can also present with a focal region of osteopenia in the early stages of the disease, transient osteoporosis resolves in 6–8 months, whereas avascular necrosis is usually progressive. It is thought that transient osteoporosis could represent a non-traumatic form of Sudeck atrophy or reflex sympathetic dystrophy.
1587
A 42-year-old factory worker complains of chest tightness and shortness of breath during the early days of the week, settling down during the weekend over the last several months. Chest radiograph is normal. HRCT is requested for further evaluation. What do you expect the HRCT to show? A. Crazy paving pattern B. Patchy ground-glass opacities with centrilobular nodules C. Perilymphatic nodules with beaded fissures D. Central bronchiectasis E. Extensive mediastinal and hilar lymphadenopathy
B. Patchy ground-glass opacities with centrilobular nodules In acute hypersensitive pneumonitis, symptoms may begin after patients return to an environment from which they have been absent for a while (e.g., resuming work following weekends or holidays). Chest radiographs obtained in many patients with hypersensitivity pneumonitis are normal. HRCT typically shows patchy ground-glass opacities and centrilobular nodules. Respiratory bronchiolitis–interstitial lung disease is a smoking-related lung disease that has similar imaging features. Perilymphatic nodules and beaded features are features of sarcoidosis, while central bronchiectasis is a typical feature of ABPA. The crazy paving pattern is seen in pulmonary alveolar proteinosis but can also be seen in mucinous broncho-alveolar carcinoma, exogenous lipoid pneumonia, sarcoidosis, NSIP, pneumocystis pneumonia and several other diffuse acute conditions.
1588
An 8-year-old child with right upper-abdominal pain and a palpable mass was sent for US of the liver. The US showed a large heterogeneous mass occupying at least half of the right lobe of the liver. The rest of the liver was normal in appearance. Which one of the following is the most common benign primary liver tumour in children? A. Hepatocellular carcinoma B. Hepatoblastoma C. Angiosarcoma D. Infantile haemangioendothelioma E. Cholangiocarcinoma
D. Infantile haemangioendothelioma Although primary hepatic neoplasms represent only a small percentage of solid tumours that occur in children, the finding of focal hepatic lesions in a child is not an uncommon event in a busy radiology practice. The most common neoplasm involving the liver in children, as in adults, is metastatic disease. Most primary liver tumours in children are malignant, but one-third are benign; benign lesions may be of mesenchymal or epithelial origin. The most common benign tumours are, in decreasing order of frequency, infantile haemangioendothelioma, FNH, mesenchymal hamartoma, nodular regenerative hyperplasia (NRH) and hepatocellular adenoma.
1589
A 37-year-old woman with asthma underwent HRCT of the chest and was diagnosed based on clinical examination, blood tests and HRCT to have Churg–Strauss syndrome or allergic granulomatosis. Which of the following appearances would fit with the diagnosis? A. Peripheral predominant lobular consolidation with centrilobular nodules B. Peripheral predominant homogenous consolidation without centrilobular nodules C. Peripheral predominant interstitial reticular opacities with underlying traction bronchiectasis D. Central bronchiectasis and patchy airspace opacities with mucous plugs E. Upper lobe predominant interstitial pulmonary fibrosis with architectural distortion
A. Peripheral predominant lobular consolidation with centrilobular nodules. On thin-section CT, the parenchymal abnormal findings of CSS could be classified into three patterns. The first pattern is subpleural consolidation with lobular distribution. The second pattern is centrilobular perivascular densities, diffusely scattered centrilobular nodules <5 mm in diameter, especially within the ground-glass opacity lesion. The third pattern is multiple larger nodules. Bronchial wall thickening, with or without bronchial dilatation, and hyperinflation are likely to be related to asthma. Interlobular septal thickening may reflect the interstitial pulmonary oedema attributed to the cardiac and pericardial involvement. Peripheral predominant homogenous consolidation is a feature of chronic pulmonary eosinophilia. Nodules are not a feature of chronic pulmonary eosinophilia. Central bronchiectasis and mucous plugs are a feature of ABPA.
1590
A young adult man presents with a painful left wrist following a fall from height. A lateral view of the wrist shows loss of co-linearity of the radius/lunate/capitate axis with the capitate displaced dorsally. What injury is this constellation of findings compatible with? A. Midcarpal dislocation B. Perilunate dislocation C. Lunate dislocation D. Volar intercalated segmental instability E. Dorsal intercalated segmental instability
B. Perilunate dislocation Lunate and perilunate dislocations and fracture dislocations may be easily overlooked on plain films and should be actively sought and excluded given an appropriate clinical history. On the AP view, the lines along the proximal poles of both the proximal and distal carpal rows should be smooth and uninterrupted. The lunate should appear trapezoid and not triangular in shape (AP radiograph), which may indicate dislocation and rotation. On the lateral view, the distal radius, lunate and capitate should be in alignment. Volar loss of lunate alignment is seen in lunate dislocation, and dorsal loss of capitate alignment is seen in perilunate dislocation. In addition, the scapholunate (normal 30–60 degrees) and the capitolunate (normal 0–30 degrees) should be assessed for evidence of intercalated segmental instability.
1591
An 88-year-old man with obstructive uropathy and hard nodular prostate gland on digital rectal examination is being further investigated for his prostate cancer. He has been scheduled for a staging MRI of the prostate. Which one of the following is the characteristic imaging finding of prostate cancer on DWI? A. High on DWI, low on ADC and corresponding low signal intensity on T2W MRI B. Low on DWI, high on ADC and corresponding low signal intensity on T2W MRI C. High on DWI, low on ADC and corresponding high signal intensity on T2W MRI D. Low on DWI, high on ADC and corresponding high signal intensity on T2W MRI E. High on DWI, low on ADC and corresponding normal signal intensity on T2W MRI
A. High on DWI, low on ADC and corresponding low signal intensity on T2W MRI T2-weighted images are the‘workhorse’ images for prostate cancer. The glandular peripheral zone appears high in signal, whereas the central stroma has lower signal intensity on T2-weighted images. Tumours are lower in signal intensity than normal peripheral zone glandular tissue on T2-weighted images. Peripheral zone cancers are usually round or ill-defined. Various conditions, such as prostatitis, haemorrhage, atrophy, benign prostatic hyperplasia (BPH) and post-treatment changes can mimic cancer. Cancers in the central gland are even more challenging to detect than peripheral zone cancers, because the signal characteristics of the normal and hypertrophic central gland are usually similar to those of the tumour on T2-weighted images. Prostate cancers often include tightly packed glandular elements with increased cellular density and diminished extracellular spaces, which can be detected as high-signal-intensity foci on raw DW-MRI (restricted diffusion) but are low in signal on apparent diffusion coefficient (ADC) maps. ADC maps reflect the amount of diffusion present– the lower the diffusion, the darker the lesion.
1592
A 50-year-old patient presents with shortness of breath to his GP. Following abnormalities on the chest radiograph, a CT is performed, demonstrating bilateral hilar and right paratracheal lymphadenopathy along with coarse reticulations and nodular thickening along the fissures and bronchovascular bundles. Elevated ACE serum levels are noted. What is not a typical musculoskeletal manifestation of this disease? A. Lacy lytic lesions in the phalanges of the hands B. Polyarthralgia with tenosynovitis C. Multiple bony lesions with high T2W and intermediate T1W signal characteristics D. Soft-tissue calcifications E. Dactylitis
D. Soft-tissue calcifications Acute arthritis may be the first manifestation of sarcoidosis. It is mainly oligoarticular but occasionally polyarticular, and rarely monoarticular. Ankles are the most commonly involved joints. Enthesitis, tendinosis and tenosynovitis are well recognised, especially around the ankle. The triad of acute arthritis, bilateral hilar adenopathy and erythema nodosum is known as Lofgren syndrome. Chronic arthritis tends to involve the shoulders, hands, wrists, ankles and knees. Dactylitis can be seen; it is very similar to that seen in patients with psoriatic arthritis. Osseous involvement causes cystic, reticular or destructive lesions involving mainly the hands and feet (lace-like pattern) but can affect the skull, ribs, sternum, vertebrae, nasal bones, pelvis, tibia and femur as well. MRI shows multiple bony lesions with high T2-weighted and intermediate T1-weighted signal, although diffuse involvement is also recognised. Sarcoid myopathy can also be acute and mimic polymyositis.
1593
A 77-year-old man with obstructive uropathy and TRUS biopsy–proven prostate cancer was scheduled to have an MRI for local staging of the known prostate cancer. Which one of the following statements regarding identification of prostate cancer by dynamic contrast enhancement (DCE) is true? A. Fast enhancement and slow washout suggest prostate cancer. B. Slow enhancement peak and slow washout suggest prostate cancer. C. Slow enhancement and fast washout are highly suggestive of prostate cancer. D. Small prostate tumours are detected easily. E. The higher the tumour grade, the greater the grade of enhancement.
E. The higher the tumour grade, the greater the grade of enhancement DCE-MRI evaluates the vascularity of tumours by providing quantitative kinetic parameters that reflect the flow of blood and the permeability of the vessels. DCE-MRI increases the specificity of prostate tumour detection. Tumours show early enhancement and early washout of the contrast agent– the higher the tumour grade, the higher would be the permeability parameters (Ktrans – wash-in, Kep– washout). A disadvantage of DCE-MRI is that small, low-grade tumours may not demonstrate abnormal enhancement. Furthermore, abnormal enhancement patterns can also be seen in patients with BPH, which can make assessment of the central gland difficult. However, in the glandular peripheral zone and anterior gland, DCE-MRI can be helpful in identifying lesions that are not suspected on T2-weighted images. The degree of enhancement on DCE-MRI correlates positively with tumour grade.
1594
An 81-year-old man with progressive worsening hesitancy, dribbling, incomplete voiding and haematuria was found to have a hard nodular prostate gland on digital rectal examination. He is due to have a staging prostatic MRI, which will take place shortly after TRUS biopsy. Which one of the following is characteristic of post-biopsy haemorrhage of the prostate gland on MR imaging? A. B. C. Hyperintense on T1W imaging and hypointense on T2W imaging Hypointense on T1W imaging and hypointense on T2W imaging Hypointense on T1W imaging and hyperintense on T2W imaging D. Hyperintense on T1W imaging and hyperintense on T2W imaging E. Hyperintense on T1W imaging and isointense on T2W imaging
A. Hyperintense on T1W imaging and hypointense on T2W imaging T1-weighted images are important in prostate cancer staging, because they show the presence of haemorrhage secondary to a recent biopsy; haemorrhage is almost always hyperintense compared with normal parenchyma on T1-weighted MRI. Haemorrhage typically appears low in signal on T2-weighted MRI and mimics cancer.
1595
A 52-year-old man has a lower-leg radiograph performed after a fall. No acute bony injury is identified, but the reviewing doctor notices a flame-shaped lucency involving the medulla of the tibia, extending superiorly from the distal subarticular into the diaphysis. He is unsure of the significance of this appearance and seeks the opinion of the radiologist. Which of the following is the most likely diagnosis? A. Osteomyelitis B. Giant cell tumour C. Plasmacytoma D. Paget disease (acute phase) E. Paget disease (chronic phase)
D. Paget disease (acute phase) Paget disease is a multifocal skeletal condition whereby bone remodelling becomes abnormal and eventually exaggerated. It typically affects middle-aged to elderly people and has acute and chronic phases with differing radiographic appearances. The acute phase, also known as the active or osteolytic phase, is characterised by aggressive bone resorption secondary to disordered osteoclastic activity. This manifests as lucency within the medulla of the long bones, typically in a flame shape or ‘blade of grass’ configuration. Over time, osteoblastic activity overtakes the osteoclastic resorption, resulting in osteosclerosis, cortical thickening and increased trabeculation. Giant cell tumour is a benign neoplasm of the mature skeleton, seen almost exclusively in those under 30 years of age. Although osteomyelitis can result in bony lucency, it is typically patchy with a moth-eaten texture.
1596
A previously fit and healthy 16-year-old presents with recurrent wheeze. Chest X-ray demonstrates tubular, branching structures of increased density relative to normal lung radiating from the left hilum. The surrounding lung is hyperlucent. What is the most likely diagnosis? A. Allergic bronchopulmonary aspergillosis B. Cystic fibrosis C. Bronchial atresia D. Kartagener syndrome E. Swyer–James syndrome
C. Bronchial atresia This is the classic description of a mucocoele. Congenital bronchial atresia is a congenital abnormality caused by focal interruption of a lobar, segmental or subsegmental bronchus with associated peripheral mucous impaction (bronchocoele/mucocoele) and associated hyperinflation of the obstructed lung segment. The apicoposterior segmental bronchus of the left upper lobe is most commonly affected. The condition is usually benign and asymptomatic and is often an incidental chest X-ray finding. Mean age at diagnosis is 17 years and symptoms, if present, include recurrent pulmonary infection, mild wheeze and dyspnoea. The typical appearance of Swyer–James syndrome is that of a small, hyperlucent lung, with overexpansion of the contralateral lung. A diffuse pattern of scarring or irregular vessels may also be present. Cystic fibrosis is unlikely in a previously well individual. Kartagener syndrome appears as bilateral bronchiectasis. Allergic bronchopulmonary aspergillosis is a possibility, but the minor symptoms fit best with bronchial atresia.
1597
A 37-year-old woman in her third trimester presented to the labour ward with acute onset of pain in the lower central abdomen. She had had two previous caesarean sections and an appendectomy with unremarkable recovery. On clinical examination, there was definite tenderness at the site of a previous caesarean scar. The best imaging modality for evaluation of caesarean section scar dehiscence is A. CT B. Transabdominal US C. MRI D. Angiography E. Transvaginal US
C. MRI Uterine dehiscence is characterised by incomplete rupture of the uterine wall, usually involving the endometrium and myometrium but with an intact overlying serosal layer. MR imaging may be better than CT in evaluating for uterine dehiscence because of its multiplanar capability and greater soft-tissue contrast and its ability to help identify an intact serosal layer
1598
A previously hypertensive 34-year-old post-partum woman who presented with right upper-quadrant pain, pelvic pain and hypotension was sent for an urgent CT. The CT scan showed a large, subcapsular liver hematoma and hemiperitoneum within the abdomen and pelvis. No liver lesions were seen on a prepartum US. What is the diagnosis? A. Haemorrhagic adenoma B. HELLP syndrome C. Iatrogenic injury at caesarean section D. Acute hepatic steatosis of pregnancy E. Fitz Hugh–Curtis syndrome
B. HELLP syndrome HELLP (haemolysis, elevated liver enzymes, low platelets) syndrome is one of the hypertensive disorders of pregnancy, occurring in 4%–12% of pre-eclamptic patients, from vascular endothelial injury, which results in intravascular deposition of fibrin with end organ damage and can occur prior to or after delivery. Disseminated intravascular coagulation (DIC) is seen in 20%–40% of patients. Other major complications include placental abruption, acute renal failure, pulmonary oedema, pleural and pericardial effusions, hepatic infarction, haematoma and rupture. Pelvic pain may occur in the setting of haemoperitoneum. Spontaneous intrahepatic haemorrhage and rupture likely result from toxaemia-related vasculopathy with endothelial damage, leading to haemorrhage. Supportive therapy is usually offered initially, with capsular rupture necessitating surgery. Embolisation of the hepatic artery may be attempted. CT is important in initial diagnosis and for serial follow-up. Findings may include subcapsular or intrahepatic haemorrhage, capsular rupture and areas of confluent necrosis secondary to infarction. Intravenous contrast-enhanced CT allows identification of active arterial extravasation, which cannot be seen on US. Fitz-Hugh–Curtis syndrome refers to the development of perihepatitis in association with pelvic inflammatory disease.
1599
Urgent MRI done on a patient who had recently undergone caesarean delivery showed a large (>5 cm) intermediate- to high-signal-intensity haematoma at the lower uterine incision site that communicates with the endometrium on sagittal fat-saturated T2W images. The uterine serosa was intact. What is the diagnosis? A. Uterine dehiscence B. Uterine rupture C. Bladder flap haematoma D. Subfascial haematoma E. Endometritis
A. Uterine dehiscence Uterine dehiscence is characterised by incomplete rupture of the uterine wall, usually involving the endometrium and myometrium but with an intact overlying serosal layer. Uterine dehiscence is a very difficult imaging diagnosis. The presence of a bladder flap haematoma greater than 5 cm and larger pelvic haematomas should be considered abnormal and highly suspicious for uterine dehiscence in the proper clinical setting. MR imaging may be better than CT in checking for uterine dehiscence because of its multiplanar capability and greater soft-tissue contrast, with its ability to help identify an intact serosal layer. Uterine rupture is the most severe potential complication of caesarean delivery and is defined as separation of all layers of the uterine wall, including the serosal layer, with abnormal communication between the uterine cavity and the peritoneal cavity. The presence of gas/blood within the uterine defect extending from the endometrial cavity to the extrauterine parametrium in association with haemoperitoneum increases the likelihood of rupture in the appropriate clinical setting.
1600
A 66-year-old woman with discomfort in the epigastric region was sent for US of the upper abdomen. GB and bile ducts were normal but an incidental mass was identified in the pancreas. A dual-phase CT was requested for further characterisation. The CT showed a large, lobulated, multicystic lesion with a central scar, enhancing septae and some stellate calcification. Which one of the following is the most likely diagnosis? A. Solid pseudopapillary tumour B. Intraductal papillary mucinous neoplasm C. Mucinous cystic neoplasm D. Serous cystadenoma E. Gastrinoma
D. Serous cystadenoma In 70% of cases, Serous cystadenomas demonstrate a polycystic or microcystic pattern consisting of a collection of cysts (usually more than six) that range from a few millimetres up to 2 cm in size. Fine, external lobulations are a common feature, and enhancement of septa and the cyst wall may be seen. A fibrous central scar with or without a characteristic stellate pattern of calcification is seen in 30% of cases and, when demonstrated with CT or MR imaging, is highly specific and is considered to be virtually pathognomonic for serous cystadenoma. Pancreatic ductal dilatation is an uncommon finding in these patients. The macrocystic or oligocystic variant of these tumours is very uncommon and is seen in less than 10% of cases. Either of these variants can take the form of a single dominant macrocavity, in which case it will appear as a unilocular cyst or may contain fewer large (2 cm) cysts. The latter variant is classified as a macrocystic lesion and may be difficult to differentiate from a mucinous cystic tumour. Mucinous cystic neoplasms (mucinous cystadenomas) mainly involve the body and tail of the pancreas and do not communicate with the pancreatic duct. At CT/MR, they are multilocular macrocystic lesions occasionally containing debris or hemorrhage. The complex internal architecture of the cyst, including septa and an internal wall, is best appreciated at MR imaging and endoscopic US, allowing differentiation from serous cystadenomas. Although peripheral eggshell calcification is not frequently seen at CT, it is specific for a mucinous cystic neoplasm and is highly predictive of malignancy. Pseudopapillary tumour is a solid tumour with a cystic component. Others in this group are islet cell tumour, pancreatic adenocarcinoma, and metastasis.
1601
A middle-aged man undergoes CT of the abdomen for chronic low-grade abdominal pain, early satiety and weight loss; the CT reveals a large retroperitoneal mass. Which one of the following primary retroperitoneal tumours is most likely to appear on CT as a soft-tissue density mass with a large area of central necrosis? A. Liposarcoma B. Lymphoma C. Leiomyosarcoma D. Lymphangioma E. Paraganglioma
C. Leiomyosarcoma The presence of fat in a retroperitoneal tumour limits the differential diagnosis. A mass that is homogeneous and well defined and consists almost entirely of fat represents lipoma. When the mass is somewhat irregular and ill-defined but contains fat, a diagnosis of liposarcoma should be considered. Liposarcomas are the most common sarcomas of the retroperitoneum. Extremely hypervascular tumours such as paragangliomas sometimes contain haemorrhagic necrosis and manifest with fluid–fluid levels. Necrotic portions within tumours have low attenuation without contrast enhancement at CT and are hyperintense at T2-weighted MR imaging. Necrosis is usually seen in tumours of high-grade malignancy such as leiomyosarcomas, which have central necrosis more commonly than other sarcomas. Lymphomas grow and extend into spaces between pre-existing structures and surrounding vessels without compressing their lumina, manifesting with the‘CT angiogram sign’ or ‘floating aorta sign’. Some tumours, like lymphangioma, are completely cystic in appearance. Solid tumours with a partially cystic portion include neurogenic tumours
1602
A 5-year-old girl known to the endocrinologists presented to the A&E department after a fall and had radiographs of her lower limb taken to exclude fractures. No fractures were identified, but there were several well-defined lytic lesions in the metaphysis and diaphysis of the femur and tibia with an internal ground-glass matrix. Clinical examinations revealed several irregular areas of skin pigmentation. What is your diagnosis? A. McCune–Albright syndrome B. Mazabraud syndrome C. NF1 D. NF2 E. Cherubism
A. McCune–Albright syndrome Fibrous dysplasia is a relatively common benign skeletal disorder typically seen in adolescents and young adults. It can be monostotic (affecting one bone) or polyostotic (affecting multiple bones). Polyostotic fibrous dysplasia is commonly associated with café au lait spots, with irregular edges (‘coast of Maine’), in contrast to regular edges (‘coast of California’) observed in neurofibromatosis. Multiple endocrine disorders are described in association with fibrous dysplasia. McCune– Albright syndrome refers to the triad of polyostotic fibrous dysplasia (usually unilateral), café au lait spots and precocious puberty. Fibrous dysplasia is also associated with hyperthyroidism, hyperparathyroidism, acromegaly, diabetes mellitus and Cushing syndrome. Mazabraud syndrome refers to the association of polyostotic fibrous dysplasia with multiple soft-tissue myxomas, which are typically intramuscular. Cherubism is a special form of fibrous dysplasia with symmetric involvement of both the maxilla and mandible. It typically affects men and tends to regress after adolescence.
1603
A 45-year-old man with shortness of breath and bronchiectatic changes in the basal lobes showed moderate to marked dilatation of trachea on axial CT images. Which one of the following conditions is most likely? A. Cystic fibrosis B. Mounier–Kuhn syndrome C. Williams–Campbell syndrome D. Pulmonary sarcoidosis E. Allergic bronchopulmonary aspergillosis
B. Mounier–Kuhn syndrome Mounier–Kuhn syndrome is characterised by distinct tracheobronchial dilation that is due to atrophy of the muscular and elastic tissues in the trachea and main bronchial wall. Diagnosis is often made by using CT, through which abnormally large air passages are detected. In adults, the diagnostic criteria are trachea >30 mm, right main bronchus >20 mm and the left main bronchus >18 mm. Upper zone emphysema, lower zone bronchiectasis and tracheal diverticulae are recognised. Connective-tissue diseases, ataxia-telangiectasia, ankylosing spondylitis, Ehlers–Danlos syndrome, Marfan syndrome, Kenny–Caffey syndrome, Brachmann–de Lange syndrome and cutis laxa are also associated with secondary tracheobronchial enlargement. All of these conditions should be considered in the differential diagnosis. Williams–Campbell syndrome is a rare form of congenital cystic bronchiectasis, in which distal bronchial cartilage is defective with preservation of central bronchi and trachea
1604
A 65-year-old man with a cystic lesion of the pancreas detected on US was sent for a dual-phase pancreatic CT for further characterisation. The most commonly encountered cystic lesion of the pancreas is A. Serous cystadenoma B. Mucinous cystic neoplasm C. Pancreatic pseudocyst D. Main duct intraductal papillary mucinous neoplasm E. Side branch intraductal papillary mucinous neoplasm
E. Side-branch intraductal papillary mucinous neoplasm IPMNs can be classified as main duct, branch duct (side-branch) or mixed IPMNs, depending on the site and extent of involvement. Main duct IPMN is a morphologically distinct entity and cannot be included in the discussion of pancreatic cysts. However, a side-branch IPMN or a mixed IPMN (in which a side-branch tumour extends to the main pancreatic duct) can have the morphologic features of a complex pancreatic cyst, making clear-cut distinction from a mucinous cystic neoplasm difficult. Identification of a septated cyst that communicates with the main pancreatic duct is highly suggestive of a side-branch or mixed IPMN. However, it is important to be aware that lack of communication with the main pancreatic duct at imaging does not exclude an IPMN. Currently, MR cholangiopancreatography is considered the modality of choice for demonstrating the morphologic features of the cyst (including septa and mural nodules), establishing the presence of communication between the cystic lesion and the pancreatic duct, and evaluating the extent of pancreatic ductal dilatation. Because these lesions are considered premalignant, surgical resection has typically been recommended. The occurrence of malignancy is significantly higher in main duct and mixed IPMNs than in side-branch IPMNs. Therefore, in cases of side-branch IPMN, the treatment decision should be based on the risk–benefit ratio,
1605
A 23-year-old man presents with a congenital swelling underneath the left jaw, which has not changed over the past few years. Clinical examination reveals a horizontal scar overlying the same area. US shows multiloculated fluid-filled spaces at the site of the swelling in the neck. What is the diagnosis? A. Cystic hygroma B. Abscess C. Thyroglossal cyst D. Branchial cyst E. Cystic metastasis
A. Cystic hygroma A cystic hygroma is the most common form of lymphangioma and constitutes about 5% of all benign tumours of infancy and childhood. The overwhelming majority (about 80%–90%) are detected by the time the patient is 2 years old. These lesions are characteristically infiltrative in nature and do not respect fascial planes; hence recurrence post-surgery is not uncommon. On US scans, most cystic hygromas manifest as a multilocular predominantly cystic mass with septa of variable thickness. The echogenic portions of the lesion correlate with clusters of small, abnormal lymphatic channels. Fluid–fluid levels can be observed with a characteristic echogenic, haemorrhagic component layering in the dependent portion of the lesion. The most common pattern is that of a mass with low or intermediate signal intensity on T1-weighted images and hyperintensity on T2-weighted images. Infrequently, this lesion may be hyperintense on T1-weighted images, a finding associated with clotted blood or high lipid (chyle) content. In the case of haemorrhage, fluid–fluid levels may be observed.
1606
A 23-year-old man presented with a hard swelling of the left side of the jaw. Plain radiograph showed a well-defined oval lucent lesion in the left side of the mandible with the crown of a unerrupted lower third molar. What is the most likely diagnosis? A. Odontogenic keratocyst B. Dentigerous cyst C. Dental cyst D. Periapical cyst E. Adamantinoma
B. Dentigerous cyst Odontogenic keratocysts are believed to arise from the dental lamina and other sources of odontogenic epithelium. At radiography, an odontogenic keratocyst usually appears as a unilocular, lucent lesion with smooth, corticated borders that is often associated with an impacted tooth. Such lesions are indistinguishable from dentigerous cysts at radiography. They are more likely to show aggressive growth than other odontogenic cysts and may have undulating borders and a multilocular appearance; these characteristics make odontogenic keratocysts indistinguishable from ameloblastomas. The dentigerous (follicular) cyst is the most common type of non-inflammatory odontogenic cyst and the most common cause of a pericoronal area of lucency associated with an impacted tooth. At radiography, dentigerous cysts appear as well-defined, round or ovoid, corticated, lucent lesions around the crowns of unerupted teeth, usually t
1607
A 28-year-old woman is referred for a CT KUB for loin pain. The study unexpectedly shows grossly enlarged and abnormal kidneys with areas of fat density tissue within it. Small cystic areas are noted within the scanned lung bases. Further investigations reveal small subependymal lesions. What is a common musculoskeletal manifestation of this disease? A. Osteochondroma B. Sclerotic bone lesions C. Enostoses D. Fibrous dysplasia E. Enchondroma
B. Sclerotic bone lesions The patient has tuberous sclerosis, an inherited autosomal dominant multisystem disorder with multifocal systemic hamartomas. Osseous manifestations include cyst-like lesions, hyperostosis of the inner table of the calvaria, osteoblastic changes, periosteal new bone formation and scoliosis. These osseous lesions can occur anywhere in bone, commonly in the calvaria, short tubular bones of the hand or foot, spine and pelvis. The cyst-like lesions are usually irregularly circumscribed and have a sclerotic appearance peripherally.
1608
Which of the following is the correct order of normal myelination? A. B. C. D. E. Peripheral to central, caudal to rostral, ventral to dorsal Central to peripheral, caudal to rostral, ventral to dorsal Central to peripheral, rostral to caudal, dorsal to ventral Peripheral to central, rostral to caudal, dorsal to ventral Central to peripheral, caudal to rostral, dorsal to ventral
E. Central to peripheral, caudal to rostral, dorsal to ventral Myelination begins at 16 weeks in utero and is far from complete at term birth. It continues during the first 2 years of life until it reaches the level of myelination seen in a normal adult brain. Learning development coincides with myelination. CT and MRI can be used to assess myelination. On CT, unmyelinated white matter will appear hypodense. Unmyelinated white matter will appear hypointense on T1-weighted and hyperintense on T2-weighted. This can be confusing as many other conditions of interest in the neonate can cause T2-weighted hyperintensity. Knowing the pattern of normal myelination is therefore essential when interpreting paediatric neuroimaging.
1609
A 35-year-old woman with a palpable nodule in the left lobe of the thyroid gland showed a corresponding area of low activity on nuclear medicine study consistent with a cold nodule. How would you investigate this patient further? A. MRI neck B. CT neck with contrast C. US neck D. US neck with FNA E. Sialogram
D. US neck with FNA Thyroid scanning using pertechnetate (99MTc) is traditionally used to screen thyroid nodules for malignancy. The finding of a hyperfunctioning or‘hot’ nodule (uptake of tracer within the nodule with suppression of uptake in the surrounding normal thyroid tissue) excludes malignancy in almost all patients. A non-functioning or‘cold’ nodule was thought to indicate increased risk of malignancy, with 5%–15% of these being malignant. FNA should be the first-line investigation for assessment of all solitary nodules or a dominant nodule in a multinodular goitre. US is well established as a primary investigation for patients presenting with a lump in the neck; moreover, the cost-effectiveness and diagnostic accuracy of FNA can be increased by using US guidance and the presence of an on-site cytopathologist.
1610
A 53-year-old woman with upper abdominal discomfort was sent for an abdominal US, which showed a hypoechoic mass in the pancreas. A CT was performed, which reported a possible serous cystadenoma. Which one of the following statements regarding serous cystadenomas of the pancreas is true? A. They are rich in mucin. B. They are rich in glycogen. C. They have malignant potential. D. They appear only as a unilocular cyst on CT. E. They are more common in men than in women.
B. They are rich in glycogen. Serous cystadenomas are benign cystic neoplasms of the pancreas that occur frequently in older women (median age 65 years). Serous cystadenomas are composed of numerous small cysts that are conjoined in a honeycomb-like formation. The size of these cysts ranges from 0.1 to 2.0 cm but typically is less than 1 cm. The cysts are lined by glycogen-rich epithelium and separated by fibrous septa that radiate from a central scar, which may be calcified. This formation has led to the use of the more descriptive term microcystic pancreatic lesion. Serous cystadenomas are usually discovered incidentally at imaging; however, those that are large may cause symptoms such as abdominal pain or, more rarely, jaundice. Progressive enlargement of serous cystadenomas– especially those with a size of 4 cm or more at initial manifestation may be seen at serial follow-up imaging examinations performed over a period of months or years. Multiple serous cystadenomas may occur in von Hippel–Lindau disease. At MR imaging, a serous cystadenoma appears as a cluster of small cysts within the pancreas, with no visible communication between the cysts and the pancreatic duct. The cysts show signal intensity of simple fluid on T2-weighted images, and the thin fibrous septa between them enhance on delayed contrast-enhanced MR images.
1611
A 29-year-old woman with three previous miscarriages, not explained by any hormonal, biochemical or metabolic abnormality, was being investigated for a structural cause to explain the recurrent miscarriage. A pelvic MRI was scheduled, as it was the most definitive investigation for congenital structural anomalies and/or uterine masses. Which one of the following descriptions would suggest the diagnosis of uterus didelphys? A. Two separate uterine cavities with two cervices and two proximal vagina B. Two separate uterine cavities with two cervices C. D. E. Two separate uterine horns with common uterine cavity, with one cervix External indentation of the uterine fundus with one uterine cavity Single uterine horn connected to a single fallopian tube
A. Two separate uterine cavities with two cervices and two proximal vagina Uterus didelphys results from complete failure of Müllerian duct fusion. Each duct develops fully with duplication of the uterine horns, cervix and proximal vagina. A fundal cleft greater than 1 cm has been reported to be 100% sensitive and specific in differentiation of fusion anomalies (didelphys and bicornuate) from reabsorption anomalies (septate and arcuate). Bicornuate uterus involves duplication of uterus with possible duplication of cervix (bicornuate unicollis or bicornuate bicollis).
1612
A 3-year-old girl is referred to an endocrine clinic with unilateral jaw swelling noted at the dentist. Her general practitioner has also reported that she has signs of precocious puberty. An X-ray of the facial bones demonstrates expansion of the frontal bone and right side of the mandible. She is likely to have which other associated condition? A. Neurofibromatosis B. Madelung deformity C. Lisch nodule D. Hyperthyroidism E. Hypothyroidism
D. Hyperthyroidism The child has McCune–Albright syndrome (MAS), which is defined as the association of polyostotic fibrous dysplasia (PFD), precocious puberty, café au lait spots and other endocrinopathies caused by the hyperactivity of various endocrine glands. Among the endocrine syndromes described in association with MAS are (1) hyperthyroidism, (2) acromegaly, (3) gonadotrophinomas, (4) hyperprolactinaemia, (5) Cushing syndrome, (6) hyperparathyroidism, (7) gynaecomastia and (8) hypophosphataemic rickets. Lisch nodules are associated with neurofibromatosis. Fibrous dysplasia in MAS can involve any bone but most commonly affects the long bones, ribs, skull and facial bones. There is no association between Madelung deformity and MAS.
1613
A 54-year-old man with suspicious findings on US was recommended for an MRI of the orbits for further evaluation. Sagittal MR images of the globe showed a focal area of thickening in the posterior aspect of the globe with hyperintense signal on T1W sequence and strongly hypointense signal on T2W sequence. What is the diagnosis? A. Malignant melanoma of the choroid B. Rhabdomyosarcoma C. Coats’ disease D. Neuroblastoma metastasis E. Retinoblastoma
A. Malignant melanoma of the choroid Primary orbital melanoma is the most common primary intraocular malignancy in adults. MR imaging is superior to CT in the evaluation of choroidal melanomas, as melanin has intrinsic T1 and T2 shortening effects, thereby manifesting with increased T1 signal intensity and decreased T2 signal intensity. CT is non-specific, often demonstrating a hyperattenuating choroidal mass. MR imaging is also valuable for identifying other features, such as large tumour size, extraocular extension and ciliary body infiltration, all of which also portend a poorer prognosis. In addition, MR imaging is superior to CT for identifying retinal detachment and extrascleral spread. Notably, approximately 20% of melanomas are amelanotic, thereby lacking characteristic T1 and T2 shortening effects on MR images. In addition, MR signal characteristics may not always allow melanoma to be reliably distinguished from ocular metastases.
1614
Which of the following is not a recognised radiographic finding in a patient with haemochromatosis? A. Chondrocalcinosis B. Arthropathy with iron deposition in the synovium C. Generalised increased bone density D. Joint space narrowing E. Osteophyte formation
C. Generalised increased bone density Haemochromatosis may either be primary or secondary. It is most commonly primary and congenital with an autosomal recessive (AR) inheritance. It is relatively common in Caucasian populations with an incidence of 1 in 300 to 1 in 400. Men are affected about 10 times more commonly than women, and at an earlier age. It is often characterised radiographically by beak-like osteophytes projecting from the second and third metacarpal heads. The other hallmark radiographic findings of haemochromatosis include the following: generalised osteoporosis (not increased bone density), arthropathy with iron deposition in the synovium (50%), joint space narrowing and enlargement of metacarpal heads. Chondrocalcinosis is also relatively common in this condition, most often affecting the knees and triangular fibrocartilage.
1615
Foetal MRI is most commonly organised to evaluate inconclusive but potentially significant findings detected on ultrasound or to characterise definite abnormality detected on anomaly scan. Which one of the following MR imaging sequences is the most popular for foetal imaging? A. Fast spin echo T1W images B. STIR C. Fat-suppressed T1W images D. Single-shot, fast spin echo (SS FSE) T2W images E. FLAIR
D. Single-shot, fast spin-echo (SS-FSE) T2W images Because foetal MR imaging is performed without maternal or foetal sedation, image acquisition is susceptible to foetal motion; therefore, foetal MR imaging is performed primarily using ultrafast MR imaging techniques known as single-shot, fast spin-echo (SS-FSE) or half-Fourier acquired single-shot, turbo spin-echo (HASTE). Using these rapid pulse sequences, a single T2-weighted image can be acquired in less than 1 second, reducing the likelihood of foetal motion during image acquisition. Because each image is acquired separately, foetal motion typically affects only the particular image that was acquired while the foetus moved.
1616
A skeletal survey is performed on a 2-year-old boy with short stature. The lateral film of the spine reveals abnormal vertebral bodies with a central anterior‘beak’ and generalised flattening. Radiographs of the hands shows a pointed proximal fifth metacarpal base with a notch at the ulnar aspect. Which of the following is the most likely diagnosis? A. Hunter syndrome B. Hurler syndrome C. Morquio syndrome D. Achondroplasia E. Nail-patella syndrome
C. Morquio syndrome The mucopolysaccharidoses are a group of inherited diseases characterised by abnormal storage and excretion in the urine of various mucopolysaccharides. These patients have short stature and characteristic plain film findings. A characteristic finding in the hands is a pointed proximal fifth metacarpal base that has a notched appearance to the ulnar aspect. There is generalised flattening of the vertebral bodies (platyspondyly) (cf. not a feature of Hurler syndrome). Hunter and Hurler syndromes demonstrate an anterior vertebral beak that is inferiorly positioned, whereas Morquio syndrome demonstrates an anterior vertebral beak that is centrally positioned. Although achondroplasia can cause rounded anterior beaking in the vertebra of the upper lumbar spine, the findings described within the hands are more typical of the mucopolysaccharidoses.
1617
A macrosomic neonate (secondary to maternal diabetes) is noted to be in mild respiratory distress following delivery by caesarean section (CS). A chest X-ray demonstrates mild cardiomegaly mild hyperexpansion and small pleural effusions of the lungs. No focal lung abnormality is seen. What is the most likely diagnosis? A. Respiratory distress syndrome B. Meconium aspiration C. Staphylococcal pneumonia D. Group B streptococcal pneumonia E. Transient tachypnea of the newborn
E. Transient tachypnoea of the newborn Although RDS is seen in association with maternal diabetes and caesarean section, hyperexpansion is not a feature of RDS. Transient tachypnoea of the newborn appears soon after birth (<4 hours) and has been identified as occurring with caesarean birth and infant sedation. Longer labour intervals, macrosomia of the foetus, and maternal asthma have also been associated. It may be accompanied by chest retractions, by expiratory grunting, or by cyanosis (which can be relieved with minimal oxygen). Recovery is usually complete within 3 days. The lungs are usually affected diffusely and symmetrically, and the condition is commonly accompanied by small pleural effusions. The clinical course of transient tachypnoea is relatively benign when compared with the severity suggested by chest films. Radiographic resolution by the second or third day characterises this entity and differentiates it from other possible disorders; if radiographic resolution is not complete by the third day or if respiratory symptoms persist longer than 5 days, an alternative diagnosis should be sought. Findings of transient tachypnoea of the newborn on chest radiographs may include mild, symmetrical lung overaeration, prominent perihilar interstitial markings and small pleural effusions. The radiographic appearance at times can mimic the diffuse, granular appearance of hyaline membrane disease but without pulmonary underaeration. Neonates with transient tachypnoea are usually delivered at term. Radiographic lung changes may also resemble the coarse, interstitial pattern seen with other causes of pulmonary oedema or the irregular pattern of lung opacification seen in meconium aspiration syndrome.
1618
A 6-year-old boy presents with increasing pain within his upper back, which came on insidiously over a few weeks. The child is otherwise well. A radiograph of his thoracic spine reveals collapse of the T9 vertebral body. The disc spaces are preserved; there is no kyphosis, and no involvement of the posterior elements. Which of the following is the most likely diagnosis? A. Ewing’ s sarcoma B. Metastasis C. Tuberculosis D. Fracture E. Eosinophilic granuloma
E. Eosinophilic granuloma The vast majority of‘vertebra plana’ lesions in relatively healthy children are caused by an eosinophilic granuloma. The other available options are all possible, but less common, differential diagnoses. There is usually preservation of the disc space and no kyphosis. The posterior elements are rarely involved.
1619
A 42-year-old man presented with right-sided proptosis with mass in the inner canthus of the eye. An axial CT scan showed soft-tissue density and expansion of the right anterior ethmoid air cells bulging into the orbit. The extraocular muscles were displaced but not involved. What is the diagnosis? A. Encephalocele B. Anterior ethmoid mucocoele C. Destructive midline granuloma D. Wagner’s disease E. Esthesioneuroblastoma
B. Anterior ethmoid mucocoele Paranasal sinus mucocoeles are benign, expansile cystic masses covered by respiratory epithelium, resulting from accumulation and retention of mucus secretion in cases where the sinus drainage is obstructed. They primarily occur in the frontal sinuses (60%–65%) but may also be found in ethmoid sinuses (20%–25%). Usually, mucocoeles are seen as an isodense or mildly hyperdense sinus opacity in relation to the cerebral tissue, but in cases of acute infection it may appear as a more dense and peripherally enhanced image. The neighbouring bone structure is remodelled with areas of thickening, expansion and erosion. Additionally, in the areas of greater fragility, one may observe herniation into adjacent structures, displacing structures rather than invading them. Invasion would suggest malignancy. The radiological appearance at MRI varies with the time of evolution of the disease. Initially, the contents will be predominantly aqueous, so the corresponding image will be hypointense on T1-weighted sequences and hyperintense on T2-weighted sequences. Over time, the protein contents may increase, resulting in hyperintense images both on T1-weighted and T2-weighted sequences.
1620
A 61-year-old man with difficulty in swallowing was sent by his family doctor for a barium swallow. The examination showed a smooth filling defect in the mid-lower oesophagus with minor hold-up of contrast and proximal oesophageal dilatation. CT performed for further evaluation did not show any extra oesophageal organ involvement or lymphadenopathy. The most common mesenchymal tumour of the oesophagus is A. Lipoma B. Gastrointestinal stromal tumour C. Haemangioma D. Leiomyosarcoma E. Leiomyoma
E. Leiomyoma Leiomyomas are neoplasms of mature smooth muscle cells and are the most common benign oesophageal neoplasm, although they are about 50 times less common than oesophageal carcinoma. They are also the most common mesenchymal tumours of the oesophagus, unlike in the remainder of the gastrointestinal tract, where GISTs predominate. Oesophageal leiomyomas are nearly twice as common in men as in women and have been reported in patients between 4 and 81 years of age, although they rarely occur in the paediatric population. Most patients are asymptomatic, but dysphagia and pain may develop, depending on the size of the lesion and amount of encroachment on the oesophageal lumen, in contrast to patients with malignant oesophageal tumours. Affected individuals usually have long-standing symptoms, with a duration of more than 2 years in most cases. Treatment options include endoscopic resection, surgical enucleation and observation. Oesophageal leiomyomas have a benign clinical course and typically do not recur after surgery.
1621
A 70-year-old man with a history of weight loss and iron deficiency anaemia is referred for a CT colonography study. There is no malignancy on the scan but a 5 cm cystic structure is seen next to the distal ileum, in keeping with a duplication cyst. The most common site of a duplication cyst of the gastrointestinal tract is the A. Oesophagus B. Stomach C. Duodenum D. Jejunum E. Ileum
E. Ileum Enteric duplication cysts are an uncommon congenital abnormality. They can occur anywhere along the digestive tract on the mesenteric side. The small intestine is most commonly involved, with the order from most to least common being the ileum, jejunum and duodenum. Most duplication cysts manifest during the first year of life, although some occasionally manifest in older patients. Children can present with a variety of symptoms including abdominal distention, vomiting, bleeding, a palpable abdominal mass and rarely urinary frequency and hesitancy. Complications include perforation, intussusception, bowel obstruction from adjacent pressure or mass effect, volvulus and associated malignancy. Malignant lesions arising from duplication cysts are rare, particularly in children.
1622
A 45-year-old man with ulceration of the nasal septum was investigated further with CT sinuses. CT showed bone destruction involving the nasal cavity, turbinates and paranasal sinuses without associated soft-tissue masses. Chest radiograph of the same person done to exclude infection showed multiple nodules of varying sizes. A. Churg–Strauss disease B. Ethmoid carcinoma C. Polyarteritis nodosa D. Granulomatosis with polyangitis E. Leprosy
D. Granulomatosis with polyangitis CT scanning is likely to reveal a common mucosal thickening similar to that encountered in chronic sinusitis. However, the presence of bony erosions of the sinonasal wall is very suspect of the diagnosis of a granulomatous disease. Bone erosion commonly affects the lamina papyracea (orbital wall), inter sino-nasal wall and nasal septum. Bone destruction usually involves the nasal septum and then extends to the sinonasal wall, destroying the turbinates. Differential diagnoses including traumatic lesions (accidental, iatrogenic) or toxic lesions (cocaine abuse, chromium salts) should be considered. Isolated septal perforation should suggest the diagnosis of GP. There are no specific imaging findings to distinguish GP from‘lethal midline granuloma’, but the destructive lesions seem to be more extensive in lethal midline granuloma. Pulmonary nodules and masses are the most common radiologic findings of Wegener’s granulomatosis.
1623
A 30-year-old sustains a knee injury while skiing. MRI shows that there is a high signal in the lateral femoral condyle and the posterolateral part of the lateral tibial plateau. What is the most likely injury? A. ACL B. PCL C. MCL D. LCL E. MPFL (Medial patellofemoral ligament)
A. ACL The pivot shift injury is a non-contact injury commonly seen in skiers or US football players. The resulting bone contusion (pivot shift oedema) pattern involves the posterior aspect of the lateral tibial plateau and the midportion of the lateral femoral condyle. The exact location of the lateral femoral condyle injury depends on the degree of flexion of the knee at injury. Another recently described bone contusion pattern associated with the pivot shift injury is oedema within the posterior lip of the medial tibial plateau. Pivot shift oedema is typically associated with ACL tear, most commonly mid-substance. Dashboard oedema is seen at the anterior aspect of the tibia and, occasionally, at the posterior surface of the patella. The associated soft-tissue injuries include disruption of the PCL (most commonly mid-substance) posterior joint capsule. Hyperextension of the knee can result in kissing contusion involving the anterior aspect of the tibial plateau and the anterior aspect of the femoral condyle. ACL, PCL and meniscal injury can be associated depending on the force. The classic bone contusion pattern seen after lateral patellar dislocation includes involvement of the anterolateral aspect of the lateral femoral condyle and the inferomedial aspect of the patella. This is associated with tear of MPFL. With clip injury (pure valgus stress), bone marrow oedema is usually most prominent in the lateral femoral condyle secondary to the direct blow, whereas a second smaller area of oedema may be present in the medial femoral condyle secondary to avulsive stress to the MCL (mostly at the femoral attachment).
1624
A 14-year-old girl with chronic constitutional symptoms and sinonasal disease presents to the outpatient department. A CT of the paranasal sinuses reveals diffuse sinusitis and thickening of the paranasal sinus walls. On a plain chest radiograph, a cavitating pulmonary lesion is identified. Which of the following vasculitides would be the most likely diagnosis? A. Kawasaki’s disease B. Cogan’s syndrome C. Takayasu’s arteritis D. Granulomatosis with polyangiitis E. Polyarteritis nodosa
D. Granulomatosis with polyangiitis Granulomatosis with polyangiitis (GPA), previously known as Wegener’s granulomatosis, affects small to medium-sized vessels and most commonly involves the upper and lower respiratory tracts and kidneys. In comparison to adults, children are more likely to have multi-organ involvement with renal disease, subglottic stenosis, nasal deformity and pulmonary findings. Differential diagnosis of GPA may include those that present with pulmonary-renal features such as mixed connective tissue diseases, Goodpasture syndrome or systemic lupus erythematosus. Takayasu arteritis is a large-vessel vasculitis with characteristic abnormalities of the aorta and its main branches. Kawasaki’s disease and polyarteritis nodosa (PAN) are medium vessel vasculitides. The hallmark imaging feature in Kawasaki’s disease includes coronary artery aneurysms. In PAN, angiographic findings of aneurysms, stenoses or occlusions of medium-sized vessels is a key diagnostic criterion. This is predominantly seen in renal and mesenteric arteries, although not pathognomonic. Cogan’s syndrome is an autoimmune disorder characterised by interstitial keratitis and bilateral audiovestibular deficits. It may be associated with a systemic vasculitis and typically presents in young adulthood.
1625
An 8-year-old boy presents with a 5-week history of left hip pain and limp. Several previous pelvic X-rays were normal. A bone scan shows reduced uptake in the left femoral epiphysis. A line drawn from the lateral aspect of the femoral neck intersects the femoral head. What is the most likely diagnosis? A. SUFE B. Transient synovitis C. Perthes disease D. Developmental dysplasia of the hip E. Sickle cell disease
B. Transient synovitis Acute transient synovitis is the most common non-traumatic cause of hip pain in young children. It tends to affect children between 2 and 9 years of age and boys are affected two to four times more often. Where history is typical no imaging may be required; ultrasound can be used to identify the effusion. Initial bone scan uptake can be reduced, mimicking SUFE, but can increase later on. Juvenile arthritis affects children above 4–5 years of age. Radiographs may show erosions and loss of joint space. MRI and ultrasound are more sensitive for soft-tissue changes, allowing demonstration of synovitis, distinguishing pannus from simple effusion and identifying cartilage destruction and cortical erosions. Septic arthritis is an emergency. The majority of patients are less than 2 years old and are usually unwell with pain on passive movement of the hip. Avascular necrosis (AVN) of the femoral head is a condition induced by compromised blood supply, resulting in progressive destruction of bone. It is most commonly idiopathic (Perthes) but may be seen following trauma, infection, steroid treatment and in association with haematological diseases, such as sickle cell anaemia. Perthes disease usually affects children between 4 and 10 years of age and is more common in boys. Plain radiography is insensitive at early changes. The epiphysis may appear small, sclerotic or flattened with subchondral lucency or more marked fragmentation. Where symmetrical changes are seen, hypothyroidism or epiphyseal dysplasia should be considered. Meyer’s dysplasia mimics Perthes disease unilaterally by appearance but is asymptomatic.
1626
A 26-year-old man with facial pain shows a large, well-demarcated expansile cystic lesion in the petrous apex with erosion of the internal auditory meatus. No associated erosion of the scutum is identified, and the tympanic membrane is intact. MRI reveals a corresponding high-signal lesion on both T1W and T2W sequences. What is the diagnosis? A. Cholesterol granuloma B. Acquired cholesteatoma C. Congenital cholesteatoma D. Carcinoma of petrous temporal bone E. Arachnoid cyst
A. Cholesterol granuloma Cholesterol granulomas of the temporal bone can occur in the mastoid segment, the middle ear and the petrous apex. They are the most common primary petrous apex lesions. Temporal bone CT reveals an expansile, sharply defined and often rounded mass of the petrous apex with cortical thinning and trabecular breakdown. The general appearance is that of a slowly progressive benign process. There is central soft-tissue density without an internal matrix, a calcification or residual septations. If the lesion is sufficiently enlarged, frank bony dehiscence is observed. On MRI, cholesterol granulomas are typically hyperintense on both T1 and T2-weighted sequences because of the accumulation of blood breakdown products and proteinaceous debris. Small lesions may be relatively homogeneous, whereas large lesions show more heterogeneity. Often cholesterol granulomas have a distinct hypointense peripheral rim on T2-weighted images due to hemosiderin deposition. After contrast administration, there may be subtle peripheral enhancement secondary to inflammatory response but no central enhancement that would indicate solid tissue.
1627
A 6-year-old girl complains of pain in her left shoulder. X-ray shows a lucent lesion in the proximal humeral epiphysis. The lesion has sclerotic borders with specks of calcification within the lesion. What is the most likely diagnosis? A. Simple bone cyst B. Aneurysmal bone cyst (ABC) C. Non-ossifying fibroma D. Fibrous cortical defect E. Chondroblastoma
E. Chondroblastoma Chondroblastomas are rare, benign, cartilaginous tumours that affect the epiphysis of children. On MR images, chondroblastomas are seen as epiphyseal lesions with high T2 signal intensity surrounded by a halo of oedema in the adjacent marrow and soft tissues. A characteristic thin (<1 mm) low-signal-intensity ring that corresponds to peripheral sclerosis is seen in more than 90%. Fluid–fluid levels similar to Aneurysmal bone cyst (ABCs) are seen in 20%–30% of cases. Differential considerations include epiphyseal osteomyelitis and osteoid osteoma. Neuroblastoma metastasis and Langerhans cell hystiocytosis can also affect the epiphyses. Simple bone cyst (SBC), fibrous cortical defects and ABC affect the metaphysis.
1628
A 10-year-old girl complains of severe pain in her left hip. Plain X-ray shows there is a 5 mm lucent lesion in the medial femoral neck. On MRI, there is a 2 mm central focus, which is isointense on T1W and high on T2W images. There is surrounding high signal on T2W image. Dynamic imaging reveals peak enhancement on arterial phase with early partial washout of the lesion. There is slower progressive enhancement of the adjacent bone marrow. What is the likely diagnosis? A. Osteoid osteoma B. Osteoblastoma C. Stress fracture D. Cortical desmoid E. Osteochondroma
A. Osteoid osteoma Osteoid osteoma is a benign bone tumour that occurs most frequently in men and boys between 7 and 25 years old. Most patients experience pain that worsens at night and is promptly relieved by the administration of salicylates. Typical radiographic findings of osteoid osteoma include an intracortical nidus, which may display a variable amount of mineralisation, accompanied by cortical thickening and reactive sclerosis in a long bone shaft. The nidus is round or oval and usually smaller than 2 cm. At CT, the nidus is well defined and round or oval with low attenuation. The nidus has low to intermediate signal intensity on T1-weighted images and variable signal intensity on T2-weighted images, depending on the amount of mineralisation present in the centre of the nidus. Oedema in adjacent bone marrow and soft tissue and joint effusion may also be seen. Enhancement of a hypervascular nidus may be seen at dynamic CT.
1629
On a routine neonatal clinical examination, the paediatric registrar notes a simple sacral dimple at the natal cleft. It is blind-ended without any associated tuft of hair at the site of the dimple. What is the most reasonable subsequent management plan? A. No further management is required. B. Ultrasonography of the spine. C. Ultrasonography of the spine and cranial contents. D. MRI of the spine. E. MRI of the brain and spine.
A. No further management is required. Typical indications for spinal US in newborns and infants are skin-covered masses and midline cutaneous malformations of the back (e.g., dimple, haemangiomatous or hairy lesion), which are suggestive of associated dysraphic anomalies of the spinal cord. Spinal dysraphism is often associated with tethering of the spinal cord. The US appearance of tethering is a low-lying or blunt-ended conus medullaris due to abnormal fixation of the spinal cord. Moreover, movement of the spinal cord and cauda equina can be evaluated with real-time US with M-mode scanning. Typically, the tethered cord is positioned eccentrically and demonstrates reduced or absent movement. Dorsal dermal sinus manifests as a small dimple or pinpoint ostium, which is often associated with an area of hyperpigmented, angiomatous skin or hypertrichosis and occurs in a midline location or rarely in a paramedian location. Soft-tissue asymmetry and bone anomalies are common findings. Typical complications are infections such as recurrent meningitis, epidural or subdural abscess, and intramedullary spinal cord abscess. In particular, dorsal dermal sinus occurring in a paramedian location is often associated with an intraspinal dermoid or epidermoid cyst, which causes compression of neural structures with neurologic symptoms. For these reasons, dorsal dermal sinus has to be differentiated from simple sacral dimple or pilonidal sinus. The latter two anomalies do not extend to neural structures.
1630
A 77-year-old man with progressive worsening of breathing and chest pain presented to his family doctor, who sent him for a chest X-ray to exclude infection. The chest X-ray showed multiple ring shadows and he was sent for an HRCT. The HRCT confirmed the presence of multiple small cystic spaces in both lungs. All of the following disorders of the lung can be associated with this HRCT pattern, except: A. Tuberous sclerosis B. Langerhans cell histiocytosis C. Lymphocytic interstitial pneumonia D. End-stage interstitial fibrosis E. Coal worker’s pneumoconiosis
E. Coal worker’s pneumoconiosis Many diffuse lung diseases may manifest cysts as the primary abnormality, although lymphangioleiomyomatosis and Langerhans cell histiocytosis (LCH) are the most common to present with diffuse lung cysts. Others include lymphocytic interstitial pneumonia, follicular bronchiolitis, amyloidosis, light-chain deposition disease, Birt–Hogg–Dube syndrome, end-stage fibrosis (honeycombing) and cystic metastasis (leiomyosarcoma, synovial cell sarcoma, epithelioid cell sarcoma and endometrial stromal sarcoma).
1631
A 59-year-old woman with an apparent lump in the lower abdomen, weight loss and new onset tremor was sent for an MRI of the pelvis to investigate. Sagittal T2W MRI showed a multiloculated, heterogeneous left ovarian lesion with very low signal intensity. Corresponding axial in-phase and out-of-phase images revealed a hypointense mass with chemical shift artefact in its ventral aspect. Axial post-contrast T1W MRI showed significant enhancement of the ovarian lesion. What is the diagnosis? A. Brenner tumour B. Struma ovarii C. Ovarian thecofibroma D. Mucinous cystadenoma E. Endometrioma
B. Struma ovarii Struma ovarii is a rare ovarian lesion that accounts for 2% of ovarian teratomas. Struma ovarii is a highly specialised form of ovarian teratoma and is composed entirely or predominantly of thyroid tissue. About 5% of patients develop clinical evidence of hyperthyroidism. At US, struma ovarii has a non-specific solid, cystic appearance. MRI demonstrates a loculated cystic mass with variable signal characteristics. Cystic spaces may show marked T2 hypointensity and intermediate T1 signal intensity due to the thick, gelatinous colloid of the struma. Some locules may contain microscopic fat, as indicated by signal drop-off and chemical shift artefact on opposed-phase T1-weighted MRI. Struma ovarii typically demonstrates strong enhancement of the solid components on post-contrast T1-weighted MRI. Struma ovarii are benign in 95% of cases and usually occur in premenopausal women; therefore, preoperative diagnosis is essential to avoid unnecessary radical surgery.
1632
A 67-year-old man with worsening abdominal pain and LFTs shows a peripheral mass in the right lobe of the liver. Contrast-enhanced dynamic MRI done for further characterisation confirmed the lesion seen on US, with evidence of liver capsular retraction consistent with desmoplastic reaction commonly associated with peripheral cholangiocarcinoma. All of the following are risk factors, except: A. Radium exposure B. Chronic hepatitis C. Primary sclerosing cholangitis D. Thorotrast exposure E. Clonorchis sinensis infection
B. Chronic hepatitis There are a number of recognised risk factors for cholangiocarcinoma that all share the common feature of chronic biliary inflammation. Among these risk factors, infection with liver flukes (e.g., Opisthorchis viverrini and Clonorchis sinensis) and hepatolithiasis are common causes of cholangiocarcinoma in endemic areas. Dietary or endogenous nitrosamine compounds associated with parasitic infections also play an important role as cofactors in carcinogenesis, probably due to the carcinogenic effect of nitrosamine compounds on the proliferation of epithelial cells of the bile duct. Cholangiocarcinoma arising from a cirrhotic liver may be surrounded by a fibrotic pseudocapsule, which is an unusual finding in cholangiocarcinoma arising from a non-cirrhotic liver. In such cases, capsular retraction is noted along the tumour surface. This capsular retraction may be seen in some hepatocellular carcinomas (HCCs) with cirrhotic stroma but is more suggestive of cholangiocarcinoma. Cholangiocarcinoma can develop in a congenital choledochal cyst, with a lifetime risk of 10%–15%. In addition, a European study showed that a history of alcohol-related liver disease, cirrhosis, various bile-duct diseases, chronic inflammatory bowel disease or diabetes may increase the risk of development of cholangiocarcinoma.
1633
A 48-year-old woman with mid and lower back pain and shortness of breath presented to the the A&E department. Plain radiographs showed diffusely osteopaenic bones and an old superior end plate depression fracture of L1. CTPA showed acute pulmonary emboli. Plain X-ray of her hand done a year ago at a different hospital showed a metacarpal index of 9.8. What is your diagnosis? A. Marfan’s syndrome B. Homocystinuria C. Systemic lupus erythematosus D. Acromegaly E. Soto’s syndrome
B. Homocystinuria Homocystinuria is an AR disorder secondary to deficiency of cystathionine synthase. Arachnodactyly (metacarpal index >8.4 or >9.4; depending reference standard used) is seen in one in three patients (cf. 100% in Marfan syndrome). Lens dislocation is downwards and inwards (cf. upwards and outwards in Marfan syndrome). Homocystinuria is also associated with osteoporosis, bowing/fractures, pectus deformities and biconcave vertebra. There is increased propensity of thromboembolic phenomena due to increased stickiness of platelets. Death is often from occlusive vascular disease. Sotos syndrome is an autosomal dominant syndrome considered a form of cerebral gigantism.
1634
A 22-year-old man with a rapidly enlarging painful right maxilla showed an opacified right maxillary antrum on plain radiograph with destruction of the lateral wall. Axial CT showed extensive new bone formation on both sides of the anterolateral wall of the maxillary antrum with sun ray spiculations anteriorly. What is the diagnosis? A. Ewing’ s sarcoma B. Synovial sarcoma C. Antral carcinoma D. Myeloma E. Osteogenic sarcoma
E. Osteogenic sarcoma Fewer than 10% of osteosarcomas arise in the craniofacial bones, with most such tumours developing in the mandible and maxilla. The most common sites of involvement are the body of the mandible and the alveolar ridge or the antral area of the maxilla. It may be secondary to radiation, fibrous dysplasia, Paget disease, trauma, osteomyelitis, ossifying fibroma and giant cell tumour. On CT, the tumour displays a spectrum of bone changes from well-demarcated borders, notably the low-grade osteosarcoma (uncommon), to lytic bone destruction with indefinite margin and variable cortical bone erosion, to the osteoblastic form, where the bone is sclerotic. The majority of osteosarcomas have matrix mineralisation, calcifications of the osteoid or osteoid-like substance within the tumour and some tumours show a sunburst effect caused by radiating mineralised tumour spiculae. Ewing’s sarcoma can also occur in this area, although the expected age would be younger. On CT, it often shows the characteristic onion-skin appearance of periosteal reaction and less often a sunburst type of periosteal reaction.
1635
A chest radiograph of a 3-year-old child demonstrates marked right lower zone consolidation with a large pneumatocoele. A diagnosis of necrotising pneumonia is made. What is the most likely causative organism? A. Staphylococcus aureus B. Streptococcus pyogenes C. Bordatella pertussis D. Mycobacterium tuberculosis E. Aspergillus
A. Staphylococcus aureus Pneumatocoeles are thin-walled, air-filled intraparenchymal cysts that develop secondary to localised bronchiolar and alveolar necrosis, which allow one-way passage of air into the interstitial space. They commonly occur in immunocompetent patients and are most commonly associated with S. aureus, followed by Staphylococcus pneumoniae infections. Although there is no clear correlation between the development of pneumatocoeles and mechanical ventilation, patients receiving mechanical ventilation have an increased risk for developing complications related to pneumatocoeles, including an increase in their size. Other than in hyperimmunoglobulin E syndrome, there is no known genetic or familial tendency for pneumatocoeles. The majority of pneumatocoeles (more than 85%) resolve spontaneously, partially or completely over weeks to months without clinical or radiographic sequelae.
1636
A 3-month-old full-term infant with normal antenatal history presents with multiple seizures. On clinical examination, there is no facial asymmetry, dysmorphology or opthalmoplegia. There is marked hypotonia of the limbs. An MRI of the brain revealed a reduction in the number of cortical sulci and shallow Sylvian fissures. What is the most likely diagnosis? A. Holoprosencephaly B. Lissencephaly C. Band heterotopia D. Hemimegalancephaly E. Schizencephaly
B. Lissencephaly Lissencephaly (smooth brain) is a severe malformation of the cerebral cortex that results from impaired neuronal migration during the third and fourth months of gestation. The affected brain shows either an absence or a paucity of gyri (agyria or pachygyria, respectively). The most common clinical manifestations include severe psychomotor retardation, developmental delay, seizures and failure to thrive. The prognosis depends on the degree of failure of cortical development. In severe cases, death occurs in infancy or early childhood. Prenatal diagnosis of an affected foetus allows appropriate counselling and optimisation of obstetric management. Abnormal cortical development is the main manifestation of lissencephaly, although other associated cranial and extracranial abnormalities may be present.
1637
A 38-year-old patient presents with right-sided abdominal pain. She underwent renal transplantation 2 years previously for autosomal dominant polycystic kidney disease. A CT scan of the abdomen revealed oedematous terminal ileum, caecum and ascending colon. Which of the following is the most likely diagnosis in this patient? A. Appendicitis B. Tuberculosis C. Crohn’s disease D. Typhlitis E. Ischaemic bowel
D. Typhlitis Typhlitis, also known as neutropenic colitis, is a recognised acute colitis affecting the caecum ± the terminal ileum and ascending colon with a predisposition for children with leukaemia, lymphoma and patients on immunosuppressive treatment (i.e., neutropenia). The CT findings include circumferential wall thickening of the caecum, which may extend to the terminal ileum and ascending colon, pericaecal fluid and localised fat stranding. Intestinal TB may be primary or secondary to haematogenous spread from pulmonary TB, and predominantly affects the colon and ileum. Appendicitis may also produce caecal wall thickening, but it is usually asymmetric in nature and rarely extends into the terminal ileum. Backwash ileitis is a chronic complication of ulcerative colitis in which the terminal ileum is affected with a patulous ileocaecal valve, absent peristalsis and granularity of the mucosa. Pseudomembranous colitis usually has a predisposition for the distal colon but may affect the colon in its entirety.
1638
An elderly woman with progressive worsening of back pain is initially investigated with plain films of the spine. Plain radiographs show Grade III collapse of L3 with Grade II collapse of at least two other mid-thoracic vertebrae. MRI suggested osteoporotic collapse as the most likely cause. Which one of the following is the expected progression of osteoporotic vertebral compression fractures as seen on MR imaging? A. A partial return to normal fatty marrow. B. No change. C. The progression is unpredictable. D. E. An increase in oedema and fibrovascular tissue. A decrease in normal fatty marrow.
A. A partial return to normal fatty marrow. The signal intensity of the fractured vertebral body would appear low on T1-weighted images in the acute phase and would gradually be restored to normal intensity from the periphery to the centre of the body, as healing progresses. On T2-weighted images, the signal intensity of the fractured vertebral body would appear high, with or without some strongly lowered area in it, in the acute phase, and would be gradually restored to normal intensity with time. Acute fracture and metastatic compression fracture can both show enhancement post-contrast injection. However, contrast enhancement decreases with time in benign vertebral fractures as normal marrow signal is restored.
1639
A 29-year-old woman with a pelvic mass on US is sent for MRI. Axial TSE MRI shows a well-defined, ovoid solid mass with low signal on T1W images and intermediate signal with multiple round internal cysts on T2W images. Gadolinium-enhanced TSE T1 FS image shows the mass as having a moderately enhancing solid portion. All the following ovarian tumours typically show solid enhancing elements, except A. Sclerosing stromal tumour B. Sertoli–Leydig cell tumour C. Struma ovarii D. Serous cystadenoma E. Cystadenofibroma
D. Serous cystadenoma Ovarian tumours with highly enhancing solid portions, although uncommon, include sclerosing stromal tumour, Sertoli–Leydig cell tumour, struma ovarii and cystadenofibroma.
1640
Which of the following is a joint that is not characteristically involved in primary osteoarthritis (OA)? A. Distal interphalangeal joint (DIPJ) B. First metatarsophalangeal joint (MTPJ) C. Metacarpophalangeal joint (MCPJ) D. Knee E. Lumbar spine
C. Metacarpophalangeal joint (MCPJ) Primary OA describes degenerative joint disease with no local aetiological factor. It is age related and caused by high mechanical forces of a repetitive nature on a normal joint. Secondary OA describes degenerative changes within a joint with an underlying aetiological factor. These may include trauma, CPPD, inflammatory arthritis, haemochromatosis, developmental dysplasia of the hip (DDH), AVN or loose bodies. The joints most commonly involved in primary OA are distal interphalangeal, proximal interphalangeal, first carpometacarpal joint, hips, knees, spine and first metatarsophalangeal. The joints commonly spared include metacarpophalangeal, wrist, elbow, shoulder and ankles.
1641
A 3-day-old neonate presents with bilious vomiting and clinical suspicion for malrotation. Which of the following imaging features would best fit with this diagnosis? A. A duodeno-jejunal (DJ) flexure located left of the midline, above the gastric pylorus B. A double bubble sign on supine abdominal radiograph C. Retroperitoneal location of the third part of the duodenum D. Superior mesenteric artery located to the right of the superior mesenteric vein E. Distended bowel loops throughout the whole abdomen on plain radiography
D. Superior mesenteric artery located to the right of the superior mesenteric vein The upper GI series remains the imaging reference standard for the diagnosis of malrotation with or without volvulus. The normal position of the DJ junction is to the left of the left-sided pedicles of the vertebral body at the level of the duodenal bulb on frontal views and posterior (retroperitoneal) on lateral views. In children with acute duodenal obstruction, the upper GI series may depict a Z-shaped configuration of the duodenum in the presence of obstructing peritoneal bands or a corkscrew-shaped duodenum in the presence of volvulus. In children who have bowel malrotation without volvulus, the upper GI series shows an abnormal position of the DJ junction. The detection of the double bubble sign suggests duodenal obstruction. In infants in whom the radiograph demonstrates a double bubble, one should consider both intrinsic and extrinsic causes of obstruction. The intrinsic causes are duodenal atresia, duodenal stenosis and duodenal webs; the extrinsic causes include annular pancreas, malrotation of the gut with obstruction produced by mid-gut volvulus or by Ladd bands and preduodenal position of the portal vein. The proximal left-sided bubble is the air- and fluid-filled stomach. The duodenum represents the second bubble to the right of the midline. The third part of the duodenum is a retroperitoneal structure. Normally SMV is to the right of the SMA. Distended bowel loops throughout the abdomen would exclude any significant mid-gut obstruction.
1642
A 58-year-old patient is reported to have a carcinoid tumour of the gastrointestinal (GI) tract on an abdominal computed tomography scan. What is the most common primary site of GI carcinoids? A. Stomach B. Colon C. Rectum D. Small bowel E. Appendix
E. Appendix Carcinoid tumours are the most common primary tumour of the small bowel and appendix. Gastrointestinal carcinoids account for about 85% of all carcinoid tumours, with the remaining 15% occurring in the lungs and bronchi. These tumours arise from the enterochromaffin cells of Kultchitsky; these express serotonin and other histamine-like substances. The appendix is the most common site of carcinoids, accounting for 30%–45%, the small bowel for 25%–35%, the rectum 10%–15%, the colon 5% and the stomach <3%. Most tumours are clinically silent but may cause pain, obstruction, weight loss and, rarely, bowel perforation. In rare cases (7% of small bowel carcinoids), the hormonal load from the tumour may overwhelm the liver’s capacity to metabolise serotonin, causing a carcinoid syndrome–recurrent diarrhoea, right-sided endocardial fibroelastosis, wheezing/bronchospasm and flushing of the face and neck.
1643
A 53-year-old man with history of haematuria which shows a gel-like polypoid filling defect on cystoscopy is sent for an MRI. The MRI shows a low T1 signal, heterogeneous T2 signal (central high and peripheral low) lesion. On post-contrast T1W FS images, the peripheral portion enhances more than the central potion, resembling a ring-like pattern. What is the diagnosis? A. Endometriosis B. Inflammatory pseudotumour C. Malakoplakia D. Cystitis glandularis E. Eosinophilic cystitis
B. Inflammatory pseudotumour Inflammatory pseudotumour is an interesting entity that has been reported in every organ of the body. At imaging, it usually appears as a solitary exophytic or polypoid bladder mass, which may be ulcerated. On T2-weighted MRI, it is heterogeneous, with a central hyperintense component surrounded by a low-signal-intensity periphery; on post-contrast images the periphery enhances, whereas the central region enhances poorly. The central region consists of necrotic tissue, and the periphery comprises fascicles of spindle cells in oedematous stroma with myxoid components, vessels and inflammatory cells (hence the name pseudosarcomatous fibromyxoid tumour). This structure may produce the pattern of ring-like enhancement observed on CT and MR images suggestive of the diagnosis, but histologic confirmation is essential. In young adults, the presence of luminal clot surrounding an enhancing bladder mass may also suggest this diagnosis. MRI shows single or multiple masses. On T2-weighted images, cystitis glandularis shows low signal intensity with a central branching high-signal pattern. The hyperintense area shows the most contrast enhancement and corresponds to the vascular stalk. Eosinophilic cystitis nodules are hyperintense to muscle on T1, isointense on T2-weighted images, and enhanced after intravenous contrast administration.
1644
A 14-year-old boy with family history of pulmonary chondroma in his elder brother is investigated with a CT of the chest and abdomen. The CT shows a 3 × 3 cm calcified perihilar lung mass and a large mixed density mass in the left upper quadrant, anterior to the spleen inseparable from the stomach. What other finding(s) would you expect on the CT? A. Bilateral renal carcinoma B. Hepatoblastoma C. Multiple cysts in the lung, kidney and pancreas D. Wilms tumour on the right E. Multiple extra-adrenal neuroblastomas
E. Multiple extra-adrenal neuroblastomas The question describes the Carney triad: pulmonary chondroma, gastric GIST and multiple extra-adrenal neuroblastoma. Cancer predisposition syndrome (CPS) is the term that is generally reserved to describe familial cancers in which a clear mode of inheritance can be established. Individuals may present with one or more key physical features or congenital anomalies (e.g. hemihypertrophy). Patients may have specific tumours that are known to be highly associated with a CPS (e.g. haemangioblastomas in VHL (Von Hippel-Lindau syndrome) disease). Some physical features suggesting CPS are café-au-lait spots (NF1, NF2, Bloom’s), angiofibromas (tuberous sclerosis, TS), pits in palms and soles (Gorlin), macrocephaly (Sotos, Cowden, Gorlin), macroglosia [BWS ( (Beckwith–Wiedemann syndrome)], hyperpigmentation (NF1, Fanconi anaemia, Blooms), spotty skin pigmentation (Carney complex), hemihypertrophy (NF1, BWS, Klippel–Trénaunay syndrome), thumb malformation (Fanconi anaemia), aniridia [WAGR (Wilms tumor- anirida syndrome with genitourinary anomalies)] and so on. Tumours associated with CPS include Wilms tumour (WAGR, BWS and several others), haemangioblastoma (VHL), clear cell renal carcinoma (VHL, TS), pheochromocytoma (VHL, MEN 2B, NF1), hepatoblastoma (BWS, Familial adenomatous polyposis, FAP), adrenocortical and breast carcinoma (Li–Fraumeni syndrome, LFS), optic glioma and neurofibrosarcoma (NF1), retinoblastoma (familial retinoblastoma), gastric cancer and GIST (FAP, NF1, Carney triad, HNPCC [hereditary non-polyposis colorectal cancer] or lynch syndrome, LFS, MEN1), neuroblastoma (NF1, BWS), rhabdomyosarcoma (LFS, NF1, BWS, hereditary retinoblastoma) and so on. Screening tests and pathways have been established for several of these CPSs. These include US abdomen for BWS, LFS, FAP and VHL; prophylactic thyroidectomy for MEN 2; and so on.
1645
A 34-year-old man with history of measles infection showed marked cerebral atrophy on MRI with high signal in the deep white matter bilaterally on T2 and FLAIR images. What is the most likely diagnosis? A. Adrenoleukodystrophy B. Alexander’s disease C. CJD D. Progressive multifocal leukoencephalopathy (PML) E. Subacute sclerosing panencephalitis
E. Subacute sclerosing pan encephalitis Creutzfeldt–Jakob disease (CJD) shows high signal intensities in the basal ganglia (putamen and caudate nucleus) and in the cortex on DW images. The high signal intensities in the basal ganglia are also prevalent on T2-weighted and FLAIR images. The cortical hyperintensities are usually not visualised on T2-weighted and FLAIR images (advantage of DW imaging). Progressive multifocal leukoencephalopathy (PML) is a demyelinating disease of immunocompromised patients caused by human papovaviruses. Subacute sclerosing panencephalitis (SSPE) occurs several years after measles infection. SSPE typically starts with mental and behavioural abnormalities, myoclonia, tremor and seizures. Multifocal, hyperintense foci in white matter and the basal ganglia have been reported in PML and SSPE on T2-weighted images. On T2-weighted and FLAIR images, PML and SSPE are associated with white matter lesions, whereas CJD is not. The high-signal-intensity cortical lesions on DW images may be also a hallmark of CJD.
1646
An adult patient was admitted to hospital with abdominal pain, jaundice and a palpable epigastric mass. Ultrasound demonstrated isolated dilatation of the common bile duct with otherwise normal appearance of the proximal biliary tree. What is the most likely diagnosis based on the sonographic findings? A. Choledochal cyst B. Caroli disease C. Choledochocoele D. Common bile duct diverticulum E. Impacted common bile duct calculus
A. Choledochal cyst The Todani classification system is used to differentiate the cystic processes of the biliary tree into five groups. Type 1 cysts, known as choledochal cysts, are responsible for 90% of cystic biliary disease. These cysts are further subclassified into IA (dilation of the entire extrahepatic bile duct), IB (focal segmental dilation of the extrahepatic duct) and IC (dilation only affecting the common duct). Patients may present with the triad of vague abdominal pain, jaundice and a palpable epigastric mass, although this is only reported in 10%–20% of patients. Type 2‘cysts’ are true diverticulae of the bile duct. Type 3, known as a choledochocoele, is a focal protrusion of CBD into the duodenum. Type 4, consists of multiple communicating intra and extra-hepatic duct cysts. Type 5, known as Caroli's disease, represents cystic dilatation of intra-hepatic ducts.
1647
You are performing an abdominal ultrasound scan on a woman who has been complaining of chronic abdominal pain. There is a large 20 cm multiloculated, ovoid anechoic mass in the right lobe of liver. The internal septations are well visualised and hyperechoic. Further investigation with CT demonstrates enhancement of its thick wall and internal septations. What is the most likely diagnosis? A. Simple hepatic cyst B. Choledochal cyst C. Echinococcal cyst D. Caroli disease E. Biliary cystadenoma
E. Biliary cystadenoma Biliary cystadenomas are benign neoplasms originating in bile ducts, most commonly seen within the right lobe of liver. They rarely affect the extrahepatic biliary system. Women are affected more than men and typically present with chronic, vague abdominal pain. Biliary cystadenomas can reach up to 35 cm in size and are usually demonstrated as large multiloculated anechoic masses on ultrasound. A CT scan typically demonstrates a water density mass that shows enhancement of its walls and septations following administration of contrast, differentiating these from simple cysts. An echinococcal cyst (hydatid cyst) is a parasitic infection causing cystic transformation in the liver.
1648
A 45-year-old woman presents with right hip pain. An AP radiograph of the hip shows a large lucent lesion with stippled calcification and a wide destructive-appearing zone of transition. Which of the following is the most likely diagnosis? A. Osteomyelitis B. Osteosarcoma C. Chondrosarcoma D. Fibrous dysplasia E. Chondromyxoid fibroma
C. Chondrosarcoma The radiographic description is aggressive and the stippled calcification is suggestive of a chondroid matrix. The most likely diagnosis is therefore a chondrosarcoma. These may be primary or develop from an enchondroma or osteochondroma. Although chondromyxoid fibroma would be a possibility in this age group, it is an exceedingly rare benign chondral lesion that would normally have well-defined, thickened sclerotic margins. Chondrosarcomas are quite common tumours that may have relatively benign radiographic appearances. They are normally metaphyseal, located centrally within the skeleton and may exhibit endosteal scalloping. The diagnosis should also be considered in patients with increased pain or growth in centrally located enchondromas or osteochondromas. Surgical resection is the treatment of choice and the 5-year survival rate is approximately 75%.
1649
A 65-year-old man presents with severe upper thoracic back pain, of insidious onset, with little relief from analgesia. Lateral radiographs of his thoracic spine reveals uniformly increased density within the T7 and T9 vertebral bodies, with retention of the vertebral body size and contour. Which of the following is the most likely cause for this finding? A. Bone metastasis B. Osteoid osteoma C. Tuberous sclerosis D. Osteopetrosis E. Fluorosis
A. Bone metastasis The‘ivory vertebra’ sign, as described in this case, refers to a diffuse and homogeneous increase in the density of a vertebral body, which retains its size and contours. It can occur in both adults and children but is more common in the former. In adults, ivory vertebra has been associated most commonly with fractures (compression or healing), haemangiomas, lymphoma, myelosclerosis, with metastatic disease (especially prostate, breast and carcinoid), chronic sclerosing osteomyelitis, Paget disease and renal osteodystrophy. An ivory vertebra at one or more vertebral levels in an elderly man is most compatible with a diagnosis of metastatic disease, commonly as the result of prostate carcinoma. The other options are all much less common causes of ivory vertebrae.
1650
A 40-year-old golfer presents with sudden-onset right medial palmar wrist pain during a golf swing. On examination, he has paraesthesia in the right fourth and fifth digits. Plain AP and oblique radiographs are normal. Which of the following is the most appropriate next investigation? A. Nerve conduction studies B. CT scan of the wrist with 1-mm slices C. Hook of hamate radiographic views D. MRI of the wrist E. Ultrasound scan of the wrist
D. MRI of the wrist The patient’s presentation is a classic description of a hook of hamate fracture. These often occur during racquet, bat or club sports. A fracture of the hook of hamate can narrow Guyon’ s canal, compressing the ulnar neurovascular bundle and causing distal neuropathy of vascular compromise. Conventional radiographs even with multiple dedicated views only have a sensitivity of 72% for diagnosing this carpal fracture. CT with multiplanar reformatting would be a reasonable next investigation; however, in view of this patient’s neurological signs of ulnar nerve compression, MRI is the imaging modality of choice. This gives better soft-tissue definition while still having a sensitivity and specificity approaching 100% for diagnosing fractures.
1651
A 5-year-old girl with history of seizures and learning difficulty is referred for an abdominal ultrasound due to non-specific abdominal pain. On imaging, you notice bilateral renal cysts and multiple hyperechoic lesions in the kidneys and spleen. The patient also has small red skin lesions on her face. What is the most likely unifying diagnosis? A. Sturge–Weber Syndrome B. Von Hippel–Lindau C. Autosomal dominant polycystic kidney disease D. Tuberous sclerosis E. PHACE syndrome
D. Tuberous sclerosis The US images suggest renal cysts, bilateral angiomyolipomas and splenic hamartoma. Polycystic kidney disease PCKD1 gene lies next to Tuberous sclerosis gene TSC2 and renal cysts are common in TS. TS is an autosomal, dominant, inherited neurocutaneous syndrome characterised by a variety of hamartomatous lesions in various organs. Classically, TS demonstrates a triad of clinical features (Vogt triad): mental retardation, epilepsy and adenoma sebaceum. PHACE syndrome constitutes, posterior fossa malformations–hemangiomas–arterial anomalies–cardiac defects–eye abnormalities–sternal cleft and supraumbilical raphe syndrome. Typically infants have large plaque like facial haemangiomas.
1652
A 25-year-old woman had radiographs of her pelvis and both her lower limbs following a road traffic accident. Although no fracture was identified, multiple incidental findings were observed, including bilateral posterior iliac horns, protuberant anterior iliac spines and rudimentary patellae. Which of the following is the most likely diagnosis? A. Osgood–Schlatter B. Bipartite patella C. Diastrophic dysplasia D. Protrusio acetabuli E. Nail–patella syndrome
E. Nail–patella syndrome Nail–patella syndrome is a rare autosomal dominant disorder characterised by symmetrical mesodermal and ectodermal abnormalities. Radiographic abnormalities include bilateral posterior iliac horns in 80% (diagnostic), flared iliac crests with protuberant anterior iliac spines, genu valgum owing to asymmetrical development of the femoral condyles, prominent tibial tubercles, fragmentation/hypoplasia/absence of the patella, radial head/capitellum subluxation with dislocation of the radial head dorsally, short fifth metacarpal, clinodactyly of the fifth finger, scoliosis and, occasionally, mandibular cysts. Protrusio acetabuli and bipartite patella are not disease entities as such; they are radiological descriptions of the acetabular floor bulging into the pelvis and a congenital fragmentation or synchondrosis of the patella, respectively. Diastrophic dysplasia refers to severe rhizomelic dwarfism, and Osgood–Schlatter refers to rupture of the growth plate at the tibial tuberosity secondary to an apophyseal traction injury.
1653
A 5-month-old infant presents with one episode of urinary tract infection. The patient responds well to treatment and urinalysis reveals E. coli as the causative organism. There is no family history of ureteric reflux or renal disease. According to NICE guidelines, what imaging test(s) should be recommended? A. Urgent urinary tract ultrasound B. Routine urinary tract ultrasound within 6 weeks C. Routine urinary tract ultrasound with DMSA within 4–6 months D. Routine urinary tract ultrasound, DMSA and micturating cystourethrogram E. Magnetic resonance cystourethrogram
1654
A 55-year-old woman is found to have an incidental, pleurally based 7 cm ovoid mass with smooth tapered margins, on an otherwise unremarkable CT urography examination. MR imaging is performed for further assessment. Which of the following imaging characteristics is most suggestive of a diagnosis of benign mesothelioma? A. T1 MR hypointense; T2 MR hyperintense; contrast enhancement on CT avid B. T1 MR hypointense; T2 MR hyperintense; contrast enhancement on CT minor C. T1 MR hyperintense; T2 MR hyperintense; contrast enhancement on CT avid D. T1 MR hypointense; T2 MR hypointense; contrast enhancement on CT avid E. T1 MR hyperintense; T2 MR hyperintense; contrast enhancement on CT minor
A. T1 MR hypointense; T2 MR hyperintense; contrast enhancement on CT avid Benign mesothelioma is otherwise known as solitary fibrous tumour of the pleura. Unlike malignant mesothelioma, it has no recognised association with asbestos exposure. It is usually solitary and more commonly arises from the visceral pleura. It is asymptomatic in 50% and is a recognised cause of finger clubbing, hypertrophic pulmonary arthropathy and episodic hypoglycaemia. It is usually sessile but may be pedunculated, in which case shape and location may vary with the patient’s position. Imaging characteristics are as above, and contrast enhancement may be heterogeneous if there is myxoid degeneration and haemorrhage. About 14%–30% undergo malignant degeneration. Surgical excision is curative.
1655
A 33-year-old man with chronic sinus disease is being considered for a functional endoscopic sinus surgery (FESS) operation. The ENT surgeon refers him for CT sinuses prior to surgery. Regarding the ethmoidal sinuses, the ethmoidal air cells occasionally pneumatise laterally and posteriorly around the frontal recess. What is the name of this anatomical variation? A. Concha bullosa B. Agger nasi cell C. Haller cell D. Onodi cell E. Paradox middle turbinate
B. Agger nasi cell Agger nasi is a Latin term literally meaning‘nasal mound’. At rhinoscopy, the agger nasi appears as an eminence located on the lateral nasal wall at the leading edge of the middle turbinate; it represents the intranasal portion of the frontal process of the maxilla. As noted earlier, the agger nasi serves as the anterior limit of the frontal recess. Pneumatisation of the agger nasi (resulting in the so-called agger nasi cell) occurs in 78%–98.5% of individuals. When present, agger nasi cells are considered the most anterior of all ethmoid cells and can pneumatise posteriorly to narrow the frontal recess. Coronal and sagittal reformatted CT images are most helpful in identifying the agger nasi cell. On coronal images, the agger nasi appears as a laterally placed sinus below the frontal sinus and anterior to the middle turbinate. Sagittal images demonstrate the anterior location of the air cell. Haller cells are extramural ethmoidal air cells extending to the inferomedial orbital floor. Onodi cells are posterior-most ethmoidal cells which lie superolateral to the sphenoid sinus in close relation to the optic nerve and internal carotid artery.
1656
A 65-year-old woman with known history of gallstones is referred for an out-of-hours ultrasound scan to investigate fever, rigors and deranged liver function tests. Ultrasound demonstrates a common bile duct measuring 11 mm diameter and mild intrahepatic biliary dilatation, and a gallbladder containing multiple calculi associated with wall thickening, pericholecystic free fluid and a positive Murphy’s sign. What diagnosis should be fed back to the referring clinicians? A. Ascending cholangitis B. Biliary colic C. Acute cholecystitis D. Primary sclerosing cholangitis E. Impacted calculus in common bile duct
A. Ascending cholangitis The overwhelmingly important diagnosis not to be missed in this scenario is ascending cholangitis. This is because ascending cholangitis needs immediate treatment with broad-spectrum antibiotics and biliary drainage via either cholecystectomy or acute laparoscopic cholecystectomy. Although a sonographic-positive Murphy’s sign and pericholecystic free fluid suggest a diagnosis of acute cholecystitis, this should not distract the clinician from the presence of cholangitis, which is confirmed by the presence of fever and jaundice and is primarily a clinical diagnosis rather than a radiological one. Acute cholecystitis is not a clinical emergency and in this case is an associated finding that likely predates ascending cholangitis.
1657
A 16-year-old boy presents with shin pain. A tibia and fibula radiograph demonstrates a well-defined, cortically based osteolytic lesion within the diaphysis measuring 4 cm cranio-caudally without periosteal reaction. On MRI, the lesion is solitary, intermediate signal on T1 sequences (same as muscle) and high signal on T2. There is homogenous and avid enhancement post-contrast administration. There is no soft-tissue mass associated with the bony lesions. Which one of the following differential diagnoses is the most likely? A. Simple bone cyst B. Non-ossifying fibroma C. Fibrous cortical defect D. Chondroid myxoid fibroma E. Adamantinoma
E. Adamantinoma Adamantinoma appears as a well-circumscribed, slightly expansile lesion, usually with a narrow zone of transition, a finding consistent with its indolent nature. It is often multilocular, with sclerosis and lysis seen in a‘soap bubble’ pattern. The lesion is typically oriented longitudinally along the anterior tibial diaphysis, with an average length of 10 cm. MR imaging is the best modality for delineating the extent of adamantinoma in the medullary cavity, soft tissues and satellite lesions. These tumours typically demonstrate intermediate signal intensity relative to muscle on T1-weighted images and intensity similar to that of fat on T2-weighted images obtained without fat saturation. Static-enhanced images demonstrate intense homogeneous enhancement within the lesion.
1658
A 30-year-old woman with dysmenorrhoea and menorrhagia has had uterine artery embolisation of her fibroid uterus. She presents with lower abdominal pain and a malodorous vaginal discharge. Ultrasound demonstrates a distended debris- and fluid-filled uterine cavity with a separate hypoechoic large mass noted at the internal cervical os, causing cervical obstruction. Which of the following is the most likely explanation for these findings? A. Embolisation and subsequent swelling surrounding a cervical fibroid B. Detachment of a large subserosal pedunculated fibroid following embolisation C. Detachment of a large submucosal fibroid following embolisation D. Incidental large endometrial polyp close to the internal cervical os E. Cervical carcinoma
C. Detachment of a large submucosal fibroid following embolisation Uterine artery embolisation is an increasingly popular treatment that is used for the treatment of troublesome uterine fibroids; however, complications have been reported. Fibroid passage is one such complication; this is defined as detachment of the treated uterine fibroid. An increased risk of fibroid passage is seen in fibroids abutting the endometrial cavity (e.g. submucosal fibroids). Fibroid passage can precipitate severe pain, infection or recurrent bleeding. In this case, fibroid passage has caused cervical obstruction and pyometra. Cervical carcinoma and a large endometrial polyp can both cause cervical stenosis and consequent pyometra, but they are both much less likely in this circumstance.
1659
A 20-year-old man was seen in the A&E department after an injury to his foot during a football game. On examination, there was tenderness on palpation in the forefoot. A plain film performed showed a step in the alignment of the medial aspect of the second metatarsal bone and middle cuneiform bone. The second to fourth metatarsal had moved laterally. Normal alignment was noted at the articulation between the first metatarsal and medial cuneiform bone. Which type of fracture is demonstrated on the plain film? A. March fracture B. Jones fracture C. Lover’s fracture D. Homolateral Lisfranc fracture dislocation E. Divergent Lisfranc fracture dislocation
D. Homolateral Lisfranc fracture dislocation Lisfranc injury is a fracture dislocation of the tarso-metatarsal joints in a dorsolateral direction. The second metatarsal is held in a mortise formed by the medial and lateral cuneiform bones. Transverse metatarsal ligaments, both on the plantar and dorsal surfaces, connect the second to fifth metatarsals proximally. There is no transverse ligament between the first and second metatarsal. The plantar ligaments are stronger; hence most dislocations occur dorsally. If there is medial dislocation of the first metatarsal and lateral dislocation of the remaining metatarsals, it is considered a divergent type of Lisfranc injury. If the alignment of the first metatarsal is normal, or if it displaces laterally along with the remaining metatarsals, it is considered the homolateral type. On AP radiography of the foot, the medial aspect of the second metatarsal and middle cuneiform bone should align and, on the oblique view, the medial aspect of the third metatarsal and the lateral cuneiform bone should align. March fracture is another name for stress fracture, which commonly occurs at the distal shafts of the second and third metatarsals. They may appear on plain radiograph as a lucent line or their presence may be noted secondary to faint periosteal reaction or callus formation. If not visible on plain films and clinical suspicion persists, MRI or radionuclide scan may be helpful. Lover’s fracture is another term for fracture of the calcaneus. Jones fracture occurs in the proximal shaft of the fifth metatarsal. It has a high rate of non-union and is found within 1.5 cm of the fifth metatarsal tuberosity.
1660
Routine mammograms of a woman show a 1 cm retro-areolar lesion. Ultrasound demonstrates an 8 mm retro-areolar lesion with a dilated duct. Which of the following is the most likely diagnosis? A. Papillomatosis B. Ductal cancer C. Ductal ectasia D. Plasma cell mastitis E. Papilloma
E. Papilloma Papillomas are benign lesions (hyperplastic epithelium on a stalk) usually found in an ectatic subareolar major duct. They are the most common cause of blood or serous discharge. Papillomatosis refers to multiple peripheral papillomas that are located in the duct lumen just proximal to the lobule; it carries an increased risk of malignancy. Ductal ectasia is a benign entity that represents accumulation of cellular debris in enlarged subareolar ducts. Presentation is usually with non-bloody discharge and/or pain. It is the second most common cause of blood or serous discharge. Plasma cell mastitis refers to an inflammatory component associated with extensive secretory calcification.
1661
A 45-year-old woman is diagnosed with a biopsy-proven cancer. A lymph node is also demonstrated in the upper outer quadrant of the breast. Which of the following features is most suggestive of malignancy in the node? A. Hyperechoic centre B. Round and hypoechoic node C. Posterior acoustic shadowing D. A 1.5 cm node in the upper outer quadrant E. Homogeneous enhancement on MRI
B. Round and hypoechoic node US criteria for benign versus malignant nodes:
1662
A 60-year-old man with neurofibromatosis Type 1 presents with vague abdominal pain and bloating. A CT scan reveals a well-defined hyperenhancing rounded exophytic mass within the left upper quadrant, which has a broad base with the body of the stomach with no fat plane between the two. The spleen, transverse colon and left kidney are displaced by this mass. It measures 18 cm. There is no lymphadenopathy. Which of the following is the most likely diagnosis? A. Ectopic pancreas B. Gastric lipoma C. Gastric adenocarcinoma D. Malignant GIST (gastrointestinal stromal tumour) E. Benign GIST (gastrointestinal stromal tumour)
D. Malignant GIST (gastrointestinal stromal tumour) Gastrointestinal stromal tumours are the most common mesenchymal tumours of the gastrointestinal tract. They are usually found around the stomach and are often exophytic in nature. Commonly, they are found incidentally and can become very large before causing symptoms. These are difficult to distinguish from leiomyomas, leiomyosarcomas, schwannomas and neurofibromas radiologically and are characterised on immunohistochemistry by their expression of the KIT gene. It is important to appropriately identify GISTs as these can be treated successfully with tyrosine kinase inhibitors (Glivec®). Gastrointestinal stromal tumours can metastasise to the liver and peritoneum but lymphadenopathy is very uncommon. Once these tumours are over 5 cm malignancy becomes more likely. Lymphoma would usually be associated with lymphadenopathy. A gastric adenocarcinoma of this size would also be likely to have associated lymphadenopathy. Gastric lipoma should not enhance and be of low attenuation. Ectopic pancreas is usually endoluminal with an ill-defined border and is more often found in the antrum of the stomach and duodenum.
1663
A 26-year-old diabetic man who is known to binge drink is referred to you with deranged liver function tests following antibiotics for a chest infection. On ultrasound, you see a hyperechoic area anteriorly within the left lobe of liver within Segment 4. Which of the MRI sequences from the list below will be the most helpful in determining the nature of this lesion (although all would be required for a definitive diagnosis)? A. In-phase and out-of-phase T1-weighted images B. T1 and T2W images C. Diffusion-weighted images and apparent diffusion coefficient D. Dynamic contrast-enhanced phase images E. T2 with TE of 80 ms and T2 with TE of 160 ms
A. In-phase and out-of-phase T1-weighted images The hyperechoic area seen on ultrasound anteriorly within Segment 4 is most commonly caused by focal fatty infiltration and is a common finding at ultrasound and is more common in heavy drinkers, hyperlipidaemia and in obese patients. It is also commonly seen at the portal bifurcation. Although all of the above sequences will be important in accurately defining the lesion, the most useful sequence when this is likely to be focal fatty infiltration will be the in-phase and out-of-phase T1-weighted images. Focal fat should lose signal between the in-phase T1-weighted images and the out-of-phase T1-weighted images.
1664
A patient presents with deranged liver function tests, right upper quadrant pain, nausea, flushing and recurrent bouts of diarrhoea. The surgeon thinks the patient may have gallstone disease and requests a MRCP. A single 3 mm gallstone is noted within the gallbladder, but the bile ducts are within normal limits. A markedly hyperintense lesion on T2 is noted within the mesentery and a few of the small bowel loops in this region appear tethered. The lesion enhances avidly following administration of gadolinium. There are further hyperintense lesions noted throughout the liver, which also avidly enhance in the arterial phase. Which one of the following is the most likely diagnosis? A. Metastatic melanoma B. Cholangiocarcinoma with liver and mesenteric metastasis C. Retractile mesenteritis D. Carcinoid syndrome E. Cholecystitis with mesenteric and liver abscesses
D. Carcinoid syndrome Carcinoid syndrome is a collection of symptoms that are caused by a small proportion of carcinoid tumours. This nearly always occurs following metastasis to the liver, although it can happen if a carcinoid tumour occurs outside the gut and its excreted chemicals (adrenocorticotropic hormone, histamine, serotonin, etc.) bypass liver metabolism. These chemicals cause recurrent diarrhoea, bronchospasm, flushing and right-sided heart failure. A spiculated mesenteric mass with calcification and a desmoplastic effect on the adjacent bowel in a patient with these symptoms is characteristic of carcinoid syndrome. These tumours are often hyperintense on T2-weighted magnetic resonance imaging as they are very vascular. They also produce hypervascular metastases. If there is carcinoid syndrome, the carcinoid tumour is usually located in small bowel. Malignant melanomas are often bright on T1 and do not usually give a desmoplastic mesenteric reaction or give flushing. Cholangiocarcinomas produce hypervascular liver metastases but mesenteric deposits are unusual and the biliary tree appeared normal in this case. Retractile mesenteritis would not cause liver lesions. Mesenteric abscesses from cholecystitis would be rare and only the rim should enhance with gadolinium. This condition would also not be expected to produce a desmoplastic reaction.
1665
An 80-year-old patient with history of rheumatoid arthritis, has been involved in a 40 mph head-on car crash. Air bags were deployed. You are asked to review a cervical spine trauma series. No fracture is identified. The patient has mid-cervical spine tenderness. The patient wants to go home. Which of the following is the best option? A. Let the patient be discharged since trauma series is normal. B. Repeat lateral and AP views. C. Request a flexion and extension lateral cervical spine. D. Cervical spine CT. E. Bone scintigraphy.
D. Cervical spine CT. In high-risk patients or patients with limited physical examination, given the relative insensitivity of plain films, CT should be performed. In a rheumatoid patient there is an increased risk of cervical injury. Repeat views would not be of benefit. Age and mechanism of injury would indicate CT cervical spine according to NICE guidelines.
1666
A 42-year-old female patient with shortness of breath undergoes CT pulmonary angiogram for suspected pulmonary embolism (PE). In addition to the presence of subsegmental PE, the scan reveals two well-defined, rounded intraluminal polypoid masses on the posterior wall of the trachea and multiple small nodules in both lungs. Disseminated malignancy is suspected. Which primary malignancy is most likely to metastasize to the trachea? A. Malignant melanoma B. Ovarian carcinoma C. Sarcoma D. Transitional cell carcin
A. Malignant melanoma About 90% of all tracheal tumours are malignant and of those metastases are the most common group. Malignant melanoma, renal cell tumour, breast carcinoma and colonic adenocarcinoma are the most common sources of haematogenous metastatic spread to the tracheal mucosa. The lesions are usually multiple, polypoid, well defined and without tracheal wall thickening or extratracheal extension, as commonly seen in primary tracheal tumours. Cancers of the thyroid, oesophagus, larynx and lung may invade the tracheal wall by direct extension from the primary source, and this type of spread is the most common in secondary tracheal malignancies. The symptoms of tracheal malignancies are non-specific and include haemoptysis, dyspnoea, chest pain, cough and wheeze.
1667
A 72-year-old woman with B12 deficiency and achlorhydria has a barium meal. The stomach is featureless with no demonstrable rugal folds, and there is narrowing of the body of the stomach. Which of the following is the most likely diagnosis? A. Infectious gastritis B. Ménétrier disease C. Eosinophilic gastritis D. Atrophic gastritis E. Linitis plastica
D. Atrophic gastritis Atrophic gastritis has a high association with B12 deficiency. It occurs in the older population and is often caused by autoimmune disease. However, it can also be caused by Helicobacter pylori, leading to chronic inflammation of the gastric mucosa and eventually fibrosis. Atrophic gastritis particularly affects the chief and parietal cells, resulting in achlorhydria and B12 deficiency, and causes the classic featureless stomach. It is associated with an increased risk of gastric carcinoma. Infectious gastritis, Ménétrier disease and eosinophilic gastritis are often associated with thickened folds. Linitis plastica would have a history of weight loss and iron deficiency anaemia as opposed to B12 deficiency.
1668
A 78-year-old man presents with weight loss and frank haematuria. He had a stricture of his urethra dilated many years ago. He has no other relevant past history. Initial ultrasound and IVU show a normal upper urinary tract and bladder. As part of the standard work-up, he proceeds to flexible cystoscopy, but the urologist is unable to pass the cystoscope beyond the proximal penile urethra. He is therefore referred for an ascending urethrogram, which demonstrates an irregular filling defect in the bulbar urethra. Which of the following is the likeliest diagnosis? A. Prostatic carcinoma B. Transitional cell carcinoma of the urethra C. Squamous cell carcinoma of the urethra D. Adenocarcinoma of the urethra E. Recurrent benign stricture
C. Squamous cell carcinoma of the urethra Primary urethral carcinoma is rare. About 50%–75% occur in the bulbar urethra and nearly 90% are squamous type, particularly if there has been a stricture in the past. Urethral carcinoma is often advanced at diagnosis, explaining the weight loss. Primary transitional cell carcinomas (TCC) are closer to the bladder neck, although secondary TCCs can occur anywhere in the urethra when seeded from the bladder in patients with a bladder TCC history, when they are often multiple (these are the most commonly encountered male urethral tumours). Adenocarcinomas are extremely rare and tend to occur at the prostatic utricle or glands of Littre. Haematogenous metastases to the urethra are exceedingly rare but are most likely to be from melanoma. Prostatic carcinoma would be more likely to involve the prostatic urethra. Benign strictures can appear a little irregular, but weight loss would not be expected.
1669
CT of the head of a young man for chronic headache showed erosion of the left frontal sinus. Post-contrast showed focal meningeal enhancement with mucosal enhancement of the right frontal sinus. What is the most likely diagnosis? A. Mucocoele B. Inverted papilloma C. Pott’ s puffy tumour D. Antrochoanal polyp E. HIV
C. Pott’ s puffy tumour Pott’ s puffy tumour is defined as a subperiosteal abscess of the frontal bone with frontal osteomyelitis. Frontal sinus infection can spread directly through the thin bone wall of this sinus or through the network of small veins that drain its mucosa. Today, this is a rare complication, given the widespread use of antibiotics. Trauma and frontal sinusitis are the most common causes of this condition. The bacteria causing Pott’ s puffy tumour usually reflect the type of bacterial species responsible for community-acquired chronic sinusitis. The most common causal organisms are streptococci, staphylococci and anaerobic bacteria. Cultures frequently reveal polymicrobial involvement. The infection may spread as a thrombophlebitis from the frontal sinus through the diploic veins, involving the intracranial space with consequent epidural or subdural empyema, meningitis, brain abscess and venous sinus thrombosis.
1670
A middle-aged patient who sustained RTC 6 months ago presents with progressive visual loss and exophthalmos on the right. MRI demonstrated a dilated superior ophthalmic vein with flow voids in the cavernous sinus. What is the likely diagnosis? A. Buphthalmos B. Carotid-cavernous fistula C. Orbital pseudotumour D. Arteriovenous malformation E. Dural fistula
B. Carotid-cavernous fistula Carotid-cavernous fistula (also described as caroticocavernous fistula) is an abnormal communication between the internal carotid artery (ICA) and the veins of the cavernous sinus. It is mostly due to trauma with laceration of the ICA within the cavernous sinus usually due to a skull base fracture or penetrating trauma. Ultrasound and MRI usually show arterial flow in the cavernous sinus and superior ophthalmic vein.
1671
Which of the following is a recognised cause of a‘bone within bone’ appearance? A. Renal osteodystrophy B. Paget’s disease C. Hyperparathyroidism D. Melorheostosis E. Osteopathia striata
B. Paget’s disease A‘bone within bone’ appearance describes the radiographic appearance whereby a bone appears to have another bone within it, which results from endosteal new bone formation. Recognised causes include Paget’s disease, sickle cell disease, thalassemia, Gaucher’s disease, acromegaly, hypervitaminosis D, scurvy and rickets, among many others. It can also be a normal finding in infants, particularly in the thoracolumbar spine.
1672
A 30-year-old woman was involved in a severe road traffic accident and sustained direct high-energy trauma to her pelvis. Among other injuries, she was found to have a fracture on her left sacroiliac joint and left ischiopubic ramus. What type of fracture has she sustained? A. Open book B. Straddle C. Bucket handle D. Duverney E. Malgaigne
E. Malgaigne Pelvic fractures can be divided into stable and unstable fractures. The Malgaigne, open book, straddle and bucket-handle fractures are all unstable, as the pelvic ring is interrupted in two places. The Malgaigne fracture is described in this case. The open book fracture implies fracture/ diastasis of both ischiopubic rami and sacroiliac joints, the straddle fracture involves both obturator rings and the bucket-handle fracture refers to an SI joint fracture with a contralateral ischiopubic ramus fracture. Patients with unstable fractures are at significant risk of pelvic organ injury and haemorrhage. Duverney fracture is an isolated fracture of the iliac wing and is a stable fracture.
1673
A 6-year-old boy presents with adrenal insufficiency and developmental delay. Magnetic resonance imaging demonstrates diffuse T2 hyperintensity in the deep white matter, most predominant in the posterior parieto-occipital region and splenium of the corpus callosum. Which of the following is the most likely cause for this finding? A. Metachromatic leukodystrophy B. Acute disseminated encephalomyelitis C. X-linked adrenoleukodystrophy D. Alexander disease E. Canavan disease
C. X-linked adrenoleukodystrophy Dysmyelinating diseases, or leukodystrophies, encompass a wide spectrum of inherited neurodegenerative disorders affecting the integrity of myelin in the brain and peripheral nerves. Most of these disorders fall into one of three categories– lysosomal storage diseases, peroxisomal disorders and diseases caused by mitochondrial dysfunction– and each leukodystrophy has distinctive clinical, biochemical, pathological and radiological features. X-linked adrenoleukodystrophy is an inherited white matter disorder caused by gene mutation (ALD gene) resulting in abnormal formation of myelin. The childhood cerebral form (CCALD) is the most common and affects males aged between 4 and 10 years. Hyperpigmentation can occur as a result of adrenal insufficiency. The diagnostic clue is symmetric, peritrigonal white matter abnormality involving the splenium. Alexander disease characteristically involves the frontal white matter preferentially, and Canavan disease causes diffuse white matter abnormality.
1674
A 26-year-old woman who had an intrauterine contraceptive device (IUCD) coil inserted 6 years ago presents to her general practitioner complaining of right iliac fossa pain, constipation, night sweats and fevers. The practitioner refers her for a transvaginal ultrasound, which shows a right-sided convoluted cobra-shaped structure containing fluid echogenicity and some polypoidal outgrowths from the wall. Adjacent to this is a cystic left adnexal mass containing internal echoes. Which of the following is the likely diagnosis? A. Actinomycosis B. Appendix abscess C. Diverticulitis with pericolic abscess D. Migrated IUCD causing hydroureter E. Salpingitis secondary to tuberculosis
A. Actinomycosis Pelvic actinomycosis is a rare chronic bacterial infection but is commonly seen in the setting of a long-standing IUCD. It can also be associated with recent surgery. It often causes abdominal pain, low-grade fever and an abdominal or pelvic mass/abscess, which can mimic a malignant mass as it can get quite large if left untreated. The earlier it is diagnosed and treated, the less likely the patient will require surgery. The cobra-shaped structure is an infected dilated tube. Endometriosis can give cysts containing low-level echoes but fever would be unusual with pain. Appendix and diverticulitis can cause irritation of the adjacent tube but the clue is the long-standing IUCD.
1675
An 8-month-old boy presents with a right upper quadrant mass. Blood results reveal a raised alpha-fetoprotein (AFP). Ultrasound of the abdomen demonstrates a large 7-cm, hypervascular, heterogeneous hyperechoic mass in the liver with a few cystic regions. There is no vascular invasion. No renal or suprarenal lesions are present. Which of the following differential diagnoses is most likely? A. Hepatoblastoma B. Infantile haemangioendothelioma C. Hepatic haemangioma D. Mesenchymal hamartoma of the liver E. Fibrolamellar hepatocellular carcinoma
A. Hepatoblastoma Hepatoblastoma is the most common primary hepatic tumour in children. Hepatoblastoma has been associated with several syndromes, including Beckwith–Wiedemann syndrome, Gardner syndrome, familial adenomatous polyposis, type 1A glycogen storage disease and trisomy 18. Heptoblastomas are most often hyperechoic relative to adjacent liver on US. A spoke-wheel appearance with areas of alternating echogenicity may be seen at antenatal imaging. CT shows a sharply circumscribed mass that is slightly hypoattenuating relative to the adjacent liver on unenhanced and contrast-enhanced images. Epithelial hepatoblastomas demonstrate a more homogeneous appearance, while mixed tumours are more heterogeneous in attenuation. Speckled or amorphous calcification is seen in more than 50% of lesions. The tumour enhances slightly, but less than adjacent liver. At MR imaging, epithelial hepatoblastomas are homogeneously slightly hypointense on T1-weighted images and hyperintense on T2-weighted images relative to adjacent liver parenchyma. Mixed tumours demonstrate more heterogeneous signal intensity characteristics. Fibrotic septa are hypointense on both T1- and T2-weighted images and enhance after intravenous administration of gadolinium contrast material. Infantile haemangioendothelioma (IHE) is a vascular tumour and enhances much more than adjacent liver, while hepatoblastoma typically enhances much less than adjacent liver. Occasionally, the peripheral rim enhancement on arterial phase images seen in hepatoblastoma may suggest IHE, but IHE is distinguished by intense nodular or corrugated peripheral enhancement with centripetal fill-in on delayed phase images. Mesenchymal hamartoma of the liver (MHL) is a benign tumour that manifests in the same age group as hepatoblastoma. It can usually be distinguished from hepatoblastoma by normal serum AFP levels in MHL, predominantly cystic appearance and age at diagnosis >5 years (cf. hepatoblastoma generally diagnosed <5 years).
1676
A 13-year-old boy presents with symptoms and radiographic evidence of a slipped capital femoral epiphysis (SCFE). It is noted on his radiographs that the physes are generally wide with flaring of the metaphyses. Which of the following is the most likely diagnosis? A. Rickets B. Hypophosphatasia C. Blount’s disease D. Achondroplasia E. Renal osteodystrophy
A. Rickets Rickets is the paediatric equivalent of osteomalacia. It affects the metaphysis of bones as these are the most metabolically active sites. Common sites of involvement include proximal humerus, proximal tibia and proximal and distal femur. Appearances include widened and irregularly shaped physeal lucencies and metaphyseal flaring. There may be long bone deformation with lower limb bowing. Patients with rickets are at increased risk of Salter–Harris I fractures of the epiphyses that most commonly occur at the proximal femur (SCFE). While Blount’s disease is often associated with bow legged-ness in infants and children, it is an abnormality at the knee with an increase in the tibial metaphyseal angle. There are a few cases in the literature of SCFE in association with Blount’s disease. While hypophosphatasia may give similar findings to rickets, its incidence 1:10 0000 is several orders of magnitude less than rickets (1:100–1:1000).
1677
A 73-year-old woman with weight loss, previous history of endometriosis and a CA-125 of 983 µg/mL, attends for an magnetic resonance imaging (MRI) scan of the pelvis after a cystic mass with nodules was noted in the left adnexa on ultrasound. There is a 6 cm predominately unilocular cystic mass in the left adnexa, which is bright on T1W & T1W fat-saturated images with enhancing solid mural nodules along its wall. It remains high signal on fat-saturated imaging. Which of the following is the most likely diagnosis? A. Dysgerminoma B. Brenner tumour of the ovary C. Endometrioma D. Ovarian dermoid E. Clear cell carcinoma of the ovary
E. Clear cell carcinoma of the ovary Clear cell carcinoma and endometrioid carcinomas are commonly found in patients with previous endometriosis. Clear cell carcinoma is an aggressive carcinoma. It is frequently cystic with enhancing mural nodules. The cystic and solid components are high signal on both T1- and T2-weighted images. Brenner tumour of the ovary is low signal on T1- and T2-weighted imaging. Endometrioma may have a bright T1 signal but should not have enhancing mural nodules or be associated with weight loss. Dysgerminoma is a solid germ cell tumour of the ovary, typically solid with a fibrous capsule and fibrous septa. Ovarian dermoid should contain fat and would not be associated with weight loss.
1678
A 32-year-old lady presents with acute sudden onset headache to the A&E department. CT shows haemorrhage within the fourth ventricle. Which vessel is most likely to be involved? A. Anterior cerebral communicating artery B. Anterior cerebral artery C. Posterior cerebral artery D. Posterior cerebral communicating artery E. Posterior inferior cerebellar artery
E. Posterior inferior cerebellar artery With posterior inferior cerebellar artery (PICA) aneurysms, the rate of intraventricular haemorrhage is high due to reflux of blood. If there is an isolated intraventricular haemorrhage, a peripheral PICA aneurysm, lying in or near the fourth ventricle, may be suspected.
1679
A 4-year-old child undergoes a CXR for suspicion of chest infection. The request mentions that the child has a congenital cardiac anomaly, which is, as yet, untreated. No further information regarding the type of anomaly is provided. The only abnormalities you can detect on the CXR include mild generalised cardiomegaly and increased pulmonary arterial flow. You note from the A&E department notes that the child is not cyanosed. What is the most likely diagnosis? A. Ebstein anomaly B. Ventricular septal defect C. Tetralogy of Fallot D. Atrial septal defect E. Truncus arteriosus
B. Ventricular septal defect Congenital cardiac anomalies can be categorised according to whether or not the child is cyanosed. Thereafter, assessment of both heart size and pulmonary arterial flow allows considerable shortening of the differential diagnosis:
1680
An elderly patient presents to the orthopaedic clinic with progressive hip pain. He has a history of a hip replacement performed 5 years ago. A radionuclide bone scan is ordered and demonstrates increased uptake around the proximal aspect of the prosthesis. This finding is sensitive and specific for which of the following conditions? A. Prosthetic fracture B. Mechanical loosening C. Periprosthetic fracture D. Particle disease E. None of the above
E. None of the above A cemented component in a total hip joint replacement (THR) will demonstrate increased uptake on a radionuclide bone scan for up to 1–2 years. Following this time, increased uptake is good evidence to support mechanical loosening or infection with a 50%–100% sensitivity. It is, however, not specific and therefore correlation with the patient’s clinical presentation and plain radiography is required. Aspiration under fluoroscopic guidance may be required to diagnose or exclude infection. An arthrogram can be performed at the same time, which may allow a confident diagnosis of mechanical loosening to be made; however, a negative arthrogram does not exclude this. Cementless THR components may have persistently increased radionuclide uptake. This is secondary to bone ingrowth into the prosthesis and needs to be differentiated from pathology.
1681
A neonate is noted to be markedly cyanosed, worsening when she cries. A CXR performed on day 1 is normal. Which of the following types of congenital cardiac anomaly is most likely? A. Patent ductus arteriosus B. Ebstein anomaly C. Coarctation of the aorta D. Tricuspid atresia E. Tetralogy of Fallot
D. Tricuspid atresia Congenital cardiac anomalies can be categorised according to whether or not the child is cyanosed. Thereafter, assessment of both heart size and pulmonary arterial flow allows considerable shortening of the differential diagnosis In the case provided, the child is cyanosed; this excludes patent ductus arteriosus and coarctation as the cause. Ebstein anomaly classically produces a grossly enlarged heart. While tetralogy of Fallot is the most common congenital cardiac anomaly to cause cyanosis, it has characteristic CXR features; upturned cardiac apex and deficient main pulmonary artery creates a ‘boot-shaped’ heart. In addition, there is usually decreased pulmonary vascularity associated with Fallot’s tetralogy. Tricuspid atresia is the third most common congenital cardiac cause of cyanosis after tetralogy of Fallot and transposition of the great vessels.
1682
Pick’s disease affects which of the following? A. Fronto-parietal lobe B. Temporo-parietal C. Temporo-frontal D. Parieto-occipital E. Frontal
C. Temporo-frontal Pick’s disease is a neurodegenerative disease, and one of the tauopathies (group of neurodegenerative diseases characterised by abnormal metabolism of tau proteins leading to intracellular accumulation and formation of neurofibrillary tangles, similar to Alzheimer’s disease) characterised by the accumulation of the Pick bodies. It is sometimes used synonymously with fronto-temporal lobar degeneration (FTLD), although strictly it is incorrect since all causes of FTLD isn’t pathologically Pick’s disease. The primary imaging abnormality is that of cortical atrophy of the frontal and temporal lobes. These changes can be markedly asymmetric and affect one region much more than another. Volume of the head of the caudate nucleus may also be reduced. Differentials include Alzheimer’s disease and corticobasal degeneration where parietal lobe involvement is more pronounced.
1683
A normally well 8-year-old child presents to the A&E department short of breath and pyrexial complaining of joint pain. Chest X-ray shows an enlarged heart with upper lobe venous blood diversion and small bilateral pleural effusions. No focal collapse/consolidation is evident, but there are patchy interstitial infiltrates in a perihilar distribution. Further questioning reveals that the child recently had a sore throat. What is the most likely underlying diagnosis? A. Toxic synovitis B. Juvenile rheumatoid arthritis C. Reiter’s disease D. Rheumatic fever E. Septicaemia
D. Rheumatic fever The imaging features described are those of acute congestive heart failure. In the context of joint pain following a recent sore throat and with no previous cardiac history, rheumatic fever should be considered. Rheumatic fever commonly causes a pancarditis with valve insufficiency that can lead to acute congestive heart failure in severe cases. This follows a streptococcal pharyngitis. Chronically, rheumatic heart disease can cause valve stenosis with varying degrees of regurgitation, arrhythmias and ventricular dysfunction. Rheumatic fever as a child is the most common cause of valvular heart disease in adults in the western world. The mitral valve is the most commonly affected. Currently, the disease rarely affects children in the West, but the condition remains common in the developing world.
1684
A 38-year-old woman is undergoing investigations for infertility. She is otherwise asymptomatic. On hysterosalpingography, there is a large filling defect within the uterine fundus with a linear defect that extends into the filling defect. She subsequently has an MRI scan, which demonstrates a myometrial mass with indistinct margins, which abuts the junctional zone and has lower signal on T2 when compared with the adjacent myometrium. There are a few focal high T2 signal intensity areas within, some which appear more linear. It demonstrates less enhancement than its adjacent myometrium. Which of the following is the most likely diagnosis? A. Leiomyosarcoma B. Leiomyoma C. Endometrial carcinoma D. Adenomyosis E. Hydatidiform mole
D. Adenomyosis Adenomyosis can be focal or diffuse, the latter being the more common. The aetiology is unclear, but prominent endometrial glands extend into the myometrium with adjacent smooth muscle hyperplasia. Leiomyoma tends to be sharply demarcated as opposed to ill-defined and tends to have low-intensity T1 and T2 signals, although multiple signal characteristics are seen. It would be unusual for leiomyomas to have linear bands extending from the endometrium; this is more commonly seen in adenomyosis. Leiomyoma and leiomyosarcomas cannot be differentiated accurately on imaging; secondary features of lymphadenopathy or metastasis can help raise the suspicion of leiomyosarcoma. Endometrial carcinoma usually presents in an older age group and tends to present with symptoms of irregular bleeding, which is why it is often diagnosed at an early stage. There would likely be endometrial thickening also. Hydatidiform moles would usually be identified clinically, particularly in a patient being investigated for infertility, as she would have raised beta-human chorionic gonadotrophin levels.
1685
A 62-year-old postmenopausal woman has a magnetic resonance imaging (MRI) scan to look for a hernia. In the right ovary is a 1 cm sharply demarcated low T1 and T2 signal solid mass with adjacent calcification. There is also a multilocular cystic lesion within the same ovary, containing multiple thin-walled septa. Which of the following is the most likely cause of the well-demarcated low-signal lesion? A. Ovarian fibroma B. Ovarian Brenner tumour C. Ovarian dermoid cyst D. Endometrioma E. Corpus luteal cyst
B. Ovarian Brenner tumour Brenner tumours are uncommon tumours that are almost always benign. These tumours have a large fibrous component and therefore have a similar appearance to an ovarian fibroma (low signal on T1- and T2-weighted imaging) on both ultrasound and MRI. Brenner tumours are commonly found with an adjacent epithelial tumour of the same ovary (usually mucinous cystadenoma); hence, in this case, the diagnosis of Brenner tumour is more likely than ovarian fibroma. An ovarian dermoid would usually contain fat but would also be in the differential diagnosis. However, the patient is postmenopausal and so endometrioma and corpus luteal cyst are unlikely.
1686
A 63-year-old man presents with progressive vertical gaze abnormality and cognitive symptoms. MRI of the brain demonstrates volume loss of the mid brain, which was described by the reporting radiologist as Hummingbird sign. What is the diagnosis? A. Multisystem atrophy B. Progressive supranuclear palsy C. Parkinson’s disease D. Shy–Drager syndrome E. Amyotrophic lateral sclerosis
B. Progressive supranuclear palsy Conventional MRI is usually not helpful in the diagnosis of early Parkinson’s disease because it most often yields normal findings. In advanced disease, abnormalities of the substantia nigra, including volume loss, decreased T2 signal reflecting iron deposition and blurring of the margins, can be seen. However, the primary role of MRI is to exclude structural abnormalities that potentially mimic Parkinson’s disease (e.g., NPH– normal pressure hydrocephalous, intracranial mass and bilateral subdural haematomas). FDG PET images are most often normal and show preserved metabolism in the putamen and globus pallidus. This is a defining feature of Parkinson’s disease and allows differentiation from both PSP (progressive supranuclear palsy) and MSA (multisystem atrophy, Shy–Drager syndrome), which commonly demonstrate reduced basal ganglia FDG activity. In patients with MSA–Parkinson’s type, abnormalities are confined to the putamen and include atrophy, symmetric hypointensity on T2 and T2*-weighted images and‘slitlike’ marginal T2 hyperintensity. Putaminal atrophy appears to help discriminate MSA from Parkinson’s disease, whereas T2 hypointensity is a non-specific sign that can be seen in PSP, Wilson’s disease, neurodegeneration with brain iron accumulation and other acquired conditions. Patients with PSP exhibit atrophy of the midbrain and tegmentum, manifesting as third ventricular dilatation, reduced midbrain AP diameter or flattening of the superior midbrain. Reduced midbrain AP diameter at the level of the superior colliculi on axial images gives rise to the Mickey Mouse sign. Midbrain atrophy with relative preservation of pons produces the hummingbird sign or penguin sign. Additional findings include superior cerebellar peduncle atrophy and increased FLAIR signal, both of which have reasonably high sensitivity and specificity in distinguishing PSP from Parkinson’s disease and MSA.
1687
A malformation is defined as a congenital morphologic anomaly of a single organ or body part due to an alteration of the primary developmental program caused by a genetic defect. All of the following are examples of posterior fossa cystic malformation, except A. Blake’s pouch cyst B. Dandy–Walker malformation C. Arachnoid cyst D. Rhombencephalosynapsis E. Giant cisterna magna
D. Rhombencephalosynapsis Dandy–Walker malformation is characterised by hypoplasia or agenesis of cerebellar vermis and cystic dilatation of the 4th ventricle, which can result in enlargement of the posterior fossa. Blake’s pouch cyst is a result of absence of communication between the 4th ventricle and the subarachnoid space leading to tetraventricular hydrocephalus. The cerebellum has a normal size and shape. Typical neuroimaging findings include the presence of a cyst in a retro/infracerebellar location, which is essentially a diverticulum of the enlarged 4th ventricle. Giant cisterna magna is an enlarged cisterna magna (≥10 mm on mid-sagittal images) with an intact vermis, normal 4th ventricle and, in some patients, an enlarged posterior fossa. Consistent presence of hydrocephalus allows the differentiation of Blake’s pouch cyst from mega cisterna magna. Arachnoid cysts are well-defined CSF density extra-axial lesions in the posterior fossa, which do not communicate with the 4th ventricle or the subarachnoid space. They may enlarge during infancy and produce mass effect on the cerebellum and vermis, which may cause a secondary obstruction of the ventricular system, hydrocephalus and/or remodelling or thinning of the overlying occipital bone. Isolated hypoplasia of the inferior vermis with normal 4th ventricle has been described variably in literature as Dandy–Walker variant. Rhombencephalosynapsis is characterised by the absence of the vermis and continuity of the cerebellar hemispheres, dentate nuclei and superior cerebellar peduncles, which creates a horseshoe-shaped arch across the midline, resulting in a keyhole-shaped fourth ventricle. It is a key feature of Gómez–López–Hernández syndrome (parietal alopecia, trigeminal anaesthesia and craniofacial dysmorphic signs) and may be seen with VACTERL. Posterior coronal T2-weighted images show the horizontal folial pattern.
1688
A 37-year-old woman, with a previous history of cervical carcinoma and radiotherapy, presents with abdominal pain. On magnetic resonance imaging, there is a large cystic midline structure, which has high T1 signal and some internal debris, extending anteriorly and superiorly from the cervix. Coincidentally, in the cervical region, there is high/intermediate T2 signal surrounding the endocervical canal and disruption of the low-intensity cervical stromal ring. Which of the following is the most likely diagnosis? A. Cervical carcinoma recurrence causing haematometra B. Cervical carcinoma with invasion of the uterus C. Radiotherapy-induced cervical stenosis causing haematometra D. Cervical carcinoma recurrence with metastasis to ovary E. Imperforate hymen
A. Cervical carcinoma recurrence causing haematometra Cervical carcinoma tends to be of higher signal on T2 than the surrounding cervical stroma and, in the above case, there is also disruption of the cervical stromal fibrous ring, increasing the likelihood of microscopic parametrial invasion. In this case, the cervical tumour is causing a malignant cervical stenosis and haematometra (high T1 signal is consistent with haemorrhage; tumour is usually intermediate signal/hypointense, similar to myometrium on T1 imaging). After radiotherapy to the cervix, cervical stenosis with haematometra can occur; however, post radiotherapy, the cervix becomes low signal because of fibrosis. Ovaries usually lie to either side of the midline, and it would be unusual for a cervical carcinoma metastasis of the ovary to have a high T1 signal. Imperforate hymen is congenital and is normally picked up either neonatally or at the time of menarche, if the latter commonly presents with haematometrocolpos; it would be inconsistent with a cervical cancer history, as this tumour is not seen in patients who are virgo intacta.
1689
You are called to perform a catheter angiogram on a 26-year-old biker who was involved in a road traffic accident. You are told that he has suffered a dislocated knee. The clinician informs you that his foot is perfused but has sluggish capillary refill. He has a palpable dorsalis pedis pulse but no posterior tibial pulse. Which of the following would you expect to find on the angiogram? A. Severed anterior tibial artery B. Severed tibioperoneal trunk C. Traumatic popliteal artery dissection D. Atheromatous plaque at the peroneal origin E. Normal anatomy
B. Severed tibioperoneal trunk The most common anatomical configuration of the popliteal trifurcation is the anterior tibial (AT) artery branching off first and terminating as the dorsalis pedis artery. The tibioperoneal trunk arises distal to the AT origin and gives rise to the peroneal artery and posterior tibial (PT) arteries.
1690
A 13-year-old post-pubescent girl presents to the emergency department with acute abdominal pain sited predominantly within the right iliac fossa. An ultrasound scan is performed. This reveals an echogenic mass within the right side of pelvis measuring approximately 4 cm. The sonographer thinks it is adjacent to and inseparable from the right ovary. What is the most likely diagnosis? A. Acute appendicitis B. Ovarian dermoid C. Ovarian torsion D. Ectopic pregnancy E. Haemorrhagic ovarian cyst
E. Haemorrhagic ovarian cyst Acute pelvic pain in adolescent girls is a common problem, but ultrasound scanning is very useful in differentiating the many possible causes. Haemorrhagic ovarian cysts are a common cause of pelvic pain in adolescent girls and appear as an echogenic mass in relation to the ovary. Acute appendicitis is likely to occur as a blind-ending tubular structure. This may appear like a ‘target lesion’ in cross section and there may be fluid/collection adjacent to it. An acute appendix should be clearly distinct from the right ovary. Ovarian dermoids are usually predominantly fat filled and therefore echogenic on ultrasound, but these tend to be a painless, incidental finding. Ovarian torsion can certainly produce an echogenic mass within the right pelvis, but this is less common than haemorrhagic cysts and would not appear distinct from the ovary. Ectopic pregnancy usually appears as a‘doughnut’-shaped complex mass in relation to one of the uterine tubes; a foetal heartbeat may be present.
1691
A young adult presented with complex partial seizures and amnesia. His MRI scan demonstrated T2 hyperintensity within the medial right temporal lobe, loss of hippocampal head digitations and dilatation of the ipsilateral temporal horn of lateral ventricle. Which of the following is the most likely diagnosis? A. Herpes simplex encephalitis B. Choroidal fissure cyst C. Mesial temporal sclerosis D. Early-onset Alzheimer’s disease E. Post-seizure appearance
C. Mesial temporal sclerosis Mesial temporal sclerosis typically demonstrates an atrophic, T2 hyperintense hippocampus on imaging. Other pathology affecting the temporal lobe including infection and tumour would cause enlargement of the hippocampus. Alzeheimer’s dementia results in medial temporal atrophy, but T2-weighted hyperintensity is not a feature. Herpes encephalitis is caused by Herpes simplex virus (HSV) type 1 in adults and HSV type II in children. It involves the limbic lobe (hypothalamus, parahippocampal gyrus, cingulate gyrus); and typically appears bilateral but asymmetric. Gyriform enhancement and haemorrhage are late features. Choroidal fissure cyst is a benign incidental finding, similar to arachnoid cysts in the brain. These occur in the region of the hippocampal fissure.
1692
A 39-year-old woman taking human menopausal gonadotrophins presents with pelvic pain, bloating and weight gain. On transvaginal ultrasound, both ovaries are enlarged and contain multiple bilateral cysts, some of which are 8 cm. There is also free fluid within the pouch of Douglas and surrounding the uterus. Which of the following is the most likely diagnosis? A. Large corpus luteal cysts B. Endometriomas C. Polycystic ovarian syndrome D. Ovarian hyperstimulation syndrome E. Ovarian torsion
D. Ovarian hyperstimulation syndrome Human menopausal gonadotrophins are an infertility drug, which contains follicle-stimulating hormone and luteinising hormone, derived from the urine of postmenopausal women and given as an injection. One of its main side effects is ovarian hyperstimulation syndrome, which can also occur because of high levels of beta-human chorionic gonadotrophin seen in multiple pregnancies and hydatidiform moles. Typical symptoms are those mentioned in the question, vomiting and nausea. Corpus luteal cysts are singular and usually unilateral. Ovarian torsion is more common in infertility treatment but is usually unilateral, with a very oedematous ovary containing small, subcentimetre cysts arranged around its periphery. Polycystic ovarian syndrome causes bilaterally increased ovarian volume, but cysts are usually small and subcentimetre. Endometriomas usually contain low-level echoes.
1693
You are asked to perform an ultrasound scan on an asymptomatic 30-year-old pregnant woman 4 weeks after the obstetrician incidentally notes a 3 cm hyperechoic lesion in the right lobe of the liver. On today’s scan, you notice a 7 cm hyperechoic lesion involving segments VI & VII of the liver and subcapsular fluid. What is the most likely diagnosis for this lesion? A. Focal nodular hyperplasia B. Liver haemangioma C. Metastasis D. Hepatic adenoma E. Hepatic abscess
D. Hepatic adenoma Hepatic adenomas are benign encapsulated liver lesions, which are usually 8–15 cm in diameter but can grow up to 30 cm. They are associated with the oral contraceptive pill and can often increase in size during pregnancy. They appear as hyperechoic lesions on ultrasound and are usually of low density on plain computed tomography and enhance in the arterial phase with rapid washout. They have a risk of haemorrhage and, if rupture occurs, can develop into a subcapsular haematoma. Focal nodular hyperplasia, haemangiomas and metastases would not be expected to grow this fast. Hepatic abscesses can grow quickly but the patient would be unwell, especially with spread into a subcapsular collection.
1694
A 72-year-old man presents with recurrent episodes of flash pulmonary oedema. Magnetic resonance angiography shows bilateral renal artery stenosis with atherosclerotic disease at both renal artery ostia. Which of the following do you advise? A. No endovascular option B. Bilateral renal artery radiofrequency (RF) denervation C. Bilateral renal artery angioplasty and stenting D. Continue with best medical management E. Refer to vascular surgeons for bilateral renal artery endarterectomy
C. Bilateral renal artery angioplasty and stenting The results of the ASTRAL trial have shown no benefit of endovascular treatment when compared with best medical management of hypertension. However, recurrent flash pulmonary oedema is a good indication for intervention. Surgical endarterectomy is a procedure that carries huge risks and has no role in this case. Radiofrequency (RF) denervation is a relatively new method. The renal arteries are catheterised via a common femoral artery approach. Using an RF generator, the artery is denervated, excluding the kidneys, from the sympathetic control. The long-term results are unknown.
1695
A previously healthy 36-year-old woman presents with acute hepatic failure of unknown cause. Her international normalized ratio (INR) is 2.6 and fails to normalise despite treatment. An ultrasound scan shows ascites. The liver team asks you to consider a liver biopsy. Which of the following is the most appropriate reply? A. Unable to perform because of INR and ascites B. Advise the patient of the increased risk of bleeding and go ahead with liver biopsy C. Drain the ascites and the perform the biopsy D. Perform transjugular liver biopsy E. Perform transarterial liver biopsy
D. Perform transjugular liver biopsy This is a commonly occurring scenario. Liver biopsy is contraindicated in the presence of ascites and coagulopathy. The transjugular approach is a good alternative where available. The right internal jugular vein is punctured and a catheter is advanced into a hepatic vein, usually the right. Through this, a long specially designed biopsy needle is introduced and a biopsy is taken. The method is used for generalised liver disease and not for a focal lesion. An alternative method is the plugged liver biopsy; here the biopsy is done over a sheath, and coils are dropped after the core is taken to plug the hole
1696
An elderly man presents to his GP with pain in his left middle finger in the region of the nail bed. This has worsened over the past 2 months. Plain film reveals a destructive lytic lesion within the distal phalanx of his left middle finger. What is the likely diagnosis? A. Breast cancer metastasis B. Renal cancer metastasis C. Lung cancer metastasis D. Gastric cancer metastasis E. Pancreatic cancer metastasis
C. Lung cancer metastasis The most common sites for bone metastases are bones that contain red bone marrow. Hence, bone metastases have a predilection for the axial skeleton. Peripheral metastases are rare, and 50% of these are caused by lung cancer. The other options are also known causes of lytic metastases, but would be less likely to affect the hands.
1697
A 34-year-old pregnant woman with severe abdominal pain attends for an MRI scan after an ultrasound showing a uterine mass. As well as the normal gestational sac, on T1W imaging there is a low-signal mass noted within the posterior myometrial wall, which displaces the uterine cavity anteriorly. This mass has a high T1 signal intensity rim. On T2W images, this lesion is intermediate to low signal and has a low-signal rim. Which of the following is the most likely diagnosis? A. Non-degenerated leiomyoma B. Hyaline degeneration within a leiomyoma C. Red degeneration within a leiomyoma D. Myxoid degeneration within a leiomyoma E. Cystic degeneration within a leiomyoma
C. Red degeneration within a leiomyoma Fibroids (leiomyomas) are benign tumours made of smooth muscle. Afro-Caribbean women are more prone than Caucasian women to developing fibroids. Fibroids can outgrow their blood supply and undergo degeneration causing them to look very different on MRI depending on the type of degeneration that they undergo. Typically, a non-degenerated fibroid is a well-defined low-intensity lesion on T1 and T2-weighted images, and enhances post contrast injection. Hyaline degeneration is caused by necrotic central change and increasing collagen content; this causes variable signal on T1-weighted images but low signal on T2-weighted images. With calcific degeneration, the fibroid has similar signal characteristics on T1 and T2 to typical non- degenerative fibroids. Cystic degeneration can produce areas of fluid signal within the fibroid, which does not enhance. Myxoid degeneration results in a very high T2 signal with a little enhancement. Red degeneration is a very rare type of degeneration; it typically causes severe pain and can cause a fever. It is the result of thrombosis of the vessels surrounding the fibroid leading to the high T1 and low T2 signal rim, and it usually occurs in pregnancy.
1698
A 10-year-old girl of African origin presents with colicky right upper quadrant pain and deranged liver function tests. Ultrasound of the abdomen demonstrates acute calculus cholecystitis. What is the most likely diagnosis? A. Sickle cell anaemia B. Familial hypercholesterolaemia C. Inflammatory bowel disease D. Choledochal cyst E. Thalassaemia minor
A. Sickle cell anaemia Gallstones are an uncommon occurrence in childhood irrespective of the symptomatology and therefore should always raise the possibility of a background predisposing condition leading to their formation. In the African population, sickle cell anaemia is the most likely diagnosis. One must also consider other conditions such as malabsorption, Crohn disease, thalassemia major (in patients of Mediterranean origin), total parenteral nutrition (TPN), cystic fibrosis and short gut syndrome. The radiologist is often the first to raise the possibility of a predisposing condition. Children are at risk of the same gallstone-related complications as adults, including severe acute pancreatitis, which may be life threatening.
1699
A 40-year old with recurrent pulmonary emboli is due to have a hip replacement and it is decided to deploy a temporary inferior vena cava (IVC) filter. What is the preferred site of deployment of an IVC filter? A. Suprarenal IVC B. Infrarenal IVC C. Proximal to the clot load, no matter the level D. At the confluence of common iliac vessels E. At the junction of IVC and right atrium
B. Infrarenal IVC In the presence of normal anatomy, IVC filters should be placed inferior to the renal veins. This is to avoid potential clot propagation and renal vein thrombosis. If the patient has aberrant anatomy such as double IVC, a single suprarenal filter or twin IVC filters can be placed. Access is via a right internal jugular puncture or a right femoral vein puncture. A cavogram is performed to visualise the renal veins and to look for aberrant anatomy as described above. Indications for IVC filter include deep vein thrombosis where anticoagulation is contraindicated, where the patient is non-compliant with medical treatment or when there is free-floating thrombus in the IVC. Retrievable devices are available and should be removed within 14 days from insertion. Pulmonary embolism can still occur despite an IVC filter with an incidence of 2.7%–4%.
1700
Which of the following is an appropriate puncture site for a percutaneous nephrostomy in a patient with normal sonographic anatomy? (You anticipate no need for further intervention.) A. Posterior calyx, middle/lower pole B. Posterior calyx, upper pole C. Direct puncture of the renal pelvis D. Anterior, middle pole calyx E. Anterior calyx, upper pole
A. Posterior calyx, middle/lower pole When further intervention such as antegrade stent placement or percutaneous nephrolithotomy is anticipated, a posterior upper pole calyx may be appropriate. Effort should be made to puncture below the 12th rib to avoid traversing the pleura. However, this is not always possible and puncture between the 11th and 12th ribs can also be used with care. Direct puncture of the renal pelvis should be avoided as the adjacent vessels here are large. Anterior calyces should also be avoided, as puncturing these from a posterior approach increases the risk of renovascular injury because of the volume of renal parenchyma traversed.
1701
An 18-month-old toddler with a history of prematurity presents with an acutely distended abdomen and vomiting. He has not passed flatus for 24 hours. On examination, the referring accident and emergency registrar is concerned that the patient may have a torted testis, which he believes is causing the abdominal pain. An ultrasound demonstrates mixed echogenicity in the right scrotum. The testis could not be identified. What is the most likely diagnosis? A. Acute testicular torsion B. Testicular carcinoma C. Inguinal hernia D. Femoral hernia E. Sacro-coccygeal teratoma
C. Inguinal hernia The child in this scenario has acute bowel obstruction, which can sometimes be missed on initial assessment. When assessing plain films, the hernial orifices should be remembered as an important ‘review area’ as inguinal hernia are common and should not be missed, both in children and adults alike. In this case, an ultrasound was performed first but the same principles apply and bowel loops in the scrotum are diagnostic. The radiologist may be the first to raise this possibility. An incarcerated hernia associated with obstruction or strangulation is an acute surgical emergency and requires urgent surgical intervention. Femoral hernia is uncommon in children. Acute testicular torsion may have a similar presentation and should always be considered. At least some part of a testis should be identifiable in case of torsion, tumour or injury. Non-visualised testis strongly suggests hernia.
1702
A healthy 12-year-old boy presents with painful ankles (no history of trauma). A plain radiograph of his ankles demonstrates bilateral symmetrical periosteal reaction of his tibia and fibula. His father reports similar symptoms in adolescence but never had it investigated. No treatment is given. The boy had a further plain film performed after a football injury to his ankle at the age of 15 years. No periosteal reaction was seen at this time. Which of the following is the most likely diagnosis? A. Thyroid acropathy B. Pachydermoperiostosis C. Hypertrophic pulmonary osteoarthropathy D. Hypervitaminosis A E. Venous insufficiency
B. Pachydermoperiostosis Pachydermoperiostosis is a self-limiting condition with similar radiological findings to those of hypertrophic osteoarthropathy: symmetrical bilateral periosteal reaction. It is seen in adolescents and is of autosomal dominant inheritance pattern. Males tend to be affected more often than females. Differential diagnoses of these radiological findings include thyroid acropachy, hypervitaminosis A, hypertrophic osteoarthropathy, metastases and chronic venous insufficiency. All of the above require treatment before resolution of the periosteal reaction. Hypertrophic osteoarthropathy is caused by many pulmonary and non-pulmonary diseases such as benign and malignant thoracic tumours, chronic infection/inflammation (bronchiectasis and lung abscesses), cyanotic congenital heart disease, inflammatory bowel disease and liver cirrhosis. In HOA, resolution of the radiological findings is only really seen following treatment of the underlying condition.
1703
You are performing a varicocele embolisation. The patient suddenly develops left loin pain. He remains haemodynamically stable, but you notice contrast extravasation proximal to the coils. Which of the following is the most appropriate management? A. Inject polyvinyl alcohol (PVA) at the extravasation site B. Stent grafting to exclude the extravasation site C. Alert the surgical team– need to proceed to laparotomy D. Take the patient for a computed tomography scan of the abdomen/pelvis E. Conservative management
E. Conservative management Varicocele is a commonly identified correctable cause of male factor infertility. Surgical correction has a failure rate of less than 5%. An alternative to surgery is the selective catheterisation and embolisation of the gonadal vein. The gonadal vein is catheterised via a common femoral vein puncture and embolic material is introduced. Indications include symptomatic varicocele, recurrence of varicocele post treatment and varicocele with associated infertility. Complications include pain, recurrence and reaction to iodinated contrast. Rupture of the testicular vein is a known complication, but it needs no specific treatment. Unlike surgery, embolisation is not associated with postoperative hydrocele or testicular loss from inadvertent injury to testicular artery. However there are case reports of renal loss from coil migration.
1704
A 40-year-old man presents with acute onset of III cranial nerve palsy. The unenhanced CT shows subarachnoid blood. Where is the aneurysm likely to be? A. Anterior communicating cerebral artery B. Anterior cerebral artery C. Middle cerebral artery D. Posterior cerebral artery E. Posterior communicating cerebral artery
E. Posterior communicating cerebral artery The third nerves exit the brain stem medial to the cerebral peduncles, and course forward and laterally in the interpeduncular cistern between the posterior cerebral arteries–posterior communicating arteries (PcomA) above and superior cerebellar arteries below. The pupillary fibres are located dorsomedially and peripherally at this segment. Common pathologies involving this segment include aneurysm at PcomA or basilar tip, dolichoectatic vessels, microvascular ischaemia, SAH, meningitis, neoplasms (leukaemia, lymphoma, neurogenic tumours and leptomeningeal carcinomatosis), inflammatory disease (neurosarcoidosis, Wegener’s granulomatosis), demyelinating disease, transtentorial herniation (from supratentorial tumour, haemorrhage or brain swelling) and head trauma with nerve avulsion (usually at the posterior petroclinoid ligament where the third nerve is stretched because of downward displacement of the brain stem at the time of impact).
1705
A 7-year-old girl presents with abdominal pain, which has come on and off over the past year, and failure to thrive. An X-ray performed revealed dense metasphyseal bands in both lower limbs. The blood profile was unremarkable. What is the most likely diagnosis? A. Leukaemia B. Lead poisoning C. Congenital rubella D. Osteopetrosis E. Osteopathia striata
B. Lead poisoning The differential list for dense metaphyseal bands is plenty but includes normal variant in neonates, growth acceleration lines (usually in patients with chronic illnesses like diabetes, asthma), treated rickets, chronic anaemia, scurvy and lead poisoning. Recurrent abdominal pain is associated with lead poisoning. None of the other conditions, except for leukaemia, is associated with abdominal pain, but the dense metaphyseal band sign is acknowledged only in treated leukaemia rather than active leukaemia. Although the blood profile is often normal, the patient may sometimes have a mild microcystic, microchromic anaemia.
1706
A taxi driver is brought to the accident and emergency department after a high-speed road traffic accident. The fire brigade needed 2 hours to extricate him from his wrecked car. On arrival, he is hypothermic but stable, complaining of severe abdominal pain. An urgent contrast enhanced CT scan is performed and the radiologist on call issues a report describing features suggestive of mesenteric injury with segmental small bowel ischaemia. Which finding is the most specific for a mesenteric injury with associated bowel wall ischaemia? A. Generalised increased bowel wall enhancement B. Patchy and irregular Localised bowel wall enhancement C. Localised bowel wall thickening D. Decreased or absent bowel wall enhancement E. segmental bowel dilatation
D. Decreased or absent bowel wall enhancement Abnormalities in bowel wall enhancement after administration of IV contrast are very common and important positive findings in the evaluation of abdominal CT in a trauma scenario and should always be assessed. Generalised increased enhancement is typical for bowel wall injury with vascular involvement but without ischaemia, and is thought to be caused by increased permeability of the hypoperfused wall. Localised, patchy and irregular hyperenhancement is suggestive but not diagnostic of a full- thickness tear. Areas of decreased or absent enhancement in a segment of a bowel are indicative of bowel ischaemia; lack of enhancement is considered to be highly specific.
1707
A 5-year-old girl presents with left-sided abdominal pain. On examination, there is a palpable mass in the left flank. Computed tomography demonstrates a well-circumscribed multiseptated cystic renal mass, which is replacing the lower pole of the left kidney. The intervening septa are thick and enhance post-contrast, and the cysts appear to be herniating into the renal pelvis. Which of the following is the most likely diagnosis? A. Multicystic dysplastic kidney B. Multilocular cystic nephroma C. Nephroblastomatosis D. Polycystic kidney disease E. Mesoblastic nephroma
B. Multilocular cystic nephroma Multilocular cystic nephroma is a benign renal tumour that occurs in children and, less commonly, adult women. There is no known association with Wilms’ tumour. It is usually a unilateral abnormality that replaces an entire renal pole and presents as a large mass, often around 8–10 cm in diameter. Radiological appearances, while not entirely specific, can help to differentiate this lesion from other renal mass lesions. A sharply well-circumscribed, multiseptated cystic mass is typical, with a thick surrounding capsule. The cysts may appear to herniate into the renal pelvis– an appearance that is relatively specific. Unsurprisingly, these lesions are excised as a definitive radiological differentiation from malignancy is often not possible. Multilocular cystic nephroma can be differentiated from multicystic dysplastic kidney by the presence of normal functioning renal parenchyma and symmetrical renal excretion. Polycystic kidney disease involves the entire kidney, unlike multilocular cystic nephroma, which tends to be localised around a renal pole.
1708
A 24-year-old man sustained an injury to his wrist following a fall while skiing. There is tenderness in the anatomical snuffbox, particularly on ulnar deviation. Plain radiograph performed in casualty revels a scaphoid fracture. Which one of the following statements regarding fractures of the scaphoid bone is true? A. The distal fragment is at risk for avascular necrosis when the waist of the scaphoid is fractured. B. C. D. They usually are the result of a direct blow to the wrist. Fractures of the distal pole are less common than fractures of the proximal pole. Fractures involving the proximal pole invariably lead to avascular necrosis of the proximal fragment. E. Fractures through the waist of the scaphoid do not displace because of the strong supporting intercarpal ligaments.
C. Fractures of the distal pole are less common than fractures of the proximal pole. Scaphoid fractures are common injuries but can have long-standing implications if not treated in a timely manner because of the risk of avascular necrosis and early development of osteoarthritis in cases of mal- or non-union. Scaphoid fractures most commonly occur within the waist of the scaphoid (70%), with the remaining occurring in the proximal pole (20%) and distal pole (10%). Due to the nature of the scaphoid blood supply, it is the proximal fracture fragment that is at risk of avascular necrosis. A fall on the outstretched hand is the most common mechanism. Fracture displacement and scapholunate ligamentous disruption with widening of scapho-lunate distance, (‘Terry Thomas’ sign) are common associated findings.
1709
A 21-year-old man, who presented with a growing mass in the left testis of 2–3 months duration with a palpable, non-tender, firm nodule on the surface of the left testicle, on physical examination showed a well-circumscribed hypoechoic mass with a concentric lamellar pattern of alternating hyper-and hypoechoic rings on US. What is the diagnosis? A. Tunica albuginea cysts B. Simple cyst of testis C. Epidermoid cyst D. Intratesticular varicocele E. Intratesticular spermaocele
C. Epidermoid cyst Epidermoid cysts are also known as keratocysts. They are non-tender and usually palpable. The US appearance varies with the maturation, compactness and quantity of keratin present within the epidermoid cyst. A target appearance, a solid mass with an echogenic rim and a characteristic “onion ring” configuration with alternating layers of hyper and hypoechogenicity have been described. These cysts do not show blood flow at Doppler US examination. The constellation of an onion ring configuration, negative tumour marker status and avascularity help differentiate testicular epidermoids from other germ cell tumours. The US findings of intratesticular varicocele are similar to those of extratesticular varicocele and include multiple anechoic, serpiginous and tubular structures of varying sizes. Colour flow and duplex Doppler US show a venous flow pattern with a characteristic venous spectral waveform, which increases with the Valsalva manoeuvre. An intratesticular spermatocele is a cystic intraparenchymal lesion that is attached to the mediastinum in the area of the rete testis.
1710
A 2-year-boy presents with a 2-week history of melena culminating in an acute episode of bright red blood per rectum. Ultrasound was unremarkable. Upper gastrointestinal endoscopy was negative. Technetium pertechnetate demonstrates increased uptake in the left upper and right lower quadrants. What is the most likely diagnosis? A. Acute appendicitis B. Intussusception C. Meckel’s diverticulum D. Gastrinoma E. Non-specific inflammatory bowel disease
C. Meckel’s diverticulum The findings on this technetium pertechnetate scan are typical of a Meckel’s diverticulum containing ectopic gastric mucosa. The uptake in the left upper quadrant should not mislead the radiologist as it is the result of normal tracer uptake in the stomach mucosa. Secretions from ectopic gastric tissue in a Meckel’s diverticulum can cause ulceration of the diverticulum or adjacent small bowel and can lead to bleeding, which, if profuse, can simulate an upper gastrointestinal bleed. Meckel’s diverticulum may simulate acute appendicitis on clinical examination; however, a pertechnetate scan is only performed if a Meckel’s diverticulum is suspected clinically. A gastrinoma is usually located in the pancreatic islet cells, not the right iliac fossa.
1711
A 69-year-old man with a history of non-muscle invasive urothelial carcinoma of bladder treated with transurethral resection and intravesical BCG therapy presented with a firm palpable nodule on PR examination. MRI showed a low T2 signal lesion with high signal on DW MR, low signal on ADC map and non-enhancement on subtracted contrast-enhanced images in the peripheral gland at 4‘ o ’ clock. The findings were stable on an MR repeated at 9 months. A. Prostatic carcinoma B. Post-inflammatory scar C. Post biopsy haemorrhage D. Granulomatous prostatitis E. Post-radiotherapy change
D. Granulomatous prostatitis Granulomatous prostatitis is an inflammatory entity that often presents with a firm nodule on digital rectal examination and elevated prostate-specific antigen, thus clinically mimicking prostate cancer. Possible causes include previous intravesical BCG therapy, TB prostatitis and previous intervention such as TURP, although most cases are idiopathic. On MRI, granulomatous prostatitis may appear as a discrete mass with markedly abnormal T2 signal, DWI and ADC map. Furthermore, there may be associated infiltration of the periprostatic fat by inflammation, thus mimicking extraprostatic tumour extension. Currently, histopathologic analysis is regarded as the only means of definitively establishing the diagnosis and thereby excluding the presence of tumour; however, a suggestive history, such as prior BCG therapy, may be useful in prompting consideration of the diagnosis. In addition, the presence on MRI of large areas of non- enhancement, indicative of necrosis within the lesion corresponding with caseous abscess on pathologic evaluation, may suggest the diagnosis. A short-term follow-up MRI may be obtained after antimicrobial treatment to assess for therapeutic response.
1712
A 9-month-old boy with recurrent urinary tract infections and renal pelvis dilatation is referred for a micturating cystourethrogram (MCUG). The patient is currently taking a prophylactic dose of trimethoprim. With regard to performing an MCUG in this patient, which of the following instructions would you give his mother? A. The patient should continue on his current regime of antibiotics until his physicians advise him to stop. B The patient should stop taking their antibiotics the day before the procedure and restart the medication the day after the procedure. C The patient should take a course of treatment dose trimethoprim, starting on the day of the procedure for 3 subsequent days. D The patient should take a 5-day course of treatment dose trimethoprim, starting on the day before the procedure. E The patient should stop their antibiotics the day before the procedure and take a 3-day course of treatment dose gentamicin.
C. The patient should take a course of treatment dose trimethoprim, starting on the day of the procedure for 3 subsequent days. Ensure that the baby undergoing MCUG receives 4 mg/kg of trimethoprim on the day of the MCUG and for three subsequent days to prevent complication of infection. If a child is on prophylactic 2 mg/kg does of trimethoprim, full dose is required as described above to cover the MCUG.
1713
A 74-year-old man with painless haematuria and weightloss with irregular thickening of the bladder wall on US underwent an MRI for staging. MRI revealed focal plaque-like thickening in the bladder wall and areas of thin calcification on plain film. All of the following are recognised findings in squamous cell carcinoma of the bladder, except A. Single enhancing bladder mass B. Papillary tumour with pure intraluminal growth C. Sessile enhancing tumour mass D. Calcification related to tumour E. Mass in a diverticula
B. Papillary tumour with pure intraluminal growth Squamous cell carcinoma accounts for <5% of bladder neoplasms; however, in parts of the world where schistosomiasis (bilharziasis) is endemic, it is a major health problem, accounting for over 50% of bladder cancers. Risk factors in non-bilharzial regions include chronic irritation from indwelling catheters, bladder calculi or chronic infection. All of these risk factors may be present in paraplegic patients, putting them at increased risk. Cyclophosphamide, smoking and intravesical BCG has also been implicated in the pathogenesis of squamous carcinoma of the bladder. The imaging findings in squamous carcinoma are non-specific. Tumours may appear as a single enhancing bladder mass or as diffuse or focal wall thickening. Intradiverticular squamous tumours are soft-tissue masses, sometimes with surface calcification. In contrast to urothelial carcinoma, squamous carcinoma is sessile rather than papillary, and pure intraluminal growth is not seen. Bladder wall thickening and calcification, from chronic inflammation or infection with Bilharzia, may coexist and complicate the diagnosis.